Bei Vorträgen, Lesungen, Gesprächen, per Email oder auf sonstigen Wegen stellen mir die Menschen immer wieder Fragen zur Astronomie. Das ist auch gut so, denn Fragen zu beantworten gehört ja auch zu meinem Job. Einige Fragen sind dabei aber deutlich beliebter als andere und ich dachte mir es wäre eine gute Idee, diese “Standardfragen” auch mal in eigenen Blog-Artikeln zu beantworten. Also werde ich das ab jetzt in unregelmäßigen Abständen tun. Auf dieser Seite gibt es einen Überblick über alle bisher beantworteten Fragen. Und wenn ihr selbst eine Frage stellen wollt, könnt ihr das im Kommentarbereich tun. Wichtig: Stellt hier wirklich nur Fragen! Darüber diskutieren können wir auch anderswo im Blog; wenn das hier passiert, wird alles unübersichtlich (und wenn doch andere Antworten auftauchen, dann übernehme ich keine Garantie für deren Richtigkeit!). Ich kann nicht versprechen, dass ich alle Fragen beantworten werde und wie lange es bis zu einer Antwort dauert. Aber ich werde mein bestes tun!

————————————————-

Kommentare (1.491)

  1. #1 Markus
    21. Juli 2014

    Ich hätte eine Frage zu schwarzen Löchern und Gravitonen.

    In einem schwarzen Loch der Raum so gekrümmt ist, dass nach dem Schwarzschildradius kein Teilchen mehr selbst mit Lichteschwindigkeit entkommen kann.
    Außerdem wird vermutet, dass die Gravitation durch ein Austauschteilchen, dem Graviton vermittelt wird – welches sich widerum nur mit Lichtgeschwindikeit fortbewegen kann.

    Warum hat ein schwarzes Loch eine von außen sichtbare Masse, wenn das Graviton, welches die Masse vermittelt, den Ereignisshorizont gar nicht verlassen können sollte?

  2. #2 Nick
    Alsdorf
    21. Juli 2014

    Was macht die Wissenschaft so sicher dass es nichts schnelleres als die Lichtgeschwindigkeit gibt? Könnte es sein dass Licht z.B. in weiter Entfernung anders unterwegs ist als in unserer Galaxie?

  3. #3 Florian Freistetter
    22. Juli 2014

    BITTE beachtet meinen Hinweis im Text. Dieser Ort ist dazu da, UM FRAGEN ZU STELLEN. Nicht, um darüber zu diskutieren. Das macht alles nur unübersichtlich. Zum Diskutieren könnt ihr gerne anderswo im Blog kommentieren. Danke.

  4. #4 Martin
    Münster
    24. Juli 2014

    Eine Frage zur Forschung über die Dunkle Materie: Eines der wichtigsten Indizien, das als Bestätigung der Theorie der Dunklen Materie gilt, betrifft ja die Rotationsgeschwindigkeit der Galaxien, die sich als “zu hoch” herausgestellt hat. Dann kann man auf Bildern von Galaxienclustern die farblich erkennbar gemachte mutmaßliche Verteilung der Dunklen Materie sehen, wie man sie aus Gravitationslinseneffekten heraus berechnet hat. Kann man diese beiden Wege der Schätzung von DM bereits zusammenbringen, also bspw. bei starken lokalen Gravitationslinseneffekten genau dort entsprechend besonders hohe galaktische Rotationsgeschwindigkeiten messen? Ist dies bereits versucht worden, oder ist dies ggf. beim derzeitigen Stand der Beobachtungstechnik noch unmöglich? Ist es absehbar, dass die Beobachtungstechnik dies in einigen Jahren oder Jahrzehnten ermöglichen könnte?

  5. #5 Higgs-Teilchen
    24. Juli 2014

    @Florian
    Wollte auch schon länger mal fragen, wie denn der momentane Erkenntnisstand bezüglich der Dunklen Materie ist. Gibst da schon irgendeine/n mögliche/n Erklärung/Hinweis ect…..?

    Lg H.

  6. #6 Florian Freistetter
    24. Juli 2014
  7. #7 Daniel Schmid
    Schweiz
    28. Juli 2014

    Wie ist der momentane Wissensstand was Elementarteilchen angeht?
    Woraus bestehen Quarks und andere Elementarteilchen?
    Gibt es sowas wie eine kleinste Einheit der Materie?
    und wenn ja, könnte es sein, dass sie sich binär verhaltet?

  8. #8 eh i
    29. Juli 2014

    wieso fliegt ein lichtteilchen immer mit der selben geschwindichkeit ?
    egal von welcher quelle, kerze, lampen, leuchtkäfer, sonne ?
    wie kann man sich den “abschuss” der lichtteilchen bzw. welle vorstell ?

  9. […] war vor dem Urknall? Und auch wenn die Antwort darauf schwierig ist, möchte ich sie trotzdem in meine Serie zu den grundlegenden Fragen in der Astrononomie […]

  10. #10 Michael
    4. August 2014

    Wie ermittelt man die Distanz in Lichtjahren?

    Wenn man weit entfernte Objekte analysiert, kann es dann nicht zu vielen Abweichungen z.b. durch unterschiedlich starke Ausdehnungen des Raums kommen?

  11. #11 Eberhard Bauer
    4. August 2014

    Eine Frage zum Sonnensystem: Kann sich im Asteroidengürtel wieder ein größeres Objekt bilden, in dem z. B. Ceres die kleineren Objekte “einsammelt”?

  12. #12 SonnenKlar
    5. August 2014

    Ich träume gern ein bisschen herum. Daher ein paar was wäre wenn Fragen 🙂

    Hypothetisch betrachtet: Wie würde das Weltall von der “Oberfläche” eines schwarzen Lochs aus aussehen?
    Extrem beschleunigt?

    Noch eine Frage:
    Was würde man sehen, wenn man von einem Raumschiff, das mit exakt Lichtgeschwindigkeit (hypotethisch!) fliegt nach hinten schaut, von wo man herkommt?
    Schwärze?

  13. #13 Hans-Walter Bronder
    Schiffweiler
    7. August 2014

    Der Raum hat sich nach dem Urknall mit Überlichtgeschwindigkeit ausgedehnt. Könnte es daher nicht sein, dass wir heute theoretisch Bilder unserer eigenen Galaxie in ihrer Anfangszeit sehen könnten, da das Licht aus dieser Zeit uns jetzt erst erreicht ? Es stellt sich somit auch die Frage, ob wir dann nicht auch andere Galaxien “doppelt” sehen, also aus der Anfangszeit des Universums und der Zeit nach dem Abklingen der starken Anfangsexpansion und dass die Anzahl der Galaxien im Universum dadurch weitaus geringer ist, als bisher angenommen, was auch Auswirkung auf die Theorie von der dunklen Energie hätte ?

  14. […] meiner Serie “Fragen zur Astronomie” geht es heute um die Lichtgeschwindigkeit. Denn dieses Thema ist auch ein Dauerbrenner bei den […]

  15. #15 andy
    12. August 2014

    auch ich hätte mal eine frage,es heist doch je weiter entfernt die galaxien sind desto schneller entfernen sie sich.aber es heist doch auch je weiter eine galaxie weg ist desto mehr blicken wir in die vergangenheit.das müsste doch auch heissen das weit entfernte galaxien sich in der vergangenheit schneller entfernt haben als heute, oder? danke schon mal im vorraus für die antwort.

  16. #16 T2
    13. August 2014

    Wieso treffen Photonen eines Sterns, hundert Lichtjahre weit weg, immer genau in mein Auge? Und in das meines Freundes, der neben mir steht, gleichermaßen? (Und das ganze auch noch kontinuierlich…. )

  17. #17 T
    13. August 2014

    Hallo Namensvetter, ich benutze diesen (zugegebenermaßen nicht sehr originellen) nickname hier seit einigen Jahren und wäre Ihnen dankbar, wenn Sie sich einen anderen Buchstaben aussuchen würden. Danke. Offenbar lässt SB doppelte nicks zu und ich gehe davon aus, dass es sich um einen Zufall handelt.
    Ihre Frage habe ich übrigens nicht verstanden. Sterne strahlen in alle Richtungen, warum also sollten Photonen eines von der Erde aus sichtbaren Sternes nicht sichtbar sein? Oder ist das eine Scherzfrage?

  18. #18 Florian Freistetter
    13. August 2014

    Der Kommentarbereich hier ist eigentlich nur für Fragen. Nicht für Diskussionen.

  19. #19 T2
    14. August 2014

    Meine (durchaus ernstgemeinte) Frage zielt darauf wie es eigentlich sein kann, dass ein so winziges Teilchen immer genau dort ankommt, wo es ankommen muss: es legt tausende von Lichtjahren zurück und landet dann genau in meinem Teleskop. Und dasselbe gilt bei dem Alien, der den Stern von der anderen Seite des Universums aus beobachtet. Hat das mit der Wellenfunktion des Lichts zu tun?
    (Das “T” war ein bloßer Zufall und wurde wunschgemäß geändert.)

  20. #20 Florian Freistetter
    14. August 2014

    Ok. Da anscheinend niemand auf mich hören will, muss das hier jetzt doch wieder ein unübersichtliches Diskussionsforum werden anstatt ein Ort, um Fragen zu stellen. Es wird mir dann aber DEUTLICH schwerer fallen, aus den Diskussionen Fragen zu extrahieren, die noch nicht beantwortet worden sind, um darüber einen eigenen Beitrag zu schreiben. Und ich übernehme auch absolut keine Verantwortung für die Korrektheit der Antworten, die hier die diversen anderen Leute geben!

    (P.S. Deine Frage ist vergleichbar mit: Woher weiß der Regentropfen, dass er genau auf meinen Kopf fallen und mich nass machen soll? Darauf ist die Antwort doch klar, oder?)

  21. #21 Desolace
    14. August 2014

    Ich hatte diese Frage schon anderswo gestellt, allerdings finde ich das grad nicht mehr (weiß also auch nicht,obs da “”damals”” ne Antwort gab…):
    Wie ist der neuste Stand zum Aussehen unserer Galaxie (also, wieviele Arme hat sie)? Und, wahrscheinlich spannender: wie kann man sowas “messen”, bzw. herausfinden?

  22. #22 T2
    14. August 2014

    Hallo Florian, hui, jetzt weiss ich gar nicht mehr, ob ich noch antworten darf, denn ich verstehe deine Bitte, hier kein Diskussionsforum aufzumachen; aber offensichtlich versteht meine Frage keiner 😉
    Deine Analogie geht an meiner Frage vorbei, denn der Regentropfen kommt da runter, wo sich die Wolke nunmal befindet. Ein Stern strahlt aber überall hin. EIne Analogie wäre: Es regnet in Thüringen und in Hamburg werden davon die Kinder nass. Und da würde sich doch offensichtlich auch jeder drüber wundern, oder?
    Ich habe es wahrscheinlich falsch angestellt mit meinem Beispiel des Beobachters. Ich hätte einfach nur formulieren sollen: Wie breitet sich das Licht im Universum aus?
    Das ist meine Frage. (Und mit diesem 3. Post ist’s nun gut, keine Angst…)

  23. #23 Higgs-Teilchen
    14. August 2014

    Hi Florian

    Das Alpha Magnetic Spectrometer hat 2013 Positronen entdeckt. Meine Frage: Woher kommen die denn? Ich dachte jegliche Antimaterie sei beim Urknall vernichtet worden…..

    Lg Higgs

  24. #24 Chris
    17. August 2014

    Der Raum hat sich ja nach dem Urknall sehr schnell ausgedehnt. Wie schnell hat er sich denn ausgedehnt?

    Danke

  25. #25 Terraner
    24. August 2014

    Dunkle Materie hat ja Gravitation, könnte es Schwarze Löcher aus Dunkler Materie geben(theoretisch/mathematisch)?

  26. #26 Gerard
    Nürnberg
    28. August 2014

    Hallo Florian,

    mich würde interessieren, wie es mit Außerirdischen auf der Erde aussieht, wenn man dies aus biologischer Sicht sieht (Viren, Sauerstoff etc.).

    Darüber hab ich mal vor Jahren was gesehen/gehört/gelesen – würde jedoch gern mehr Informationen für die Unwahrscheinlichkeit aus biologischer Sicht erhalten.

  27. #27 Gerard
    28. August 2014

    Nachtrag: ich meine damit keine Verschwörungtheorien etc. sondern eher das Gegenteil:
    Aus welchen biologischen Gründen ist es äußerst unwahrscheinlich das außerirdische “einfach so” bei uns auftauchen – bsp. Virenbefall der Außerirdischen, Sauerstoff ist giftig etc.

  28. […] meiner Serie “Fragen zur Astronomie” ist heute eine besonders beliebte Frage an der Reihe: Was ist Antimaterie?. […]

  29. #29 Oliver O'Donnay
    Düsseldorf
    1. September 2014

    Mich würde interessieren, wie real schwarze Löcher sind?
    Diese Frage ist m.E. in so fern berechtigt, als diese kompakten Gebilde allesamt an der pathologischen Struktur der intrinsischen Singularitäten kranken.
    Weil unsere momentanen Theorien an den Singularitäten schwarzer Löcher wie ein Kartenhaus zusammen fallen, besteht doch zwangsläufige Notwendigkeit, über Alternativen nachzudenken und damit die Realität schwarzer Löcher in Frage zu stellen.

    Grüße aus Düsseldorf
    Oliver O’Donnay

  30. #30 Eberhard
    Berlin
    2. September 2014

    Sehr geehrter Herr Freistetter,

    wären Sie so nett eine Podcastfolge über den Mond zu machen? Gerade war doch der berühmte Supermond… und warum ist der Mond beim Aufgehen größer als ein paar Stunden später?
    Ich würde mich sehr freuen!
    Grüße aus Berlin!

  31. #31 Florian Freistetter
    3. September 2014

    @Eberhard: “wären Sie so nett eine Podcastfolge über den Mond zu machen? Gerade war doch der berühmte Supermond… und warum ist der Mond beim Aufgehen größer als ein paar Stunden später?”

    Siehe hier: https://scienceblogs.de/astrodicticum-simplex/2009/06/16/grosser-mond-kleiner-mond-die-mondtauschung/
    Und die Folgen 15 und 16 der Sternengeschichten.

  32. #32 R. Rose
    Bochum
    4. September 2014

    Nach der neuen Karte von Lanaikea liegt die Milchstrasse am Rand eines ‘Local Voids’. Mir kamen da gleich wieder die “Dark Energy vs Local Void” Diskussionen in den Sinn… würde mich freuen, wenn Sie zu diesen neuen Ergebnissen etwas schreiben könnten.

  33. #33 Florian Freistetter
    8. September 2014

    Über Laniakea habe ich hier was geschrieben: https://scienceblogs.de/astrodicticum-simplex/?p=16172

  34. […] wie das auf der Erde? Kann es nicht auch ganz anders sein?”. Grund genug, sie in meine Serie “Fragen zur Astronomie” aufzunehmen und einmal ausführlicher zu […]

  35. #35 Doris
    8. September 2014

    Mich würde interessieren, wie die vielen Monde in unserem Sonnensystem entstanden sind – waren es tlw. planeten, die von einem größeren planeten als mond eingefangen wurden (hier sind die großen monde von jupiter und saturn interessant). ich weiß, dass einige eingefangen sind, aber wo und wie entstanden io, europa, titan, … Danke!

  36. #36 Detlef
    8. September 2014

    Wie alt ist die Erde und wie kann man das wirklich korrekt ermitteln?
    Soweit ich weiß, hat man zur Feststellung des Alters der Erde die Zerfallsreihe des Uran bis hin zum Blei genutzt. Damit kann aber nach meinem Verständnis nur das Alter der “Baustoffe” unseres Planeten bestimmt werden, da der Uran-Zerfall ja unmittelbar nach dessen Entstehung in einer Supernova einsetzte. Daraus würde ich schließen, dass unser Planet jünger als die derzeit angegebenen 4,5 Milliarden Jahre ist, da der Zeitraum bis zur Entstehung unseres Sonnensystems von den 4,5 Milliarden Jahren abgezogen werden müsste.

  37. #37 Florian Freistetter
    8. September 2014

    @Detlef: ” Daraus würde ich schließen, dass unser Planet jünger als die derzeit angegebenen 4,5 Milliarden Jahre ist, da der Zeitraum bis zur Entstehung unseres Sonnensystems von den 4,5 Milliarden Jahren abgezogen werden müsste.”

    Da das aber nur ein paar Dutzend Millionen Jahre sind, fällt das kaum ins Gewicht.

  38. #38 ZufallsGast
    11. September 2014

    Was würden Sie als Antwort auf das Fermi-Paradox geben?
    Sind wir eine der ersten (Intelligenten) in unserer Galaxie oder gar in unserem Universum – und nur deshalb allein?

    Und die Frage ist ja noch beliebig erweiterbar:

    Falls Zeitreisen in die Vergangenheit möglich sind, wo sind die Zeitreisenden?

    Falls unser Universum Teil eines Multiversums ist – beweist die Abwesenheit anderer Intelligenzen, dass ein interdimensionaler Transfer von Information ummöglich ist?

    Falls unser Universum Teil eines Multiversums ist, das schon immer existiert hat, dann haben schon unendlich viele Intelligenzen existiert. Wenn wir von unendlich vielen Universen sprechen, dann haben auch wir schon exakt so wie wir sind unendlich oft existiert. Und in keiner “Runde” (BigBang, Expansion, Entstehung der Naturkontanten, Raum, Zeit, Galaxien, Planeten, Leben, Intelligenzen) ist eine Intelligenz entstanden, die es geschafft hat, die Endlichkeit zu überlisten und auszubüxen? Wenn es nur eine Intelligenz mal geschafft hätte, müssten es dann nicht auch unendlich viele andere geschafft haben?

    Macht die Antwort “Multiversum” nur Sinn, wenn man annimmt, dass es unendlich viele Universen gab, gibt und geben wird? (Weil man nur so die Fragen “Was war vor dem Nullpunkt” und “Was ist außerhalb des Universums” umgehen kann?)

    Und die Bonus-Frage: was würden Sie machen, wenn Sie zu einem Gott würden? (Forum-Gott zählt nicht :P)
    Ich verrat Ihnen schon mal, was ich machen würde: ich würde Gnade walten lassen und das Ganze abschalten. Denn ständig zuzusehen, wie Intelligenzen entstehen, die ihre eigene Endlichkeit vor Augen haben, ist ziemlich sadistisch.

  39. #39 Alex Gerstl
    Regensburg
    28. September 2014

    Hallo Florian!
    Toller Blog, Fan seit 2012…
    Meine Frage:
    Eigentlich WAR ich ein Fan des Zyklischen Universums, also Big Bang, dann Big Crunch, neuer Big Bang, weil damit (soweit ICH das verstehe die Frage, wieso ausegerechnet WIR hier so viel “Glück” haben mit Naturkonstanten etc., dass wir hier Leben haben ausgeschaltet ist, weil, NUR WIR uns Geadnken darüber machen können, EBEN WEIL hier Leben möglich ist und ALLES passt.
    Meine Frage nun: Nach der Entdeckung der dunklen Energie, die ja einen BIG CRUNCH unmöglich (?) macht: Ist es möglich, dass eben diese dunkle Energie dazu führt, dass wir nach 10 hoch 100 (?) Jahren ein quasi leeres Universum hätten, dass dann zu einem neuen BIG BANG führen könnte, also dann doch wieder zu einem zyklischen Universum? Vielen dank von einem naiven Bewunderer Alex

  40. #40 Alex Gerstl
    28. September 2014

    das ausgedünnte Universum sozusagen als Neubeginn eines VÖLLIG leeren Universums, das durch einen Quantenfluktuation zu einem Big Bang führt?

  41. #41 Danie
    Zwickau
    29. September 2014

    Hallo! Ich habe eine Frage, die die Astronomie aber auch ein wenig die Esoterik betrifft. Ich habe ein Buch gelesen, da lautete eine These”All das was ist, ist Gleichzeitig”. D.h die Zukunft aber auch die Vergangenheit. Dann habe ich eine Erklärung dazu gelesen, die besagte. Das Licht, was wir von den Sternen sehen, wurde vor vielen Jahren “gesendet”. Also ist die Vergangenheit noch irgendwie da. Könntest Du das noch ein bisschen greifbarer für mich machen, mit dem Licht was bei uns ankommt? Bitte 🙂 LG

  42. […] wird wieder Zeit, ein paar Fragen zur Astronomie zu beantworten und neben den diversen kosmologischen Fragen zum Anfang von Allem ist natürlich […]

  43. #43 Florian Freistetter
    6. Oktober 2014

    @Alex Gerstl: Hab gerade einen Artikel zur Zukunft des Universums geschrieben: https://scienceblogs.de/astrodicticum-simplex/2014/10/06/big-rip-big-bounce-und-waermetod-wie-endet-das-universum/

  44. #44 Alex Gerstl
    Regensburg
    8. Oktober 2014

    @Florian Freistetter, Vielen Dank für die ausführliche Antwort!!! Jetzt muss ich nur noch alles verstehen 😉

  45. #45 Hans-Walter Bronder
    Schiffweiler
    8. Oktober 2014

    Hallo Florian Freistetter,
    ich bitte herzlich um Deinen Kommentar zu meiner Frage vom 07. August 2014 : nachstehend nochmals der Text:
    Der Raum hat sich nach dem Urknall mit Überlichtgeschwindigkeit ausgedehnt. Könnte es daher nicht sein, dass wir heute theoretisch Bilder unserer eigenen Galaxie in ihrer Anfangszeit sehen könnten, da das Licht aus dieser Zeit uns jetzt erst erreicht ? Es stellt sich somit auch die Frage, ob wir dann nicht auch andere Galaxien “doppelt” sehen, also aus der Anfangszeit des Universums und der Zeit nach dem Abklingen der starken Anfangsexpansion und dass die Anzahl der Galaxien im Universum dadurch weitaus geringer ist, als bisher angenommen, was auch Auswirkung auf die Theorie von der dunklen Energie hätte ?

  46. #46 Florian Freistetter
    8. Oktober 2014

    @Hans-Walter: “ich bitte herzlich um Deinen Kommentar zu meiner Frage vom 07. August 2014”

    Wie im Text oben steht: Ich kann nicht versprechen, ob und wann ich auf Fragen antworten kann. Abgesehen davon, dass ich nicht auf jede Frage eine Antwort habe, möchte ich die Fragen ja auch in einem entsprechenden Artikel beantworten und das kann oft dauern. Aber wenn man irgendwelche “Spiegelbilder” von Galaxien sehen würde, dann hätte man das sicher schon gemerkt. (Nach sowas sucht man im Rahmen von Gravitationslinsen ja regelmäßig)

  47. #47 Hans-Walter Bronder
    Schiffweiler
    8. Oktober 2014

    @Florian:
    Vielen Dank für die Antwort !

  48. #48 Mirko
    HH
    8. Oktober 2014

    Frage: die Expansion des Jungen Universums wird der Ausdehnung des Raumes zugeschrieben. Die jetzige Ausdehnung der dunklen Energie. Meint man jetzt, dass die dunkle Energie heute auch den Raum dehnt oder nur die Materie auseinander treibt?

    Danke!

  49. #49 Florian Freistetter
    12. Oktober 2014
  50. […] den Fragen zur Astronomie, die mir bei diversen Anlässen gestellt werden, sind schwarze Löcher immer weit vorne mit dabei. […]

  51. #51 Thomas Freund
    Fürstenzell
    13. Oktober 2014

    Wie sehen Meteoriten von innen und ausen aus?
    und wie fiel würde ein 100 Gramm schwerer an Geld bringen?

  52. #52 SSRMKK
    Hanau/MKK
    14. Oktober 2014

    Mich würde interessieren, wovon es abhängt, ob ein Planet bzw. Mond eine Atmosphäre halten kann. Nur von der Masse? Wäre es insbesondere physikalisch möglich, den Mars mit einer Atmosphäre auszustatten?

  53. #53 Florian Freistetter
    14. Oktober 2014

    @SSRMKK: Gute Frage! Werde ich beantworten – aber wohl erst in ein bis vier Wochen (viel zu tun und viele Fragen…). Jeden Montag gibts im Blog eine neue Antwort!

  54. #54 Higgs-Teilchen
    15. Oktober 2014

    @Florian

    Lese gerade dein Buch “Der Astronomieverführer”.
    Kompliment! Abwechslungsreich und verständlich geschrieben. Dazu evt. eine Frage: Seite 75.
    Das Mondgestein sei dem Erdgestein “viel zu ähnlich”…..
    Wie genau ist das gemeint? Gibt es denn Zweifel an der Theia-Impakt-Theorie? Gibt es eine Aternative?

    Lg Higgs

  55. #55 Florian Freistetter
    15. Oktober 2014
  56. #56 Mirko
    HH
    15. Oktober 2014

    Eine Frage dazu, dass die Rotverschiebung als Beweis für die Ausdehnung der Raumzeit, sprich des Universums verwendet wird:
    Diese Ausdehnung soll die Ursache dafür sein, dass sich andere Galaxien und Sterne/Supernovas von uns entfernen und das Entfernen zu einem ‘Auseinanderziehen’ der elektromagnetischen Strahlung, also einer längeren Wellenlänge = Rotverschiebung führt. Mich wundert das insofern, dass Raumdehnung doch auch heißt, dass die Vergleichseinheit (für Wellenlänge das Meter) quasi gedehnt wird und die Wellenlänge vom Zahlenwert her gleich bleiben müsste? Das Meter seinerseits wird abgelitten von Sekunden, die durch einem bestimmten atomaren Zerfallsprozess definiert sind. Dieser Zerfall dauert bei einer Dehnung/Stauchung der Zeit auch länger/kürzer, ist aber in Sekunden ausgedrückt immer gleich lang.
    Wo steckt der Fehler?

    Danke, Gruß Mirko.

  57. #57 Peter
    D
    18. Oktober 2014

    Danke für Ihre Sternengeschichten!
    Hätten Sie nicht Lust mal eine über Astronomie und Gottesvorstellungen zu machen? Oder wie denken moderne Astronomen über Gott. Liegt es nicht nahe sich solche Fragen zu stellen, wenn man beim Urknall angelangt ist? Sowas in der Art. Auch ein Buchtip für einen interessieren Laien, wäre schön.
    In jedem Fall nochmals herzlichen Dank für die Sternengeschichten, die mir sehr viel Freude bereiten!

  58. […] den meisten Fragen aus meiner Serie “Fragen zur Astronomie” geht es um die eher “spektakuläreren” Themen wie schwarze Löcher oder den Urknall. […]

  59. #59 Wasserhund
    Chemnitz
    20. Oktober 2014

    Hallo!
    Meine Frage ist: Wieso ist es in unserer Galaxie so heiß? Die Materie der Lokalen Flocke hat eine Temperatur von ca. 6000 Kelvin, in der Lokalen Blase sind es sogar ca. 1,4 Millionen Kelvin. Woher kommen diese enormen Temperaturen und sollte diese Materie in der Kälte des Universums nicht längst abgekühlt sein?
    Danke und LG, Ronny

  60. #60 Roman Himmes
    21. Oktober 2014

    Sind die Bezeichnungen dunkle Materie und dunkle Energie nicht eigentlich nur Hilfskonstrukte um die Ahnungslosigkeit der aktuellen Physik zu verbergen? Können die Existenz dieser Hilfskonstrukte nicht aufzeigen, dass ein genereller Gedankenfehler im aktuellen Modell ist?

    Ein Beispiel war ja der sogenannte Äther, der mal als Hilfskonstrukt herhalten musste.

  61. #61 AmbiValent
    22. Oktober 2014

    @Roman Himmes

    Die Frage ist falsch gestellt. Es sind erst einmal Hilfskonstrukte, um bestimmte Beobachtungen zu beschreiben, die von den Erwartungen abweichen. Das dient aber nicht zum Verbergen einer Ahnungslosigkeit, sondern als Grundlage verschiedener Modelle, die zur Erklärung der Beobachtungen aufgestellt werden. Die Prognosen aufgrund dieser Modelle können dann mit neuen Beobachtungen verglichen werden, wodurch man schrittweise zu besseren Modellen kommt.

  62. #62 Florian Freistetter
    22. Oktober 2014

    @Roman: “Sind die Bezeichnungen dunkle Materie und dunkle Energie nicht eigentlich nur Hilfskonstrukte um die Ahnungslosigkeit der aktuellen Physik zu verbergen? Können die Existenz dieser Hilfskonstrukte nicht aufzeigen, dass ein genereller Gedankenfehler im aktuellen Modell ist?”

    Nein, sind sie nicht. Darüber habe ich schon sehr oft gebloggt:

    https://scienceblogs.de/astrodicticum-simplex/?p=10955
    https://scienceblogs.de/astrodicticum-simplex/2009/12/01/dunkle-energie-ist-keine-erfindung/
    https://scienceblogs.de/astrodicticum-simplex/2012/02/07/die-dunkle-materie-ist-keine-erfindung/

  63. #63 Cakir
    22. Oktober 2014

    Sorry, wusste nicht wo hin damit, aber bitte Florian, schau dir diesen Schwachsinn mal an. Schlechte Schlagzeilen!!

    https://www.focus.de/wissen/videos/ampeln-kanonen-krokodile-nasa-bilder-zeigen-spuren-mysterioeser-marskulturen_id_4215826.html

  64. […] ab? Und könnten wir das vielleicht irgendwann ändern? Die Frage, die ich heute in der Serie “Fragen zur Astronomie” beantworten will, lautet daher: Wovon hängt es ab, ob ein Himmelskörper eine Atmosphäre halten […]

  65. #65 Florian Freistetter
    27. Oktober 2014
  66. #66 Stephan
    27. Oktober 2014

    Terraforming: Können wir dem Mars eine neue Atmosphäre geben?
    Wäre es nicht möglich, einfach die Masse des Mars zu erhöhen? [das fliegen doch genug Asteroiden rum 😉

  67. #67 David
    28. Oktober 2014

    Was hältst du von Mars One?

  68. #68 Florian Freistetter
    28. Oktober 2014

    @David: Nix. Ist eine PR-Aktion. Da wird niemand je den Mars erreichen. Nichtmal den Weltraum.

  69. #69 Captain E.
    28. Oktober 2014

    @Stephan:

    Vertu dich mal nicht! Nehmen wir einmal den berühmten Asteroidengürtel zwischen Mars und Jupiter. Neben den relativ großen Brocken wie Ceres, Pallas, Juno und Vesta gibt es zwar noch unzählige weitere Brocken, aber zusammen machen die gerade einmal 5% der Masse unseres Mondes aus. Der Kuipergürtel hinter Neptun mag mehr Masse enthalten, die dafür aber sehr weit entfernt und weit verstreut ihre Bahnen zieht. Wie willst du damit die Masse des Mars erhöhen?

  70. […] der heutigen Frage zur Astronomie geht es um Sterne. Und um eine eigentlich sehr simple Frage: Wie groß ist ein Stern?. Und: Wie […]

  71. #71 Terraner
    6. November 2014

    hallo florian freistetter

    hat die wissenschaft schon eine ahnung wie dunkle materie beschaffen sein könnte, also zb gasförmig, kompakt, staubförmig etc, oder ist da etwas ganz neues?

  72. #72 Captain E.
    6. November 2014

    Klein, sehr klein. Die bereits bekannten Neutrinos, die durch die gesamte Erde fliegen können, ohne mit einem einzigen subatomaren Teilchen wechselzuwirken, aus denen die Erde, ihre Lufthülle und all ihre Lebewesen bestehen, dürften ein Teil dieser dunklen Materie sein. Es gibt aber nach aktuellen Abschätzungen nicht genug von ihnen, um die Beobachtungen zu erklären. Daher gibt es vermutlich weitere Teilchen mit ähnlichen Eigenschaften. Da ballt sich aber nichts zusammen, weder zu festen Körpern noch zu Flüssigkeiten, auch nicht zu Gas oder Plasma. Es gibt einfach nur diese winzigen Teilchen.

  73. #73 Florian Freistetter
    6. November 2014

    @Terraner: Ich habe (wie auch schon weiter oben gesagt) hier sehr ausführlich über die dunkle Materie geschrieben: https://scienceblogs.de/astrodicticum-simplex/2013/06/26/dunkle-welten-alles-uber-dunkle-materie-die-komplette-serie/

  74. #74 Erwin Anton
    6. November 2014

    Eine – vielleicht dumme – Frage zur Fluchtgeschwindigkeit.
    Die Fluchtgeschwindigkeit zum Verlassen der Erde beträgt 11,2 km/s; die nötige Geschwindigkeit um das Sonnensystem verlassen zu können beträgt ca 16,7 km/s;
    Voyager 1 ist zurzeit mit 17,037 km/s (lt Wikipedia) unterwegs. – kein Problem.
    Voyager 2 ist zurzeit mit 15,413 km/s unterwegs. Da Voyager 2 vorher nicht schneller war, sollte es ihr nicht möglich sein, das Sonnensystem zu verlassen. Andererseits ist sie jetzt schon so weit, dass die Gravitation der Sonne bereits soviel abgenommen hat, dass auch eine geringere Geschwindigkeit reichen müsste.

    Wann muss die Fluchtgeschwindigkeit erreicht werden?
    Würde ein Projektil, dass von der Erdoberfläche mit 11,2 km/s abgeschossen wird, trotz bremsender Atmosphäre die Erdanziehung überwinden können?
    Wo liegt mein Verständnisfehler?
    (obwohl ich in Mathematik maturiert habe, würde ich doch um sparsamen Einsatz von Formeln ersuchen.)

  75. #75 Alderamin
    7. November 2014

    @Erwin Anton

    Die Fluchtgeschwindigkeit zum Verlassen der Erde beträgt 11,2 km/s;

    Korrekt. Aus niedriger Höhe. Es gibt nicht die Fluchtgeschwindigkeit, diese hängt vom Abstand ab, da ja die Gravitation mit zunehmender Entfernung abnimmt. Die Fluchtgeschwindigkeit ist stets √2 mal die Kreisbahngeschwindigkeit. In niedriger Umlaufbahn ist die Kreisbahngeschwindigkeit der Erde 7,9 km/s, also die Fluchtgeschwindigkeit 1,414 * 7,9 km/s = 11,2 km/s. In Mondentfernung beträgt die Kreisbahngeschwindigkeit noch ca. 1,02 km/s, die Fluchtgeschwindigkeit 1,44 km/s. Das ist Dein Denkfehler.

    die nötige Geschwindigkeit um das Sonnensystem verlassen zu können beträgt ca 16,7 km/s;

    Nicht korrekt, aus der Erdbahn sind es 42,1 km/s. Da aber die Erde selbst schon mit 29,8 km/s unterwegs ist, braucht man nur die Differenz aufzubringen, man kann den Schwung der Erdbewegung um die Sonne nutzen, wenn man in die richtige Richtung startet. Das wären dann 42,1 – 29,8 km/s =12,3 km/s. Aber man muss zusätzlich die Gravitation der Erde überwinden, wenn man von ihr aus startet.

    Die Geschwindigkeiten addieren sich nicht linear, sondern die Energien tun das. Und zwar muss die Startgeschwindigkeit die Gesamtenergie für das Verlassen der Erde und der Sonne aufbringen. Die kinetische Energie zum Verlassen der Erde ist 1/2 m v(E)² (v(E) sei die Fluchtgeschwindigkeit der Erde aus niedriger Höhe). Die Energie zum Verlassen der Sonne ist 1/2 m v(S)² (v(S) ist die Fluchtgeschwindigkeit der Sonne aus Höhe der Erdbahn), wobei wir bereits durch die Erdbewegung mit der Kreisbahngeschwindigkeit der Erde v(e) unterwegs sind, es ist also nur 1/2 m (v(S)-v(e))² = 1/2 m Δv² aufzubringen. Dann ist die Gesamtenergie 1/2 m v(G)² = 1/2 m Δv² + 1/2 m v(E)². Wir kürzen 1/2 m heraus (die Hälfte der Masse der Rakete) und es bleibt v(G)² = Δv² + v(E)². Wurzel ziehen ergibt v(G) = √( Δv² + v(E)²) = √[(12,3 km/s)² + (11,2 km/s)²] = 16,6 km/s. Wenn man mit dieser Geschwindigkeit in Richtung der Erdbewegung um die Sonne diese verlässt, verlässt man auch das Sonnensystem.

    Voyager 1 ist zurzeit mit 17,037 km/s (lt Wikipedia) unterwegs. – kein Problem.
    Voyager 2 ist zurzeit mit 15,413 km/s unterwegs. Da Voyager 2 vorher nicht schneller war, sollte es ihr nicht möglich sein, das Sonnensystem zu verlassen. Andererseits ist sie jetzt schon so weit, dass die Gravitation der Sonne bereits soviel abgenommen hat, dass auch eine geringere Geschwindigkeit reichen müsste.

    Diese Geschwindigkeiten gelten relativ zur Sonne und richtig, weiter weg von der Sonne ist die Fluchtgeschwindigkeit viel kleiner. Als die Sonden starteten, waren sie 15 bzw. 14,5 km/s relativ zur Erde unterwegs (dazu kommen die 29,8 km/s der Erdbewegung). Das war weniger als die Fluchtgeschwindigkeit, aber die beiden Sonden haben sich an den großen Planeten zusätzlich Schwung durch Gravity-Assist-Manöver geholt. Da, wo sie jetzt sind, weit draußen in rund vierfacher Neptun-Entfernung von der Sonne, haben sie bei weitem genug Geschwindigkeit, um das Sonnensystem zu verlassen.

    Würde ein Projektil, dass von der Erdoberfläche mit 11,2 km/s abgeschossen wird, trotz bremsender Atmosphäre die Erdanziehung überwinden können?

    Nein, die Geschwindigkeit muss natürlich außerhalb der Atmosphäre erreicht werden, sie muss da erreicht sein, von wo aus das Raumschiff im freien Fall ungebremst den Keplerschen Gesetzen folgen kann. Die Atmosphäre ist dünn, in 36 km Höhe ist der Druck nur noch 1% des Drucks an der Erdoberfläche, in 100 km Höhe nur noch 0,00027%. In dieser Höhe würde die Fluchtgeschwindigkeit wohl schon reichen. Meist steigt man zuerst auf eine niedrige Erdumlaufbahn in 300-400 km Höhe, checkt die Raumsonde dort durch, und startet dann mit einer zweiten Brennphase um richtigen Zeitpunkt durch bis auf Fluchtgeschwindigkeit.

    So weit, so klar?

  76. #76 Erwin Anton
    7. November 2014

    recht herzichen Dank.
    die dritte kosmische Geschwindigkeit hatte ich aus wikipedia. dort steht nach “… für die notwendige Geschwindigkeit ergibt sich damit insgesamt 16,5 km/s” (die 16,7 muss ich woanders abgeschrieben haben). dann wird noch die Rotationsgeschwindigkeit berücksichtigt (ca 460m/s).
    Voyager 2 ist mit 14,5 km/s gestartet und hat durch ein “swing by Manöver” auf 15 km/s erhöht.
    Jetzt nur für mich zum Verständnis: Dieses Manöver wurde durchgeführt um schneller zum Saturn zu kommen (und für die weitere Mission). Für das erreichen der 3. kosmischen Geschwindigkeit war es nicht mehr notwendig, da diese auf Höhe des Jupiter bereits geringer als 14,5 gewesen wäre?

  77. #77 Alderamin
    7. November 2014

    @Erwin Anton

    Voyager 2 ist mit 14,5 km/s gestartet und hat durch ein “swing by Manöver” auf 15 km/s erhöht.

    Nein, Du vergleichst hier Geschwindigkeiten verschiedener Bezugssysteme. Voyager 2 ist mit 14,5 km/s relativ zur Erde gestartet (das ist die Geschwindigkeit, die die Rakete aufgebracht hat, deswegen ist diese Zahl wichtig). Relativ zur Sonne war die Sonde da mit ca. 44,3 km/s unterwegs. Diese (als “heliozentrisch” bezeichnete) Geschwindigkeit nahm dann bis zur Jupiterbahn auf nur ca. 10 km/s ab und hätte die Sonde nur noch bis zur Hälfte zwischen Jupiter und Saturn gebracht, dann wäre die Sonde zurück zur Erdbahn gefallen. Durch den Gravity Assist am Jupiter wurde die Geschwindigkeit auf über 20 km/s (heliozentrisch) erhöht und erst von da an war sie höher als die Fluchtgeschwindigkeit der Sonne. Danach folgten weiter Assists an Saturn und Uranus, die allesamt die Geschwindigkeit weiter erhöhten und die Flugzeiten zu den folgenden Zielen verkürzten. Am letzten Wegpunkt Neptun gab es noch einen Assist, der die Geschwindigkeit wieder etwas verringerte, aber die Richtung der Sonde aus der Ebene der Planetenbahnen heraus änderte (das Manöver diente, wenn ich mich recht entsinne, hautpsächlich zum Überflug einer der Pole des Neptunmondes Triton).

    Das kann man alles aus einem wunderbaren Diagramm in der englischen Wikipedia ablesen. Man sieht in dem Diagramm bei den Vorbeiflügen an den Planeten scharfe Spitzen, weil die Sonde natürlich im Anflug von den Planeten angezogen und beschleunigt wird, aber der größte Teil der Geschwindigkeitsänderung wird nach dem Passieren des Planeten wieder “aufgefressen”, weil die Schwerkraft des Planeten nach dem Passieren in Gegenrichtung zur Bewegung der Sonde wirkt und diese verlangsamt. Man sieht jedoch, dass die Kurve bei allen Assists außer Neptun danach höher verläuft als vorher, d.h. man hat Netto Geschwindigkeit gewonnen, indem man sich von der Bahngeschwindigkeit des Planeten ein Stück mitziehen ließ. Da, wo die rote Kurve oberhalb der blauen verläuft, wird die Fluchtgeschwindigkeit der Sonne übertroffen, die nach außen hin stetig abnimmt. Die Skala geht bis 40 AU, das ist etwa die mittlere Entfernung Plutos von der Sonne; da ist die Fluchtgeschwindigkeit nur noch ca. 5 km/s. Mittlerweile sind die Sonden schon bei ca. 107 und 129 AU und haben einen sehr hohen Geschwindigkeitsüberschuss gegenüber der Fluchtgeschwindigkeit.

    Wo sie sind und wie irre schnell sie unterwegs sind, zeigt diese schöne Seite. Die letzten Stellen bei den KM-Angaben sind volle km, das Komma dient im Englischen zum Trennen der Tausender-Stellen. Die stetigen Änderungen in diesen Stellen muss man sich mal plastisch auf der Erde vorstellen, und wie die kleinen Sonden sie im Tiefflug zurücklegen würden. Immer weiter so, dann erreicht sie in einigen zehntausend Jahren nahe Sterne (die Sonden sind nicht geradewegs zu irgendwelchen Fixsternen unterwegs, sondern passieren einige in großem Abstand).

  78. #78 Erwin Anton
    7. November 2014

    recht herzlichen Dank für diese ausführliche Erklärung (und Geduld) Durch den Hinweis auf das Diagramm hat sich die restliche Unsicherheit geklärt.

  79. […] zur bevorstehenden Kometenlandung der Rosetta-Mission gibt es heute in meiner Serie mit Fragen zur Astronomie eine typische Kometenfrage. Wenn ich in meinen Vorträgen über Asteroide, Kometen und andere […]

  80. #80 bruno
    10. November 2014

    was ist davon zu halten? 50% der Sterne ausserhalb ihrer Galaxien?
    https://www.bbc.com/news/science-environment-29917082

    Rein intuitiv kann ich mir schwer vorstellen, dass so viele Sterne durch was auch immer ihrer Galaxie entfliehen.
    Und gibt es grobe Schätzungen, wiewiele Planeten aus ihren Sonnensystemen geworfen wurden – und wieviele davon galaktische Fluchtgeschwindigkeit erreichten?

  81. #81 Florian Freistetter
    10. November 2014

    Naja, wenn man sich den Artikel durchliest, wird das Ende ja schon wieder deutlich relativiert. Ich kann dazu allerdings nicht viel sagen, da der Fachartikel in Science nicht frei zugänglich ist.

  82. #82 bruno
    10. November 2014

    …das alte Publikationsproblem…danke dir!

  83. #83 Stefan
    12. November 2014

    Ich habe eine recht simple Frage, ich hoffe nicht zu simpel:
    Was versteht der Astronom unter “Staub”?
    In einer der letzten Folgen die ich gehört habe hieß es, Sternschnuppen sind “Staub” der in der Atmosphäre verglüht.
    Staub ist für mich Zeug bei dem ich die einzelnen Körner mit bloßem Auge nicht identifizieren kann, also feiner als Sand.
    Es fällt mir etwas schwer mir vorzustellen das diese so hell verglühen dass man das auf der Erdoberfläche beobachten kann.
    Wenn man aber nun täglich mit Objekten wie Planeten und Sternen zu tun hat, nennt man aber ja vielleicht auch Objekte von der Größe eines Kleinwagen “Staub”, ok, Kleinwagen ist vermutlich übertrieben, aber Wassermelone? Tennisball? Erbse?

    Danke,
    Stefan

  84. #84 Alderamin
    12. November 2014

    @Stefan

    Staub im Weltall ist normalerweise ungefähr so fein wie die Partikel des Zigarettenrauchs. Es gibt sogenannte “Dunkelwolken”, die bestehen aus solchem Material. Aus ihnen können Sterne und Planeten entstehen.

    Die Teilchen, die sichtbare Meteore verursachen, stammen hingegen aus unserem Sonnensystem, zumeist von Kometen (oder evtl. Asteroiden), die sind größer, aber immer noch in der Größenordnung von Sandkörnern und kleinen Kieselsteinen. Kirschkerngroße Teilchen machen schon sehr helle Feuerkugeln, und alles, was deutlich größer ist, einen Boliden, der lange nachleuchtet. Von letzteren fallen dann auch manchmal Teile auf den Erdboden, ihre Meteore sind sogar am Tage zu sehen und verursachen gelegentlich einen hörbaren Donner (Überschallknall aus großer Höhe).

    Die Leuchterscheinung, die man am Himmel sieht (Meteor), ist nicht das glühende Teilchen selbst (“Meteoroid” genannt), sondern die davon ionisierte Luft, die das Teilchen umgibt und hinter ihm zurück bleibt. Diese wird wie in einer Leuchtstoffröhre zum Leuchten angeregt. Falls etwas von dem Meteoroiden auf den Boden fällt, nennt man dieses einen Meteoriten.

    Meteoroiden von mehr als 1 m Größe werden schon als Asteroiden bezeichnet. Mit (interplanetarem) Staub meint man Partikel kleiner als Sand. Davon fallen tausende Tonnen täglich auf die Erde (“Mikrometeroiden”), ohne dass man dies mit bloßem Auge sehen könnte.

  85. #85 Higgs-Teilchen
    12. November 2014

    @Florian

    Ich “meditiere” gerade über folgender Frage:
    Entfernen sich die Ringe der Planeten wie z.B. Saturn durch die Gezeitenkräfte von ihren Planeten? Wenn ja, wie viel pro Jahr? Oder ist das nicht messbar?

    Lg H.

  86. #86 Karin Triebscher
    12. November 2014

    Hallo,

    wenn man eine heutige Rakete (z.B. Ariane 5) nimmt und sie in Richtung Alpha Centauri schickte — wie lange wäre die unterwegs? (Swing-Bys sind erlaubt)?

  87. #87 Karin Triebscher
    13. November 2014

    Darf ich bitte noch was fragen?
    Eine Rakete (Masse = m) startet und schwenkt in eine Umlaufbahn ein, dann erst bricht sie aus und verlässt das Schwerefeld der Erde.
    Um wieviel mehr Treibstoff würde sie benötigen, wenn sie kerzengerade nach oben fliegen und straight on das Schwerefeld der Erde verlassen würde? Wie ist der Faktor?

    Danke für die Antwort im Voraus, falls Sie die Zeit finden!

  88. #88 PDP10
    13. November 2014

    @Karin Triebscher:

    “Hallo,

    wenn man eine heutige Rakete (z.B. Ariane 5) nimmt und sie in Richtung Alpha Centauri schickte — wie lange wäre die unterwegs? “

    Du kannst das pi-mal-daumen recht schnell selbst ausrechnen.

    Die “Fluchtgeschwindigkeit”, die eine Rakete braucht um das Gravitationsfeld der Sonne – also unser Sonnensystem – zu verlassen, ist ungefähr 17 km / s.
    Also auf diese Geschwindigkeit müsste man die Rakete mindestens beschleunigen um aus dem Sonnensystem raus und zum Alpha Centauri zu kommen.

    Jetzt rechne der Einfachheit halber mit einer Entfernung zum Alpha Centauri mit 4 Lichtjahren.

    Ein Lichtjahr sind ungefähr 9,8 Billionen Kilometer. Und das mal 4.

    Jetzt musst du nur noch die Entfernung durch die Geschwindigkeit teilen (kleine Hausaufgabe 🙂 ) und weisst, wie lange das dauert in Sekunden … das kannst du dann in Tage / Wochen / Jahre umrechnen …

    Also so ungefähr. Grössenordnungsmässig.

    PS: Ja ich weiss Florian … aber mein Rechenfinger juckt gerade …

  89. #89 PDP10
    13. November 2014

    @Karin Triebscher:

    Sag mal, du schreibst nicht zufällig morgen eine Physik Klausur?

  90. #90 krypto
    13. November 2014

    @PDP10#88:
    …Wobei die übrigbleibende Reisegeschwindigkeit äußerst gering ausfällt…

  91. […] den “Fragen zur Astronomie” geht es heute um ein Phänomen, das man in so gut wie jeder klaren Nacht beobachten kann: […]

  92. #92 Gerhard
    München
    18. November 2014

    Kann es erd-ähnlichen Planeten mit einem flüssig-wasser-Klima überhaupt in einem Doppelsternsystem geben?

    Meine Basisgedanken:
    1. Der Planet sollte natürlich auch ein ähnliches Alter haben, wie die Erde, dann müßte eine der Sonnen auch unserer Sonne ähneln?
    2. Die Zweite Sonne dann auch? (sehr mutige These)
    3. Im Orbit um eine Sonne wirds dann sehr heiß?
    4. Im Orbit um beide Sonnen wirds dann sehr kalt?

  93. #93 Anderland
    20. November 2014

    Bzgl. der Beschaffenheit von Kometen möchte ich gerne folgend Frage stellen.

    Was ist im o.g. Zusammenhang eigentlich an so ‘nem Artikel dran?

    https://farsight3.wordpress.com/2014/11/12/esa-kometen-mission-rosetta-basiert-auf-falschem-glauben/

    Genauer gesagt, wie gut oder schlecht sind die dort aufgeführten Argumente bzgl. Elektrizität und Plasma (unabhängig davon, daß die Site nicht besonders wissenschaftlich ausschaut)?

  94. #94 Alderamin
    20. November 2014

    @Anderland

    Das ist völliger Unsinn. Z.B. dass Kometen aus Gestein bestünden und das Wasser erst außerhalb des Kometen vom Sonnenwind “erzeugt” werde.

    Man weiß sehr genau, woraus Kometen bestehen, man hat bei mehreren das Verhältnis von schwerem Wasser zu normalem Wasser bei ihnen gemessen, und die Werte sind individuell verschieden, der Wasserstoff kommt also sicher nicht alle von der Sonne. Das Wasser, das der Komet Shoemaker-Levy 9 1994 in der Jupiteratmosphäre deponiert hat, ließ sich noch 2013 nachweisen (unmittelbar nach dem Einschlag sowieso).

    Man wusste aber bisher nicht, in welcher Form das Eis auf der Oberfläche des Kometen vorliegt, ob locker geschichtet oder dicht gepackt, und dieser Komet ist nun auch schon ein paarmal im inneren Sonnensystem gekreist und von der Sonne erwärmt worden, man weiß also immer noch nicht, ob ein frischer Komet aus der Oortschen Wolke genau so beschaffen ist oder nicht. Aber nun hat man etwas gelernt, und das ist doch der Sinn der Wissenschaft. Wenn man alles schon vorher wüsste, bräuchte man sie doch gar nicht.

    Außerdem besteht der Mond auch aus Gestein und ist dem Sonnenwind ausgesetzt, warum hat der dann keine Koma und keinen Schweif?

    Diese Webseite braucht niemand ernst zu nehmen.

  95. #95 Anderland
    20. November 2014

    @Alderamin: vielen lieben Dank für die prompte Antwort

  96. #96 PDP10
    20. November 2014

    @Alderamin:

    Langsam nervt der Schwachsinn mit dem elektrischen Universum echt!

    Ich frage mich so langsm wo der Unsinn überhaupt herkommt.
    Das erste mal, das ich davon gehört habe war hier im Blog und ich habe mich sofort gefragt, ob die Komiker, die das propagieren eigentlich die Maxwellschen Gleichungen hinschreiben können.
    Wenn sie das könnten müssten sie eigentlich gemerkt haben, dass das nicht funktioniert.

    Seltsame Sache das.

    Schwachsinn kann man offensichtlich nicht ausrotten und das Internet ist daran ganz gewiss nicht schuld.
    Die ganzen Deppen waren vorher schon da.
    Die konnten nur nicht irgendwo was hinschreiben, wo’s jeder lesen kann …

  97. #97 Mirko
    HH
    21. November 2014

    Hallo,

    Eine Frage in Sachen Gravitation: Ein wesentliches Ziel aktueller Physikforschung ist (zB laut Wikipedia), die Gravitation mit den übrigen Wechselwirkungen zu einer „Großen Vereinheitlichten Theorie“ (GUT) zu vereinen, um somit eine Theorie zu formulieren, die alle Naturkräfte auf einmal beschreiben kann. Laut ART ist die Gravitation doch aber nur eine Scheinkraft. Wie passt das zusammen?

    Danke!

  98. #98 Matthias
    Fellbach
    21. November 2014

    Hallo,
    ich habe zwei Fragen zur Raumfahrttechnologie, die wahrscheinlich total trivial sind.

    (1) Alufolienknüddl. Mir ist aufgefallen, dass Satelliten und Sonden häufig mit goldfarbener Folie umwickelt sind. Warum macht man das, und vor allem, warum sieht das so aus, als wenn ich zu Hause einen Schokokuchen mit Alufolie als Reiseproviant einpacke. Das ganze wirkt auf mich irgendwie lieblos und improvisiert. Aber darfür gibt es bestimmt einen guten Grund, warum auf viel Falten und Knicke Wert gelegt wird. Wäre ein passend geformtes Compartment nicht besser und robuster?

    (2) Außerdem ist mir aufgefallen, dass beim Start einer Rakete immer irgendwelches weißes Zeug von der Raketen-Außenhat abfällt. Das ist sowohl in Raketenstarts der letzten Jahre, als auch in historischen Aufnahmen erkennbar. Auch beim Film Apollo13 wurde das wohl aus dramaturgischen Gründen gezeigt. Warum und vorallem was fällt denn da runter?

  99. #99 Alderamin
    21. November 2014

    @Matthias

    zu (1): die Folie reflektiert Sonennlicht und hält die Sonnenwärme draußen, die innere Wärme drinnen. Kennt man auch als Rettungsdecke im Auto-Verbandkasten, die Verletze warm (oder kühl) hält und ungemein platzsparend unter zu bringen ist. Der Vorteil der Folie ist, dass sie praktisch nichts wiegt (im Gegensatz zu einem “Compartment”). Wenn man bedenkt, dass ein Kilogramm Masse zum niedrigen Erdorbit heute noch $10000 – $20000 kostet (tiefer in den Weltraum noch deutlich mehr), ist das eine Menge gespartes Geld.

    zu (2): Was da herunter fällt sind Eisplatten. In einem der Tanks befindet sich flüssiger Sauerstoff, in dem anderen (meistens, wenn’s kein Kerosin ist) flüssiger Wasserstoff. Diese lassen sich nur bei ca. -250°C lagern (wenn man keinen Hochdrucktank verwendet, der natürlich in dieser Größe viel zu schwer wäre). Man kann sich vorstellen, dass trotz Isolation die Rakete außen ziemlich abkühlt und da im feuchten Florida durch Kondensation Eisplatten entstehen, die bei den Vibrationen des Starts abplatzen. Auch sieht man häufig Dämpfe austreten, das ist verdampftes Gas, das aus den Tanks zum Druckausgleich abgelassen wird und die umgebende Luft durch Abkühlung zur Nebelbildung bringt.

  100. […] Das sind alles gute Fragen und ein paar davon will ich im heutigen Artikel aus der Serie “Fragen zur Astronomie” […]

  101. #101 ImNetz
    Zimba
    25. November 2014

    @Florian
    Bitte um Antwort auf meine Frage zur Verteilung der Materie im Universum soweit momentan bekannt/vermutet:
    In Plasmaphase (> 90% … ?)
    feste Phase
    flüssige Phase (“exotisch” wie flüssiges Methan?)
    gasförmig …
    ????

    Merci

  102. #102 Florian Freistetter
    25. November 2014

    @ImNetz: Ich denke nicht, dass sich diese Frage sinnvoll beantworten lässt. Wenn du dunkle Materie ignorierst, dann ist die überwiegende Mehrheit der Materie in Form von Sternen (also Plasma) organisiert. Das bisschen, was Planeten ausmachen fällt da nicht ins Gewicht…

  103. #103 mundes
    hagen
    26. November 2014

    die peitsche gottes
    wenn mich jemand nach dem zustand vor dem urknall fragt, sage ich ihm immer “nimm eine peitsche und lass sie knallen. welches geräusch gab es davor ? keins.
    erst das zusammentreffen von verschiedenen zuständen und parametern hat den knall ermöglicht.
    nun aber zu meiner frage.
    könnte es auch jetzt zu einem “urknall kommen, wo das universum schon existiert, oder muss dafür alle masse oder energie auf einem punkt konzentriert sein.

  104. #104 Alderamin
    26. November 2014

    @mundes

    könnte es auch jetzt zu einem “urknall kommen, wo das universum schon existiert

    Grundsätzlich ja. Nach Alan Guth würde ein hinreichend dichtes Planckvolumen von selbst eine kosmische Inflation auslösen und wohlmöglich sogar ein ganzes Multiversum erzeugen. Laut Guth würde solchermaßen entstehender Raum sich von unserem Raum abschnüren und hätte dann keinen Kontakt mehr mit unserem Universum. Es könnte sogar sein, dass innerhalb schwarzer Löcher genau ein solcher Prozess abläuft und neue Universen entstehen, was allerdings hochspekulativ ist.

  105. #105 Ferrer
    30. November 2014

    Wunschgemäß hier on-topic (danke für den Hinweis auf diese Sektion) und hoffentlich genauer formuliert: Auf der Erde sieht man regelmäßig Meteorschauer, manche, die jährlich an bestimmten Tagen vorkommen, haben eigene Namen (Leoniden, Perseiden…). Ich habe gelesen, dass an diesen Tagen die Erde die Reste eines Kometenschweifs durchkreuzt; da die Erde sich um die Sonne dreht, passiert das immer zur selben Jahreszeit. Und nun meine Frage: Wie kommt es, dass sich die Kometenschweifreste auf der Bahn, die der Komet durchflog, nicht ebenfalls um die Sonne drehen? Denn wenn sie sich drehen, dann kreuzt die Erde den Schweifrest auf der alten Bahn nicht am selben Datum, da müssten sie woanders sein (so definiert man landläufig Bewegung, nicht wahr?) Wenn sich die Partikel nicht um die Sonne drehen, müssten sie gravitationsbedingt in die Sonne fallen, aber wenn sie sich doch um die Sonne drehen, würde die Erde sie nicht immer am selben Datum durchkreuzen. Hast Du eine Erklärung für diesen sicher nur scheinbaren Widerspruch?
    Anders gefragt: Wie bewegt sich der Schweif eines Kometen um die Sonne verglichen mit der Bahn eines Planeten? Gelten für winzige Partikel die Keplerschen Gesetze nicht (z.B. weil der Sonnenwind die Gravitation überlagert)?

  106. #106 Florian Freistetter
    30. November 2014

    @Ferrer: Ok, ich hatte gehofft, meine Erklärung nebenan hätte schon gereicht. Eine andere kann ich dir auch nicht geben. Ich weiß auch nicht, was du mit “drehen” meinst. Staub bewegt sich um die Sonne, wie auch die Erde. Der Komet verliert auf seine Bahn ständig Staub und deswegen ist die ganze Bahn voll mit Staub der sich entlang der Kometenbahn um die Sonne bewegt. Wenn die Erde diese staubige Kometenbahn kreuzt, gbt es Meteorschauer. Und das tut sie jedes Jahr zur gleichen Zeit.

  107. #107 Alderamin
    30. November 2014

    @Florian

    Nebenan schriebst Du:

    Der Komet verliert ja nicht nur einmal einen Haufen Staub, der dann als “Klumpen” um die Sonne kreist, sondern verliert konstant Staub.

    Ein Komet verliert aber vor allem in Sonnennähe besonders viel Staub, der ihn dann zunächst als “Klumpen” begleitet. Deswegen sind die Leoniden alle 33 Jahre besonders intensiv, wenn die Erde mit dem Staub eines der vergangenen Perihelia des Kometen Tempel-Tuttle zusammentrifft. In anderen Jahren sind die Leoniden eher enttäuschend.

    Es dauert eine Weile, bis sich die Teilchen, die vom Kometen mit einer Anfangsgeschwindigkeit ausgestoßen wurden und auch vom Sonnenwind beeinflusst werden, über die ganze Kometenbahn verteilt haben, so dass es bei jedem Durchlauf der Erde durch den Kreuzungspunkt mit der Kometenbahn einen verlässlichen Meteorschwarm gibt. Ich denke, diese Info fehlt Ferrer noch zum Verständnis der Meteorschwärme.

  108. #108 Florian Freistetter
    30. November 2014

    @Alderamin: ” so dass es bei jedem Durchlauf der Erde durch den Kreuzungspunkt mit der Kometenbahn einen verlässlichen Meteorschwarm gibt. Ich denke, diese Info fehlt Ferrer noch zum Verständnis der Meteorschwärme.”

    Dass sich der Staub über die KOmetenbahn verteilt, habe ich ja erwähnt. Ich wollte das nur nicht so kompliziert machen, mit Strahlungsdruck und Perihel und so weil ich das Gefühl hatte, dass die Verständnisprobleme hier wesentlich grundlegender angesiedelt waren.

  109. […] Beispiel den Voyager-Sonden – benutzt wird? Eine gute Frage und daher gibt es in meiner Serie “Fragen zur Astronomie” diese Woche darauf eine […]

  110. #110 Higgs-Teilchen
    4. Dezember 2014

    Weiß man eigentlich was von Planeten auf denen es schneit, bzw. schneien kann? Außer der Erde natürlich 😉

  111. #111 Florian Freistetter
    4. Dezember 2014

    @Higgs-Teilchen: Schöne Frage! Die werde ich am Montag vor Weihnachten in der Artikelserie beantworten – da passt sie schön 😉

  112. #112 Gerhard
    4. Dezember 2014

    Weiß man eigentlich was von Planeten auf denen es schneit, bzw. schneien kann?

    Wie sieht das in systemem mit mehreren Sonne aus? Heist Schneien = habitable Zone ? Besteht Schnee immer aus Wasser?

    Ich freu mich auf Weihnachten!

  113. […] geht es in der Serie “Fragen zur Astronomie” um den Mond. Das ist der einzige Himmelskörper, den wohl jeder sofort und ohne Probleme am Himmel […]

  114. #114 ulfi
    8. Dezember 2014

    Ich hab eine kleine Frage:

    Als ich heute späten nachmittag/abend (17:10 ca) abhob konnte ich durch die Flughöhe den Mond, bzw den Mondaufgang sehen. Allerdings hat mich die Mondfarbe etwas irritiert: sie war eher gelblich/orange. Der Effekt verschwand innerhalb weniger Minuten als der Mond etwas höher stieg. Kann das sowas wie eine Mondfinsterniss gewesen sein?

  115. #115 Alderamin
    9. Dezember 2014

    @ulfi

    Nein, das ist nur der längere Lichtweg durch die Atmosphäre, der auch die Sonne rot auf- und untergehen lässt. Die Lufthülle ist ja recht dünn, die Dichte nimmt mit der Höhe schnell ab, und so blickt man effketiv nur durch ein paar Kilometer Luft, wenn man schräg oder senkrecht nach oben in den Weltraum schaut. In Richtung des Horizonts ist der Lichtweg durch die Lufthülle jedoch hunderte Kilometer lang, und weil die Luft das kurzwellige blaue Licht stärker streut als das langwellige rote, wird rötliches Licht besser durchgelassen. Staub und Dunst tragen ebenfalls zur Färbung von Sonne und Mond bei, deswegen sehen Sonnen- und Mondaufgänge immer wieder verschieden aus.

  116. #116 Florian Freistetter
    9. Dezember 2014

    @ulfi: “Kann das sowas wie eine Mondfinsterniss gewesen sein?”

    Ne – das liegt daran, dass das Licht mehr Atmosphäre durchqueren muss, wenn der Mond tief steht und deswegen rötlicher erscheint. Siehe hier: https://scienceblogs.de/astrodicticum-simplex/2013/12/18/was-ist-besser-vollmond-im-winter-oder-im-sommer/

  117. #117 Lothar
    Ludwigshafen
    10. Dezember 2014

    Meine Fragen gehen zu optischen Großteleskopen.

    In der Radioastronomie kann man entfernt voneinander liegende Teleskope zu einem großen Verbund zusammenschließen und erzielt dadurch eine viel größere Auflösung. Geht so was auch bei opt. Großteleskopen, die weiter auseinander liegen?
    Im Bau sind Teleskope mit 20, 30 und 40 Meter Spiegeldurchmesser. Wird es da in der Zukunft Grenzen der Größe und Machbarkeit geben, wo liegen hier die techn. Grenzen? Wäre ein Spiegeldurchmesser von 100 oder mehr Metern machbar und auch sinnvoll. Weiterer mmöglicher Erkenntnisgewinn wäre dann von techn. Realisierbarkeit abhängig. Wäre das das Ende der Astronomie, wenn größer nicht mehr ginge? Vielleicht vergleichbar mit an die Grenzen stoßen bei der Teilchenphysik in Cern, wo ein noch deutlich größerer Beschleuniger auch schon fast nicht mehr denkbar und finanzierbar ist.
    1o. Dez. 2014

  118. #118 Mark S
    10. Dezember 2014

    Bahnmechanik (Vereinfacht zum Mitrechnen). “Früher” brauchte es drei Tage um zur ISS zu kommen, nun fliegt Orion binnen Stunden in 5800km Höhe und fällt “genauso schnell” wieder zurück(!!). Wie “weit” wäre Orion mit einer sinnvollen Flugbahn gekommen?

  119. #119 Gerhard
    10. Dezember 2014

    Bei Erinnerung an den Pysikuntericht ist mir mal wieder eingefallen, dass sich geworfene Objejte auf der Erde auf einer Parabelbahn bewegen, laut Newton’s Gravitationsgesetz.
    Wenn ich aber einen Planeten ins Sonnesystem werfe ist es eine Elliptische Bahn.
    Da beides auf Newtons Gesetzen beruht – mßten dann nicht auch die Objekte auf der Erde in elliptischen Bahnen fliegen ?

  120. #120 Alderamin
    10. Dezember 2014

    @Mark S

    Die Flugzeit hat weniger mit der erreichten Höhe zu tun (die niedrige Erdumlaufbahn von ca. 400 km erreicht eine Rakete in guten 10 Minuten) als vielmehr damit, dass man die ISS einholen muss und zwar auf der gleichen Bahn. Wenn man aber auf der gleichen Bahn unterwegs ist, ist man auch ziemlich genau so schnell unterwegs, also dauert es lange, die Entfernung zu überbrücken. Da man möglichst wenig Treibstoff mitnehmen will (jedes Kilo in den Orbit kostet $10000 bis $20000), beschränkt man sich auf kleinste Korrekturen der Bahn, man kann nicht einfach Gas geben, die ISS einholen und dann wieder abbremsen, sondern wählt eine Bahn, die der ISS irgendwann mit geringem Geschwindigkeitsunterschied begegnet und dockt dann nur noch mit Hilfe der kleinen Steuerdüsen an.

    Mittlerweile können die Russen ihre Soyuz genauer steuern, treffen die ISS-Bahn auf Anhieb besser und schaffen das Andocken schon nach ca. 6 Stunden (was für die Insassen der engen Soyuz ein Segen ist).

    Die Orion hat man einfach auf eine möglichst große Höhe geschossen und hat dabei noch eine Extra-Umkreisung um die Erde eingelegt, zum Durchchecken und weil man das Wiederstarten der Antriebsstufe auch mit testen wollte. Die Orion-Kapsel war dabei mit rund 9 km/s unterwegs, während man im niedrigen Orbit mit 7,8 km/s kreist. So dramatisch viel schneller war die Orion also nicht unterwegs. Mehr war nicht möglich, weil derzeit keine stärkere Rakete verfügbar ist (und dabei war die Orion ziemlich leer, keine Sitze, keine Bedienkonsole, keine Fracht, nichts). Die zugehörige SLS-Rakete startet erst 2018.

  121. #121 Alderamin
    10. Dezember 2014

    @Gerhard

    Das täten sie, wenn der Erdboden nicht im Weg wäre. Die Parabelbahn gilt näherungsweise für ein konstantes Schwerefeld mit parallelen Feldlinien, als das, was man auf kleinem Raum näherungsweise hat. Wenn die Erde auf einen winzigen Punkt geschrumpt würde, aber ihre Masse beibehalten, dann würde ein geworfener Gegenstand auf diesen Punkte zu fallen und dabei immer schneller werden. Hat er dann eine Seitwärtskomponente, würde er den Punkt verfehlen, um ihn herum fallen (wobei ihn die Fliehkraft bei der hohen Geschwindigkeit nach außen drückt) und dann mit dem Schwung des Absturzes wieder zurück zum Ausgangspunkt zurück steigen. Das wäre dann die Ellipse.

    Das gleiche gilt für einen Satelliten, da ist die Seitwärtskomponente der Bewegung lediglich so groß, dass er immer an der Erde vorbeifällt. Natürlich will man meistens eine Kreisbahn erreichen, das ist aber einfach nur eine spezielle Form der Ellipse.

    Auch im Weltraum gibt es Parabelbahnen, und zwar, wenn ein Objekt mit Anfangsgeschwindigkeit 0 aus dem Unendlichen kommt, dann wird die Ellipse beliebig lang und verwandelt sich in eine Parabel.

  122. #122 Alderamin
    10. Dezember 2014

    @Lothar

    In der Radioastronomie kann man entfernt voneinander liegende Teleskope zu einem großen Verbund zusammenschließen und erzielt dadurch eine viel größere Auflösung. Geht so was auch bei opt. Großteleskopen, die weiter auseinander liegen?

    Bis jetzt noch nicht, da müssen die Lichtwege analog zusammengebracht werden. Bei der Radioastronomie kann man die Signale hinreichend genau mit Zeitmarken abtasten und aufzeichnen und die Interferenz dann im Rechner simulieren. Optisch geht das wegen der kurzen Wellenlänge nicht – man müsste die Zeit auf einen Bruchteil einer Wellenlänge genau aufzeichnen und die Wellen mit mindestens ihrer doppelten Frequenz abtasten, und dann auch noch die entsprechenden Datenmengen speichern. So weit sind wir noch nicht. Aber die Lichtwege benachbarter Teleskope können über Spiegelsysteme an einem Ort zusammengebracht werden, das geschieht z.B. schon beim Very Large Telescope auf dem Cerro Paranal in Chile. Bei Keck 1 und 2 auf Hawaii ebenso.

    Im Bau sind Teleskope mit 20, 30 und 40 Meter Spiegeldurchmesser. Wird es da in der Zukunft Grenzen der Größe und Machbarkeit geben, wo liegen hier die techn. Grenzen? Wäre ein Spiegeldurchmesser von 100 oder mehr Metern machbar und auch sinnvoll.

    Es gab mal eine Studie für ein Overwhelmingly Large Telescope das genau so groß hätte werden sollen. Klar träumen die Astronomen davon, mehr Öffnung ist immer sinnvoll, aber die technischen Probleme bei so einem Projekt würden es wohl extrem teuer machen. So werden nun immerhin das E-ELT mit 39 m Öffnung und das GMT (25 m) und TMT (30 m) gebaut. Wenn man mit denen Erfahrung und Daten gesammelt hat, vielleicht werden dann zukünftig auch einmal noch größere Teleskope gebaut werden.

    Wäre das das Ende der Astronomie, wenn größer nicht mehr ginge?

    Sicher nicht. Erstens wird auch heute noch mit sehr viel kleineren Teleskopen beobachtet, es tut sich ja auch immer mal wieder was neues am Himmel, und mit der heutigen Digitaltechnik hat man viel bessere Möglichkeiten als früher. Zweitens entwickeln sich die Sensoren weiter, gerade neulich las ich von einem Sensor, der neben dem Bild auch gleich ein Spektrum aufnimmt, da jedes Photon mit seiner individuellen Energie registriert wird. Oder bei der Messung von Planeten durch den Dopplereffekt, da wird die Genauigkeit der Spektrometer immer besser, die entdeckbaren Planeten immer kleiner. Drittens eröffnet der Weltraum noch Möglichkeiten, in Frequenzen zu beobachten, die die Atmosphäre nicht durchdringen (Röntgen- und Gamma-Teleskope “sehen” z.B. noch recht unscharf), oder dort Interferometer zu stationieren (wie den früher geplanten Terrestrial Planet Finder). Man könnte viertens sogar eines Tages die Sonne als Linse eines Gravitationsteleskops nutzen. Da ist noch viel Luft.

  123. #123 Crazee
    12. Dezember 2014

    Ich bastele gerade an einem Quartettspiel mit Himmelskörpern und da habe ich mal eine seltsame Frage, die dabei auftauchte:

    Wie groß ist die scheinbare Helligkeit eines schwarzen Lochs? Unendlich? Oder gibt es durch die abgegebene Strahlung da doch einen Wert? Durch googeln bin ich irgendwie nicht auf ein Ergebnis gestoßen.

  124. #124 A-P-O
    15. Dezember 2014

    Warum ist der Betrag der Lichtgeschwindigkeit 299 792 458 m/s?
    Warum nicht 400 000 000 m/s oder nur 30 000 m/s?
    Anders gefragt: Was bestimmte beim Big Bang diesen Betrag?

  125. […] der Serie “Fragen zur Astronomie” geht es heute um ein Thema, das viele Menschen zu beschäftigen scheint. Es ist jedenfalls eine […]

  126. #126 Lothar
    Ludwigshafen
    15. Dezember 2014

    Ich habe Fragen zu Exoplaneten und dem Nachweis von Leben auf diesen bis zu “Außerirdischen Zivilisationen”.

    Wenn es uns schon wegen der großen Entfernungen nicht gelingen wird auch in Zukunft dorthin zu fliegen, ob bemannt oder unbemannt, so frage ich mich, was können wir durch unsere immer bessere Technik von der Erde oder vom Erdorbit aus entdecken und nachweisen. Wo werden in Zukunft da unsere Erkenntnisgrenzen liegen. Sollten wir anhand von z.b. Atmosphärenanalysen incl. “Biomarkern” sagen können (Planetengröße, habitable Zone usw…schon mitberücksichtigt) auf diesem Planeten könnte Leben existieren ? Werden wir je sagen können, wenn dort Leben existiert, so ähnlich wie wir es von unserer Erde kennen, welche Organisationsstufe das Leben dort erreicht haben könnte, incl. der Entwicklung einer techn. Zivilisation? Wir würden gerne wissen, gibt es sowas wie unseren schönen blauen Planeten mit seiner Flora und Fauna, nochmals in unserer Heimatgalaxie. Ich bin mir bewußt, daß die Exoplanetenforschung noch nicht lange betrieben wird und neue sehr leistungsstarke Teleskope bald zum Einsatz kommen. Wird es je möglich sein ferne Exoplaneten wenigstens annäherungsweise so zur opt. Darstellung zu bringen, wie die Planeten in unserem Sonnensystem oder werden das immer nur ganz winzige Lichtpunkte bleiben, egal wie groß unser Teleskop in Zukunft sein wird ?
    15.Dez.2014

  127. #127 Diana
    26. Dezember 2014

    Ich habe gerade ein Bild gesehen, in dem eingetragen wurde wie weit hinaus ins Weltall (in Lichtjahre) die Radioübertragungen von Christmassongs der letzen Jahrzenten übertragen wurden – also was die Aliens von uns hören.

    So und nun Frage ich mich umgekehrt, würden wir überhaupt ein Radioprogramm von Aliens bemerken? Ich meine natürlich nicht, dass ich das in mein heimischen Radio empfangen kann, sondern ehr ob wissenschaftliche Gerätschaften dies könnten und wenn ja ob uns das überhaupt auffallen würde?

    Umgekehrt müssten ja fortgeschrittene Aliens eine ähnliche Entwicklung hinlegen wie wir und eine Art Kultur haben. Und irgendwann ähnlich Unterhaltungsgeräte/Kommunikationsgeräte entwickeln wie wir. Also könnten da draußen die “Radioprogramme” von Aliens rumschwirren und wir diese, wenn deren Planeten nahe genug ist, auch empfangen?

  128. #128 Florian Freistetter
    26. Dezember 2014

    @Diana: Eine gute Frage! Die werde ich im nächsten Artikel aus der Serie ausführlich beantworten; Mitte Januar, wenn ich vom Urlaub wieder zurück bin!

  129. #129 Alderamin
    26. Dezember 2014

    @Lothar#117, @myself #122

    Aber die Lichtwege benachbarter Teleskope können über Spiegelsysteme an einem Ort zusammengebracht werden, das geschieht z.B. schon beim Very Large Telescope auf dem Cerro Paranal in Chile. Bei Keck 1 und 2 auf Hawaii ebenso.

    Wie ich in der Januar-Sky&Telescope gelesen habe, hat man 2012 das Keck-Interferometer nach 10 Jahren Verzögerung stillschweigend eingestampft. Es gab Probleme mit der Baugenehmigung (heiliger Berg der Hawaiianer, Widerspruch von Umweltschützern) und der Finanzierung (konkurrierende Projekte) der Zusatz (“Outrigger-“) Teleskope sowie mit der Technik.

  130. #130 Alderamin
    26. Dezember 2014

    @Lothar

    Wird es je möglich sein ferne Exoplaneten wenigstens annäherungsweise so zur opt. Darstellung zu bringen, wie die Planeten in unserem Sonnensystem oder werden das immer nur ganz winzige Lichtpunkte bleiben, egal wie groß unser Teleskop in Zukunft sein wird ?

    Die Planeten im Sonnensystem sind ein paar AU entfernt, die nächsten Sterne ein paar hunderttausend bis Millionen AU. Um auf diese Entfernung Planeten auflösen zu können, müssten unsere Teleskope also hunderttausende bis millionen Mal bessere Auflösung haben und entsprechend größer sein. Theoretisch wäre dies möglich, wenn man mehrere optische Teleskope an gegenüber liegenden Orten der Welt zur Interferenz bringen könnte. Dazu müsste man ihr Licht mit doppelter Frequenz aufzeichnen (also mit ca 10^15 Hz, eine Million Samples pro Nanosekunde) und im Rechner interferieren lassen. Ich sehe im Moment nicht, wie das technisch möglich sein soll (Speicherzugriffe werden immer noch in Nanosekunden gemessen), aber vielleicht ist die Technik in ein paar hundert Jahren so weit.

    Vielleicht findet man auch einen anderen Weg, etwas das Signal per Laser über Satelliten an einem Ort zusammen zu bringen, was bei Weltraumteleskopen vielleicht am ehesten machbar wäre (deren Position zueinander dann auf 250 Nanometer genau bekannt sein müsste). Sehe ich aber auch nicht für die absehbare Zukunft.

    Also wird man weiter auf Spektroskopie und Biomarker setzen müssen. Alleine dies muss erst einmal verwirklicht werden. Die neuen im Bau befindlichen Großteleskope sollten dazu in der Lage sein.

  131. […] Antworten findet ihr auf der Übersichtsseite zu den Fragen, wo ihr selbst auch Fragen stellen […]

  132. #132 Bernd Heilmann
    50259 Pulheim Wupperstr.92
    29. Dezember 2014

    Frage: Wie entsteht Licht oder wie enstehen die Bereiche des elektromagnetischen Spektrums?

  133. #133 Gegenschein (Sabine)
    Hofbieber, Deutschland
    30. Dezember 2014

    Hallo Florian. Meine Frage lautet: wohin geht das (künstliche) Licht, wenn ich meine Taschenlampe wieder aus mache? Saust es für immer durch das Universum, wenn es nicht auf einen Gegenstand trifft, der das Licht aufnimmt???
    sternreiche Grüße aus dem Sternenpark Rhön
    http://www.sternenpark-Rhön.de

  134. #134 Alderamin
    31. Dezember 2014

    @Gegenschein

    Klar, das Licht verlässt die Lampe (nicht erst, wenn Du sie ausschaltest, sondern die ganze Zeit, während sie leuchtet) und wird dann entweder von einem Hindernis absorbiert (oder reflektiert) oder es bewegt sich ewig weiter im All, wobei es sich über eine immer größere Fläche verteilt, bis es nicht mehr nachweisbar ist. Die Photonen verteilen sich auf der Fläche und es wird immer unwahrscheinlicher, eines davon an einem bestimmten Ort innerhalb des Lichtkegels aufzufangen. Sie bewegen sich mit Lichtgeschwindigkeit von der Quelle fort und werden in ferner Zukunft durch die kosmologische Expansion zu immer längeren Wellenlängen auseinander gezogen, und sich vermischen sich mit der übrigen Strahlung, die das All ausfüllt.

  135. #135 Alderamin
    31. Dezember 2014

    @Bernd Heilmann

    Elektromagnetische Strahlung entsteht immer dann, wenn Ladung beschleunigt wird. In einer Radioantenne liegt ein wechselndes elektrisches Feld an, das die im Metall frei beweglichen Elektronen hin- und herschwingen lässt. Eine Schwingung bedeutet eine Beschleunigung der Elektronen, also entsteht eine elektromagnetische Welle. Nach den Gleichungen von James Clerk Maxwell erzeugt ein sich änderndes elektrisches Feld ein sich ebenfalls änderndes magnetisches Feld, das wieder ein elektrisches Feld hervorbringt, und so pflanzt sich die Welle elektromagnetisch fort.

    Nichts anderes tut Licht. Nur ist die Wellenlänge hier viel kürzer, die Schwingung viel schneller. Licht entsteht zum Beispiel, wenn ein Elektron in einem Atom von einer höheren Schale auf eine niedrigere springt, auch das ist eine Art von Beschleunigung. Diese Art von Licht entsteht zum Beispiel in Leuchtstoffröhren oder Laserpointern. Sie hat nur ganz bestimmte Frequenzen (bzw. Farben bzw. Wellenlängen).

    Das meiste Licht entsteht jedoch durch thermische Strahlung. Bekanntlich beginnt ein erhitztes Stück Eisen mit zunehmender Temperatur irgendwann rötlich (“rotglühend”), dann gelblich bis zum Weißen (“Weißglut”) hin zu leuchten. Wärme ist nichts anderes als das Zittern von Atomen oder Molekülen um ihren Ruheort. Wenn sie das heftig genug tun, dann wirken sie wie kleine Antennen und senden elektromagnetische Strahlung aus, denn auch hier sind in den Atomen geladene Teilchen, die durch die thermische Bewegung Wellen aussenden können. So leuchtet z.B. die Sonne, oder aber auch eine Glühlampe, in der ein Faden aus Wolfram vom ihn durchfließenden Strom bis zur Weißglut erhitzt wird. Schon bei Zimmertemperatur, ja sogar weit darunter strahlen Moleküle elektromagnetische Wellen aus, nur bei längeren Wellenlängen im Mikrowellenbereich (Millimeter und kleiner) und Infrarot (Mikrometer, das sind tausendstel Millimeter). Bei zunehmender Temperatur schwingen sie schneller, die Frequenz wird höher, die Wellenlänge entsprechend kürzer (Licht hat ein paar zehntel Mikrometer Wellenlänge).

    Noch kürzere Wellenlängen erreicht man, wenn man Ladung schnell genug im Kreis herum jagt (Synchrotronstrahlung; ein Synchrotron ist ein kreisförmiger Teilchenbeschleuniger, in dem zum Beispiel Elektronen durch ein Magnetfeld beschleunigt und auf eine Kreisbahn gezwungen werden), denn auch eine Kreisbewegung ist eine Beschleunigung; oder wenn man sie abrupt abbremst (etwa Elektronen beschleunigt und dann auf eine Metallplatte schießt: dabei ensteht sogenannte Bremsstrahlung, die liegt normalerweise im Röntgenbereich, zehntausendstel Mikrometer).

    Licht ist nur ein kleiner Bereich der elektromagnetischen Strahlung, die das Auge wahrnehmen kann. Langwellige Strahlung fühlen wir als Wärme auf der Haut, weil sie ihrerseits Moleküle in der Haut zum Schwingen bringt, und Wärme ist ja das Schwingen von Molekülen. Radiowellen und Röntgenstrahlung sind von exakt der gleichen Natur wie Licht, nur dass wir für diese Wellenlängen keine Sinne haben.

  136. #136 Gerhard
    Warum sind Exoplaneten so nah an Ihren Sternen?
    7. Januar 2015

    Wenn es um Exoplaneten geht habe ich das Gefühl, dass die Aussage: Der Planet ist seinem Stern sehr nah überdurchschnittlich oft fällt. Die Fragen die sich für mich daraus ergeben:
    – Finden wir tatsächlich vorwiegend sternennahe Planeten (oder sind die sternennahen die interessanteren und werden daher öfters erwähnt) ?
    – Ist die “Methode des Planeten findens” die wir z.Z, nutzen einfach erfolgreicher bei Planeten mit kurzen Umlaufzeiten?

  137. #137 Spritkopf
    7. Januar 2015

    – Finden wir tatsächlich vorwiegend sternennahe Planeten (oder sind die sternennahen die interessanteren und werden daher öfters erwähnt) ?

    Ersteres.

    – Ist die “Methode des Planeten findens” die wir z.Z, nutzen einfach erfolgreicher bei Planeten mit kurzen Umlaufzeiten?

    Ja, aus zwei Gründen. Erstens ist bei sternennahen Planeten der Einfluss der Gravitation auf den Stern höher, d. h., der Stern wackelt mehr, als wenn der Planet weit entfernt wäre. Zweitens muss man mehrere Umläufe des Planeten um den Stern beobachten, um aus der Periodizität des Ereignisses schließen zu können, dass das Wackeln von einem Planeten verursacht wurde. Was bei Umlaufzeiten von mehreren Jahren natürlich eine langwierige Angelegenheit wird.

  138. #138 Jens
    10. Januar 2015

    Warum ist im Zentrum fast aller Galaxien ein supermassives schwarzes Loch?

  139. #139 Florian Freistetter
    11. Januar 2015

    @Jens: Sehr gute Frage! Die ist als nächstes an der Reihe und ich werde sie in einem eigenen Artikel ausführlich beantworten.

  140. #140 Jens
    Langen
    11. Januar 2015

    Warum ist die Lichtgeschwindigkeit konstant und hat so einen großen Wert?

  141. #141 peer
    15. Januar 2015

    Ist vielleicht nicht ganz “Astronomisch”, aber: Angenommen die ISS wird durch ein Schrotteil (oder Sattelit oder Meteor) schwer beschädigt – wie holt man die Astronauten oben schnellstmöglich runter? Was für Sicherheitsmöglichkeiten haben die?

    (Full Disclosure: Ich frage auch, weil ich gerade überlege, ob das nicht auch ein spannendes, kooperatives Brettspiel mit dem Thema ergeben würde und ich nicht völlig falsche Dinge da einbauen wollen würde…)

  142. #142 Zoost
    köln
    16. Januar 2015

    Hallo,
    ich habe keinen Kommentar sonder eher eine Frage, die mich beschäftigt:

    Wie lange strahlt Licht oder welche Distanz kann es zurück legen. Ist Licht unendlich ? Oder wie kann/ soll man es deuten ?

    Danke

  143. #143 Alderamin
    16. Januar 2015

    @peer

    Angenommen die ISS wird durch ein Schrotteil (oder Sattelit oder Meteor) schwer beschädigt – wie holt man die Astronauten oben schnellstmöglich runter? Was für Sicherheitsmöglichkeiten haben die?

    Für solche Fälle sind immer hinreichend viele Sojus-Kapseln angedockt. Es sind nie mehr Leute gleichzeitig in der ISS, als in den angedockten Kapsel Platz finden (normalerweise 3 und eine Kapsel, beim Mannschaftswechsel 6 und zwei Kapseln).

    Da die ISS jeden Tag (nicht immer zur gleichen Uhrzeit) jeden Ort der Erde, der zwischen 51,6° nördlicher und südlicher Breite liegt, überfliegt, kann man auch täglich im üblichen Landegebiet in Kasachstan herunter kommen.

    Die einzelnen Sektionen der ISS können außerdem durch Luken voneinander getrennt werden, falls eine mal undicht werden sollte (oder wie neulich, wenn irgendwo Gas austreten sollte).

  144. #144 Cliff
    16. Januar 2015

    @Zoost: Bin zwar kein gelernter Physiker, aber ich versuche es mal.

    So lange Licht nicht auf Materie trifft, breitet es sich unendlich aus. Sobald es auf Materie trifft gibt es unterschiedliche Möglichkeiten (Absorption, Brechung, Streuung etc.)

  145. #145 Hicks
    München
    16. Januar 2015

    Spontaner Zerfall von Licht?

    Besteht nicht für jedes Elementarteilchen eine geringe Wahrscheinlichkeit, dass es spontan zerfällt?

    Aber wenn ein Photon zerfällt und Energie entsteht, entsteht dann wieder ein Photon ?

    Dann noch die Sache mit der geringen Wahrscheinlichkeit: Solange ein Photon sich mit Lichtgeschwindigkeit bewegt, steht die Zeit still, und dann ist die Zerfallswahrscheinlichkeit sowieso Null?

    Ist unser Universum überhaupt groß genug, so das sich der Zerfall von Licht überhaupt auswirken würde?

  146. #146 Alderamin
    16. Januar 2015

    @Jens

    Die Physik kann “warum”-Fragen eigentlich nicht beantworten, nur Fragen nach dem “wie”: wie verhält sich die Natur, wie bewegen sich Objekte oder Licht, wie kann man diese mathematisch beschreiben. Oft lassen sich Eigenschaften auf andere, grundlegendere Gesetzmäßigkeiten zurückführen, aber manchmal nicht.

    Man kann die Lichtgeschwindigkeit auf zwei andere Größen zurückführen, die elektrische Permittivität und die magnetische Permeabilität, die im wesentlichen beschreiben, wie sich elektrische und magnetische Felder in verschiedenen Stoffen ausbreiten (die elektrische Permittivität gibt beispielsweise an, welche elektrische Feldstärke herrscht, wenn eine bestimmte Ladung pro Flächeneinheit auf die Platten eines Kondensators aufgebracht wird; je nach dem verwendeten Material zwischen den Kondensatorplatten ergibt sich mehr oder weniger Feldstärke (die Feldstärke bestimmt, welche Kraft etwa ein geladenes Teilchen durch das Feld erfährt); die magnetische Permeabilität gibt an, wie ein Magnetfeld in einem Material verstärkt oder geschwächt wird). Auch das Vakuum hat eine Permittivität ε0 (sprich “Epsilon-Null”) und eine Permeabilität µ0 (sprich “Mü-Null”), die sich auf elektrische und magnetische Felder im Vakuum auswirken, und Licht ist ja nichts anderes als ein elektrisches Wechselfeld, das ein magnetisches Wechselfeld mit sich zieht (und umgekehrt). Es gilt das interessante Gesetz

    c= 1/√(ε0*µ0)

    für die Geschwindigkeit c des Lichts (und elektromagnetischer Wellen überhaupt, z.B. auch Radiowellen und Röntgenstrahlung). In Stoffen mit anderen Permeabilitäten ε und Permittivitäten µ (Glas, Wasser, bei Radiowellen auch Stein) gilt tatsächlich eine andere Lichtgeschwindigkeit

    c’=1/√(ε*µ)

    (die stets geringer als die im Vakuum ist) in Abhängigkeit von den jeweiligen Materialkonstanten ε und µ.

    Insofern hat die Lichtgeschwindigkeit im Vakuum ihren Wert, weil die Konstanten ε0 und µ0 ihre jeweiligen Werte haben. Warum dies der Fall ist, kann niemand beantworten. Niemand weiß, ob es einen tieferen Grund für die Größe dieser Konstanten ist, der ihren Wert unvermeidlich macht, oder ob sie Zufall sind.

    Aus den Beobachtungen der Lichtausbreitung ergab sich jedoch auch, dass die Lichtgeschwindigkeit eine fundamentale Eigenschaft der Raumzeit ist: Licht erscheint immer mit dieser Geschwindigkeit, egal wie schnell man sich relativ zu ihm bewegt. Das Licht einer Taschenlampe kommt genau so schnell auf mich zu, wenn ich ihr entgegenrenne, oder wenn ich stillstehe, stets misst man exakt den selben Wert. Das ist die (aus Beobachtungen folgende) Kernaussage der Relativitätstheorie von Albert Einstein, aus der alle weitergehenden Gesetze der Relativitätstheorie automatisch folgen (dass die Zeit eines bewegten Beobachters langsamer zu scheinen läuft, dass Strecken verkürzt erscheinen etc.). Die Werte der Vakuum-Permeabilität und -Permissivität stecken also irgendwie mit in den Grundeigenschaften von Raum und Zeit, denn Licht bewegt sich genau so schnell, wie es nach der Relativität maximal möglich ist (und wie sich kein Objekt mit einer Masse bewegen kann, das kann die Lichtgeschwindigkeit nie ganz erreichen).

    Man kann, glaube ich, zeigen, dass unsere Welt so nicht funktionieren würde, wenn die Lichtgeschwindigkeit unendlich wäre oder sehr gering, aber warum sie genau diesen bestimmten Wert hat, kann niemand beantworten. Nur, wie es sich mit dieser Geschwindigkeit ausbreitet und was daraus folgt.

  147. #147 krypto
    16. Januar 2015

    @Hicks: So einfach ist es auch nicht.
    Aber der Chef hat ja darum gebeten, hier nur Fragen zu stellen und nicht zu diskutieren 😉
    Ich meine, mich aber dunkel erinnern zu können, dass das schon mal thematisiert wurde:
    https://scienceblogs.de/astrodicticum-simplex/2013/07/29/instabiles-licht-konnen-photonen-zerfallen/

  148. #149 Florian Freistetter
    16. Januar 2015

    @krypto: Ach, wenn ihr wollt, könnt ihr diskutieren. Das lässt sich effektiv ja eh nicht verhindern. Mir ging es nur darum, dass ich für die Antworten keine Garantie übernehmen kann, solange sie nicht von mir selbst kommen.

  149. #150 Gerhard
    16. Januar 2015

    @krypto und @adlerarmin
    Danke der Artikel und die Kommentare sind genau das richtige zum Lichtzerfall.

    Die Frage die bleibt: Warum können masselose Teilchen nicht (spontan) zerfallen?

    Die populistische Antwort wäre wohl: “Weil für ein Photon die Zeit still steht, hat es keine Zeit zum zerfallen”

    Niels schieb aber:
    ” Tatsächlich kann man für masselose Teilchen nämlich gar keine Eigenzeit definieren. Das ist eine ganz andere Aussage als die “Eigenzeit ist Null”. Für ein (masseloses) Photon gibt es aber schlicht kein Inertialsystem.”

    Könnt ihr mir das erklären (oder erlinken), und dürfen masselose Teilchen jetzt doch zerfallen?

    P.S. Im verlinkten Artikel wird erwähnt, dass selbst wenn Licht zerfällt, es dies leider extrem unspekuakurär langsam tut.

  150. #151 Jens
    Langen
    16. Januar 2015

    @Alderamin
    Vielen Dank für die interessanten Anmerkungen zur Lichtgeschwindigkeit. Ich hatte vermutet dass man ihre Größe irgendwie aus der Geometrie des Raumes (drei Raumdimensionen, eine Zeitdimension) ableiten kann. Würde die Lichtgeschwindigkeit vielleicht in einer anders dimensionierten Raumzeit (z.B. vier Raumdimensionen oder zwei Zeitdimensionen) einen anderen Wert annehmen?

  151. #152 Jens
    Langen
    16. Januar 2015

    Wie groß war die Geschwindigkeit ( in km/s) mit der sich das Universum während der inflationären Phase ausgedehnt hat?

  152. #153 Alderamin
    17. Januar 2015

    @Jens

    Würde die Lichtgeschwindigkeit vielleicht in einer anders dimensionierten Raumzeit (z.B. vier Raumdimensionen oder zwei Zeitdimensionen) einen anderen Wert annehmen?

    Puh, keine Ahnung. Vielleicht wissen Martin Bäker oder Niels das (falls sie mitlesen; ansonsten bei “Hier wohnen Drachen” nachfragen).

    Wie groß war die Geschwindigkeit ( in km/s) mit der sich das Universum während der inflationären Phase ausgedehnt hat?

    Man kann der Expansion keine bestimmte Geschwindigkeit zuordnen, da diese vom Abstand der betrachteten Endpunkte einer Strecke abhängt. Das ist auch heute bei der kosmologischen Expansion so. Der Hubble-Parameter beträgt ca. 70 km/s / Mpc, also dehnt sich eine Strecke von 1 Mpc (ca. 32,6 Millionen Lichtjahre) pro Sekunde um 70 km aus. Eine Strecke von 2 Mpc entsprechend um 140 km, usw.

    Während der Inflation verdoppelte sich jedenfalls jede Strecke in 10^-35 s einmal. Da das Licht in 10^-35 s eine Strecke von 3*10^8 m/s * 10^-35 s = 3*10^-27 m zurücklegt, wuchs eine solche Strecke während der Inflation also mit Lichtgeschwindigkeit. Das ist eine Billion mal kleiner als ein Proton (10^-15 m).

  153. #154 Alderamin
    17. Januar 2015

    @Jens

    Würde die Lichtgeschwindigkeit vielleicht in einer anders dimensionierten Raumzeit (z.B. vier Raumdimensionen oder zwei Zeitdimensionen) einen anderen Wert annehmen?

    Puh, keine Ahnung. Vielleicht können Martin Bäker oder Niels dazu was sagen (falls sie mitlesen; ansonsten bei “Hier wohnen Drachen” nachfragen).

    Wie groß war die Geschwindigkeit ( in km/s) mit der sich das Universum während der inflationären Phase ausgedehnt hat?

    Man kann der Expansion keine bestimmte Geschwindigkeit zuordnen, da diese vom Abstand der betrachteten Endpunkte einer Strecke abhängt. Das ist auch heute bei der kosmologischen Expansion so. Der Hubble-Parameter beträgt ca. 70 km/s / Mpc, also dehnt sich eine Strecke von 1 Mpc (ca. 32,6 Millionen Lichtjahre) pro Sekunde um 70 km aus. Eine Strecke von 2 Mpc entsprechend um 140 km, usw. Für eine hinreichend große Strecke von 4300 Mpc = 13,9 Milliarden Lichtjahre wächst auch das heutige Weltall mit Lichtgeschwindigkeit (das ist auch ungefähr die Lichtlaufzeitentfernung vom Beginn des Universums bis zu uns).

    Während der Inflation verdoppelte sich jede Strecke in 10^-35 s einmal. Da das Licht in 10^-35 s eine Strecke von 3*10^8 m/s * 10^-35 s = 3*10^-27 m zurücklegt, wuchs eine solche Strecke während der Inflation also mit Lichtgeschwindigkeit. Das ist eine Billion mal kleiner als ein Proton (10^-15 m). Das war wirklich eine heftige Expansion.

  154. #155 Alderamin
    17. Januar 2015

    Ups, sorry für den Doppelpost, bin auf die falsche Taste geraten.

    @Gerhard

    Könnt ihr mir das erklären (oder erlinken), und dürfen masselose Teilchen jetzt doch zerfallen?

    Erklären kann ich’s nicht, aber Ich hab’ dazu das hier gefunden. Ansonsten: Niels oder Martin fragen.

  155. #156 Krypto
    17. Januar 2015

    @Gerhard: Ich denke, dass ein masseloses Teilchen eigentlich gar nicht mehr “Teilchen” genannt werden dürfte, sondern bestenfalls noch “Energieform” Ist ja eh äquivalent.
    Ich will nur verdeutlichen, dass “masselos” schon das geringste Energieniveau darstellt und dem armen Teilchen eigentlich nur noch ein Impuls bleibt, um es von nichts zu unterscheiden.

  156. #157 Alderamin
    19. Januar 2015

    @Krypto

    Ich will nur verdeutlichen, dass “masselos” schon das geringste Energieniveau darstellt und dem armen Teilchen eigentlich nur noch ein Impuls bleibt, um es von nichts zu unterscheiden.

    Das erzähl’ mal einem Gammaquant… in einem von 511 keV steckt z.B. die komplette Masseenergie eines Elektrons oder Positrons… das ist die Energie der Annihilationsstrahlung von Elektronen und Positronen.

  157. […] einer etwas längeren Weihnachtspause geht es mit der montäglichen “Fragen zur Astronomie”-Serie weiter. Und heute geht es wieder einmal um die Objekte, über die es die meisten Fragen zu […]

  158. #159 Jens
    20. Januar 2015

    @Florian
    Kannst du mal einen Beitrag bzw. Sternengeschichte zur Supernova SN1987a machen? Mich interessiert hier besonders was sie für neue Erkenntnisse sie für die Supernovaforschung gebracht hat. Danke schon mal im Voraus.

  159. #160 Jens
    25. Januar 2015

    Wie kann man sich automatisierte Überwachungsprogramme für Supernovaereignisse vorstellen und warum wurde die sehr helle SN2014J im letzten Jahr fast übersehen?

  160. #161 Hans-Walter Bronder
    Schiffweiler
    25. Januar 2015

    @Florian
    Könnte man durch die Zusammenschaltung von weit entfernten Radioteleskopen eine so hohe Auflösung erreichen, dass man extrasolare Planeten direkt erkennen kann, bzw. ist die Radiostrahlung von Planeten überhaupt stark und unterschiedlich genug um sich selbst bei hoher Teleskop-Auflösung von ihrem Mutterstern abzuheben ?

  161. #162 Alderamin
    25. Januar 2015

    @Hans-Walter Bronder

    Theoretisch ja, wenn die Planeten im Radiobereich hell sind (Jupiter ist das, d.h. eigentlich sein starkes Magnetfeld, das mit dem Sonnenwind wechselwirkt; das dürfte von seinem Kern aus metallischem Wasserstoff verursacht sein, der exzellent den Strom leitet). So ein schlappes Feld wie das der Erde wohl eher nicht. Das Feld des Sterns mag vorhanden und stark sein, aber die lange Basislinie sorgt ja für eine exzellente Auflösung, d.h. Trennung beider Quellen.

  162. #163 Alderamin
    25. Januar 2015

    @Jens

    Wie kann man sich automatisierte Überwachungsprogramme für Supernovaereignisse vorstellen

    Z.B. so oder so. Die Teleskope haben eine feste Zahl von Galaxien im Auge und nehmen diese immer wieder auf, wobei die Bilder dann mit Referenzaufnahmen verglichen werden (automatisch oder von menschlichen Auswertern). Wenn im Differenzbild ein heller Stern übrig bleibt, hat man eine potenzielle Supernova (vermutlich wird man von der gleichen Galaxie mehrere Bilder pro Nacht machen, um nicht auf Pixelfehler, Satelliten oder dergleichen herein zu fallen).

    und warum wurde die sehr helle SN2014J im letzten Jahr fast übersehen?

    Meinst Du das? https://astro.berkeley.edu/bait/public_html/2014/sn2014J.html

    Zu wenig Sterne zum automatischen Überlagern der Bilder und die studentischen Freiwilligen haben sie auch übersehen, weil die Galaxie so komplex geformt ist, steht da.

  163. […] viele populärwissenschaftliche Überlegungen. Und daher auch eine beliebte Kategorie in der Serie “Fragen zur Astronomie”. Dabei geht es aber nicht immer nur um die absichtliche Kontaktaufnahme mit eventuell vorhandenen […]

  164. #165 Hans-Walter Bronder
    Schiffweiler
    26. Januar 2015

    @Florian und @Alderamin
    Frage: Wenn sich das Universum für eine Strecke von 13,9 Mrd. Lichtjahren mittlerweile gemäß der Hubble-Konstanten mit Lichtgeschwindigkeit ausdehnt, müssten dann die vom kurz nach dem Urknall ausgesandten Lichtwellen, wenn ich sie als Vektor betrachte, jetzt nicht so stark in die entgegengesetzte Richtung “gedehnt” werden, dass die Photonen quasi “auf der Stelle stehen” und uns gar nicht mehr erreichen ?
    Wo liegt mein Denkfehler ?

  165. #166 Hans-Walter Bronder
    Schiffweiler
    26. Januar 2015

    Nachtrag: @Florian u. @Alderamin
    Zu meiner Frage von vorhin: Oder gilt hier das relativistische Gesetz, dass die Lichtgeschwindigkeit unabhängig vom Bezugssystems immer gleich bleibt (und demnach auch nicht durch Dehnung der Raumzeit “verlangsamt” wird) ?

  166. #167 Florian Freistetter
    26. Januar 2015
  167. #168 Alderamin
    26. Januar 2015

    @Hans-Walter Bronder

    Da der Urknall überall statt fand, gibt es für jede Zeit eine geeignete Entfernung, aus der uns die Photonen der Hintergrundstrahlung jeweils gerade erreichen. Für größere Entfernungen als die aktuellen 13,9 Milliarden Jahre Lichtlaufzeit gilt, dass die Photonen uns entweder erst zukünftig erreichen werden, oder (noch weiter entfernt) niemals, weil die Entfernung von dort zu uns schneller wächst, als sich die Photonen in der gleichen Zeit in unsere Richtung fortbewegen. Z.B. werden uns Photonen, die gerade jetzt in der Lichtlaufzeitentfernung von 13,9 Milliarden Jahren in unsere Richtung ausgesendet werden, uns niemals erreichen. Diejenigen, die uns in 1 Milliarde Jahren erreichen, sind schon lange unterwegs und beinahe schon hier, und diejenigen, die uns in 13,9 Milliarden Jahren erreichen werden, sind auch schon lange auf dem Flug und über die Hälfte der Strecke. Bis sie hier ankommen, wird sich die Strecke jedoch noch mehr als verdoppeln (beschleunigte Expansion!) deswegen brauchen sie viel länger, als die Photonen, die uns heute erreichen.

    Bis in alle Ewigkeit werden Photonen der Hintergrundstrahlung bei uns ankommen. In der Zukunft wird die Rotverschiebung (d.h. Temperatur) der Hintergrundstrahlung jedoch immer mehr zu- und ihre Intensität immer mehr abnehmen, bis sie in einer fernen Zukunft nicht mehr nachweisbar sein wird, weil sie im Quantenrauschen untergeht.

  168. #169 Hans-Walter Bronder
    26. Januar 2015

    @Florian u. @Alderamin:
    Vielen Dank, hatte dann doch keinen Denkfehler in meiner Überlegung 🙂

  169. #170 Alderamin
    26. Januar 2015

    @myself

    n der Zukunft wird die Rotverschiebung (d.h. Temperatur) der Hintergrundstrahlung jedoch immer mehr zu-

    Ist so nicht ganz korrekt formuliert, die Rotverschiebung wird zunehmen und die Temperatur wird damit abnehmen, denn die Temperatur ist umgekehrt proportional zur Wellenlänge (Wiensches Verschiebungsgesetz). Die Mikrowellen-Hintergrundstrahlung ist nämlich Schwarzkörperstrahlung. Sie hat eine Rotverschiebung von ca. 1100 und war somit bei ihrer Aussendung 1100-mal heißer als die heutigen 2,8 K, macht gute 3000 K Temperatur des Feuerballs, als dieser erstmals durchsichtig wurde und die Photonen freie Bahn bekamen.

  170. #171 Stefan
    26. Januar 2015

    Folgende Frage treibt mich schon länger um:

    In Diagrammen, die die Entwicklung des Universums nach dem Urknall visualisieren, sind i.d.R. auf der x-Achse die Zeit und auf der y-Achse die Größe dargestellt.
    Auch liest man oft Erklärungen dazu wie z.B. 10 hoch -xy Sekunden nach dem Urknall war das Universum so und so groß.

    Welche Basis wird denn eigentlich dieser Zeit- und Größenmessung zugrunde gelegt?

    Denn nach der ART kommt es ja in der Nähe von großen Massen (ist ja beim Urknall der Fall) bei Zeit- und Distanzmessungen auf den Beobachter an.
    Wo befindet sich beim Urknall dieser (fiktive) Beobachter mit seiner Uhr und seinem Zollstock?
    Beim Rückwärtsgehen in der Zeit in Richtung Urknall sollte doch die Zeit irgendwann nicht mehr linear verlaufen, oder doch?
    Oder habe ich hier einen Denkfehler?

  171. #172 Hans-Walter Bronder
    26. Januar 2015

    @Florian
    Noch eine Frage zur Interferometrie bei Radioteleskopen: Mir ist bekannt, dass durch die Zusammenschaltung von voneinander weit entfernten Radioteleskopen sehr große Auflösung erreicht werden kann, die einem imaginären Radioteleskop von der Größe des Abstandes der einzelnen in die Interferometrie eingebundenen Teleskope entspricht. Leider ist es mir trotz intensiver Suche im Internet nicht gelungen, die für diesen Sachverhalt physikalisch/mathematische Grundlage bzw. Erklärung zu finden. Wo muss ich suchen ?

  172. #173 PDP10
    26. Januar 2015

    @Hans-Walter Bronder:

    Ich bin zwar nicht Florian, fand die Erklärung hier:

    https://de.wikipedia.org/wiki/Interferometer_%28Radioastronomie%29

    aber recht verständlich.

    Wenn du auch noch dem einen oder anderen Link folgst (dem zu Auflösungsvermögen zB oder unter “Siehe auch”) solltest du eigentlich alles dazu finden, was du wissen willst …

  173. #174 Hans-Walter Bronder
    26. Januar 2015

    @PDP10
    Danke für den Tipp: Ist wirklich echt gut erklärt !

  174. #175 AmbiValent
    26. Januar 2015

    Zum Thema Gasriesen: Ich meine gelesen zu haben, dass bei den kleineren Riesenplaneten Uranus und Neptun auch im Kern noch im Wesentlichen “normaler” (thermischer) Druck herrscht, bei Jupiter und Saturn dagegen die Verhältnisse so extrem wären, dass der Entartungsdruck (Pauli-Prinzip) in den Vordergrund rückt. Habe ich das richtig verstanden? Und wenn, weiß man schon, bei welcher Masse die Grenze zwischen den beiden Formen liegt?

  175. #177 krypto
    27. Januar 2015

    Oha…das ist auch nicht Curiosity, sondern Nr. 5:
    https://www.kino.de/kinofilm/nummer-5-lebt/fotoshow/9361/651631
    😉

  176. #178 Hans-Walter Bronder
    Schiffweiler
    28. Januar 2015

    @Florian:
    Danke für den Hinweis auf deinen Artikel
    “Die Rotverschiebung …”
    Toller Artikel, sehr gut verständlich !

  177. #179 Jens
    28. Januar 2015

    Kann man den Schwarzschildradius des sichtbaren Universums berechnen? Wenn ja war das Universum in der Vergangenheit mal kleiner als sein Schwarzschildradius?

  178. #180 krypto
    29. Januar 2015

    @Jens: Meiner Meinung nach sind Deine sehr interessanten Fragen eigentlich nur so zu beantworten:
    Ein Radius ist eine Strecke, die sich in einer der 3 Raumdimensionen erstreckt. Da mit dem Universum auch die Raumzeit entstanden ist, beißt sich die Katze da ein wenig in den Schwanz 😉

  179. #181 Tina
    3. Februar 2015

    Ich hätte da mal eine Frage: Ich habe gehört, dass Wissenschaftler kürzlich festgestellt haben, dass die Gravitationswellen, die sie angeblich beobachten konnten (veröffentlicht im März 2014), sich nun doch als Verwirbelungen im Kosmischen Staub rausgestellt haben.
    Man möge mir meine laienhafte Ausdrucksweise verzeihen, aber genau das bringt mich zu der Frage:
    Was genau ist da jetzt rausgefunden worden? Und was bedeutet das? Florian, kannst du das in einfach verständlichen Worten erklären?
    (Hier ein Link zur Quelle: https://www.ardmediathek.de/radio/Forschung-Aktuell-Deutschlandfunk/Doch-keine-Gravitationswellen-Urknall-F/Deutschlandfunk/Audio-Podcast?documentId=26240668&bcastId=21676038)

  180. #182 Florian Freistetter
    3. Februar 2015

    @Tina: Hier hab ich früher schon mal was geschrieben: https://scienceblogs.de/astrodicticum-simplex/2014/07/11/anhoeren-bicep2-die-entdeckung-der-kosmologischen-inflation-und-staubige-zweifel-an-den-daten/ Irgendwann kommt dazu aber auch sicher noch mal mehr.

  181. #183 Alderamin
    3. Februar 2015

    @Tina

    Hier gibt’s was auf Englisch dazu. Das BICEP2-Team hatte gemeint, eine Stelle des Himmels angepeilt zu haben, die frei von Staub sei (aufgrund von Karten des PLANCK-Weltraumteleskops), was jedoch nicht der Fall war. Staubteilchen, die teilweise länglich sind und auch Eisen enthalten, können sich entlang von Magnetfeldern in der Milchstraße ausrichten (man denke an Eisenfeilspäne) und diese wirken dann als schwache Polarisationsfilter, für normales Licht und erst recht für Mikrowellen. Nach Polarisationsmustern hatte BICEP2 gesucht und war dann auf das falsche Signal hereingefallen. Das gefundene Signal war auch viel stärker, als man es eigentlich erwartet hätte. Zukünftige Experimente werden empfindlicher sein und vielleicht die echten B-Moden aus der Hintergrundstrahlung herauskitzeln.

    Einerseits sehr schade (und einige “Wissenschaftskritiker” werden sicher darüber herziehen), aber es zeigt mal wieder, dass die Wissenschaft sich selbst korrigiert. Wer Unsinn behauptet, wird irgendwann von einem anderen Team widerlegt. Und gibt dann (anders als die unzähligen Cranks) auch seinen Irrtum zu.

  182. #184 Tina
    3. Februar 2015

    Ah ja, danke!!

  183. #185 Lutz Komischke
    3. Februar 2015

    Hallo Herr Freistetter,
    ich habe eine Frage zur beschleunigten Ausdehnung des
    Raumes bzw. der grösseren Fluchtgeschwindigkeit der
    Galaxien je weiter sie von uns entfernt sind.

    Wenn jetzt anerkannt ist, das der Raum nicht mit “Nichts”
    sondern mit “Dunkler Energie” erfüllt ist, würde das für mich
    bedeuten, das auch die Lichtgeschwindigkeit relativ ist und
    zwar zu dieser Energie.
    Deshalb stellt sich die Frage warum man davon ausgeht
    das sich das Licht ohne Energieverlust Milliarden Jahre
    durch den Raum bewegt.
    Ich würde meinen, das ein Grossteil der Rotverschiebung
    des Lichtes anderer Galaxien durch Energieverlust zustande kommt und deshalb die Entfernungs- und
    Altersbestimmungen des Universums korrigiert werden
    müssten.

    m.f.G. Lutz Komischke

  184. #186 Alderamin
    3. Februar 2015

    @Lutz Komischke

    Wenn jetzt anerkannt ist, das der Raum nicht mit “Nichts”
    sondern mit “Dunkler Energie” erfüllt ist

    Es war auch schon vor der Entdeckung der Dunklen Energie klar, dass das Vakuum nicht einfach “Nichts” ist, sondern dass darinständig virtuelle Teilchen entstehen und wieder verschwinden, dass es eine Vakuumenergie gibt. Nur nicht, dass diese den Raum zusätzlich aufbläht.

    würde das für mich
    bedeuten, das auch die Lichtgeschwindigkeit relativ ist

    Das war sie schon seit Einstein, daher “Relativitätstheorie”.

    und
    zwar zu dieser Energie.

    Die Lichtgeschwindigkeit ist gleich groß für jeden unbeschleunigten Beobachter, egal wie dieser sich bewegt (abgesehen davon, dass man der Dunklen Energie keine Geschwindigkeit zuschreiben kann, sie ist ja an kein konkretes Objekt gebunden). Vielleicht meinen Sie die kosmische Hintergrundstrahlung, relativ zu der wir einigermaßen in Ruhe sind (bis auf die Eigenbewegung der Erde, Sonne und Milchstraße von insgesamt ca. 400 km/s, was klein gegen die Lichtgeschwindigkeit ist). Im Prinzip ist alles in relativer Ruhe zu dieser Strahlung, überall im Weltall. Jeder sieht sie mit der gleichen Rotverschiebung in jeder Richtung, bis auf eine kleine Eigenbewegung, die aus dem gravitativen Kollaps der nach dem Urknall entstandenen, ursprünglich gleichmäßig verteilten Materie zu Galaxien und Sternen resultiert.

    Deshalb stellt sich die Frage warum man davon ausgeht
    das sich das Licht ohne Energieverlust Milliarden Jahre
    durch den Raum bewegt.
    Ich würde meinen, das ein Grossteil der Rotverschiebung
    des Lichtes anderer Galaxien durch Energieverlust zustande kommt

    Das ist im Prinzip völlig korrekt. Durch die Expansion des Raums werden die Wellenlängen der Photonen größer und damit nimmt ihre Energie ab, denn die ist h*f mit der Frequenz f des Lichts (und Wellenlänge λ = c/f). Das Licht wird aber nicht “von selbst müde”, ein solcher Effekt wurde nie beobachtet und könnte mit unserer Physik auch nicht erklärt werden; er stünde aber vor allem im Widerspruch zu den Beobachtungen (dass nämlich nicht nur Lichtwellen gestreckt werden, sondern überhaupt jeder zeitliche Prozess, z.B. eine Supernova-Lichtkurve oder ein Gamma-Burst; durch die Raumexpansion werden zwei beliebige Pulse, die mit einem Zeitversatz losgesendet werden, immer weiter zeitlich auseinandergezogen, egal, ob es sich um zwei Wellenberge des Lichts oder Punkte auf der Lichtkurve einer Supernova handelt).

    und deshalb die Entfernungs- und
    Altersbestimmungen des Universums korrigiert werden
    müssten.

    Der Effekt der Dunklen Energie wird in der Altersbestimmung des Universums berücksichtigt. Ohne diese wäre das Weltall nur ca. 10 Milliarden Jahre alt und damit jünger als die ältesten bekannten Sterne. Auch das müsste mit “müdem Licht” zuerst einmal erklärt werden.

  185. #187 Nemesis
    3. Februar 2015

    Kleine Frage eines unbedarften Laien:

    Ich sehe desöfteren (und gerade auch wieder) am nichtmal völlig dunklen Abend ein sehr helles, grosses Objekt am Himmel, ziemlich genau im Süden, ist das die Venus?

  186. #188 luca2004
    3. Februar 2015

    ich habe eine frage zu schwarzen löchern : wo ist das ende von schwarzen löchern

  187. #189 luca2004
    3. Februar 2015

    naja eigentlich habe ich sehr viele fragen aber dass ist eine davon

  188. #190 Krypto
    3. Februar 2015

    @Nemesis:
    Soweit ich weiß, können Venus (und erst recht Merkur) gar nicht genau im Süden(und erst recht nicht im Norden) am Nachthimmel stehen, weil sie zu nahe an der Sonne sind.
    Daher kann man sie wenn überhaupt entweder morgens im Osten oder abends im Westen sehen.
    Die Planeten, welche weiter von der Sonne entfernt sind als die Erde, lassen sich auch im Süden beobachten. Und da ist Jupiter mit Abstand am hellsten.

  189. #191 Krypto
    3. Februar 2015

    @myself: Bei extrem klarer Sicht und wenn man weiß, wo man an der Sonne vorbeischauen muss, kann man Venus auch tagsüber sehen.

  190. #192 Florian Freistetter
    3. Februar 2015

    @Luca: Schwarze Löcher saugen nicht: https://scienceblogs.de/astrodicticum-simplex/2010/01/12/schwarze-locher-sind-keine-staubsauger/

    @Nemesis: Besorg dir Stellarium. Da kannst du sowas nachschlagen.

  191. #193 Alderamin
    3. Februar 2015

    @Nemesis

    Die Venus steht abends tief im Südosten und ist der hellste “Stern” am Himmel, der schon in der Dämmerung zu sehen ist, geht aber bald nach Sonnenuntergang ebenfalls unter. Genau im Osten steht abends halb hoch Jupiter (im Augeblick rechts schön flankiert vom Vollmond), der ist nicht ganz so hell und hat einen gelblichen Farbton. Im Süden findet man (je nach Uhrzeit, eher gegen 22:00 Uhr) das Sternbild Orion und links unterhalb davon den hellen Stern Sirius, den hellsten Fixstern am Himmel. Sirius leuchtet bläulich-weiß und flackert meistens deutlich, was die Planeten nicht tun.

  192. #194 Alderamin
    3. Februar 2015

    @luca2004

    Schwarze Löcher haben kein “Ende”. Das sind einfach Sterne, die so weit in sich zusammenstürzen, dass nichts mehr ihrer Schwerkraft (die an der Oberfläche des Sterns immer größer wird, je kleiner er schrumpft) entkommen kann, nicht einmal Licht. Wie es genau hinter jenem “Horizont” aussieht, den das Licht nicht mehr verlassen kann, weiß niemand genau. Vielleicht ist da drinnen nur ein unendlich dichter Punkt als Sternenrest übrig.

  193. #195 Nemesis
    4. Februar 2015

    @Alderamin

    Dankeschön für Deine fachliche Auskunft. Dann muss es Sirius sein, denn das Objekt befindet sich im Süden und flackert eindeutig, die Uhrzeit kommt auch etwa hin. Cool, wieder was dazugelernt.

    @Krypto

    Auch Dir besten Dank!

  194. #196 luca 2004
    5. Februar 2015

    Vielen Dank an Alderamin und F.F

  195. #197 Nemesis
    5. Februar 2015

    @FF

    Ooops, ich hatte Deinen Hinweis auf Stellarium gestern übersehen! Hey, das ist richtig cool, 1000 Dank für den Hinweis!

  196. #198 Lutz Komischke
    6. Februar 2015

    Ich habe eine Frage zur Expansion des Univerums.

    Wenn sich das Universum seit 13,7 Milliarden Jahren
    ausdehnt und sich alle Galaxien voneinader entfernen,
    müsste es nicht einen riesigen Raumbereich geben der
    völlig leer ist ?
    Und wiso ist die Andromeda Galaxie dann auf
    Kollisionskurs mit unserer Milchstrasse ?

  197. #199 Hicks
    6. Februar 2015

    Die Frage geht in die selbe Richtung, wie die von Lutz Komischke.
    Wissen wir wo das Zenturm des Universums ist?
    Oder gibt es überhaupt eine sinnvolle Definition für das Zentrum des Universums?

    P.S. Zur Zeit der Galaxienenstehung, hätten sich Galaxien “entschieden” könne Richtung Zentrum zu fliegen?

    P.S.P.S Haben Galaxien gemessen am absolutem Raum – ups – überhaupt signifikant unterschiedliche Geschwindigkeiten?

  198. #200 Florian Freistetter
    6. Februar 2015

    @Lutz: Schau dir auch mal die schon beantworteten Fragen an. In der Liste oben sind besonders Frage 1 und 2 interessant für dich. (und Frage 8)

    @Hicks: Zentrum gibt es keines… https://scienceblogs.de/astrodicticum-simplex/2013/03/03/video-wie-gros-ist-das-universum-und-wo-ist-der-mittelpunkt/
    Zur Frage der Galaxiengeschwindigkeit kann ich dir meinen Podcast empfehlen: https://scienceblogs.de/astrodicticum-simplex/2015/01/23/sternengeschichten-folge-113-das-kopernikanische-prinzip-und-der-grosse-attraktor/

  199. #201 Alderamin
    6. Februar 2015

    @Lutz Komischke, Hicks

    Ihr scheint davon auszugehen, dass sich eine Explosion innerhalb eines vorhandenen Raums ereignet hätte. Die Allgemeine Relativitätstheorie besagt aber, dass der Raum nichts festes, unveränderliches ist, sondern dass er sich krümmen und wachsen kann, und das nimmt die Urknalltheorie an.

    Man kann sich das vorstellen wie die Oberfläche eines Ballons (die aber nur zweidimensional ist; sie soll trotzdem für den dreidimensionalen Raum stehen, nur kann man sich keinen dreidimensionalen “Oberraum” eines vierdimensionalen Ballons vorstellen). Wenn der Ballon aufgeblasen wird, wird die Oberfläche gleichmäßig überall vergrößert und der Abstand von auf den Ballon gemalten Punkten vergrößert sich. Jeder Punkt würde alle anderen Punkte von sich fortstreben sehen und sich im Mittelpunkt der Ausdehnung wähnen. So geschieht das auch bei der Expansion des Raums, nur in drei Dimensionen (der Hohlraum des Ballons dient nur der Vorstellung, in Wahrheit gibt’s den wahrscheinlich nicht, am besten gleich wieder vergessen).

    Insofern gibt es keinen Mittelpunkt des Universums. Jeder Punkt ist gleichberechtigt. Deswegen sieht es so aus, als ob wir zufällig genau der Mittelpunkt der Ausdehnung seien. Das denken die anderen aber von sich mit der gleichen Berechtigung.

    Wenn sich das Universum seit 13,7 Milliarden Jahren
    ausdehnt und sich alle Galaxien voneinader entfernen,
    müsste es nicht einen riesigen Raumbereich geben der
    völlig leer ist ?

    Wie gesagt, außen herum ist da keine Leere. Ob das All geschlossen ist wie die Ballonoberfläche oder doch eher unendlich wie ein unendlich großes Gummituch, an dem von allen Seiten gezogen wird, ist dabei nicht klar. Innerhalb des Universums verteilt sich die Materie nur immer weiter, die Dichte nimmt ab. Jedenfalls im Großen.

    Und wiso ist die Andromeda Galaxie dann auf
    Kollisionskurs mit unserer Milchstrasse ?

    Weil sie mit anderen Galaxien zur Lokalen Gruppe gehört, die aneinander gebunden sind. Nach dem Urknall war das Weltall ziemlich gleichmäßig mit Gas ausgefüllt, das lediglich winzige Dichteunterschiede aufwies. Aufgrund der etwas höheren Schwerkraft der dichteren Stellen fing das Gas an, zu diesen hin zu fallen und verdichtete sich dort zu Galaxien aus Sternen und Galaxienhaufen aus vielen Galaxien. Diese Objekte bewegten sich dann auch weiterhin aufeinander zu und umeinander herum und verschmolzen zu immer größeren Objekten. Es wirkt zwar zusätzlich die Raumexpansion aber erst jenseits ca. 60 Millionen Lichtjahren, wo sie eine ähnliche Größenordnung wie die Galaxien-Eigenbewegung erreicht (sie nimmt ja mit der Entfernung immer mehr zu: wenn ein Meter um 1% pro Sekunde wächst, dann sind das 1 cm/s, wenn ein Kilometer um 1% pro Sekunde wächst, sind es 10 m/s), kann sie die Galaxien allmählich voneinander entfernen. Die Andromeda-Galaxie ist nur 2,5 Millionen Lichtjahre von der Milchstraße entfernt und wird in 5 Milliarden Jahren mit ihr verschmelzen. Vielleicht kollidieren noch weitere Galaxien mit der neuen Riesengalaxie. So passiert das überall im All.

    P.S. Zur Zeit der Galaxienenstehung, hätten sich Galaxien “entschieden” könne Richtung Zentrum zu fliegen?

    Nein, das Gas war nach dem Urknall gewissermaßen in Ruhe und fiel dann durch die Eigenschwerkraft auf zufällig dichtere Stellen zu. Die Schwerpunkte dieser Dichtezentren ruhen immer noch gegenüber der kosmischen Hintergrundstrahlung, wenn man diese als Bezugspunkt für die Bewegung nimmt. Die Galaxien bewegen sich um diese Zentren herum.

    P.S.P.S Haben Galaxien gemessen am absolutem Raum – ups – überhaupt signifikant unterschiedliche Geschwindigkeiten?

    Einen absoluten Raum gibt’s nicht, aber mit der Hintergrundstrahlung als Referenz (man ruht genau dann in Bezug zur Hintergrundstrahlung, wenn sie in jeder Richtung die gleiche Rotverschiebung hat) bewegen sich die Galaxien mit den Geschwindigkeiten, die aus dem freien Fall aufeinander zu resultieren und die somit knapp unter der Fluchtgeschwindigkeit der Galaxienhaufen liegen (ein Fall aus dem Unendlichen aus dem Ruhezustand heraus hin zu einem Massezentrum ergibt genau Fluchtgeschwindigkeit dieser Masse). Das sind dann maximal ein paar hundert km/s. Die “Fluchtgeschwindigkeit” (wie das früher mal hieß) der Galaxien aufgrund der Expansion des Raums geht hingegen bis fast Lichtgeschwindigkeit hinauf. Aber, wie gesagt, in Wahrheit flieht da nichts, da bewegt sich nichts, dazwischen kommt nur ständig mehr Raum hinzu, wie beim aufgeblasenen Ballon oder einem aufgehenden Rosinenkuchen im Backofen (wo sich die Rosinen relativ zum Teig auch nicht bewegen).

  200. #202 Jens
    6. Februar 2015

    In wievielten Jahren sehen wir auf Grund der Expansion des Universums nur noch die Galaxien aus der lokalen Gruppe am Nachthimmel?

  201. #203 Alderamin
    7. Februar 2015

    @Jens

    Gerade habe ich nebenan erklärt: eigentlich nie. Eigentlich wird immer noch Licht von weiter her zu uns kommen, es wird nur zunehmend älter und rotverschobener sein, so dass man es nicht mehr mit optischen Teleskopen sehen kann, anfangs noch als Mikrowellen oder Radiowellen, bis diese im Rauschen untergehen.

    Im Buch “Ein Universum aus dem Nichts” erklärt Lawrence Krauss jedoch (gerade nachgeschlagen), dass die Objekte spätestens dann aus dem Bereich von möglichen Messungen verschwinden, wenn die Wellenlänge ihrer Strahlung länger als der überblickbare Bereich des Universums wird. Für die lokale Gruppe gibt Lawrence M. Krauss eine Zeit von 2 Billionen Jahren an, bis dies der Fall ist. Das ist lange, aber zu dieser Zeit wird es noch Sterne geben, die es heute schon gibt (z.B Proxima Centauri, der sonnenächste Stern, wäre ein Kandidat, unsere Sonne wird dann aber längst ein kalter “Schwarzer Zwerg” sein).

    Die lokale Gruppe werden wir im Sichtfeld behalten, sie expandiert nicht mit dem Universum. Wenn es keinen “Big Rip” gibt.

  202. #204 ron
    7. Februar 2015

    Hallo Florian,

    was passiert eigentlich mit dem Licht welches wir täglich mit unseren elektrischen Geräten emittieren? Also wohin geht das Licht aus der Mikrowelle oder dem Kühlschrank oder der Wohnzimmerlampe. Wird es irgendwann von weltlicher Materie geschluckt oder besteht die Wahrscheinlichkeit, dass dieses Licht als Teilchen weit ins Unsiversum “fliegt”?

    Danke für deine Antwort!

    r

  203. […] der Serie “Fragen zur Astronomie” geht es heute mal wieder um das Licht. Eine Frage bekomme ich dazu sehr oft zu hören: Was passiert […]

  204. #206 luca2004
    9. Februar 2015

    warum werden objekte immer schneller je weiter sie sich von uns entfernen?

    ich hab mal gehört das es was mit dunkle marterie

  205. #207 luca2004
    9. Februar 2015

    in meinem letzten kommentar hab ich mein satz nicht zu ende geschrieben ich wollte schreiben

    ich mal gehört das es was mit dunkle marterie zu tun hat oder dunkle energie

  206. #209 Hicks
    10. Februar 2015

    @luca (@alle fals ich Unrecht habe)

    Das Universum ist an jeder Stelle mehr oder weniger gleich – folglich:
    Galaxien die sehr weit weg sind, “bewegen” sich nicht wirklich schneller als Galaxien, die nicht ganz so weit weg sind – es gibt schließlich keinen Anhaltspunkt das entfernte Galaxien irgendwie anders sein könnten als nahe.

    Die flapsige Verwendung von “bewegen” wirkt widersprüchlich, da ich keine Inertialsystem nennen kann in dem man diese gleiche Geschwindigkeit messen kann. Aber ich finde die Vorstellung sehr anschaulich.

    Zurück zur Messung der Geschwindigkeit: Der Raum selbst dehnt sich aus, je weiter eine Galaxie entfernt, desto mehr Raum ist zwischen ihr und uns und umso mehr kan sich dieser Raum während der Messung ausdehen (die Dauer der Messung ist nicht die Zeit die der Astronom “vorm Fernrohr sitzt” sonder die Zeit die das Licht von der fernen Galaxie zu uns braucht). Summa sumarum wir messen eine höhere Geschwindigkeit entferntern Galaxie, die Galaxie entfernt sich auch wirklich von uns, aber sie wird dabei nie schneller (nie beschleunigt).

    Ich hoffe das ist verständlich.

  207. #210 Alderamin
    10. Februar 2015

    @Hicks

    Ist so richtig, wie Du das erklärst. Es gibt einen Wert, um den eine bestimmte Strecke pro Sekunde wächst, der nennt sich “Hubble Konstante” (oder neuerdings “Hubble Parameter”, weil der Wert über Jahrmilliarden gar nicht konstant ist). Und zwar wächst eine Entfernung von einem Megaparsec (1 parsec sind 3,26 Lichtjahre, also 1 Megaparsec = 3,26 Millionen Lichtjahre) jede Sekunde um rund 72 km (die Gelehrten streiten noch, ob es eher 70 oder eher 74 sind, wie verschiedene Messungen ergeben). 10 Millionen Lichtjahre wachsen also um ungefähr das dreifache, 220 km/s.

    So schnell scheinen sich Galaxien in dieser Entfernung von uns weg zu bewegen (obwohl sie sich eigentlich gar nicht bewegen, der Raum zwischen ihnen und uns wird einfach größer), 100 Millionen Lichtjahre wachsen um 2200 km pro Sekunde. 1 Milliarde Lichtjahre um 22000 km/s usw. Je weiter, desto schneller.

    Der Raum wächst eigentlich nur um einen winzigen Prozentanteil, etwa 1% Längenzunahme in 135 Millionen Jahren, wenn ich richtig gerechnet habe, ein Meter wächst in 13,5 Millionen Jahren gerade mal um einen Millimeter. Aber 1% ist auf eine große Strecke bezogen halt ein entsprechend großes Stück. Und 3,26 Millionen Lichtjahre sind eine sehr große Strecke, etwa 30.839.600.000.000.000.000 Kilometer (das sind 19 Dezimalstellen hinter der ersten 3). Da macht halt auch das Fünfbillionstel Wachstum pro Tag ein ziemliche Strecke aus.

    Wir alle, die Erde und das Sonnenssystem, die Milchstraße und ihre Nachbargalaxien wachsen jedoch nicht mit, wir rutschen sozusagen über das Wachstum des Raums hinweg, weil die Kräfte, die uns zusammen halten, viel größer sind, als diese winzige Kraft des Vakuums. Erst auf ca. 20 Millionen Lichtjahre wird ihr Einfluss so groß, dass er die Kräfte zwischen Galaxien in dieser Entfernung übertrifft. Das ist etwa die 8-fache Entfernung zu unserer Nachbar-Spiralgalaxie, dem Andromedanebel. Der entfernt sich deswegen auch nicht von uns, sondern kommt uns im Gegenteil immer näher.

  208. #211 Spritkopf
    10. Februar 2015

    @Alderamin

    Wir alle, die Erde und das Sonnenssystem, die Milchstraße und ihre Nachbargalaxien wachsen jedoch nicht mit, wir rutschen sozusagen über das Wachstum des Raums hinweg,

    Ist das so? Dann hatte ich das bisher falsch verstanden. Ich dachte immer, die Expansion des Raumes würde nur dort stattfinden, wo keine oder nur geringe Gravitationskräfte dem Druck der dunklen Energie entgegenwirken, also vor allem in den Voids.

  209. #212 Alderamin
    10. Februar 2015

    @Spritkopf

    Nein, der Raum wächst überall. Aus den neuesten Messungen von Planck wurde erst letzte Woche auf einer Konferenz gefolgert, die Dunkle Energie sei ununterscheidbar von einer kosmologischen Konstanten, d.h. sie ist eine Eigenschaft des Vakuums schlechthin, sie ist überall und auch überall gleich groß.

    Es ist aber so, dass Voids schneller wachsen, weil da ja kaum Materie ist, die mit ihrer Gravitation das Wachstum bremsen kann (das All würde auch ohne Dunkle Energie wachsen, aber das Wachstum würde durch die Gravitation gebremst). Im Gegenteil fällt die Materie in den Voids (ganz leer sind die nämlich nicht, dazu gab’s in der Februar-Sky&Telescope einen Artikel, ihre Dichte ist lediglich rund 1/10 der Dichte außerhalb, da gibt’s auch Galaxien drin) nach außen zu den Rändern hin, deswegen vergrößern sich die Voids noch. Da gibt’s im Moment Untersuchungen, die durch das Ausmessen sehr vieler Voids noch genauere Parameterwerte für das kosmologische Modell ermitteln wollen, als PLANCK und WMAP das aus der Hintergrundstrahlung bestimmen konnten.

  210. #213 Spritkopf
    11. Februar 2015

    @Alderamin: Merci bjeng.

  211. #214 krypto
    11. Februar 2015

    @Hicks:
    Ich verstehe das ein klein wenig anders, aber vielleicht meinst Du das genauso:
    Galaxien, die weit weg sind, sehen schon anders aus und verhalten sich anders, weil wir sie in einer früheren Evolutionsstufe sehen und sich die Expansion des Raums beschleunigt.

  212. #215 Hicks
    11. Februar 2015

    @krypto
    Ich nehme mal das Risiko der unützen Haarspalterei in Kauf …

    Wenn wir in die ferne sehen, sehen wir in die Vergangenheit, und wegen der (logischen) Uniformität des Universums
    a) ist die entfernte Vergangenheit gleich(uniform) der unseren.
    b) die ferne Gegenwart ist auch gleich der unseren, nur das wir Sie nicht beobachten können.

    (bis hierhin macht das auch unabhängig von jeglicher Raumausdehnung perfekt Sinn)

    Wenn wir nun sagen, sie sehen aber doch anders aus, dann ist das eine Aussage darüber wie wir Sehen, bzw das gesehene interpretieren.

  213. #216 krypto
    11. Februar 2015

    @Hicks: Das Haar ist nun so klein, dass ich es nicht weiter spalten mag 😉
    Es ging mir einfach nur darum, dass Luca das nicht falsch versteht. Da kommt nämlich schnell Relativität in´s Spiel.

  214. #217 bikerdet
    14. Februar 2015

    Hallo

    Ich hätte da mal eine Frage zum mittleren Abstand der Asteroiden. Ich erinnere mich an eine Aussage zur Oortschen Wolke, das die Objekte dort im mittel 10 AU von einander entfernt seien. Ist das korrekt ? Und wie sieht das beim Asteroidengürtel zwischen Mars und Jupiter aus oder dem Kuipergürtel ?
    Leider ist es kaum möglich im Blog nach solchen Allerweltsbegriffen wie ‘Abstand´’ oder ‘Asteroiden’ zu suchen. Die kommen praktisch in jedem Beitrag vor …

  215. #218 Alderamin
    15. Februar 2015

    @bikerdet

    Vielleicht hast Du diesen Post gelesen?

    Ansonsten habe ich das hier gefunden, das im wesentlichen dasselbe aussagt (“billions of km” für Oort-Wolke, aber auch Kuiper-Gürtel, “millions of km” für Asteroidengürtel). Die Schwierigkeit ist, die untere Größe für die Objekte festzulegen, denn je kleiner, desto mehr Objekte und desto geringer die Abstände. Die Zahlen sind also ein wenig willkürlich, daher wirst Du es genauer als auf die Größenordnung nicht finden.

  216. […] der heutigen Frage aus der Serie “Fragen zur Astronomie” wird es wieder einmal kosmologisch. In der letzten Woche habe ich mich ja mit der Dunkelheit […]

  217. #220 bikerdet
    16. Februar 2015

    @ Alderamin :

    Ja, Danke. Das ist genau genug.
    Klar, dass das nur auf die auch beobachtbaren Objekte angewendet werden kann. Mehr kennen wir ja nicht.
    Solche Grenzen sind immer irgendwie willkürlich, ansonsten müßte man es wohl bis auf die Plancklänge runterbrechen.

  218. #221 Jens
    17. Februar 2015

    Warum sind die Naturgesetzte/Konstanten so auf einander abgestimmt, dass sich Sterne bilden können?

  219. #222 Alderamin
    17. Februar 2015

    @Jens

    Die Kurzform der Antwort auf diese Frage lautet: das weiß kein Mensch.

    Mögliche Lösungen wären:
    – die Naturkonstanten müssen zwangsläufig bestimmte Werte haben, aber bisher gibt es keine Theorie, wie diese Werte zustande kommen
    – ein riesengroßer Zufall
    – es gibt andere Orte (in unserem Universum oder in anderen parallelen / früheren / späteren), in denen zufällig andere Werte angenommen werden, dann kann natürlich nur an Orten, wo sie genau die richtigen Werte zur Entstehung von Sternen, Planeten und Leben haben, jemand wie wir entstehen, der sich die Frage stellt, warum die Konstanten genau diese für das Leben günstigen Werte haben (schwaches anthropisches Prinzip). Genau so, wie höheres Leben nur auf einem der möglicherweise seltenen Planeten entstehen kann, der einen hinreichend langlebigen Stern im richtigen Abstand umkreist, einen großen Mond hat, Plattentektonik etc., und wir uns dann auch genau auf einem solchen Planeten wiederfinden. An einem anderen Ort kann es uns (oder sonst jemanden) schlicht und ergreifend nicht geben, und die Frage von niemandem gestellt werden.

    Was von diesen drei Möglichkeiten zutrifft, oder ob es noch eine ganz anderer Erklärung gibt, kann derzeit aber niemand sagen.

  220. #223 ImNetz
    18. Februar 2015

    Ein Beitrag über die Verdienste der morgenländischen Astronomiegeschichte vielleicht – mit Bezug zur “exotischen Moderne (https://www.spiegel.de/panorama/saudi-arabien-islamgelehrter-sagt-dass-sich-die-erde-nicht-dreht-a-1018916.html) wäre nett 😉

  221. #224 Florian Freistetter
    18. Februar 2015

    @ImNetz: Die Seite hier ist dafür da, um konkrete Fragen zur Astronomie zu stellen; nicht für allgemeine Artikelwünsche (und was sollte ich zu dem Spinner denn noch schreiben, außer “Ist falsch!”).

  222. #225 Hans-Walter Bronder
    Schiffweiler
    20. Februar 2015

    @Florian:
    Da man ständig größere Strukturen wie z.B. Filamente im Universum findet, darüber hinaus über Multiversen nachdenkt und auch im Kleinen unterhalb der Quarks noch Kleineres annimmt, kann es da nicht sein, dass es nie ein Größtes und nie ein Kleinstes gibt ? Wäre sowas mathematisch darstellbar ?

  223. #226 Joseph Quimby
    Wörth
    23. Februar 2015

    Ist für Sie als Wissenschaftler Religion bzw. Gott mit den heutigen astronomischen Erkenntnissen in Einklang zu bringen?

    Toller Blog! Weiter so! 🙂

  224. #227 Krypto
    23. Februar 2015

    @Joseph:
    Der tolle Blog hat auch eine ebenso tolle Such-Funktion 🙂
    Einfach mal die beiden Schlagwörter eingeben!

  225. #228 Florian Freistetter
    23. Februar 2015

    @Joseph Quimby: “Ist für Sie als Wissenschaftler Religion bzw. Gott mit den heutigen astronomischen Erkenntnissen in Einklang zu bringen?”

    Nein. Siehe hier: https://scienceblogs.de/astrodicticum-simplex/2011/10/13/wissenschaft-und-religion-vertragen-sich-einfach-nicht/

  226. #229 Waldi
    24. Februar 2015

    Hallo Florian,
    gestern hieß es auf zdfinfo ( Das Universum – Planeten, ZDFinfo, 23.02.2015 23:15, noch in der zdf mediathek verfügbar), dass der Jupiter die Erde vor dem Einschlag von Asterioden schützt. Ohne Jupiter würden in der Größenordnung 100 mehr Asteroiden auf die Erde kommen. So richtig gut erklärt wurde das aber nicht. Meine Frage: Wie macht das der Jupiter? Wie schafft er es, immer an der richtigen Stelle zu sein, um 99% aller Asterioden einzufangen? Schöne Grüße, W.

  227. #230 Dietmar
    24. Februar 2015

    @Waldi: Die Gravitation des Gasriesen lässt viele Asteroiden auf ihm niedergehen.

  228. #231 Mirko
    24. Februar 2015

    Ich denke mal, so schlau war der Waldi auch. Die Frage ist doch wohl anders gemeint: betrachten wir das Sonnensystem von oben, Jupiter befindet sich am großen Zeiger in 1 Uhr. Man kann annehmen, dass er auf Asteroiden in 1 – und vielleicht auch in 12 und 2 Uhr anziehend wirkt, obwohl diese ja dichter am Mittelpunkt liegen. Wie kann er die Erde aber beschützen, wenn die sich quasi auf dem kleinen Zeiger irgendwo zwischen 3 und 11 Uhr befindet?

  229. #232 Alderamin
    24. Februar 2015

    @Waldi

    Er räumte bei seiner Wanderung durch das Sonnensystem vor 3,8 Milliarden Jahren die meisten Objekte mti seiner Schwerkraft einfach ab. Wenn sie ihm zu nahe kamen, änderte sich ihre Bahn, so dass sie aus dem Sonnensystem heraus oder nach innen abgelenkt wurden (einige sind auch sicher mit Jupiter kollidiert).

    Die nach außen abgelenkten Objekte sammelten sich in der Oortschen Wolke, wo ihre Bahn sich durch die Gravitation der Milchstraße zirkularisierte, so dass sie nicht mehr ins innere Sonnensystem zurück kamen. Die nach innen gelenkten Objekte stürzten in die Sonne oder kollidierten mit den inneren Planeten (und gaben z.B. unserem Mond sein verkratertes Antlitz). Nach diesem “Schweren Bombardement (Heavy Bombardment) war dann deutlich weniger an Kleinkörpern im Sonnensystem unterwegs, so dass es heutzutage unwahrscheinlich ist, dass die Erde noch einmal von einem großen Objekt getroffen wird.

  230. #233 Alderamin
    24. Februar 2015

    @Mirko

    Das war ein langwieriger Prozess, man stelle sich eher vor, der Jupiter saust millionenmal um die Sonne herum und begegnet dabei ständig den Kleinkörpern, die ihm nahe kommen. Durch die ständigen Begegnungen verändert sich seine Bahn, sie ändert ihre elliptische Form und wird auch enger, so dass er einen breiten Streifen leerräumt.

    Objekte, die auf Bahnen unterwegs sind, die kleine Vielfache von Jupiters Umlaufzeit haben, trifft der Planet immer an der selben Stelle ihres Orbits (Orbitalresonanz) und zieht diesen durch seine Anziehungskraft in die Länge, was ihre Bahn chaotisch werden lässt, sie kreuzt die wiederholt die Bahn der inneren Planeten, bis sie mit diesen kollidieren o.ä. Der Jupiter wirkt als eine Art Schneepflug, der das Sonnensystem leergeschaufelt hat.

  231. #234 Hicks
    24. Februar 2015

    @Waldi und @Alle ob das auch stimmt, was ich mir vorstelle

    Die Asteroiden im Asteroidengürtel fliegen alle ungfär in die gleiche Richtung, denn Geisterfahrer habe auch im Weltall eine gesenkte Lebenserwartung. Wenn nun also doch mal wieder welche zusammenstoßen dann fliegen die neuen Splitter tendenziell wieder in die selbe Richtung und wenn wir pech haben eiert so ein “Querulanten” dann so herum, dass sich seine Umlaufbahn langsam der unsrigen annährt. Wie von Alderamin beschrieben wird er dabei aber noch sehr oft in Jupiters Einflußbereich kommen und evtl. ausgesiebt.

    Ein anderer (Vorstellungs-) Aspekt ergibt sich noch wenn man sich vorstellt, es sitzt jemand im Asteroidengürtel und versucht nun ein Steinchen direkt auf die Erde zu werfen. Bei einem so steilen Winkel Richtung inneres Sonnensystem, hat die Erde nicht mehr die Möglichkeit das Steichen gravitativ einzufangen und ein Treffer ist sehr unwahrscheinlich.

  232. #235 Alderamin
    24. Februar 2015

    @Hicks

    Wenn nun also doch mal wieder welche zusammenstoßen dann fliegen die neuen Splitter tendenziell wieder in die selbe Richtung

    Tendenziell schon, je nachdem, wieviel Wumms die Kollision hatte, können einige davon auf anderen Bahnen landen.

    Wie von Alderamin beschrieben wird er dabei aber noch sehr oft in Jupiters Einflußbereich kommen und evtl. ausgesiebt.

    “Ausgesiebt” heißt aber auch, dass Jupiter gelegentlich durch Orbitalresonanz den Vorrat an NEA (near-Earth-Asteroids) auffrischt. Die Objekte haben im inneren Sonnensystem keine so hohe Lebensdauer, irgendwann kommen sie dann doch einmal einem Planeten oder der Sonne in die Quere. Im inneren Sonnensystem ist es eng und die Objekte kreisen schneller als im Asteroidengürtel und außerhalb.

    Bei einem so steilen Winkel Richtung inneres Sonnensystem, hat die Erde nicht mehr die Möglichkeit das Steichen gravitativ einzufangen und ein Treffer ist sehr unwahrscheinlich.

    Wenn die Bahn des Asteroiden die Erdbahn kreuzt, ist eine enge Begegnung nur eine Frage der Zeit. Die Erde kann die Bahn des Asteroiden dann verändern (z.B. dass er gar nicht mehr in den Asteroidengürtel zurückkehrt, Absenkung seines Aphels), was die Begegnungen noch häufiger macht. Oder ihn weiter nach innen lenken. Aber jede solcherart geänderte Bahn kehrt wieder zum Ort der Änderung zurück, wenn es keine weitere Änderung durch einen anderen Planeten gibt.

    Das ist wie bei der Raumfahrt, ich kann den erdfernsten Punkt eines Satelliten anheben, indem ich am erdnächsten Punkt den Antrieb feuere. Der Satellit kehrt dann aber wieder zum ursprünglichen erdnächsten Punkt zurück.

    Umgekehrt kann ich den erdnächsten Punkt anheben, indem ich am erdfernsten Punkt Gas gebe – und der Satellite kehrt dann wieder zum ursprünglichen erdfernsten Punkt zurück. Ändert also die Erde die Bahn des Asteroiden, kommt er trotzdem wieder zur Erdbahn zurück.

    Und irgendwann trifft er dann gerade so mit der Erde zusammen, dass diese frontal getroffen wird. Es sei denn, die Venus oder der Merkur räumen ihn vorher ab.

  233. #236 Dietmar
    24. Februar 2015

    @Mirko: Ich hatte Waldi so verstanden, dass er gedacht hätte, Jupiter würde dann irgendwie schützend vor der Erde stehen. Deshalb wollte ich auf die Gravitation hinweisen. Das war in keiner Weise herabsetzend gemeint.

  234. #237 Noonscoomo
    Berlin
    24. Februar 2015

    David Wiltshire hat eine Theorie entwickelt, welche die beschleunigte Expansion des Universums ohne dunkle Energie erklären kann indem er ein zeitlich inhomogenes Universum annimmt (timescape cosmology). 2011 wurde das heiß diskutiert und es hieß, sein Modell würde sogar in manchen Aspekten besser passen als das Standardmodell. Was ist draus geworden? Ist das verworfen/widerlegt inzwischen? Wenn nicht, warum wird es nicht als Alternative genannt?

  235. #238 Hicks
    25. Februar 2015

    @Alderamin

    [quote]Aber jede solcherart geänderte Bahn kehrt wieder zum Ort der Änderung zurück, wenn es keine weitere Änderung durch einen anderen Planeten gibt. [/quote]

    Ich bin dem Irrglauben aufgesessen, dass die erste Sonnenannäherung eines “neuen” Asteroiden etwas besonders sei und dessen Bahn zwangsläufig ändert – babei fliegt das Ding ja nur einfach “geradeaus”.

  236. #239 monotom
    25. Februar 2015

    Hey Florian!
    Ich hätte mal eine Frage:
    Hintergrund: Ich bin auf der Suche nach einer Übersicht, wie weit alle Planeten des Sonnensystems IN DIESEM MOMENT (also im Idealfall Sekundenaktuell) von der Erde entfernt sind. Natürlich ohne Anspruch auf 100% Genauigkeit!

    Bisher habe ich nur den Mars gefunden, auf einer Seite des Department of Physics Atlanta:

    https://www.physics.emory.edu/astronomy/events/mars/calc.html

    Gibt es eine Formel oder einen Algorithmus, mit dem man die Entfernung aller Planeten berechnen könnte?

    Gruß und Kuss,
    Tom

  237. #240 Spritkopf
    25. Februar 2015

    @monotom

    Ich bin auf der Suche nach einer Übersicht, wie weit alle Planeten des Sonnensystems IN DIESEM MOMENT (also im Idealfall Sekundenaktuell) von der Erde entfernt sind.

    Die Astronomieprogramme Stellarium und Celestia können das. Sind beide kostenlos herunterzuladen. Stellarium ist ein Planetariumsprogramm und mehr für die Himmelsbeobachter gedacht, die gerne wissen möchten, wo ein bestimmter Stern, Planet usw. am Himmel zu finden ist. Celestia legt den Fokus auf die Simulation von Körpern im Sonnensystem und ist vermutlich für deine Zwecke etwas anschaulicher.

    Guck mal bei Wikipedia. Beide Programme haben ihre eigenen Einträge mit Link zur Downloadseite.

  238. #241 Alderamin
    25. Februar 2015

    @monotom

    Die Berechnung der Entfernung der Planeten erfordert die Berechnung ihrer Bahn, darüber sind Bücher geschrieben worden, das ist nicht mit einer einfachen Formel erledigt.

    Die meisten Planetariumsprogramme wie Stellarium oder Cartes du Ciel (googeln) können die Entfernung der Planeten anzeigen, wenn man Datum und Uhrzeit angibt, das Objekt sucht und sich dann dessen Eigenschaften anzeigen lässt.

    Eine tolle Web-Ressource ist der JPL Solar System Simulator, mit dem kann man sich Ansichten im Sonnensystem berechnen lassen und dabei werden auch die Entfernungen mit angegeben. Um etwa die Entfernung zum Mars angezeigt zu bekommen, wählt man “show me <Mars> as seen from <Earth> on <Datum/Uhrzeit>”, die anderen Einstellungen sind beliebig (man kann die angezeigten Orbits weglassen und das Blickfeld ändern, um die Beschriftungen besser lesen zu können). Nach Klicken auf “run simulator” wird Mars mit einer Entfernung von 331,739 Millionen km angezeigt.

  239. #242 Waldi
    26. Februar 2015

    @all: Danke für eure Beiträge zum “Ausputzen” von Asteroiden durch den Jupiter.

    In dem Film sah man zu der Aussage, dass der Jupiter das tun würde, lediglich in einer Sequenz einen Asteroiden auf ihm einschlagen. Ich habe es als Astronomie-Laie mir dann tatsächlich so vorgestellt, dass die meisten Asteroiden, die der Erde gefährlich werden könnten, von “irgendwo” her aus dem dreidimensionalen Raum kommen und die Umlaufbahn des Jupiter in einem zufälligen und damit meist ziemlich steilen Winkel kreuzen würden – wenn sie es denn überhaupt tun und nicht von “oben” kommen. Wie der Jupiter von denen 99% abfangen sollte, war mir völlig unverständlich.

    Wenn sie fast alle aus dem Asteroidengürtel “nahe” beim Jupiter stammen und sich allmählich in einer Eliipsen-förmigen Umlaufbahn dem Jupiter nähern und wenn man außerdem den Einfluss des Planeten von vor Urzeiten mit heranzieht, dann fängt die Aussage an Sinn zu machen. Aber solche Asteroiden müssten ja außerdem noch an Saturn usw. vorbei, der würde dann ja auch noch einen großen Prozentteil ablenken bzw. einfangen. Demzufolge können wir uns nicht nur beim Jupiter sondern auch beim Saturn bedanken, dass uns nicht so oft der Himmerl auf den Kopf fällt?!

    Ein wenig schlauer grüßt -W.

  240. #243 Waldi
    26. Februar 2015

    Das mit Saturn war natürlich Quatsch, merke ich gerade. Wenn dann Mars, aber der ist vermutlich nicht gewichtig genug. – W.

  241. #244 monotom
    26. Februar 2015

    @ Alderamin
    @ Spritkopf

    Super Tipps! Besten Dank! 🙂

  242. #245 ron
    1. März 2015

    Hallo FF-Community,

    gibt es das hochauflösende Foto vom 14.02.15 von Tschurjumow-Gerassimenko auch mit einem eingefügten “Größenverhältnisbeispiel”? Zum Beispiel ein Fußballfeld… 😉

    Ich kann mir leider gar kein Bild machen welche Größe der Bildausschnitt hat, da ich von jeher eine Entfernungs- und Größenschätzschwäche hab.

    Danke für die Infos!

    ron

  243. […] gibt es in der Serie “Fragen zur Astronomie” eine Frage, die mir oft gestellt wird und die eigentlich ziemlich simpel klingt: Wo beginnt der […]

  244. #247 Jens
    3. März 2015

    Verlieren die Planeten beim Umkreisen der Sonne Bewegungsenergie durch Abstrahlen von Gravitationswellen?
    Wenn ja hat dies Auswirkung auf deren Umlaufbahn?

  245. […] gibt es in der Serie “Fragen zur Astronomie” wieder einmal einen Klassiker. Die Frage “Gibt es andere Universen?” wird mir recht […]

  246. #249 Jabir
    Leipzig
    9. März 2015

    Die Expansion des Universums wird ja schön veranschaulicht durch einen sich ausdehnenden kugelförmigen Luftballon mit Galaxienkreuzen – wobei es einen Mittelpunkt im Zentrum der Kugel.gibt. Das legt die Frage nahe: Was spricht gegen die Annahme, dass unser Universum in ein räumlich vierdimensionales Gebilde eingebettet wäre und dort einen Mittelpunkt hätte?

  247. #250 Ferrer
    9. März 2015

    Guten Tag! Ist es prinzipiell möglich, den Schwerpunkt des Universums zu bestimmen? Wenn nicht, was spricht dagegen?

  248. #251 Mischa Weg
    wald am arlberg
    9. März 2015

    Hallo ich bin erst 16 jahre alt
    Ich kenne mich sehr gut mit dem universum aus
    Ich schätze oder weiß das es vor unseren UNIVERSUM schon ein universum gab
    Es gab ein VORUNIVERSUM es war so groß.
    oder doppelt so groß wie das jetzige ist

    Als es wuchs dehnte es sich aus und immer
    weiter biss es schrumpfte und so dicht und
    heiß wurde heißer als alle Sonnen und
    schwarze löcher und alles zusammen
    Es schrumpfte bis auf die größe eines atoms vileicht auch kleiner
    Und weil es so dicht und heiß war explodierte es wir betzeichnen das als urknall
    Und das widerholte sich unentlich lang
    Wie ein baby das zum kind wird dann zum erwachsenen und bis zum tod

  249. #252 Krypto
    10. März 2015

    @Jabir:
    Abgesehen davon, dass unser Universum bemerkenswert flach ist und der sich füllende Ballon nur die Ausdehnung visualisieren soll, spricht nichts gegen weitere Dimensionen.
    @Ferrer: Nein, weil ein erheblicher Teil des Universums mit anderen Teilen nicht mehr wechselwirken kann.
    @Mischa: Super, dass Du so ein großes Interesse hast!
    Für Deine Idee sieht es leider nicht so gut aus, weil man festgestellt hat, dass sich das Universum immer schneller ausdehnt und keine Kraft entdeckt wurde, die dem effektiv entgegenwirken könnte.

  250. #253 Alderamin
    10. März 2015

    @Krypto, Ferrer

    Und weil es keinen Mittelpunkt hat, jede Galaxie sieht sich im Mittelpunkt. Möglicherweise ist das Weltall unendlich, oder aber auch ein endlich großer Raum mit einer geschlossenen Topologie (z.B. ein 3-Torus). Die Annahme ist jedenfalls, dass es keinen Rand gibt, dass das Universum homogen und isotrop ist, und deswegen taugt jeder Punkt gleich gut als Schwerpunkt oder nicht (bis auf lokale Unebenheiten durch Massekonzentrationen wie Galaxienhaufen).

  251. #254 Ferrer
    10. März 2015

    @Krypto, Alderamin: Vielen Dank, einfach und einleuchtend.

  252. #255 Mischa Weg
    10. März 2015

    Hir schreibt nochmal mischa weg
    Ich meine nicht jetzt unser universum sondern die davor weil vileicht diese gröser aber auch vileicht doppelt so viele galaxyen besitzen
    Meine zweite teorie wäre das das universum eine riesiege galaxy war inmitten war ein schwarzes loch mit der grösse von locker 1000 bis 1000000 galaxyen das war so dicht und stark war das es in sich koliebierte

  253. #256 Hans-Walter
    Schiffweiler
    10. März 2015

    Hallo Mischa,
    bei allem Wohlwollen: Lerne doch lieber erst mal richtig schreiben !

  254. #257 Hans-Walter
    10. März 2015

    @Mischa
    Hallo Mischa,
    war nicht so gemeint, hab’ eben erst gesehen, dass Du erst 16 Jahre alt bist, toll, dass Du Dich schon so intensiv für Astronomie interessierst.
    Ich bitte Dich nochmals um Entschuldigung für meinen Kommentar von vorhin, sorry !!!

  255. #258 Mischa Weg
    10. März 2015

    Passt schon nichts pasiert

  256. #259 Florian Freistetter
    10. März 2015

    Ich weiß, dass sich mittlerweile schon lange niemand mehr an das hält, was ich oben im Text geschrieben habe, aber darf ich trotzdem nochmal darauf hinweisen, dass die Kommentare hier eigentlich nur dazu da sind, um FRAGEN zu stellen, aber nicht für eine allgemeine Diskussion? Wenn, dann beschränkt euch bitte auf konkrete ANTWORTEN – und wenn ihr keine Antwort habt/kennt, dann schreibt bitte auch nichts. Diskutiert werden kann anderswo im Blog ja eh überall – und hier sorgen die vielen (nicht zielführenden) Kommentare (wie zB zur Orthografie) dafür, dass die Seite immer langsamer und langsamer lädt. Und dann wird mir nichts anderes übrig bleiben, als ALLE Kommentare hier zu löschen, weil die Seite dauerhaft als Übersicht über die “Fragen”-Serie dienen soll.

  257. #260 Hans-Walter Bronder
    10. März 2015

    @Florian
    Tut mir Leid, Du hast Recht, war dumm von mir, kommt nicht wieder vor !!

  258. #261 bikerdet
    10. März 2015

    @ Mischa :
    Auch Deine zweite Idee kann nicht funktionieren. Wie soll ein schwarzes Loch kollabieren ? Rein nach unserer Erklärung besteht ein schwarzes Loch aus einer Singularität, sprich einem punktförmigen ‘Irgendwas’. Es ist auch nicht möglich, das sich das noch zusammenzieht. Außerdem würde ein schwarzes Loch die umgebenden Galaxien nicht anziehen.
    Ich möchte Dir aber einen Tipp geben, wie Du Dir das Wissen ganz schnell und toll aneignen kannst. Höre Dir bitte den Podcast von Florian an. Er heißt ‘Sternengeschichten’ und hat aktuell 118 Folgen. Dort erklärt Dir unser Gastgeber hier, der ja einen Doktor in Astronomie hat und sich super gut auskennt, sehr genau und ausführlich die Zusammenhänge im Weltall. Suche Dir zuerst ein paar Folgen aus, die Dich brennend interessieren. Aber es wäre schon super, wenn Du Dir nach und nach alle Folgen anhörst. Ich habe die Folgen (alle) auf meine I-Pod und höre sie beim Joggen. Die Folgen gibt es übrigens auch bei YouTube, falls Dir das lieber ist. Wenn Du Dir alles genau angehört hast, wirst Du SELBER wissen warum Deine Ideen nicht funktionieren.
    Viel Spaß beim hören !

  259. #262 Jens
    10. März 2015

    Leider bleiben viele hier gestellten Fragen unbeantwortet. Wie gehn wir damit um?

  260. #263 Mischa Weg
    10. März 2015

    Hallo hier schreibt mischa weg
    Kann das sein weil es gipt die quards und ich habe mir überlegt ob in den quards ein anderes universum gipt und von diesem univerum die quards vileich unser universum oder ein anderes ist könnt ihr mir die lösung schreiben und auch schreiben ob ich recht habe

  261. #264 Florian Freistetter
    10. März 2015

    @Jens: “Wie gehn wir damit um?”

    Müsst ihr denn damit umgehen? Wie gesagt: Es gibt viele Seiten für allgemeine Diskussionen hier im Blog. Und ich beantworte Fragen ja auch. Und ich kann eh niemanden hindern, hier Fragen zu beantworten und wild zu diskutieren. Ich wollte halt nur anregen, vielleicht wirklich nur dann zu schreiben, wenn man auch eine Antwort HAT und nicht alles mit Meta-Diskussion zuzuschreiben (so wie das jetzt wieder passiert). Aber egal – wenns zu voll wird, dann lösche ich das halt wirklich raus. Und wenn unbedingt Bedarf für NOCH eine Extra-Diskussionsseite besteht (wie gesagt – das gibts schon), dann kann ich sowas auch aufmachen. Ich möchte diese Fragenseite halt nur praktikabel halten.

  262. #265 bikerdet
    11. März 2015

    @ Mischa Weg :
    Höre Dir bitte die Sternengeschichten an, Florian hat dort alle Deine Fragen sehr ausführlich und leicht verständlich beantwortet.
    Solltest Du DANACH noch Fragen haben, so kannst Du sie gerne hier stellen, z.B. so : Ich habe Sternengeschichten-Folge xx gehört und noch Fragen dazu …
    Darauf wirst Du eine korrekte Antwort erhalten. Aus Deinen bisherigen zusammenhanglosen Ideen wird man nicht so richtig schlau, leider.

  263. #266 Mischa Weg
    13. März 2015

    Ich habe schon so viele bücher und dokumentationen angesehen bin schon auf wissenschaftskonkressen gewessen und habe mit meinem teleskop schon bis zur andrommeda galaxy geschaut ich kenne auch einen von meinen alten lehrern der einen doktortitel in astronomie hat
    Der hat mir viel beigebracht

  264. #267 Jens
    15. März 2015

    Ist unser Planetensystem ein typisches Planetensystem?

  265. […] der Frage zu Paralleluniversen sehr philosophisch und abstrakt wurde, geht es heute in der Serie “Fragen zur Astronomie” wieder ganz konkret und bodenständig mit einem Klassiker weiter. Die Frage ist eine, die mir sehr, […]

  266. #269 bikerdet
    16. März 2015

    @ Jens :
    Nein, es gibt kein ‘typisches’ Planetensystem. Die Entstehung von Planeten ist von sehr vielen Faktoren abhängig. Z.B. von der Größe und Zusammensetzung der Sonne, wieviele Sonnen und wie nah sie ‘nebenan’ entstehen. Sobald die Planetesimale entstehen, in unserem System soll es rund 100 gegeben haben, geht es ziemlich chaotisch zu. Kollisionen der Planetesimale sind ‘an der Tagesordnung’ , Merkur, Erde und Uranus sollen getroffen worden sein, Venus sogar zwei mal. Die äußeren Gasplaneten haben die Kometen (und übrigen Kleinkörper) ordendlich durchgerührt und im Sonnensystem aufgeräumt. Dabei kam es zu einer planetaren Migration. Juputer rückte näher an die Sonne, Saturn, Neptun und Uranus weiter weg. Uranus und Neptun tauschten dabei sogar ihre Plätze. Bei jeder dieser Aktionen hätte es im schlimmsten Fall auch zu einer Zerstörung einzelner Planeten kommen können. Ob das Planetensystem danach noch stabil gewesen wäre oder sich komplett umgestaltet hätte kann natürlich niemand sicher sagen. Die Simulationen lassen viele Möglichkeiten zu
    Du siehst, ein typisches Sonnensystem gibt es nicht und auch unseres ist weit davon entfernt.

  267. #270 Dietmar
    16. März 2015

    @Jens: Was man aber meines Wissens sagen kann, ist, dass Planetensysteme an sich häufig, also insofern typisch, sind und die Zahl der Planeten die der Sonnen wohl übersteigt.

  268. #271 robsn
    16. März 2015

    Wie kann man Licht bremsen? Wie weit wurde Licht bereits gebremst? Und wenn Licht tatsächlich gebremst werden kann, weshalb hört man immer von der konstanten Lichtgeschwindigkeit?

  269. #272 bikerdet
    16. März 2015

    @ robsn :
    Die Lichtgeschwindigkeit ist nur im Vakuum konstant. Sobald sich Licht durch ein Medium ( Luft, Wasser, usw) bewegt, wird es abgebremst. Wie weit das Licht abgebremst wird, hängt vom durchquerten Medium ab.

  270. #273 robsn
    16. März 2015

    Danke, bikerdet. So halbwegs ist mir das auch alles klar. Ein ausführlicher Artikel von Florian darüber, wäre sicher lesenswert. Mir schwirren da etliche weiterführende Fragen durch den Kopf. Zum Beispiel ob das Licht nach dem Durchqueren des bremsenden Mediums, beim Widereintritt ins das Vakuum wieder beschleunigt (was für ein Wahnsinn =D) und so weiter und so weiter.

  271. #274 robsn
    16. März 2015

    Das Licht, das uns von fernen Sternen aus erreicht, hat ja sicher auch nicht nur Vakuum hinter sich. Erreicht uns also aus allen erdenklichen Richtungen verschieden schnelles Licht? Das könnte doch sicher nen Artikel füllen.

  272. #275 bikerdet
    16. März 2015

    @ robsn :
    Hat Florian doch schon. Schaue mal ganz an den Anfang des Artikels, ein Link zu Frage 5. Da wirst Du fündig.

  273. #276 Alderamin
    16. März 2015

    @robsn

    Das Licht ist im Vakuum immer gleich schnell, das besagt die spezielle Relativitätstheorie. Sogar, wenn sich die Erde einer Lichtquelle entgegenbewegt oder von ihr weg, misst man stets genau c (nur die Frequenz ändert sich).

    Wenn Licht durch ein Medium läuft, wird es nicht wirklich abgebremst, sondern von den Elektronen absorbiert und wieder emmitiert, es wird also immer wieder kurz aufgehalten. Zwischen diesen Vorgängen breitet es sich mit c aus. Wenn es ein Medium verlässt, wird es nicht mehr aufgehalten und deswegen braucht es auch nicht zu beschleunigen.

    So ähnlich funktionierte auch dieses Experiment. Das Licht bleibt nicht wirklich stehen, es überträgt sich nur auf Elektronen, die einen Lichtpuls aufnehmen, verzögern und wieder abgeben können.

  274. #277 Mischa Weg
    16. März 2015

    Gibt es eine unentlichkeit des raumes der zeit und der geschwindichkeit im universum ?

  275. #278 bikerdet
    16. März 2015

    @ Mischa :
    Toll, ich hätte auch gerne Astronomie in der Schule gehabt. Allerdings kann man die Andromedagalaxie bereits mit bloßem Auge sehen, unter halbwegs dunklem Himmel gelingt das problemlos.
    Zu Deinem letzten Beitrag : Wir wissen es nicht. Hättest Du allerdings meinen Tipp beherzigt und die Sternengeschichten gehört, wüßtest Du das bereits. Deine ‘Ausbildung’, die Du uns in #266 geschildert hast, hat Dir nicht geholfen. Schade um die vertane Zeit.

  276. #279 Mischa Weg
    18. März 2015

    Ich habe keine ausbildung
    Das habe ich in der hauptschule gelernt
    Der lehrer hat nur mich unterrichtet weil ich mich dafür interessire.

    Die andern schüler haben sich nicht interresiert.
    Und glaubt ihr jeder für sich auch ohne forschungen
    Gipt es parraleluniversen. Also 2 dimension, 3 dimension 4,5,6…

  277. #280 Florian Freistetter
    18. März 2015

    @Mischa: Lieber Mischa: Schau dir doch mal die ganzen schon beantworteten Fragen an, die oben auf der Seite verlinkt sind. Klick die Links an und lies dir die Artikel durch. Da sind viele deiner Fragen schon ausführlich beantwortet worden. Zum Beispiel die zu den Paralleluniversen: https://scienceblogs.de/astrodicticum-simplex/2015/03/09/gibt-es-andere-universen/

  278. […] heutigen Artikel der Serie “Fragen zur Astronomie” geht es zur Abwechslung mal wieder um Physik. Und um eine Frage, die die Wissenschaftler schon seit […]

  279. #282 B. Lothar
    Ludwigshafen am Rhein
    24. März 2015

    Das Hubble-Teleskop war ja nun schon eine teure Angelegenheit. Aber man konnte bei einem Fehler immerhin mit dem Space-Shuttle hinfliegen, um die Angelegenheit wieder in Ordnung zu bringen. Das geht bei dem JWST (dem Hubble Nachfolger) nicht, da es zu weit von der Erde aus positioniert wird. Warum geht man, bei einem solchen extrem anspruchsvollen techn. und finanziellen Unternehmen (die Rede ist von derzeit 8 Milliarden Euro?), so ein hohes Risiko ein ?
    Fehler können immer auftreten, da können die Ingenieure
    vorher noch so viel prüfen und testen. Das muss man sich mal vorstellen: das JWST ist noch teuerer als der riesige LHC bei Genf.

    Lothar

  280. #283 Krypto
    24. März 2015

    @B.Lothar:
    Ich bin überzeugt davon, dass sich die Planer und Finanzierer über dieses Thema intensiv ausgetauscht haben.
    Wahrscheinlich wirst Du bei ESA,NASA & Co. entsprechende Informationen finden.

  281. #284 Alderamin
    24. März 2015

    @B. Lothar

    Eigentlich sollte das JWST ja nur 1,5 Milliarden kosten…

    Der Grund für den Lagarange-2-Punkt ist, dass es sich um ein Infrarot-Teleskop handelt, und das ist im Erdschatten leichter zu kühlen (als etwa LEO-Infrarot-Teleskope, die z.B. mit Helium gekühlt werden; JWST guckt allerdings nicht im fernen Infrarot, wie diese, und kommt ganz ohne Helium aus).

    Das Shuttle gibt’s mittlerweile ohnehin nicht mehr, und mit der Orion-Kapsel wird man problemlos auch bis zum JWST kommen. Das war zwar ursprünglich nicht geplant, aber ich las mal von Vorschlägen, solche Wartungsmissionen doch durchzuführen. Man weiß ja gar nicht, wo man mit der Orion hin soll, solange der Mars noch zu ambitioniert ist.

  282. #285 Alderamin
    24. März 2015

    @B. Lothar

    Noch ein Grund für L2 ist, dass man ohne Unterbrechung fast in jeder Richtung lange belichten kann. Beim HST-Orbit kam nach 45 Minuten meistens die Erde ins Bild. Manchmal störte der Mond. Im L2 hat man alles Störende (Sonne, Erde, Mond) in der gleichen Richtung und sieht von Erde und Mond auch nur die dunklen Seiten.

  283. #286 Jens
    28. März 2015

    Gibt es Kugelsternhaufen auch um Zwerggalaxien?

  284. #287 Alderamin
    28. März 2015

    @Jens

    Sieht so aus.

    Nochmal @B. Lothar:

    Das JWST hat einen Docking-Mechanismus für Orion-Kapseln mitbekommen.

  285. […] die Sonne herum ändern? Das sind interessante Fragen, die ich deswegen auch heute für die Serie “Fragen zur Astronomie” ausgewählt […]

  286. #289 Hans-Walter
    8. April 2015

    @Florian
    Hab’ vor kurzem gelesen, dass man plant, Interferometrie mit optischen Teleskopen zu betreiben. Wie realistisch ist das, wegen des dazu benötigten “Supercomputers” ?

  287. #290 Hans-Walter
    8. April 2015

    @Florian
    Nachtrag: Ich meine natürlich Interferometrie mit sehr weit voneinander entfernt stehenden optischen Teleskopen, z.B. Basislängen von mehreren hundert-, vielleicht mehreren 1000 Kilometern.

  288. #291 Jens
    18. April 2015

    Wie werden Gravitationlinsen für die astronomische Forschung genutzt?

  289. #292 bikerdet
    24. April 2015

    Hallo Florian

    Ich habe am Mittwoch einen Vortrag an der Sternwarte gehört. Das Thema war die Entstehung des Sonnensystems. Dort wurde erklärt, das die Supernovaexplosion, die zur Entstehung des Sonnensystems führte, nur max. 1 Lichtjahr entfernt stattgefunden hat. Die Planetenentstehung sei durch einen Vorbeizug einer Sonne in ähnlicher Entferung initiert woren. Dabei sei alle Materie außerhalb des (heutigen) Kuipergürtels weggerissen worden. Hierbei sei die Ursprungswolke so verformt worden, das innerhalb des Kuipergürtels die Planeten entstanden sind.

    In wieweit ist diese Entstehungsgeschichte wahrscheinlich und wie passt die Oortsche Wolke dazu. Könnte es sie nach diesem Szenario überhaupt geben ?

    Leider konnte ich von dem Vortragenden keine Info’s zur Quelle erhalten.

  290. #293 Florian Freistetter
    24. April 2015

    @bikerdet: “In wieweit ist diese Entstehungsgeschichte wahrscheinlich und wie passt die Oortsche Wolke dazu.”

    Unwahrscheinlich ist es nicht; ob es aber so exakt gesagt werden kann, kann ich nur beurteilen, wenn ich dazugehörige wissenschaftliche Arbeit lesen könnte. Und die Oortsche Wolke entstand erst nach/während der Planetenentstehung. Da hat sich das ganze Zeug angesammelt, das während der Entstehung durch nahe Begegnungen zwischen den Objekten aus dem inneren Sonnensystem rausgeschleudert worden ist.

  291. #294 bikerdet
    24. April 2015

    @ Florian :
    Vielen Dank für die Info. Ich hatte mal gelesen, das das Sonnensystem etwa 750.000 bis 800.000 Jahre nach der SN-Explosion entstanden sei und war davon ausgegangen das sie weiter weg gewesen wäre. Müßte so ein Überrest (z.B. Neutronenstern) nicht die nahe Entstehung einer Sonne be- oder sogar verhindern.

    Leider konnte ich über die wissenschaftliche Arbeit nichts herausfinden, da der Vortragende sehr schnell gegangen ist. Ich erinnere mich nur daran, das auf der Folie auch ein paar Bilder waren, die den Vorgang schematisch darstellten. Leider war die Beschriftung so klein, das ich sie nicht entziffern konnte. Ich habe aber eine Mailadresse und werde mal sehen, ob ich eine Kopie des Vortrages bekommen …

  292. #295 bikerdet
    24. April 2015

    @ Florian :
    Leider ist meine Antwort im Nirgendwo verschwunden. Da ich den Text auch nicht nochmal verschicken kann, möchte ich Dich bitten mal nachzusehen ob er noch zu finden ist. Er ist jedenfalls nicht in der Moderation …

  293. #296 Mark S
    26. April 2015

    Warum gibt es keine Pläne für ein Weltraumteleskop mit Interferometrie im optischen oder IR Bereich?

    Gibt es technische Probleme, bzw. macht es keinen Sinn,
    ein Weltraumteleskop aus 2-3 “kleinen” Spiegeln auf einer langen “Stange” (Grundlinie) aufzubauen?

    Natürlich ist die Lichsammellleistung “gering” aber die Winkelauflösung müsste doch bombastisch sein!(?)

  294. #297 Jens
    26. April 2015

    Wie groß ist die größte Galaxis im Universum?

  295. […] der Serie Fragen zur Astronomie geht es heute wieder Mal um schwarze Löcher. Und diesmal um die interessante aber auch […]

  296. #299 Karsten
    27. April 2015

    Welcher Stern (außer der Sonne selbst) stellt eine Gefahr für das Leben auf der Erde dar? Ich denke da an solche Sachen wie Gammablitze bei Supernova-Explosionen (einer davon hat die Erde ja wohl schon mal getroffen, vor ca 450 Mio Jahren, und soll für das erste Massenaussterben verantwortlich gewesen sein) oder braune Zwerge, die möglicherweise am Sonnensystem vorbeiziehen und dabei mit Kuipergürtel und/oder Oortscher Wolke gravitativ interagieren

  297. #300 Alderamin
    27. April 2015

    @Karsten

    Möglicherweise dieser hier, der vor 70000 Jahren wahrscheinlich durch die Oortsche Wolke geflogen ist. Was immer er dort durcheinander gebracht haben sollte, braucht allerdings rund 2 Millionen Jahre bis ins innere Sonnensystem, das ist also nichts, was uns jetzt Kopfzerbrechen machen müsste.

    Eta Carinae könnte einen Gamma-Burst verusachen, ist aber wahrscheinlich weit genug weg, dass dieser keinen großen Schaden verursachen würde, und da die Bursts hauptsächlich entlang der Drehachse des Sterns abgefeuert werden und diese anscheinend bei Eta Carinae nicht in unsere Richtung weist (im Abstract steht 60°Abweichung von der Sichtlinie), auch hier Entwarnung.

  298. #301 Alderamin
    27. April 2015

    @Jens

    Wie groß ist die größte Galaxis im Universum?

    Niemand kann alle Galaxien des Universums überblicken, aber die größte bekannte Galaxie ist vermutlich IC 1101 mit 212000 +/- 39000 Lichtjahren Radius und einem umgebenden schwach, leuchtenden Halo von sogar bis zu 2 Millionen LJ Radius. Es ist laut Artikel unklar, ob es sich eher um eine flache Scheibe, die wir von oben sehen, oder um ein in der Tiefe ausgedehnteres Objekt handelt. Die Galaxie wird auf 100 Billionen Sterne geschätzt (das wäre etwa ein Faktor 30 zu Milchstraße).

    Übrigens sagt man “Galaxis” (= die Galaxie) nur zur Milchstraße, bei allen anderen sagt man “Galaxie” mit “ie” am Ende).

  299. #302 Alderamin
    27. April 2015

    @Jens

    Sorry, Link auf den oben zitierten Artikel vergessen:

    https://en.wikipedia.org/wiki/IC_1101

  300. #303 Chytharo
    NRW
    29. April 2015

    Könnte man Objekte zwischen dem Mond und der Erde mit bloßem Auge bei vollem oder zumindest fast vollem Zustand sehen?
    Also wenn zum Beispiel Raumstationen, Satelliten, Asteroiden ect. genau in der Blicklinie von der Erde auf dem Mond durchfliegen.
    Würde man dann so etwas wie einen kleinen dunklen Fleck sehen können, der “über” den Mond fliegt, wenn man genau zu dem Zeitpunkt nachts von der Erde aus auf den Mond schauen würde?

  301. #304 Florian Freistetter
    29. April 2015

    @Chytharo: Gute Frage! Werd ich mir Gedanken drüber machen und sie in einem meiner nächsten Artikel zum Thema beantworten.

  302. #305 Kyllyeti
    29. April 2015

    @Chytharo

    Hier gibt’s übrigens ein schönes Bild der ISS vor der dunklen Hälfte des Mondes.

  303. […] Weiter ging es auch bei den Fragen zur Astronomie: […]

  304. […] der heutigen Ausgabe von “Fragen zur Astronomie” geht es nicht um das ferne Universum, Galaxien oder schwarze Löcher. Heute wird es wieder einmal […]

  305. #308 Ridi
    4. Mai 2015

    1- Gibt es Planeten und/oder Sonnen, die sich nicht drehen und/oder nicht bewegen?

    2- Weshalb müssen sich Himmelskörper auf Bahnen umkreisen, anstatt direkt aufeinander zu fallen? z.B. weshalb haben sich die Planeten so in Bewegung gesetzt, dass diese sich um die Sonne kreisen?

  306. #309 Alderamin
    4. Mai 2015

    @Ridi

    zu 1: wir kennen keine. Alle Himmelskörper drehen sich. Es gibt lediglich einige, die für einen Umlauf um einen anderen, größeren Körper genau so lange brauchen, wie sie für eine Umdrehung brauchen, und dem größeren Körper somit stets die gleiche Seite zuwenden, so wie das der Mond zur Erde tut (oder der Pluto und sein Mond Charon jeweils beide zueinander). Das nennt man gebundene Rotation.
    Dass sich ein Körper gar nicht dreht, ist einfach extrem unwahrscheinlich, rein statistisch gesehen. Jede noch so kleine Abweichung vom Stillstand wäre eine langsame Rotation. Der Planet mit der langsamsten Rotation im Sonnensystem ist übrigens die Venus, sie braucht ca. 243 Tage für eine Umdrehung. Das ist länger als ihre Umlaufzeit um die Sonne von 224,7 Tagen.

    zu 2: Die Himmelskörper fallen nicht direkt aufeinander zu, weil der sogenannte Drehimpuls eine Erhaltungsgröße ist, er kann sich nicht ändern. Vereinfacht gesagt ist der Drehimpuls eines Planeten seine Masse mal Abstand zum Stern mal Umlaufgeschwindigkeit (der Anteil der Geschwindigkeit, der nicht senkrecht auf den Stern weist). Eine Umlaufgeschwindigkeit > 0 ergibt sich schon dann, wenn der Planet nur ein wenig am Stern vorbeizielen würde; nur bei einem frontalen Flug direkt auf den Stern zu wäre die Umlaufgeschwindigkeit und der Drehimpuls beide genau 0.

    Wenn nun aber der Drehimpuls nicht 0 ist, dann bedeutet dies, dass die Umlaufgeschwindigkeit größer werden muss, wenn der Abstand kleiner wird (die Masse ändert sich ja nicht, also muss die Vergrößerung der Geschwindigkeit die abnehmende Entfernung ausgleichen, damit der Gesamtbetrag gleich bleibt). Eine höhere Umlaufgeschwindigkeit heißt aber auch, dass die Fliehkraft beim Umlauf zunimmt, die den Planeten nach außen drückt, während ihn die Anziehungskraft nach innen zieht. Deswegen wird er um den Stern herum gelenkt und saust mit dem Schwung der gewonnen Geschwindigkeit wieder weg vom Stern, wobei er (Abstand nimmt zu!) wieder langsamer wird, bis er wieder zurück fällt. So ergibt sich eine mehr oder weniger elliptische Bahn.

    Die Planeten und die Sonne haben sich urpsrünglich aus einer Gaswolke gebildet, die unter ihrer eigenen Schwerkraft kollabierte. Auch hier galt die Erhaltung des Drehimpulses: eine (stets vorhandene, siehe Punkt 1) zufällige turbulente Drehung der Wolke führte dazu, dass sie sich zu eine Scheibe abflachen musste, die wegen der Erhaltung des Drehimpulses nicht komplett in die Sonne stürzen konnte. Alle Bahnen von Teilchen, die nicht in der Scheibenebene lagen, oder deren Bahnen zu elliptisch waren, waren auf Kollisionskurs mit der Scheibe, und solange diese noch aus Staub und Gas bestand, war die Chance groß, dass solche Teilchen dann auch wirklich mit anderen zusammenstießen. Durch Zusammenstöße und Reibung blieb nur eine flache Scheibe übrig, in der sich die Teilchen auf annähernden Kreisbahnen um die Sonne drehten. In der Scheibe enstanden aus diesem Material dann die Planeten, die heute noch alle in der gleichen Ebene der ursprünglichen Scheibe um die Sonne kreisen, und deren Bahnen immer noch sehr nahe an Kreisen sind (bis auf die von Mars und Merkur, die merklich elliptisch sind, was auf die wechselseitige Anziehung zwischen den Planeten zurückzuführen ist). Die Planeten waren somit ein Mittel der Sonne, um den überflüssigen Drehimpuls der ursprünglichen Gaswolke loszuwerden.

  307. #310 B. Lothar
    Ludwigshafen am Rhein
    7. Mai 2015

    Hallo,
    meine Frage zielt ab, auf die Leistungsfähigkeit der Großteleskope.
    In einem Vortrag auf You Tube/MPI HD) hörte ich, daß das LBT von der räumlichen Auflösung her gesehen, derzeit das stärkste auf der Welt ist. Wenn beide 8,4m Spiegel zusammengeschaltet werden, dann hat es die räumliche Auflösung eines 23m Spiegels.
    Über das VLT in Chile habe ich gelesen, dass alle vier Spiegel kombiniert werden können, sodaß sie wirken, wie ein Teleskop mit einem Durchmesser von 200m. Die beiden Keck-Teleskope lassen sich sicher auch zusammenschalten und erzielen dann auch eine deutlich größere Auflösung als ein Spiegel alleine.
    Ich frage mich dann, wozu muss man dann noch für sehr viel Geld, in den nächsten Jahren 30 bzw. 40m Spiegel bauen, wenn das VLT heute schon die Auflösung eines 200m Spiegels bringt?
    Können Sie mir bitte die Dinge hier mal erklären, sodass ich es verstehen kann. Welche Teleskope bringen was genau und worin liegen die Unterschiede, ob das Teleskop alleine oder mit anderen Teleskopen zusammengeschaltet betrieben wird.
    Viele Grüße
    Lothar

  308. @Lothar: Das VLT ist ein Radioteleskop. Da kannst du die Signale der einzelnen Teleskope aufzeichnen und sie nachträglich am Computer interferieren und so ein entsprechend großes Teleskop simulieren. Bei optischen Teleskopen geht das aber nicht. Da müsstest du die Daten der einzelnen Teleskope in Echtzeit austauschen und entsprechend schnelle Leitungen durch die ganze Welt legen. Abgesehen davon funktioniert Interferometrie bei den viel kleineren optischen Wellenlängen auch nicht so gut wie bei den größeren Radiowellenlängen.

  309. #312 Lutz Komischke
    8. Mai 2015

    Hallo zusammen,

    ich habe gestern den Podcast über die Missverständnisse
    zu Schwarzen Löchern gesehen.

    Dort hieß es das die Materie zu etwas zusammengepresst wird,
    wovon unbekannt ist zu was.

    Zu reiner Energie kann es ja nicht sein, sonst wäre ja auch
    keine Gravitation mehr vorhanden.

    Wäre es nicht naheliegend anzunehmen, das es zu
    “Dunkler Materie” zusammengepresst wird.

    Die Gravitation ist noch vorhanden und keine
    Elektromagnetische Strahlung mehr da.

    Und wenn die Supermassiven Schwarzen Löcher in den
    Galaxien “verdunsten” weis man ja wohin die Dunkle Materie
    sich unsichtbar hin verteilt. hat oder nicht ?

  310. #313 Alderamin
    8. Mai 2015

    @Florian

    Das VLT ist ein Radiotelekop.

    Ich glaube Du verwechselst das VLT mit dem VLA..

  311. @Alderamin: Da hast du vollkommen Recht. Aber im Prinzip bleibt mein Argument bestehen. Die VLTs stehen alle direkt nebeneinander, da kriegt man die Daten leicht synchronisiert. Wenn die eine größere Distanz überspannen sollen, wirds kompliziert.

  312. #315 Jens
    8. Mai 2015

    Sind die Kugelsternhaufen im Halo der Milchstraße älter als die Milchstraße selbst?

  313. #316 Alderamin
    8. Mai 2015

    @Lutz Komischke

    Zu reiner Energie kann es ja nicht sein, sonst wäre ja auch keine Gravitation mehr vorhanden.

    Selbstverständlich erzeugt Energie Gravitation. Im sogen. Energie-Impuls-Tensor , der die Raumkrümmung in der allgemeinen Relativitätstheorie bestimmt, gehen Masse, Energie und andere Größen mit ein. Da 95% der Nukleonenmasse in der Bindungsenergie der Quarks steckt, aus der sie bestehen, muss diese Energie ja offenbar auch Gravitation verursachen.

    Kein Mensch weiß bisher, was Dunkle Materie genau ist, aber vermutlich besteht sie einfach aus bisher unentdeckten Teilchen, die nicht elektromagnetisch wechselwirken. Dass schwarze Löcher dunkel sind, rührt aber daher, dass ihre Schwerkraft kein Licht entweichen lässt, das hat nichts mit dem Dunkelsein der dunklen Materie zu tun.

    Was genau in der Singularität eines Schwarzen Lochs passiert, weiß man nicht, nicht einmal, ob es diese überhaupt gibt, vielleicht ist das ein Zustand extremer Energiedichte wie beim Urknall, in dem es keine unterscheidbaren Teilchen mehr gibt, wer weiß. Wenn Schwarze Löcher durch Hawking-Strahlung zerfallen, erzeugen sie auch nicht (nur?) dunkle Materie (wenn überhaupt), sondern alle möglichen Teilchen.

  314. #317 Lutz Komischke
    9. Mai 2015

    @Alderamin

    Wenn Du von Nukleonenmasse und Bindungsenergie
    sprichst ist ja wieder “normale Materie” im Spiel
    die in Schwarzen Löchern ja nicht mehr
    vorhanden zu sein scheint und nicht “reine Energie” .
    (Wozu sucht man sonst nach Gravitronen und Higgs Teilchen um die
    Gravitation und die Masse zu erklären ?).

    Ausserdem meine ich in den “Missverständnissen zu
    Schwarzen Löchern” gelernt zu haben, das es dort keine Singularität gibt.
    (Das ist eine rein mathematische Betrachtungsweise und hat mit
    der physikalischen Realität nichts zu tun).

    Ich hoffe Du verstehst was ich meine.

    Ich denke einfach das die “Dunkle Materie” einfach der Zwischenschritt
    zwischen “Energie” und der “normalen Materie” die mit der elektromagnetischen
    Strahlung interagiert ist, und in Schwarzen Löchern wieder zu diesem Zustand
    “zerfällt” .

  315. #318 B. Lothar
    Ludwigshafen am Rhein
    9. Mai 2015

    Hallo
    Meine Herren “Aldemarin” und Florian Freistetter!
    Das Very Large Teleskop in Chile besteht aus vier 8,2m Spiegeln, welche zusammengeschaltet werden können.
    Das VLT ist definitiv kein Radioteleskop. Zu lesen eigentlich überall, auch in der Wikipedia. Ich verwechsle hier mal gar nichts, wie mir unterstellt wird.
    Meine Fragen vom 7. Mai sind nach wie vor unbeantwortet.
    Gruß
    Lothar

  316. #319 Alderamin
    9. Mai 2015

    @B. Lothar

    Ich habe nicht Dir, sondern Florian unterstellt, das VLT mit dem VLA verwechselt zu haben, was er dann bestätigt hat.

    Die optische Interferometrie ist äußerst komplex und benötigt viel Rechenaufwand, es kommt dabei kein Bild heraus, sondern ein Muster aus Interferenzstreifen, aus dem man erst ein Bild synthetisieren muss. Das Blickfeld dabei ist winzig klein, flächige Objekte wie Nebel oder Galaxien kann man nicht abbilden. Meist reichen den Beobachtern die Streifen aus, um z.B. den Durchmesser eines Sterns daran zu bestimmen (wenn die Streifen verschwinden, ist das Objekt aufgelöst). Das Keck-Interferometer wurde daher nie fertig gestellt und eingestampft, und die Beobachtungen mit dem VLTI kann man fast schon an den Fingern abzählen.

    Ein Großteleskop wie das EELT oder TMT werden auf einen Schlag riesige Blickfelder als Klarbilder liefern, ohne Extraaufwand. interferometrie wird diese nie ersetzen können.

  317. @B.Lothar: “Ich verwechsle hier mal gar nichts, wie mir unterstellt wird.”

    Niemand unterstellt etwas. ICH habe das VLT mit dem Radioteleskop VLA verwechselt und darauf hat sich Alderamin bezogen.

    “Meine Fragen vom 7. Mai sind nach wie vor unbeantwortet.”

    Also meine Antwort, dass es nicht so einfach ist, optische Teleskope zwecks Interferometrie weltweit zu synchronisieren, reicht nicht?

  318. #321 bikerdet
    9. Mai 2015

    In der letzten Zeit sieht man immer öfter Tabellen um Sonnenfinsternisse auf den Jupitermonden beobachten / fotografieren zu können. Also keine Bedeckungen, sondern Sonnenfinsternisse durch andere Jupitermonde. So wird z.B. am 16.5. um 00:38 Uhr der Mond IO für 4 Min. den Mond Europa verfinstern.

    Meine Frage ist jetzt : Ist es damit vorbei wenn Jupiter seine Achsstellung ändert und wir nicht mehr genau auf die Äquatorebene Jupiters schauen ? Es kommt doch eigendlich nur darauf an, ob die Monde in der Ebene bleiben. Oder übersehe ich da was ?

  319. #322 B. Lothar
    Ludwigshafen am Rhein
    9. Mai 2015

    Hallo,

    die Mißverständnisse sind beseitigt und die neue Antwort von “Aldemarin” hilft mir weiter.
    Vielen Dank
    Lothar

  320. #323 Alderamin
    9. Mai 2015

    @bikerdet

    Ja, damit ist es jetzt vorbei, Jupiter bewegt sich weiter und die Knotenlinie der Jupitermondbahnen zeigt nicht mehr in Richtung Erde/Sonne. Wechselseitige Ereignisse der Monde gibt es erst wieder in 6 Jahren, wenn die “andere Seite” der Knotenlinie in unsere Richtung zeigt. Ist genau wie bei den Saturnringen, die sich alle 15 Jahre schließen und dann “verschwinden”, nur ist die Achsneigung von Jupiter und seinen Monden nicht so groß wie beim Saturn.

  321. #324 bikerdet
    11. Mai 2015

    @ Alderamin :
    Vielen Dank, an die auf- und absteigenden Knoten habe ich nicht gedacht ….

  322. #325 Cakir
    13. Mai 2015

    Ich habe einige Fragen bezüglich die Rotations Zeit der Planeten.
    Der Mars braucht ja für die Rotation 24 h 37 min, also knapp 40 Minuten mehr als die Erde.
    Wenn wir aber die anderen Planten im Sonnensystem beachten, brauchen z.B Gasriesen weniger Zeit für die Rotation.
    Jupiter und Saturn haben ja auch fast ne identische Rotationsperiode. Jupiter braucht 9 h 55 min und Saturn braucht 10 h 33 min, also braucht der Saturn auch knapp 40 Minuten mehr als Jupiter.
    Gasriesen Rotieren also schneller, aber wieso? Welche Faktoren spielen da eine Rolle? Masse? Dichte? usw.
    Wieso braucht aber die Venus so verdammt lange für ne Rotation? (243 Tage 27 min)
    Der Merkur z.B braucht 58 d 15 h 36 min, also desto kleiner der Planet, desto länger die Rotation? Ergibt aber auch irgendwie kein Sinn, da der Pluto deutlich kleiner ist und deutlich weniger Zeit braucht, als der Merkur. (6 d 9 h 17 min)

  323. #326 Torq Serpilian
    Hannover
    15. Mai 2015

    Per Swing-by zu Proxima Centauri?

    Wäre es zumindest theoretisch möglich, eine Raumsonde mit einer von Vielzahl von Swing-Bys so zu beschleunigen, dass sie innerhalb einer halbwegs sinnvollen Zeit Proxima Centauri erreichen kann? Wäre die denkbare Geschwindkeit, die eine Raumsonde durch Swing-Bys erreichen kann, durch die Gegebenheiten der Himmelskörper in unserem Sonnensystem begrenzt? Und wenn ja, wie hoch wäre diese Geschwindigkeit?

    Ich würde mich auch sehr über eine Sternengeschichte zum Thema Gravity-Assist/Swing-By freuen!

    um innerhalb einer halbwebs sinnvollen Zeit

  324. #327 Spritkopf
    15. Mai 2015

    @Torq Serpilian

    Wäre es zumindest theoretisch möglich, eine Raumsonde mit einer von Vielzahl von Swing-Bys so zu beschleunigen, dass sie innerhalb einer halbwegs sinnvollen Zeit Proxima Centauri erreichen kann?

    Die Endgeschwindigkeit, die du erreichen kannst, ist tatsächlich auch dann begrenzt, wenn du mehrere Swing-Bys hintereinander nutzt.

    Bei jedem Swing-By-Manöver kann deine Raumsonde nicht mehr als um das Doppelte der Bahngeschwindigkeit des Planeten schneller werden, den sie für das Manöver benutzt. Und spätestens dann, wenn du die Fluchtgeschwindigkeit für das Sonnensystem überschritten hast, geht es unweigerlich nach draußen. Das heißt, du könntest dann höchstens noch einmalig die Planeten zum Swing-by nutzen, deren Bahnen sich außerhalb deiner gegenwärtigen Position zur Sonne befinden (und das auch nur, wenn sie halbwegs auf deiner Flugbahn liegen).

    Hier ist eine Grafik, die die Swing-by-Manöver von Voyager 2 an Jupiter, Saturn, Uranus und Neptun zeigt. Manöver, die aber nur deswegen geflogen werden konnten, weil zu dem Zeitpunkt die Planetenkonstellation äußerst günstig dafür war:

    https://upload.wikimedia.org/wikipedia/commons/2/2c/Voyager_2_velocity_vs_distance_from_sun.svg

    Das nächste Mal gibt es eine Konstellation, in der diese vier Swing-bys möglich sind, erst wieder im 22. Jahrhundert.

  325. #328 Krypto
    15. Mai 2015

    @Cakir: Soweit ich weiß, kann man da keine generellen Regeln aufstellen, wenn man mal von gebundenen Rotationen/Resonanzen und Obergrenzen absieht. So kann z.B. ein ordentlicher Impakt in der Vergangenheit eine größere Rolle spielen als die ursprüngliche Rotation bei Planetenbildung.

  326. #329 Cakir
    15. Mai 2015

    @Krypto

    Nope, es gibt generelle Regeln, wie ich grad nun gefunden habe.

    “Dennoch rotiert zum Beispiel Jupiter aufgrund seiner geringen mittleren Dichte viel schneller als zum Beispiel die Erde.”

    Ich war irgendwie sicher, dass die Dichte eine Rolle spielt. 😉

    Quelle: https://www.sterngucker.de/artikel/allgemeines/jupiter-der-gasriese-im-sonnensystem/

    Bezüglich Venus, hast kannst du aber recht haben.
    Habe folgendes gefunden:

    “man vermutet aber, dass es etwa mit einem gewaltigen Ereignis in der Frühgeschichte der Venus zu tun hat. Ursprünglich wird der Planet ein ganz “normale” erdähnliche Rotation gezeigt haben. Eine Kollision mit einem großen Objekt könnte das dann aber geändert haben. Solche Kollisionen mit Überbleibseln aus der Entstehungsphase des Sonnensystems waren in der Frühphase unseres Sonnensystems noch deutlich häufiger.”

    Quelle: https://www.astronews.com/frag/antworten/1/frage1137.html

  327. #330 Spritkopf
    15. Mai 2015

    @Myself

    Hier ist eine Grafik, die die Swing-by-Manöver von Voyager 2 an Jupiter, Saturn, Uranus und Neptun zeigt. Manöver, die aber nur deswegen geflogen werden konnten, weil zu dem Zeitpunkt die Planetenkonstellation äußerst günstig dafür war:

    Wobei ich bei nochmaligem Blick auf die Grafik feststelle, dass der letzte Swing-By kein beschleunigendes, sondern ein abbremsendes Manöver für Voyager 2 war, weil es nicht um den Swing-By an sich ging, sondern darum, Neptun so nahe zu kommen, um ihn besser studieren und Bilder von ihm machen zu können.

  328. #331 Torq Serpilian
    Hannover
    15. Mai 2015

    @Spritkopf:

    Herzlichen Dank für Dein Feedback! Und ich bitte für meine “unordentliche” Eingangsfrage um Entschuldigung:(

  329. #332 Jens
    16. Mai 2015

    Es wird vermutet, dass jenseits des Kuipergürtels noch ein etwa erdgrosser Planet um die Sonne kreist. Wie kommt man darauf und könnte man ihn in absehbarer Zeit entdecken? Z.B. New Horizon?

  330. #333 Florian Freistetter
    16. Mai 2015

    @Jens: Such mal in meinem Blog nach “PLanet X” – da findest du jede Menge Artikel zu diesem Thema. Hier ist ein Einstieg: https://scienceblogs.de/astrodicticum-simplex/2009/10/11/nicht-jeder-planet-x-ist-gefahrlich/

  331. #334 bikerdet
    16. Mai 2015

    @ Cakir in #329 :
    Mich wundert es etwas, das Dir niemand widersprochen hat.
    Aber unser Erde dreht sich nicht wegen der größeren Dichte so langsam, sondern weil sie vom Mond abgebremst wird. Im Gegenzug entfernt sich der Mond von der Erde. Aktuell bremst der Mond die Erde um 1sek / 100.000 Jahren, er entfernt sich dabei um 4 cm / Jahr. In seiner Anfangszeit war der Mond der Erde viel näher, die Gezeitenkräfte deshalb sehr viel stärker und ebenso die Abbremsung. Vor etwa 100 Mio. Jahren betrug die Jahreslänge noch 400 Tage. Die Erde drehte sich also in weniger als 22 Std. um sich selbst. Als der Mond entstand, drehte sich die Erde sogar in nur 8 Std. um sich selbst. Und damit schneller als es aktuell der Jupiter tut. Da Jupiter immer kleiner wird, die Schwerkraft komprimiert die Gase, könnte er sich früher langsamer gedreht haben.
    Leider habe ich dazu keine Info’s finden können, evtl. weis aber jemand anders hier mehr….

  332. #335 Alderamin
    16. Mai 2015

    @bikerdet

    Doch, es gibt offenbar einen Zusammenhang, allerdings zwischen der Masse und der Rotationsrate:

    https://www.skyandtelescope.com/astronomy-news/exoplanets/first-exoplanet-quick-spin/

    Dass die Venus durch einen Impact so langsam rotiert, habe ich auch mal gelesen (Merkur möglicherweise auch).

  333. #336 Cakir
    16. Mai 2015

    @bikerdet

    Das der Mond die Erde ausbremst, war mir schon klar. 😉
    Mich würde auch interessieren, ob die zahlreichen Monde von Saturn, Jupiter etc. den Planeten auch ebenfalls ausbremsen?
    Würde z.B der Saturn ohne die ganzen Monde schneller rotieren? Oder sind die Monde im Vergleich einfach zu klein? Der Saturn ist ja riesig, daneben sind die Monde praktisch Staubkörner. 🙂

  334. #337 Krypto
    17. Mai 2015

    @Cakir: Du kennst doch bestimmt, wie Eiskunstläufer Pirouetten machen? So verhalten sich auch Planeten. Stichwort wäre Impulserhaltung. Dementsprechend wird durch Radiusverkleinerung die Rotation beschleunigt. Bei Gasriesen gibt es noch Bremsvorgänge durch Reibungs- und Zentrifugaleffekte, welche bei erdähnlichen Planeten nicht so stark ausfallen. Irgendwo habe ich einmal aufgeschnappt, dass die Übergangsschichten oberhalb des metallischen Wasserstoffs auch eine Rolle im Rotationsverhalten spielen sollen. Was die Monde betrifft, kann ich mir nicht vorstellen, dass sie einen unserer Gasriesen nennenswert abbremsen könnten. Im Wesentlichen behalten die Planeten jedoch den bei ihrer Entstehung bzw. schweren Impakten erhaltenen Drehimpuls. Wie Alderamin geschrieben hat, könnten wir bald grobe Regeln ermitteln, was einen Zusammenhang zwischen Masse und Tageslänge betrifft, aber dazu reicht die momentan vorhandene Datenbasis noch nicht aus.

  335. #338 bikerdet
    17. Mai 2015

    Der Zusammenhang müßte Masse vs Durchmesser sein, also die von Cakir angeführte Dichte. Durch den Massezuwachs erhöht sich auch der ‘eingefangene’ Drehimpuls. Somit müssten Körper, die viel Masse auf kleinem Volumen haben auch schneller rotieren. Leider ist es mir nicht möglich auszurechnen, ab welchem Verhältnis die Größe den Drehimpuls dominiert. So dreht sich die Sonne, trotz größerer Masse bei ähnlicher Dichte wie Jupiter, am Äquator in etwa 25 Tagen um sich selbst. Sie müßte sich aber nach der in #329 angegebenen Definition viel schneller drehen. Hier spielt also der Durchmesser eine Rolle. So gibt es ja Neutronensterne, die im Millisekundentakt rotieren. Allerdings ist das Verhältniss Masse/Durchmesser dort sehr extrem. Ich habe von SL gelesen, die mit fast Lichtgeschwindigkeit rotieren. Umgekehrt ist bekannt, das die Wolkenbänke, die im Winterhalbjahr die Nordhalbkugel der Erde bedecken, ebenfalls eine Verlangsamung (im gerade noch messbaren Bereich..) erzeugen.

    Ob die Monde der Gasplaneten messbaren Einfluss auf diese haben ist mWn unbekannt. Wir sehen ja nur die äußerste Atmosphäre. Selbst wenn die Kerne der Gasplaneten verlangsamt würden, messen können wir es nicht.

    Evtl. üben auch die Planeten einen Gezeiteneffekt auf die Sonne aus. Die Erde entfernt sich mit etwa 15cm/Jahr von der Sonne. Als einziger Grund ist noch die Übernahme des Drehimpulses von der Sonne im Rennen. Die Abbremsung der Eigendrehung der Sonne läge dabei rechnerisch nur im Millisekundenbereich je Jahrhundert und ist für uns nicht messbar.

    btw : Venus soll in ihrer Frühphase zweimal von Protoplaneten getrofen worden sein. Der erste Impakt bremste sie ab, der zweite soll die Achsdrehung verursacht haben. Der zweite Impakt soll auch den beim ersten Einschlag entstandenen Mond zerstört / aus der Bahn geworfen haben. Merkur hingegen konnte durch seine Sonnennähe keinen Mond bilden. Seine beim Impakt ausgeschleuderte Masse wurde von der Sonne angezogen.

  336. #339 A-P-O
    19. Mai 2015

    Stichwort Eismonde.
    Könnte man die kilometerdicken Eispanzer von Enceladus und Europa entfernen, würde dann das darunter liegende Wasser schlagartig in den Raum vaporisieren und von den Monden buchstäblich nichts mehr übrig bleiben (abgesehen vom Gesteinskern)?
    Wenn Ja, wie konnten dann solche Eis-und-Wasser-Monde überhaupt entstehen?

  337. #340 Captain E.
    19. Mai 2015

    @A-P-O:

    Nur zum Teil, würde ich annehmen. Der Rest würde gefrieren. Kalt genug sollte es ja sein.

  338. #341 Crazee
    19. Mai 2015

    Das ist eine schöne Frage für What-if von XKCD. Darf ich die da einreichen?

  339. #342 Alderamin
    19. Mai 2015

    @A-P-O

    Der Tripelpunkt des Wassers liegt bei 0°C (das ist ja gerade der Nullpunkt der Skala). Bei geringeren Temperaturen gibt’s nur noch Eis oder Dampf. Da die Eismonde außerhalb der Schneegrenze im Sonnensystem liegen, würde freigelegtes flüssiges Wasser durch Kochen schnell oberflächlich abkühlen (Verdunstungskälte) und zufrieren und danach nur noch langsam zu Wasserdampf (besser: -gas) sublimieren. Unter dem Eis bliebe das (dichtere, schwerere) Wasser unter Druck und könnte flüssig bleiben, solange es eine innere Wärmequelle gibt, die es über dem Schmelzpunkt hält (radioaktiver Zerfall, Gezeitenreibung).

  340. #343 A-P-O
    19. Mai 2015

    @Alderamin: soweit verständich. Die Monde würden also “nur” kleiner werden. Wie steht es aber mit der ursprünglichen Entstehung? Lagerten sich hier Eiscluster an einem Gesteinskern, der ja auch erst entstanden sein musste (Planetesimal)? Was sollte das Wasser dazu bringen? Viel wahrscheinlich wäre da schon, dass das Wasser sich als Ringe um den Mutterplanten anordnet. Die Kometen-Kompakt-Theorie dürfte hier wohl nicht greifen, da ja kaum ein genügend groß Kern vorhanden war, auf den ein Einschlag stattfinden hätte können?

    @Crazee: Ich kenne Randall Munroes Blog und What-If-Buch? Nimmt er noch Fragen an? Dann mach ich das gleich mal selber. Danke für den Tipp?

  341. #344 Alderamin
    19. Mai 2015

    @A-P-O

    Nein, Eis und Staub waren gut gemischt, und Eis haftet gut an einander, so werden sich die befrorenen Staubteilchen aneinandergeheftet haben und zu größeren Strukturen gewachsen sein. Ein Gesteinskern kann sich erst ausbilden, wenn das Wasser im Inneren eines größeren Objekts aufschmilzt (Differentiation; das Gestein enthielt ja auch radioaktive Bestandteile) und das schwere Material nach unten sinkt. Im Prinzip passiert das gleiche bei noch größeren Objekten mit dem Gestein und Eisen.

    Falls durch den Einfall von größeren Objekten und die damit einhergehende Erhitzung ein Teil des Wasser verloren ging, dann konnte danach immer noch wasserreicher Staub dieses ersetzen (und das verlorene Waser fror ja auch in der Umlaufbahn wieder, also könnte das Objekt das Eis teilweise wieder aufgesammelt haben).

    Es ist einfach sehr viel Wasser in der protoplanetaren Scheibe gewesen (jenseits der Schneegrenze), irgendwo muss es am Ende geblieben sein. Nur auf den inneren Planeten, in deren Umlaufbahn es zu warm für Wasser war, musste es duch Asteroiden und Kometen nachgeliefert werden.

  342. […] Diese Frage wurde mir in letzter Zeit häufig gestellt und ich möchte sie daher in meiner Serie “Fragen zur Astronomie” […]

  343. #346 willi1
    22. Mai 2015

    hier finde ich euch also wieder!

    habt ihr die frage nach urknall zurück gestellt auf später? in dem blog ist ja tote hose. ziemlich abrupt.
    ich habe allerdings auch keine neuen erkenntnisse.

  344. […] wird es in der Serie “Fragen zur Astronomie” wieder einmal ganz grundlegend. Jeder kennt Sternbilder und von Sternzeichen hat sicher auch jeder […]

  345. #348 Janico
    25. Mai 2015

    Könnte man mit einem Lichtsegel oder Sonnensegel auch wie mit einem normalen Segel aufkreuzen, um so im Zickzackkurs näher zur Sonne zu fliegen? Bei Segeln im Wind funktioniert das über den Tragflächeneffekt, gibt es diesen Effekt auch mit elektromagnetischer Strahlung?

  346. #349 Simon Deutschl
    26. Mai 2015

    Am Ende des Lebens einer entsprechend großen Sonne wird diese zum schwarzen Loch. Sobald das schwarze Loch da ist, “frisst” dieses gnadenlos alles an Materie, die den Ereignishorizont überschreitet.

    Gibt es eigentlich einen Zustand, in dem das schwarze Loch wieder zu etwas anderem werden könnte? Oder wächst es einfach immer weiter, je mehr Materie es verschlingt? Und könnte so ein schwarzes Loch theoretisch groß genug werden, um das Universum zu vernichten (vorausgesetzt, es gibt genügend Materie im Universum, um das schwarze Loch so groß werden zu lassen)? Und was passiert eigentlich, wenn zwei schwarze Löcher aufeinander treffen?

    Ich weiß, das ist jetzt mehr als eine Frage, aber diese Reihe an Fragen schwirrt schon länger in meinem Kopf herum.

  347. #350 Jens
    26. Mai 2015

    In welcher Entfernung kann man die Erde mit heutiger Beobachtungstechnik grad noch nachweisen?

  348. #351 T
    26. Mai 2015
  349. […] der Serie “Fragen zur Astronomie” geht es heute zur Abwechslung mal um die Planeten. In unserem Sonnensystem unterscheiden wir da ja […]

  350. #353 Karsten
    1. Juni 2015

    Gibt es eigentlich irgendetwas neues bezüglich der Nemesis-Theorie? Das wir also in einem Doppelsternsystem leben, dessen 2. Komponente ein sehr dunkler roter Zwergstern (eben besagter Nemesis) sein soll, der etwa alle 26 Mio Jahre der Sonne und damit der Erde so nahekommt, dass er Einfluss auf das innere Sonnensystem nehmen kann? Indem er zum Beispiel die Bahnen der Kometen in der Oortschen Wolke stört? Und damit zusammenhängend: kann man inzwischen (ohne Nemesis zu Hilfe nehmen zu müssen) die Umlaufbahn des Zwergplaneten Sedna befriedigend erklären? Der ja wohl in einem Orbit um die Sonne kreist, in dem man normalerweise keinen Himmelskörper vermuten würde?

    Und was ist mit dem vermuteteten Gasriesen, der ebenfalls irgendwo zwischen Kuiper-Gürtel und Oortscher Wolke “herumgeistern” soll – weiß man da inzwischen Genaueres?

  351. #354 Alderamin
    1. Juni 2015

    @Jens

    In welcher Entfernung kann man die Erde mit heutiger Beobachtungstechnik grad noch nachweisen?

    Das Designziel des Weltraumteleskops Kepler (das ich mal als das weitreichendste Gerät zur Entdeckung erdähnlicher Planeten deklariere) war, bei einem Stern 12. Größe Helligkeitsänderungen von 20 Teilen pro Million zu entdecken; ein erdähnlicher Planet würde etwa 80 Teile pro Million an Verdunklung (vor einem sonnengroßen Stern) verursachen. Andere Sterne erwiesen sich als etwas variabler als die Sonne, aber mal ausgehend davon würde man die Erde noch im Transit vor der Sonne aus einer Entfernung entdecken, aus der sie 12. Größe hätte. Dazu wären (nach Designziel) mindestens drei Transits, also mindestens etwas mehr als zwei Jahre Beobachtung, nötig.

    Die absolute Helligkeit der Sonne beträgt 4,83 Größenklassen, das ist die Helligkeit bezogen auf 10 pc (32,6 Lichtjahre) Entfernung. 12. Größe ist 7,17 Größenklassen oder den Faktor 10^(7,17*0,4) = 738 mal dunkler als 4,83 Größenklassen. Da die Helligkeit mit dem Quadrat der Entfernung abnimmt, hat man diesen Faktor bei Wurzel aus 738 = 27,16-facher Entfernung, das sind rund 270 pc oder knapp 900 Lichtjahre.

    So weit entfernt könnte Kepler die Erde vor der Sonne gerade noch nachweisen, vorausgesetzt, sie würde aus dieser Richtung auch Transits vor der Sonne durchführen.

    Falls nicht, käme noch die Dopplermethode in Frage, mit der gemessen wird, wie stark die Gravitation des Planeten seinen Mutterstern beim Umlauf zum Wackeln bringt. Diese Methode ist derzeit in der Lage, Geschwindigkeitsänderungen von ca 1 m/s nachzuweisen. Die Erde veranlasst mit 1/333000 der Sonnenmasse und 150 Millionen km mittlerem Abstand zur Sonne diese dazu, pro Jahr einen (Beinahe-)Kreis von 450 km um das gemeinsame Baryzentrum zu drehen. Damit komme ich auf eine maximale Geschwindigkeit von 0,09 m/s, ein Faktor 10 zu wenig. Daher kann der verlinkte HARPS (Stand der Technik) bisher nur Supererden (ca. 10 Sonnemassen) um kleine Sterne nachweisen, die Erde bliebe unentdeckt.

  352. #355 bruno
    Könnten (Terroristen) Asteroiden als Waffe missbrauchen?
    3. Juni 2015

    YEAH. Geile Frage!
    …die Idee kam mir gerade erst…

    Was wäre wenn…
    Terroristen … Raketen… entwickelten/ einsetzten … um… dann… spontan … einen Asteroiden von seiner Bahn (zB Apophis) auf die Erde zu lenken … um.. die (kapitalistische/ korrupte/ ungläubige) Menschheit alternativ zu erpressen oder auszulöschen…

    worst case scenario…. die “jungs” haben irgendwas (massereiches), um Apophis die letzten nullkommagrad … in Richtung Erde umzuleiten… wie realistisch wäre eine Konterraktion und wie lange würde das im besten Falle dauern … den “Counter-strike” loszuschicken???
    (anbetracht dessen was “die jungs” realistischerweise ins All schicken könnten?)

  353. #356 Krypto
    3. Juni 2015

    @Bruno:
    Wer solche Raketen ungestört bauen kann, braucht keine Asteroiden, um ultimative Zerstörungen anzurichten.

  354. #357 Jens
    3. Juni 2015

    @Alderamin: Danke für diese ausführliche Erläuterung.

  355. #358 Jens
    4. Juni 2015

    Warum sind die Einschlagkrater auf der Vorderseite des Mondes größer als auf der Rückseite?

  356. #359 Florian Freistetter
    4. Juni 2015

    @Jens: Auf der Rückseite des Mondes findest du den größten Krater des ganzen Sonnensystems!

  357. #360 bikerdet
    4. Juni 2015

    @ Jens :
    Das liegt an der Dicke der Mondkruste. Auf der Rückseite des Mondes ist sie bis zu 150 Km dick, auf der Vorderseite im Mittel nur 1/2 so dick. Teilweise ist die Kruste dort aber nur wenige Km dick. Einschlagende Meteoriten durchschlugen die Kruste und Lava lief hinein -> Bildung der Mare. Die wärmere und dünnere Kruste führte auch dazu, das das Material beim Einschlag vorrangig nach außen geschleudert wurde, auf der dickeren und härteren Kruste der Rückseite aber vorrangig in den Krater zurück fiel. Rechnet man diese rein optische Veränderung raus, sind die Krater auf Vorder- und Rückseite statistisch gleich groß.

  358. #361 Kyllyeti
    4. Juni 2015

    @ Jens

    … und warum die Kruste auf der Rückseite des Mondes so deutlich dicker ist, wurde übrigens hier erklärt.

  359. #362 Jens
    5. Juni 2015

    @Kyllyeti
    leider funktioniert dein Link nicht. Wäre aber schon ineressant zu wissen warum die Kruste auf der Rückseite des Mondes deutlich dicker ist.

  360. #363 Kyllyeti
    5. Juni 2015

    @Jens

    Oh, tut mir leid – also noch mal:

    die Beschreibung der derzeit plausibelsten Theorie findest du hier.

  361. #364 Jens
    7. Juni 2015

    Der Mond zeigt der Erde immer die gleiche Seite weil er eine gebundene Rotation um sie ausführt. Welche Monde in unserem Sonnensystem tun das auch?

  362. #365 Alderamin
    7. Juni 2015

    @Jens

    Charon, Phobos, Deimos… es gibt ein paar Hundert Monde im Sonnensystem, niemand wird im Kopf haben, welche davon alle gebunden rotieren (sofern Rotationraten überhaupt bekannt sind). Es ist eine Fleißarbeit, diese alle im Web zu suchen, das kannst Du z.B. auch selbst versuchen. In Wikipedia steht ja eigentlich alles drin, was man über die Körper des Sonnensystems weiß.

  363. #366 AmbiValent
    7. Juni 2015

    @Jens

    Florian hat auch gerade etwas über die Plutomonde Nix und Hydra geschrieben, die eben nicht gebunden rotieren. Damit sind sie die große Ausnahme – mir fällt sonst nur der Saturnmond Hyperion ein.

  364. #367 Alderamin
    7. Juni 2015

    @Ambivalent

    Phoebe (Saturn) und Nereide (Neptrun) rotieren z.B. auch nicht gebunden, und viele der kleinen Monde mit großen Bahnneigungen und großen Entfernungen von ihren Mutterplaneten vielleicht auch nicht (aber die Rotationsperioden sind da meist unbekannt).

    Die großen Jupiter-, Saturn- und Uranusmonde schon, und (für mich) überraschenderweise auch Triton, obwohl er den Neptun retrograd umläuft.

  365. […] denn normalerweise verhalten sich Monde ganz anders. Es lohnt sich also, in der Serie “Fragen zur Astronomie” einmal genauer darüber nachzudenken. Also: Wie rotieren die Monde in unserem […]

  366. #369 f.sonnleitner
    wien
    11. Juni 2015

    Tritt im Zentrum einer Spiralgalaxie Materie ein oder aus? Funktioniert sie also wie ein Feuerwerksrad oder wie ein Badewannenauslauf??

  367. #370 klauszwingenberger
    11. Juni 2015

    @ f.sonnleiter:

    “Wie ein…” ist ein gefährlicher Ansatz, weil kosmische Vorgänge mit Alltagsphänomenen nur sehr schwer vergleichbar sind. Auch wenn sich die Naturgesetze nicht unterscheiden: man betrachtet ganz andere Skalen.

    Die in den Zentren von Spiralgalaxien befindlichen extremen Masseansammlungen bis zum supermassiven SL akkretieren in der Regel etwas Materie. Die Mengen sind aber gemessen an der Gesamtmasse der Galaxie nicht der Rede wert.

    Das, was in angesammelt wird, landet wegen des Grundsatzes der Drehimpulserhaltung erst einmal in einer rotierenden Scheibe. Durch die Abbremsung in der Scheibe geht ein Teil des Impulses verloren (und wird als elektromagnetische Strahlung abgegeben). Das, was vom Innenrand der Scheibe auf ein SL heruntergezogen wird, verschwindet auch nicht sämtlich hinter dem Horizont, sondern kann zu einem beträchtlichen Teil durch die enorme erfahrene Beschleunigung die Oberhand über die Schwerkraft behalten und verlässt den Galaxienkern, hauptsächlich entlang der Rotationsachse, mit relativistischen Geschwindigkeiten. So entstehen die “Jets” aktiver Galaxienkerne.

    Es ist, wenn Du so willst, eine Kombination aus Feuerwerksrad und Badewannenauslauf unter besonderer Berücksichtigung der Allgemeinen Relativitätstheorie – und damit unter extremen Bedingungen.

    Aber um das ganze in die richtigen Verhältnisse zu rücken: das betrifft nur ganz kleine Teile der Masse einer Spiralgalaxie. Der allergrößte Teil dreht im gravitativen Gleichgewicht bedächtig seine Runden.

  368. #371 Alderamin
    11. Juni 2015

    @klauszwingenberger

    Ich glaube, es ging f.sonnleitner eher um die Spiralarme, nicht das zentrale Schwarze Loch.

    @f.sonnleitner

    Tatsächlich sind zwischen den Spiralarmen auch Sterne und zwar nicht weniger als in den Spiralarmen. Nur sind in den Spiralarmen die Sternentstehungsgebiete, und die hellsten Sterne, die dort ebenfalls entstehen, leben nicht sehr lange (ein paar Millionen Jahre; Sterne wie die Sonne leben tausendmal länger, noch kleinere sogar bis zu millionenmal), man sieht sie nur dort, wo gerade Sterne entstehen. Da die hellsten Sterne so leuchtstark sind, markieren sie deutlich die Orte der Sternentstehung, die Spiralarme.

    Die Sternentstehungszone in einem Spiralnebel bewegt sich nun langsam um die Spiralgalaxie herum, warum ist noch nicht so genau verstanden. In einer Sternentstehungszone muss das (überall in der Milchstraße vorhandene) Gas jedenfalls ein wenig komprimiert werden, damit es unter seiner eigenen Schwerkraft zusammenfällt und Sterne bildet. Das könnte z.B. durch Stoßwellen von Supernova-Explosionen geschehen, die von den hellen, kurzlebigen Sternen am Ende ihres Lebens erzeugt werden, und deren ausgestoßenes Material das umliegende Gas komprimiert. So könnte eine Kette von Supernovae das Gas vor sich weiter verdichten, dort Sternentstehung auslösen, die wieder zu Supernovae führt, welche wiederum das voranliegende Gas komprimieren. Weil der Umlauf der Stoßwelle außen einen längeren Weg hat, dauert er dort länger, daher bleiben die Außenbereiche gegenüber den inneren Bereichen zurück – eine Spiralform entsteht.

    Eine andere Theorie, die Dichtewellentheorie, besagt, dass in den Spiralarmen aufgrund der gegenseitigen Anziehung der Sterne (und gewisser Resonanzen) die Dichte ein wenig höher ist (was den Kollaps des Gases mit auslöst) und die Sterne dort hindurch wandern wie Autos durch einen Stau. Die Verdichtungszone (Dichtewelle) umkreist die Galaxie dann mit einer eigenen Geschwindigkeit (so wie sich auch ein Stau typischerweise unabhängig von der Fortbewegung der Fahrzeuge, typischerweise sogar in Gegenrichtung der Fahrbahnrichtung, fortbewegt). Auch hier ergibt sich eine Spiralform, weil der Umlauf der Dichtewelle außen länger dauert.

    Wie gesagt, die wirkliche Ursache ist noch nicht geklärt, aber sicher ist, dass die Sterne die Galaxie umkreisen und nicht rasensprengerartig aus dem Zentrum heraus fliegen, und dass die Struktur der Spiralarme lediglich zeigt, wo sich junge, helle, heiße (und damit blau-weiß leuchtende) Sterne befinden (Spiralarme erscheinen auf Farbfotos deswegen immer blau-weiß).

  369. #372 klauszwingenberger
    11. Juni 2015

    @ Alderamin:

    So oder so, jetzt ist es jedenfalls komplett 😉

  370. #373 primus
    12. Juni 2015

    Gibt es Exoplaneten in Regionen mit erhöhter Sterndichte (z.B. Kugelsternhaufen wie M80)?

  371. […] sich durchaus lohnt. Und darum habe ich diese Frage auch für den heutigen Eintrag in meiner Serie “Fragen zur Astronomie” […]

  372. #375 Michael Jachan
    15. Juni 2015

    Hallo! Bei einer Typ 1a- Supernova sind 2 Sterne beteiligt: Der große gibt dem kleinen stetig Materie, bis der eine gewisse (einheitliche) Größe erreicht und zur Nova wird. Was passiert dann, bei der Nova, mit dem großen Stern?

    Thx:)

  373. #376 Thorsten
    16. Juni 2015

    Woher weiss man, dass es zig Milliarden Sterne gibt? Wie zählt man die?

  374. #377 Jens
    16. Juni 2015

    Schwarze Löcher scheint es jede Menge im Universum zu geben und bald werden wir sie sogar mit Teleskopen direkt beobachten können. Aber wie sieht es mit weißen Löchern aus? Kann es die auch geben und wie lassen sie sich beobachten?

  375. #378 Alderamin
    17. Juni 2015

    @Thorsten

    Woher weiss man, dass es zig Milliarden Sterne gibt? Wie zählt man die?

    Man zählt sie nicht, man wiegt sie… und misst ihre Helligkeit.

    Aus der Umlaufzeit der Sonne um die Milchstraße kann man mit Hilfe des Gravitationsgesetzes die Masse berechnen, die innerhalb der Sonnenbahn vorhanden ist. Sternmassen wiederum kennt man aus Doppelsternsystemen und von der Sonne, so kann man die Zahl der Sterne abschätzen, die dort vorhanden sein müssten.

    Man kann außerdem die Helligkeit von Sternhaufen und Galaxien messen. Wenn man weiß, wie diese sich auf Sterne verschiedener Massen verteilt, kann man abschätzen, wie viele Sterne da leuchten.

    Und stellt dann fest, dass in der Milchstraße 5mal so viel Masse steckt, wie an leuchtenden Sternen vorhanden ist. So kommt man auf die Dunkle Materie. Dazu hat Florian schon einige Artikel geschrieben.

  376. #379 Thorsten
    18. Juni 2015

    @ Alderamin
    Vielleicht erkennt Florian an deiner Antwort, dass die Frage auch für weitergehende Ausführungen geeignet ist – denn ehrlich gesagt hätte ich so zu jedem deiner Absätze gleich drei neue Fragen… (die ich aber hier nicht stelle, weil das ganze ja kein Diskussionsforum sein soll).
    Dir, Alderaim, aber trotzdem erstmal Danke für diese Ausführungen!

  377. #380 Alderamin
    18. Juni 2015

    @Thorsten

    Hier kann man über alles diskutieren. Aber das Thema wäre sicherlich auch einen Artikel wert, wenn Cheffe Zeit hat.

  378. #381 Hanne
    22. Juni 2015

    Hallo Florian, ich möchte Dir meinen Dank aussprechen. Dein Blog hier ist super und hat meinen Horizont und mein Verständnis für Astronomie schon vielfach erweitert. Danke noch mal. 🙂

  379. […] sehr beliebte Frage zur Astronomie” beschäftigt sich mit der Rotation der Planeten: Warum rotieren die Planeten um ihre Achse? Gerne […]

  380. #383 Rother Uhu
    Karlsruhe
    24. Juni 2015

    Mein naturwissenschaftlicher Ohrwurm!

    Mir gehen Fragen durch den Kopf, die so hartnäckig sind wie ein Ohrwurm oder schlimmer. Sie haben mit dem zu tun, was ich über die Planetenentstehung gelesen habe. Da gibt es eine Lücke in Staubkoagulation bei der Planetenentstehung wegen der Fragmentation beim Zusammenstoß der Staub- klumpen, der im Bereich vom 10 cm bis 10 m liegt und die durch die derzeitigen Modelle der Planetenentstehung nicht erklärt werden können.

    Nun gibt es eine Eis- oder Schneegrenze in Sternen- systemen!

    Gibt es zum Stern hin auch eine Silizumgrenze?

    Sublimiert festes Silizium oberhalb einer bestimmten Temperatur im interplanetaren Raum ebenso zu Dampf wie Wassereis? (Eher nicht. Silizium für die Chipfertigung wird zur Reinigung z.B. im Hochvakuum aufgeschmolzen.)

    Oder gibt es um einen Stern, wegen der relativ großen Differenz von 1850 C° zwischen Schmelzpunkt und Siedepunkt, eine Zone in der Silizium in halbfester bis flüssiger Form (wie Magma?) vorkommen kann?

    Warum ich das Frage!
    Ehem – Weil ich kein Fachmann für das Verhalten von Silizium im Hochvakuum bin, vielleicht?
    Weil Magmen in der Regel silikatische Gesteinsschmelzen sind.
    Weil Magma „klebrig“ ist.
    Und dadurch die oben erwähnte Lücke in Akkretionsrate überbrückt werden könnte.

    Ach ja, und ich durch keinerlei Ahnung diesbezüglich vorbelastet bin.
    (Selig sind die Ahnungslosen) 🙂

  381. #384 Higgs-Teilchen
    Im Standardmodell oben rechts
    26. Juni 2015

    Meine Frage hat nicht direkt was mit Astronomie zu tun, aber ich stelle sie trotzdem:
    Woher weiß man, welches Element welche Halbwertszeit hat? Thorium z.B. 14.050.000.000 Jahre. Woher weiß man das?

    Lg Higgs

  382. #385 PDP10
    26. Juni 2015

    @Higgs-Teilchen:

    Die Halbwertzeit kann man messen.

    Man misst dazu im Prinzip die Aktivität – aus der sich die “Zerfallsrate” ergibt, die im wesentlichen umgekehrt proportional zur Halbwertzeit ist.

    Kurze Halbwertzeiten kann man auch einfach durch “wiegen” bestimmen …

    Guck mal hier:

    https://de.wikipedia.org/wiki/Halbwertszeit#Messung_radioaktiver_Halbwertszeiten

  383. #386 Simon Deutschl
    1. Juli 2015

    Im Universum gib es viel Staub. Doch ab wann spricht man im Weltall eigentlich von Staub? Wie groß sind diese Staubteilchen und ab wann ist es nicht mehr Staub sondern ein Stein oder Felsbrocken oder doch etwas kleineres, unterhalb von Staub?

  384. #387 Stefan
    1. Juli 2015

    Frage:
    Ist Licht von Sternen polarisiert?

  385. #388 Alderamin
    1. Juli 2015

    @Simon Deutschl

    Die Teilchen sind typischerweise etwa so groß wie die Partikel des Zigarettenrauchs, im Schnitt 0,3 µm, siehe hier. Eine klare Grenze zu Mikrometeoroiden und Meteoroiden wüsste ich nicht; Mikrometeoroiden sind aber typischerweise so groß wie ein Sandkorn, 1/10 mm = 100 µm und Meteoroiden so groß wie Kieselsteine oder größer. Und ab 1 m kann man schon von Asteroiden sprechen, aber auch diese Grenze ist nirgends eindeutig definiert.

    @Stefan

    Nein, warum sollte es? Sterne sind Temperaturstrahler und die senden erst mal kein polarisiertes Licht aus. Dazu bedarf es einer Streuung im 90°-Winkel (das kann z.B. insterstellarer Staub leisten, etwa die Reflexionsnebel in den Plejaden) oder Durchgang durch magnetisch ausgerichteten Staub (schwach polarisierend).

  386. #389 Stefan
    1. Juli 2015

    @Alderamin
    Ich hatte gehofft, der Sternenrand würde derartiges leisten, ohne allerdings länger darüber nachgedacht zu haben.

    Die Frage hinter der Frage war: Zerfällt Polarisation?
    Wir kennen unpolarisiertes Licht und können dies polarisieren.
    Die Frage war nun: Zerfällt diese Polarisation wieder?
    – nicht in der Theorie, aber gibt es Messungen?

    Sie sagen, es gibt polarisierenden Staub. Wie weit ist dieser etwa von uns entfernt?

  387. #390 Alderamin
    2. Juli 2015

    @Stefan

    Ich hatte gehofft, der Sternenrand würde derartiges leisten

    Nö, jeder einzelene Punkt auf der Oberfläche des Sterns strahlt unpolarisiertes Licht in alle Richtungen aus.

    Die Frage hinter der Frage war: Zerfällt Polarisation?

    Nein. Habe ich noch nie von gehört. Es braucht stets eine Art von “Filter”, der eine bestimmte Schwingungsrichtung bevorzugt durchlässt (oder ein “Verzögerungsglied” für eine Richtung, dann gibt es zirkulare Polarisation). Eine streifende Reflexion an einer glatten Fläche (z.B. Wasseroberfläche) polarisiert das Licht z.B., weil die Elektronen in der Ebene der Fläche der durch die Lichtwelle angeregten Schwingung leichter folgen können, als senkrecht dazu, in den reflektierenden Stoff hinein. Mit einer Polfilter-Brille kann man deshalb besser ins Wasser oder auch durch Glasflächen schauen.

    Sie sagen, es gibt polarisierenden Staub. Wie weit ist dieser etwa von uns entfernt?

    In den Plejaden gibt es Reflexionsnebel, die das Licht zur Seite streuen, da passiert dasselbe wie bei der Streuung von Sonnenlicht in der Erdatmosphäre, die den Himmel blau färbt (weil blaues Licht stärker gestreut wird als längere Wellelängen). Die Plejaden sind ca. 400 Lichtjahre entfernt.

    Ansonsten durchziehen Gas und Staub die gesamte Milchstraße, die selbst ein Magnetfeld hat. Da der Staub auch Eisen enthält und die Staubpartikel meist nicht rund, sondern länglich sind, können sie sich wie kleine Kompassnadeln ausrichten und eine Art Polfilter-Gitter bilden, das durchlaufendes Licht schwach polarisiert. Da kann ich keine Entfernung angeben, die gibt’s an allen möglichen Stellen, von ein paar hundert bis zu zehntausenden Lichtjahren entfernt (ich hab’ jetzt auch kein konkretes Beispiel parat).

  388. #391 Stefan
    2. Juli 2015

    @Alderamin: Danke für die ausführliche Antwort. Sie haben mir sehr geholfen.

  389. #392 stefan
    2. Juli 2015

    Hi,
    folgende Frage beschäftigt mich schon seit längerem.

    Wenn in einem schwarzen Loch keine Zeit existiert, bzw. keine Zeit vergeht, wie ist es dann möglich, dass sich schwarze Löcher bewegen. Dass schwarze Löcher statisch sind und sich der Raum um sie herum ausdehnt und sich nicht die schwarzen Löcher sondern der Raum selbst bewegt war meine erste Idee, jedoch dürften sich dann schwarze Löcher nicht aufeinander zu bewegen. Und wenn sie sich doch selbst bewegen, dann müsste es doch auch Zeit in einem schwarzen Loch geben, da es sich ohne jeglichen Ablauf von Prozessen im Innersten ja nicht bewegen könnte, da es nun mal Zeit braucht, um von einem Ort zum anderen zu gelangen. Über eine Beantwortung meiner Frage würde ich mich sehr freuen.

  390. #393 Captain E.
    3. Juli 2015

    Das dürfte vom jeweiligen Beobachter abhängen. Wir mögen bemerken, dass ein Schwarzes Loch in einer Zeit t1 eine Strecke s1 zurück legt. Ein Beobachter am Ereignishorizont nimmt unter Einfluss des Schwarzen Lochs zwei völlig andere Werte wahr, einer im Schwarzen Loch selbst theoretisch auch, abgesehen davon, dass der vielleicht gar nichts mehr von außen wahrnehmen kann. Laut Einstein sind Zeiten und Strecken eben nichts absolutes, sondern werden von Schwerkraft und Geschwindigkeit bestimmt.

  391. #394 stefan
    3. Juli 2015

    Mir ist schon klar, dass die ganze Sache relativ ist. Bei einem Stillstand der Zeit im Inneren des schwarzen loches, der ja gegeben sein muss wenn das Licht unendlich lange braucht um die Raumkrümmung zu überwinden, da die Lichtgeschwindigkeit ja nun nicht relativ ist, ist mir dennoch nicht klar wie es sich bewegen kann. Weil eine Bewegung von 0 nunmal auch relativistisch eine Bewegung von 0 bleiben sollte oder?

  392. #395 Jens
    4. Juli 2015

    Am 19.01.2006 startete die NASA-Sonde New Horizons zum Plutosystem. Am 24.08.2006 bekommt Pluto von der IAU den Planetenstatus aberkannt. Ist das Zufall oder wollte die IAU mit der Aberkennung bewusst warten bis die Sonde gestartet ist? Wäre die Mission auch durchgeführt wurden wenn Pluto damals schon als Zwergplanet eingestuft gewesen wäre?

  393. #396 Florian Freistetter
    4. Juli 2015

    @Jens: ” Wäre die Mission auch durchgeführt wurden wenn Pluto damals schon als Zwergplanet eingestuft gewesen wäre?”

    Natürlich. Das Problem mit der Aberkennung hat fast ausschließlich die Öffentlichkeit. Die Astronomen sind mit großer Mehrheit schon lange vor 2006 der Meinung gewesen, das Pluto nie Planet genannt werden hätte dürfen. Es war die IAU die sich immer geweigert hat, das anzuerkennen und es waren die Astronomen bei der IAU-Versammlung, die das mit der Aberkennung beschlossen haben. Ob das Ding jetzt “Planet” heißt oder nicht, ist ja auch völlig schnurz, was die wissenschaftlichen Erkenntnisse angeht! Pluto ist um nichts weniger interessant, nur weil er seit 2006 anders klassifiziert wird als vorher! Der Termin 2006 war Zufall – eine Generalversammlung wie die damals in Prag findet nur alle 3 Jahre statt.

  394. #397 Jens
    4. Juli 2015

    @FLorian
    Ja aus wissenschaftlicher Sicht ist klar, dass das Plutosystem hochinteressant ist. Doch so eine Mission ist vermutlich ziemlich teuer und da entscheiden sicher nicht nur reine wissenschaftliche Aspekte ob sie durchgeführt wird oder nicht.
    Ich vermute mal da haben auch einige Politiker ein Wort mit zu reden und ob die von einer Mission zu einem “Zwergplaneten” angetan gewesen wären? Wer weis?

  395. #398 Alderamin
    4. Juli 2015

    @Jens

    Dawn war ja auch eine Mission zu zwei Asteroiden (Vesta, Ceres), von denen einer kurz vor dem Start zum Zwergplaneten befördert wurde.

    Die Auswahl der Roboter-Missionen der NASA ist eher wissenschaftsgetrieben (die der humanen Missionen eher politisch). Es werden regelmäßig die Missionen der kommenden 10-Jahre aus zahlreichen Vorschlägen vorausgewählt, wobei die Vorschläge mit dem besten Wissensgewinn für’s Geld die besten Chancen haben.

  396. #399 Florian Freistetter
    5. Juli 2015

    @jens: “Ich vermute mal da haben auch einige Politiker ein Wort mit zu reden und ob die von einer Mission zu einem “Zwergplaneten” angetan gewesen wären? Wer weis?”

    Missionen werden meist schon Jahrzehnte geplant, bevor sie starten. Die Planung von New Horizons ging schon in den 1990ern los. Was ein paar Monate vor dem Start über den Status von Pluto entschieden wird, ist da völlig egal.

  397. #400 Jens
    5. Juli 2015

    Okay eure Argumente leuchten mir ein. Danke für die Erläuterungen.

  398. […] der Serie “Fragen zur Astronomie” wird es heute mal ein klein wenig wissenschaftstheoretisch. Es geht um die Frage, wie […]

  399. #402 Pedro
    6. Juli 2015

    Das Hubble Gesetz – ein Irrtum?

    Soweit mir bekannt, schlußfolgert Hubble aus der gemessenen Rotverschiebung, dass sich Objekte, die weiter entfernt von der Erde sind, schneller von uns wegbewegen als Objekte, die näher bei uns sind. Dieser Schluss ist aus meiner Sicht unzulässig, weil hier Beobachtungen bzw. Zustände zu verschiedenen Zeiten miteinander verglichen werden. Wenn wir zwei Objekte, eine Galaxie, die 1 Million Lichtjahre entfernt ist und eine weitere, die 10 Millionen Lichtjahre von uns entfernt ist, betrachten, dann betrachten wir Zustände von vor einer Million Jahren und vor 10 Millionen Jahren. Daraus lässt sich leider nichts ableiten.

    Ähnlich wäre es, wenn wir eine Studie zum Kraftstoffverbrauch machen sollten und hätten Daten aus Frankreich und Italien von 2012, von Marokko von 2000 und von Brasilien und Argentinien von 1985. Diese Daten würden wir in Beziehung setzen und bekämen – voilá – je weiter entfernt ein Land ist, umso höher der Verbrauch. Möglicherweise stimmt das sogar, der Schluss aus den vorliegenden Daten ist jedoch nicht zulässig.

    Mit dem Hubble Gesetz ist es ähnlich: Wir können sagen, dass sich Objekte (einmal angenommen, unsere Methoden zur Entfernungsbestimmung sind tatsächlich richtig), die weiter entfernt sind, in weiter zurück liegender Zeit schneller bewegt haben. Das lässt die Vermutung zu, dass sich früher alle Objekte schneller bewegt haben – Sicherheit haben wir nicht, da wir keine gleichzeitigen, direkten Beobachtungen machen können.

    Sind meine Gedankengänge richtig?

  400. #403 Alderamin
    6. Juli 2015

    @Pedro

    Dieser Schluss ist aus meiner Sicht unzulässig, weil hier Beobachtungen bzw. Zustände zu verschiedenen Zeiten miteinander verglichen werden.

    Doch, ist zulässig, denn wenn man die zurückgelegte Strecke bei der beobachteten Geschwindigkeit und der wechselseitigen Gravitation zurückrechnet, kommt man (jedenfalls für die näheren Galaxien, wo die Dunkle Energie noch keine Rolle spielt) auf einen gemeinsamen Ursprung. Das wäre nicht der Fall, wenn sich die Geschwindigkeiten willkürlich oder systematisch geändert hätten. Ganz abgesehen davon, dass es dafür keinen bekannten Mechanismus gibt (bis auf, wieder, die Dunkle Energie, die allerdings selbst durch eine einfache Konstante mit in die Gleichung eingebracht werden kann).

    Die Messungen z.B. von Supernovae ergeben Graphen wie diesen, wo dann die entsprechende Bewegungsgleichung (Friedmann-Gleichung, eine Lösung der Einsteinschen Feldgleichung, die aus der Allgemeinen Relativitätstheorie folgt) druntergelegt wird und man anhand der gewählten Parameter (hier: Dichte der Materie ΩM und Dichte der Dunklen Energie, ΩΛ) den besten Fit sucht (derzeit bei ΩM = ca. 0,32, ΩΛ=0,68).

    Wir können sagen, dass sich Objekte (einmal angenommen, unsere Methoden zur Entfernungsbestimmung sind tatsächlich richtig), die weiter entfernt sind, in weiter zurück liegender Zeit schneller bewegt haben

    Tatsächlich ist die Annahme, dass sich die Galaxien gar nicht bewegen (bis auf ein wenig Herumeiern um lokale Schwerpunkte von Galaxienhaufen), sondern dass sich der Raum zwischen ihnen und uns ausdehnt, und folglich ein weiter entferntes Objekt mehr absolute Entfernungszunahme pro Zeiteinheit erfährt als ein näheres. Daher erscheint die Expansion auch so, als ob wir im exakten Mittelpunkt lägen. Darüber hinaus gibt’s aber auch etliche andere Belege für die Korrektheit der Urknalltheorie, wie etwa die Hintergrundstrahlung (Existenz, Korrelation von Temperaturvariationen über den Winkelabstand), das Verhältnis der Elemente im ursprünglichen Gas, die Entwicklung der Galaxien über ihr Alter, das Alter der ältesten Sterne usw. Die Rotverschiebung ist nur eines von vielen Puzzlestückchen, und wenn man das in Frage stellt, muss man die ganzen anderen Teile des Puzzles, das doch ein eindeutiges Bild ergibt, anders erklären können.

    Sicherheit haben wir nicht, da wir keine gleichzeitigen, direkten Beobachtungen machen können.

    Sicherheiten gibt’s in der Naturwissenschaften nicht, nur bestmögliche Beschreibungen der Beobachtungen durch entsprechende Theorien. Florian schrieb heute darüber.

  401. #404 Thomas
    8. Juli 2015

    Die Urkallttheorie geht ja davon aus, dass sich das gesamte Universum so verhält wie das beobachtbare Universum, oder? Wäre es aber nicht vorstellbar, dass sich Teile des Universum gegensätzlich zum beobachtbaren Universum verhalten? Während sich der eine Teil ausdehnt, könnte sich ja ein anderer, benachbarter Teil zusammenziehen. So ähnlich wie bei einer Wasseroberfläche sich abswechselnde Wellenberge und Wellentäler entstehen.
    Zur Verdeutlichung: Wenn man sich das “gesamte” Universum als eine Art unendliches Gummituch vorstellt, dann ziehen sich Bereiche des Tuches zusammen während benachbarte Bereiche sich ausdehnen. Die Ausdehnungen und Zusemmenziehungen würden dann in einer Wechselwirkung stehen. Könnte man damit nicht erklären, wieso sich das beobachtbare Univerum mit zunehmender Geschwindigkeit Ausdehnt, weil darauf Kräfte wirken von angrenzenden Bereichen die sich zusammenziehen?
    Damit wäre natürlich nicht beantwortet warum sich überhaupt etwas ausdehnt oder zusammenzieht.
    Ist diese Idee kompett auszuschließen? Wenn ja, warum?

  402. #405 Captain E.
    8. Juli 2015

    Das Universum ist groß, größer als unser theoretisch möglicher Beobachtungshorizont. Also ja, denkbar wäre so etwas. Beobachtungsdaten dazu existieren aber wohl bislang nicht.

  403. #406 Captain E.
    8. Juli 2015

    @stefan

    Mir ist schon klar, dass die ganze Sache relativ ist. Bei einem Stillstand der Zeit im Inneren des schwarzen loches, der ja gegeben sein muss wenn das Licht unendlich lange braucht um die Raumkrümmung zu überwinden, da die Lichtgeschwindigkeit ja nun nicht relativ ist, ist mir dennoch nicht klar wie es sich bewegen kann. Weil eine Bewegung von 0 nunmal auch relativistisch eine Bewegung von 0 bleiben sollte oder?

    Zu einer Bewegung gehört eine Geschwindigkeit (und gemäß Einstein ein Bezugssystem). Wenn sich die zurückgelegte Strecke der 0 annähert, konvergiert die Geschwindigkeit gegen 0. Wenn sich andererseits die vergangene Zeit der 0 annähert, divergiert die Geschwindigkeit gegen unendlich. Teilen wir also eine Strecke fast 0 durch eine Zeit fast 0, kann somit praktisch alles herauskommen zwischen 0 und unendlich. Schneller als Lichtgeschwindigkeit wird es schon rein physikalisch gesehen allerdings nicht werden.

  404. #407 Cakir
    10. Juli 2015

    @FF

    Es kommt wieder ein “guter” Wissenschaftsfilm
    Kannst ja, wenn du willst, ein Review darüber machen, wie bei “Interstellar” 😉

    Der Film heißt: “Der Marsianer”. Kinostart: 26 Nov 2015

    Hier ein Trailer:

  405. #408 bikerdet
    10. Juli 2015

    @ Cakir :

    Nachdem Florian eine Resension des Buches gemacht hat, haben es sich vele hier gekauft und verschlungen. Wir warten alle sehnsüchtig auf den Filmstart…

  406. #409 bikerdet
    10. Juli 2015

    @ Thomas in #404 :

    Das Universum ist sicherlich größer als der für uns sichtbare Bereich. Allerdings verhält sich der für uns sichtbare Bereich sehr gleichmäßig. Sowohl was die Verteilung der Materie als auch deren Bewegungsrichtung beträgt.

    Es wäre nicht nur sehr schwer zu erklären, warum sich das Universum überhaupt anders verhalten soll. Es würde sich zudem die Frage stellen, warum dieses andere Verhalten NUR außerhalb unseres Gesichtsfeldes stattfindet.

    Achtung, wir reden hier von sehr großen Maßstäben, größer als Galaxienhaufen !

    Nur weil wir es nicht direkt beobachten können, heißt das ja nicht, das dort alles möglich ist. Zumindest in den ‘Randbereichen’ unseres Gesichtfeldes müßte es dann zu Anomalien kommen. Aber, wie Captain E. bereits schrieb, Beobachtungsdaten liegen uns dazu nicht vor.

  407. #410 Alderamin
    10. Juli 2015

    @bikerdet

    Zumindest in den ‘Randbereichen’ unseres Gesichtfeldes müßte es dann zu Anomalien kommen.

    Nicht notwendigerweise; wir sind in der Situation eines Schiffes im weiten Ozean, das um sich herum überall nur Wasser sieht. Ob hinter dem Horizont Land ist, kann man von dort nicht sehen.

    Allerdings werden wir nie herausfinden können was hinter dem kosmischen Horizont passiert, unsere Lebenszeit ist viel zu kurz. Nichts von jenseits des Horizonts werden wir je zu sehen bekommen oder kann uns irgendwie zukünftig beeinflussen. Deswegen kann man sich auf den Standpunkt stellen: braucht uns auch nicht zu interessieren.

  408. #411 UMa
    10. Juli 2015

    @Pedro
    Allein aus den Messungen der Rotverschiebung und der Entfernung allein kann man, ohne Kenntnis der Bewegungsgleichungen, in der Tat nicht unterscheiden, ob die Zunahme der Geschwindigkeit daran liegt, dass die Galaxie weiter weg ist oder wir sie in weiter Vergangenheit beobachten.

    Nimmt man allerdings das Trägheitsgesetz hinzu, kann man folgern, dass sich die Geschwindigkeiten der Galaxien in wenigen Millionen Jahren gleich bleiben.
    Die zusätzlichen Beschleunigungen, die man z.B. aus der gegenseitigen Anziehung berechnen kann sind sehr klein. Daher stellt für kurze Zeiträume bis wenige hundert Millionen Jahre und kleine Entfernungen bis wenige hundert Millionen Lichtjahre, die Annahme der konstanten Geschwindigkeit ohne äußere Kräfte eine gute Näherung dar.
    https://de.wikipedia.org/wiki/Newtonsche_Gesetze#Erstes_newtonsches_Gesetz

  409. #412 Thomas
    10. Juli 2015

    @bikerdet
    Theoretisch müsste man die Auswirkungen, sich anders verhaltender Bereiche des Universums, auch von unserem Standpunkt aus nachmessen können. Aber da man nicht wissen kann, wie weit diese Bereiche von uns entfernt sind, könnte man auch davon ausgehen, dass deren Auswirkungen mit unseren momentanen Mitteln garnicht messbar sind.
    Ich frage mich nur, ob die ganze Idee überhaupt physikalisch möglich ist. Bei einem schwabbelnden Gummituch ist recht klar, wieso es dort Stellen gibt die sich ausdehnen während die Stelle daneben sich zusammenzieht. Aber ich gehe davon aus, dass sich der Weltraum nicht mit einem Gummituch vergleichen lässt, weil das einfach zwei verschiedene Dinge mit unterschiedlichen Eigenschaften sind.
    Allerdings sehe ich auch ein Fehlerpotential darin, sich auf die Vorgänge innerhalb des beobachtbaren Universums zu beschränken, um diese Vorgänge zu erklären. Die Idee von der dunklen Energie könnte vollkommen unnötig sein, wenn die Ursache für die beschneunigte Ausdehnung des sichtbaren Universums außerhalb davon liegt.
    Es ist ein bischen so als würde man von der Erde aus die Bewegung der Sonne beobachten, um dann zu dem Schluß zu kommen, dass sich die Sonne um die Erde bewegt. (Natürlich kann man auch von der Erde aus beobachten, dass das nicht der Fall ist, wenn man nur die Bewegungen der anderen Himmelskörper berücksichtigt. Aber darauf musste man ja auch ersteinmal kommen.)

  410. #413 bikerdet
    10. Juli 2015

    @ Alderamin in #410 :
    Okay, aber die Inseln hinterm Horizont verlangen keine andere Gesetzmäßigkeit. Sollte sich aber das Universum NUR außerhalb unseres Gesichtsfeldes zusammenziehen, müßte man dafür aber erst einmal einen Grund haben. Ich kann natürlich einfach erklären, das das ganze unbeobachtbare Universum aus Antimaterie besteht und dort die Zeit rückwärts läuft. ABER an den Grenzen muss das Auswirkungen haben. Und dann bleibe noch die große Frage, warum das nur außerhalb unseres Gesichtsfeldes passiert. Dies würde die Annahme der Gleichförmigkeit durchbrechen und uns wieder an einen ‘besonderen Platz’ stellen.

    @ Thomas :
    Ein Gummituch läßt sich einfach verformen. Ein Maß dafür ist das ‘E-modul’ das die Verformbarkeit in Zahlen presst :
    Gummi : 0,01GPa
    Stahl : 210GPa
    Diamant : 1000Gpa
    Raumzeit : 10^24 GPa

    Wenn sich also, in Analogie zum Gummituch, der Raum in Nord-Süd – Richtung ausdeht und dafür in Ost-West – Richtung zusammenzieht, so geht das ja nur, wenn der Raum sich zuerst auch in Ost-West ausgedeht hat und sich nun die Ausdehnung umkehren würde. Dabei müßte die Kontraktion auch größer sein als die Ausdehnung, ansonsten würde sich das Universum ja weiter ausdehnen, nur halt langsamer. In den Grenzbereichen würde sich also der Raum extrem stark verformen müssen. Da hierzu gigantische Energiemengen benötigt würden, so müssen diese irgendwo herkommen. Naja, einen physikalischen Grund müßte man auch definieren.

    Das sich die Erde um die Sonne dreht, kann man nur über die Paralaxenverschiebung beweisen. Und das gelang Friedrich Wilhelm Bessel 1838 zum ersten Male. Erst seit diesem Zeitunkt war das heliozentrische Weltbild verifiziert.

    Leider ist das mit der dunklen Energie nicht so einfach. Es reicht nicht sich vorzustellen, das da von ‘Außen’ jemand am Universum zieht. Wir können die Auswirkungen der DE auch im Kleinen messen. Und wie immer bei einem komplexen System : Wenn man eine Komponente entfernt, muss man eine Neue einfügen, die die alte Komponente in ALLEN Auswirkungen mind. genauso gut ersetzt.

  411. #414 UMa
    13. Juli 2015

    @bikerdet

    Das sich die Erde um die Sonne dreht, kann man nur über die Paralaxenverschiebung beweisen. Und das gelang Friedrich Wilhelm Bessel 1838 zum ersten Male. Erst seit diesem Zeitunkt war das heliozentrische Weltbild verifiziert.

    Was ist mit der Aberration?
    https://de.wikipedia.org/wiki/Aberration_%28Astronomie%29

  412. #415 Alderamin
    13. Juli 2015

    @bikerdet

    Eigentlich soll hier ja nicht diskutiert werden, aber eine letzte Anmerkung von mir:

    Sollte sich aber das Universum NUR außerhalb unseres Gesichtsfeldes zusammenziehen, müßte man dafür aber erst einmal einen Grund haben.

    Wir wissen bisher nicht einmal den Grund für die Dunkle Energie. Es gibt zwar eine Erklärung über eine Vakuumenergie und inneren negativen Druck, aber wir wissen nicht wirklich, was die Ursache dafür ist, und die Inflation zeigt, dass sich die Vakuumenergie auch ändern kann. Möglicherweise kann das Vakuum auch noch weiter auf niedrigere Stufen fallen, vielleicht auch so weit, dass es woanders nicht mehr expandiert. Wir wissen nicht einmal, ob die Naturkonstanten in einer sehr großen Entfernung noch dieselben sind.

    Es gibt zwar keinen Grund, anzunehmen, dass es hinter dem Horizont anders aussieht als hier, aber es gibt auch keinen, das nicht zu tun. Wir wissen schlicht und einfach nichts darüber. Und deswegen kann man weder ausschließen, dass die Expansion dort anders verläuft als hier, noch gibt es einen Grund, dies anzunehmen. Und daher ist die Antwort auf Thomas’ Frage: ja, es ist vorstellbar, dass sich das Universum jenseits des Horizonts anders benimmt als hier. Auch wenn wir darauf keinen Hinweis haben.

    Ich kann natürlich einfach erklären, das das ganze unbeobachtbare Universum aus Antimaterie besteht und dort die Zeit rückwärts läuft. ABER an den Grenzen muss das Auswirkungen haben.

    Wenn sich die Grenze erst weit hinter dem kosmologischen Horizont befindet (oder zu weit in der Vergangenheit liegt). Tatsächlich würde etwa ein Vakuumzerfall, der irgendwo in der Nähe (kosmologisch gesehen, sagen wir, etwa in der Andromedagalaxie) ein so abrupter Übergang sein, der sich mit Lichtgeschwindigkeit fortpflanzt, dass wir ihn erst bemerken würde, wenn er hier einträfe (bzw. hätten wir dann keine Gelegenheit mehr, es zu bemerken, weil die Materie instantan aufhören würde, zu existieren).

    Und dann bleibe noch die große Frage, warum das nur außerhalb unseres Gesichtsfeldes passiert.

    Anthropisches Prinzip? Nur in gewissen, seltenen Bereichen sind die Bedingungen so, dass Sterne, Planeten, Materie, Leben möglich sind, deswegen befinden wir uns notwendigerweise in einem solchen Gebiet. Das muss nicht notwendig überall so sein. Wäre zumindest eine Möglichkeit. Wir können als lebende Wesen nicht davon ausgehen, dass unsere Gegend hier typisch sein muss, solange wir nicht erklären können, warum die Naturkonstanten genau die passenden, scheinbar zufälligen Werte haben, die unsere Existenz überhaupt erst ermöglichen. Wir können niemals völlig neutrale Beobachter sein. Deswegen sollten wir einfach keine Aussagen über Orte machen, die außerhalb unserer Beobachtungs-Reichweite liegen.

  413. #416 SMario
    Villach
    15. Juli 2015

    Hallo Zusammen,

    eine Frage interessiert mich schon seit langem.

    Beim Eintritt von Materie in ein schwarzes Loch sammelt sich diese in einer Akkretionsscheibe am Ereignishorizont und heizt sich extrem auf.Wie heiß kann die Materie dabei werden und ist es möglich, dass dabei Fusionsprozesse zünden?

  414. #417 klauszwingenbeger
    15. Juli 2015

    @ SMario

    Fusionsprozesse setzen nicht nur hohe Temperaturen, sondern gleichzeitig hohe Drücke, also hohe Teilchendichten voraus. Die trifft man dort nicht an.

  415. #418 RootingBill
    Köln
    21. Juli 2015

    Hallo zusammen,

    ich habe die letzten Tage das Buch “Paradoxo: Am Abrund der Ewigkeit” verschlungen und habe mich gefragt, was an der Theorie des Vakuumzerfalls dran ist. Worum geht es dabei eigentlich konkret, wo liegen die tatsächlichen Gefahren?

  416. #419 Jens
    21. Juli 2015

    Warum ist das Prinzip des zureichenden Grundes so fundamental für die moderne Naturwissenschaft?

  417. #420 PDP10
    22. Juli 2015

    @Jens:

    Warum ist Gschwafel heutzutage eine notwendige aber offenbar keine hinreichende Bedingung für hermeneutisch-synkretische und ontologische Betrachtungen in der Philosophie?

    Will meinen: Was willst du uns mit deiner Frage sagen?

    Fragen über Fragen … tja …

  418. #422 Herbert
    23. Juli 2015

    @#416 Mario: “heiss” musst du dir eher als “schnell” vorstellen. Da Akkretionsscheiben (harte) Röntgenstrahlung aussenden, kann man von bis zu 10 mio. Grad ausgehen. Aber die Dichte…
    …and as always, mal die Wiki befragen: https://de.wikipedia.org/wiki/Akkretionsscheibe
    lg

  419. #423 Braunschweiger
    23. Juli 2015

    @Jens #419: Das ist keine Frage zur Astronomie, sondern eben zur Philosophie.

    “Zureichender Grund” wurde mehr oder weniger durch eine unbedingte Kausalität ersetzt. Jedes Phänomen muss begründbar und auf Anderes zurückzuführen sein; Esoterik, bloßer Glaube, Meinung und Politik reichen nicht aus. Zusammen mit Widerspruchsfreiheit gehört dies zu den Säulen der Wissenschaft, weil diese aus Stabilitätsgründen nur auf Fakten und Logik beruhen soll, unabhängig dagegen von Menschen, Sprachen, Zeiten, Weltsystemen etc. — In der Mathematik gibt es die vergleichbaren Begriffe der “Vollständigkeit” und der “Korrektheit”.

  420. #425 Patrik L
    26. Juli 2015

    Wie ist das Hexagon am Nordpol des Saturns zu erklären?

  421. #426 PDP10
    29. Juli 2015

    @Patrik L.:

    Dazu hat Florian früher schon mal was geschrieben:

    https://scienceblogs.de/astrodicticum-simplex/2010/04/10/das-hexagon-am-saturn-im-labor-reproduziert/

    Jetstreams auf der Erde bilden übrigens ganz ähnlich Strukturen:

    https://www.heise.de/newsticker/meldung/NASA-Sonde-Cassini-filmt-Saturn-Sturm-Hexagon-2061747.html

  422. #427 Alex G
    10. August 2015

    Hab ein Vorstellungsproblem bei der Planetenentstehung: Wenn aus der Akkretionsscheibe aus kleinsten Partikeln/Staubkörnchen durch Adhäsion immer größere Staubpartikel entsehen, wachsen sie immer weiter an (laut Literatur) werden zu Steinchen und größeren Steinen bis zu Felsen, bis dann die Graviation alles immer weiter anwächsne lässt bis zu Protoplaneten hin zum GESTEINSplaneten.
    Meine Frage, wie kann man den Übergang von SDtaubpartikel zu Steinchen/Steinen erklären oder sich vorstellen. Sind zB die Asteroiden und kleineren Objekt im Asteroidengürtel noch zum Teil Staubpartikel bzw lockere Ansammlungen davon ?

  423. #428 Alderamin
    10. August 2015

    @Alex G

    Das ist in der Tat ein heiß diskutiertes Thema, wie genau dieser Schritt abläuft.

    Die Teilchen können sich verhaken, aneinander festkleben (“Koagulation”, z.B. durch Eis) und durch elektrostatische Kräfte (Van-der-Waals-Kräfte) aneinander haften bleiben. Wenn eine gewisse Größe erreicht ist, können auch größere neu auftreffende Teilchen inelastisch abgebremst und festgehalten werden, solange die Relativgeschwindigkeit nicht zu groß ist. Das Gas in der Akkretionsscheibe wirkt dabei bremsend auf die enthaltenen Staubteilchen.

    Ab 1 km greift dann irgendwann auch die Schwerkraft. Hier wird behauptet, das Wachstum könne man bis dahin schon simulieren.

    Hier gibt’s Folien aus einer Vorlesung zum Thema Planetenentstehung, sehr schön gemacht, aber auch anspruchsvoll (Vorsicht, 11 MB!)
    https://www.tat.physik.uni-tuebingen.de/~kley/lehre/planeten/folien/kap5x.pdf

    Asteroiden und Kometen sind z.T. noch porös, wie man an ihren geringen Dichten merkt (der Komet Tschurjumov-Gerassimenko, der gerade von der Sonde Rosetta besucht wird, hat z.B. trotz teilweise steinharter Oberfläche nur eine Dichte von 0,4 g/cm³, weniger als die Hälfte von flüssigem Wasser). Jedenfalls solche Objekte, die nicht Teil eines größeren Objekts waren, das schon differenziert war (innen aufgeschmolzen) und dann wieder in kleinere Stücke zertrümmert wurde. Es kann sein, dass Asteroiden teilweise lose “Schotterhaufen” (“rubble piles”) sind. Dafür spricht auch, dass viele von ihnen Monde haben, die z.B. durch die Fliehkraft der Rotation (die etwa vermöge des YORP-Effekts durch Sonnenbestrahlung immer schneller werden kann) losgerissene Bruchstücke sein könnten.

  424. #429 Alex G
    12. August 2015

    @Alderamin
    Vielen Dank für die rasche und gute Antwort. Zusatzfrage, wenn ich den Text richtig verstehe: kleine umhertreibende echte massive Steine sind dann wohl eher das Ergebnis von Stößen und zerfallenen größeren Objekten, als das Resultat immer größer werdender Staubpartikel. Mein größtes Verständnisproblem war die Vorstellung wie Staubballen ohne gravitative Kräfte und ohne Zusammenbacken unter Hitze zu Steinen werden sollten. Vielen Dank nochmals!

  425. #430 Alderamin
    12. August 2015

    @Alex G (echter Kurzname oder eine Hommage an Astro-Alex?)

    kleine umhertreibende echte massive Steine sind dann wohl eher das Ergebnis von Stößen und zerfallenen größeren Objekten, als das Resultat immer größer werdender Staubpartikel.

    Genau. Der Kleinkram aus der Entstehungszeit des Sonnensystems ist lange weg, mit anderen Körpern kollidiert, abgebremst oder weggeblasen vom Sonnenwind.

    Man kann aber einige Meteoriten z.B. dem Asteroiden Vesta zuordnen. Wann immer im Sonnensystem irgendwas kollidiert oder einschlägt, gibt’s kleine Trümmer, so wird das kleine Material immer wieder aufgefrischt. Und Kometen sondern auch Partikel ab, z.B. die Teilchen, die die Sternschnuppen verursachen. Eben las ich auf Twitter, dass Rosetta beobachtet, wie sich 1-40 m große Blöcke von Tschurjumow-Gerassimenko lösen. Wow.

    Cooler Nick übrigens.

  426. #431 Hans-Walter Bronder
    12. August 2015

    @Florian
    Wäre es denkbar, dass die beschleunigte Ausdehnung des Universums die Folge der Gravitation eines unser Universum umhüllendes, größeres Universum ist ?

  427. #432 Maximalgeschwindigkeit von Impaktoren
    SSRMKK
    16. August 2015

    @Florian oder wer sich sonst zu einer Auskunft berufen dünkt.
    In der Wikipedia ist zu lesen:
    “Meteoroiden, die aus dem Sonnensystem stammen, haben im Bereich des Erdorbits eine maximale heliozentrische Geschwindigkeit von etwa 42 km/s (siehe Dritte kosmische Geschwindigkeit). Da die Bahngeschwindigkeit der Erde etwa 30 km/s beträgt, sind Relativgeschwindigkeiten von maximal 72 km/s oder 260.000 km/h möglich.”

    Ich sehe ein, dass Körper mit höherer Geschwindigkeit das Sonnensystem verlassen. Aber könnte ein Meteoroid oder Asteroid durch SwingBy-Effekte nicht auf wesentlich höhere Geschwindigkeiten beschleunigt werden (bevor er auf der Erde einschlägt)?
    Besten Dank.

  428. […] habe ich “Fragen zur Astronomie” […]

  429. #434 Mickey
    5. September 2015

    Als Beweis für die beschleunigte Ausdehnung des Universums nach den Erkenntnissen von Hubble wird immer angeführt, dass sich die am weitest entfernt liegenden Galaxien am schnellsten von allen anderen Objekten wegbewegen .. weil sie die größte Rotverschiebung aufweisen.
    Nun gilt doch aber eigentlich auch .. je weiter die Galaxie entfernt liegt, desto tiefer geht der Blick in die Vergangenheit.
    Nach meinem Verständnis wäre daraus also der umgekehrte Schluss logisch, also dass sich die Beschleunigung des Universums vermindert .. Da ja die am weitesten entfernt liegenden Galaxien (also die ältesten) sich schneller voneinander fortbewegen als sie näheren (jüngeren) Galaxien. Ergo: “je älter die Objekte desto schneller und je jünger dagegen die Objekte sind umso geringer die Fluchtgeschwindigkeit”? Das würde doch für eine Verminderung der Ausdehnungsgeschwindigkeit des Universums sprechen. Nun ist mir klar, dass ich an der Stelle irre, da Hubble für diese Erkenntnis ja wohl den Nobelpreis erhalten hat.
    Als Laie in dem Gebiet verstehe ich die Schlussfolgerung aber trotzdem nicht.

  430. #435 Mickey
    5. September 2015

    Warum ist die Erklärung der dunklen Energie nicht einfach die anhaltende Inflation ?

  431. #436 Alderamin
    6. September 2015

    @Mickey

    Als Beweis für die beschleunigte Ausdehnung des Universums nach den Erkenntnissen von Hubble wird immer angeführt, dass sich die am weitest entfernt liegenden Galaxien am schnellsten von allen anderen Objekten wegbewegen .. weil sie die größte Rotverschiebung aufweisen.

    Das stimmt so nicht ganz, die Urknalltheorie besagt, dass sich die Galaxien nicht bewegen (sie stehen bis auf kleine Bewegungen um nahe andere Galaxien fast still im Raum und sehen z.B. die kosmische Hintergrundstrahlung um sich herum in allen Richtungen mit der gleichen Rotverschiebung und Temperatur, so wie wir auch; die grundlegende Annahme ist, dass das Weltall von überall aus gleich aussieht), sondern dass der Raum zwischen ihnen expandiert.

    Die kosmologische Rotverschiebung entsteht alleine dadurch, dass ein Lichtwellenpaket, das lange Zeit durch den Raum eilt, durch diese Expansion gestreckt wird. Je länger es unterwegs ist, desto mehr. Das bedeutet, dass das Licht fernerer Galaxien, von denen aus es länger unterwegs ist, stärker rotverschoben erscheint, als das von näheren.

    Ergo: “je älter die Objekte desto schneller und je jünger dagegen die Objekte sind umso geringer die Fluchtgeschwindigkeit”?

    Nein, je weiter weg, desto länger war das Licht der Raumexpansion unterworfen und desto stärker ist die Rotverschiebung. Man kann es auch anders ausdrücken: Durch die konstante Raumexpansion wächst eine bestimmte Strecke pro Zeiteinheit um einen bestimmten Faktor (oder Prozentanteil). Der Hubble-Parameter ist derzeit ungefähr 70 km/s/Megaparsec, wobei ein Megaparsec 32,6 Millionen Lichtjahren entspricht. Die doppelte Strecke wächst demnach um den doppelten Wert: 140 km pro Sekunde. Die tausendfache Strecke wächst um das Tausendfache: 140.000 km pro Sekunde, etc. Aber die Expansionsrate ist konstant, eine Strecke wächst pro Zeiteinheit um den gleichen Bruchteil.

    Diese Zahl ist auch nicht durch die Lichtgeschwindigkeit begrenzt, es bewegt sich ja nichts. Aber die Rotverschiebung lässt das Wachstum der Entfernung zu einer fernen Galaxie wie eine Geschwindigkeit im Raum erscheinen. Da eine größere Strecke schneller in absoluten Zahlen wächst, scheinen ferne Galaxien sich schneller zu bewegen als nahe, auch bei konstanter Expansionsrate.

    Tatsächlich nahm die Expansion zu Beginn des Universums noch ab als die Dichte sehr groß war und sich die Materie stärker gegenseitig anzog und abbremste (Materie beeinflusst die Raumkrümmug, daher hat dies auch einen Effekt auf die Raumexpansion). Vor 7 Milliarden Jahren wurde die Dichte dann so gering, dass der Effekt der Dunklen Energie größer wurde als die Abbremsung durch Gravitation, und seit dem expandiert das All zunehmend schneller.

    Ergo: “je älter die Objekte desto schneller und je jünger dagegen die Objekte sind umso geringer die Fluchtgeschwindigkeit”?

    Dass die Dunkle Energie von einer Vakuumenergie verursacht wird, ist eine der plausibelsten Annahmen, aber nicht die einzige Möglichkeit. Zumal der Wert der Vakuumenergie, der sich aus der Quantenphysik ergibt, um den Faktor 10^120 größer ist, als der beobachtete Effekt, da fehlt noch etwas Wichtiges, das diesen Effekt enorm verkleinert.

    Allerdings war die Inflation ungleich heftiger als die heutige Expansion. Bevor sich bei der heutigen Rate eine Entfernung verdoppelt, müssen fast 10 Milliarden Jahre (3,16*10^17 Sekunden) vergehen. Während der Inflation reichten 10^-35 Sekunden. Ein Faktor 3,16*10^52 schneller!

  432. #437 Mickey
    6. September 2015

    @alderadmin
    Danke für die Erklärungen .. jetzt verstehe ich wieder ein bische n mehr von diesem Puzzle. Wobei die Gegensätze jedesmal wieder überwältigend sind .. unsere meist leicht verständlichen Alltagserfahrungen .. die unfassbare Komplexität dessen was wir (oder besser ihr Kosmologen, Astronomen u. Astrophysiker) über den Aufbau des Kosmos schon herausgefunden haben .. aber eben auch den Dingen, wo wir vielleicht nur erahnen wieviel mehr an Komplexität es wohl noch geben muss.

  433. #438 Noonscoomo
    Berlin
    7. September 2015

    @Alderamin
    Mit Aussagen wie “die Galaxien stehen still im Raum” wäre ich etwas vorsichtig, das suggeriert, dass Galaxien sich auch in Bezug auf den Raum bewegen könnten. Das ist aber nicht so. Der Raum ist keine sinnvolle Bezugsgrösse.
    Und wenn ich das richtig sehe ist auch sowohl die beschleunigte Ausdehnung als auch das mit der kosmischen inflation noch heiss umstritten. Für beides gibt es wahlweise andere Erklährungen bzw. keine zweifelsfreien Belege. Wiltshire beispielsweise liefert plausible und bislang meines Wissens unwiederlegte Gründe dafür, warum wir eine beschleunigte Ausdehnung warnehmen, obwohl sie gleichförmig ist. Und der Nachweis der Inflation ist bislang auch meines wissens noch nicht geglückt, Bizep2 musste da ja einen Rückzieher machen.

  434. #439 Spritkopf
    7. September 2015

    @Alderamin

    wobei ein Megaparsec 32,6 Millionen Lichtjahren entspricht.

    Könnte es sein, dass dir da ein Komma verrutscht ist? 😉

  435. #440 Alderamin
    7. September 2015

    @Spritkopf

    Könnte. Und das tausendfache von 70 ist auch nicht 140000…

    @Noonscoomo

    Mit Aussagen wie “die Galaxien stehen still im Raum” wäre ich etwas vorsichtig, das suggeriert, dass Galaxien sich auch in Bezug auf den Raum bewegen könnten. Das ist aber nicht so. Der Raum ist keine sinnvolle Bezugsgrösse.

    Die Hintergrundstrahlung ist aber eine, die im ganzen Weltall zu bestehen scheint. In Bezug auf die Hintergrundstrahlung kann man schon in Ruhe sein (oder eben nicht). Wir bewegen uns mit ca. 600 km/s relativ zur Hintergrundstrahlung, das ist eine typische Geschwindigkeit für eine Galaxie in einem Galaxienhaufen.

    Und wenn ich das richtig sehe ist auch sowohl die beschleunigte Ausdehnung als auch das mit der kosmischen inflation noch heiss umstritten.

    Zur Inflation gibt es Alternativen (wie das Ekpyrotische Universum), aber die beschleunigte Ausdehung ist ein Messergebnis, das ist nicht umstritten. Der Mechanismus dahinter, der ist nicht geklärt.

    Wiltshire beispielsweise liefert plausible und bislang meines Wissens unwiederlegte Gründe dafür, warum wir eine beschleunigte Ausdehnung warnehmen, obwohl sie gleichförmig ist.

    Quelle, Link? Am besten im Verschwörungsgeplauder, hier soll nicht diskutiert werden.

  436. #441 Noonscoomo
    Berlin
    7. September 2015

    Ich halte Prof. David Wiltshire nicht für einen Verschwörungstheoretiker: https://www2.phys.canterbury.ac.nz/~dlw24/universe/MU1404.pdf
    Diese Theorie hat auch Martin Bäker hier auf scienceblogs schon mal besprochen: https://scienceblogs.de/hier-wohnen-drachen/2011/03/15/gibt-es-die-dunkle-energie/

    Daraus ergibt sich für mich eine Frage: Was ist von dieser Theorie zu halten? Und sollte man nicht, solange sie nicht fundiert widerlegt ist, von einer “scheinbar beschleunigten Ausdehnung” sprechen?

  437. #442 Alderamin
    7. September 2015

    @Noonsoomo

    Ich halte Prof. David Wiltshire nicht für einen Verschwörungstheoretiker

    Der Verschwörungsplauder-Thread hat nicht wirklich was mit Verschwörung zu tun, sondern da kann man halt über jedes Thema reden. Hier hingegen sollen nur Fragen gestellt und beantwortet werden. Aber wo Du schon einen Artikel von Martin zum Thema verlinkst, machen wir doch da weiter

  438. #443 Theodor
    21. September 2015

    Hallo, bin ich hier richtig, um Fragen zu stellen?

    Ich hätte nämlich eine, für die ich irgendwie noch keine schlüssige Antwort gefunden habe. Oder diese übersehen habe.

    Wie sind die supermassiven schwarzen Löcher in den Zentren der Galaxien entstanden? Wie aus massiven Sternen schwarze Löcher werden, wird ja hinreichend erklärt. Aber dies passt doch nicht auf die schwarzen Löcher in Galaxie-Kernen?

    LG
    Theodor

  439. #444 HF(de)
    21. September 2015

    @Theodor: ich bin nur Laie aber ich vesuch mal, zwei Fragen ein wenig zu beantworten:
    zu a) (“richtig hier?”): in diesem Blog ist man für solche Fragen genau richtig.
    zu b) gebe ich oben rechts in die Suchfunktion “supermassiv” ein, komme ich u.a. auf
    https://scienceblogs.de/astrodicticum-simplex/2009/11/30/ein-schwarzes-loch-erzeugt-eine-galaxie/
    Ich denke, das ist noch eine Art Henne-Ei-Problem, ich kann aber auch falsch liegen…

  440. #445 Florian Freistetter
    21. September 2015

    @Theodor: “Wie sind die supermassiven schwarzen Löcher in den Zentren der Galaxien entstanden”

    Die Antwort ist einfach: Weiß man noch nicht! 😉

  441. #446 Florian Freistetter
    21. September 2015

    @Theodor: Hier wurde die Frage übrigens beantwortet: https://scienceblogs.de/astrodicticum-simplex/2015/01/19/wieso-befindet-sich-im-zentrum-jeder-galaxie-ein-riesiges-schwarzes-loch/ (steht auch oben in der Liste)

  442. #447 Theodor
    22. September 2015

    Danke und ups, habe ich doch tatsächlich ausgerechnet diese Frage übersehen 🙂

    Das man es nicht genau weiß, aber daran arbeitet, ist trotzdem einigermaßen befriedigend für mich 😉

    Denn in vielen populär-wissenschaftlichen Berichten und den berühmt-berüchtigten TV-Serien werden solche Sachen immer als gegeben hingenommen, ohne nun zu erklären, was da passiert ist.

  443. #448 Almut
    Fellbach
    29. September 2015

    Hallo Florian!
    Ich habe eine Frage zur Rotation des Erd-Mondes.
    Bei der Beobachtung des roten Mondes bei der Mondfinsternis am 28.09.15 ist mir aufgefallen, dass der Mond um fast 90 Grad nach rechts gedreht war, schön zu erkennen war der Krater Tycho nun fast ganz links.
    So habe ich den Mond noch nie gesehen.
    Ich habe auch entsprechende Bilder dieser Mondfinsternis im Internet gefunden, z. Tl. war der Mond sogar noch extremer gedreht um ca. 100 Grad.
    Von der Libration des Mondes habe ich gelesen, aber die ist ja nur minimal.
    Gestern Abend habe ich den Mond nochmal beobachtet und der Krater Tycho war wieder wie gewohnt unten.

    Danke und liebe Grüße
    Almut

  444. #449 klauszwingenberger
    29. September 2015

    Hallo Almut,

    deine Beobachtung könne darauf beruhen, dass Du den Mond in einer Lage am Westhimmel noch nie so bewusst beobachtet hast. Mit der Mondrotation hat das gar nichts zu tun, der Effekt beruht auf der Rotation des Himmelsanblicks und also auf der Rotation der Erde.

    Bei Sternbildern tritt er genauso auf. Nehmen wir den Löwen: er geht im Osten mit der Schnauze voraus auf, liegt sechs Stunden späte, wenn er durch den Meridian geht, auf dem Bauch, und weitere sechs Stunden später geht er, Schnauze voraus und Schwanz in die Höhe unter.

  445. #450 Kyllyeti
    29. September 2015

    Ganz ähnlich gelagert ist übrigens auch die Sache mit der liegenden Mondsichel – hier erklärt,.

  446. #451 Almut
    1. Oktober 2015

    Danke für die Antworten! 🙂
    Ja, das leuchtet ein! Ich habe heute morgen tatsächlich nochmal (trotz Sonnenschein) den Mond beobachten können und den Effekt wieder gesehen.
    Aber dass der Mond dann um ca. 90 Grad “gekippt” sichtbar ist, finde ich sehr erstaunlich!
    Wie weit geht das denn im Laufe einer Nacht bzw. einer sichtbaren Phase (oder was sagt man dazu?)
    Ich habe ja Bilder vom 28.09. um 04:23 im Internet gefunden, wo der Krater Tycho auf “10-Uhr”-Position stand, und die Bilder wurden in Gilching (nahe München) gemacht.
    5. Bild in der Galerie:
    https://www.br.de/themen/wissen/mondfinsternis-mofi-mond-totale-100.html

    1 Stunde später sah ich (in Stuttgart) den Krater Tyche in “9 Uhr”-Position (müsste das nicht eigentlich andersrum sein? Wie rum dreht sich die Sicht denn eigentlich?)
    Ich habe im Internet nur Berichte über den “gekippten” Mond in Bezug auf die Mondsichel gefunden, aber das hat ja eher mit der Bestrahlung durch die Sonne zu tun…

    Liebe Grüße
    Almut

  447. #452 klauszwingenberger
    1. Oktober 2015

    Diese (scheinbare) Drehung geht im Uhrzeigersinn; jedenfalls von der Nordhalbkugel der Erde aus gesehen – unterm Äquator ist es andersrum. Von Mondaufgang bis Monduntergang, also etwas um die 12 Stunden herum, sind es natürlich 180 Grad. Stelle Dir die Mondbahn als Schiene vor, die sich über den Himmel spannt und auf der der Mond einen halben Tag lang läuft. Dann wird es ganz klar.

    Die Lage eines bestimmten Mondkraters auf einem Bild kann von vielen Dingen abhängen: welche Optik wurde bei der Aufnahme verwendet – gerade unterschiedliche Teleskoptypen ergeben unterschiedlich orientierte Bilder; wie ist das Teleskop aufgestellt – ein parallaktisch nachgeführtes behält die einmal eingestellte Bildorientierung bei, bei einem azimutal monintierten wird das Mondbild sich drehen.

  448. #453 Alderamin
    1. Oktober 2015

    @Almut

    Stelle Dir 2 Extrempunkte vor.

    1. Du stehst am Nordpol, die Erde dreht sich. Der Mond kreist dann in konstanter Höher über dem Horizont um Dich herum und verändert seine Neigung nicht. Denke Dir einen Pfeil auf dem Mond, der parallel zur Erdachse ist und nach Norden zeigt. Der Pfeil wird dann immer genau nach oben zeigen, denn Deine Körperachse ist parallel zur Erdachse.

    2. Du stehst am Äquator, der Mond gehe genau im Osten auf. Der gedachte Pfeil zeigt nach Norden, also links. Der Mond steigt höher bis zum Zenit. Der gedachte Pfeil zeigt immer noch nach Norden, entlang des Meridians, die gedachte Linie am Himmel, die vom Südpunkt durch den Zenit zum Nordpunkt geht. Wenn man nach Süden blickt, würde man sagen, der Pfeil steht aufrecht. Der Mond sinkt dann schließlich im Westen gegen den Horizont, der gedachte Pfeil zeigt immer noch nach Norden, welcher aber beim Blick nach Westen rechts liegt. Der Pfeil hat sich also im Laufe der Nacht scheinbar um 180° von links nach rechts gedreht.

    Von unseren mittleren Breiten aus ergibt sich eine Drehung genau dazwischen, etwas weniger als 90°. Was sich dreht, ist die Erde mit der Körperachse des Beobachters, nicht der Mond. Am Nordpol wird man nur um die Längsachse (Füße Richtung Kopf) gedreht, das ändert die scheinbare Ausrichtung des Mondes nicht (wir drehen uns ja genau um diese Achse entgegen der Erddrehung zurück, damit wir den Mond im Blick behalten, obwohl uns die Erde ständig wegdreht).

    Am Äquator hingegen werden wir um eine Achse durch den Bauch umgedreht und auf den Kopf gestellt, die Füße zeigen nach einer halben Erddrehung genau in die Gegenrichtung. Wir müssten einen Handstand machen, um das zu kompensieren. Also dreht sich auch der Mond bei einer halben Erddrehung scheinbar auf den Kopf. Die Situation bei uns setzt sich aus Teildrehungen um beide Achsen zusammen, was nur mit Text schwer zu veranschaulichen ist, aber von der Logik her schon zwischen den beiden Extremen liegen muss, es gibt ja keinen Breitengrad, wo sich die Situation sprunghaft ändert, sondern der Drehwinkel muss sich mit der geografischen Breite stetig ändern.

    Das ganze gilt übrigens für die Lage der Krater genau so, wie für die Beleuchtung, z.B. als Halbmond oder Sichel, denn die ändert sich in einer Nacht kaum. Der Mond bewegt sich nur ein bisschen auf seiner Bahn weiter und ändert minimal die Phase. Man kann näherungsweise zum Verständnis des Problems der Drehung die Phase als fest betrachten, und damit sind stets bestimmte Gebiete auf dem Mond im Licht und andere im Schatten.

  449. #454 Kyllyeti
    1. Oktober 2015

    @Almut

    Ich habe im Internet nur Berichte über den “gekippten” Mond in Bezug auf die Mondsichel gefunden, aber das hat ja eher mit der Bestrahlung durch die Sonne zu tun…

    Nicht nur … der Winkel, in dem die Sonne den Mond beleuchtet, schwankt im Laufe des Jahres höchstens um etwa den halben Wert, den wir hier auf der Erde haben (wenn ich da richtig gerechnet habe). Während einer bestimmten Phase werden deswegen größtenteils immer dieselben Regionen beleuchtet. Ein Gutteil der ‘gekippten Mondsichel’ ist darum auch auf den ‘gekippten Gesamtmond’ zurückzuführen.

  450. […] pünktlich jeden Freitag erscheinen. Ich hoffe, zumindest ab und zu wieder montags ein paar Fragen zur Astronomie beantworten zu können. Darüber hinaus kann ich aber keine Versprechungen machen. Mal sehen […]

  451. […] etwas längerer Pause habe ich endlich wieder Zeit, ein paar Fragen zur Astronomie zu beantworten. Und es geht gleich mit einer sehr interessanten Frage weiter: Wie groß können […]

  452. #457 Andre Wulff
    Hamburg
    12. Oktober 2015

    Eine Anmerkung noch zur Größe von Teleskopen. Refraktoren größer als der Yrekes sind neben der Kostenfrage auch deswegen problematisch, weil die Linsen anfangen würden sich unter ihrem eigenen Gewicht durchzubiegen.

  453. #458 Alderamin
    12. Oktober 2015

    @Andre Wulff

    Es ist auch schwer, einen so großen Glasblock völlig homogen und ohne eingeschlossene Luftblasen und dergleichen zu gießen, und man muss mindestens zwei Oberflächen schleifen (meistens jedoch vier oder sechs, weil Objektive, je nach Bauart, aus zwei bis drei Linsen bestehen), was Aufwand bedeutet. Bei einem Teleskopspiegel muss nur eine einzige Oberfläche perfekt sein und das Innere des Glases ist weitgehend belanglos (es sollte sich nur nicht unter Temperaturschwankungen unterschiedlich ausdehnen).

    Außerdem sind Spiegel absolut farbrein und verursachen keine Brechungsdispersion (chromatische Aberration; weswegen man bei Linsenteleskopen mehrere Linsen aus verschiedenen Glassorten benötigt, deren Fehler sich gegenseitig kompensieren sollen).

  454. #459 R. Steidle
    Schwäbisch Gmünd
    13. Oktober 2015

    Sehr geehrter Herr Freistetter,

    via Kickstarter versucht derzeit ein anonymer Abenteurer, eine Expedition zum Nordpol zu finanzieren, wo er einen Zugang zur “Hohlerde” zu finden hofft, welche freilich als Hirngespinst abgetan werden kann. Mich würde allerdings interessieren, ob überhaupt irgendwelche Himmelskörper bekannt sind, welche dieser Hypothese entsprechend lediglich aus Kruste und eventuell Mantel bestehen, aber keinen Kern aufweisen. Könnte sich ein derartiger Himmelskörper von nennenswertem Umfang überhaupt auf natürlichem Wege bilden? Wäre das nach den gängigen Theorien zur Planetenentstehung nicht zumindest höchst unwahrscheinlich?

    Gruß und danke für Ihre Arbeit!

  455. #460 Jens
    13. Oktober 2015

    Warum hat der Mond keinen Mond? Gibt es im Sonnenensystem überhaupt Monde die ihrerseits von einem Mond umrundet werden oder ist das aus irgendeinem Grund physikalisch ausgeschlossen?

  456. #461 Florian Freistetter
    13. Oktober 2015

    @Jens: Die Frage ist schon beantwortet. Steht oben in der Liste.

  457. #462 Jens
    13. Oktober 2015

    Sorry. Stimmt das hattest du schon unter #19 ausführlich beantwortet.

  458. #463 Polygon
    15. Oktober 2015

    Hallo,
    ich frage mich seit geraumer Zeit, wie Monte-Carlo-Markov-Chain-Techniken eigentlich in der Astrophysik verwendet werden. Was für Daten hat man da, die man auswerten möchte? Wie effizient (sprich: schnell und korrekt) ist es? Welche Alternativen gibt es zu MCMC? Wird eigentlich MultiNest (siehe https://arxiv.org/abs/0809.3437) in der Astrophysik häufig verwendet? Ich kenne es nur aus der Teilchenphysik, wenn Neutrinos detektiert werden sollen, aber von dem, was ich so gelesen habe, scheint es auch für “größere Dimensionen” geeignet sein.
    (Und bitte nicht zu viel Ahnung auf meiner Seite voraussetzen. Meine Kenntnisse in Bayessche Statistik sind doch begrenzt)
    Und vielen Dank für deinen Blog mit den vielen, wertvollen Beiträgen.
    Beste Grüße
    Polygon

  459. #464 Jens
    21. Oktober 2015

    Würden unsere Neutrino- und Gravitationswellendetektoren eine Supernovaexplosion in der Milchstraße nachweisen auch wenn sie optisch nicht sichtbar wäre?

  460. #465 Alderamin
    21. Oktober 2015

    @Jens

    Ja.

    Neutrinos wurden ja schon von der Supernova 1987A nachgewiesen, und die war in der Großen Magellanschen Wolke, die mehr als doppelt so weit entfernt ist, wie der Durchmesser der Milchstraße. Heute gibt’s noch mehr und empfindlichere Detektoren als damals, wir dürften also keine Supernova verpassen.

    Und was Gravitationswellen betrifft: LIGO in den USA bekam kürzlich einen Upgrade und heißt jetzt aLIGO (a wie “advanced”, fortgeschritten). Wie man am Diagramm unten sieht, sind Supernovae jetzt erstmals in Reichweite des Detektors (sogar solche außerhalb der Milchstraße, laut Artikeltext).

  461. #466 imNetz
    22. Oktober 2015

    Sehr geehrter Herr Dr. Freistetter,

    ich hätte eher einen Vorschlag wie folgt:

    Könnten Sie nicht eine gepflegte Liste mit all den vielen Meldungen in den Mainstreammedien über nahende Asteroiden, Kometen und sonstige kosmische Unheilbringer in ihrem geschätzten Blog unterbringen?
    Mein erster Beitrag wäre:

    https://www.vol.at/grosser-kuerbis-im-anflug-asteroid-fliegt-zu-halloween-an-der-erde-vorbei/4495230

    Dem Kürbis da nicht allzu böse sein (-‘

  462. #467 ron
    23. Oktober 2015

    @FF: Ich bin gespannt auf deine Bewertung zu KIC 8462852!

  463. #469 ron
    24. Oktober 2015

    Paul, der Link funktioniert leider nicht…

  464. #470 ron
    24. Oktober 2015

    edit: Da war ein Zeichen zuviel.Habs jetzt.Danke!

  465. #471 Paul
    24. Oktober 2015

    @ron:

    Ok, war wohl schon zu spät oder zuviel Rotwein. Aber Du hast den Artikel ja auch so gefunden.

    Trotzdem, hier nochmal der korrigierte Link:

    Der “geheimnisvollste Stern”: KIC 8462852 und die angebliche Alien-Superzivilisation

  466. #472 mpingu
    26. Oktober 2015
  467. #473 Alderamin
    26. Oktober 2015

    @mpingu

    Ist korrekt und habe ich auch schon woanders gehört.

    Wäre auch nicht die erste Raketenstufe, die für einen Asteroiden gehalten wurde.

  468. […] Und trotz Tour-Stress habe ich zumindest kurz Zeit gefunden, eine Frage zur Astronomie zu beantworten. Diesmal geht es um die Beobachtung des Himmels: Wie groß ist die kleinste Struktur […]

  469. #475 Jan
    5. November 2015

    Hallo, hab dann auch mal eine Frage. Soweit ich weiß braucht man eine Geschwindigkeit von ungefähr 11km/s um die Gravitation der Erde zu überwinden. Wenn man aber eine verdammt lange Leiter nach oben, nach “irgendwo ins Nichts” hätte, und auf dieser Leiter mit 1m/s klettern würde, könnte man dann beliebig weit von der Erde “wegklettern”? Wenn nicht, was würde das verhindern?
    Hab die Frage vor einiger Zeit auch schon auf anderen Seiten gestellt, aber bekam keine Antwort. Hoffe die Frage ist nicht zu blöd, ich hab echt keine Ahnung wo das Problem sein sollte, angenommen man könnte 24/7 klettern 😉

  470. #476 Spritkopf
    6. November 2015

    @Jan

    Soweit ich weiß braucht man eine Geschwindigkeit von ungefähr 11km/s um die Gravitation der Erde zu überwinden.

    Die 11 km/s sind die sogenannte Fluchtgeschwindigkeit. Diese bezieht sich nur auf Objekte, die die Erde umkreisen.

    Ich versuchs mal, zu erklären. Jedes Objekt (ein Satellit o. ä.), welches die Erde umkreist, sich also in einem sogenannten Orbit befindet, benötigt eine bestimmte Bahngeschwindigkeit, um die Erdumkreisung aufrecht zu erhalten. Ist ja auch klar – die Gravitation zieht das Objekt in Richtung Erde, die Bahngeschwindigkeit sorgt dafür, dass sich das Objekt parallel zur Erdoberfläche bewegt. Ist die Bahngeschwindigkeit genau angepasst, dann fällt das Objekt quasi um die Erde herum. Es bewegt sich so schnell, dass die Bahngeschwindigkeit parallel zur Erde und die Fallgeschwindigkeit in Richtung Erde in Kombination eine Kreisbahn um die Erde herum ergeben.

    Verringert man nun die Bahngeschwindigkeit, wird auch die Bahnhöhe über der Erdoberfläche kleiner. Je weiter man die Bahngeschwindigkeit verringert, umso tiefer sinkt die Bahn, bis sie irgendwann die Erdoberfläche berührt und das Objekt abstürzt (in der Realität würde schon bei Bahnhöhen von unter 100 km das Objekt durch die Atmosphäre so stark abgebremst, dass es in jedem Fall abstürzen würde).

    Das Ganze geht aber auch in umgekehrte Richtung. Erhöht man die Bahngeschwindigkeit, wird auch der Orbit immer höher. Und bei einer bestimmten Bahngeschwindigkeit ist dann sozusagen die Grenze des Orbits erreicht, sprich, das Objekt wird nicht mehr durch die Erdanziehung in seiner Umlaufbahn gehalten und verschwindet in den Weiten des Weltalls. Und genau diese Bahngeschwindigkeit nennt man die Fluchtgeschwindigkeit.

    Das ist allerdings eine stark vereinfachte Darstellung. Die Umlaufbahnen sind nicht kreisförmig, sondern wie eine Ellipse geformt und wenn man die Bahngeschwindigkeit verringert, dann “springt” das Objekt nicht einfach auf eine niedrigere Kreisbahn, sondern die Ellipse “verzieht” sich so, dass der Bahnpunkt, der der aktuellen Objektposition gegenüberliegt, in Richtung Erdoberfläche wandert. Genauso, wenn man die Bahngeschwindigkeit erhöht: Dann wandert der gegenüberliegende Punkt von der Erdoberfläche weg, wird also höher.

    Wenn man aber eine verdammt lange Leiter nach oben, nach “irgendwo ins Nichts” hätte, und auf dieser Leiter mit 1m/s klettern würde, könnte man dann beliebig weit von der Erde “wegklettern”?

    Ja, könnte man.

  471. #478 Spritkopf
    6. November 2015

    @Jan
    Oops, mein letzter Kommentar ist in der Moderation gelandet und muss erst freigeschaltet werden.

    Aber als Nachtrag dazu: Wenn dich das Thema näher interessiert, dann empfehle ich dir den Orbiter, ein freier Raumfahrtsimulator, bei dem spielerisch lernt, wie Bahngeschwindigkeit und Umlaufbahn zusammenhängen.

  472. #479 Spritkopf
    6. November 2015

    Florian, ich habe noch zwei Kommentare in der Mod. Könntest du die bitte freischalten?

  473. #480 Florian Freistetter
    6. November 2015

    @Spritkopf: Florian, ich habe noch zwei Kommentare in der Mod. Könntest du die bitte freischalten?

    So wie in den bisherigen 7 Jahren in denen mein Blog existiert mache ich das auch heute ganz von selbst und sobald ich die Zeit dazu habe. Auch ohne explizite Aufforderung…

  474. #481 Spritkopf
    6. November 2015

    Ich war der Auffassung, dass du sie übersehen hattest, weil der erste Kommentar ja noch vor deinem geschrieben wurde.

  475. #482 Jan
    6. November 2015

    Danke für die schnellen Antworten und die Links 🙂

  476. #483 Alderamin
    6. November 2015

    @Jan

    Wenn man so eine Leiter oder einen Weltraumlift hinaufklettern würde, dann würde man auch seitlich beschleunigt, denn die Erde dreht sich ja und der Umfang eines Kreises nimmt mit dem Abstand zu: doppelter Abstand vom Erdmittelpunkt = doppelter Umfang. Da die Umlaufzeit gleich bleibt, wird man also immer schneller. In 35800 km Höhe hätte man dann Kreisbahngeschwindigkeit und in 50630 km Höhe Fluchtgeschwindigkeit. Wenn man den Lift also lang genug machte, könnte man damit Raumsonden in das Sonnensystem starten. Die nötgie Energie für die Beschleunigung würde man der Erddrehung entnehmen, denn die Erde zieht die Lasten mit sich mit, während sie das Seil hochklettern (vereinfacht gesagt, das Gegengewicht am anderen Ende des Seils spielt auch eine Rolle).

  477. #484 AmbiValent
    9. November 2015

    Zu braunen Zwergen gab es schon mal Artikel, aber ich hätte doch noch ein paar Fragen: Bezeichnet der Name “Brauner Zwerg” wirklich sowohl das Objekt während der Deuteriumfusionsphase als auch das danach? Welcher Anteil der Sternmaterie wird ungewandelt? Weniger als 1 Promille oder doch mehr? Und gibt es irgendwo Temperaturkurven für die Entwicklung sowohl bei leichten und schweren Braunen Zwergen als auch bei minimalen Sternen, Sub-Braunen Zwergen und Jupiter zum Vergleich?

  478. #485 Thomas
    13. November 2015

    Der Artikel zum Objekt “V774104” hat bei mir eine (für mich interessante) Frage aufgeworfen. Dort geht es ja u.a. um die Entfernung des Objektes zur Sonne. Von Sedna wissen wir ja, dass diese Objekte eine ziemlich große Abweichung von Perihel und Aphel haben können. Wäre es theoretisch möglich, dass es “da draußen” Objekte von vielleicht mehreren hundert Kilometer Durchmesser gibt, die im Perihel näher als Pluto oder vielleicht sogar Neptun und im Aphel aber soweit entfernt, dass wir sie noch nicht entdeckt haben (können)?

    Vielen Dank schon mal.

  479. #486 robsn
    23. November 2015

    “Liegt” unser Sonnensystem in der Milchstraße, oder “steht” es? Stellt man sich die Milchstraße als große Scheibe und unser Sonnensystem als kleine Scheibe vor, haben beide die gleiche Ausrichtigung, oder steht unser Sonnensystem quasi hochkant? (Ich hoffe man versteht die Frage) Inspiriert durch diese Darstellung (ab ca 8 sec.): https://www.youtube.com/watch?v=PWxcgJUCDxg

  480. #487 Alderamin
    23. November 2015

    @robsn

    Nach dem hier (Thomas Schmidt) beträgt der Winkelabstand zwischen dem Ekliptik- und dem galaktischen Nordpol 60,2°. Also mehr hochkant als liegend, aber noch nicht ganz hochkant.

  481. #488 Jens
    26. November 2015

    Ich habe gehört, dass man aus sogenannten Lichtechos interpretieren kann wann eine Supernova in der Vergangenheit explodiert ist und von welchem Typ sie war. Wie funktioniert das genau?

  482. #489 Alderamin
    26. November 2015

    @Jens

    Wie funktioniert das genau?

    Die verzögerten Lichtwege, die uns erreichen, verlaufen alle über eine Ellipse, in deren einem Brennpunkt die Erde und in deren anderem Brennpunkt die Supernova liegt. Eine Ellipse ist nämlich eine Menge von Punkten, deren Summe der Abstände von zwei festen Punkten (= Lichtlaufstrecke von der Quelle [Punkt 1] über die reflektierende Wolke [Ellipsenumfang] zur Erde [Punkt 2]!) gleich groß ist – deswegen kann man sie zeichnen, indem man eine geschlossene Schnurschleife um die Brennpunkte (z.B. zwei Reißzwecken) legt, mit einem Stift die Schleife stramm zieht und eine Linie um die Brennpunkte malt. Die Schleife ist immer gleich lang, der Abstand zwischen den Brennpunkten ist fest, also muss die Summe der Strecken von den Reisszwecken zum Stift konstant sein.

    Aus dieser Geometrie kann man bei bekannter Entfernung und der scheinbaren Bewegung des Lichtechos (man sieht nur die nach außen laufende Bewegung) bestimmen, wo genau die Linie von der Supernova den Ellipsenrand berührt und wie lang diese Strecke ist. Daraus kann man dann die Laufzeit des Lichts bis zu diesem Punkt und damit das Alter der Supernova berechnen.

    Man kann auch die Entfernung berechnen; ich habe dazu keine Anleitung gefunden, denke aber, dass es funktioniert, wenn man zwei verschiedene Lichtechos hat (wie in der verlinkten Grafik) und auf eine gemeinsame Laufzeit kommen will, dann wird es für die Entfernung eine eindeutig Lösung geben, vermute ich.

  483. #490 Jens
    26. November 2015

    super. Vielen Dank für die detaillierte Erläuterung der Lichtecho-Interpretation zur Bestimmung des Alters/Entfernung. Wird aus dem Spektrum des
    Lichtechos dann auch auf den Typ der Supernova geschlossen?

  484. #491 ina
    Stuttgart
    28. November 2015

    Hallo,

    deine/Ihre Erklärungen sind sehr interessant.
    Ich hätte auch noch zwei Fragen:
    1.) Wie kam die Neigung der Erdbahn zustande? Was sind also die Ursachen? War sie also zu Anfang bereits geneigt oder entstand es durch z.B. Masseverlagerung im Innern oder äußere Anziehungskräfte?
    2.) Sind auch die anderen Planeten unseres Sonnensystems geneigt?
    3) Monde sind wohl nie gegenüber ihren Planeten geneigt, oder?

    Antwort bitte an:
    aschenblume@arcor.de

    Vielen Dank,

    Ina

  485. #492 PDP10
    28. November 2015

    @Ina:

    Zu 1.) Das weiss niemand. Ist halt so. Es gibt Modelle, die die Achsneigung erklären können. Aber sicher weiss man das nicht.

    Zu 2.) Ja. Der Uranus zB um mehr als 90° zur Ebene seiner Umlaufbahn.

    Zu 3.) Doch. Der Mond der Erde ist zB ca. 6° zu seiner Umlaufbahn um die erde “geneigt”. Und die Ebene seiner Bahn ist um mehr als 5° relativ zur Ebene der Erdbahn geneigt.

    Das hättest du aber alles einfach in der Wikipedia nachlesen können … Ihr jungen Leute seid doch angeblich alle “Digital Natives”. Wieso fällt es euch immer so schwer, mal einfach ein paar Fakten selber nach zu schlagen?
    Kommt das daher, dass da bei Facebook nix drüber steht?

    BTW: Schreibst du Montag zufällig eine Klassenarbeit in Erdkunde?

  486. #493 JaJoHa
    29. November 2015

    @Ina
    Zu den Bahnebenen, die werden meines Wissens meistens auf die Erdbahn bezogen. Im Zweikörperproblem ist dann die Bahnebene fest, das ist eine direkte Folge davon, das man eine Zentralkraft (Kraft entlang Verbindungslinie) und Drehimpulserhaltung hat.
    Im Sonnensystem hast du aber mehr als zwei Körper, und in der Frühphase zusätzlich noch Gas und viel mehr Kleinkram. Die Wechselwirkungen damit können dann die Bahnen verändern. Da gibt es hier einige Artikel zu, wahrscheinlich mit Himmelsmechanik als tag.
    Oder meinst du die Neigung der Erdachse gegen ihre Bahnebene?
    Dann schau mal hier
    Zu Uranus gibt es hier einen Artikel

  487. #494 AmbiValent
    29. November 2015

    @Ina
    Zu deinen Fragen 1 und 2:
    Bei der Bildung des Sonnensystems komprimierte sich die Urwolke zur präsolaren Scheibe. In dieser Scheibe rotierte noch alles in derselben Richtung.

    Erst als das regelmäßige Wachstum kleiner Körper in die Zusammenstöße größerer Körper überging, konnten sich durch diese Kollisionen andere Drehachsen entwickeln.

    Aber auch nachdem sich das Sonnensystem entwickelt hatte, konnten sich die Achsen der Planeten durch Übertragung von Drehimpuls (durch die Gravitation der anderen Planeten) weiterhin ändern.

    Zur Frage 3:
    Um die entstehenden Gasriesen bildeten sich ebenfalls Scheiben aus Gas und Staub, aus denen sich dann Monde bildeten. Wahrscheinlich entstanden die weiter innen kreisenden sogenannten regulären Monde mit den Planeten, während die weiter außen kreisenden irregulären erst später eingefangen wurden.

    Die Orbits der regulären Monde sind meist sehr wenig gegen den Äquator des Planeten geneigt (oft weniger als 1 Grad), während die Orbits der irregulären Monde oft sehr stark geneigt sind.

    Der Erdmond ist wieder ein anderer Typ, weil er sich nicht aus einer Staubscheibe entwickelt hatte, sondern aus einer Trümmerscheibe nach der Kollision des marsgroßen Protoplaneten Theia mit der jungen Proto-Erde.

    Die Mondbahn um die Erde ist zwar nur wenige Grad gegen die Bahn der Erde um die Sonne geneigt, aber kann von 18 bis zu 28 Grad gegen den Äquator geneigt sein (abhängig davon, ob die Neigung der Erdachse und die der Mondbahn zusammenfallen oder gegenläufig sind).

  488. […] ob es vielleicht irgendwo auch noch andere, größere Objekte geben kann. Die heutige Ausgabe von “Fragen zur Astronomie” möchte ich daher für eine Antwort nutzen. Also: Kann es noch unentdeckte Planeten im Sonnensystem […]

  489. […] gibt es in der Serie “Fragen zur Astronomie” wieder mal einen Klassiker: Können Pluto und Neptun miteinander zusammenstoßen? Eine gute und […]

  490. #497 Higgs-Teilchen
    Im Standardmodell oben rechts
    1. Dezember 2015

    Hi zusammen.

    Habe gerade in den Einsichten der LMU München Sonderausgabe von 2013 auf Seite 89, folgendes gelesen:

    “So wie bei der Gaswolke im Zentrum der Galaxis: 2011 haben Stefan Gillessen und Reinhard Genzel vom Max-Planck-Institut für Extraterrestrische Physik in Garching überraschend einen Lichtfleck
    erspäht, der auf das Schwarze Loch im Herzen der Milchstraße zurast.
    Offensichtlich handelt es sich um einen kleinen Tropfen aus Gas.
    Eigentlich passt er nicht in die heiße unwirtliche Umgebung, weshalb er schon in wenigen Jahren vom Schwarzen Loch verschluckt
    werden könnte. Astronomen bietet sich damit die einmalige Gelegenheit zu beobachten, wie Materie hinter dem sogenannten Ereignishorizont eines Schwarzen Loches verschwindet – ein direkter Test
    für die Vorhersagen der Allgemeinen Relativitätstheorie.”

    Ist das jetzt schon ppassiert und beobachtet worden oder nicht?

    Lg H.

    https://www.uni-muenchen.de/aktuelles/medien/einsichten/archiv/sonderheft_2013/einsichten_2013_sonderheft.pdf

  491. #498 Alderamin
    2. Dezember 2015

    @Higgs-Teilchen

    G2 schwang um das Schwarze Loch herum und nichts auffälliges passierte.

    https://www.spektrum.de/news/gaswolke-gewinnt-gegen-schwarzes-loch/1319848
    https://www.mpg.de/8775605/gasfluss_galaktisches_zentrum

  492. #499 AmbiValent
    2. Dezember 2015

    Man weiß ja, dass größere Sterne nicht nur Wasserstoff zu Helium fusionieren, sondern auch schwerere Elemente. Sie tun dies allerdings erst, wenn sie schon relativ alt sind (verglichen mit ihrer Gesamtlebenszeit).

    Gibt es nun bekannte Sterne, von denen man weiß, dass sie schon Helium zu Kohlenstoff fusionieren oder gar schwerere Elemente?

  493. #500 Christian
    7. Dezember 2015

    Hallo Florian,

    das Universum ist ja eher ein sehr kalter Ort. Die Sonne hingegen ist an und für sich ein eher heißer Ort. Beide möchte man ja eher meiden, was die Temperaturen betrifft.

    Meine Frage: Wie weit muss man sich der Sonne nähern, als hypothetischer Weltraumreisender, um an einem Ort in unserem Sonnensystem zu sein, der angenehme Raumtemperatur hat? Sagen wir mal: 25°C.

  494. #501 klauszwingenberger
    7. Dezember 2015

    @ AmbiValent:

    Rote Riesen fusionieren Helium zu Sauerstoff und Kohlenstoff. Aldebaran und Arktur zum Beispiel. Überriesen fusionieren bis hinauf zum Eisen, dann ist schlagartig Schluss. Antares und Beteigeuze zum Beispiel.

    @ Christian:

    Das kann man so nicht angeben, weil die 25° Celsius, die Du meinst, ein atmosphärisches Phänomen sind. Sie setzen eine Atmosphäre voraus, in der “Wärme” in dem Sinne vorhanden ist, dass viele Teilchen pro Raumvolumen mehr oder weniger schnell in ungeordneter Bewegung sind – und sich dadurch ständig anrempeln. Im Weltraum gibt es so etwas nicht – es gibt hier nur Teilchenenergien, denen man wohl ein Temperaturäquivalent zurechnen kann, die aber etwas ganz anderes sind als Behaglichkeitsgrade. Wohl gibt es im interstellaren Raum Gegenden, in denen infrarote Strahlung von 25° C (oder genauer: etwa 298 K) vorhanden ist. Die Teilchendichten dort sind aber so niedrig, dass das bißchen Strahlung, das sie freisetzen, Dich nicht wirklich “wärmen” kann. Du würdest dort gleichzeitig erfrieren und von hochenergetischer Strahlung gegrillt.

  495. #502 Alderamin
    7. Dezember 2015

    @Christian, klauszwingenberger

    Man kann zumindest ausrechnen, welche Temperatur ein Körper annehmen würde, der dem Sonnenlicht ausgesetzt ist und hinreichend schnell rotiert (so dass er auf der Schattenseite gegenüber der mittleren Temperatur nicht zu sehr abkühlt und auf der Sonnenseite nicht zu warm wird). Die Temperatur hängt aber davon ab, welche Albedo (Rückstrahlvermögen) das Objekt hat. Ich setz’ mal einfach 0,37 an, das ist ungefähr auch der Wert der Erde (beim Mond sind’s 0,12). Außerdem spielt die Form eine Rolle. Ich nehme mal einen Zylinder mit einer Achse senkrecht zur Sonnenrichtung an, um die der Körper rotieren soll, weil ich dann nicht viel nachdenken muss. 😉 Der Zylinder wird dann bei schneller Rotation überall gleichmäßig warm.

    In Erdentfernung von der Sonne beträgt die Solarkonstante ca. 1380 W/m². Bei einer Albedo von 0,37 werden davon 37% reflektiert, der Rest absorbiert, macht 63% * 1380 W/m² = 869,4 W/m² Absorption.

    Die absorbierende Oberfläche entspricht dem Querschnitt des Zylinders. Die abstrahlende entspricht seiner Mantelfläche. Der Querschnitt ist Q = 2 * Radius * Höhe, die Mantelfläche A = Umfang * Höhe = 2 * Radius * π * Höhe = π * Q. Folglich strahlt das Objekt über die Oberfläche π-mal soviel ab, wie es über den Querschnitt absorbiert (hier wird offensichtlich, dass die Berechnung für komplexe Formen aufwendig werden würde).

    Nun gilt:
    Absorption [W] = Q [m²] * 869,4 W/m² (hier steckt die Albedo schon drin!)
    Emission[W] = σ * A * T^4 (Stefan-Boltzmann-Gesetz)
    = σ * π * Q * T^4 ( σ=5,67*10^-8 W / (m²K^4), Stefan-Boltzmann-Konstante)

    Eine Gleichgewichtstemperatur stellt sich ein, wenn Emission und Absorption gleich groß sind:

    869,4 W/m² * Q = σ * π * Q * T^4, also 869,4 W/m² = σ * π * T^4, mithin
    T = (869,4 W/m² / (σ * π) )^¼ = 264,31 K = -8,84 °C

    Wir hätten gerne 25°C, das sind 298,15 K, das ist das 298,15/264,31 = 1,128-fache der Temperatur, also müsste man etwas näher an die Sonne heran. Wir hätten gerne die 1,128^4=1,619-fache Strahlungsleistung = 869,4*1,619 = 1407 W/m² oben in der Temperaturformel, denn
    T’=(1407 W/m² / (σ * π) )^¼ = 298,15 K = 25 °C

    Da die Strahlungsleistung mit 1/r² abnimmt, hätte man bei der, √(1/1,619) = 0,786-fachen Entfernung von der Sonne genau die gewünschte Strahlungsleistung pro Quadratmeter, um die gewünschte Temperatur zu erreichen. Dann wäre nämlich die Solarkonstante = (1/0,786)² * 1380 W/m² = 2233 W/m², mit der Albedo von 0,37 (63% Absorption) würden davon 1407 W/m² vom Zylinder aufgenommen (siehe oben, Formel für T’). Der “Sweet Spot” läge also bei 0,786 AE (zum Vergleich: die Venus befindet sich bei 0,723 AE).

    Wie gesehen hängt das Ganze jedoch von Albedo und Form des Objekts ab. Eine Kugel würde sich oben um die Pole herum weniger stark erwärmen als ein Zylinder, ein Mensch im Raumanzug würde mehr Licht reflektieren etc. Es gibt also keine feste Entfernung, bei der alles so wäre wie auf der Erde, weil, wie klauszwingenberger schon bemerkte, im Weltraum eben keine Luft ist, die eine Temperatur annehmen könnte (und die dann alles auf die gleiche Temperatur brächte).

    Die Abweichung zwischen Zylinder und Kugel macht aus, dass die Temperatur der Erde bei -18°C liegen würde (statt der oben berechneten -8,8°C für den Zylinder gleicher Albedo) – wenn es den Treibhauseffekt nicht gäbe.

  496. […] der Serie “Fragen zur Astronomie” geht es heute wieder einmal um die Grundlagen. Eine Frage, die mir vergleichsweise oft gestellt […]

  497. #504 ImNetz
    18. Dezember 2015

    Frage an Hrn. Freistetter und die Fachleute hier im Blog:
    Falls der rote Zwergstern Wolf 1061 drei Gesteinsplaneten in habitabler Zone hat – wie kann ein Laie sich dies vorstellen? Gibt es dazu gesicherte Erkenntnisse z. B. über Tag / Nacht Rhythmus; Helligkeit, Atmosphäre, Wasser, evt. Klima, Tektonik (Gesteinsbeben ….)?

  498. #505 Florian Freistetter
    18. Dezember 2015

    @ImNetz: “Gibt es dazu gesicherte Erkenntnisse z. B. über Tag / Nacht Rhythmus; Helligkeit, Atmosphäre, Wasser, evt. Klima, Tektonik (Gesteinsbeben ….)?”

    Nein. Die kann es auch nicht geben, weil dazu noch die Instrumente fehlen. Was man momentan messen kann, sagt uns nur den Abstand des Planeten vom Stern, seine Größe und seine Masse.

  499. #506 Alderamin
    18. Dezember 2015

    @ImNetz

    Nein, von den Planeten weiß man nicht mehr als ihre Umlaufbahnen und grob Masse, Größe und Dichte. Die habitable Zone ist einfache der Entfernungsbereich, in dem der Planet gerade so viel Licht von seinem Stern erhält, dass theoretisch dort Wasser flüssig sein kann. Praktisch hängt das aber von der Atmosphäre ab, die ein höllisches Treibhaus wie die Venus sein könnte, oder so arm an Kohlendioxid, dass der ganze Planet gefroren ist. Und selbst wenn flüssiges Wasser vorhanden wäre, muss daraus nicht folgen, dass es dort wirklich Leben gibt, dafür wissen wir noch zu wenig darüber, wie und unter welchen Bedingungen es entstehen kann.

    Theoretisch könnte es jedoch sogar einfaches Leben in den Ozeanen unter den Eispanzern der Monde Europa (Jupiter) und Enceladus (Saturn) geben, die weit außerhalb der habitablen Zone der Sonne liegen. Auch das wissen wir noch nicht, aber die NASA grübelt gerade über einen Lander für Europa nach, der irgendwann in den 2030ern dort im Eis scharren soll (und hoffentlich Bakterienspuren erkennen kann). Wir werden sehen.

  500. #507 AmbiValent
    21. Dezember 2015

    @ImNetz

    Aus den bekannten Daten (Entfernung vom Stern, Leuchtkraft des Sterns), die man auf Wikipedia findet, kann man die Helligkeit auf den Planeten berechnen. Fürs gesamte Spektrum ergäbe sich da für den mittleren Planeten Wolf 1061c 111% der Sonnenhelligkeit auf der Erde. Das meiste davon ist aber Infrarot, im sichtbaren Spektrum hat man noch 20%. (Wie warm der Planet davon wird, hängt von seiner Atmosphäre ab)

    Außerdem könnte man annehmen, dass die inneren beiden Planeten durch Gezeitenwirkung dem Stern immer die selbe Seite zuwenden. Das ist allerdings nicht gesichert (und der Merkur im Sonnensystem tut das auch nicht).

  501. #508 Jens
    22. Dezember 2015

    Gibt es für schwarze Löcher eine Massenober- bzw. Untergrenze?

  502. #509 Alderamin
    22. Dezember 2015

    @Jens

    Nein. Zwar können nur Sterne von mindestens 10 Sonnenmassen zu einem Schwarzen Loch kollabieren, das dann mindestens 3,x Sonnenmassen hat (die exakte Grenzmasse zu den Neutronensternen ist nicht bekannt), aber theoretisch könnten beim Urknall auch viel kleinere Schwarze Löcher entstanden sein. Schwarze Löcher werden irgendwann durch Hawking-Strahlung zerfallen und dabei immer kleiner werden. Am Ende sind sie mikroskopisch klein. In den letzten Sekunden ist der Zerfall geradezu explosiv mit der Gewalt von Milliarden Atombomben.

    Nach oben gibt’s eigentlich auch keine Grenze, aber beim Einfall von Materie entsteht Strahlung, die den Einfall bremst, insbesondere bei Supermassiven Schwarzen Löchern. Es ist dann eine Frage der verfügbaren Menge an Material, wie groß das Schwarze Loch werden kann; man nimmt an, dass die Entstehung Supermassiver Schwarzer Löcher auf das Wachstum der umgebenden Galaxie zurück wirkt, denn es gibt einen linearen Zusammenhang zwischen der Masse der Galaxie (bei Spiralgalaxien nur die Masse der zentralen Verdickung) und der Masse des enthaltenen Schwarzen Lochs. Das in M87 hat über 3 Milliarden Sonnenmassen, die Galaxie ungefähr 2,7 Billionen Sonnenmassen, das ist eine der größten, die wir kennen. Aber theoretisch könnten Schwarze Löcher noch größer werden. Sie können sich ja auch vereinigen, wenn Galaxien kollidieren.

  503. #510 Paul
    22. Dezember 2015

    @Jens

    Hier steht etwas zu Obergrenzen:

    Spektrum.de: Wie groß können Schwarze Löcher werden?

  504. #511 Noonscoomo
    Berlin
    29. Dezember 2015

    Ich hab da mal ne Frage:
    Wenn Arthur in ein schwarzes Loch fällt und Beate das von weit weg beobachtet, dann sieht Beate doch, dass Arthur immer langsamer und immer roter wird und dann am Ereignishorizont steckenbleibt, oder? So steht das zumindes überall. Müsste dann aber Arthur nicht umgekehrt sehen, dass die Galaxie um das Schwarze Loch herumm immer Blauer wird und immer schneller dreht und schliesslich alle Sterne ausgehen und dann das ganze Universum zum Teufel geht? Und was heisst das eigentlich für die Hawking Strahlung? Die entsteht ja genau am Ereignishorizont. Müsste die aus Beates Sicht nicht extremst langwellig sein, also fast unendlich langwellig, weil die Zeit dort von Beate aus gesehen so irre langsam vergeht?

  505. #512 Noonscoomo
    Berlin
    30. Dezember 2015

    Ah, und gleich noch ne Frage: Ein Gravitationswellen-Messgerät funktioniert doch in etwa wie ein interferrometer, bei dem also Ständig die Entfernungen verschiedener Massen gemessen werden und wenn jetzt eine Gravitationswelle vorbeikommt ändert sich diese Entfernung ein ganz klein wenig. Aber ist es nicht so, dass die Gravitationswelle die Raumzeit verbiegt und damit auch ein klein wenig die Zeit und das gilt dann natürlich auch für das Licht, sodass sich dann doch keine interferenz zeigt? Ist das verständlich, was ich meine?

  506. #513 Alderamin
    30. Dezember 2015

    @Noonscoomo

    Müsste dann aber Arthur nicht umgekehrt sehen, dass die Galaxie um das Schwarze Loch herumm immer Blauer wird und immer schneller dreht und schliesslich alle Sterne ausgehen und dann das ganze Universum zum Teufel geht?

    Sieh’ selbst. Erläuterungen weiter unten.

    Müsste die aus Beates Sicht nicht extremst langwellig sein, also fast unendlich langwellig, weil die Zeit dort von Beate aus gesehen so irre langsam vergeht?

    Sie ist stark rotverschoben, und das ist bei ihrer Angabe (Hawking-Temperatur) schon berücksichtigt. Sie entsteht aber knapp außerhalb des Ereignishorizonts, daher ist sie nicht unendlich rotverschoben.

  507. #514 Noonscoomo
    Berlin
    31. Dezember 2015

    @Alderamin
    Vielen Dank. Irgendwie drückt sich aber auch diese Beschreibung um die Beantwortung der Frage, oder ich verstehs einfach nur nicht. Ich probier noch mal darzustellen, was ich meine. Beate sieht, dass Arthur immer langsamer wird, wenn er sich dem Schwarzschild Radius nähert, richtig? Denn aus Sicht von Beate geht Arthurs Uhr immer langsamer bis sie schliesslich stehen bleibt. Richtig? Mal ganz davon abgesehen, dass Arthur sich aus Beates Sicht auch noch erheblich verzerren und verschmieren wird und so, aber das kann uns kurz mal egal sein. Ausserdem wird Arthur immer roter. Umgekehrt müsste das doch aber bedeuten, dass aus Arthurs Sicht Beates Uhr immer schneller geht bis sie schliesslich unendlich schnell geht und währenddessen wird Beate und der Rest des Universums immer blauer. Wenn die Uhr von Beate jetzt aber so rassant schnell geht, von Arthur aus gesehen, dann müsste doch Arthur, mal davon ab, dass sich das aus seiner Sicht auch alles verzerrt, sehen, wie das Universum immer schneller altert und, wenn er schliesslich fast am Ereignishorizont angekommen ist, ist das Universum schon so alt, dass alles schon auseinander geflogen ist und alle Sterne schon dunkel und ausgebrannt sind.
    Oder zusammengefasst, wenn Arthur durch den Ereignishorizont durch ist, muss aus seiner Sicht Beates Universum unendlich alt sein. D.h. Aber doch, dass ein schwarzes Loch niemals wachsen kann, denn weder aus Beates, noch aus Arthurs sicht kommt zu “Lebzeiten” des Universums irgendwas duch den Ereignishorizont. Und das kann doch jetzt irgendwie nicht sein, oder?

  508. […] bei den Fragen zur Astronomie ging es nach einer kleinen Pause wieder […]

  509. #516 Noonscoomo
    Berlin
    1. Januar 2016

    …oh, und würde das nicht bedeuten, dass, wenn ein Stern zum schwarzen Loch wird er so lange komprimiert, bis ein schwarzes Loch entstanden ist und dann alles was an Masse/Energie noch nicht durch den Ereignishorizont durch ist von aussen gesehen da auch nie mehr durch kommt sondern alle Energie auf der “Oberfläche” des Ereignishorizonts stecken bleibt?
    Das würde dann ja aber trotzdem dazu führen, dass der Schwarzschildradius grösser wird… Hm, ich bin verwirrt. Dass hiesse doch, dass der Schwarzschildradius vom Betrachter abhängt.
    Auwei, ich hab nen Knoten im Hirn.

  510. #517 Alderamin
    1. Januar 2016

    @Noonscoomo

    So wie ich das verstanden habe (und ich hoffe, ich schreibe da jetzt keinen Quatsch), gibt es da zweierlei Effekte: Zum einen wird der Dopplereffekt am Ereignishorizont unendlich, d.h. jegliche Schwingung von Lichtwellen und damit überhaupt jegliche zeitliche Periode, die man visuell oder per elektromagnetischer Strahlung beobachtet, erscheint von außen gesehen unendlich lang zu werden, weil das Licht gegen die Schwerkraft nach oben klettern muss, was ihm Energie raubt. Zum anderen geht die Zeit im Schwerefeld tatsächlich langsamer. Der zweite Effekt äußert sich so, dass ein entfernter Beobachter (Beate) z.B. die berühmte Lichtuhr eines Arturs auf einem Planeten mit hoher Schwerkraft langsamer laufen sähe, während Artur eine entsprechende Uhr bei Beate schneller laufen sähe.

    Beim ersten Effekte sollte es meiner Ansicht nach eine Rolle spielen, ob Artur sich bewegt, oder nicht. Wenn er auf der Planetenoberfläche ruht, fällt ihm das Licht von Beate entgegen und wird damit blauverschoben, was jegliche Schwingungsperiode verkürzt (also sieht er Beates Uhr schneller laufen). Fällt er jedoch selbst z.B. in ein schwarzes Loch, dann ist er immer näher dran an diesem, als Licht, das ihn von hinten einholt, und er erfährt mehr Schwerebeschleunigung als dieses, das müsste dann eigentlich auch wieder rotverschoben sein. Aber da bin ich echt überfragt, deswegen habe ich auch noch nicht geantwortet.

    Jedenfalls heißt es immer, aus Sicht des Hereinfallenden dauert es nicht sehr lange bis zur Singularität (siehe auch meinen Link), während aus der Sicht eines entfernten Beobachters alles am Ereignishorizont in die Dunkelheit verblasst und ihn scheinbar nie überschreitet.

    Frag’ doch mal bei Martin Bäker, er oder Niels kennen sich da viel besser aus als ich.

  511. #518 Krypto
    2. Januar 2016

    @Noonscoomo:
    Das ist ein schwieriges Thema.
    Der absolute Fachmann dafür ist Andreas Müller:
    https://www.wissenschaft-online.de/astrowissen/lexdt_s03.html#sing
    Er hat, soweit ich weiß, auch mal nachgewiesen, dass ein einfallendes Objekt die Singularität in endlicher Zeit erreicht.

  512. #519 Anne M.
    Leipzig
    3. Januar 2016

    Hallo Herr Freistetter,

    ich schaue mir gerade eine Sendung über unseren Planeten Sonne an. Dort beschreibt ein Astrophysiker gleich zu beginn folgendes:
    “Ein Planet ist ein Körper, der selber keine Energieproduktion im innern hat.
    Die Sonne produziert ja, setzt Energie frei, ein Planet tut das im allgemeinen nicht. Es gibt Planeten, die sind an der Zwischengrenze eines Planeten und einem Stern, wie zB. Jupiter”…
    Meine Frage:
    Aber was ist mit unserer Erde, sie ist und wird immer heißer, umso tiefer wir ins innere der Erde gelangen. Was ist das für eine Energie und ist das keine Energieproduktion? Ich bedanke mich schonmal im voraus.

  513. #520 Florian Freistetter
    3. Januar 2016

    @anne: die Energie der erde ist quasi “alte” Energie. Die ist noch übrig aus der zeit ihrer Entstehung. Damals war sie heiß und kühlt seitdem ab bzw. hatte sie anfangs viele radioaktive Elemente, die seitdem zerfallen und Energien abgeben. Aber da wird keine neue Energie produziert. Im Gegensatz zur Sonne, wo das der Fall ist durch die ständige Kernfusion.

  514. #521 Andreas
    7. Januar 2016

    Dazu habe ich nichts gefunden:

    ist es eig. ausgeschlossen, dass innerhalb eines bestehenden Universums ein (weiterer) Urknall stattfinden könnte? Andersrum: ist für den Urknall ein absolutes Raum-Zeit-Nichts – um die Singularität herum – ein grundlegende Voraussetzung? Oder beißt sich hier einfach die Katze…

  515. #522 Andreas
    7. Januar 2016

    ach ja, es gibt ja kein “um die Singularität herum”…glaub ich. hm… Ich lass die Frage trotzdem mal stehen.

  516. #523 bikerdet
    7. Januar 2016

    Hallo andreas

    Zumindest ist es pysikalisch und mathematisch möglich, das es zu einem neuen Urknall käme. Es gibt innerhalb der Voids (= extrem leere Gebiete zwischen Galaxienhaufen mit bis zu 500 Mio. LJ Durchmesser) schon ein (fast) absolutes Vakuum.
    Ob dabei ein Universum im Universum entsteht, unser ‘altes’ Universum zerstört oder einfach beide freidlich nebeneinander her existieren ist spekulativ. Gesicherte Aussagen dazu wie es sich ausprägen würde können wir ebensowenig machen wie über die Zeit vor ‘unserem’ Urknall.

    Wenn Du Dich näher in das Thema einlesen möchtest, würde ich Dir :

    Brian Greene : Das elegante Universum
    Lawrence M. Krauss : Ein Universum aus Nichts

    empfehlen.

  517. #524 Andreas
    7. Januar 2016

    @bikerdet
    schon mal interessant, dass es nicht grundsätzlich ausgeschlossen ist. Danke – auch für die Buchtipps.

  518. #525 Alderamin
    7. Januar 2016

    @Andreas

    Es fallen mir 3 Möglichkeiten (theoretische, je nach Korrektheit der Inflationstheorie) für einen “Urknall” innerhalb unseres Universums ein:

    1) Falls unser Vakuum hat nicht den niedrigst möglichen Energiegehalt hätte, und es irgendwo und irgendwann spontan auf eine niedrigere Energiestufe fallen würde. Wenn dies irgendwo passieren würde (und es wurde letztes Jahr ein Papier veröffentlicht, dass ein zerfallendes winziges Schwarzes Loch so einen Zustand herbeiführen könnte), dann würde sich von dort aus eine Art Feuerball mit Lichtgeschwindigkeit ausbreiten, der die überschüssige Vakuumenergie als Strahlung frei setzen würde. Das wäre dann nicht schön für jemanden, dem diese Feuerwand über den Weg läuft (aber sie käme mit Lichtgeschwindigkeit ohne Vorwarnung und man wäre nicht mehr existent, bevor man es merken könnte). Wir brauchen uns darüber allerdings (siehe Link) derzeit keine Sorgen zu machen, da Schwarze Löcher erst in fernster Zukunft (in ca. 10^67 Jahren) zur relevanten Größe zerfallen sein werden und der LHC erzeugt offenbar auch keine Mini-Black-Holes, wie manche für möglich hielten, oder das betreffende Paper hat einfach Unrecht; der LHC tut nämlich nichts, was die Natur nicht schon seit Milliarden Jahren mit kosmischen Partikeln noch viel höherer Energie “ausprobiert” hätte, und da ist ja auch nichts passiert. Also stimmt entweder das Paper nicht, oder solche Prozesse erzeugen einfach keine Mini-Black-Holes. Das war auch nur eine Hypothese unter gewissen Annahmen innerhalb der String-Theorie; man hat im LHC noch keine zerfallenden Mini-Black-Holes gesehen, und er arbeitet mittlerweile mit der vollen Kollisionsenergie.

    2) Nach Alan Guth, dem Vater der kosmischen Inflationstheorie, braucht es möglicherweise nur eine genügend hohe Materie-/Energiedichte von so um die 10^85 g/cm³, um ein Vakuum mit sehr hoher Energie zu erzeugen, das spontan inflationär zu wachsen beginnt (kosmische Inflation). Wir wissen heute noch zu wenig über das Innere von Schwarzen Löchern, und die Inflation als Ursprung des Universums ist bisher noch nicht belegt, aber es wäre zumindest theoretisch denkbar, dass die Materie innerhalb eines Schwarzen Lochs zu so einer Dichte komprimiert wird, dass dies innerhalb des Schwarzen Lochs einen Urknall auslöst, der jedoch von außen unbeobachtbar bliebe, weil kein Licht und keine Information das Schwarze Loch je verlassen kann. Das neue Universum würde sich dann von unserem Universum abnabeln (irgendwann zerfällt das Schwarze Loch in unserem Universum durch Hawking-Strahlung, aber das dauert sehr lange).

    3) Nach der Heisenbergschen Unschärferelation kann ein kleines Volumenelement des Vakuums für kurze Zeiten keine scharfen Energiegehalt haben, sondern der Energiegehalt schwankt zufällig um einen Mittelwert. Für beliebig kurze Intervalle kann die Abweichung sehr groß werden. Wenn dabei zufällig irgendwo eine so hohe Abweichung entstünde, dass dabei die oben genannte Dichte eines inflationären falschen Vakuums erreicht würde, wäre, falls Alan Guth recht hat, ein spontane Entstehung eines Universums aus dem Vakuum möglich (@bikerdet: dazu bedürfte es keines kosmischen Voids, im Gegenteil; das Innere eines Protons hat ohnehin schon eine hohe Energiedichte, das wäre doch ein guter Ausgangsort). Dieses würde sich dann (wenn ich das richtig verstanden habe) seinen eigenen Raum erzeugen und sich somit auch von unserem Universum abnabeln, wir würden davon nichts mitbekommen. Während der Recherche für meinen Artikel über die Zukunft des Universums im Blog-Schreibwettbewerb 2015 fand ich einen Zeitraum von 10^10^56 Jahren, den ein so seltenes Ereignis in unserem Universum statistisch im Mittel benötigen würde. Das ist eine Zahl mit so vielen Nullen, dass man sie im beobachtbaren Universum nicht hintereinander schreiben könnte, aber unser Universum wird ja voraussichtlich ewig expandieren und hat sehr viel Zeit.

    Wie angedeutet ist das jedoch alles höchst spekulativ und hängt an einigen unbelegten Annahmen, aber Deine Frage lautete ja, ob es “denkbar” sei. Ja, ist es.

  519. #526 bruno
    ok -> ot
    8. Januar 2016

    Käptn – es nähert sich uns ein Raumschiff mit Warp-Geschwindigkeit

    Zitat aus einem Star-Treck-Film. Ich bin in der SF-Terminologie nicht bewandert.
    Also gut … Warp. Raumzeit aufraffen, Alcubierre-Zeugs.
    .
    Aber wie kann ich ein mit Warp-v auf mich zu fliegendes Schiff detektieren?

  520. #527 bruno
    ok, wieder on topic
    8. Januar 2016

    (Ex)Methan-Flüsse auf Pluto?

  521. #528 ron
    11. Januar 2016

    Hallo FF,

    ich hab seit einiger Zeit einen dystopischen Gedanken der mich nicht los lässt:

    Angenommen ein massiver Asteroid trifft die Erde und sie bricht anschliessend auseinander in ziemlich große Teile. (Kontinentgröße)
    Wie lange wäre ein Überleben auf einem dieser Teile möglich, wenn man den direkten Impact und die Folgen (Erdbeben, Vulkanausbrüche etc.) übersteht? Auch in Hinsicht auf die veränderte Umlaufbahn…

    Danke für die Beantwortung dieses Gedankenspiels.

    LG

    Ron

  522. #529 Bullet
    11. Januar 2016

    das sind dann wohl nur Minuten … das wäre etwa so, als ob man fragt, wie lange eine Mücke einen Hieb mit einem Vorschlaghammer überlebt, wenn man die direkten Folgen (Lärm, Wärmeentwicklung an der Trefferstelle) vernachlässigt. Immerhin wird, wenn der Planet zerbricht, auch die Atmosphäre a) glühend, und währenddessen b) entfernt.
    Du solltest ein raumflugtaugliches Gefährt haben, wenn das passiert. Und dieses eine Woche vorher sinnentsprechendd verwenden.

  523. #530 PDP10
    11. Januar 2016

    @bruno (#526):

    “Aber wie kann ich ein mit Warp-v auf mich zu fliegendes Schiff detektieren?”

    Das ist recht leicht zu erklären:

    Wer die Technologie hat mit Warp-Geschwindigkeit zu fliegen, kann auch ein Schiff detektieren, dass mit Warp-X auf ihn zu fliegt – sonst gäbs nämlich ein böses Logik-Loch im Drehbuch 😉

  524. #531 Mario
    Wien
    12. Januar 2016

    “.. wir stellen uns das Universum, das Weltall immer als etwas vor, das ganz weit weg ist, irgendwo da draußen. Aber das ist ja nicht der Fall, wir sind ja mittendrin.” (Zitat Radio Ö1, vor wenigen Tagen)

    Gibt es eine Möglichkeit, genau das besser wahrnehmbar zu machen?

    Wenn ich nachts auf der Wiese liege fängt das Weltall ja sofort bei meiner Nasenspitze an, aber die dreidimensionale Wahrnehmung des Raumes funktioniert nicht recht. Ist es nur die Gravitation, die den Unterschied macht zur Wahrnehmung eines Astronauten, wenn der sich mit dem Rücken an einem großen runden Satelliten festschnallen würde?

    Danke!

  525. #532 Florian Freistetter
    12. Januar 2016

    @Mario: “Gibt es eine Möglichkeit, genau das besser wahrnehmbar zu machen?”

    Naja, nicht einfach so per Fingerschnippen… Aber wenn man halt ein bisschen drüber nachdenkt, merkt man schnell, wie nah uns das Universum ist (ich hab ein ganzes Buch darüber geschrieben – “Der Komet im Cocktailglas”).

  526. #533 klauszwingenberger
    12. Januar 2016

    @ Mario:

    Mit etwas Vorstellungsvermögen geht das sogar in ziemlich großem Maßstab. Mache Deine Liegekur im Hochsommer an einem richtig dunklen Ort, und schau in Richtung Süden. Nach einer Weile wrst Du das Band der Milchstraße aus dem Zenit herunter verfolgen können, bis es sich knapp über dem Horizont zu einem dicken Wulst verbreitert – dort ist das Zentrum, um das sich alle ihre Sterne und Materiewolken drehen. Stelle Dir vor, zu diesem riesigen rotierenden System zu gehören.

    Ich habe schon durch manches Teleskop geschaut, aber kaum ein Beobachtungsmoment war so bewegend wie dieser.

  527. #534 Helena
    14. Januar 2016

    Hallo, danke für diese tolle Seite und die vielen anschaulichen Antworten! 🙂
    Nun habe ich auch eine Frage: Man geht ja heute davon aus, dass das Universum vermutlich eukldisch, flach und unendlich ist.
    Wie passt ein solches unendliches Universum in einen räumlich begrenzten Urknall, wenn der Raum nicht irgendwie aufgewickelt – und damit gekrümmt – ist? (Die gesamte Raumzeit muss ja dort drin gewesen sein und nicht irgendwie “außerhalb”.)
    Oder war der Urknall gar nicht auf ein abgeschlossenes Volumen begrenzt, sondern einfach nur ein Zustand maximaler Dichte an jedem Punkt in einem unendlichen Raum?
    Falls ja, würde das Horizontproblem da nicht schon innerhalb des Urknalls auftreten – dass einzelne Bereiche des Universums schon beim Urknall (vor der Inflation) so weit von einander entfernt sind, dass sich sich gar nicht homogen entwickeln können, weil Information nicht rechtzeitig übertragen werden kann?
    Und es würde auch bedeuten, dass das Universum unendlich viel Masse hat. Ist das so/ kann das sein?
    Danke schon mal für die Antwort! =)

  528. #535 Captain E.
    14. Januar 2016

    Mal eine ganz andere Frage: Ist der theoretischer Physiker John W. Moffat vielleicht auf der richtigen Spur? Seine “Modified Gravity”, kurz MOG, scheint wissenschaftlich durchdacht zu sein, auch wenn sie mangels Belegen bislang reine Hypothese ist. Zumindest versucht sie das, was eine Nachfolgetheorie zu Einsteins Relativitätstheorien leisten muss: Sie erklärt sie zu einem Spezialfall. Wenn es hinhaut, war das Universum immer genau so flach, wie es nach heutiger Beobachtung ist, und Dunkle Materie und Energie würden wohl überflüssig.

    https://de.wikipedia.org/wiki/John_Moffat

  529. #536 Helena
    16. Januar 2016

    Ich habe noch zwei Ergänzungen zu meiner Frage oben.
    1) Der Urknall war ein Zustand sehr hoher Massendichte – kann es da sein dass der Raum, auch wenn er *im Prinzip* flach ist, im Urknall durch die hohe Masse stark gekrümmt war (und deswegen hineinpasst), so wie er sich um ein schwarzes Loch krümmt? Und sich dann im Laufe der Expansion geebnet hat?
    2) Ein Teil meiner Frage war, ob der Urknall auf ein abgeschlossenes Volumen beschränkt war. Macht die Frage überhaupt Sinn? Wenn der komplette (unendliche) Raum sich im Urknall befindet, ließe sich die Größe des Volumens nicht bestimmen, und damit auch nicht, ob es beschränkt ist. Außerdem müsste ein abgeschlossenes Volumen einen Rand haben, aber wie geht das wenn es “außen” nichts gibt? (Oder wäre eine topologische Struktur denkbar, die überall an sich selber grenzt?)

  530. #537 Krypto
    16. Januar 2016

    @Helena:
    Die Flachheit wird aktuell erklärt über die Inflation.
    Die Frage 2 macht wohl schon Sinn, aber nicht so wie Du wahrscheinlich meinst:
    Mit dem Urknall ist der Weltraum entstanden und seitdem dehnt er sich aus. Der Urknall fand also überall statt.

  531. #538 Johann
    Amsterdam
    17. Januar 2016

    Hallo Florian,

    es gibt abscheinend eine neue Theorie zum wow Signal.
    https://www.welt.de/wissenschaft/article151076922/Neue-Theorie-zum-mysterioesen-Wow-Signal-aus-dem-All.html

    Ich finde dieses Thema sehr interessant (und bestimmt auch viele weitere Leser) und würde mich total freuen, wenn Du mal einen Artikel oder Statement dazu veröffentlichen würdest.

    Danke für Deinen tollen Blog! 🙂

  532. #539 Florian Freistetter
    17. Januar 2016

    @Johann: “Ich finde dieses Thema sehr interessant (und bestimmt auch viele weitere Leser) und würde mich total freuen, wenn Du mal einen Artikel oder Statement dazu veröffentlichen würdest.”

    Ich finde das auch interessant. Aber mir fehlt leider die Zeit, zu allem etwas zu sagen, was interessant ist… https://scienceblogs.de/astrodicticum-simplex/2016/01/01/veraenderungen-was-2016-in-meinem-blog-anders-werden-wird/

  533. #540 Helena
    17. Januar 2016

    @krypto, danke für deine Antwort.
    Heißt dass, dass man davon ausgeht, dass die Raumzeit erst mit der Inflation flach geworden ist und im Urknall gekrümmt war?
    Dass der Urknall überall war (im heutigen Raum) ist schon klar. Die Frage war, ob der Urknall (trotzdem) in einem begrenzten Volumen stattfand, in dem die gesamte unendliche Raumzeit drin war (und darin Platz hatte, weil sie gekrümmt/ aufgewickelt war).
    (Analogie: Eine unendliche Summe kann gegen einen endlichen Betrag konvergieren und somit in ein endliches Intervall passen, obwohl sie unendlich viele Glieder hat.)

  534. #541 Helena
    17. Januar 2016

    Ich habe gleich noch eine weitere Frage, die mit dem Problem oben zu tun hat:
    Oben habe ich ja gefragt, ob das Horizontproblem dann nicht schon im Urknall besteht – dass einzelne Bereiche des Universums schon beim Urknall (vor der Inflation) so weit von einander entfernt wären, dass sich sich gar nicht homogen entwickeln können, weil Information nicht rechtzeitig übertragen werden kann. Mir ist nun aufgefallen, dass dieses Problem durch einen im Urknall gekrümmten Raum evtl. nicht gelöst wird, weil Licht/Information ja den gekrümmten Raumlinien folgt. Zumindest tut es das bei Gravitationslinsen. Dh., selbst wenn das Urknall-Universum in einem kompakten Volumen untergebracht wäre, das in einer höheren Dimension endlich ist, wäre das Licht dann trotzdem nicht gezwungen, denn gekrümmten 4D-Raumlinien zu folgen und könnte somit das Volumen niemals ganz durchqueren?

    Kurz gefasst lautet meine Frage:
    Wie breitet Licht sich im gekrümmten Raum aus?
    Muss es der Krümmung folgen oder kann es eine Abkürzung durch eine höhere Dimension nehmen?

    Florian, ich habe jetzt erst gelesen dass du 2016 weniger Zeit zum bloggen haben wirst. Schade, aber ich freue mich darauf, dich in den Science Busters zu sehen! Ich poste meine Frage trotzdem mal, für den Fall dass du Zeit findest zu antworten.

  535. #542 Alderamin
    17. Januar 2016

    @Helena

    Die Inflation löst das Horizontproblem, d.h. dass Regionen der Hintergundstrahlung, die ohne Inflation niemals in kausalem Kontakt gestanden haben können (weil ihre Entfernung zu groß war, als dass Licht sie zum damaligen Weltalter hätte zurücklegen können), die gleiche Temperatur haben, z.B. gegenüberliegende Seiten des Himmels (aber mehr als zwei scheinbare Vollmonddurchmeser reichen auch schon). Vor der Inflation war demgemäß dieser Raum so klein, dass die Materie darin einen Temperaturauschgleich erreichen konnte. Sollte das ursprüngliche Universum so groß gewesen sein, dass da dennoch kein vollständiger Temperaturausgleich möglich war, dann wären diese Zonen heute so weit weg, dass wir sie nicht sehen können und insofern machen sie uns kein Kopfzerbrechen, sie sind für uns nicht nachweisbar und damit nicht existent. Die Inflationstheorie löst das Horizontproblem im beobachtbaren Universum.

    Zu der anderen Frage: Licht kann keine Abkürzung nehmen, sondern folgt dem Raum, und wenn dieser gekrümmt ist (z.B. um eine große Masse herum), dann krümmt sich auch der Weg des Lichts entsprechend, das ist das, was wir Gravitation nennen. Das Weltall ist im großen Maßstab flach, nur um Massen herum ist es lokal “hügelig”, wie ein Weg mit Kieselsteinen oder das Meer mit seinen Wellen (Erdkrümmung mal vernachlässigt).

    So ganz verstehe ich Deine Folgerungen aber auch nicht. Der Artikel hier soll aber nicht zum Diskutieren verwendet werden, ich schlage vor, Du suchst mit der obigen Suchfunktion nach dem Schlagwort “Inflation” und fragst in einem der gefundenen Artikel weiter, dann versuchen wir, das zu klären. Aber jetzt gehe ich erst mal joggen…. 😉

  536. #543 Krypto
    17. Januar 2016

    @Helena:
    Die Anfangsbedingungen sind ein ziemlich heißes Thema; die Lösung wäre z.B. eine Theorie der Quantengravitation.

    Dass der Urknall überall war (im heutigen Raum) ist schon klar. Die Frage war, ob der Urknall (trotzdem) in einem begrenzten Volumen stattfand, in dem die gesamte unendliche Raumzeit drin war (und darin Platz hatte, weil sie gekrümmt/ aufgewickelt war).

    Wenn Dir das wirklich klar wäre, würde sich damit Deine Frage erübrigen, denke ich.

  537. […] heutige Frage aus der Serie “Fragen zur Astronomie” wird mir oft gestellt. Es geht um die tollen Bilder, die Teleskope und Raumsonden vom Universum […]

  538. […] der Serie “Fragen zur Astronomie” gibt es heute wieder einen Klassiker. Oft werde ich gefragt: “Sind die Sterne die wir sehen […]

  539. #546 Valentin
    Köln
    20. Januar 2016

    Hallo Florian,

    ich habe eine Frage zum möglichen neunten Planeten in unserem Sonnensystem, dessen mögliche Entdeckung gerade diskutiert wird.

    Auf der Seite des New Scientist heißt es in einem Artikel:

    “Two astronomers say they have found evidence that a planet around 10 times the mass of Earth is lurking in the outer reaches of the solar system, on an orbit that comes no closer than 200 times the distance between the sun and Earth.”

    Gleichzeitig schreibst du in deinem Artikel über “Planet X und Co.”:

    “Ein Planet mit der Masse der Erde müsste mindestens 750 Astronomische Einheiten (AE) weit weg sein. Er muss also 750 Mal weiter von der Sonne entfernt sein, als unser Planet.”

    Mich interessiert wie es zu dieser Diskrepanz kommt, dass jetzt die Annahme eines so gewichtigen Planeten so “nah” an der Sonne getroffen wird?

  540. #547 vortex
    20. Januar 2016

    Hallo Florian,

    Vermutlich hast du es schon mitbekommen: Neuer Planet im Sonnensystem berechnet (?), Beobachtung steht noch aus?

    Mein Englisch ist leider zu schlecht. Eventuell kannst du dich dieser aktuellen Veröffentlichung annehmen?

    https://www.caltech.edu/news/caltech-researchers-find-evidence-real-ninth-planet-49523

    https://iopscience.iop.org/article/10.3847/0004-6256/151/2/22

  541. #548 Captain E.
    26. Januar 2016

    Durch die Medien geistert gerade ein Bericht über eine Vorlesung von Stephen Hawking. Er hat da doziert, dass die Wahrscheinlichkeit, dass die Erde in ein Schwarzes Loch fällt, für jedes einzelne Jahr extrem gering sei. Über die nächsten fünf -bis zehntausend Jahre sei es dennoch unausweichlich. Es sei ein Naturgesetz, dass Planeten unter ihre eigenen Masse zusammenfallen und in einem Schwarzen Loch verschwinden würden. Trotzdem sollten wir angesichts der steigenden Anzahl an Bedrohungen auf der Erde zusehen, dass wir uns nicht vorher selbst auslöschen würden.

    Selbsttätig in sich zusammen fallende Planeten? Ein Schwarzes Loch mitten in unserem schönen Sonnensystem in allerspätestens zehntausend Jahren? Was hat den Mann denn da geritten bzw. was hat er eigentlich sagen wollen?

  542. #549 Krypto
    26. Januar 2016

    Ich frage mich eher, was die Medien geritten hat oder an welcher Stelle da “stille Post” gespielt wurde.

  543. #550 Theodor
    26. Januar 2016

    Ich mich auch.

    Sogar im relativ seriösen RBB Inforadio wurde der Quatsch 1:1 übernommen. Mit O-Ton Ausschnitten von Hawking.

    Scheint, das da jeder die selbe dumme Pressemitteilung übernommen hat, ohne überhaupt zu prüfen, was denn im O-Ton gesagt wurde.

  544. #551 Theodor
    26. Januar 2016

    Da frage ich mich übrigens, wenn man schon bei solch leicht überprüfbaren Meldungen solchen Blödsinn übernimmt, welchen Blödsinn man dann bei etwas komplexeren Sachverhalten wie Wirtschaft und Politik übernimmt?

  545. #552 Higgs-Teilchen
    Im Standardmodell oben rechts
    30. Januar 2016

    Hi.

    Habe gerade in einem Buch gelesen, dass Saturn fast doppelt so viel Energie abgibt, wie er von der Sonne bekommt. Eine mögliche Erklärung ist der sogenannte „Heliumregen“.
    Das Buch ist von 2008. Gibt es dazu mittlerweile ein bessere/andere Erklärung.

    Lg H.

  546. #553 Alderamin
    30. Januar 2016

    @Higgs-Teilchen

    Nein. 2008 ist doch relativ frisch, oder nicht?

  547. #554 bruno
    2. Februar 2016
  548. #555 Claudia
    Tamm
    3. Februar 2016

    Hallo Florian, tolles Blog, bin durch Deinen Podcast zur Wissenschaft mit Holgi darauf gekommen. Da Du ja ein Experte bist, hätte ich eine eher triviale Frage. Ich hoffe, die ganzen Leute (und Du auch), die hier posten werden mir das verzeihen: Es gibt eine Liste aller Sterne auf Wikipedia mit den entsprechenden HIP Nummern aufhttps://starnames.wikia.com/wiki/Stars_Beginning_with_B
    Zu jedem Stern müsste es doch auch eine Konstellation geben, in der er erscheint, beispielsweise “Alula Borealis” etc.
    Wie kann ich die herausfinden?
    Hintergrund: wir möchten die Hunde aus unserer Zucht nach Sternnamen benennen, jetzt ist B dran. Wir hatten uns die Namen Balik 102506, Balzac 3532 und Banaak 59831 herausgesucht. Über Balik konnte ich nur herausfinden dass das wohl das Sternzeichen Fisch auf türkisch ist. Mehr konnte ich über Google leider nicht ermitteln. Vielen herzlichen Dank für einen Tipp wo wir weitere Informationen finden!
    Claudia
    k9scout.blogspot.com

  549. #556 Florian Freistetter
    3. Februar 2016

    @Claudia: Sternnamen sind knifflig; die haben alle oft gleichzeitig verschiedene Namen, je nachdem in welchem Katalog sie auftauchen. Du könntest es mal mit Stellarium probieren: https://scienceblogs.de/astrodicticum-simplex/2010/03/16/der-ganze-himmel-am-computer-stellarium/ Mit dem Programm kann man auch nach Sternen suchen und dann solltest du auch sehen, wo in welchem Sternbild sie liegen. Ansonsten gibts auch noch speziellere Kataloge (https://simbad.u-strasbg.fr/simbad/sim-fbasic) – die sind aber für Laien kaum zu durchblicken. Wenn du da zB “HIP 59831” eingibst, kriegst du jede Menge Infos zurück: https://simbad.u-strasbg.fr/simbad/sim-basic?Ident=HIP+59831&submit=SIMBAD+search Unter anderem 30 verschiedene Namen/Katalognummern; in dem Fall auch gleich groß zu Beginn die Bezeichnung “*2 CVn” und das “CVn” ist das Kürzel für das Sternbild Jagdhunde (https://de.wikipedia.org/wiki/Jagdhunde_%28Sternbild%29). Alle Sternbildkürzel findest du in dieser Liste: https://de.wikipedia.org/wiki/Liste_der_Sternbilder

  550. #557 Wolle
    Barcelona
    11. Februar 2016

    Lieber Florian,

    bin vor einigen Tagen auf Deinen Blog gestoßen und kann gar nicht aufhören zu lesen. Eine fantastische Arbeit machst Du da, vielen Dank dafür. Ich will mich mit einer Frage beteiligen, die vor kurzem im Gespräch mit einem Freund (wir sind beide blutige aber interessierte Laien) aufkam, als er erzählte, er hätte irgendwo gelesen (Spektrum der Wissenschaft?!), dass die Milchstraße nicht quasi “flach” sei, sondern die Arme in der Vertikalen eine Wellenform aufwiesen. Kannst Du das bestätigen? Und wenn ja: Woher kommt das? Eigentlich müssten die Kräfte die Milchstraße doch ausschließlich in der Horizontalen auseinandertreiben?

  551. #558 Damian
    Berlin
    13. Februar 2016

    Hey Florian,

    auch ich bin ein riesen Fan von deinem Blog und hätte auch eine Frage:
    Was sind die Auswirkungen von starken Gravitationswellen? (So stark, das sie sich makroskopisch bemerkbar machen)

    Angenommen, ich hätte eine Gravitationsbombe (also ein Gerät, das Gravitationswellen erzeugt welche den Raum im makroskopisch sichtbare Größenordnung strecken und stauchen). Was würde dann passieren? Sind die Auswirkungen ähnlich der Druckwelle einer normalen Bombe? Was würde mit einem Lineal passieren, durch das eine Gravitationswelle hindurch läuft? Wären die Zentimeter auf dem Lineal ( für mich als Beobachter den die Gravitationswelle noch nicht erreicht hat) unterschiedlich lang? Und wäre das Lineal danach kaputt?

    Vielen Dank für deine sehr spannende Arbeit!

  552. #559 Chytharo
    15. Februar 2016

    Mal von den erheblichen Risiken beim Start und den finanziellen Kosten abgesehen: Wenn man Atommüll in den Weltraum schießen würde, wie würde sich die Gravitation der Planeten unseres Sonnensystems auf eine Rakete voll mit Tonnen von Atommüll auswirken? Und wie sähe es mit dem Antrieb aus? Würde ein Sonnensegel und ein Schub in eine leere Gegend ausreichen oder müsste man die Rakete Jahrzehnte lang antreiben, um sie aus unserem Sonnensystem zu werfen?

  553. #560 Krypto
    15. Februar 2016

    @Chytharo:
    Ohne das grob durchzurechnen:
    Sonnensegel reichen definitiv nicht, um aus dem SS zu gelangen.
    Was die Gravitationswirkung durch Planeten betrifft:
    Auch das SS besteht vor allem aus ganz viel leerem Raum. Irgendwann in vielen, vielen Jahren(durchaus Millionen oder Milliarden möglich) würde das Raumschiff evt. von der Sonne oder einem Planeten eingefangen werden. Ist aber reine Wahrscheinlichkeitsrechnung, wenn das Raumschiff einfach irgendwo herumfliegt.

  554. #561 Noonscoomo
    Berlin
    15. Februar 2016

    @Cytharo
    Es wäre viel einfacher, den Müll in die Sonne zu schiessen. Da braucht man bloss grob die richtige Richtung einstellen, nen kleinen Schubs geben und der Rest passiert von alleine.

  555. #562 Alderamin
    15. Februar 2016

    @Damian

    Gerade hat Martin Bäker nebenan einen Artikel darüber geschrieben, was Gravitationswellen mit einer Eisenstange anstellen könnten (und was nicht).

    https://scienceblogs.de/hier-wohnen-drachen/2016/02/14/was-bedeutet-es-wenn-sich-der-raum-verzerrt/

  556. #563 Alderamin
    15. Februar 2016

    @Noonscoomo

    Es wäre viel einfacher, den Müll in die Sonne zu schiessen. Da braucht man bloss grob die richtige Richtung einstellen, nen kleinen Schubs geben und der Rest passiert von alleine.

    Das ist falsch. Um das Sonnensystem zu verlassen, muss man einer Rakete 16,6 km/s in Richtung der Erbewegung um die Sonne mitgeben (3. kosmische Geschwindigkeit), dann verlässt sie sowohl das Schwerefeld der Erde wie auch das der Sonne.

    Um etwas in die Sonne zu schießen, muss man es gegen die Erdbewegung um die Sonne aus dem Schwerefeld der Erde schießen, und ihm dann noch die Umlaufgeschwindigkeit der Erde um die Sonne (29,8 km/s) nehmen. Das braucht 31,8 km/s (nach den Formeln im Link oben), fast die doppelte Geschwindigkeitsänderung und die vierfache Energie = Treibstoffmenge. Wenn man weniger Geschwindigkeitsänderung erreicht, fällt unser Atommüll auf einer elliptischen Bahn an der Sonne vorbei, um sie herum, und kommt dann wieder genau zum Ausgangspunkt auf der Erdbahn zurück, wo irgendwann mal mit der Erde wieder zusammentrifft. Will man nicht haben.

  557. #564 noonscoomo
    Berlin
    15. Februar 2016

    @Alderamin
    Huch, du hast recht. Ich hatte erwartet es sei einfach, in die Sonne zu fliegen…

  558. #565 Chytharo
    16. Februar 2016

    Vielen Dank für eure Antworten, das ist sehr interessant!

  559. #566 Dirk
    17. Februar 2016

    bewegen sich Gravitationswellen schneller als Lichtgeschwindigkeit? Wenn ja,könnte man dies nutzen z.B. für intertellare Reisen?

  560. #567 Captain E.
    17. Februar 2016

    Laut Albert Einstein und seiner Allgemeinen Relativitätstheorie bewegen sich Gravitationswellen genau mit Lichtgeschwindigkeit. Damit dürfte sich der zweite Teil der Frage erübrigen, oder?

  561. #568 Florian Freistetter
    17. Februar 2016

    @Dirk: Gravitationswellen bewegen sich mit Lichtgeschwindigkeit.

  562. #569 Theodor
    17. Februar 2016

    Ich dachte, man ist noch beim Forschen, ob Gravitonen Träger dieser Wellen sind und ob sie masselos wie Photonen sind? Dies müsste man messen können, wenn Gravitationswellen eben nicht genau Lichtgeschwindigkeit sind. Sie wären dann etwas langsamer?

  563. #570 bikerdet
    19. Februar 2016

    Hallo Florian

    Ich habe ja auch Dein E-Book zur Rosettamission, welches mir gerade wieder mal ‘auf den Schirm’ gekommen ist. Ist dazu eigendlich mal eine Fortsetzung geplant ? Das Buch endet ja vor der Landung von Philae, da gibt es doch bestimmt noch eine spannende Anschlussgeschichte ….

  564. #571 Florian Freistetter
    19. Februar 2016

    @bikerdet: “st dazu eigendlich mal eine Fortsetzung geplant ?”

    MWn nicht…

  565. #572 Alderamin
    19. Februar 2016

    @Theodor

    Gravitation reicht unendlich weit. Das kann sie nur sein, wenn ihre Trägerteilchen masselos sind. Wie die Photonen bei der elektromagnetischen Kraft, die auch unendlich weit reicht. Die Eichbosonen der starken und schwachen Kernkraft haben hingegen eine Ruhmasse und damit nur eine sehr kleine Reichweite.

  566. #573 Theodor
    19. Februar 2016

    Ok, danke.

    Hatte meine Frage aus folgenden Kurzartikel aus Spektrum entnommen:

    Bewegen sich Gravitationswellen mit Lichtgeschwindigkeit fort?
    […]
    Aber einmal angenommen, LIGO und Virgo würden Gravitationswellen einer Verschmelzung zweier Neutronensterne auffangen und ein klassisches Teleskop den damit einhergehenden Gammastrahlenausbruch, und es ergäbe sich eine minimale zeitliche Diskrepanz zwischen beiden – dann hätte dies enorme Konsequenzen für die Grundlagenphysik. Man nimmt an, dass die Schwerkraft durch ein Teilchen vermittelt wird, das man als Graviton bezeichnet – analog zum Photon der elektromagnetischen Wechselwirkung. Wenn dieses Teilchen genau wie das Photon keine Masse hat, sollten sich Gravitationswellen mit Lichtgeschwindigkeit ausbreiten. Nichts anderes erwarten die allermeisten Physiker. Aber möglich wäre es dennoch, dass Gravitonen eine winzige Masse haben – in diesem Fall sollte LIGO “langsame” Gravitationswellen sehen.

    https://www.spektrum.de/news/6-fragen-die-uns-gravitationswellen-beantworten-koennten/1398802?utm_source=sdw-nl&utm_medium=newsletter&utm_campaign=sdw-nl-daily&utm_content=heute

  567. #574 Alderamin
    19. Februar 2016

    @Theodor

    Ja, das stimmt. Theoretisch könnten aber auch Photonen eine winzige Ruhemasse haben und etwas langsamer als “Lichtgeschwindigkeit” sein (also als c aus den Maxwellschen Gleichungen und der Relativitätsheorie). Allerdings müsste diese Masse wirklich sehr, sehr klein sein, bisher wurde derartiges nicht beobachtet.

    In zeitlicher Nähe zum LIGO-Event gab es möglicherweise tatsächlich einen Gammaburst mit 0,4 s Zeitdifferenz, aber er kam später als die Gravitationswellen an. Dazu gibt’s hier eine kuriose Theorie, aber es kann auch sein, dass der Burst eine Fehldetektion war oder gar keine Beziehung zum LIGO-Event hatte.

  568. #575 Captain E.
    19. Februar 2016

    Ist das nicht sogar sehr wahrscheinlich, dass die Gammastrahlung etwas verzögert ankommt? Die Neutrino einer Supernova sind auch meistens etwas früher da als die Gammastrahlung, und die fliegen nur fast Lichtgeschwindigkeit, einfach weil die EM-Strahlung von der Materie ein klein wenig verzögert wird. Oder sehe ich das falsch?

  569. #576 Alderamin
    19. Februar 2016

    @Captain E.

    Bei der Supernova 1987 kamen die Neutrinos sofort aus dem kollabierenden Kern heraus, da breitete sich die Explosion erst von innen her nach außen aus. Erst als die Fusionsfront an die Oberfläche kam, konnte das Licht entkommen und sich auf den Weg zur Erde machen. Das war aber keine Gammastrahlung, die man nachwies, weil es damals noch kein Compton-Teleskop gab.

    Bei fusionierenden Schwarzen Löchern spielt sich alles am Ereignishorizont ab, und der ist winzig gegenüber einem normalen Stern (in den 0,4 Sekunden Zeitunterschied legt das Licht 120.000 km zurück, während die Schwarzen Löcher nur um die hundert Kilometer durchmessen). Außerdem sollte eigentlich gar kein Material mehr um die Schwarzen Löcher kreisen, das beim Absturz noch Gammastrahlen verursachen könnte, siehe den Link unter “kuriose Theorie”. Insofern ist es nicht klar, warum der Gammaburst so viel später kam, und warum er überhaupt kam.

  570. […] doch nur aus ungeladenen Neutronen?. Eine gute Frage und eine, über die ich im Rahmen meiner Serie “Fragen zur Astronomie” ein wenig mehr sagen […]

  571. #578 Dortel von Döns
    25. Februar 2016

    Meldung auf gmx.net: “Alien-Rätsel ist gelöst: Wow!-Signal kam von Kometen” (https://www.gmx.net/magazine/wissen/alien-raetsel-geloest-wow-signal-kometen-31373550) – also, Komenten sollen das Wow!-Signal verursacht haben. Klingt vernünftig, aber ist es zutreffend? Da Meldungen&Artikel auf solchen Seiten immer mit äußerster Vorsicht zu genießen sind, frage ich lieber nach!

  572. #579 Alderamin
    25. Februar 2016

    @Dortel von Döns

    Ja, ist zutreffend, habe ich auch gehört. Ist zumindest eine Erklärungsmöglichkeit, die noch umstritten ist.

    https://www.newscientist.com/article/dn28747-famous-wow-signal-might-have-been-from-comets-not-aliens/

    https://planetary-science.org/wp-content/uploads/2016/01/Paris_Davies-H-I-Line-Signal.pdf

  573. #580 Jens
    28. Februar 2016

    Würde es die Sonne auch geben wenn die Lichtgeschwindigkeit einen viel kleineren Wert hätte ( z.B. nur ein Tausendstel des heutigen Wertes)?

  574. #581 Krypto
    28. Februar 2016

    @Jens:
    Abgesehen davon, dass unser Universum mit seiner Feinabstimmung ein Problem hätte, wäre es auch ziemlich dunkel:
    Die Fluchtgeschwindigkeit unserer Sonne, einem relativ kleinen Stern, beträgt gut 600km/s, also doppelt soviel wie die von Dir als Beispiel genannte LG.
    Damit wären die Sonne und auch noch viel kleinere Sterne Schwarze Löcher 😉

  575. #582 kleiner grüner
    1. März 2016

    Da das Universum sich ausdehnt könnte es irgendwann so groß sein das es keine Anreihung von Molekülen mehr gibt so das zB. einen Menschen oder so zweimal gibt. Ich hoffe die Frage basiert nicht auf einer Fehlannahme.

  576. #583 Captain E.
    1. März 2016

    Vermutlich wird eher das Problem sein, dass selbst Protonen letzten Endes instabil sind und zerfallen. Aber deine Aussage ist nicht so recht klar geworden. Kannst du das noch einmal neu formulieren?

  577. #584 Alderamin
    2. März 2016

    @kleiner grüner, Captain E.

    Ich hab’s so verstanden, dass er folgendes meint, und da hat er möglicherweise Recht:

    https://scienceblogs.de/astrodicticum-simplex/2011/07/22/unendlich-viele-doppelganger-aus-der-parallelwelt/?all=1

    Allerdings gilt das nicht für eine fortgesetzte Expansion (die das All immer weiter “verdünnt”), sondern für den Fall, dass es bereits jetzt sehr, sehr groß und homogen (überall gleich dicht) ist. Wir überblicken ja auf jeden Fall nur einen kleinen Teil des Universums, begrenzt durch die Lichtlaufzeit seit dem Urknall. Hinter dem kosmologischen Horizont geht es jedoch weiter, möglicherweise sogar unendlich weit, und in diesem Fall wären Wiederholungen der Anordnung von Atomen und Molekülen unvermeidlich. Auch die Wiederholung des gesamten beobachtbaren Universums. Inklusive aller möglichen kleinen Variationen davon. Z.B. dass ich jetzt schon im Bett liege.

  578. #585 Daniela
    Heroldsbach
    8. März 2016

    Hallo Herr Freistetter,

    im Falle man weiss im voraus, dass ein Asteroid mit der Erde kollidieren wird, inwieweit kann man anhand von Berechnungen den Einschlagsort und den betroffenen Zerstörungsradius bestimmen. Wie genau wären diese Berechnungen? So wie ich das verstanden habe gibt es verschiedene Unbekannte die in solch eine Berechnung einfließen würden – z.B. die Zusammensetzung des Asteroiden.

    Mich würde außerdem interessieren, was für Möglichkeiten es für mich als Nicht-Astronom es gibt Asteroidensuchprogramme bzw. Asteroidenabwehrprogramme zu unterstützen? Ich bin im Internet bereits auf einige Seiten gestoßen bei denen man Geld spenden könnte, kann diese aber leider nicht auf ihre Seriosität hin beurteilen.

    Gibt es für mich eine Möglichkeit das Interesse der Regierung an solchen Projekten in irgendeiner Weise zu beeinflussen? Die einzige Möglichkeit die ich im Moment sehe wäre ein Brief – aber wohin sollte ich den schicken und irgendwie bezweifle ich, dass so etwas überhaupt beachtet wird.

    Dankeschön.

  579. #586 Crazee
    8. März 2016

    @Daniela:

    Ein Brief an den Bundestagsabgeordneten des eigenen Wahlkreises bzw. ein persönliches Gespräch bei einem seiner “Meet-and-greets” ist mEn tatsächlich das dichteste, wie man an die Bundespolitik drankommt.

  580. #587 Claudia
    8. März 2016

    @Florian Noch herzlichen Dank für Deine Tipps bezüglich der Sternbilder. Das hat mir sehr weitergeholfen da ich ein bisschen (ganz ganz wenig) verstanden habe, dass sie auf verschiedene Arten klassifiziert werden und ich so im Internet weiter suchen konnte bezüglich unserer Sternennamen.

  581. #588 BöHsling
    14. März 2016

    Hi Florian,
    ich verfolge deinen Blog seit einigen jahren hab in letzter Zeit einigen Populärwisschenaftlichen Quatsch zu “weißen Löchern” gelesen, vielleicht könntest du mal was darüber schreiben?

  582. #589 Florian Freistetter
    14. März 2016

    @BöHsling: Ist notiert!

  583. #590 BöHsling
    14. März 2016

    klasse, danke 🙂

  584. #591 Hans-Walter
    Schiffweiler
    14. März 2016

    @Florian
    Ich habe gelesen, dass wenn ein Raumschiff in der Lage wäre, in ein schwarzes Loch zu fliegen, man von der Erde aus das Raumschiff am Ereignishorizont unendlich lange sehen würde, da für das Raumschiff ab dort wegen der unendlich hohen Gravitation keine Zeit mehr vergeht. Liegt hier denn nicht ein Gedankenfehler vor, da der Zeitstillstand am oder auch im schwarzen Loch doch nur für das Raumschiff gilt, nicht aber für den Beobachter auf der Erde ? Nach obiger Theorie dürfte man dann von der Erde ja auch nie z.B. eine Gaswolke oder einen Stern in ein schwarzes Loch verschwinden sehen, sondern von der Erde aus wäre eine mit der Zeit stets größer werdende “Anhäufung” von Objekten am Ereignishorizont zu sehen (die in Wahrheit schon längst im schwarzen Loch verschwunden sind), oder ?

  585. #592 Krypto
    14. März 2016

    @Hans-Walter:
    Soweit ich weiß, “entschwindet” das gedachte Raumschiff in einer extremen Rotverschiebung. Da sammelt sich nichts am EH.

  586. #593 Hans-Walter
    15. März 2016

    @Krypto
    Das sehe ich genauso !! Deswegen kam mir auch das mit dem “unendlich lange sichtbaren Raumschiff” so dubios vor.
    Danke für Deine Antwort !

  587. #594 Marco
    19. März 2016

    Hallo
    ich habe mal folgende Frage:
    Wenn die Zeit an einem Ort der sich relativ zu mir schneller bewegt langsamer vergeht, wie schnell vergeht dann die Zeit für ein Elektron?
    Wenn ich das richtig verstanden habe soll sich ein Elektron doch mit fast Lichtgeschwindigkeit bewegen … aber wie kann das sein wenn seine Zeit fast still steht?

  588. #595 Alderamin
    19. März 2016

    @Marco

    Elektronen bewegen sich nicht immer mit hoher Geschwindigkeit, Du meinst vermutlich Neutrinos.

    Die Relativitätstheorie baut darauf auf, dass die Lichtgeschwinddigkeit immer gleich schnell erscheint. Egal, ob Du mit halber oder 99,9% Lichtgeschwindigkeit unterwegs bist, erscheint Dir ein Lichtstrahl immer gleich schnell. Nimm’ mal an, das Neutrino fliegt neben einem Lichtstrahl her, dann sähen wir es (wenn wir es sehen könnten) fast gleichauf mit dem Lichtstrahl und würden naiverweise annehmen, das Neutrino sähe den Lichtstrahl nur langsam Vorsprung gewinnen. Aus Sicht des Neutrinos würde der selbe Lichtstrahl aber mit voller Lichtgeschwindigkeit davon rasen. Wir können das damit erklären, dass beim Neutrino die Zeit langsamer läuft, so dass ihm der kleine Geschwindigkeitsunterschied viel größer erscheint, eben genau wie Lichtgeschwindigkeit.

    Damit muss dem Neutrino aber auch eine bestimmte Strecke viel kürzer erscheinen. Wenn es sich von A nach B bewegt und es nach unserer Zeitmessung dafür eine bestimmte Dauer braucht, dann braucht es nach seiner Eigenzeit eine kürzere Zeit (z.B. könnte in dieser Zeit der gleichzeitig gestartete Lichtstrahl die doppelte Strecke zurückgelegt haben, was für uns ruhende Beobachter wegen des uns kleiner erscheinenden Geschwindigkeitsunterschieds viel länger dauert als aus Sicht des Neutrinos, das den Lichtstrahl schneller davonjagen sieht). Dies erklärt die Relativitätstheorie damit, dass dem Neutrino Strecken in seiner Bewegungsrichtung verkürzt erscheinen. Die Strecke A-B erscheint ihm um den gleichen Faktor gestaucht, wie uns seine Eigenzeit verlangsamt erscheint.

    So, und nun kommen wir dazu, warum die Relativitätstheorie so heißt, wie sie heißt. Was sieht das Neutrino, wenn es uns betrachtet? Die oben genannten Argumente gelten dann ganz genau so. Aus Sicht des Neutrinos bewegen wir uns schnell relativ zu ihm. Wenn es uns nach dem Vorbeiflug einen Lichtstrahl hinterher schickt, eilen wir fast mit dem selben Tempo davon, wie der Lichtstrahl, und es dauert aus Sicht des Neutrinos lange, bis er uns erreicht. Aus unserer Sicht ist der Strahl aber sehr schnell im Vergleich zu uns, er bewegt sich mit fast doppelter Lichtgeschwindigkeit vom Neutrino weg, das ja mit fast Lichtgeschwindigkeit in die Gegenrichtung fliegt. Deswegen erscheint des dem Neutrino so, als ob unsere Zeit verlangsamt läuft.

    Solange sich die Geschwindigkeiten und ihre Richtungen nicht ändern, beobachtet also jeder gleichförmig bewegte Beobachter, dass die Zeit anderer Beobachter, die sich relativ zum ihm bewegen, langsamer läuft. Er selbst kann sich in Ruhe betrachten und jeder Lichtstrahl (ob von ihm oder anderen ausgesendet) erscheint ihm immer genau gleich schnell. Erst wenn einer die Richtung umkehrt und sich mit einem anderen Beobachter bei dessen Geschwindigkeit trifft, würde man durch Uhrenvergleich feststellen, dass die Zeit bei demjenigen weniger vorangeschritten ist, der die Richtung geändert hat (was dann zum Zwillingsparadoxon führt), aber ich will das jetzt nicht ausführen, sonst wird’s zu verwirrend.

    Das ist, wenn man tiefer drüber nachdenkt, ziemlich starker Tobak, aber genau so wurde es zigfach nachgemessen, genau so funktioniert die Welt, und deswegen ist die Lichtgeschwindigkeit nicht nur irgendeine Geschwindigkeit wie die des Schalls, die man mit ein bisschen Aufwand auch überwinden kann, sondern eine grundlegende Konstante der Raumzeit (wohingegen das, was wir als Stillstand betrachten, genau das nicht ist, sondern jeder kann sich im Prinzip als ruhend betrachten und die ganze Physik funktioniert dann bei ihm genau so, wie man sie kennt).

    Wie sieht das Neutrino also das Universum? Es würde ihm mit fast Lichtgeschwindigkeit bewegt und in der Bewegungsrichtung zusammengedrückt erscheinen, so dass ihm beispielsweise die Milchstraße als breite Ellipse erschiene, wenn es sie in deren Ebene durchqueren würde. Es wäre deshalb aus seiner Sicht in ein paar Stunden oder Minuten schon am gegenüberliegenden Ende, während es aus unserer Sicht 100.000 Jahre bräuchte.

    Ich hoffe, das beantwortet Deine Frage.

  589. #596 Marco
    19. März 2016

    @Alderamin

    Danke Dir für die Erklärung. Ehrlich gesagt muss ich darüber nachdenken. Ich bin froh das Forum gefunden zu haben und habe so viele Fragen das es mir schwer fällt es in Worte zu fassen. Das mit der Zeit ist sehr komisch … aber jetzt sind ja Ferien … da hab ich Zeit darüber nachzudenken. Ich melde mich bald wieder.

  590. #597 MArco
    20. März 2016

    Mal angenommen (ich weiß es nicht) Neutrinos bewegen sich mit 80% der Lichtgeschwindigkeit.
    Das Neutrino und das Licht starten vom gleichen Punkt in die gleiche Richtung.
    Ich stehe still und warte 10 Sekunden ab.

    Dann hat das Licht eine Strecke von 3.000.000 KM und das Neutrino hat für uns dann eine Strecke von 2.400.000 KM zurückgelegt. Der Abstand zwischen Licht und Neutrino beträgt also „nur“ 600.000 KM.
    Für das Neutrino sind also nur zwei Sekunden vergangen (da sich ja auch für das Neutrino das Licht mit 300.000 KM/s entfernt).

    Sind meine Überlegungen richtig gedacht?

    Andererseits hat das Neutrino ja in dieser Zeit (2 Sekunden) für mich eine Entfernung von 2.4000.000 KM zurückgelegt. Also z.B. von Planet A zu Planet B.
    Sooo schnell kann es ja gar nicht sein.
    Und wie wurde nun die Strecke für das Neutrino verkürzt, wie muss ich mir eine Stauchung vorstellen? Wird der Raum kleiner?
    Wird das Neutrino selbst auch gestaucht und was ist dann die echte Entfernung?
    Sind diese beiden Planeten für das Neutrino dann näher aneinander?
    Wird das Universum für schnelle Objekte immer kleiner?
    Wenn ich dann noch weiter denke … ist das Universum für das Licht dann nur noch 2 Dimensional?
    Oder nein … das Licht braucht ja auch Zeit um eine Strecke zurück zu legen.

    Würdest Du sagen die wahre Entfernung zwischen zwei Punkten ist die „maximale gestauchte“ Entfernung bei Lichtgeschwindigkeit (weil die einzige Konstante)?

  591. #598 Marco
    20. März 2016

    Ach und noch eine Frage:
    Wie schnell entfernt sich eigentlich für einen Lichtstrahl ein anderer Lichtstrahl in die gleiche Richtung?
    Steht die Zeit für ein Photon still?

  592. #599 Marco
    20. März 2016

    Oh … ich meinte Meter … nicht KM in meinem vorletzten Eintrag …. sorry.

  593. […] Als ich letzte Woche in meinem Podcast über Berge auf dem Mars gesprochen habe, wurde mir dabei auch eine Frage gestellt, die mir in diesem Zusammenhang oft gestellt wird: Wie misst man eigentlich die Höhe eines Bergs auf dem Mars? Dort gibt es ja keinen “Meeresspiegel” auf den man die Höhe beziehen kann? Eine gute Frage und deswegen ist sie heute auch das Thema in meiner Serie “Fragen zur Astronomie”. […]

  594. #601 Captain E.
    21. März 2016

    Auch wenn der Hausherr es nicht so gerne sieht, hier mal meine Antwort dazu:

    Ja, für ein lichtschnelles Photon vergeht keinerlei Zeit. Die Rechnung mit dem Abstand zwischen Photon und Neutrino und der aus Sicht des Neutrinos vergangenen Zeit ist vermutlich so nicht ganz korrekt – relativistische Effekte sind etwas schwerer zu berechnen. So wäre auch zu betrachten, wie weit das Neutrino das vorausgeeilte Photon von sich entfernt sieht. In die ungefähr richtige Richtung hast du aber schon gedacht. Aufgrund der Geschwindigkeit ist für das Neutrino die Zeit anders vergangen als für den Beobachter – und das Photon war für beide exakt lichtschnell. Die Höchstgeschwindigkeit von Neutrinos ist übrigens sogar noch größer und dermaßen nahe an c, so dass man lange Zeit nicht ausschließen konnte, dass sie keine Ruhemasse haben können. Mittlerweile ist man sich aber ziemlich sicher, dass Neutrinos eine Ruhemasse haben und sich etwas langsam als das Licht bewegen. Trotzdem kann es passieren, dass sich eine Milliarden von Lichtjahren entfernte Supernova durch ihren Neutrinoausstoß ankündigt, bevor das erste Photon eintrifft. Grund: Neutrinos verlassen den Ort der Supernova augenblicklich, Photonen werden durch die Materie verschluckt und wieder abgegeben und müssen sich erst durch diese “Suppe” wühlen, bevor sie endgültig frei abgestrahlt werden können.

    Was soll übrigens eine “wahre Entfernung zwischen zwei Punkten” sein? Strecken sind relativ, und bei Lichtgeschwindigkeit ist das gesamte Universum nur ein Punkt.

  595. #602 Alderamin
    21. März 2016

    @Marco

    Mal angenommen (ich weiß es nicht) Neutrinos bewegen sich mit 80% der Lichtgeschwindigkeit.
    Das Neutrino und das Licht starten vom gleichen Punkt in die gleiche Richtung.
    Ich stehe still und warte 10 Sekunden ab.

    Dann hat das Licht eine Strecke von 3.000.000 KM und das Neutrino hat für uns dann eine Strecke von 2.400.000 KM zurückgelegt. Der Abstand zwischen Licht und Neutrino beträgt also „nur“ 600.000 KM.
    Für das Neutrino sind also nur zwei Sekunden vergangen (da sich ja auch für das Neutrino das Licht mit 300.000 KM/s entfernt).

    Sind meine Überlegungen richtig gedacht?

    Im Prinzip ja, aber wie der Captain schon sagte, nicht exakt. Nehmen wir ruhig mal die 80% c, auch wenn Neutrinos noch viel schneller sind.

    Aus Sicht eines ruhenden Beobachters eilt der Lichtstrahl in 10 Sekunden 3.000.000 km weit. Aus seiner Sicht vergehen beim Neutrino in dieser Zeit 10s * √(1-v²/c²) = 10s * √(1-0,8²) = 10s * 0,6 = 6s. Die 3.000.000 km erscheinen dem Neutrino längenverkürzt um den Faktor 0,6: 3.000.000 * 0,6 = 1.800.000 km. Für das Neutrino hat sich das Licht also in 6 seiner Sekunden 1.800.000 km aus seiner Sicht weit bewegt, das macht eine Lichtgeschwindigkeit von 1.800.000km / 6s = 300.000 km/s aus. Genau die gleiche Lichtgeschwindigkeit, die der ruhende Beobachter für den selben Lichtstrahl misst.

    Andererseits hat das Neutrino ja in dieser Zeit (2 Sekunden) für mich eine Entfernung von 2.4000.000 KM zurückgelegt. Also z.B. von Planet A zu Planet B.
    Sooo schnell kann es ja gar nicht sein.

    Aus Sicht des ruhenden Beobachters hat sich das Neutrino in 10 Sekunden 10s*0,8c = 2.400.000 km weit bewegt. Aus seiner eigenen hat es sich aber nur 6s*0,8c = 1.440.000 km weit bewegt. Seine eigene zurückgelegte Strecke erscheint ihm auch verkürzt.

    Und wie wurde nun die Strecke für das Neutrino verkürzt, wie muss ich mir eine Stauchung vorstellen? Wird der Raum kleiner?
    Wird das Neutrino selbst auch gestaucht und was ist dann die echte Entfernung?
    Sind diese beiden Planeten für das Neutrino dann näher aneinander?

    Der Raum wird in Bewegungsrichtung gestaucht (wie gesagt, aus der scheibenförmigen Milchstraße würde z.B. eine Ellipse werden). Auch für uns erschiene das Neutrino (bzw. ein größeres Objekt, Neutrinos sind ja im Bereich der Messgenauigkeit Punkte) in Bewegungsrichtung verkürzt. Eine “echte Entfernung” gibt’s nicht, wie der Captain schon sagte, echt ist immer, was man misst, und wer sich sehr schnell relativ zu einer Strecke bewegt, misst die selbe Strecke kürzer, als wenn er in Bezug zu ihr ruht.

    Man könnte auch die Strecke zwischen zwei sich mit konstantem Abstand folgenden Neutrinos betrachten. Diese erschiene uns verkürzt (wie die Länge des Neutrinos), aber dem Neutrino nicht. Es würde diese Strecke länger wahrnehmen, als wir das tun. Ihm erscheinen “unsere” Strecken verkürzt, relativ zu denen wir ruhen (also das nahe Weltall) und damit auch der Abstand zwischen zwei Planeten.

    Wird das Universum für schnelle Objekte immer kleiner?
    Wenn ich dann noch weiter denke … ist das Universum für das Licht dann nur noch 2 Dimensional?

    Im Prinzip ja. Kein Objekt und auch kein denkendes Wesen kann allerdings lichtschnell sein, man ist immer einen Tick langsamer, und das Weltall wird einem daher nie völlig platt erscheinen. Und ein Lichtstrahl wird einem immer sehr schnell erscheinen.

    Oder nein … das Licht braucht ja auch Zeit um eine Strecke zurück zu legen

    Nein, Licht hat keine Eigenzeit. Aus seiner Sicht gibt es keine Zeit. Es ist sofort bei der Aussendung auch schon am Ziel.

    Würdest Du sagen die wahre Entfernung zwischen zwei Punkten ist die „maximale gestauchte“ Entfernung bei Lichtgeschwindigkeit (weil die einzige Konstante)?

    Na, da ist die Entfernung ja gerade Null. Die wahre Entfernung ist die, die man misst, und was man misst, hängt von der eigenen Bewegung ab. Allenfalls könnte man die Strecke, relativ zu der man ruht, als “die wahrste” bezeichnen. Wenn ich also die Strecke zwischen zwei Planeten betrachten wollte, würde ich diese bei relativem Stillstand zu den Planeten messen wollen. Wenn ich die Länge eines sehr schnellen Raumschiffs messen wollte, müsste ich hingegen bei der Messung parallel zum Raumschiff fliegen (oder eben die Formeln der Relativitätstheorie verwenden, um die gemessene Länge entsprechend zu korrigieren).

    Wenn ich aber im Raumschiff zu einem fernen Stern unterwegs bin und wissen will, wann ich mein Ziel nach Bordzeit erreiche, dann ist die mir verkürzt erscheinende Strecke zu diesem Stern schon genau die richtige, die multipliziert mit meiner Geschwindigkeit die verbleibende Flugzeit ergibt.

  596. #603 Marco
    21. März 2016

    @Captain und Alderamin
    Vielen Dank für die Antworten

    Wow …
    “Nein, Licht hat keine Eigenzeit. Aus seiner Sicht gibt es keine Zeit. Es ist sofort bei der Aussendung auch schon am Ziel.”
    Das hab ich gedacht aber es erschien mir irgendwie falsch. Das es überhaupt “etwas” gibt das die Zeit still steht ist echt krznumpf(:-)?!

  597. #604 Captain E.
    22. März 2016

    Tja, Marco, das Universum ist nun einmal ein völlig durchgeknallter Ort. Wenn das wirklich von irgendeinem Schöpfer so ersonnen worden ist, will man gar nicht so genau wissen, was der sich vorher alles eingeworfen hat. 😉

  598. #605 H.B.
    30. März 2016

    Meine Fragen:
    Welche Rolle spielen hypothetische “magnetische Monopole” bei der Entstehung:
    – des Universums
    – der Gravitation ?

    Wie erhielt ein Atom seine Elektronenschalen ?
    Gibt es wirklich nur sieben Elektronenschalen ?

    Könnten hyp. “magnetische Monopole” bei der Entstehung von Elektronenschalen eine Rolle gespielt haben ?

    Wie entstand das Elektron und was veranlasst ein Elektron dazu eine bestimmte Schale zu besetzen ?

    Spielen beim Aufbau von Elektronenschalen Fermionen oder Bosonen eine tragende Rolle ?

    Warum kann eine Schale nur eine bestimmte max Anzahl von Elektronen aufnehmen ?
    Die Formel dazu lautet: 2*n^2.

    Was versteht man eigentlich unter einem “Teilchen” in der Physik ?
    Wie beschreibt der Physiker eigentlich ein “Teilchen” bzw. “Anti-” Teilchen ?

  599. #606 Bea
    Südliche Weinstraße
    30. März 2016

    Hallo Herr Freistetter!

    Ich habe eine Frage zum Frühlingsbeginn.
    Bisher dachte ich, dass Frühlingsbeginn und Tagundnachtgleiche zusammenfallen, aber wenn diese Sonnenaufs- und -untergangstabelle stimmt, war die TNG bereits am 15. März.
    https://www.sonnenaufgang-sonnenuntergang.de/?resulting=1&e=1&location=Berlin&longitude=13.404953999999975&latitude=52.52000659999999#.VvvZGr-0dBg
    Habe ich etwas Falsches geglaubt oder ist die Tabelle nicht korrekt?

    MfG, Bea

  600. #607 Bea
    30. März 2016

    Sorry, am 16. – vertippt.

  601. #608 H.R.
    30. März 2016

    Was sind Eich-Symmetrien ?

  602. #609 Micha
    unterwegs
    30. März 2016

    @Bea
    Schau dir einfach den dazugehörigen Wikipedia-Artikel an: https://de.wikipedia.org/wiki/%C3%84quinoktium

    Hier genauer den Unterpunkt “Equilux”:
    Mit „Equilux“ wird ein Kalendertag bezeichnet, an dem auf der Erdoberfläche bei idealem (mathematischen) Horizont die Belichtungsdauer, gemessen zwischen dem ersten Sonnenstrahl morgens und dem letzten Sonnenstrahl abends, genau zwölf Stunden betragen würde; diese Definition bezieht sich also auf den Rand der Sonnenscheibe, nicht deren Mitte. Das Datum des Equilux fällt daher nicht auf das Datum eines Äquinoktiums („Equinox“), sondern findet im Jahreslauf einige Tage vor dem Primar- bzw. nach dem Sekundaräquinoktium statt. Im Gegensatz zu den erdmittelpunktbezogenen und so weltweit gleichen Äquinoktien hängt das Equilux-Datum darüber hinaus jeweils vom Breitengrad des Standortes ab. Für den 40. Breitengrad liegt es um den 17. März bzw. den 26. September, für den 5. Breitengrad um den 25. Februar bzw. den 15. Oktober.[5]

    Die Sonne geht auf, wenn ihr oberer Rand über der Horizontlinie sichtbar wird, bevor also ihr Mittelpunkt erscheint. Der Sonnenuntergang ereignet sich, nachdem der Sonnenscheibenmittelpunkt scheinbar unter den Horizont gesunken ist, wenn der letzte Sonnenstrahl des oberen Sonnenrandes erlischt. Gegenüber einer punktförmigen Betrachtung der Sonnenmitte kommt damit je ein halber Durchmesserbogen der Sonne (etwa 0,25° bzw. 16′ Bogenminuten) hinzu. Außerdem bewirkt die Lichtbrechung durch die Erdatmosphäre jeweils eine scheinbare Anhebung der Sonnenscheibe (um etwa 0,6° bzw. 34′). Diese Verlängerung des lichten Tages auf Kosten der Nacht um knapp 7 Minuten (1,7 Winkelgrad × 4 Minuten/Winkelgrad) am Äquator (in Mitteleuropa um knapp 11 Minuten) wird bei der Bestimmung des Equilux berücksichtigt.

  603. #610 Alderamin
    30. März 2016

    @Bea

    Dafür gibt es zwei Gründe.

    Zur Tag- und Nachtgleiche steht die Sonne genau über dem Äquator und es liegen 12 Stunden zwischen den Zeitpunkten, zu denen der Mittelpunkt der Sonne auf dem geometrischen (Erde als perfekte Kugel, ohne Atrmosphäre gedacht) Horizont liegt. Sonnenauf- und untergangszeiten beziehen sich jedoch darauf, wann der erste bzw. letzte Lichtstrahl über den Horizont scheinen, also auf den oberen Rand der Sonne. Und der berührt den Horizont morgens früher und abends später als der Mittelpunkt der Sonne, der Tag ist also zur Tag- und Nachtgleiche schon mehr als 12 h lang.

    Und dann gibt es die Refraktion der Atmosphäre. Die Lufthülle der Erde wirkt ein wenig wie ein Prisma, das Lichtstrahlen umlenkt. Wenn die Sonne sehr tief steht, fällt ihr Licht streifend durch die Atmosphäre, die oben dünner als unten ist. Das lenkt die Lichtstrahlen ein wenig nach unten ab. Der Effekt ist, dass das Licht einen krummen Weg durch die Atmosphäre zurück legt und die Sonne höher zu stehen scheint, als sie es eigentlich tut (siehe auch diese Seite). Das geht so weit, dass die Sonne, wenn sie den Horizont unten berührt, geometrisch eigentlich schon komplett unterhalb des Horizonts stehen müsste (ein anderer Effekt ist, dass die Sonne abgeflacht erscheint, denn ihr unterer Rand wird stärker angehoben als ihr oberer). Somit tritt der Sonnenaufgang schon mehrere Minuten früher ein, als er rein nach der Geometrie stattfinden sollte, und der Untergang mehrere Minuten später.

    In Summe beider Effekte tritt der 12h-Tag schon mehrere Tage vor Frühlingsanfang ein und dann wieder erst mehrere Tage nach Herbstanfang. Die beiden Daten, an denen zwischen Auf- und Untergang genau 12 h liegen, nennen sich Equilux (“gleiches Licht”).

    Die Tabelle dürfte also korrekt sein. Passt auch ungefähr zu den Daten des Equilux im Wikipedia-Artikel (wobei sich Berlin ein gutes Stück nördlicher als die im Artikel angegebenen 40° befindet, das macht den Sonnenbogen flacher und verschiebt die Equilux-Daten noch ein wenig weiter von den Tag- und Nachtgleichen weg).

  604. #611 Bea
    30. März 2016

    Hallo Micha und Alderamin!

    Dh dann wohl, dass man mit Fug und Recht sagen kann, dass der Frühlingsbeginn NICHT mit der “wahren” TNG zusammenfällt, bzw, dass das Datum der TNG wohl mehr aus Gewohnheit, als aus astronomischer Exaktheit auf den Frühlingsbeginn datiert wird – oder?

  605. #612 Alderamin
    31. März 2016

    @Bea

    Vielleicht ist der Name “Tag- und Nachtgleiche” etwas unglücklich gewählt (zumal es ja bereits ca. 1h vor Sonnenaufgang bzw. nach Sonnenuntergang schon/noch hell ist), aber astronomisch gesehen ist die Tag- und Nachtgleiche (Äquinoktium) eben genau dann definiert, wenn die Sonne den Äquator überschreitet. Bei einer perfekt glatten Erdkugel ohne Atmosphäre und einer punktförmigen Sonne ginge der Erdschatten dann durch beide Pole entlang eines Längengrads um die Erde, d.h. für jeden Breitengrad wäre genau die Hälfte des Breitengrads beleuchtet und die andere Hälfte im Schatten der Nacht.

    Das ist genau die Mittelstellung zwischen den Positionen zu Sommeranfang (Erdachse zur Sonne hin geneigt) und Winteranfang (Erdachse von der Sonne weg geneigt), und es ging den Astronomen darum, für diese Position der Erdachse einen Namen zu vergeben. Ungefähr stimmt es ja auch. Als der Name geprägt wurde, gab es noch keine sekundengenauen Uhren und niemand hätte auf die Minute genau die Tageslänge messen können, sonst hätte man sich vielleicht etwas anderes als Namen einfallen lassen.

    Für 12 h zwischen Sonnenauf- und Untergang gibt’s ja mit “Equilux” schon eine eigene Bezeichnung.

  606. #613 Bea
    31. März 2016

    Dankesehr, Alderamin! 🙂

  607. #614 Higgs-Teilchen
    Im Standardmodell oben rechts
    7. April 2016

    Hi.

    Eine Verständnisfrage:
    Sind Elektronen nicht eigentlich Singularitäten?
    Sie haben eine Masse die sich auf einem Punkt konzentriert…..
    Was verhindert die Singularität?
    Oder verstehe ich da was ganz gewaltig falsch?

    Lg H.

  608. #615 H.A.B
    7. April 2016

    Kann mir vorstellen, dass im Zuge der weltbewegenden Ereignisse auch immer öfters kritischere Fragen gegen Astrophysik gestellt werden.

    Und zwar bezüglich der Kosten, Nutzen und Sinn der Astrophysik für die Wirtschaft, Politik und Jugend.

    * Hat Astrophysik in der heutigen Zeit eigentlich so noch eine Bedeutung, wenn diese der Jugend in direkter Folge keinen Arbeitsplatz bieten kann ?

    * Oder anders rum gefragt:
    Hat die Astronomie eigentlich jemals jungen Menschen eine Arbeit bieten können ?
    Sie hat ja bis zum heutigen Tag als Wissenschaft den Bereich der Akademien nicht verlassen und ist und bleibt seit eh und je ausschliesslich eine sehr teure elitäre Wissenschaft.

    * Warum soll z.b. Astronomie in den Grundschulen gelernt werden ?
    Heutzutage weiss jeder warum eine Sonnenfinsternis entsteht. Und jene Menschen die es nicht wissen, können sich sehr kostengünstig Videomaterial aus dem Internet besorgen. Die Kosten Astronomie zu Unterrichten überschreitet definitiv den Nutzen – (aus meiner Sicht heraus zumindest)

    * Die Grundschulen mit nicht praktisch verwertbarem Wissen zu belasten halte ich für nicht wirklich sinnvoll.
    Vor allem auch deshalb weil es an Fördergeldern (für wichtigere Dinge) fehlt um den normalen Schulbetrieb weiter zu führen.
    Ich kriege auch hier irgendwie das Gefühl nicht los, dass der Astronom versucht in den Grundschulen zu unterrichten umso (im Falle einer Wirtschaftskrise) selbst besser vor ihr geschützt zu sein.

    * Geht durch den massiven wissenschaftlichen Fortschritt und der zunehmenden weltpolitischen Problemen die Sinnhaftigkeit für Astronomie nicht selbst verloren ?

    * Wer profitiert eigentlich heute noch von wissenschaftlichem Fortschritt: Beispiel: Gravitationswellen-Astrophysik

    * Unter welchen wirtschaftlichen, politischen Umständen würde ein Astrophysiker selbst zur Erkenntnis kommen, dass es keinen weiteren Sinn mehr macht eine derartig teure Wissenschaft weiter zu führen ?
    Gibt es hier einen Break-Point der Einsicht ?

    Fragen solcher Art würde ich in Zukunft vermehrt stellen bzw. erwarten.

  609. #616 Spritkopf
    7. April 2016

    @H.A.B.

    * Wer profitiert eigentlich heute noch von wissenschaftlichem Fortschritt: Beispiel: Gravitationswellen-Astrophysik

    Vermute ich recht, dass Sie Anhänger der These sind, dass nur Forschung, die sich unmittelbar in klingender Münze auszahlt, ihre Berechtigung hat? Was, wenn Wissenschaftler vor 200 Jahren ihre Forschungen zur Elektrizität eingestellt hätten, bloß weil ihre Zeitgenossen nach der Sinnhaftigkeit, an Metallstifte angeschlossene Froschschenkel zucken zu lassen, gefragt haben? Dann würden Sie Ihre Fragen nicht einem Blog und damit im übertragenen Sinne der ganzen Welt stellen, sondern allerhöchstens mit Stift und Papier und Ihrem besten Freund.

    * Unter welchen wirtschaftlichen, politischen Umständen würde ein Astrophysiker selbst zur Erkenntnis kommen, dass es keinen weiteren Sinn mehr macht eine derartig teure Wissenschaft weiter zu führen ?
    Gibt es hier einen Break-Point der Einsicht ?

    Sie schreiben etwas von “Einsicht”, eine Wortwahl, die suggeriert, dass Sie uns – die wir Grundlagenforschung befürworten, weil man eben nur mit vielen Jahrzehnten Verzögerung erfährt, was bei Grundlagenforschung an Zählbarem herauskommt – gegenüber einen Erkenntnisvorsprung haben, der die Sinnlosigkeit eben dieser Grundlagenforschung einwandfrei belegt hätte. Darf ich fragen, wie Sie zu diesem Erkenntnisvorsprung gelangt sind?

  610. #617 PDP10
    7. April 2016

    @H.A.B:

    Ich nehme an. Sie stehen auch im Louvre vor der Mona Lisa und fragen sich, wofür das gut ist und wieso darum soviel Gewese gemacht wird?

    Wissenschaft? Kunst? Musik? Wieso sollten Schüler so etwas heute lernen?

    Es kommt schließlich auf Kompetenzen an, nicht wahr?

    Wer braucht schon Bildung …

    Ein braver Bürger, der stets nach der ökonomischen Verwertbarkeit von Allem – und selbstverständlich auch nach der seiner Selbst – fragt, sicher nicht.

    Viel Spass mit dem Rest Ihres Lebens.

  611. #618 Higgs-Teilchen
    Im Standardmodell oben rechts
    7. April 2016

    @H.A.B

    “Heutzutage weiss jeder warum eine Sonnenfinsternis entsteht”

    Naja, da wäre ich mir bei solchen Kandidaten hier nicht so sicher:
    https://de.webfail.com/689d57c6e24?tag=ffdt
    https://de.webfail.com/0561a130823?tag=ffdt
    https://de.webfail.com/bb793882705?tag=ffdt

    Lg H.

  612. #619 Captain E.
    8. April 2016

    Mir fällt da gerade die Postkarte ein, die ich vor etwa15-16 Jahren in einem Geschäft gesehen habe. Darauf gedruckt waren ein paar Sätze, die sich mit dem Mon beschäftigten. Unter anderem fand sich die Aussage, dass bei Neumond der Mond nicht zu sehen ist, weil er sich in diesem Zeitraum im Erdschatten befände. Noch Fragen?

  613. #620 Alderamin
    8. April 2016

    @Higgs-Teilchen #614

    Da sonst keiner was schreibt und obwohl Teilchenphysik nicht so mein Bereich ist: Welche Abmessungen das Elektron wirklich hat, kann man nicht genau genug messen, es erscheint halt in allen Messungen mit der erreichbaren Auflösung als Punkt. Das heißt aber nicht, dass es wirklich ein Punkt sein muss. Es könnte ja auch ein String sein. Oder so was. Uns fehlt dazu einfach noch die Theorie, und vor allem die Theorie der Quantengravitation, die die Wirkung der Schwerkraft auf kleinstem Raum beschreibt.

  614. #621 Theodor
    8. April 2016

    Hatte in diesem Blog bei einem andern Verfasser mal aufgeschnappt, das solche Teilchen wegen der Unschärferelation eher als Feld, ähnlich einer Temperaturkarte, anzusehen sind. Diese punktförmigen Darstellungen sollen nur dem Verständnis Ungeübter dienen.

    Muss aber leider gestehen, ob ich dies in diesem Zusammenhang richtig deute.

  615. #622 Theodor
    8. April 2016

    Und @ H.A.B

    Den Wert von Wissenschaft, Kunst, überhaupt Kultur kann man nicht an der Fingerbreite Wurst auf dem Brot messen.

  616. #623 PDP10
    8. April 2016

    @Theodor:

    Muss aber leider gestehen, ob ich dies in diesem Zusammenhang richtig deute.

    Nee, da liegst du schon richtig. Vor allem mit dem Wort “deute”. Alles was wir bis jetzt über die Dinger wissen sind eben nur “Deutungen” wie @Alderamin sinngemäss schon geschrieben hat.

    Der Theoretische Physiker Chris Quigg hat mal gesagt, er stelle sich Elektronen weder als Teilchen noch als Feld vor, sondern als kleine gelbe Kugeln. :-).

    Weiter sind wir halt noch nicht ….

  617. #624 Chemiker i.R,
    9. April 2016

    Frage: Wieso gibt es trotz Sublimation noch Eis (egal wovon) in unserem Sonnensystem? Fast jeder kennt den den Effekt der Gefriertrocknung – löslicher Kaffee, Gefrierbrand, Astronautennahrung…Zumal die Gasphase ja ständig durch den Sonnenwind “weggeschossen” wird (s. Marsatmosphäre), ein Gleichgewicht also auf die Gasphase verschoben wird?
    Anm. zu #615
    Endlich ein Beweis für Zeitreisen! Ein Urahn hat sich gemeldet!

  618. #625 Chemiker i.R,
    9. April 2016

    Ergänzung: Wenn ich mich nicht total verrechnet habe (Bezug SG Folge 166) hat Pluto seit seiner Entstehung rund 5% seiner Masse durch diesen Prozess verloren. Kein Problem für seine Umlaufbahn im PlanetenSystem.(s.Dreikörperproblem, Folge 175 SG). Problem aber für seine Begleiter (Monde?)?

  619. #626 Alderamin
    9. April 2016

    @Chemiker i.R,

    Frage: Wieso gibt es trotz Sublimation noch Eis (egal wovon) in unserem Sonnensystem?

    Weil’s weit genug weg von der Sonne so kalt ist, dass die Sublimation sehr langsam abläuft und die Objekte im äußeren Sonnensystem verdammt viel Eis haben. Noch viel flüchtiger als Wasser ist ja Stickstoff. Und für Pluto gilt beispielsweise folgendes (Wikipedia):

    Weakness of Pluto’s gravitation and high temperature of upper layers of its atmosphere causes quick atmospheric escape into outer space. The escape rate is estimated to be 1027–1028 molecules (50–500 kg) of nitrogen per second. It corresponds to loss of surface layer several hundred meters or several kilometers thick during lifetime of solar system

    Mehrere Kilometer Verlust ist bei einem Objekt von 2400 km Durchmesser nicht sehr viel. Enstprechendes gilt dann für Wasser auch näher an der Sonne, z.B. für den Eismond Europa.

  620. #627 Jens
    Langen
    19. April 2016

    Gibt es einen Zusammenhang zw.
    dem ersten Nachweis von Gravitationswellen und dem fast gleichzeitig registrierten Gammastrahlenausbruch?

    https://www.astronews.com/news/artikel/2016/04/1604-025.shtml

  621. #628 Alderamin
    20. April 2016

    @Jens

    Nichts genaues weiß man nicht (nicht einmal, ob der Gamma-Burst wirklich echt war, nur ein Instrument hat ihn gesehen), aber möglicherweise ja, z.B. so:

    https://www.cfa.harvard.edu/news/2016-05

  622. #629 Alderamin
    20. April 2016

    @Jens

    Mehr: https://cosmos4u.blogspot.de/2016/04/fermi-vs-wave-battle-of-press-releases.html

    Also ist weiterhin unklar, ob es den Burst überhaupt gab.

  623. […] der Serie “Fragen zur Astronomie” ist heute ein großer Klassiker an der Reihe. Eine Frage, die mir immer wieder gestellt wird: Wozu […]

  624. #631 Wolle
    Ratingen
    13. Mai 2016

    Hallo,
    ich habe mal eine vielleicht peinliche Anfängerfrage und hoffe auf eine aufklärende Antwort.

    Unsere Erde soll sich mit ca. 108.000 km/h durchs Weltall bewegen.
    Angenommen eine Rakete startet von der Erde mit ca. 28.000 km/h entgegengesetzt zur Flugbahn der Erde, so müsste sich die Rakete doch mit insgesamt 136.000 km pro Stunde von der Erde wegbewegen.
    Nun wird immer wieder von unseren bisher schnellsten Raketen mit ca. 50.000 km/h gesprochen.
    Würde ich z.B. mit einem Auto 100 km/h fahren, entfernt sich das entgegenkommende Fahrzeug mit ebenfalls 100 km/h von mir 200 km in der Stunde.
    Wo ist hier mein Denkfehler ?

    Wo ist hier mein Denkfehler ?

  625. #632 Alderamin
    13. Mai 2016

    @Wolle

    Unsere Erde soll sich mit ca. 108.000 km/h durchs Weltall bewegen.
    Angenommen eine Rakete startet von der Erde mit ca. 28.000 km/h entgegengesetzt zur Flugbahn der Erde, so müsste sich die Rakete doch mit insgesamt 136.000 km pro Stunde von der Erde wegbewegen.

    Du meinst in Richtung der Flugbahn der Erde. Ja, theoretisch hätte die Rakete dann eine auf die Sonne bezogene (“heliozentrische”) Geschwindigkeit von 136000 km/h. Wobei sie die Anziehungskraft der Erde wieder verlangsamen würde. Um die Erde dauerhaft zu verlassen, muss sie mit 11,2 km/s = 40300 km/h von der Erde gestartet werden. Dann würde sie auf einer Bahn ähnlich derjenigen der Erde mit rund 108000 km/h um die Sonne kreisen – falls man sie in Richtung der Erdbahn abschießt.

    Nun wird immer wieder von unseren bisher schnellsten Raketen mit ca. 50.000 km/h gesprochen.

    Diese Geschwindigkeit wird dann halt auf die Erde bezogen sein (“geozentrische Geschwindigkeit”), denn dies ist die Geschwindigkeit, die die Rakete selbst aufbringt. Tatsächlich wurden New Horizons mit 16,2 km/s = 58320 km/h von der Erde weggeschossen, schneller als jede Sonde zuvor. Heliozentrisch war sie aber nicht die schnellste, sondern das waren die Helios-Sonden, die 70,2 km/s = 252720 km/h relativ zur Sonne hatten, weil sie der Sonne nahe kamen und dabei durch die Schwerkraft der Sonne immer schneller wurden. New Horizons kam nur auf 23,2 km/s und wurde mit zunehmender Entfernung von der Sonne immer langsamer, bis sie sich im Vorbeiflug am Jupiter neuen Schwung von diesem holte (“Gravity Assist”).

    Ein paar weitere Zahlen finden sich hier.

    WIr halten also fest:
    – Die Geschwindigkeit kann sich auf die Erde beziehen, was sinnvoll zur Bestimmung der Leistung der Rakete ist, oder auf die Sonne, was mehr Sinn macht, wenn die Raumsonde weit weg von der Erde ist.
    – Die Geschwindigkeit hängt nicht nur von der Startrichtung ab, in der man die Raumsonde schießt, sondern auch von ihrer Sonnenentfernung; was sich der Sonne nähert, wird schneller, was sich von ihr entfernt, wird langsamer.
    – Die Geschwindigkeit kann durch Vorbeiflüge an Planeten geändert werden (erhöht, aber auch verringert).

  626. #633 Wolle
    Ratingen
    15. Mai 2016

    @ Alderamin #632

    dankeschön für die verständliche Antwort, welche mir sehr geholfen hat.

  627. #634 Wolle
    Ratingen
    15. Mai 2016

    @ Alderamin #632

    dankeschön für die verständliche Antwort, welche mir schon ein wenig geholfen hat, aber nun spinne ich meine Theorie mal weiter.
    Würde die Rakete in die entgegengesetzte Richtung starten und irgendwann aus dem Anziehungsfeld der Erde sein, entfernt sie sich doch auch mit 108.000 km/h plus ihrer eigenen Geschwindigkeit.

    Bei gleicher Flugrichtung zur Erde ergibt sich dann nicht auch das gleiche Ergebnis?
    Erdgeschwindigkeit plus Raketenspeed.

    LG, Wolle

  628. […] der Serie “Fragen zur Astronomie” geht es heute wieder mal um Teilchenphysik. An hypothetischen Partikeln herrscht in der […]

  629. #636 Wolle
    Ratingen
    16. Mai 2016

    @ Alderamin #632

    sorry, hab da noch was übersehen und war etwas zu schnell mit meiner Frage .

    Du hattest geschrieben:

    – Die Geschwindigkeit hängt nicht nur von der Startrichtung ab, in der man die Raumsonde schießt, sondern auch von ihrer Sonnenentfernung; was sich der Sonne nähert, wird schneller, was sich von ihr entfernt, wird langsame

  630. #637 Alderamin
    16. Mai 2016

    @Wolle

    Bin ganz Ohr Auge…

  631. #639 Hendrik
    Innsbruck
    14. Juni 2016

    Hi alle!
    Ich habe noch ein kleines Problem mit dem Verständnis des Higgs-Bosons bzw des Higgs-Feldes. Interesssanter Weise hat das Higgs-Teilchen ja eine Eigenmasse von 125 GeV. Wie ist das nun zu interpretieren? Warum und woher bekommt das Teilchen, welches für den Austausch der Masse zuständig ist, eine Eigemasse? Steht die Antwort im Zusammenhang damit, dass das Higgs-Feld das einzige Kraftfeld ist, welches Energie benötigt um zur Ruhe zu kommen?
    Lg
    Hendrik

  632. #640 Krypto
    14. Juni 2016

    @Hendrik:
    Das ist im Grunde die etwas schludrige Ausdrucksweise, die sich eingebürgert hat.
    Eigentlich geht es um Energien bzw. Massenäquivalenzen; ganz nach Einsteins berühmter Formel E=mc^2.
    Und die Energie des Higgs-Bosons beträgt 125 GeV/c^2. Weil der Nenner “c^2” eine Konstante ist, lässt man ihn meistens unter´m Tisch bzw. die Bruchkante fallen 😉

  633. #641 Krypto
    14. Juni 2016

    @myself: Ooops:
    Korrekt ist natürlich:
    Und die Energie des Higgsbosons entspricht einer Masse von 125 GeV/c^2

  634. #642 Braunschweiger
    18. Juni 2016

    Es gibt den Erdbegleiter 2016 HO3, der auch als Quasi-Mond bezeichnet wird. — Warum ist er kein richtiger Mond der Erde, sondern eigentlich ein Sonnensatellit?

    Ist das Argument lediglich, dass sein Durchschittsabstand zur Erde zu groß ist und seine zu erwartende Masse zu klein? Bedeutet dies, dass Masse, Abstand und Umlaufgeschwindigkeit nicht zu einem Erdtrabanten passen?

    Oder ist es sogar so, dass die Ebene seiner Umlaufbahn um die Erde nicht den Erdmittelpunkt enthält (und er also von der Sonne als anderer Masse abhängen muss), oder ergibt seine Erdumrundung noch nicht einmal eine eindeutige Ebene?

    Gehört er eigentlich zu den Erdtrojanern oder ist ein Lagrangepunkt-Begleiter? Und definieren derartige Objekte nicht eine ganz eigene Klasse von Satelliten (und damit Umlaufbahnen), die regelmäßig von zwei Meistern (d.h. Massen) bestimmt werden?

  635. #643 Alderamin
    18. Juni 2016

    @Braunschweiger

    Schau’ Dir mal den Artikel von Phil Plait an, da hat er auch eine Orbitsimulation verlinkt. Da sieht man, wie das Objekt die Sonne umkreist. Es bewegt sich anders als unser Mond außerhalb der Hill-Sphäre der Erde, innerhalb derer Objekte permanent von ihr festgehalten bleiben. Es ist aber auch kein Trojaner, die befinden sich bei +/- 60°auf der Umlaufbahn der Erde, was hier nicht der Fall ist. Ist irgendwie so ein Zwischending, scheint mir, eine Resonanz zwischen der Bahn des Objekts und der Erde. Und die ist wohl auch nicht von ewiger Dauer, wie man es für einen Satelliten der Erde erwarten würde.

  636. #644 AmbiValent
    20. Juni 2016

    @Braunschweiger

    Gäbe es keine störenden Einflüsse (zB von der Sonne), hätte ein Objekt, das die Erde in genau einem Jahr umkreist, etwa eine große Halbachse von 2,2 Mio km. Bei einer sehr exzentrischen Bahn mit Perigäum nahe an der Erde wäre noch ein Apogäum von fast 4,4 Mio km möglich. Alles, was noch weiter von der Erde weg ist, würde für eine Umrundung mehr als ein Jahr brauchen.

    2016 HO3 mit seiner scheinbaren Umlaufzeit von einem Jahr kommt aber nie so nah an die Erde heran, ist also ganz klar kein echter Mond.

  637. #645 Stefan
    26. Juni 2016

    Frage: Oortsche Wolke und unsere Nachbarsterne

    Hallo Florian,

    beim “Stöbern in der Oortschen Wolke” ist bei mir die Frage aufgekommen, ob es eigentlich um andere Sterne eine vergleichbare Wolke gibt.

    Insbesondere finde ich die Frage vor dem Hintergrund spannend, dass die Region der Oortschen Wolke mit zw. 0.8 und 3.2 Lichtjahren angegeben wird und der nächste Stern ja schon bei etwas über 4 Lichtjahren auf uns wartet. Hätte dieser eine ähnliche Wolke, wäre es ja nicht unwahrscheinlich, dass die Oortschen Wolken der beiden sich überschneiden. Ob es tatsächlich zu Interaktionen kommt, hängt sicherlich von vielen Parametern ab (Dichte, durchschnittlicher Abstand zweier Objekte etc. pp.), aber ähnlich wie beim Kolliedieren zweier Galaxien – könnte man hier ähnliche Gezeiteneffekte erwarten?

    Beste Grüße!

  638. #646 vortex
    7. Juli 2016

    Hallo Florian,

    Finde leider nur sehr wenig Informationen über primordial black holes und wollte fragen ob du dazu vllt etwas mehr weißt.

    Laut den LIGO Ergebnissen wäre es denkbar, dass die beobachteten Daten aus der Kollision zweier primordial black holes resultiert, deren Massen angeblich zwischen 20 und 100 Sonnenmassen liegen sollen.

    Der Nachweis von primordial black holes soll theoretisch via Hawking Strahlung möglich sein. Gleichzeitig wird aber argumentiert, dass (falls Hawking radiation doch nicht existiert) es sehr schwierig bis unmöglich wäre, die primordial black holes zu finden, da sie ziemlich klein wären und kaum gravitativ wechselwirken. Ich beziehe mich dabei auf diesen Wiki-Artikel, der momentan die einzige Quelle zu sein scheint die irgendwie irgendwas zu dem Thema erläutert:

    https://en.wikipedia.org/wiki/Primordial_black_hole#Possible_detection

    Ich stehe nun etwas auf dem Schlauch, denn wenn ich mich nicht irre kennen wir doch bereits SL die unterhalb 20 Sonnenmassen liegen, dh diese haben wir doch schon beobachten können.

    Und die gravitativen WW dieser SL mit ihrer Umgebung findet ja durchaus statt, dh primordial black holes die größer sein sollen müssten doch auffindbar sein bzw mit ihrer Umgebung wechselwirken – wenn ich das richtig verstehe, dann kann das also nicht der einzige Grund sein. Wo also liegt das Problem?

    Würde mich sehr über eine Antwort freuen.

    Liebe Grüße!

  639. #647 Alderamin
    8. Juli 2016

    @vortex

    Die primordial black holes, die man durch Hawking Strahlung heute finden könnte, müssten Massen von ursprünglich Asteroidenmasse (um 10^11 kg; die Sonne hat etwa 2*10^30 kg, die Erde rund 6*10^24 kg) haben, weil der Zerfall ansonsten im heutigen Alter des Universums noch nicht erfolgen könnte (stellare Schwarze Löcher legen derzeit schon durch die kosmische Hintergrundstrahlung mehr an Masse zu, als sie durch Hawking-Strahlung verlieren, denn die nimmt mit ihrer Masse ab). Solche Objekte können nicht aus Sternen entstanden sein, sondern müssten Relikte des Urknalls sein.

    Die Möglichkeit, dass dabei auch sehr massive Schwarze Löcher entstanden, ist auch gegeben, aber die würden wir nicht durch Hawking-Strahlung entdecken können, sondern nur durch die bei Kollisionen entstehenden Gravitationswellen, die Strahlung von Materie, die in sie hinein fällt, die Schwerkraftwirkung auf nahe Sterne, oder das Umlenken von Licht dahinter liegender Sterne (gravitational microlensing event).

  640. #648 Sophie
    Düsseldorf
    8. Juli 2016

    Warum vergeht die Zeit, wenn man sich nahezu mit Lichtgeschwindigkeit fortbewegt, langsamer?

  641. #649 AmbiValent
    9. Juli 2016

    Warum-Fragen sind bei solchen Themen schwierig zu beantworten – ich könnte sagen “weil es keinen absoluten Raum und keine absolute Zeit gibt, sondern nur die Raumzeit”, aber das ist wohl auch nicht besonders hilfreich.

    Wie-Fragen sind da leichter. Angenommen, ein Raumschiff fliegt von der Erde los, beschleunigt auf 99% Licht, fliegt nach Alpha Centauri und bremst dort wieder ab und landet auf einem Planeten.

    Dann sieht es für einen Beobachter auf der Erde so aus, als stände die Erde still, das Raumschiff bewegt sich mit 99% Licht, und die Zeit an Bord des Raumschiffs scheint langsamer zu vergehen. Man kann sogar exakt berechnen, wieviel langsamer, weil eine 100%ige Bewegung durch die Raumzeit herauskommen muss.

    Aus der Formel Raumbewegung^2 + Zeitbewegung^2 = 1 (entsprechend der Formel eines rechtwinkligen Dreiecks) erhält man 0,99^2 + x^2 = 1, daraus ergibt sich x ~ 0,14, also etwa 1/7, so dass die Zeit scheinbar 7mal langsamer vergeht. Nach etwas über 4 Jahren ist das Raumschiff am Ziel angekommen, und von den Signalen, die das Raumschiff von sich gibt, kann man annehmen, dass an Bord etwas über 7 Monate vergangen sind.

    Da alles relativ ist, sieht es für einen Beobachter an Bord des Raumschiffs anders aus. Auch der misst Bewegungen und Entfernungen, aber aus einer Perspektive, in der das Schiff stillsteht und sich die Erde mit 99% Licht entfernt… aber erstmal sollte ich fragen, ob das vorige verstanden wurde (ich hatte nur geantwortet, weil Florian und Alderamin sich noch nicht gemeldet hatten).

  642. #650 Alderamin
    11. Juli 2016

    @Sophie

    Wie Ambivalent schon sagte, Physik beantwortet keine “warum”-Fragen, die Antwort wäre: ist halt so. Man kann aber meistens physikalische Gesetze auf grundlegendere Sätze zurück führen. So auch hier.

    Der grundlegendste Satz der speziellen Relativitätstheorie lautet: Die Lichtgeschwindigkeit ist konstant. Sie erscheint jedem Beobachter immer gleich groß. Das weiß man, weil man es nachgemessen hat (Michelson-Morley-Experiment).

    Wenn etwa ein Beobachter in einem Raumschiff mit halber Lichtgeschwindigkeit unterwegs ist und er einen Lichtstrahl in Richtung seiner Bewegung aussendet, erscheint ihm dieser nicht etwa nur halb so schnell, wie die Lichtgeschwindigkeit c, sondern genau so schnell (bei Schallwellen vor einem Auto/Linienflugzeug oder Wasserwellen vor einem langsamen Ruderboot wäre das anders!). Und ein ruhender Beobachter, der den Lichtstrahl messen würde, würde ihn nicht mit 1,5 c messen, sondern auch mit c.

    Jetzt denke Dir mal eine Uhr, die darauf beruht, dass ein Lichtstrahl zwischen zwei Spiegeln auf und ab läuft. Immer, wenn der Strahl beim oberen Spiegel ankommt, ist ein Takt der Uhr vorbei. Ein Beobachter in einem schnellen Raumschiff würde den Strahl nur senkrecht auf- und ablaufen sehen (wenn man das könnte…), während sich das Raumschiff vorwärts bewegt. Ein Außenstehender würde hingegen sehen, wie sich das Raumschiff mit der Uhr fortbewegt und der Lichtstrahl eine Zickzack-Linie nehmen muss, um die sich fortbewegenden Spiegel zu treffen. Diese Linie ist länger, als die Senkrechte zwischen den Linien. Da dem Außenstehenden das Licht genau so schnell erscheint wie dem bewegten Beobachter, wird der Außenstehende sagen, die Uhr ticke langsamer, denn der Weg, den das Licht zwischen den Spiegeln zurücklegen muss, erscheint dem Außenstehenden länger, das Licht beiden aber gleich schnell. Längerer Weg mal gleiche Geschwindigkeit bedeutet längere Zeit.

    Wenn man sich jetzt noch beim Außenstehenden eine zweite solche Lichtuhr denkt, und sich überlegt, dass der bewegte Beobachter im Raumschiff auch für diese Uhr eine Zickzacklinie wahrnehmen würde, dann kommt man auf die merkwürdige Folgerung, dass dem bewegten Beobachter auch die Uhr des Außenstehenden verlangsamt erscheint, d.h. jeder von beiden sieht die Uhr des anderen langsamer vergehen! Das erscheint komplett widersinnig zu sein, ist aber unvermeidliche Schlussfolgerung aus der Konstanz der Lichtgeschwindigkeit.

    Solange die beiden ihre Uhren nicht nebeneinander stellen und die vergangene Zeit im gleichen Bezugssystem (gleiche Geschwindigkeit) vergleichen können, gibt es dadurch allerdings kein Problem. Für solch einen Vergleich müsste aber der bewegte Beobachter umkehren oder der Außenstehende ihm hinterher fliegen, dann würde sich ein Bewegungszustand ändern, und man hätte eine neue Situation, einen Wechsel des Bezugssystems, die auf das berühmte “Zwillingsparadoxon” führt.:

    Damit etwa der Außenstehende den Raumschiffinsassen einholt, muss er beschleunigen und schneller als dieser unterwegs sein, seine Uhr wird also aus Sicht eines dritten Außenstehenden eine Zeit lang noch langsamer vergehen, und wenn er das Raumschiff eingeholt hat, wird seine Uhr insgesamt weniger weit gelaufen sein. Aus Sicht des Raumschiffs (wenn sich dieses als stillstehend betrachtet, was in der Relativitätstheorie stets erlaubt ist, solange man seine Geschwindigkeit und Richtung nicht ändert) hätte sich der Außenstehende zunächst entfernt, dann die Richtung geändert und wäre zum Raumschiff zurück gekommen, und seine Uhr würde weniger Zeit zeigen.

    Letzteres wäre die symmetrische Situation dazu, dass der Außenstehende gewartet hätte, bis das Raumschiff zurück kehrt; in diesem Fall wäre dann die Raumschiffuhr insgesamt langsamer gegangen. Beim Zwillingsparadoxon zeigt also am Ende immer diejenige Uhr weniger Zeit an, die die Geschwindigkeit bzw. Richtung auf dem Weg zum Treffpunkt geändert hat.

    Aus der Konstanz der Lichtgeschwindigkeit kann man die ganze spezielle Relativitätstheorie ableiten: die Zeitverlangsamung, die Längenverkürzung, die Massenzunahme und die Äquivalenz von Masse und Energie (E=mc²). Die Antwort auf die Frage “warum vergeht die Zeit eines bewegten Beobachters langsamer” wäre also: “weil die Lichtgeschwindigkeit konstant ist.” Und warum ist die Lichtgeschwindigkeit konstant? Ist halt so. Darauf hat die Physik keine tiefer gehende Antwort.

  643. […] längerer Pause gibt es heute wieder einmal eine “Frage zur Astronomie”. Diesmal wird es apokalyptisch: Kann man den Mond auf die Erde fallen lassen? Und wie viel Energie […]

  644. #652 Multi
    5. August 2016
  645. #654 Multi
    10. August 2016

    Was versteht man eigentlich genau unter Quanten-Gravitation ? bzw. wieso braucht man überhaupt eine Quantisierte Gravitation ?

    Eigentlich müsste ja die normale Newtonsche Anschauung
    F = G * (m1 *m2)/r^2
    auch zw. 2 Atome gelten. Wenn ich als Masse die Atommasse einsetze, so müsste auch hier die entsprechende Gravitationskraft (als analoge Kraft) wirken, nur dass diese halt vernachlässigbar klein gegenüber der EMK wäre und somit nicht auffällt. Aber “da” müsste diese trotzdem sein.

  646. #655 Bullet
    10. August 2016

    @Multi:

    bzw. wieso braucht man überhaupt eine Quantisierte Gravitation ?

    Na ja, jetzt wird sowas ja nicht “erfunden”, weil man es “braucht”…die Quantisierung der elektromagnetischen Wellen war auch keine Frage des “Brauchens”… aber wenn man so tut, als sei elektromagnetische Energie kontinunierlich, stolpert man sofort über die Ultraviolettkatastrophe. Mithin ist die Welt eben nicht so simpel (= kontinuierlich). Und wenn das so für den Elektromagnetismus gilt, dann liegt der Verdacht nahe, daß auch die Gravitation von Quantisierung betroffen ist. Dann gibt es auch Effekte. Man muß sie nur erkennen. Einer davon ist, daß eine kontinuierliche Gravitation unendliche Werte zuläßt (wie eben bei genau der UV-Katastrophe in der Elektrodynamik). Ich glaube, das hätten wir bemerkt. 🙂

  647. #656 Multi
    10. August 2016

    Sagt die String-Theorie eigentlich eine quantisierte Gravitation vorraus ?

  648. #657 Alderamin
    10. August 2016

    @Multi

    Es ist wohl auch so, dass eine nicht-quantisierte Gravitation in Schwierigkeiten mit der Quantentheorie gerät und für allerkleinste Abstände unsinnige Ergebnisse liefert. Ich kenne da aber auch keine Details.

    Die String-Theorie sagt ein Teilchen voraus, das genau die Eigenschaften des Gravitons hätte, eines Teilchens, das in einer Quantengravitation die Schwerkraft vermitteln würde (so wie das Photon die elektromagnetische Kraft vermittelt).

  649. #658 guest
    12. August 2016

    Was ist der Unterschied zw.
    Warmer dunkler Materie und kalter dunkler Materie ?

  650. #659 Captain E.
    12. August 2016

    Vermutlich die Zusammensetzung, generell aber die Temperatur – und damit auch die Geschwindigkeit.

  651. #660 Alderamin
    12. August 2016

    Genau, die Geschwindigkeit. Denn ist die zu hoch (wie die von Neutrinos, die normalerweise mit fast Lichtgeschwindigkeit unterwegs sind), dann kann sie sich nicht zu Filamenten und Galaxienhaufen verdichten, weil die Teilchen schneller als die Fluchtgeschwindigkeit solcher Strukturen unterwegs sind. Teilchen, die sich nur langsam bewegen, können das hingegen. Da in einem (normalen, baryonischen) Gas die Teilchenbewegung direkt mit der Temperatur zusammenhängt, spricht man von “warmer” bzw. “heißer” und “kalter” Dunklen Materie – obwohl das eigentlich Unsinn ist, denn die DM kann keine Wärmestrahlung (elektromagnetisch!) abstrahlen und auch keine Wärme durch Stöße an normale Materie übertragen. Ist halt Astrophysiker-Jargon.

  652. #661 Alexander
    12. August 2016

    Was verrät uns der Umstand, dass SPON die Meldung “Wissenschaftliche Sensation: Mögliche zweite Erde in unserer Nachbarschaft entdeckt” unter der Rubrik “Astrologie” einordnet? 😉

    https://www.spiegel.de/wissenschaft/weltall/astrologie-erdaehnlicher-planet-beim-nachbarstern-entdeckt-a-1107405.html

  653. #662 PDP10
    12. August 2016

    @Alexander:

    Was verrät uns der Umstand, dass SPON die Meldung “Wissenschaftliche Sensation: Mögliche zweite Erde in unserer Nachbarschaft entdeckt” unter der Rubrik “Astrologie” einordnet?

    Dasselbe wie immer: Dass man Praktikanten, die mal irgendwas-mit-medien machen wollen, nicht für 3eurofuffzich-die-Stunde das Content-Management machen lassen sollte …

  654. #663 Alexander
    12. August 2016

    Die Panne wurde mittlerweile behoben. Ich hätte aber noch einen Screenshot!

  655. #664 PDP10
    12. August 2016

    @Alexander:

    Nich ganz. Guck mal auf den Titel ganz oben im Browser :-).

  656. #665 Alexander
    12. August 2016

    Bitte sehr: https://img5.fotos-hochladen.net/uploads/sponastrologievs9bcxu18p.jpg

    Das erinnert mich an einen Besuch an einem Standort eines sehr bekannten Herstellers optischer Instrumente. Der Herr, der uns durch den Betrieb führte, wusste zu berichten, dass vor Ort auch “astrologische Geräte” gefertigt würden …

  657. #666 PDP10
    12. August 2016

    Der Herr, der uns durch den Betrieb führte, wusste zu berichten, dass vor Ort auch “astrologische Geräte” gefertigt würden …

    Oh! Fertigen die etwa so was wie ein Astrolaberum? Oder gar Spiegelhoroskope? :-).

  658. #667 Wälchli Jürg
    CH-3033 Wohlen
    21. August 2016

    Frage: gibt es zwischen den Wendekreisen zwei Sonnenstände im Zenith und somit zwei längste Tage? Berechnung am Beispiel von Recife PE Brasilien? Vielen Dank.

  659. #668 Alderamin
    22. August 2016

    @Wälchli Jürg

    Nein, es gibt zwei Zenitstellungen, aber nur einen längsten Tag (um den 21. Dezember). Berechnung hier (einfach mal die Monate durchprobieren und auf die Tageslänge achten).

    Für die Tageslänge entscheidend ist nicht die Höhe des Sonnenstandes (am Äquator steht die Sonne im Sommer höher und die Tageslänge ist trotzdem kürzer als bei uns), sondern die Länge des Bogens, den die Sonne zieht. Und der ist dann am längsten, wenn die jeweilige Erdhalbkugel sich maximal der Sonne zuwendet. Recife liegt südlich des Äquators, also ist das zur Wintersonnenwende (auf Europa bezogen) der Fall.

    Wenn nicht klar ist, warum der Bogen dann länger ist, denke Dich an die Stelle der Sonne und schaue auf das gedachte Gradnetz der Erde, wenn sich einer ihrer Pole Dir zuwendet. Die Erdhälfte, die sich Dir zuneigt, zeigt mehr von ihren Breitenkreisen als die Hälfte auf der anderen Seite des Äquators. Um den Pol sieht man sogar komplette Breitenkreise, zum Äquator hin werden die Bögen kürzer, vom Äquator sieht man genau die Hälfte.

    Am Äquator sind die Tage deshalbe das ganze Jahr hindurch gleich lang. Siehe etwa Quito in Ecuador. Die Tageslänge schwankt nur um ein paar Minuten.

  660. #669 Wälchli Jürg
    28. August 2016

    Vielen Dank. Noch eine Frage eines Unwissenden: Warum wird für Recife die längste Sonnenscheindauer im November angegeben (anscheinend Monatsmittel)?

  661. #670 Alderamin
    28. August 2016

    @Wälchli Jürg

    Könnte an der Bewegung der Erde um die Sonne liegen. Im (Nord-) Winter ist die Erde der Sonne näher und bewegt sich auf ihrer Bahn wegen des zweiten Keplerschen Gesetzes in 24h weiter als im (Nord-) Sommer, d.h. die Sonne bewegt sich scheinbar am Sternenhimmel schneller nach Osten, und die Erde muss sich deswegen etwas weiter um sich selbst drehen als im Sommer, um die Sonne einzuholen (damit sie z.B. mittags auf dem Meridian steht). Außerdem wandert die Sonne im Frühjahr und Herbst relativ zum Sternhimmel etwas schräg, nach Norden bzw. Süden, während sie im Sommer und Winter parallel zu den Breitenkreisen wandert und sich somit schneller nach Osten verschiebt. Beides führt in Kombination dazu, dass die Tageslänge von Mittag zu Mittag nicht immer genau 24h beträgt, die Sonnenuhr kann bis zu 15 Minuten falsch gehen. Vermutlich verschiebt dies den längsten Tag gegenüber dem Datum des Solstitium (Sonnenwende am 21.12.).

    Siehe auch https://de.wikipedia.org/wiki/Zeitgleichung

  662. #671 Jens
    31. August 2016

    Bisher wurden von Ligo zwei Gravitationswellenereignisse gemessen die durch die Verschmelzung von jeweils zwei stellaren Schwarzen Löchern hervor gerufen wurden, die über 1 Mrd Lichtjahre entfernt waren. Kann Ligo auch Gravitationswellen messen die bei der Verschmelzung von supermassiven Schwarzen Löchern entstehen?

  663. #672 AmbiValent
    31. August 2016

    @Jens

    Diese Gravitationswellen wären viel stärker und entsprechend leichter zu messen – aber supermassive Schwarze Löcher sind schon um viele Größenordnungen seltener als stellare, und eine Verschmelzung wäre noch viel seltener – nicht im Sinne von “Es passiert nicht”, sondern im Sinne von “Es passiert irgendwann in etlichen Millionen bis Milliarden Jahren”.

  664. […] der Serie “Fragen zur Astronomie” geht es heute mal nicht um eine konkrete Frage zu einem astronomischen Thema. Sondern um ein etwas […]

  665. #674 Jens
    10. September 2016

    Noch eine Frage zu Gravitationswellen:
    Erfahren Gravitationswellen so etwas wie eine Rotverschiebung? Oder anders formuliert hat die kosmische Expansion Einfluss auf die Wellenlänge der Gravitationswellen?

  666. #675 Alderamin
    10. September 2016

    @Jens

    Selbstverständlich. Die Raumexpansion dehnt den Raum und alles, was durch Kräfte nicht aneinander gebunden ist. Licht genauso wie Gravitationswellen. Deswegen ist man bei der Suche nach Gravitationswellen des Urknalls auf der Suche nach sehr langwelliger Strahlung, für die es ein Weltraumteleskop wie eLisa braucht.

  667. #676 Rai
    14. September 2016

    *Ausdehnung des Raums vs. Annäherung durch Gravitation*

    das wurde ja am Beispiel der Andromeda Galaxie recht gut erklärt (Gravitation gewinnt, wird aber aufgrund der Expansion nicht voll wirksam).

    Etwas ähnliches müsste doch auch in anderen Zusammenhängen passieren: Wenn ein sich ein Himmelskörper um eine eine Sonne bewegt, dann müsste der Bahnabstand von der Sonne aufgrund der Raumexpansion etwas größer sein, als wenn die Bahn nur durch die Gravitation allein bestimmt wäre.

    Falls das der Fall ist, wie große ist dieser Effekt z. B. bei der Erdbahn und wie wird er berechnet?
    Oder gibt es hier einen Denkfehler?

    Vielen Dank und Grüße

    Rai

  668. #677 Andreas Gerber
    Winterthur
    18. Oktober 2016

    Guten Abend Herr Freistätter

    Wie kann ich’s mir vorstellen, dass mehrere Millionen Kelvin heisses Gas in Galaxien oder Galaxienhaufen nicht sofort abfliesst oder durch Abstrahlung von irgendwas in kürzester Zeit abkühlt? Und wie wird dessen Existenz nachgewiesen?

    Freundliche Grüsse und herzlichen Dank!

  669. #678 AmbiValent
    19. Oktober 2016

    Eine Frage zur Allgemeinen Relativitätstheorie:

    Angenommen, ein Lichtstrahl/Lichtsignal geht knapp an einem kompakten Objekt vorbei (trifft es also nicht).
    Wäre der Weg dieses Lichtsignals durch die Raumzeit exakt symmetrisch zum Punkt der größten Annäherung an das Objekt?

    Wer könnte diese Frage sicher beantworten?

  670. #679 AmbiValent
    19. Oktober 2016

    Ich habe in einem Fachbuch sogar eine Formel gefunden, aus der meines Erachtens diese Symmetrie folgt. (Wen es interessiert – Thorsten Fließbach, Allgemeine Relativitätstheorie, Kapitel 26 “Lichtablenkung”, Formel 26.5) Es ist eine exakte Formel, und die Terme sind abstandsabhängig, was schon eine solche Symmetrie suggeriert.

  671. #680 Alderamin
    20. Oktober 2016

    @Andreas Gerber

    [1] Wie kann ich’s mir vorstellen, dass mehrere Millionen Kelvin heisses Gas in Galaxien oder Galaxienhaufen nicht sofort abfliesst oder [2] durch Abstrahlung von irgendwas in kürzester Zeit abkühlt? [3] Und wie wird dessen Existenz nachgewiesen?

    ad 1): es fließt ja ab: Supernovae erzeugen solches Gas und es breitet sich aus, wird aus der Galaxie herausgeblasen, aber das dauert Millionen Jahre für eine bestimmte Supernova, und die explodieren andauernd irgendwo in der Galaxie. Oft rammt das heiße Gas aber auch in kühleres umgebendes und wird dort gebremst und abgekühlt.

    ad 2): Abstrahlung gibt’s nur, wenn die Teilchen des Gases mit anderen Teilchen wechselwirken. So lange sie einfach geradeaus mit hoher Geschwindigkeit durch den Raum rasen (hohe Geschwindigkeit = hohe Temperatur) ohne mit anderen Teilchen zu kollidieren, ändert sich ihre Geschwindigkeit nicht, und dann strahlen sie nicht (nur beschleunigte Ladung strahlt). Erst durch Stöße oder Ablenkung durch andere Teilchen kann es zur Abstrahlung von elektromagnetischen Wellen kommen. Ist das Gas dünn genug, passiert das hinreichend selten und das Gas kühlt kaum ab.

    ad 3): Natürlich stoßen einige wenige Teilchen dann doch zusammen und dies führt zur Aussendung von Röntgenstrahlung, die man messen kann. Anhand der Wellenlänge dieser Strahlung kann man die Temperatur ermittlen. Ist übrigens genau so bei der Sonnenkorona, auch die hat Millionen K. und strahlt im Röntgenbereich.

  672. #681 Bernhard
    Harz
    21. Oktober 2016

    Es geht um dunkle Materie. In diesem Zusammenhang wurden Neutrinos erwähnt. Soweit sogut. Aus einem unbekannten Grund formte sich mir folgender Gedanke:

    – Alles im (bekannten) Universum ist in Bewegung. Kein Objekt ist statisch bzw. stationär.
    – Bewegung hat eine enge Beziehung zu dem Phänomen Zeit.
    – Zeit wiederum ist unabdingbare Voraussetzung jeglicher Existenz.

    Fest steht, das Neutrinos alles durchdringen, und nur in den allerseltensten Fällen mit der “Normalmaterie” reagieren. Nun meine Frage: Könnte es sein, das die Neutrinos stationär sind und lediglich die sichtbare Materie da quasi hindurchfliegt? Mir ist zwar klar, das Neutrinos in Sonnen u.a. entstehen, welche sich ja bewegen – aber könnte es nicht sein, das diese Erzeugung dazu führt, das diese Neutrinos einfach so da liegenbleiben, während die Sonne sich ja weiterbewegt?
    Ich frage deswegen, weil dies ja ein Ansatz sein könnte, die ganze Sache (dunkle Materie u.a.) mal grundsätzlich anders zu denken – und das es ja vielleicht möglich wäre, eine Art Gegenstück zu der Schlüsselformel E=Mc² sich vorstellen zu können.

    Und da wäre der Grundgedanke: Existenz gibt es nur durch Reaktionen der Materie untereinander – mithilfe des Mediums Zeit. Zeit wäre meinen Vorstellungen nach das Bindeglied der “Welt der sichtbaren” zur “Welt der nicht sichtbaren” Materie.

    Sichtbar=dynamisch=positiv; Nicht sichtbar=statisch=negativ; Zeit= Neutrum, Katalysator.
    Hier wäre ja dann die Reaktion positiv zu negativ dann etwas, was etwas anstösst/bewirkt. Das also zwei “Systeme” sich untereinander bedingen und untrennbar verbunden sind.
    Ob dieser Ansatz gegen irgendwelche Grundsätzlichkeiten oder Axiome verstösst, müsste dann geklärt werden, weil der “perpetuum mobile” ja geradezu ketzerich wäre, oder?!
    Jedenfalls ist es ein gedanklicher Ansatz, der ungewöhnlich, abseits von Mainstream ist.

  673. #682 AmbiValent
    21. Oktober 2016

    @Bernhard
    Nein, es kann nicht sein, dass Neutrinos einfach an Ort und Stelle bleiben, während die Sonne, in der sie entstanden ist, sich weiterbewegt. Zum einen werden Neutrinos von der Sonne ständig gemessen, unabhängig davon, wo die Erde in Bezug auf die Bewegungsrichtung der Sonne steht. Zum anderen wäre die Geschwindigkeit von Neutrinos relativ zum Stern sehr langsam, während man tatsächlich fast Lichtgeschwindigkeit misst.

  674. #683 Alderamin
    21. Oktober 2016

    @Ambivalent

    Was die Ablenkung des Lichts angeht, das folgt doch einer Geodäte, und eine Geodäte ist stets der kürzeste Weg durch die Raumzeit, egal in welcher Richtung. Wenn Du es genauer wissen willst, frag’ Martin Bäker.

  675. #684 Theodor
    23. Oktober 2016

    Hallo, welche Farbe hat ein Neutronenstern?

    Abgenommen, man würde ein Teleskop in die Nähe eines Neutronenstern schicken. Was würde man dort sehen? Oder ist es aufgrund der dortigen Energien gar nicht möglich, ein für Menschen erkennbares Bild zu liefern?

  676. #685 Alderamin
    23. Oktober 2016

    @Theodor

    Neutronensterne sind üblicherweise extrem heiß, das heißt, sie leuchten bläulich weiß. Auf der Oberfläche würde man nichts erkennen, da sie rasend schnell rotieren. Eventuell würde man nur zwei helle Streifen sehen, wo die magnetischen Pole liegen, die einen Pulsar pulsieren lassen. Durch die schnelle Rotation und die meist nicht mit der Rotationsachse zusammen fallende Ausrichtung der Magnetachse würden diese hellen Punkte für das Auge zu Bändern verschmiert.

  677. #686 Captain E.
    24. Oktober 2016

    @AmbiValent:

    Nein, es kann nicht sein, dass Neutrinos einfach an Ort und Stelle bleiben, während die Sonne, in der sie entstanden ist, sich weiterbewegt. Zum einen werden Neutrinos von der Sonne ständig gemessen, unabhängig davon, wo die Erde in Bezug auf die Bewegungsrichtung der Sonne steht. Zum anderen wäre die Geschwindigkeit von Neutrinos relativ zum Stern sehr langsam, während man tatsächlich fast Lichtgeschwindigkeit misst.

    Tja, wahrscheinlich ist das so, aber faszinierend ist die Vorstellung schon. Und dann käme die Expansion des Raumes hinzu….

    Aber da die Erde ja um die Sonne kreist, müsste man vermutlich periodische Schwankungen im Neutrinofluss messen, oder? Wenn die Sonne auf ihrer Bahn um das Zentrum der Milchstraße (und mit der zusammen in Richtung Virgo) die Neutrinos einfach zurück ließe, dann dürfte es “vor” der Sonne (in Bahnrichtung gesehen) keine solaren Neutrinos geben. Mit ähnlichen Überlegungen und Messungen hat man doch den Äther ausgeschlossen, oder?

  678. #687 Noonscoomo
    Berlin
    24. Oktober 2016

    @CaptainE
    Richtig. Z.B das Michelson-Morley-Experiment
    https://de.m.wikipedia.org/wiki/Michelson-Morley-Experiment

  679. #688 Loffti
    Oberhausen
    24. Oktober 2016

    Ich habe heute in der “Sterne und Weltraum”, Ausgabe 11/2016, auf Seite 14, einen Bericht über den galaktischen Halo gelesen, der unsere Milchstraße umgibt. Er besteht, nach heutiger Meinung, aus einem 2 Millionen Grad Celsius heißem Gas und rotiert in die selbe Richtung wie die Milchstraße. Zur Frage: Wie kann ein Gas mit einem derart großen Umfang dauerhaft so viel Energie erzeugen?

  680. #689 Alderamin
    24. Oktober 2016

    @Loffti

    Das Gas ezeugt die Energie nicht, es kühlt nur (sehr langsam) ab. Es stammt aus Supernovae in der Milchstraße und hat diese durch seine hohe Teilchengeschwindigkeit Richtung Halo verlassen, wo es dann langsam abkühlt, bis es zurückfällt.

    Siehe oben #680 und https://astro.uni-bonn.de/~deboer/astraum/mwh.html, letzter Abschnitt.

  681. #690 imNetz
    2. November 2016

    Eine Frage zu diesem Artikel:

    https://www.spiegel.de/wissenschaft/natur/parkhaus-struktur-in-zellen-und-neutronensternen-verblueffende-aehnlichkeit-a-1119384.html

    Ist dies da oben geschildertefür Astronomen eines Kommentars würdig oder nur “Schmirgel”?

    Merci

  682. #691 Bernhard
    Harz
    3. November 2016

    Eigentlich schon. Spontan fällt mir da ein das es schon seit langer Zeit solche Vorstellungen gibt, wo es ähnliche Strukturen gibt im Mikro, Normal und Makro Bereich (oder Universum).
    Ausserdem ist es ein postives Zeichen, was da gesetzt wird, nämlich fachübergreifende Verständigung/Anregung. Das ist deshalb wichtig, weil fachspezifisches bzw. Spezialisten-Denken (und zwar in allen Bereichen des Lebens) stark zu sog. “Betriebsblindheit” neigen.
    Man muss nicht zwingend etwas “verwertbares” oder “brauchbares” voraussetzen, um Erkenntnisprozesse zu beleben bzw. anzustossen. Manches “Heureka!” Ereignis fußt des öfteren auf Zusammenhänge ausserhalb einer Fachdisziplin.

    Insoweit ist dieser Artikel positiv, auch wenn er nicht zielführend oder ergebnisorientiert zu sein scheint.

  683. #692 Sven
    Hamburg
    19. November 2016

    Hallo,

    ich frage mich gerade, ob es so etwas wie die Monddämmerung gibt. Sprich: Der Mondhöhenwinkel beläuft sich auf -4,57°. Ist der Nachthimmel bei zunehmend 94,3% wirklich dunkel oder gibt es so etwas wie Streulicht?

    Ich bin gespannt.

  684. #693 gcorp
    Wiesbaden
    19. November 2016

    Stephen Hawking hat vor kurzem gesagt, dass die Menschheit nur noch ungefähr 1000 Jahre auf der Erde überleben kann.

    Ich fände einen Beitrag darüber total spannend. 🙂

    (Ist das richtig hier? Ich wusste nicht, wo ich die Idee sonst posten soll.)

  685. #694 AmbiValent
    26. November 2016

    @Sven

    Hallo,
    ich frage mich gerade, ob es so etwas wie die Monddämmerung gibt. Sprich: Der Mondhöhenwinkel beläuft sich auf -4,57°. Ist der Nachthimmel bei zunehmend 94,3% wirklich dunkel oder gibt es so etwas wie Streulicht?
    Ich bin gespannt.

    Da ist es wie bei Radio Eriwan: “Im Prinzip ja, aber…”.

    Bei der Sonne kann man das Licht der Sonne selber sehen, das gestreute Licht am Tageshimmel, und das Rest-Streulicht bei der Dämmerung.

    Beim Vollmond kann man bei einem ausreichend klaren Himmel und bei kaum menschlichem Streulicht noch außer dem Mond selbst noch das Streulicht des Monds in der Atmosphäre sehen – der Himmel erscheint dann nicht ganz schwarz, sondern in einem sehr dunklen Blau, und schwächere Sterne, die man in mondlosen Nächten noch sehen konnte, nimmt man dann nicht mehr wahr.

    Das Rest-Streulicht des Monds, wenn der Mond noch unterhalb des Horizonts steht, ist dagegen im Prinzip noch vorhanden, aber nicht mehr hell genug, um es wahrzunehmen.

  686. #695 Sven
    29. November 2016

    @AmbiValent: Klasse! Vielen Dank für die ausführliche Antwort!

  687. #696 Peter Paul
    Winnenden
    4. Dezember 2016

    Hubbles Messungen waren grandios falsch, sonst hätte er nicht den ca. 8-fachen Wert der Hubble-Konstante herausgekriegt. War es reiner Zufall, dass er bei dieser Fehlergrößenordnung überhaupt eine Proportionalität feststellte, oder was steckt dahinter?

  688. #697 Peter Paul
    Winnenden
    4. Dezember 2016

    Wieso können wir uns nur eine dreidimensionale Welt räumlich vorstellen, obwohl die Raumzeit, also die Welt, wie sie ist, vierdimensional ist?

  689. #698 Captain E.
    4. Dezember 2016

    Vielleicht weil die vierte Dimension die Zeit ist? Die Berechnung ist recht einfach, aber eine Vorstellung erlangt man dadurch nicht.

  690. #699 Peter
    Dresden
    23. Dezember 2016

    Ich habe folgende Frage: Welche Nacht ist die längste im Jahr, und wie kann man das ermitteln/berechnen?
    Es ist ja bekannt, dass der Tag der Wintersonnenwende (dieses Jahr der 21.12.2016) der kürzeste Tag des Jahres ist. Aber welche Nacht ist die längste des Jahres?
    Ist es in jedem Jahr die Nacht vor der Wintersonnenwende (z. B. vom 20. zum 21.12.2016)?
    Oder ist es in jedem Jahr die Nacht nach der Wintersonnenwende (z. B. vom 21. zum 22.12.2016)?
    Oder sind beide Nächte bis auf die Sekunde gleich lang?
    Oder ändert sich der Zeitpunkt der längsten Nacht von Jahr zu Jahr, d. h. ist in manchen Jahren die Nacht vor dem Tag der Wintersonnenwende am längsten, u. in anderen Jahren ist die Nacht nach der Wintersonnenwende am längsten?
    Wie ist es mit der Sommersonnenwende: Ist immer die Nacht vor o. immer die Nacht nach der Sommersonnenwende die kürzeste, o. wechselt dies von Jahr zu Jahr?
    Laut Wikipedia u. anderen Quellen findet die Wintersonnenwende am 21.12.2016 um 11:44 MEZ statt. Da diese Uhrzeit näher an 0:00 Uhr als an 24:00 Uhr liegt, könnte man vermuten, dass die Nacht vom 20. zum 21.12.2016 geringfügig länger ist als die Nacht vom 21. zum 22.12.2016. Allerdings weicht die MEZ ja oft von der wahren Ortszeit ab, z. B. laut https://www.heret.de/funkuhr/ortszeit.htm für Dresden um -05:05 (min/sec.), das heißt 11:44:00 MEZ entspricht 11:39:05 wahrer Ortszeit.
    Gibt es im Internet eine Seite, wo die Nachtlänge (auf die Sekunde) für einen beliebigen Ort genau angezeigt wird?
    Ich habe bisher nur Websites gefunden, auf denen die Zeiten von Sonnenaufgang und Sonnenuntergang angezeigt werden, allerdings werden diese Zeiten nur auf die Minute und nicht auf die Sekunde genau angegeben.
    Z. B. für Dresden 2016:
    20.12. Sonnenunt. 15:59 Uhr, 21.12. Sonnenauf. 8:05 Uhr, Nachtlänge 16 h 06 min.
    21.12. Sonnenunt. 16:00 Uhr, 22.12. Sonnenauf. 8:06 Uhr, Nachtlänge 16 h 06 min.
    22.12. Sonnenunt. 16:00 Uhr, 23.12. Sonnenauf. 8:06 Uhr, Nachtlänge 16 h 06 min.
    Durch diese nicht auf die Sekunde genauen Zeitangaben kann man nicht berechnen, welche Nacht für einen bestimmten Ort denn nun die längste ist.

  691. #700 David Baum
    31. Dezember 2016

    Ich habe eine Frage zum Dreikörperproblem, das von dir im Blog und im Sternengeschichten-Podcast ja schon mehrfach diskutiert wurde. So wie ich es verstanden habe, kann es per se mathematisch nicht exakt gelöst werden. In der Folge 212 sagst du, dass das verdeutlicht, wie chaotisch die Bewegung von Himmelskörpern erfolgt. Aber “chaotisch” heißt lapidar doch so etwas wie “aus kleinen Abweichungen des Anfangszustandes folgen große Änderungen”, oder? Wenn es aber keine exakte Lösung gibt, dann ist die Bewegung der Körper ja nicht deterministisch. Ist das so? Wenn wir mehrmals die gleiche Konstellation haben (gleiche Massen, Koordinaten, Geschwindigkeiten…) wäre die weitere Bewegung der Körper dann tatsächlich jedes Mal anders? Oder wie löst das Universum das Dreikörperproblem, wenn es doch deterministisch ist?

    Danke und einen guten Rutsch. 🙂

  692. #701 schufti
    Wien
    1. Januar 2017

    Hallo!
    Liege ich richtig mit der Aussage:
    Erfolgen die Rotation eines Planeten und der Mondumlauf in entgegengesetzte Richtungen, so werden der Mondumlauf als auch die Planetenrotation abgebremst.
    Wenn das stimmt wo kommt dann der Drehimpuls hin?

  693. #702 Alderamin
    2. Januar 2017

    @Peter

    Ich habe folgende Frage: Welche Nacht ist die längste im Jahr, und wie kann man das ermitteln/berechnen?
    Es ist ja bekannt, dass der Tag der Wintersonnenwende (dieses Jahr der 21.12.2016) der kürzeste Tag des Jahres ist. Aber welche Nacht ist die längste des Jahres?
    Ist es in jedem Jahr die Nacht vor der Wintersonnenwende (z. B. vom 20. zum 21.12.2016)?
    Oder ist es in jedem Jahr die Nacht nach der Wintersonnenwende (z. B. vom 21. zum 22.12.2016)?

    Es ist diejenige Nacht die längste, die am nächsten zum exakten Zeitpunkt der Wintersonnenwende liegt, als die Sonne ihren Tiefststand erreicht hatte. Das war am 21. Dezember um 10:44 UTC, das wäre 11:44 MEZ, ziemlich exakt mittags. Daher war die Nacht davor und die Nacht danach praktisch identisch gleich lang gewesen. Wenn der Zeitpunkt der Sonnenwende deutlich vormittags gelegen hätte, wäre es eher die Nacht davor gewesen, bei einer Sonnenwende deutlich am Nachmittag die Nacht danach. An anderen Orten der Welt gelten andere Uhrzeiten, etwa lag dieser Zeitpunkt in den USA noch am frühen Morgen, und da war dann tatsächlich die Nacht vom 20. auf den 21. Dezember die längste. In China wäre es hingegen die Nacht vom 21. auf den 22. Dezember gewesen.

    Der Zeitpunkt der Wintersonnenwende fällt meistens auf den 21. Dezember, aber nicht immer; das schwankt ein wenig, weil ein Jahr nicht exakt 365 Tage hat, sondern 365,24219… Tage. Deswegen gibt es Schaltjahre, die (normalerweise) alle 4 Jahre das Datum wieder auf die Sonnenwenden synchronisieren. Somit rutscht der Sonnentiefststand von Jahr zu Jahr um ungefähr 6 h nach hinten, bis ein Schaltjahr ihn wieder um 24 h nach vorne holt.

    Wie ist es mit der Sommersonnenwende: Ist immer die Nacht vor o. immer die Nacht nach der Sommersonnenwende die kürzeste, o. wechselt dies von Jahr zu Jahr?

    Genau so. Sommersonnenwende ist genau ein halbes Jahr nach der Wintersonnenwende. Ein halbes Jahr sind 365,24219…/2 Tage = 182,621095 Tage = 182 Tage 14h 54m 23s.

    Allerdings weicht die MEZ ja oft von der wahren Ortszeit ab, z. B. laut https://www.heret.de/funkuhr/ortszeit.htm für Dresden um -05:05 (min/sec.), das heißt 11:44:00 MEZ entspricht 11:39:05 wahrer Ortszeit.
    Gibt es im Internet eine Seite, wo die Nachtlänge (auf die Sekunde) für einen beliebigen Ort genau angezeigt wird?

    Ist korrekt, streng genommen muss man die wahre Ortszeit zugrunde legen, inklusive der Zeitgleichung (die Sonnenuhr geht manchmal vor oder nach, siehe unten). Ja, es gibt eine solche Seite: https://www.timeanddate.com/sun/

    Die Tageslänge dort ist auf die Sekunde genau, die Auf- und Untergangszeiten der Sonne nur auf die Minute. Außerdem bezweifle ich, dass dabei der Einfluss der Atmosphäre oder der Sonnenausdehung berücksichtigt sind; vermutlich rechnen die mit dem Zeitpunkten, wo der Mittelpunkt der Sonne bei nicht vorhandener Atmosphäre (die die Sonne um ca. 0,5° anhebt) auf der Oberfläche einer perfekten Kugel (oder einem der Erdform angenäherten Ellipsoid) von Erdgröße den Horizont schneidet. Das ist dann alles nur auf ca. 1 Minute genau.

    Laut der oben angegebenen Seite war für Dresden die Tageslänge am 20.12. 7h54m19, am 21. 7h54m16 und am 22. 7h54m20. Demnach war die Nacht vom 20. auf den 21.12. 1 s länger als die Nacht von 21. auf den 22.12. Wegen der oben angesprochenen Relevanz der Atmosphäre, ja des Wetters, der Erdform, der Lage des Horizonts usw. ist dieser winzige Unterschied allerdings rein akademisch und für die Praxis vollkommen irrelevant.

  694. #703 Alderamin
    2. Januar 2017

    @schufti

    Liege ich richtig mit der Aussage:
    Erfolgen die Rotation eines Planeten und der Mondumlauf in entgegengesetzte Richtungen, so werden der Mondumlauf als auch die Planetenrotation abgebremst.
    Wenn das stimmt wo kommt dann der Drehimpuls hin?

    Beinahe: auch wenn der Mond den Planeten schneller umrundet, als dieser sich in der gleichen Richtung dreht, wird der Mond verlangsamt. Es ist aber richtig: wenn der Mond in die entgegengesetzte Richtung den Planeten umläuft, in der jener rotiert, dann wird der Mond auf jeden Fall abgebremst.

    Der Drehimpuls beider Bewegungen ist dann entgegengesetzt, d.h. beide können dem Betrage nach abnehmen, und die Summe bleibt trotzdem gleich (Drehimpulserhaltung), weil sie verschiedene Vorzeichen haben (willkürliches Beispiel ohne Einheiten: 60 + (-10) = 59 + (-9) = 50).

  695. #704 Peter
    2. Januar 2017

    @ Alderamin
    Vielen Dank für die ausführliche Antwort!

  696. #705 Alderamin
    2. Januar 2017

    @Peter

    Oben hatte ich mit “siehe unten” auf eine nachfolgende Textpassage verwiesen, die ich dann gar nicht geschrieben hatte… ich wollte mich darauf beziehen, warum der früheste Sonnenuntergang bzw. späteste Sonnenaufgang nicht auch am Tag der Wintersonnenwende statt finden, was an jener “Zeitgleichung” liegt. Das möchte ich noch nachschieben.

    Eine Drehung der Erde bezogen auf den Sternenhimmel (also ein ruhendes Koordinatensystem) beträgt 23h56m04s (86164 s). Warum hat ein Tag dann exakt 24h? Weil er sich nicht auf die Sterne bezieht (das tut der “Sterntag”), sondern auf die Sonne, von Mittagsstellung zur Mittagsstellung gemessen. Denn weil die Erde die Sonne umkreist, scheint sich die Sonne vor dem Sternenhimmel zu verschieben. Einmal um den ganzen Himmel (Tierkreis oder Ekliptik) herum in einem Jahr, 360° in 365 Tagen. Also fast genau 1° jeden Tag. 1° sind immerhin 2 Durchmesser der Sonnenscheibe. Wenn sich die Erde also bezogen auf den Sternenhimmel nach dem letzten Mittagsstand der Sonne genau einmal gedreht hat, steht die Sonne schon zwei Durchmesser weiter östlich. Dieses fehlende Grad muss sich die Erde noch weiter drehen, damit sie wieder im Süden steht, ein knappes 1/360 von einem Sterntag. Und das sind genau die 3m56s, die an 24 h fehlen.

    Nun bewegt sich die Erde aber nicht auf einer perfekten Kreisbahn um die Sonne, sondern gemäß der Keplerschen Gesetze auf einer Ellipse mit der Sonne in einem Brennpunkt (Kepler 1), wobei sie sich in Sonnennähe schneller und in Sonnenferne langsamer bewegt (Kepler 2). Sonnennähe ist just übermorgen, am 4. Januar, d.h. im Moment und auch schon in den vergangenen und kommenden Wochen bewegt sich die Erde schneller um die Sonne, als sie das im Winter tut. Das bedeutet, dass die Sonne sich auch pro Tag weiter vor dem Sternenhimmel verschiebt, als sie das im Sommer tut. Es reicht also nicht für die Erde, sich um die oben erwähnten 3m56 weiter zu drehen, um die Sonne wieder einzuholen, sondern es dauert noch ein paar Sekunden länger. Und das jeden Tag.

    D.h. zwischen zwei Mittagen vergehen derzeit mehr als 24h. Im Sommer entsprechend weniger. Im Mittel sind es zwar genau 24h, aber im Moment ist jeder Tag ein wenig länger als 24h, und die Sonne geht entsprechend ein wenig später auf und ein wenig später unter, als sie das tun würde, wenn der Tag exakt 24h lang wäre, und das summiert sich von Tag zu Tag zu einer immer größer werdenden Abweichung zwischen der “wahren Sonne” und der “mittleren Sonne”, nach der unsere Uhren laufen. Wenn der früheste Sonnenuntergang eigentlich am 21.12. sein sollte, die wahre Sonne aber “nachgeht”, findet der Sonnenuntergang später statt. Und der früheste Sonnenuntergang ist dann ein paar Tage vorher (am 12. Dezember), wenn die Differenz zwischen wahrer und mittlerer Sonne noch kleiner ist, obwohl die Sonne da noch höher steht.

    Entsprechend gilt für den spätesten Sonnenaufgang, dass dieser erst am 30.12. stattfindet. Der Tagesbogen der Sonne nimmt zwar seit dem 21.12. schon wieder zu, aber mit dem stetig später folgenden Höchststand der Sonne verschiebt sich auch ihr Aufgang zu immer späteren Zeiten, bis die damit konkurrierende Zunahme der Tageslänge die Oberhand gewinnt.

    Die Abweichung der wahren von der mittleren Sonne wird als “Zeitgleichung” bezeichnet, und sie kann fast eine Viertelstunde betragen. Neben dem Effekt der elliptischen Erdbahn kommt noch hinzu, dass die Sonne im Frühjahr und Herbst eine zur Erdachse schräge Bahn am Himmel nimmt (sie gewinnt im Frühjahr rasch an Höhe, im Herbst verliert sie diese rasch wieder), während sie im Winter und Sommer parallel zum Äquator nach Osten wandert und somit mehr Strecke nach Osten gut macht, als im Frühjahr und Herbst. Die Überlagerung beider Effekte sorgt für die Wellenlinie der Zeitgleichung.

  697. #706 Peter Paul
    Winnenden
    3. Januar 2017

    Eine Frage zur “Dunklen Materie”: Gibt es eigentlich Licht-Licht-Wechselwirkungen? Wenn nicht: Gehören Photonen dann zur “Dunklen Materie”?

  698. #707 Bullet
    3. Januar 2017

    @Peter Paul: in diesem Wiki-Artikel ist der Satz zu finden:

    Erscheinungen des Elektromagnetismus können auch dann beobachtbar sein, wenn keine elektrische Ladung in greifbarer Entfernung vorhanden ist, beispielsweise bei den elektromagnetischen Wellen oder beim Zerfall π 0 → 2 γ des π 0-Pions in zwei Gamma-Photonen.

    Wenn ich Feynman richtig verstanden habe (was natürlich immer mit einem Fragezeichen versehen ist), ist auch der umgekehrte Vorgang erlaubt, also die Bilung eines π-Pions bei der “Kollision” zweier Photonen mit passender Frequenz. Wäre das sowas, worauf deine Frage abzielt?

  699. #708 Peter Paul
    Winnenden
    3. Januar 2017

    Ja, das wäre sowas, aber gibt´s das denn wirklich? Dann wären Photonen also keine “Dunkle Materie”?

  700. #709 Peter Paul
    Winnenden
    17. Januar 2017

    Bisher wurden meines Wissens nur zwei Gravitationswellenereignisse von LIGO nachgewiesen. Hatte man nicht viel mehr solche Nachweise erwartet? Wieso sind es bisher nur so wenige?

  701. #710 Donkieshot
    Trantor
    31. Januar 2017

    Ich dachte immer es gibt keine Anti-Gravitation. Nun lese ich gerade unter SCINEXX.de:

    Kosmos: “Zentrum der Abstoßung” entdeckt
    Leerer Bereich des Weltalls stößt die Milchstraße und ihre Nachbarn aktiv weg
    (Nature Astronomy, 20ß17; doi: 10.1038/s41550-016-0036)

    Was haltet ihr davon? – Bis zum 1. April ist noch ‘ne Weile hin.

  702. #711 Theodor
    31. Januar 2017

    Eigentlich bin ich eher selbst ein Fragensteller, fühle mich nicht kompetent genug, auch antworten zu können. Aber dort steht kein Wort über “Anti-Gravitation”. Man kann sich diese Beobachtung schlicht noch nicht erklären.

    Zitat: Warum allerdings diese Region im Weltraum abstoßend wirkt und ob sie tatsächlich mit einem “Void”, einer besonders leeren Region im All, übereinstimmt, müssen nun weitere Forschungen zeigen.

  703. #712 Donkieshot
    31. Januar 2017

    Du hast recht, das Wort “Anti-Gravitation” kommt in dem Artikel nicht vor. Da es aber der: “Shapley Attraktor” ist “, eine schwerkraftreiche Zone auf die sich die meisten Galaxien und Cluster zubewegen.” die als anziehend bezeichnet wird. Daher kam mir der Gedanke die abstoßende Kraft als Anti-Schwerkraft bzw. Anti-Gravitation zu bezeichnen. Ich wollt auch nicht die Physik, die hinter diesen Vorgängen steht, erklärt bekommen. Es geht mir um eine Einschätzung der Ernsthaftigkeit dieser Arbeit.

  704. #713 Alderamin
    31. Januar 2017

    @Donkieshot

    Abstoßende Gravitation kennt die allgemeine Relativitätstheorie sehr wohl, z.B. bei einem negativen Druck des Vakuums, was die Erklärung für die wahrscheinliche Inflationsphase des frühen Universums und möglicherweise auch für die Dunkle Energie sein soll (positiver Druck wirkt anziehend, negativer abstoßend). Bisher geht man aber davon aus, dass die Dunkle Energie das Vakuum gleichmäßig erfüllt und es nicht Orte gibt, wo die Expansion stärker verläuft als anderswo.

    Aus dem Artikel führt ein Link auf das Papier bei Nature, und da ist nur von “Underdensities” die Rede, die eine Abstoßung bewirken würden. In diesem Zusammenhang ist “Abstoßung” sehr missverständlich, tatsächlich sorgt eine geringere Materiedichte nur dafür, dass die anziehenden Kräfte der umliegenden Materie kein Gegengewicht erhalten und die unterdichte Zone gewissermaßen von außen “leergesaugen”. Das ist von Voids bekannt, aus denen die Materie in die umliegenden Filamente fließt. Der “Große Attraktor” ist so ein Filament, und es scheint mir so, als habe man hier dessen Gegenspieler in Form des nächsten großen Voids identifziert, was mit Anti-Gravitation nichts zu tun hat, nur mit der Anordnung der Materie und der daraus folgenden Wirkung der Schwerkräfte. (Das erinnert mich ein bisschen an die “Löcherleitung” in Halbleitern, wo die Abwesenheit von Elektronen eine Wirkung entfaltet, die positiven Ladungsträgern gleich kommt).

  705. #714 Theodor
    31. Januar 2017

    @Alderamin

    Aha, immer wieder danke für die weiterführenden Erläuterungen.

  706. #715 Donkieshot
    Trantor
    31. Januar 2017

    @Alderamin
    Vielen Dank für die Informationen. Da kann ich mich ja wieder mal etwas aufschlauen. Sehr schön. Suchbegriffe habe ich ja jetzt.

  707. #716 Alderamin
    31. Januar 2017
  708. #717 willi1
    bremen
    5. Februar 2017

    Ihr könnt und wollt euch also immer noch nicht vorstellen, dass der Inhalt des uns bekannten Universums nach dessen Urknall ganz einfach die Anziehungskraft der Einzelteile (Galaxien) zueinander überwunden haben und in den schon vor dem Urknall vorhandenen Raum fliegen. Warum eigentlich nicht?
    Ich warte gespannt auf neue Erkenntnisse von Teleskopen, die den “vorgeburtlichen” Zustand des Raums darstellen können, damit endlich die Behauptung “Viel kann aus Nichts entstehen” ad acta gelegt werden kann. Ich behaupte immer noch, dass der sog. Urknall ein Kollisions- oder Explosionsereignis war, was ich aber nicht beweisen kann, wie auch alle noch so klugen Wissenschaftler der Erde ihre Theorien. Es wäre ja auch zu einfach.

  709. […] einer etwas längeren Pause geht es endlich wieder weiter in der Serie “Fragen zur Astronomie”. Und diesmal gleich mit einer besonders schönen Frage: Wie lange gibt es noch Sterne im […]

  710. #719 Captain E.
    6. Februar 2017

    @willi1:

    Es ist aber schon klar, dass die moderne Naturwissenschaft prinzipiell nichts beweist, oder? Man überprüft natürlich alle Hypothesen, und wenn man sie trotz aller Bemühungen nicht widerlegen kann, nimmt man erst einmal an, dass sie einigermaßen wahr sind.

  711. #720 Alderamin
    6. Februar 2017

    @willi1

    Ihr könnt und wollt euch also immer noch nicht vorstellen, dass der Inhalt des uns bekannten Universums nach dessen Urknall ganz einfach die Anziehungskraft der Einzelteile (Galaxien) zueinander überwunden haben und in den schon vor dem Urknall vorhandenen Raum fliegen. Warum eigentlich nicht?

    Dann müsste man z.B. erklären, warum diese Explosion genau symmetrisch um unseren Ort herum erfolgt, so als ob wir genau im Zentrum sind; das wäre schon ein seltsamer Zufall. Dies gilt auch für die kosmische Hintergrundstrahlung, die dem rotverschobenen Licht des Feuerballs entspricht. Das erreicht uns aus jeder Richtung völlig symmetrisch.

    Man sieht auch in der Ferne nirgendwo einen Rand, eine Explosionsfront, sondern das Weltall ist stets homogen und isotrop. Die Dichte entspricht dabei genau der kritischen Dichte, die zu einer flachen Geometrie des Universums führt. Bei einer Explosion wäre es ein großer Zufall, wenn die Materie exakt diejenige Dichte hätte, die sie mit Geschwindigkeit 0 im Unendlichen ankommen ließe (dies entspräche einer kritischen Dichte ohne Dunkle Energie). Eine kleine Abweichung davon in jeglicher Richtung (Dichte größer 1 oder kleiner 1) würde sich sehr schnell vergrößern. Das Universum erscheint wie ein Bleistift, der auf der Spitze balanciert ist.

    Eine Explosion kann auch nicht die Beschleunigung der Expansion erklären, die man beoabachtet (eben diese Dunkle Energie). Anhand von Messungen der Rotverschiebung der Galaxien in Abhängigkeit von ihrer Entfernung zeigt sich, dass die Expansion heute schneller erfolgt als früher. Bei einer Explosion müsste sie sich durch die wechselseitige Schwerkraft verlangsamen. Nur durch Berücksichtigung dieser beschleunigten Expansion kann man übrigens das Alter der ältesten Sterne mit dem aus der Expansion folgenden Weltalter in Einklang bringen.

    Die Belege für eine Raumexpansion sind schon ziemlich erdrückend. Da unterstützen sich unterschiedliche Beobachtungen sehr verschiedener und nicht unmittelbar voneinander abhängiger Vorgänge. Das haben sich nicht ein paar Leute an einem Nachmittag überlegt, sondern viele tausend Köpfe haben da seit 80 Jahren drüber nachgedacht und Beobachtungen gesammelt. Wenn’s nur eine Explosion wäre – da kommt man sofort drau, mit dieser Idee hat man mal angefangen, als man die “Galaxienflucht” entdeckte, aber sie ließ sich nicht halten, s.o.

    Ich warte gespannt auf neue Erkenntnisse von Teleskopen, die den “vorgeburtlichen” Zustand des Raums darstellen können,

    Jeglicher Blick hinter die kosmische Hintergrundstrahlung ist uns verwehrt, weil das Licht aus einem leuchtenden Plasma stammt, das selbst für Licht undurchlässig ist (wie etwa die Sonnenoberfläche). Bestenfalls mit Gravitationswellen und Neutrinos lässt sich noch in entferntere / frühere Zonen blicken.

    Man kann aber aus bekannter Physik bis zu den ersten Bruchteilen einer Sekund zurück schließen. Z.B. kann man ausrechnen, wie viel Wasserstoff in den ersten 3 Minuten zu Helium, Lithium und Deuterium verschmolz, und findet die entsprechenden Mengenverhältnisse heute im Gas zwischen den Galaxien wieder.

  712. #721 Bullet
    6. Februar 2017

    @ willi1:

    Ihr könnt und wollt euch also immer noch nicht vorstellen, […]. Warum eigentlich nicht?

    Vorstellen könnte man sich das. Wenn man das täte und dabei genau vorginge, dann würe man feststellen, daß ein solcher Vorgang aus unserer Position ganz anders aussähe als das, was man tatsächlich beobachten kann. Warum soll ich mir also eine bereits als falsch nachgewiesene Vorstellung machen? Nur damit du deine Couchkartoffel-Denkweise fortführen kannst?
    Och nö … laß ma’.

  713. #722 willi1
    7. Februar 2017

    Danke für eure Stellungnahmen 719-721.
    Natürlich kann ich alle bekannten Thesen nicht widerlegen, wie auch umgekehrt nicht. Ich habe hier schon vor längerer Zeit geschrieben, dass ich nicht an Spuk glaube, was es für mich wäre, wenn aus nichts “unser” Weltall entstanden sein soll. Ich halte das nicht für “Couchkartoffeldenken”, nur weil viele tausend Köpfe darüber nachgedacht haben, viele hundert Möglichkeiten entwickelt haben, aber nichts bewiesen haben außer das existente zu beschreiben. Ich halte mich daran, dass Materie größeren Ausmaßes schon vorher da war. Das muss nicht 1 Körper gewesen sein, sondern kann auch eine Kollision vieler Körper aus einer anderen Ecke des vorhandenen Raums gewesen sein.
    Wir gucken immer in die Mitte des möglichen Urknalls, der Hubble und Co. schon ziemlich nahe gekommen sind. Da die Milchstraße am Rande “unseres” Weltalls liegen soll, wäre der Blick in die leere Richtung besser, die bestimmt auch schon stattfindet, sofern die Position der Erde das zulässt. Was gibt es denn von dort zu melden?

  714. #723 nur Consuela
    7. Februar 2017

    @ willi1:
    Die Milchstraße und damit die Erde liegt nicht am Rande des Weltalls, da hast du vermutlich etwas missverstanden. Wir sehen von „zuhause“ aus in jede Richtung gleich weit, nämlch ungefähr 47 Milliarden Lichtjahre – dieser Radius definiert das beobachtbare Universum.

    Der Urknall fand an jedem Ort des Universums gleichzeitig statt, daher gibt es keine Richtung, in die wir bevorzugterweise schauen könnten. Anders gesagt: Wir können gar nicht anders als in Richtung Urknall zu schauen.

    Und was deine Kollisions-These betrifft: In der String-Theorie gibt es durchaus einen Ansatz, der als Ursache des Urknalls die Kollision sogenannter Branen annimmt. Aber es sind bei diesem Thema halt zwei Dinge schwierig: Erstens die Unmöglichkeit direkter Beobachtung des Urknalls und zweitens die Beschränktheit der menschlichen Vorstellungskraft.

  715. #724 Abu
    14. Februar 2017

    Frage zu schwarzen Löchern:
    – Wieso die Bezeichnung ‘Loch’.
    Das würde suggerieren, dass da ein Riss in der Raumzeit wäre, wo etwas in einem anderen Raum hindurchfällt (Wurmlöcher z.b.)
    Wäre es nicht besser man spricht von einem Galaktischen Kern.

    – Wenn eine Masse in ein schwarzes Loch fällt,
    trifft diese auch dann mal auf einen materiellen Kern ?

    – In CERN versucht man ebenfalls ‘schwarze Löcher’ zu erzeugen. Was versteht eigentlich die Teilchenphysik darunter. Bisher gibt’s ja schwarze Löcher nur in makroskopischer Form.

    mit freundlichen Grüßen,

  716. #725 PDP10
    14. Februar 2017

    @Abu:

    – Wieso die Bezeichnung ‘Loch’.

    Siehe: https://de.wikipedia.org/wiki/Schwarzes_Loch
    Gleich der zweite Absatz erklärt wo der Begriff her kommt.

    Wäre es nicht besser man spricht von einem Galaktischen Kern.

    Nein. Zwar hat – soweit wir wissen – jede Galaxie ein schwarzes Loch im Zentrum. Es gibt aber auch schwarze Löcher (und zwar wahrscheinlich jede Menge), die nicht im Zentrum einer Galaxie sind.

    – Wenn eine Masse in ein schwarzes Loch fällt,
    trifft diese auch dann mal auf einen materiellen Kern ?

    Das wissen wir nicht. Möglicherweise werden wir das wissen, wenn es irgendwann eine Quantenfeldtheorie der Gravitation gibt.

    – In CERN versucht man ebenfalls ‘schwarze Löcher’ zu erzeugen.

    Nein. Es wurde die vage theoretische Möglichkeit diskutiert, dass bei den hohen Energien die der Teilchenbeschleuniger am CERN erzeugt mikroskopisch kleine schwarze Löcher entstehen könnten. Man hat aber nicht “versucht” solche zu erzeugen.
    Und auch keine gefunden.

  717. #726 willi1
    23. Februar 2017

    @ Nr, 723
    du hast Recht, es musste heißen: …die Erde am Rande der Milchstraße…
    die “Branen” (habe in wiki geguckt) sind auch nur ´Vorstellungen” von etwas, was sich ein Experte in Raumangelegenheiten nicht vorstellen kann oder will: Die einfache “Lösung”: Unser Knall, wie immer er erzeugt wurde, fand im vorhandenen Raum statt. Und da hinein fliegen alle Trümmer, bis sie irgendwann irgendwo wieder einen neuen Knall entfachen. Das sehen wir heute noch in den Bildern der Teleskope, die uns das bis jetzt mögliche sichtbar machen. Darum dehnt sich das Universum nicht aus, sondern die Trümmer fliegen einfach auseinander, bis sie die Anziehungskraft ihres Ursprungs überwunden haben.
    Hier kommt dein letzter Satz voll zum Zug: “…die Unmöglichkeit direkter Beobachtung des Urknalls und zweitens die Beschränktheit der menschlichen Vorstellungskraft.”
    Dann sollte der Satz auch am Ende aller “Beweisführungen” stehen.

  718. #727 Alderamin
    23. Februar 2017

    @willi1

    Die einfache “Lösung”: Unser Knall, wie immer er erzeugt wurde, fand im vorhandenen Raum statt.

    Dann erkläre bitte, warum man den Feuerball in jeder Himmelsrichtung sieht und warum die Expansion an Geschwindigkeit zulegt (oder wie die Beobachtungen der symmetrischen Hintergrundstrahlung und der Abhängigkeit der Rotverschiebung von der Helligkeit von Typ-I-Supernovae und von räumlichen Korrelationen in der Hintergrundstrahlung alternativ zu erklären sind). Deine Alternativerklärung muss alle Beobachtungen besser erklären, als die anerkannte, die seit 80 Jahren niemand widerlegen konnte, deren Vorhersagen hingegen bestätigt wurden.

    Ich halte das nicht für “Couchkartoffeldenken”, nur weil viele tausend Köpfe darüber nachgedacht haben, viele hundert Möglichkeiten entwickelt haben, aber nichts bewiesen haben außer das existente zu beschreiben.

    Du überschätzt maßlos Deine Kompetenz im Vergleich zu den Leuten, die sich ihr ganzes Leben intensiv mit dem Thema befasst haben. Wenn aus dem letzten Jahrhundert irgendetwas in der Physik klar geworden sein sollte, dann die Tatsache, dass man mit der intuitiven Anschauung aus der menschlichen Erfahrungswelt in der Physik nicht weit kommt.

    Du hast schon ein Problem zu erklären, warum die Masse des gesamten Universum an einem Punkt versammelt überhaupt “explodieren” sollte, statt einfach in einem Schwarzen Loch eine Singularität zu bilden.

  719. #728 Shadelord
    Austria
    23. Februar 2017

    Wann wird das Wasser auf der Erde komplett verdampfen? Ich habe von 200 Mil. – 1,3 und 2,6 Mrd. gehört. Gibt es eine “Sternengeschichten” Folge über das Ende der Erde und wie es ablaufen wird?
    Danke.

  720. #729 willi1
    24. Februar 2017

    @ 727:
    Ich maße mir nicht die geringste Kompetenz an. Ich urteile nur nach meiner Ansicht, dass das ganze sichtbare und noch unsichtbare Geschehen im Weltraum auf den Gesetzen der Schwerkraft basiert und nicht aus einem Nichts entstanden ist. Das ist für mich Spuk. Beweisen kann ich dass genau so wenig, wie die Wissenschaft das Gegenteil beweisen kann.

  721. #730 PDP10
    24. Februar 2017

    @willi1:

    Beweisen kann ich dass genau so wenig, wie die Wissenschaft das Gegenteil beweisen kann.

    Doch, doch. Die Wissenschaft kann.

    Für uns sieht das Universum in allen Richtungen gleich aus.
    Die kosmische Hintergrundstrahlung ist in alle Richtungen gleich und die Galaxien entfernen sich (abhängig von der Entfernung) alle mit der gleichen Geschwindigkeit von uns.

    Dafür gibt es zwei mögliche Erklärungen:

    1. Wir befinden uns zufälligerweise in der “Mitte” des Universums. Da wo die “Explosion” statt gefunden hat, oder

    2. Jene “Explosion” war keine sondern der Urknall hat überall stattgefunden, weil mit ihm erst der Raum entstanden ist (und die Zeit) – und sich seitdem ausdehnt.

    Welche Erklärung ist wohl wahrscheinlicher?

  722. #731 willi1
    24. Februar 2017

    @ 727 nochmal und 721:
    Von welchem “Hintergrund” sprecht ihr denn? Hinter was ist der Grund? Ihr sprecht von Staub und Strahlung und dunkler Materie, die sich überall im All verbreitet haben. Richtig. Die werden wohl noch heute produziert bei vorkommenden Supernovae, die wir noch nicht sehen können. Als grobes Beispiel könnte man die Orthsche Wolke nennen, die man auch mehr ahnt als sieht wegen der geringen Größe der Meteoriten.
    Und ich spreche nicht nur von möglicher Explosion, sondern auch von möglicher Kollision. Ich könnte mich auch mein ganzes Leben damit beschäftigen und entwickle irgendwann eine Theorie, an der ich mich festhalten kann. Meine “Theorie” ist aus dem Bauch gesprochen, Es hat Jahrtausende gedauert, bis sich die Menschen auf Grund von Beobachtungsmöglichkeiten zu den heutigen Erkenntnissen durchgerungen haben. Welche Kämpfe hat es gekostet, um dahin zu kommen. Damit sind sie aber noch nicht auf dem Grund der Geschichte. Neue Teleskope bringen neue Erkenntnisse. Jetzt erst sind wir in der Lage, Planeten zu orten, die möglicherweise der Erde ähnlich sind. Das ist doch schon was, aber eben nur ein erweiterter Anfang.
    Könnt ihr euch wirklich nicht vorstellen, dass sich das Geschehen “unseres” Urknalls in einem bestehenden Raum ergeignet hat? Etwa so, als wenn in einem sehr großen Luftballon ein sehr kleiner Luftballon platzt? Bullet in 721 sagte: “Vorstellen könnte man sich das.” Und weiter: “. Warum soll ich mir also eine bereits als falsch nachgewiesene Vorstellung machen?” Der “Nachweis” ist keineswegs bewiesen, sondern angenommen, egal nach welcher Berechnung. Man kann ihn für sich als endgültig erklären. Für mich ist es Spuk, wenn alles aus nichts entstanden sein soll, nur weil wir es noch nicht besser wissen.

    727 fragt noch: “… warum man den Feuerball in jeder Himmelsrichtung sieht und warum die Expansion an Geschwindigkeit zulegt…” Wissen tue ich das auch nicht. Dass man den Feuerball (den Urknall?) in jeder Himmelsrichtung sieht, ist mir sogar neu. Vielleicht, weil sich im Raum alles um alles dreht? Und das von Anfang an? Und die zunehmende Geschwindigkeit der Expansion kann durch die Anziehungskraft von Massen verursacht werden, die schon im vorhandenen Raum existent waren und sind. Ähnlich wie außerhalb des sehr großen Luftballons.

  723. #732 nur Consuela
    24. Februar 2017

    @ willi1:
    Nur um Missverständnisse auszuschließen: Mein Schlusssatz in #723 über die Beschränktheit menschlicher Vorstellungskraft gilt natürlich auch für dich.

    “Spuk” zu rufen ist nicht unbedingt die beste Argumentation, wenn man in den Naturwissenschaften weiterkommen will. Die Natur schert sich nicht darum, ob wir Menschen irgendein Phänomen als einleuchtend oder logisch betrachten. Schau dir nur die gesamte Quantenphysik an – die ist mit Hausverstand auch nicht zu begreifen. Und der Urknall selbst hat eine Menge mit Quantenphysik UND Gravitation zu tun, auch wenn man’s vielleicht lieber anders und einfacher hätte.

  724. #733 PDP10
    24. Februar 2017

    @nur Consuela:

    Mein Schlusssatz in #723 über die Beschränktheit menschlicher Vorstellungskraft gilt natürlich auch für dich.

    Nein, tut er selbstverständlich nicht.

    “Was ich nicht verstehe muss falsch sein” war schon immer der Schlussstein im höchsten Gewölbe menschlicher Einfalt.

  725. #734 Alderamin
    25. Februar 2017

    @willi1

    Ich könnte mich auch mein ganzes Leben damit beschäftigen und entwickle irgendwann eine Theorie, an der ich mich festhalten kann. Meine “Theorie” ist aus dem Bauch gesprochen,

    Wissenschaft funktioniert nicht so, dass sich jemand im Hinterstübchen irgendwas ausdenkt. Die arbeitet mit der Überprüfung von Vorhersagen. Die Hintergrundstrahlung (das stark rotverschobene Licht des Feuerballs 400000 Jahre nach dem Urknall, als das Gas sich so weit abgekühlt hatte, dass die Kernteilchen die Elektronen einfangen konnten und das Plasma erstmals durchsichtig wurde, welches nun als Radiostrahlung im Mikrowellenbereich von überall her zu empfangen ist) war so eine Vorhersage, die sich bewahrheitete. Oder die Feinstruktur in der Hintergrundstrahlung, die auf kosmische Ausdehnung vergrößerte Quantenfluktuationen darstellt, und die mit dem Satelliten COBE erstmals nachgewiesen wurde. Oder das Verhältnis der Elemente im ursprünglichen Gas (Wasserstoff zu Helium und Lithium; aus dem dann auch folgt, dass Dunkle Materie kein Staub aus Supernovae sein kann, sie kann nicht einmal aus Protonen, Neutronen und Elektronen bestehen, sonst wäre ein anderes Massenverhältnis herausgekommen, das kann man berechnen). Aus Deiner Explosionshypothese folgen auch beobachtbare Konsequenzen, und wir versuchen Dir hier wiederholt klar zu machen, dass die an den realen Beobachtungen sofort scheitern müssen, aber darauf gehst Du nicht ein. Wissenschaft muss sich mit widersprechenden Beobachtungen auseinander setzen und sie erklären.

    Dass man den Feuerball (den Urknall?) in jeder Himmelsrichtung sieht, ist mir sogar neu.

    Das sind die untersten Grundlagen der Urknalltheorie. Dann hast Du vermutlich auch noch nichts von der Vakuumenergie und dem Higgs-Feld gehört, das die ganze Energie geliefert hat, aus der die Materie entstanden ist. Bevor Du was kritisierst, lies doch zuerst mal ein Buch darüber, was Du da eigentlich kritisierst. Z.B. Brian Greene, “Der Stoff aus dem der Kosmos ist”. Dann komm’ wieder mit Verständnisfragen.

    Ich klinke mich hier aus, dieser Thread war nie für Diskussionen gedacht, sondern um astronomische Fragen zu stellen, die dann entweder in einem extra Artikel von Florian erklärt werden, oder hier kurz beantwortet werden. “Glaubt ihr eure doofe Urknalltheorie eigentlich selber?” ist nicht die Art von Frage, die hier hingehört. Dafür gibt es zig andere Artikel zum Thema von Florian, die man hier mit der Suchfunktion oder mit Google finden kann. Da kann man dann darüber in Länge diskutieren.

  726. #735 Max
    25. Februar 2017

    @”Abstoßende Gravitation kennt die allgemeine Relativitätstheorie sehr wohl, z.B. bei einem negativen Druck des Vakuums” #713..???
    Jetzt wird es schon absurd, was dem Einstein alles angedichtet wird, alles was nicht ausdrücklich widerlegt, gehört also in die Allgemeine?.
    Der mit dem Vakuum, dem Casimir-Effekt, war eben Hendrik Casimir erst im 1948

  727. #736 Alderamin
    25. Februar 2017

    @Max

    Wird ihm nicht angedichtet, sondern der Druck, der auch negativ sein kann, steht auf der Diagonalen im Energie-Impuls-Tensor, der die Raumkrümmung bestimmt.

    Eine einfache Herleitung des negativen Drucks und der abstoßenden Schwerkraft findet sich hier:

    https://www.astro.ucla.edu/~wright/cosmo_constant.html

  728. #737 Alderamin
    25. Februar 2017

    … und eine schöne einfache Herleitung, die aus der Vakuumenergiedichte den negativen Druck herleitet steht hier (aus dem 2. Link oben).

    @Martin Bäker
    Falls Du hier mitliest: Du schriebst doch jüngst über den Energie-Impuls-Tensor und auch schon über die Vakuum-Energie. Schreib’ doch mal einen kleinen Artikel über die obige Herleitung und wie die kosmologische Inflation und die Dunkle Energie damit erklärt werden können. Und was das Higgs-Feld für eine Rolle spielt.

  729. #738 rolak
    25. Februar 2017

    Ok, Alderamin, sei also Max’ zumindest uneindeutiger Kommentar als Frage aufgefasst:

    erst im 1948

    Nee, Max, der Casimir-Effekt hat nicht nur reineweg nichts mit Gravitation zu tun, er macht sich darüber hinaus auch noch durch erhöhte Anziehung bemerkbar.

  730. #739 Alderamin
    25. Februar 2017

    @willi1

    Obwohl ich mich ausklinken wollte, aber weil Max’ Kommentar und meine Antwort darauf gerade im Zusammenhang so gut passt: so geht Wissenschaft:

    – Aus dem Michelson-Morley-Experiment folgt die Konstanz der Lichtgeschwindigkeit in allen unbeschleunigten Systemen und daraus folgt zwingend die Spezielle Relativitätstheorie.

    – Aus der Speziellen Relativitätstheorie folgt mit der Äquivalenz von Beschleunigung und Schwerkraft zwingend die Allgemeine Relativitätstheorie, die den Energie-Impuls-Tensor ergibt, der auch Druckkomponenten hat. Die Relativitätstheorien haben bisher jede Überprüfung mit Bravour gemeistert.

    – Aus der Allgemeinen Relativitätstheorie folgt mit der Herleitung in #737, dass eine Vakuumenergie einen negativen Druck und damit eine abstoßende Schwerkraft verursachen muss, die den Raum aufbläht. Die Allgemeine Relativitätstheorie verletzt übrigens die Energieerhaltung, aus der Du (unbewusst?) ableitest, dass aus Nichts nicht etwas enstehen kann. Expandierter Raum enthält mehr Vakuumenergie als nicht expandierter. Kosmologisch rotverschobenes Licht hat weniger Energie als zur Zeit seiner Aussendung.

    – Dass die Vakuumenergie sehr klein ist, aber nicht Null sein kann, folgt aus den Beobachtungen, die im 2. Link von #736 beschrieben sind.

    – Und wenn man nun noch die nicht unplausible Annahme hinzu nimmt, die durch Beobachtungen wie die Flachheit des Universums und die Feinstruktur der Hintergrundstrahlung unterstützt wird, dass die Vakuumenergie ursprünglich einmal viel höher gewesen war (mit entsprechend dramatisch schnellerer Expansion des Raums) und danach auf den heutigen niedrigeren Wert gefallen ist, dann hast Du auf einen Schlag eine gigantische Menge überschüssiger Energie (die als Strahlung frei gewordene Vakuumenergie beim Übergang auf das heutige, niedrigere Niveau), die nach E=mc² teilweise in Materie übergehen kann, und so bekommst Du Dein Universum aus dem Nichts.

    Gibt’s auch ein Buch drüber: “Ein Universum aus dem Nichts” von Lawrence Krauss.

    Das waren jetzt über 100 Jahre Forschung komprimiert. Definitiv nichts, was sich jemand im Hinterstübchen aus dem Bauch heraus überlegt hat. Darüber hätte Einstein sicher noch den Kopf geschüttelt, aber das ist das, was raus kommt, und es passt zu dem, was man beoobachtet. Im Gegensatz zu einer Explosion im vorhandenen Raum, die passt zu gar nichts und erklärt auch nicht, wo die Materie herkommt (ein Vorgängeruniversum verschiebt die Frage der Herkunft nur um eine Stufe, ohne sie zu lösen).

  731. #740 Max
    26. Februar 2017

    @Alderamin
    Danke sehr, ..ich brauche wohl noch eine Weile dazu, widme mich gerade anderem Thema.

    aber, wenn´s demokratisch ginge, würde ich mich dem willi1 anschließen.
    denn im Moment kann man sich aussuchen ob es Big Crunch, Big Rip, Big freeze, ….Big Beng oder M Theorie sein soll, also glauben, sich bekennen,.. ist nicht lange hier als Prof. L. die grundlegende Fragen mit einem Theologen diskutierte (zwar nichts dagegen, aber aus einem anderen Grund). So als könnten Gedanken tatsächlich die Realität verändern (ja klar 😉 [gemeint ist 1:1 bzw. direkt].
    Nun Tunnel Effekt, eingefrorenes Licht, praktisch kein Übergang zu der Quantenmechanik, ..scheint mir die Astronomie schon schön-gerechnet und mit Konstanten gespickt,
    wie die geozentrische A. vor Kopernikus und Kepler,
    mit vielen Tabellen, Rückläufern, man hat sich das ebenfalls zurecht gerechnet.
    Die Sterne juckt das nicht^^

  732. #741 nur Consuela
    26. Februar 2017

    @Max
    Wie Alderamin (wie schon so oft) großartig dargelegt hat ist Wissenschaft ein System, das ganz anders funktioniert als Glauben. Es gibt Beobachtungen, Experimente, Hypothesen und Theorien, die ein Problem immer weiter eingrenzen und manche Lösungen ausschließen. Manchmal sind diese Wege kompliziert und unintuitiv, aber dennoch nicht beliebig.

    Nein, die Wissenschaft weiß momentan nicht, was exakt der Big Bang (also der Zeitpunkt t=0) war. Es wird aber an mehreren Hypothesen geforscht, und sehr, sehr viele Hypothesen, wie zum Beispiel jene von @willi1, kann man bereits jetzt ausschließen, weil sie nicht mit den Beobachtungen übereinstimmen.

    Ich verstehe die Verwirrung ob der Vielzahl an Hypothesen zu manchen Fragestellungen sehr gut, aber daraus zu folgern, dass Naturwissenschaft beliebig sei (von wegen „man kann sich’s aussuchen“), ist schlichtweg falsch. In der Wissenschaft ist es keine Schande zu sagen „das wissen wir noch nicht“ – ganz im Gegenteil ist das Ursprung und Triebfeder jeder Forschung.

    Viele Menschen wünschen sich eindeutige letzte Antworten („42“) von der Wissenschaft, die diese aber niemals wird liefern können, einfach weil sie selbst immer wieder neue Fragen aufwirft. Letzte Antworten gibt es nur im Glauben – dort heißen sie Dogmen.

    ——

    Bitte um Verzeihung an FF – ich höre jetzt auch auf, diesen Thread zu kapern.

  733. #742 Hans-Water
    26. Februar 2017

    Nach dem Urknall gäbe es eine inflationäre Phase des Universums, während der sich der Raum mit Überlichtgeschwindigkeit ausgedehnt hat. Könnte es da nicht sein, dass bei Deep Field – Aufnahmen von Teleskopen auch unsere Milchstraße in ihrer frühen Phase gesehen werden kann, da der Raum ja das Licht sozusagen überholt hat und das Licht uns aus dieser Anfangsphase jetzt erst erreicht ?

  734. #743 Alderamin
    26. Februar 2017

    @Hans Water

    Nein, denn das Licht bewegt sich stets relativ zum Raum mit Lichtgeschwindigkeit und kann daher von diesem nicht “eingeholt” werden. Stell’ Dir den Raum als fließendes Gewässer vor und das Licht als eine Wasserwelle darauf, dann bewegt sich da auch die Welle immer relativ zum umgebenden Wasser und wird von diesem mitgenommen.

    Je tiefer man in den Raum blickt, desto schneller wird die Strömung relativ zu uns. Die Regionen, aus denen uns das Licht heute gerade noch erreicht, sind mittlerweile so weit entfernt, dass ihr heute ausgestrahltes Licht uns nie mehr erreichen wird. Daher wird der überschaubare Teil des Universums (nicht vom Durchmesser her, sondern gemessen an den enthaltenen Galaxien) immer kleiner, bis wir (bzw. unsere fernen Nachfahren) eines Tages nur noch die lokale Galaxiengruppe am Himmel sehen werden, die an der Expansion nicht teil nimmt.

  735. #744 Hans-Walter
    26. Februar 2017

    @Alderamin Danke für die Antwort, gut erklärt, entscheidend für mein Verständnis ist die Aussage, dass das Licht bei der Expansion des Raumes von diesem “mitgenommen” wird.

  736. #745 Henner Bangert
    Hamburg
    1. März 2017

    Ich habe eine wohl eher dumme Frage zur Bewegung von Erde und Mond. Die beiden kreisen ja um ihren gemeinsamen Schwerpunkt. Es ist daher nicht die Erde, die eine schöne Ellipse um die Sonne zieht, sondern der Schwerpunkt aus Erde und Mond.

    Aber ist denn diese Bahn tatsächlich “sauber” elliptisch, oder “wackelt” der Schwerpunkt ein bisschen auf dieser Bahn??? Kreisen Erde und Mond um den Schwerpunkt als eine Fixposition (bezogen auf die “saubere” elliptische Bahn um die Sonne)??? Vielleicht ist das bei mir bloß ein Verständnisdefizit für die Grundlagen der Rotation zweier Körper umeinander. Wenn der Mond eine Kreisbewegung machen würde, hätte ich sofort gedacht, ja, der Schwerpunkt bleibt gewissermaßen “liegen” (auf der Bahn). Aber bei einer Ellipse? Kann mich jemand aufklären? (Danke!)

  737. #746 Alderamin
    1. März 2017

    @Henner Bangert

    ist denn diese Bahn tatsächlich “sauber” elliptisch, oder “wackelt” der Schwerpunkt ein bisschen auf dieser Bahn???

    Ein klitzekleines bisschen wackeln wird das Baryzentrum von Erde und Mond schon, aufgrund der Störungen durch andere Planeten im Sonnensystem (sowie die Allgemeine Relativitätstheorie, Stichwort “Periheldrehung”), aber diese vernachlässigt bewegt es sich auf einer perfekten Keplerellipse um die Sonne. Es macht dabei keinen Unterschied, ob die Bahnen von Mond und Erde elliptisch oder kreisförmig sind, der Schwerpunkt muss nach dem Schwerpunktsatz gleichförmig bewegt (also im freien Fall um die Sonne) sein, und sich auf einer Linie mit Erde und Mond befinden, so dass diese sich aus Sicht des Baryzentrums stets gegenüber liegend befinden. Das Verhältnis der Abstände r/R zum Baryzentrum entspricht dabei stets dem umgekehrten Verhältnis der Massen M/m. Daraus folgt, dass die Erde die Ellipsenbahn des Mondes exakt verkleinert um den Faktor 1/81 (der Mond hat 1/81 der Erdmasse) und in der Ausrichtung um 180° rotiert nachvollzieht: ist der Mond in Erdferne, dann ist der Erdmittelpunkt auch am weitesten vom Baryzentrum entfernt, in Erdnähe entsprechend umgekehrt.

    Man kennt das auch von Doppelsternen. Hier ist eine schöne kleine Animation, die die Orbits bei zwei Sternen gleicher Masse demonstriert. So ähnlich läuft das auch bei Erde und Mond ab. Oder Pluto und Charon, wo das Baryzentrum außerhalb Plutos liegt:

    https://www.sciencenews.org/blog/science-ticker/pluto-and-charon%E2%80%99s-orbital-dance-captured-color

  738. #747 Abert
    5. März 2017

    Was ist Gravitomagnetismus ?

  739. #748 willi1
    7. März 2017
  740. #749 willi1
    7. März 2017

    an alle:
    ich bestreite keinem das Recht auf seine Meinung und die der letzten 100 Jahre über den Urknall.
    Meine Meinung ist “ganz klar”, dass der nicht erst ein Universum erschaffen hat, sondern dass das “Ereignis” – so nenne ich es mal – in einem bereits bestehenden Raum stattgefunden hat. Ob als Explosion, Kollision oder sonstwas. Und in den schon bestehenden Raum entfleuchen die Trümmer dieses Ereignisses, bis sie von der Erde nicht mehr ausgemacht werden können. Bedenke: Lange Zeit hat man auch erwartet, dass die Galaxien sich nicht weiter entfernen, sondern wegen der Gravitation sich wieder zusammen finden müssten. Aber das war wohl nix. Man konnte die Rotverschiebung ins Spiel bringen, die das Gegenteil bewies.

    @ Aldemarin: Das Buch von Krauss hast du mir schon vor langer Zeit empfohlen, das war noch in einem anderen Blog. Unter #346 hatte ich euch wiedergefunden. Das Buch hatte ich mir gekauft, gelesen, kommentiert und erfahren, dass auch andere meine Theorie als möglich einstufen. Krauss schreibt auf Seite 169:
    “In jüngerer Zeit hat “Universum” eine einfachere und wohl auch besser nachvollziehbare Bedeutung angenommen. Mittlerweile ist es üblich, sich “unser” Universum als das vorzustellen, was einfach alles einschließt, was wir derzeit und was wir je werden sehen können. Physikalisch umfasst unser Universum alles, was je auf uns eingewirkt haben könnte.
    Sobald man diese Definition für ein Universum wählt, geraten andere “Universen”*) in den Bereich des Möglichen, zumindest im Prinzip.
    *) Regionen, die von unserem Universum seit jeher kausal agekoppelt sind und es auch immer bleiben werden – wie Inseln, die durch einen Raum-Ozean von jeder Kommunikation untereinander abgeschnitten sind.”
    +++
    Soweit das Buch und Krauss. Ich finde das gar nicht so weit weg von meiner Theorie.

  741. #750 nur Consuela
    8. März 2017

    @willi1:
    Wie bereits oben beschrieben geben die Beobachtungsdaten einfach keine Möglichkeit eines Urknalls IM bestehenden Raum her – denn dann gäbe es einen „Mittelpunkt“. Das hat nichts mit Meinung zu tun, sondern mit Physik.

    Mir ist auch nicht klar, wie du den Zusammenhang zwischen der krauss’schen Begriffsdefinition von „Universum“ und deiner Hypothese herstellst.

    ———

    Können wir diese Diskussion irgendwo anders weiterführen, damit wir diesen Thread nicht noch weiter verunstalten? ZB hier:
    https://scienceblogs.de/astrodicticum-simplex/2013/08/31/wo-war-der-urknall-uberall/

  742. #751 PDP10
    8. März 2017

    @willi1:

    Nur diesen noch. Ansonsten sollte die Diskussion wirklich umziehen ..

    ich bestreite keinem das Recht auf seine Meinung und die der letzten 100 Jahre über den Urknall.

    Wie @nur Consuela schon angemerkt hat:
    Wissenschaft hat nichts mit Meinungen zu tun.

    No, you’re not entitled to your opinion

    “You are only entitled to what you can argue for.”

    In den Naturwissenschaften ist das Experiment und Deduktion.

    Und deine Hypothese widerspricht nun mal beidem.

  743. #752 Theodor
    9. März 2017

    Ich kann mich dem nur anschließen. Ich finde es sogar den Verstand beleidigend, wenn jemand jahrhundertelange von zig Menschen betriebene Forschung mit dem lapidaren Satz “ist aber meine Meinung” hinwegfegen will.

    Dies gilt nicht nur in der Wissenschaft, sondern auch in anderen Sachthemen, das deren Grundlagen wegen “ist meine Meinung” vollkommen ignoriert werden.

    Schlimm nur, wenn solche “Meinungen” dann noch Einfluss in Politik und Gesellschaft bekommen. Dann wird es wirklich ärgerlich.

    Aber Schluss jetzt, hoffentlich.

  744. #753 willi1
    14. März 2017

    Ich lese hier nur sporadisch und so kann ich gleich mehrere negative Beiträge verarbeiten, die sich nicht von der herrschenden Meinung trennen können bzw. eine andere Meinung akzeptieren. Leute wie ich sind euch in dieser Beziehung suspekt und nicht der Rede wert – ja, sogar lästig. Das ist Mittelalter oder Religion! Dies gilt besonders für #Theodor.

    Was haltet ihr denn von diesem Beitrag, den ich heute in unserer Zeitung gelesen habe. Stichwort: “Heisenbergsche Unschärferelation und Einstein”
    Einstein hat sich zeitlebens mit der Quantenmechanik schwer getan, obwohl die Theorie immer wieder von Neuem bestätigt worden ist. Von ihm stammen diese Sätze: Der Gedanke, dass ein in einem Strahl ausgesetztes Elektron aus freiem Entschluss den Augenblick und die Richtung wählt, in der es fortspringen will, ist mir unerträglich. Wenn schon, dann möchte ich lieber Schuster oder gar Angestellter einer Spielbank sein als Physiker.”

    Bekanntlich gilt die Theorie immer noch und Einstein ist trotzdem Physiker geblieben.

    @ 750 nur Consuela:
    “Wie bereits oben beschrieben geben die Beobachtungsdaten einfach keine Möglichkeit eines Urknalls IM bestehenden Raum her – denn dann gäbe es einen „Mittelpunkt“. Das hat nichts mit Meinung zu tun, sondern mit Physik.”
    Genau, wir können noch nicht hinter oder durch das Chaos des Urknalls gucken. Deswegen können wir doch nicht die Existenz eines davor schon bestehenden Universums verleugnen.

    “Mir ist auch nicht klar, wie du den Zusammenhang zwischen der krauss’schen Begriffsdefinition von „Universum“ und deiner Hypothese herstellst.”
    Wenn du das nicht herauslesen kannst, dann frage ich mich wirklich, wie verschlossen ihr seid und warum. Vielleicht hat “Aldemarin” das ja verstanden und erklärt es dir.

  745. #754 Alderamin
    14. März 2017

    @willi1

    Umzug nach -> da.

  746. #755 willi1
    14. März 2017

    #754: Alter Hut. Der endet 2013. Der Nachfolger in 2016. Dann hast du also auch nicht Krauss verstanden. Obwohl du mir das Buch empfohlen hast. Deswegen aber brauchst du kein Schuster zu werden.
    Ergeht euch weiter in Möglichkeiten, die “tausende von Wissenschaftlern in 100 Jahren” als für möglich erarbeitet haben.

  747. […] der Serie “Fragen zur Astronomie” wird es heute ein wenig philosophisch. Wenn ich Vorträge über die Erforschung des Weltraums […]

  748. #757 Smok Danek
    24. März 2017

    Moin! Es kommt eine Frage aus der “Tardis”-ähnlichen Ecke:

    In der Kurzgeschichte “Man muss helfen” (aus dem zweiten Guslar-Band “Ganz gewöhnliche Leute”, 1975) von Kir Bulytschow reist der außerirdische Besucher namens Fywa vor- und rückwärts in der Zeit, um sich damit auch im Raum zu bewegen.

    Das Prinzip erklärt Fywa seinem irdischen Gastgeber in gebrochener, einfacher Sprache sinngemäß so:

    Das Universum dreht sich dauernd. Starten wir also eine Zeitreise auf dem Planeten P zum Zeitpunkt t und bewegen uns auf der Zeitachse um t-x, dann ist zum Zeitpunkt t-x der Planet P uns unter den Füßen längst weggeflogen. An dessen Statt ist jetzt, wenn wir alles genau berechnet haben, der Planet Q in der Galaxie G. So kann man bei exakter Kalkulation mit der Zeitmaschine tatsächlich Raum-Fahrten machen. Jedenfalls nach Fywa.

    Nun weiß ich nicht, ob in der UdSSR, der Heimat von Bulytschow, im Jahr 1975 Doctor Who bekannt war. Ich tippe auf “nein”. Um so besser für unseren humorvollen Schriftsteller.

    Inwiefern die Frage die Astronomie betrifft, möge der geschätzte Blogger entscheiden.

    Ach ja, die Frage: Hatte Fywa Recht? (Mal hypothetisch die Zeitreisen für möglich haltend.)

    Danke und viele Grüße – bin gespannt…

  749. […] geht es in der Serie “Fragen zur Astronomie” um eine ganz praktische Frage: Wieso entsorgen wir unseren Müll nicht einfach in der Sonne?. […]

  750. #759 willi1
    3. April 2017

    zu teuer

  751. #760 AmbiValent
    4. April 2017

    @Smok Danek
    Auch wenn das mit dem rotierenden Universum stimmen würde: Das Universum ist sehr sehr leer, man muss schon sehr exakt navigieren, um von einem Stern zum anderen zu kommen. Bewegt man sich von einem Stern nur entlang der Zeitkomponente, erreicht man mit ziemlicher Sicherheit nur leeren Raum.

    Aber die herkömmliche Idee, zum Beispiel im Rom von heute 2000 Jahre in die Vergangenheit zu reisen und im Rom von damals anzukommen, ist genauso falsch (selbst im Fall, dass Zeitreisen möglich wären).

  752. #761 Higgs-Teilchen
    Im Standardmodell oben rechts
    5. April 2017

    Wie kommen eigentlich so tolle Formen zustande, wie zum Beispiel die Säulen der Schöpfung im Adlernebel?

  753. #762 Alderamin
    5. April 2017

    @Higgsteilchen

    Im Adlernebel erodieren junge, blaue Sterne mit ihrer UV-Strahlung die Gas- und Staubwolke, die sich dort verdichtet und zu Sternen kollabiert. Im Schatten der Wolke kann sich das Material länger halten. Im Prinzip ganz ähnlich, wie in den USA die Mesas und Säulen aus Sandstein entstehen, die oben durch Gestein vor Regen geschützt werden und an der Seite vom Regen abgeknabbert werden.

    Siehe auch den Text unter dem zugehörigen APOD-Bild

  754. #763 Sturmkraehe
    Nürnberg
    6. April 2017

    Hi Florian und die Anderen,

    ich schau mir gerade die History Channel Dokuserie “Doomsday” an (in deutsch) – Folge über einen Neptun-Massen-grosser Planet mit Ringen, der ca. 500.000 km (5 Stunden bis zum Einschlag) vor der Erde “erscheint” und erst dort irgendwie ohne Vorzeichen auftaucht…

    Irgendwie bringt mich das zum Lachen und Weinen…. Entweder hab ich trotz super Lehrer wie dich nichts gelernt oder sowas dürfte nicht Dokumentation genannt werden…. Der Abschuss war ein U-Boot mit lebender Besatzung das mit dem Meer zum “Jupiter” gezogen wird nachdem die Erde zerstört war.

    Hierzu würde mich mal ein Debunking interessieren, also was ungefähr wann passieren würde, wenn wirklich ein streunender Planet mit 17facher Erdmasse durch unser Sonnensystem auf die Erde zurast. Denn für eine Doku find ich diese Art von Angsthase sehr bedenklich und ich würde gerne halbwegs fundiert etwas dazu sagen können, falls ich gefragt werde – ich wüsste zwar so einiges, aber da hier Wissenschaftler vieler Bereich interviewt wurden, möchte ich mich besonders versichern, das ich nicht ganz falsch liege.

    Ich wäre sehr froh, wenn jemand die Abfolge bis zu Zerstörung von Mond und Erde erklären könnte – gerne auch als Podcast-Sonderfolge 😉

  755. #764 Tamara
    Athen/Griechenland
    6. April 2017

    Frage: Gehen Sternhaufen (offene und Kugel) irgendwann auseinander, wann und wie?

  756. #765 Alderamin
    7. April 2017

    @Tamara

    Offene Sternhaufen driften irgendwann auseinander, weil sie nur lose gruppiert sind und bei nahen Begegnungen mit anderen Sternen der Milchstraße durcheinander gewirbelt werden. Sterne entstehen immer in Gruppen, auch die Sonne entstammt einem Sternhaufen, dessen Mitglieder sich nach ca. 20 Umläufen vermutlich schon über den gesamten Umfang der Milchstraße verteilt haben.

    Ein recht alter offener Sternhaufen ist Prasepe (Krippe, M44) im Krebs, den gibt die Wikipedia mit 750 Millionen Jahren an. Der enthält schon keine jungen blauen Sterne mehr. Ungefähr so alt ist die Ursa-Major-Gruppe, zu der die meisten hellen Sterne des Großen Wagens gehören, 700 Millionen Jahre, der schon ziemlich zerstreut ist. Die Hyaden sind mit 625 Millionen Jahren etwas jünger.

    Kugelsternhaufen sind hingegen so alt wie die Milchstraße, 13 Milliarden Jahre, und halten also offenbar fest zusammen. Sie bewegen sich auch nur kurz durch die Milchstraße und meistens im Bereich des Halos, da sind Begegnungen mit anderen Sternen seltener als bei offenen Sternhaufen.

  757. #766 Alderamin
    7. April 2017

    @Strumkraehe

    Ist der Nibiru-Quatsch also wieder zurück? Florian hat dazu um 2012, als mal wieder die Welt untergehen sollte, eine ganze Reihe Artikel geschrieben, auch, warum sich kein Planet unbemerkt der Erde nähern kann.

    https://scienceblogs.de/astrodicticum-simplex/?s=nibiru

  758. #767 Smok Danek
    7. April 2017

    @AmbiValent:
    1. Das mit dem Rotieren des Universums war eine gedankliche Abkürzung – zugegeben, eine nicht besonders glückliche.

    2. Dass ein bestimmter Rechenaufwand bei solchen Reisen notwendig ist, betont der außerirdische Besucher Fywa recht deutlich. Er hat auch nur gewisse Zeitfenster, in den er sich am Reiseziel aufhalten kann (erster Besuch bei Udalow: acht Minuten), sonst “findet er nicht nach Zuhause wieder”.

  759. #768 Tamara
    7. April 2017

    danke

  760. #769 Walter
    Salzburg
    21. April 2017

    Hallo zusammen!

    Hätte mal eine Frage zu den Sternen, die mich schon längere Zeit beschäftigt….
    Wenn wir zu den Sterne blicken, sehen wir ja bekanntlich nicht den Stern an sich, sondern nur das Licht, das auf dem Weg zu uns auf die Erde ist. D.h. wenn jetzt in diesem Moment ein Stern in Form einer Supernova sterben würde der 500 Lichtjahre entfernt ist, sehen wir das grelle Licht der Supernova erst in 500 Jahren.
    Aber wie erkennt man, dass das Licht der Sterne auf der Erde “angekommen” ist. Das Licht der Sonne ist ja klar und eindeutig zu sehen (Schatten etc.). Bei Sternen die hunderte oder tausende Lichtjahre entfernt sind, sieht die Sache aber schon anders aus. Wie weiß ich, wann das Licht bei uns auf der angekommen ist? Ich sehe ja keinen Lichtstrahl des Sterns vor meinen Füßen oder auf meiner Hand. Hoffe das war nicht zu verwirrend und einigermaßen verständlich geschrieben:)
    Bin gespannt auf informative Rückmeldungen!

  761. #770 Captain E.
    22. April 2017

    @Walter:

    Der Hausherr hat dazu bestimmt schon etwas geschrieben. Entfernungsbestimmung ist aber in der Tat knifflig. Auf “kurze” Entfernung funktioniert die Parallaxenmessung, darüberhinaus benötigt man “Standardkerzen”. Für “mittlere” Entfernungen bedient man sich der Cepheiden, für die wirklich großen Distanzen der Supernovae des Typs Ia.

  762. #771 Alderamin
    22. April 2017

    @Walter

    Ich weiß nicht, ob ich Dich richtig verstehe aber – wenn Du das Lichts siehst, ist es da, ob mit oder ohne Schattenwurf. Man sähe halt die genannte Supernova in 500 Jahren aufleuchten. Die Sonne sehen wir auch mit 8 Minuten 20 Verzögerung und den Jupiter mit rund 40 Minuten. Und wenn Jupiter der Erde am nächsten ist, wie gerade jetzt, dann finden Jupitermondereignisse 16:40 Minuten früher statt als wenn der Jupiter auf der gegenüberliegenden Seite der Sonne liegt, zwei Erdbahnhalbmesser weiter weg.

    Genau genommen sehen wir nichts zu der Zeit, wenn es passiert, sondern das Licht hat immer einen kleinen Weg zu uns, auch wenn der im Alltag nur Sekundenbruchteile dauert. Aber wir sehen alles immer erst dann, wenn das Licht uns erreicht. Und am Himmel erreicht uns Licht aus allen möglichen Entfernungen mit unterschiedlichen Laufzeiten, d.h. kaum etwas, was wir heute gleichzeitig am Himmel sehen, fand ursprünglich zur gleichen Zeit statt.

  763. #772 Christopher Michel
    Deutschland
    26. April 2017

    Hallo,

    Wenn sich ein schwarzes Loch vermutlich im Zentrum einer jeden Galaxie befindet, wäre es dann nicht naheliegend anzunehmen dass das loch ein Resultat des Urknalls ist bzw. Der Ursprung oder das Ergebnis der Entstehung einer galaxie?

    Wie entstehen überhaupt neue Galaxien?

    Vielen Dank schonmal für eine Antwort!

  764. #773 Florian Freistetter
    26. April 2017

    @Christopher Michel: “Wie entstehen überhaupt neue Galaxien?”

    Gute Frage die ich demnächst in einem Artikel erklären werde! Mit dem Urknall hat das aber nix zu tun.

  765. #774 Alderamin
    26. April 2017

    @Christopher Michel

    Fangen wir mal mit der letzten Frage an: Nach dem Urknall war die Materie (Wasserstoff- und Heliumgas, sowie Dunkle Materie) ziemlich homogen im Raum verteilt, mit nur kleinen Dichteschwankungen. Dort, wo die Dichte zufällig etwas höher war, herrschte eine geringfügig höhere Schwerkraft, die die umliegende Materie anzog. Da die Materie zum größten Teil (4/5) aus der Dunklen Materie besteht, spielte diese die maßgebliche Rolle. Sie verdichete sich zu einem Geflecht aus Filamenten mit großen Lücken (Voids) dazwischen und zog die sichtbare Materie mit sich.

    Da, wo sich Filamente trafen, war die Schwerkraft besonders hoch und dorthin floß die Materie zusammen. Die Dunkle Materie konnte sich nicht beliebig verdichten, sondern bildete nur große Halos aus, aber die sichtbare Materie konnte innerhalb der Halos durch Kollisionen und Strahlung Energie verlieren und sich zu kleineren Strukturen verdichten, zu Gaswolken, die dann weiter fragmentierten, bis hin zu Sternen. Die erste Generation von Sternen war dabei teilweise sehr groß mit über 100 Sonnenmassen.

    Die Herkunft der Schwarzen Löcher innerhalb der Galaxien ist noch nicht geklärt. Es gibt mehrere mögliche Erklärungen:

    – Die Materie stürzte im Zentrum von Galaxien so schnell in den Kern, dass die dabei entstehenden Sterne das einfallende Gas nicht aufhalten und wegblasen konnten, wie das normalerweise der Fall ist, so dass supermassive Schwarze Löcher gleich aus dem einfallenden Gas entstanden.

    – Es entstanden zunächst Riesensterne, die nach kurzer Lebensdauer zu Schwarzen Löchern wurden. Diese verschmolzen dann miteinander.

    – Möglicherweise verschmolzen die Riesensterne auch durch Kollisionen im Zentrum der Galaxie zu einem supermassiven Schwarzen Loch.

    – Beim Urknall entstanden gleich Schwarze Löcher, die in Galaxien nach innen wanderten und dort verschmolzen.

    Die ersten Galaxien waren eher klein. Durch Kollisionen verschmolzen kleinere Galaxien zu größeren. Die Supermassiven Schwarzen Löcher solcher Galaxien sanken dann durch Begegnungen mit anderen Sternen in das Zentrum und verschmolzen dort miteinander. Auf diese Weise konnten die supermassiven Schwarzen Löcher weiter wachsen.

    Es ist möglich, dass auf diese Weise das weitgehend lineare Verhältnis zwischen der Masse der supermassiven Schwarzen Löcher und der umgebenden Galaxien entstand und erhalten blieb.

    Die Supermassiven Schwarzen Löcher bremsten aber auch bei ihrer Entstehung das einfallende Gas durch die Strahlung ihrer Akkretionsscheiben (das Material, das in ein Schwarzes Loch fällt, muss zunächst in einer umlaufenden Scheibe seine Bewegungsenergie durch Strahlung los werden, bevor es in das SL hineinfallen kann). Die Akkretionsscheiben supermassiver SLs sind die hellsten Objekte im Universum, die sogenannten Quasare, die sich nur im weit entfernten und damit jungen Universum finden, als noch Galaxien entstanden. Die Strahlung ist viel heller als das aller übrigen Sterne der Galaxie zusammen und so stark, dass sie das einfallende Gas wegblasen kann und somit das Wachstum der Galaxie bremsen oder gar stoppen.

    Es ist noch nicht geklärt, welche der oben genannten Thesen korrekt ist und man hofft, dies durch die neuen im Bau befindlichen Teleskope und das James-Webb-Weltraumteleskop klären zu können, indem man tiefer in den Raum schaut und bei der Entstehung der Galaxien gewissermaßen live zusieht.

  766. #775 AmbiValent
    27. April 2017

    Kann man eigentlich am Äußeren eines Sterns erkennen, was für Fusionsprozesse im Innern ablaufen? Ich denke, Wasserstoffbrennphase und Heliumbrennphase lassen sich noch unterscheiden, aber kann man auch spätere Phasen erkennen?

  767. #776 Alderamin
    27. April 2017

    @Ambivalent

    So viel ich weiß, kann man nicht einmal den Heliumflash von außen erkennen. Innen ist eine Wärmequelle und außen viel Wasserstoff, mehr sieht man nicht. Bis das Schalenbrennen so weit nach außen gewandert ist, bis die Atmosphäre des Sterns zum Roten Riesen aufgeblasen wird. Oder gleich komplett weggeblasen, wie bei den Wolf-Rayet-Sternen, bei denen dann das Helium freiliegt. Dann erst sieht man zweifelsfrei, was im Stern vor sich geht.

  768. #777 Florian Freistetter
    27. April 2017

    @AmbiValent: Ebenfalls eine gute Frage, die ich demnächst beantworten werde (wenn ich denn mal endlich ein wenig mehr Zeit hätte!!)

  769. #778 UMa
    28. April 2017

    Ich habe mal eine schwierige Frage zu Neutronensternen.

    Weiße Zwerge kühlen allmählich aus, siehe hier:
    https://de.wikipedia.org/wiki/Wei%C3%9Fer_Zwerg#Energietransport

    Wie ist das aber bei Neutronensternen?
    Aufgrund der viel geringeren Oberfläche sollten sie viel langsamer auskühlen. Das tun sie aber, nach den wenigen Arbeiten die ich dazu gefunden haben nicht, sondern kühlen trotz höhere Anfangstemperatur schneller aus als Weiße Zwerge, leider ohne Erklärung und es werden nur die erste Million Jahre betrachtet oder so..

    Warum?

    Haben sie einfach auf Grund des extremen Zustandes eine sehr geringe Wärmekapazität? Setzt das Gesetz von Debye schon bei sehr hohen Temperaturen (10⁵ K?) ein?

    Was bedeutet das für alte (1-10Ga) Neutronensterne,
    und deren Sichtbarkeit in naher Sonnenumgebung (100 Lj).
    Eigentlich müsste es dort viele geben, auf 10 bis 100 weiße Zwerge sollte ein Neutronenstern kommen. Aber es wurden keine beobachtet.

    Ist das weil sie sehr kalt sind, oder wurden sie bei ihrer Entstehung mit hoher Geschwindigkeit aus der galaktischen Ebene herausgeschleudert? Oder beides?

  770. #779 Alderamin
    28. April 2017

    @UMa

    Kurze Recherche meinerseits:

    Schau’ mal hier, Thermal History. Neutronensterne kühlen alleine durch thermische Strahlung tatsächlich sehr viel langsamer ab als Weiße Zwerge, aber zu Beginn gibt es zusätzliche Prozesse (Produktion von Neutrinos und Bremsstrahlung), die die anfängliche Abkühlung stark beschleunigen.

  771. #780 UMa
    5. Mai 2017

    @Alderamin:
    Danke.
    Die Seite kannte ich schon. Eine andere Quelle die ich fand war
    https://arxiv.org/abs/1411.6803

    Die anfängliche stärkere Abkühlung durch Neutrinos usw. ist für alte/kühlere Neutronensterne nicht so wichtig, da sie nur bei hohen Temperaturen stärker als die normale thermische Strahlung.
    10⁵ K werden nach nur 1 Million Jahren erreicht. Danach scheint die Abkühlung richtig schnell zu gehen, ohne das erklärt ist warum.
    Leider habe ich bisher nichts gefunden, das weiter (10⁷ Jahre) reicht.
    Welche Temperatur und thermische Abstrahlung hat ein Milliarden Jahre alter Neutronenstern?
    Gibt es alte, kalte Neutronensterne in der nahen Sonnenumgebung (100 Lj)?

  772. #781 klauswerner
    Pommelsbrunn
    9. Mai 2017

    Ich hätte mal ne Frage zu nem Bericht neulich im Spiegel: Was genau ist “negative effiktive Masse”?
    Insbesondere scheint ja dieses “effektiv” wichtig zu sein. Was ist also der Unterschied von “uneffektiver” zu “effektiver Masse”?

    Leider verstehe ich hier Wikipedia nicht, dazu fehlt es mir wohl an Vorbildung.

    Und dann gleich in dem Zusammenhang: Daran liegt es wohl auch, das es dann doch keine negative Gravitation (ANtigravitation) geben wird, oder?

    Grüßle!

  773. #782 AmbiValent
    12. Mai 2017

    @klauswerner

    Das ist ein missverständlicher Name. Eigentlich geht es um das Verhalten des Bose-Einstein-Kondensats (Atome, die nahe des absoluten Temperaturnullpunkts abgekühlt wurden). Dass dieses nicht zerläuft, sondern zusammenbleibt, wurde mit Hilfe des Ausdrucks der “effektiven Masse” beschrieben – und die wäre in diesem Fall negativ.

    Es hat absolut nichts damit zu tun, dass eine Masse tatsächlich negativ würde. (Aber für eine Schlagzeile wirkt es natürlich toll)

  774. #783 Alex
    22. Mai 2017

    Ich habe eine Frage zur Präzession der Erdachse:
    Wenn sich die Ausrichtungen der Erdachse ändert, ändert sich auch der (vektorielle) Drehimpuls der Erde. Wie lässt sich dies mit der Drehimpulserhaltung vereinbaren? Ändert sich etwas am Drehimpuls des Mondes?

  775. #784 Meo
    23. Mai 2017

    Mich würde mal folgendes interessieren:

    Wie würde man Geräusche auf anderen Planeten/Monden mit Atmosphäre wahrnehmen?

    Ich denk mir so, ganz naiv, dass auf dem Mars, wo die Atmosphäre ziemlich dünn ist, eine Schallquelle an sich ziemlich leise sein müsste.
    Gleichzeitig aber müssten Geräusche dort über ziemlich weite Strecken zu hören sein, da die geringe Dichte der Atmosphäre ja auch bedeuten müsste, dass es weniger Reibungsverluste der Schallwellen/Impulse geben müsste. Weniger Moleküle und Atome=weniger Streuung der kinetischen Energie bei der Schallübertragung von “Luftteilchen” zu “Luftteilchen” Schall also länger brauch, bis er seine Bewegungs-Energie/Wellen durch Verluste verloren hat.

    Auf Planeten mit hoher atmosphärischer Dichte stell ich mir das Ganze umgekehrt vor: Es ist laut, gleichzeitig hat Schall aber keine große Reichweite.

    Ist dem so oder hau ich da etwas durcheinander?

  776. #785 Theodor
    23. Mai 2017

    In unserem Ozean-Wasser jedenfalls trägt Schall deutlich weiter als in unserer Luft. Und Wasser ist nun deutlich dichter als Luft.

    Keine Ahnung, ob man das so übertragen kann 😉

  777. #786 Roy
    24. Mai 2017

    Mich als Laien würde interessieren, warum sich der Raum umso schneller (und in großer Entfernung sogar angeblich mit Überlichtgeschwindigkeit) ausdehnen kann, je weiter er von uns entfernt ist und dadurch Objekte aus dem für uns beobachtbaren Universum “verschwinden” können? Hat das mit der sog. dunklen Energie zu tun?

  778. #787 Theodor
    24. Mai 2017

    Objekte aus dem “alten” Universum können deshalb aus dem für uns beobachtbaren Universum verschwinden, weil sie inzwischen weiter entfernt sind, als das ihr Licht uns erreicht haben kann. Ist auch als Laie für mich nicht einfach verständlich. Allerdings sollte ich nicht vorm Vaddertach antworten.

    Die Sache mit der “Überlichtgeschwindigkeit” verwechselt Du vielleicht mit der Inflation? Damals hatten sich aber nicht die Objekte mit Überlichtgeschwindigkeit voneinander entfernt, sondern der gesamte Raum, das Universum hatte sich inflationär ausgedehnt. Auch keine triviale Angelegenheit, das zu verstehen bzw sogar die dahinterstehende Physik, die oft Vorstellungsgrenzen sprengt ist nicht trivial.

    Mit dem Phänomen der dunklen Energie hat dies aber erst mal nichts zu tun. Die Berechnungen des beobachtbaren Universums und der Inflation gab es schon lange vor der Beobachtung dunkler Materie und Energie.

  779. #788 AmbiValent
    24. Mai 2017

    @Roy
    Der Raum dehnt sich nicht “schneller” aus in dem Sinn, dass wir stillstehen und sich alles von uns wegbewegt (auch wenn es wegen der Relativität so aussieht), er dehnt sich überall gleichmäßig aus. Also von hier bis in einer Entfernung x wächst die Entfernung um zB eine Lichtsekunde, von dort bis zur Entfernung 2x um eine weitere, von da bis zur Entfernung 3x um eine dritte usw.

    Ein Signal aus einer weit entfernten Quelle legt in einem Jahr ein Lichtjahr zurück (in alle Richtungen, also auch zu uns hin). Aber ab einer bestimmten Entfernung ist die Entfernung so groß, dass die Entfernung in diesem Jahr auch um ein Lichtjahr gewachsen ist – das Signal kommt also nicht näher, und alle Signale von weiter entfernten Quellen können uns nie erreichen.

    Die Ausdehnung scheint sich im Laufe der Zeit nicht zu verlangsamen, wie man lange gedacht hatte, sondern eher noch etwas zu beschleunigen. Dafür macht man die sogenannte Dunkle Energie verantwortlich – bei der aber noch nicht genau geklärt ist, wie sie mit allen anderen Faktoren zusammenhängt.

  780. #789 Alderamin
    24. Mai 2017

    @Ambivalent

    Die Dunkle Energie bräuchte es nicht für die Expansion des Weltalls und dafür, dass die Strecke zu hinreichend weit entfernten Galaxien mit mehr als Lichtgeschwindigkeit wachsen kann – das galt alles schon bei der klassischen Urknalltheorie vor der Entdeckung der dunklen Energie. Die gibt nur zusätzlich Gas.

  781. #790 AmbiValent
    24. Mai 2017

    @Alderamin
    Genau das, was du gerade gesagt hast, wollte ich im letzten Absatz ausdrücken… kam wohl nicht klar rüber. (Statt “Dafür” hätte ich wohl “Für diese Beschleunigung, aber nicht für die Ausdehnung an sich” einsetzen sollen)

  782. #791 Roy
    29. Mai 2017

    Meine Frage nach dem “Warum” ist damit aber nicht beantwortet, und kann es wohl auch von der Menschheit derzeit noch nicht. Signale von Objekten können uns doch nur dann nicht mehr erreichen, wenn sie sich mit Lichtgeschwindigkeit oder schneller von uns entfernen (dafür gibt es wohl den sog. Hubble-Radius). Aber wie ist so etwas möglich? Auch sollen in vielen Milliarden Jahren nur noch Objekte der lokalen Gruppe für uns sichtbar sein, da alle anderen außerhalb dieses Radius sein werden, auch das ist mir völlig unklar.

  783. #793 Roy
    29. Mai 2017

    Dieser Artikel hat ja meine Frage aufgeworfen. Aber wie Du ja selbst schreibst, können wir sie wohl noch nicht vollständig beantworten: “…Wann genau das passieren wird, kann man nicht aufs Jahr genau sagen Dazu müssten wir die Details der Kosmologie und Phänomene wie die dunkle Energie, die ja für die immer schnellere Expansion verantwortlich ist, viel besser verstehen”.

  784. #794 Alderamin
    29. Mai 2017

    @Roy

    Nochmal: die Dunkle Energie hat zunächst einmal nichts damit zu tun, dass die Expansion in hinreichender Entfernung überlichtschnell ist. Eine Strecke im All wächst aus sich heraus, der Raum quillt gewissermaßen auf, und das liegt vor allem daran, dass die Materie den Raum mit sich zieht (nach der Allgemeinen Relativitätstheorie beeinflusst die Masse den Raum). Eine hinreichend lange Strecke wächst mit Lichtgeschwindigkeit oder mehr, was kein Problem für die Allgemeine Relativitätstheorie ist – der Raum kann machen, was er will, nur Materie kann sich gegenüber dem Raum nicht mit mehr als Lichtgeschwindigkeit bewegen. Da der Raum selbst wächst, verbleiben die Galaxien relativ zu ihm praktisch in Ruhe, der Raum schiebt sie lediglich auseinander. (Um genau zu sein, gibt’s in der Relativitätstheorie keinen absoluten Raum und keine Geschwindigkeit außer der Lichtgeschwindigkeit macht Sinn ohne ein Bezugssystem, aber die Hintergrundstrahlung des Urknalls bietet ein natürliches Bezugssystem, relativ zu dem sich die Galaxien nur vergleichsweise langsam wegen ihrer wechselseitigen Gravtiation umeinander bewegen, mit ein paar hundert km/s maximal)

    Das wusste man schon vor der Entdeckung der Dunklen Energie in den 1990ern. Damals nahm man es aols selbstverständlich an, die Expansion würde sich allmählich verlangsamen, weil die Materie sich gegenseitig anzieht und abbremst, also sollte der Raum dann auch langsamer expandieren. Als man nachprüfen wollte, ob es auf diese Weise je zu einem Stillstand der Expansion kommen würde, stellte man völlig überrascht fest, dass die Expansion sich im Gegenteil beschleunigt, und dieses Phänomen nannte man “Dunkle Energie”, weil man nicht wusste, was es ist.

    Als Erklärung in Frage käme eine Vakuumenergie, die zu einer abstoßenden Schwerkraft führt – nach der ART bedingt eine Vakuumenergie einen negativen Druck, und negativer Druck bewirkt eine abstoßende Schwerkraft (der sogenannte “Energie-Impuls-Tensor”, der die Raumkrümmung beschreibt, enthält neben der Masse auch Komponenten für Druck- und Scherkräfte). Das Problem das wir momentan haben ist, dass unsere Berechnungen für die Vakuumenergie auf der Basis der Quantenphysik einen absurd hohen Wert ergeben, obwohl der Wert in Wahrheit sehr viel kleiner, aber eben nicht Null ist (was viel einfacher zu erklären wäre). Irgendetwas sorgt dafür, dass sich die Energie der Teilchen im Vakuum fast komplett auslöscht. Wie das möglich ist, ist neben der Identität der Dunklen Materie die größte Frage der Kosmologie, die bisher noch niemand beantworten konnte.

  785. #795 Bullet
    29. Mai 2017

    @Roy:

    Auch sollen in vielen Milliarden Jahren nur noch Objekte der lokalen Gruppe für uns sichtbar sein, da alle anderen außerhalb dieses Radius sein werden, auch das ist mir völlig unklar.

    die Lokale Gruppe ist ein durch Schwerkraft aneinander hängendes Gebilde aus Galaxien. Die Hubble-Konstante beträgt ja etwa 68 km/s pro Megaparsec, wenn man hier guggt. (Hier sinds dann schon 73 km/s pro Mp…)
    Die Galaxien sind aber im Raum nicht festgenagelt. Wenn die Hubble-Expansion den Raum zwischen den Galaxien vergrößert, dann zieht die Intrahaufen*-Gravitation die Galaxien wieder zurück. Das ist ja gerade das Merkmal für gravitative Bindung. Und so entfernen sich langsam aber sicher alle anderen Galaxienhaufen und individuelle Galaxien, die zu keinem Haufen gehören, von der Lokalen Gruppe. Es ist auch nicht auszuschließen, daß das eine oder andere Mitglied der lokalen Gruppe irgendwann in Milliarden Jahren so weit weg vom Baryzentrum der Lokalen Gruppe herumlungert, daß der summierte Schwerkraftzug aller Gruppenmitglieder nicht mehr ausreicht, um diese Randexistenz zurück in die Gesellschaft zu ziehen. Alle anderen Galaxien entschwinden ja sowieso, und auch diese kleine Insel wird ihnen folgen.

    * eigene Wortschöpfung. Bitte nicht in anderen Diskussionen verwenden.

  786. #796 UMa
    2. Juni 2017

    @Alex:

    Ich habe eine Frage zur Präzession der Erdachse:
    Wenn sich die Ausrichtungen der Erdachse ändert, ändert sich auch der (vektorielle) Drehimpuls der Erde. Wie lässt sich dies mit der Drehimpulserhaltung vereinbaren? Ändert sich etwas am Drehimpuls des Mondes?

    Ja. Es ändert sich als Ausgleich der Bahndrehimpuls des Mondes, also die Bahnebene in der sich der Mond um die Erde bewegt.
    Insgesamt ist es noch komplizierter, da auch die Sonne zur Präzession mit beiträgt. Dort ändert sich, sehr wenig, da der Bahndrehimpuls sehr hoch ist, die Ebene Erdbahn um die Sonne.

  787. #797 Jens
    Hessen
    2. Juni 2017

    gestern wurde ein neues Gravitationswellen-Ereignis bekannt gegeben.
    siehe https://en.wikipedia.org/wiki/GW170104
    und https://www.youtube.com/watch?v=FGC_DM7ZgAk
    und https://www.aei.mpg.de/2056384/gw170104
    Es wurde am 04.01.2017 als drittes Ereignis am LIGO gemessen und im Unterschied zu den anderen beiden nicht automatisch sondern manuell registriert. Sechs weitere Kandidaten stehen in den Startlöchern. Florian arbeitest du schon an einen Bericht für uns der uns auf den neusten Stand bringt?

  788. #798 Ambi Valent
    26. Juni 2017

    Es gibt mal wieder eine Warnung vor einem Zusammenstoß: in “nur” 1,3 Millionen Jahren würde Gliese 710 mit dem Sonnensystem zusammenstoßen.

    Bei Wikipedia steht genaueres, nämlich eine Minimalentfernung von immer noch 2 bis 3 Lichtmonaten. Also weit entfernt von den Planeten. Da steht auch, dass vor nur etwa 70000 Jahren Scholz’ Stern 7 bis 14 Lichtmonate an die Sonne herankam. Was für Auswirkungen hatte das damals?

  789. #799 Christine Schumacher
    Deutschland
    26. Juni 2017

    Es wurde festgestellt das der mond sich von der erde entfernt. Aber stimmt das auch.? Könnte es nicht auch sein das sich die erde vom Mond entfernt wegen der Anziehungskraft des schwarzen Loches in der Mitte unserer galaxy. Und das es nur deswegen nicht gleich auffällt weil die sonne sich aufbläht?

  790. #800 PDP10
    26. Juni 2017

    @Christine Schumacher:

    Es wurde festgestellt das der mond sich von der erde entfernt.

    Das ist richtig. Allerdings nur ein paar cm pro Jahr.

    Könnte es nicht auch sein das sich die erde vom Mond entfernt

    Der Mond kreist um die Erde. Wie sollte sich die Erde also vom Mond entfernen?
    Ausser man betrachtet es aus der Perspektive der Mondbewohner. Von da aus gesehen entfernt sich die Erde vom Mond.

  791. #801 Ambi Valent
    26. Juni 2017

    @Christine Schumacher

    Erde und Mond entfernen sich voneinander. Das liegt allerdings daran, dass durch die Gezeiten auf der Erde Drehimpuls der Rotation der Erde zu Drehimpuls des Erde-Mond-Systems umgewandelt wird.

    Die Erde dreht sich also langsamer, während der Abstand, in dem Erde und Mond das gemeinsame Zentrum umkreisen, langsam zunimmt.

    Das Schwarze Loch im Zentrum der Milchstraße hat zwar eine große Masse, aber während es das ganze
    Sonnensystem stark beschleunigt (so dass es das Zentrum umkreist), unterscheidet sich die Beschleunigung, die Erde und Mond bekommen, so gut wie gar nicht.

    Die Sonne dagegen beschleunigt Erde und Mond leicht unterschiedlich, das wirkt sich aber nur auf die Ausrichtung des Mondorbits aus (zB in welcher Richtung der Mond steht, wenn er in Erdnähe ist, ändert sich), aber nicht auf die Entferung von Erde und Mond.

  792. #802 Bullet
    27. Juni 2017

    @C.Sch.:

    Könnte es nicht auch sein das sich die erde vom Mond entfernt wegen der Anziehungskraft des schwarzen Loches in der Mitte unserer galaxy.

    Sag mir doch bitte mal, wie weit dieses Schwarze Loch in der Mitte unserer galaxy von der Erde entfernt ist.

  793. #803 Florian Freistetter
    27. Juni 2017

    @Bullet: “Sag mir doch bitte mal, wie weit dieses Schwarze Loch in der Mitte unserer galaxy von der Erde entfernt ist.”

    Und sagt mir doch bitte mal wo dieses Ding mit der “galaxy” herkommt? Das fällt mir immer öfter auf, dass Leute exakt dieses Wort und diese Schreibweise anstatt “Milchstraße” oder “Galaxie” verwenden. Ist das nur ein Rechtschreibfehler? Ist das Wurschtigkeit ob der Sprache? Sprachwandel? Oder steckt da doch was dahinter?

  794. #804 Spritkopf
    27. Juni 2017

    @Christine Schumacher

    Könnte es nicht auch sein das sich die erde vom Mond entfernt wegen der Anziehungskraft des schwarzen Loches in der Mitte unserer galaxy. Und das es nur deswegen nicht gleich auffällt weil die sonne sich aufbläht?

    Ich lese aus deiner Frage ein Missverständnis. Unsere Sonne samt der Planeten Merkur, Venus, Erde, Mars etc. ist keine Galaxie, sondern ein Sonnensystem. Die Galaxie, in der wir uns befinden (und die aus vielen Sonnensystemen besteht), ist die Milchstraße.

  795. #805 Bullet
    27. Juni 2017

    @FF: ja, das ist mir auch aufgefallen. Man könnte es sich vorstellen, wenn der/die Schreiber/in nicht in D aufgewachsen ist – aber dann sollten noch andere Auffälligkeiten dabei sein. Und damit meine ich nicht die übliche furchtbar schlechte Orthogravieh.

  796. #806 Theodor
    27. Juni 2017

    Hat wahrscheinlich den selben Hintergrund, wie es in letzter Zeit trotz dauernder Hinweise statt die CPU der CPU heißt

  797. #807 Alderamin
    29. Juni 2017

    @Ambivalent

    Da steht auch, dass vor nur etwa 70000 Jahren Scholz’ Stern 7 bis 14 Lichtmonate an die Sonne herankam. Was für Auswirkungen hatte das damals?

    Keine, weil die noch bevorstehen. Die Oortsche Wolke liegt zwischen mindestens 10.000 und höchstens 200.000 AE von der Sonne entfernt. 7 Lichtmonate sind etwa 37.000 AU, d.h. der Stern ist wahrscheinlich mitten durch die Oortsche Wolke geflogen und dürfte einige Objekte in Richtung des inneren Sonnensystems in Bewegung gesetzt haben. Wie lange brauchen die bis zu uns?

    Das kann man mit Kepler 3 leicht ausrechnen, welcher besagt, dass sich für Objekte, die die Sonnen umkreisen, die Quadrate der Umlaufzeiten wie die 3. Potenzen der großen Bahnhalbachsen verhalten. Also gilt mit der Umlaufzeit T und der großen Halbachse a eines solchen Objekts:

    T²/(1 Jahr)² = a³/(1 AE³) => T [Jahre]=sqrt(a [AE]³)

    wenn man die Erde mit ihrer Umlaufzeit und Halbachse als Bezug nimmt. Wenn ein Objekt von sehr weit weg aus angenäherter Bewegungslosigkeit ins innere Sonnensystem fällt, dann hat die notwendigerweise extrem schmale Ellipse eine große Halbachse, die fast genau der halben Entfernung zum Ausgangspunkt entspricht, und die Zeit bis zum Eintreffen bei uns ist die Hälfte der Umlaufzeit.

    Bei 37.000 AU Ausgangsentfernung beträgt die Fallzeit bis ins innere Sonnensystem demgemäß t = T/2 = 1/2* sqrt((37000/2)³) = 1,258 Mio Jahre. Bei 14 Lichtmonaten/74.000 AU wären es gar 3,559 Mio Jahre. Und selbst wenn die theoretische innere Grenze der Oortschen Wolke bei 10.000 AE aufgewirbelt worden wäre, wäre die Fallzeit immer noch fast 180.000 Jahre lang.

    Was immer Scholz’ Stern also da draußen angerichtet haben mag, die Auswirkungen zeigen sich frühestens in 100.000 Jahren, eher noch in Millionen Jahren. Die Frage ist vielmehr, welche Sterne haben in den vergangenen 10-15 Millionen Jahren die Oortsche Wolke gestreift oder durchkreuzt, was Auswirkungen zur heutigen Zeit haben könnte? GAIA wird diese Frage mit Sicherheit beantworten können und man wird darüber lesen.

  798. #808 Ambi Valent
    29. Juni 2017

    @Alderamin
    Das wird natürlich interessant. Ich würde aber annehmen, dass außer bei einem extrem nahen Vorbeiflug die Auswirkungen nur sehr langsam über die Jahrtausende anwachsen würden, bis sie dann wieder sehr langsam schrumpfen. Also erwarte ich innerhalb der nächsten 2000 Jahre keine größere durchschnittliche Gefahr als während der letzten…

  799. #809 Alderamin
    29. Juni 2017

    @Ambivalent

    Wer weiß, vielleicht gab es vor ein paar zehntausend Jahren, als noch niemand Geschichtsschreibung betrieb, so etwas wie “Kometen” noch gar nicht bzw. seit 60 Millionen Jahren nicht mehr… 😉

  800. #810 Ambi Valent
    29. Juni 2017

    @Alderamin
    Hm… ich habe nochmal den Swing-By-Effekt berechnet. Da wäre dann die Relativgeschwindigkeit des Oort-Objekts zu Scholz’ Stern vor und nach der Begegnung dieselbe, geht aber in eine andere Richtung.

    Da das Oort-Objekt relativ zur Sonne fast stillsteht, wäre diese Relativgeschwindigkeit zum größten Teil die der Bewegung von Scholz’ Stern relativ zur Sonne. Scholz’ Stern ist jetzt etwa 20 Lichtjahre entfernt und war vor 70000 Jahren in Sonnennähe, würde sich also mit etwa 18 AE/Jahr bewegen.

    Das würde bedeuten er war etwa 2000 Jahre vorher an einem Punkt 37000 AE von der Sonnennähe entfernt, und zu dem Zeitpunkt stünde seine Bahn in einem 45-Grad-Winkel zur Sonne. Das heißt, ein Oort-Objekt, das von dort aus losflöge, hätte 18 AE/Jahr in Flugrichtung und 18 AE/Jahr senkrecht dazu und flöge in Richtung der Sonne.

    Das wären dann ein “Start”punkt vor ca 72000 Jahren in ca 37000 * Wurzel(2) und eine “Start”geschwindigkeit von 18 * Wurzel(2) AE/Jahr relativ zur Sonne – und das Oort-Objekt würde im Lauf der Zeit weiter beschleunigen.

    Das wären dann doch nur ca 2000 Jahre Flugzeit, weit weniger, als ich gedacht hätte. Das würde dann aber auch bedeuten, dass die Objekte alle schon wieder weg sind, weil sie viel zu schnell waren, um im Sonnensystem zu bleiben.

  801. #811 Ambi Valent
    29. Juni 2017

    (Und die meisten Oort-Objekte, die Scholz’ Stern träfe, hätten nicht genau die exakt richtigen Bedingungen und würden von vornherin außerhalb des Neptun an der Planetenregion des Sonnensystems vorbeifliegen)

  802. #812 Alderamin
    29. Juni 2017

    @Ambi Valent

    Um einen Swingby-Effekt auszulösen, muss das Objekt dem Stern aber recht nahe kommen. Etwas Googelei liefert einen mittleren Abstand größerer Objekte von 10-30 AE innerhalb der Oortschen Wolke, da kann ein Stern prima hindurch, ohne allzu vielen Objekten nennenswert nahe zu kommen. Die Chance, dass in einem größeren Einzugsbereich Asteroiden leicht verzögert werden und damit zum Absturz Richtung inneres Sonnensystem gebracht, ist erheblich höher, als dass ein Objekt stark beschleunigt wird – und dann auch noch exakt auf die Sonne zu, anonsten ergäbe sich nämlich nur irgendeine weite Bahn, die das Sonnensystem höchstwahrscheinlich sogar verlassen würde.

    Das klassische Szenario ist nach wie vor: Stern wirbelt Oortsche Wolke durcheinander; ein paar der Objekte verlieren dabei ihre Orbitalgeschwindigkeit und stürzen ab Richtung Sonne.

  803. #813 W A H
    Berlin
    8. August 2017

    @FF eine Frage zum Thema beschleunigte Expansion des Weltraums:
    Angenommen es gibt das Graviton und angenommen, das Graviton hat eine Masse und weiter angenommen, dass sich daraus ergibt, dass es eine Halbwertzeit hat, ist es dann möglich, dass eine mit Zeit und Entfernung verschwindende Gravitation und daraus möglicherweise entstehende Energie einen Beitrag zur beschleunigten Expansion leistet?

  804. #814 Alderamin
    9. August 2017

    @W A H

    Kräfte, die unendlich weit reichen, haben notwendigerweise Bosonen, die masselos sind (wie die elektromagnetische Kraft das Photon, aber im Gegensatz zur schwachen Wechselwirkung, vermittelt von den massebehafteten W- und Z-Bosonen), also ist das Graviton zwingend masselos. Und Bosonen sind elementar, die zerfallen nicht, haben also auch keine Halbwertszeit. Für masselose, lichtschnelle Bosonen vergeht ohnehin überhaupt keine Eigenzeit von der Quelle zum Ziel, da machte der Begriff “Halbwertszeit” keinen Sinn. Ergo: nein, auf keinen Fall.

  805. #815 W A H
    Berlin
    9. August 2017

    @Alderamin
    “Kräfte, die unendlich weit reichen, haben notwendigerweise Bosonen, die masselos sind”.

    Das stimmt natürlich in sich, für den Fall, dass die Gravitation unendlich weit reicht. Meine Frage beruhte aber auf anderen Annahmen. Aus “GW170104: Observation of a 50-Solar-Mass Binary Black Hole Coalescence at Redshift 0.2”; B. P. Abbott et al. (LIGO Scientific and Virgo Collaboration) Phys. Rev. Lett. 118, 221101 – Published 1 June 2017 habe ich folgendes Zitat: “Assuming that gravitons are dispersed in vacuum like massive particles, we bound the graviton mass to mg≤7.7×10−23  eV/c2.”. Es scheint also zumindest erlaubt, über eine Masse zu spekulieren.

  806. #816 Alderamin
    9. August 2017

    @W A H

    Theoretische Annahmen sind das eine, Messungen sind aber immer nötig, um sie zu verifizieren (oder falsifizieren), daher ist es legitim, über die Masse des Gravitons (oder auch des Photons) zu spekulieren und sie dann durch Messungen zu überprüfen. Die Messung ergab eine Obergrenze (ich nehme an, im Rahmen der Messgenauigkeit). Und 7.7×10−23  eV/c2 ist wirklich sehr wenig (ein Elektron hat z.B. 511000 eV/c²). Das Ergebnis ist also konsistent mit einem masselosen Teilchen.

    Für Neutrinos hat man bisher auch nur eine Massenobergrenze von ca. 3 eV/c² nachweisen können, man weiß aber, dass die Masse nicht 0 sein kann, weil Neutrinos ihren Flavor (Elektron-/Tau-/Myon-Neutrino) ändern, d.h. sie haben folglich eine Eigenzeit und sind einen Ticken langsamer als mit c unterwegs. Bei Graviton oder Photon sagt die Theorie, dass sie keine (Ruhe-)Masse haben, und die Messungen widersprechen dem auch nicht.

  807. #817 Rubberduck
    10. August 2017

    Hallo und Guten Tag,

    ich verfolge Deinen Block schon länger. Zur Zeit arbeite ich mich durch die Podcast Serie Sternengeschichten.
    Zur Folge 42 (Gammablitze) habe ich ein Verständnisproblem.

    Gammastrahlen aus dem Weltraum können nicht von der Erde aus gesehen werden, da die Atmosphäre stört. Deshalb schickte man Sateliten mit entsprechenden Detektoren ins Weltall. So weit so gut. Aber warum kann man mit Sateliten Gammastrahlung auf der Erde feststellen. (Atomwaffentests) – da ist doch auch unsere Atmosphäre dazwischen. In einer Richtung (Erde>Weltall) geht es nicht – in der anderen (Weltall>Erde) anscheinend doch. Warum ?

  808. #818 fherb
    16. August 2017

    Wie real ist die Oortsche Wolke?

    In einigen Beiträgen der letzten Monate ging es auch um “Brocken” aus dem Raum ausserhalb des Wirkungsbereichs der Sonne. Oder denen, die ganz weit außen gefangen sind. Den äußeren Radius der Oortschen Wolke verstehe ich als den Bereich, wo Materie beim Einfangen mit einer Relativgeschwindigkeit zur Sonne nahe Null (bezogen auf die Umlaufgeschwindigkeit zur Sonne in diesem Abstand) unterwegs war, ohne sich durch benachbarte Sterne einfangen zu lassen.

    Da aber eingefangene Objekte aus verschiedensten Richtungen und verschiedensten Anfangsgeschwindigkeiten und -richtungen wahrscheinlich sind, müsste sich auch der Anteil abschätzen lassen, der regelmäßig als azyklischer Komet auftritt. Lässt sich daraus wiederum abschätzen, wie gut gefüllt die Oortsche Wolke tatsächlich ist?

  809. #819 Ambi Valent
    16. August 2017

    @Rubberduck
    Da bis jetzt noch keine Antwort kam: Ich bin kein Experte, hatte aber auch schon nach einer Lösung geschaut. Die beste Erklärung für das Phänomen ist für mich, dass die Gammastrahlung von der Erde direkt aus Kernprozessen kommt, und dadurch aus Photonen bestimmter Energien besteht, während die aus dem Weltall indirekt ist, dass also die ursprüngliche Strahlung geschluckt und als extrem energiereiche Schwarzkörperstrahlung wieder ausgestrahlt wurde, dann aber auf einen großen Frequenzbereich verschmiert ist.

    Und die Sensoren und Auswertungsprogramme wären besser darin, Strahlung des ersten Typs zu erkennen.

    Wenn jemand genaueres weiß, bitte melden.

  810. #820 UMa
    16. August 2017

    @Rubberduck
    Die Atmosphäre ist auch in der anderen Richtung für Gammastrahlen undurchsichtig.
    Die ersten Vela-Satelliten konnten nur Kernwaffenexplosionen im Weltraum und der oberen Atmosphäre feststellen, soweit man die Strahlung eben empfangen konnte.
    Ab 1967 gab es die verbesserten Vela-Satelliten, die auf optischen Wellenlängen auch Explosionen in der Atmosphäre feststellen konnten.

  811. #821 Carl Wenning
    87439 Kempten Allgäu
    27. August 2017

    Wie schnell in km/Tag zieht sich die Sonnenscheinkante nach der Sommersonnwende vom Polarkreis zum Nordpol zurück?
    Ich denke, es ist ein exponentieller Rückzug, mit einer linearen Rechnung komme ich nicht hin!
    Vielleicht kann mir ein Mathematiker einen Tipp geben oder gehört das schon zum Allgemeinwissen?
    Liebe Grüße,
    Carl

  812. #822 Ambi Valent
    27. August 2017

    @Carl Wenning
    Was meinst du mit “Kante”? Nach der Sommersonnenwende scheint am Nordpol noch für 3 Monate die Sonne.

    (Und wie kommst du auf die Frage?)

  813. #823 PDP10
    27. August 2017

    @Ambi Valent:

    Ich denke, er meint folgendes:

    Wenn man sich um den Nordpol einen Kreis denkt (also einen Breitengrad) an dem die Sonne zu einem gegebenen Zeitpunkt (sagen wir mal: 6:00 Uhr UTC) aufgeht, dann wird dieser Kreis von der Sommer- bis zur Wintersonnenwende immer weiter nach Süden wandern (auf einem gegebenen Breitengrad wird die Uhrzeit des Sonnenaufgangs immer später werden).

    Die Frage ist: mit welcher Geschwindigkeit.

    Und die ist gar nicht so einfach zu beantworten.

    Eine naive Näherung wäre, den Zeitraum von 6 Monaten durch 23,5 Grad zu teilen und das ganze auf den Teil des Erdumfangs um zu rechnen, den die 23,5 Grad ausmachen.

    Das kommt aber nur in erster Näherung hin, weil die Tangente an dem gegebenen Standpunkt zum Horizont nach Norden hin immer flacher wird.
    Mal abgesehen von anderen Einflüssen, wie zb der ellipsenförmigen Erdbahn um die Sonne, die dazu führt, dass sich der Winkel Erdachse bez. der Ekliptik von der Sonne aus gesehen nicht linear ändert usw.

    Eine exakte Formel dafür wäre mit Sicherheit hinreichend kompliziert und fiele in Florians Fachgebiet – vielleicht hat er ja mal Lust, was dazu zu schreiben.

    Aber ich wette, für den Hausgebrauch, muss man das nicht ausrechnen. Da gibts garantiert Tabellen für.

    Das Auffinden ebensolcher im Web sei dem Leser hiermit als Übungsaufgabe überlassen … :-).

  814. #824 Ambi Valent
    27. August 2017

    Oder vielleicht “Wie schnell zieht sich der Rand der Fläche, in der 24 Stunden am Tag die Sonne scheint, nach der Sommersonnenwende vom Polarkreis zum Nordpol zurück?”

    Ich habe schon eine Idee, wie man das löst, aber wenn das eine Vorbereitungsaufgabe für eine Matheklausur ist, und ich Carl die Lösung sage, dann hat er zwar jetzt eine Lösung, aber hat nicht gelernt, diese Lösung selber zu entwickeln – und wäre dann in der Klausur, wenn ihm keiner helfen kann, der Angeschmierte.

  815. #825 Carl Wenning
    Kempten
    30. August 2017

    Das Programm läßt mich nicht mehr rein um das weiter zu erklären, wieso?

  816. #826 Alderamin
    30. August 2017

    Den Breitengrad, von wo aus man noch um Mitternacht die Sonne sehen kann, kann man über die Deklination der Sonne leicht bestimmen (das ist der Breitengrad, über welchem die Sonne mittags im Zenit steht); theoretisch (ohne Atmosphäre) stünde die Sonne 90° nördlich davon um Mitternacht noch über dem Horizont, also zur Sommersonnenwende (23,5° Deklination) noch 113,5° weiter nördlich, das entspricht dem Breitengrad 66,5°N. Die Bewegung der Sonne in Deklination ist aber nicht konstant, sondern nach der Sommersonnenwende bewegt sie sich nur sehr gemächlich nach Süden, während sie um die Herbst-Tag- und -Nachtgleiche am schnellsten wandert.

    Man kann die Extremwerte mit einer Deklinationstabelle leicht ermitteln (diese gilt streng genommen nur für ein bestimmtes Jahr, aber die Abweichungen von Jahr zu Jahr sind klein). Vom 21. zum 22.06. ändert sich die Deklination laut Tabelle nur von 23°26’16” auf 23°26’13”, also um 3 Winkelsekunden, das entspricht 3/3600° = 0,00083°. Ein Grad Breite sind ziemlich genau 111,1 km, also zieht sich die Linie der Mitternachtssonne nur um 111,1*0,00083° = 0,00926 km = 9,26 m zurück.

    Vom 22. auf den 23. September ändert sich die Deklination laut Tabelle hingegen von 0°23′25″ auf 0°00′04″, das sind 23 Bogenminuten und 21 Bogensekunden oder 0,38917°, entsprechend 43,24 km Rückzug der Mitternachtssonne (die nach der Tag- und Nachtgleiche am Nordpol komplett verschwindet und der Ausbreitung der Polarnacht Platz macht). Danach verlangsamt sich die Änderung der Deklination wieder, bis sie bei der Wintersonnenwende zum Stillstand kommt und sich umkehrt.

    Wenn man es genau wissen will, muss man allerdings berücksichtigen, dass die Atmosphäre die Sonne beträchtlich (um ca. 1/2°, entsprechend ca. 55 km Breite, was zudem auch noch wetterabhängig ist) anhebt und dass sich die Deklination in Nordsommer und -winter nicht symmetrisch ändern, sondern wegen der elliptischen Bahn der Erde um die Sonne, welche die Erde in Sonnennähr schneller durchschreitet, liegen zwischen Frühjahrs-Tag- und -Nachtgleiche und der Sommersonnenwende 92 Tage, zwischen Herbst-Tag- und -Nachtgleiche und Wintersonnenwende hingegen nur 89 Tage. Entsprechend variiert die Geschwindigkeit, mit der sich die Sonnendeklination ändert.

    Die genaue Formel kann man durch Googeln sicher irgendwo finden.

  817. #827 Carl Wenning
    Kempten
    31. August 2017

    Danke, Alderamin, Dein Wissen hilft mir schon gut weiter, ich werde mich noch kümmern.
    Lieben Gruß,
    Carl

  818. #828 HF(de)
    5. September 2017

    Ich weiß (oder glaube zu wissen), dass Annie Jump Cannon die Einteilung von Sternen in Spektralklassen von 22 oder 26 auf 7 reduziert hat. Und sich nachher herausstellte, dass das tatsächlich sinvoll war (Cecilia Paine-G.).
    s. z.B.: https://www.skyandtelescope.com/astronomy-resources/annie-jump-cannon-star-classifier/
    Aber: was waren ihre Kriterien für die neue Einteilung? (Am besten verglichen mit den alten…)
    Ihre “Intuition” oder was auch immer dabei den Ausschlag geben hat, ist mir absolut nicht klar. Da muss doch ein handfestes Argument dahinter stecken, und nicht nur “so geht’s schneller”.
    Frage(n): wie wurden ab dann (im Vergleich zu vorher) Sterne einsortiert und warum?

  819. #829 Alderamin
    6. September 2017

    @HF(de)

    Die Details weiß ich zwar auch nicht, aber anfangs war noch nicht so klar, wie man die verschiedenen Spektren deuten und sortieren sollte. Eine der frühen Klassifikationen (es gab mehrere) ordnete sie nach der Farbe oder der Linienstärke des Wasserstoffs, mit A für die stärkste Linienbreite, dann B, C, D – N, O für Heliumsterne, P planetarische Nebel und Q für alles, was sonst nirgends passte.

    Man lernte dann, dass es sich letztlich um eine Temperaturklassifikation handelte und oberhalb der Temperatur von weißen A-Sternen die Wasserstofflinien wieder schwächer werden und dafür Helium stärker (B und noch heißer O). Bei den Klassen unterhalb A kommen dann immer mehr Metalle hinzu und schließlich auch Moleküle wie Titanoxid. Ein paar Klassen wurden gestrichen bzw. mit Nachbarklassen zusammengefasst, und so ergab sich die bekannte Reihe O B A F G K M. Man orientiert sich bei der Unterteilung am Vorkommen bestimmter Metalllinien. Die großen Klassen sind jeweils weiter unterteilt in die Unterklassen 0-9 (abnehmende Temperatur; G0 ist fast F, G9 beinahe K).

    Zwischendurch gab’s mal die Klassen R und N für Sterne mit besonders viel Kohlenstoff (R ansonsten ähnlich K, N wie M), die heute unter C laufen. W wurde für Wolf-Rayet-Sterne eingeführt, das sind heiße blaue Riesensterne, die ihre Wasserstoffhülle durch Sternwind verloren haben und wo der Heliumkern blank liegt. Jüngst wurden L, T und Y für braune Zwerge (und heiße Planeten!) eingeführt, mit weiter abnehmender Temperatur. Klasse Y ist schon im Bereich der Zimmertemperatur, solche Objekte strahlen nur noch ein wenig langwelliges Infrarot ab. Also sind wir heute bei O B A F G K M L T Y und den parallelen Sonderklassen C (neben K und M) und W (neben O).

    Dazu gibt’s dann noch die Leuchtkraftklassen, die etwas über die Größe der Sterne aussagen, von I (Überriesen) über II (helle Riesen), III (normale Riesen), IV (Unterriesen), V (Zwerge) bis VI (Unterzwerge), denn es gibt jede Spektralklasse in klein (Hauptreihenstern, Wasserstoffbrennen) und groß (Riesenast, mit höheren Fusionsstufen im Kern, die die Oberfläche aufblasen). Die Sonne ist ein Zwerg der Klasse G2V.

  820. #830 Ambi Valent
    6. September 2017

    @HF(de)
    Als Addendum zu Alderamin’s Beitrag: Die modernen Spektralklassen entstanden in Wesentlichen bei der Arbeit am Draper-Sternkatalog, für die Annie Jump Cannon und andere Frauen angestellt waren.

    Vorher gab es die Secchi-Klassen (I: blau-weiß, II: gelb-orange, III: rot, IV: Kohlenstoffsterne mit kaum blau im Spektrum). Diese Klassen wurden in den ersten Draper-Klassen erstmal nach Eigenheiten der Spektren aufgespalten (Secchi I wurde zu Draper A bis D), und zusätzliche Klassen eingefügt (O, P, Q). Die Umsortierung von B vor A stammt von Antonia Maury.

    Bei der Diskussion innerhalb der Arbeitsgruppen, wie grob oder fein die Sortierung denn sein sollte, war dann der Vorschlag von Annie Jump Cannon erfolgreich. Sie hatte bei Durchsicht des Materials festgestellt, dass einige Klassen auf Bildfehler zurückzuführen waren. Sie legte auch andere Klassen mit ihren ähnlichen Nachbarn zusammen, und fügte dafür die Unterteilung der Klassen neu ein: G5 war zum Beispiel auf der Hälfte zwischen G und K einzuordnen. So gab es dann wenige Hauptklassen (die auf den alten Draper-Klassen basierten und ihre Buchstaben übernahmen), aber auch eine feinere Unterteilung.

  821. #831 HF(de)
    6. September 2017

    Vielen Dank an Alderamin und Ambi Valent! Je mehr ich darüber nachdenke (und das wird noch einige Zeit dauern), desto mehr wird mir klar, dass die ganze Materie nicht sehr einfach ist. Umso mehr Hut ab vor den Frauen, die diese Einteilung vorgenommen haben, die hatten es ja noch viel schwieriger! Der Hausherr hat mal angedacht, etwas drüber zu schreiben, was man aus Spektrallinien herauslesen kann; ich würd mich wahnsinnig freuen. Und ein Beitrag über die Geschichte / historische Entwicklung, die dahinter steckt, würde mich erst recht begeistern (wobei ich durch Euch beide schon einiges gelernt habe — nochmal: danke).

  822. #832 Noonscoomo
    Berlin
    10. September 2017

    Wenn es dunkle Materie gibt und die außer durch Gravitation mit nichts wechselwirkt uns wenn es so viel ist wie man derzeit annimmt, wie kann die dann zwischen den Sternen verteilt sein, so wie man in diesen Diagrammen suggeriert bekommt? Sollte die dann nicht wie die restliche Materie im großen und ganzen da sein wo die Sterne und Planeten sind und zu deren Masse beitragen?

  823. #833 Ambi Valent
    10. September 2017

    @Noonscoomo
    Die normale Materie konnte sich nur zu Sternen und Planeten zusammenfinden, weil sie miteinander interagiert – wenn Materie zusammenstößt, wird Bewegungsenergie in Wärme umgewandelt, und so dass nach der Begegnung Gravitation die Materie zusammenhalten kann.

    Wenn dunkle Materie auf einen Stern zufliegt, dann fliegt sie einfach auf einem hyperbolischen Orbit um dessen Schwerpunkt herum und fliegt dann wieder weg – die Sternoberfläche macht ihr gar nichts aus.

    Dunkle Materie könnte natürlich auch ein Bestandteil eines Sonnensystems sein: ist die dunkle Materie langsam genug, würde sie die Sonne einfach immer weiter umkreisen. Aber es ist auch so, dass die benötigte Fluchtgeschwindigkeit, um einer Galaxie zu entkommen, viel höher ist als die, einem Sonnensystem zu entkommen.

    So tritt der Effekt der dunklen Materie im galaktischen Maßstab deutlich auf, während er im Sonnensystem nicht zu merken ist. Und man nimmt an, dass es weniger so ist, dass Galaxien dunkle Materie festhalten, sondern eher so, dass sich Galaxien dort gebildet haben, wo es schon vorher mehr dunkle Materie gab als anderswo, und diese dann die normale Materie angezogen hat.

  824. #834 noonscoomo
    Berlin
    11. September 2017

    @Ambi Valent
    Ah! Normale Materie-Teilchen halten sich also quasi gegenseitig aneinander fest wenn sie sich begegnen, dunkle-Materie-Teilchen durchdringen sich einfach, lenken ein klitzeklein wenig ihre Flugbahn ab und fliegen einfach weiter… Na vermutlich ist das dann ja gleichzeitig passiert. Dunkle und helle Materie fliegt nach dem Urknall in allen Richtungen durcheinander und die helle Materie knallt aneinander und bleibt kleben (vermutlich spielte da doch Elektroadhäsion auch ne Rolle, oder), die dunkle fliegt weiter.
    Wenn das wirklich so ist müsste das ja der Beleg dafür sein, dass Gravitation auch im Bereich der Elementarteilchen wirkt, denn wenn keine anderen Kräfte wirken und sich trotzdem Cluster bilden, dann geht das ja nur so.
    Komisch, ich hätte vermutet dass das seeehr lange dauert bis sich solche Massezentren bilden wenn lediglich dieser minimale Effekt der… ich nenn das jetzt mal provokativ Quantengravitation… eine Rolle spielt. Gab es die ersten Galaxien nicht schon sehr früh im Universum? Hat das jemand ausgerechnet, dass die Zeit gereicht hätte dass sich genug dunkle Materie zusammenfindet? Oder gibts einen anderen Effekt, der das Beschleunigt haben könnte? Primordiale Schwarze Löcher z.B.?

  825. #835 bikerdet
    11. September 2017

    Hallo Florian
    Ich habe vor kurzem einen Bericht im TV gesehen, wo die Aussage gemacht wurde, das sich alle Sterne in einer Galaxie durch die sie umgebende ‘dunkle Materie’ so gebunden um den Kern bewegen würden, das sie sich nach jeder Umrundung des Zentrums an der selben Stelle befänden. Galaxien würden sich in etwa so wie Schallplatten um das Zentrum bewegen und immer gleich aussehen. Ich hatte das bisher eher so verstanden, das sich auch die Sonnen nach den Keplerschen Gesetzen unterschiedlich schnell bewegen und erst in den äußeren Bereichen durch die dunkle Materie ihre Geschwindigkeit verändern. Aus diesem Grund (unterschiedliche Geschwindigkeit) befände sich auch unsere Sonne nicht mehr in ihrem ‘Geburtshaufen’.
    Könntest Du mir freundlicherweise sagen was nun richtig ist ?

  826. #836 Noonscoomo
    Berlin
    11. September 2017

    @Ambi Valent
    Wenn dunkle Materie nur Masse, also nur Gravitation hat und sonst nix, sollte das aber nicht auch weniger werden wenn sich zwei begegnen? Die verändern ja ihre Richtung dann, d.h. Masse wird beschleunigt, also Arbeit verrichtet ? Nein?

  827. #837 Ambi Valent
    11. September 2017

    @Noonscoomo
    Die Masse folgt nur der gekrümmten Raumzeit, da wird normalerweise keine Arbeit verrichtet. Streng genommen können bei asymmetrischer Massenanordnung Gravitationswellen abgestrahlt werden, aber normalerweise nur in unmessbar winzigen Mengen.

  828. #838 Ambi Valent
    11. September 2017

    @bikerdet
    Der TV-Bericht wollte nur den Effekt beschreiben, hat aber auch da danebengehauen. Du hast im Grunde schon recht – die Sterne bewegen sich unabhängig voneinander und werden von der Gravitation auf ihren Bahnen um das Milchstraßenzentrum gehalten.

    Kepler-Ellipsen sind ein Sonderfall, der sich ergibt, wenn Objekte eine Masse innerhalb von ihnen umkreisen. Dann würde die Stärke der Gravitation mit dem Quadrat der Entfernung abnehmen, da die Masse konstant bleibt. Wenn ein Stern in der Milchtraße allerdings keinen kreisrunden Orbit beschreibt, sondern sich weiter vom Zentrum entfernt, ist mehr Masse zwischen Zentrum und Stern, was der Abnahme der Gravitation entgegenwirkt.

    Dadurch ist die Geschwindigkeit der Sterne weiter außen nicht allzu unterschiedlich von der weiter innen (das ist allerdings mehr oder weniger Zufall, nicht automatisch der Fall). Das heißt aber nicht, dass die Bewegung wie die einer Schallplatte wäre – wäre das so, müssten sich die Sterne außen noch schneller bewegen, um das Milchstraßenzentrum mit der gleichen Winkelgeschwindigkeit zu umkreisen.

  829. #839 noonscoomo
    Berlin
    11. September 2017

    @Ambi Valent
    Ah, ok, aber dann ist doch alles klar. Dunkle Materie verhält sich dann also genau wie eine sich bewegende Delle (Welle) in der Raumzeit. Oder etwa nicht?
    Das nennt man gemeinhin auch Gravitationswelle. Und wenn was watschelt wie ne Ente und Quakt wie ne Ente und aussieht wie ne Ente ist es wohl auch eine. Will sagen, was spricht dagegen, dass dunkle Materie einfach nur die Summe aller Gravitationswelle ist, die irgendwann entstanden sind und jetzt ewig hin und her schwappen? Stellen wir uns einen unendlich grossen See vor, wo öfter mal überall jemand Steine reinschmeist und wo die Wellen nie Energie verlieren ausser an neue Wellen. Und hier und da gibts nen Abfluss… das sind die schwarzen Löcher. Spricht da was gegen?

  830. #840 Ambi Valent
    11. September 2017

    Gravitationswellen bewegen sich immer mit Lichtgeschwindigkeit. Dunkle Materie dagegen ist langsam (sonst könnte sie nicht zB das Milchstraßenzentrum umkreisen).

  831. #841 Noonscoomo
    Berlin
    12. September 2017

    @Ambi Valent
    Argh, du hast recht.
    Prima, Danke. Ich hab richtig was gelernt.

  832. #842 Alderamin
    12. September 2017

    @bikerdet

    Galaxien würden sich in etwa so wie Schallplatten um das Zentrum bewegen und immer gleich aussehen.

    Aussehen vielleicht (Spiralarme), aber für die Sterne trifft das keinesfalls zu. Wenn man sich die Rotationskurve einer Galaxie anschaut sieht man, dass die Umlaufgeschwindigkeit nach außen ziemlich konstant bleibt. Aber der Umfang des Orbits nimmt ja zu, d.h. eine Umdrehung dauert weiter außen trotzdem länger als weiter innen. Nach Kepler würde es halt noch länger dauern.

    Nur die Spiralarme rotieren relativ starr und wickeln sich nicht auf, aber das sind nur Gebiete vermehrter Sternentstehung, die über die Sterne hinweg rollen, keine rigiden Objekte oder Strukturen.

  833. #843 Franz
    12. September 2017

    Eine Frage die vielleicht schon zu sehr in die Physik geht aber irgendwie aus der Astronomie kommt: Wenn die allgemeine Relativitätstheorie Gravitation als ein Produkt des gekrümmten Raums beschreibt, also quasi als Scheinkraft, warum wird Gravitation dann als Grundkraft definiert die man mit den 3 anderen zu vereinheitlichen versucht und weiters auch in der Quantenmechanik Teilchen (Gravitonen) definiert. Wenn sie nur eine Scheinkraft ist wozu das alles ?

  834. #844 bikerdet
    13. September 2017

    @ Ambi Valent in #838
    @ Alderamin in #842

    Vielen Dank für die Erklärungen und den Link. Das hilft mir deutlich weiter !

  835. #845 Jens
    21. September 2017

    Seit Mitte August wird spekuliert dass Ligo/Virgo ein weiteres Gravitationswellenereignis gemessen hat. Dieses Mal sollen zwei Neutronensterne verschmolzen sein. Erstmals soll dabei auch ein zugehöriges elektromagnetisches Signal gemessen wurden sein. Wer weiß noch mehr darüber? Was kann man daraus über die Physik der Neutronensterne in Erfahrung bringen? Florian machst du dazu einen Bericht für uns?

  836. #846 Noonscoomo
    Berlin
    2. Oktober 2017

    Ich hab da mal noch ne Frage,
    vor kurzem hab ich grob Verstanden, warum sich durch die Ausdehnung des Universums ab einer bestimmten Entfernung Galaxien voneinander entfernen, aber z.B. Erde und Sonne nicht, da diese gravitativ aneinander gekoppelt sind.
    Nun war ja aber das Universum früher kleiner, bzw. der Raum zwischen der Materie war „weniger“. Und noch früher noch weniger. Und davor noch weniger. Muss dann nicht alles irgendwann mal gravitativ aneinander gekoppelt gewesen sein.
    Ah, warte, … ist das eines der Argumente für die kosmische Inflation?

  837. #847 Alderamin
    2. Oktober 2017

    @Noonscoomo

    Früher war der Hubble-Parameter einfach entsprechend höher. Mit der Inflation hat das nichts zu tun. Als die stattfand, gab es nur leeren Raum, keine Materie.

  838. #848 Percy Jackson
    @lisboa
    5. Oktober 2017

    Hallo,
    wir lesen gerade “Einstein und die Zeitmaschinen” von Luca Novelli, und da kam die Frage, ob der Mond schwerer wird, wenn die Sonne darauf scheint….

  839. #849 Florian Freistetter
    6. Oktober 2017

    @Percy Jackson: Interessante Frage! Die werd ich mir mal vormerken und hoffentlich in den nächsten Wochen Zeit finden dazu einen eigenen Artikel zur Beantwortung zu schreiben.

  840. #850 Andreas
    10. Oktober 2017

    Wie ist das Winkelverhältnis zwischen der Bewegungsrichtung der Sonne zu den Umlaufbahnen der Planeten? Die Gesamtbewegung der Planeten müsste in irgendeiner Weise spiralförmig sein und dazu habe ich nichts gefunden.

  841. #851 Spritkopf
    10. Oktober 2017

    @Andreas
    Meintest du das hier?

  842. #852 Andreas
    10. Oktober 2017

    @Spritkopf
    genau – danke! da hab ich wohl irgendwie falsch gesucht…

  843. #853 lilalo
    16. Oktober 2017

    Warum ist in Einsteins Formel e=m*c2 die Lichtgeschwindigkeit als Konstante vorhanden und wie ist er darauf gekommen?

  844. #854 Spritkopf
    16. Oktober 2017

    Warum ist in Einsteins Formel e=m*c2 die Lichtgeschwindigkeit als Konstante vorhanden

    Weil die Lichtgeschwindigkeit eine Konstante ist.

    und wie ist er darauf gekommen?

    Es sind schon andere vor ihm darauf gekommen. Eine ganz gute Zusammenfassung des Gedankenwegs findet sich bei Wikipedia.

  845. #855 Larissa
    24. Oktober 2017

    2022 sollen wir eine Nova mit bloßem Auge am Himmel sehen können. Ich habe heraus gefunden dass die 2 Sterne aber 1700 Lichtjahre von uns entfernt sind. Sind die Sterne dann nicht schon lange explodiert oder explodieren noch vor 2022?

  846. #856 Alderamin
    25. Oktober 2017

    @Larissa

    Ja, es geht um KIC 9832227

    Natürlich sind die Sterne, wenn man die Lichtlaufzeit einrechnet, längst verschmolzen, aber für uns ist nur das erfassbare Wirklichkeit, dessen Licht uns erreicht hat. Alles jenseits des sogenannten “Lichtkegels” ist prinzipiell unbeobachtbar, wir können nur darüber spekulieren, was sich dort abspielt.

    Der Lichtkegel fängt schon direkt vor Deinen Augen an. Eine Person die in 1 m Entfernung vor Dir steht, siehst Du mit 3,3 Nanosekunden (Milliardstel Sekunden) Verzögerung, ein fernes Objekt in 10 km Entfernung mit 33 Mikrosekunden (Millionstel Sekunden), was beides viel zu kurz ist, als dass man es bemerken könnte. Vom Mond braucht das Licht aber schon 1,3 Sekunden bis zu uns, von der Sonne 8 Minuten 20 Sekunden und vom Saturn fast anderthalb Stunden. Die Sterne sind dann Lichtjahre entfernt, ihr Licht braucht Jahre, Jahrzehnte, bis zu Jahrtausenden, um uns zu erreichen. Galaxien sind Millionen oder Milliarden Lichtjahre entfernt, und das Alter der Welt begrenzt unseren Blick spätestens beim Urknall.

    KIC 9832227 sehen wir entsprechend mit 1700 Jahren Verzögerung und die Verschmelzung hat schon längst stattgefunden. Aber für uns auf der Erde ist es ja interessant, wann wir das Ereignis beobachten können, und das soll eben 2022 der Fall sein. Die Angabe “fand voraussichtlich vor 1695 Jahren statt” hilft einem ja nicht wirklich, weil man dann immer auch die Entfernung mit angeben müsste, um den Zeitpunkt der möglichen Beobachtung anzugeben – und die ist bei Sternen und Galaxien oft nur ungefähr bekannt. Deswegen orientieren sich Astronomen an dem, was man sieht (oder messen kann) und wann man es sieht.

  847. #857 Udo Naumann
    Heidenau
    29. Oktober 2017

    Ich verstehe nicht, warum man Signale (Licht usw) bis relativ zeitlich nah zum Urknall empfangen kann.
    Wenn diese Signale in Lichtgeschwindigkeit von diesem Ort zu uns eilen, dann müßten wir uns mit annähernd Lichtgeschwindigkeit von diesem allgemeinen Geburtsort fortbewegen und diese Signale uns nach so langer Zeit nun einholen. Die Signale müßten uns auch deutlich langsamer als mit Lichtgeschwindigkeit erreichen.
    Ist der Raum aus dem wir kommen, aber kein geradlinig entfalteter Raum, sondern hinter uns gewissermaßen gefaltet wurden, dann könnte ich diesen Vorgang verstehen.

  848. #858 Ambi Valent
    4. November 2017

    @Udo Naumann

    Es gibt keinen “allgemeinen Geburtsort” irgendwo im Weltraum – der Weltraum wurde überall gleichzeitig geboren. Und da von allen Punkten im jungen, dichten, heißen Weltall Strahlung ausging, gibt es eben Punkte, von denen die Strahlung von damals gerade jetzt ankommt. In 1000 Jahren (oder Millionen, oder Milliarden) wird immer noch Strahlung ankommen, die in genau demselben Zeitpunkt ausgestrahlt wurde, nur eben von Punkten, die weiter weg sind als die, von denen wir heute Strahlung empfangen.

  849. #859 Thomas
    10. November 2017

    Hallo Florian, was ist dran am “Zombiestern” (SPON)?

  850. #860 Alderamin
    12. November 2017

    @Thomas

    Man vermutet dahinter eine Paar-Instabilitäts-Supernova. Die sollte nur bei sehr massiven Sternen von mehr als 100 Sonnenmassen auftreten. Solche Sterne können heutzutage wegen der “Verureinigung” des Gases mit schweren Elementen früherer Sterngenerationen eigentlich nicht mehr entstehen. Ausnahmen bestätigen mal wieder die Regel.

    https://www.scinexx.de/wissen-aktuell-22070-2017-11-09.html
    https://www.skyandtelescope.com/astronomy-news/not-dead-yet-star-wouldnt-die/

  851. #861 Ambi Valent
    13. November 2017

    Phil Plait hat inzwischen auch seinen Artikel dazu bei Bad Astronomy:

    https://www.syfy.com/syfywire/the-star-that-blew-up-a-little-then-blew-up-a-lot

  852. #863 Noonscoomo
    Berlin
    15. November 2017

    Wenn etwas in ein schwarzes Loch fällt, dann wird ein weit entfernter Beobachter, also wir, sehen (wenn wir es sehen könnten), dass das Objekt scheinbar immer stärker verzerrt und die Strahlung immer langwelliger wird. Von uns aus gesehen wird das Objekt nie den Ereignishorizont überschreiten und die Zeit wird für das Objekt scheinbar immer langsamer verlaufen. Davon merkt das Objekt selbst zwar nix, aber für uns entzieht sich dieser Vorgang der Beobachtung. Nun ist ein zweites schwarzes Loch aber auch ein Objekt. Sollte sich dann das Verschmelzen zweier schwarzer Löcher nicht auch unserer weit entfernten Beobachtung entziehen. Sollten die Gravitationwellen nicht, von uns aus gesehen, so stark rotverschoben sein, dass sie nicht mehr messbar sind? Oder entstehen die weit genug entfernt um nicht zu sehr der Zeitdillatation unterworfen zu sein?

  853. #864 Ambi Valent
    18. November 2017

    @Noonscoomo

    Die Gravitationswellen sind ein Effekt, den die Schwarzen Löcher auf die sie umgebende Raumzeit haben, und sind gewissermaßen eine spezielle Ausprägung des Gravitationsfelds. (Ähnlich wie Photonen, die eine spezielle Ausprägung des elektromagnetischen Felds sind)

    Die gegen unendlich gehende Rotverschiebung hast du, wenn das hereinfallende Objekt so nah an das Schwarze Loch herankommt, dass die beiden fast schon ein gemeinsames Objekt sind. Das passiert aber bei der Verschmelzung zweier Schwarzer Löcher erst ganz am Ende.

    Dem geht eine lange Phase voraus, wo die beiden Schwarzen Löcher weit genug auseinander bleiben, während sie einander umkreisen. Je kleiner der Radius dieses Orbits ist, desto mehr Gravitationswellen werden erzeugt. Theoretisch würden Erde und Sonne auch Gravitationswellen abstrahlen, während sie einander umkreisen, aber bei Erde und Sonne wäre der Betrag unmessbar winzig, während er bei sich eng umkreisenden Schwarzen Löchern messbar ist.

    Dadurch, dass die beiden Schwarzen Löcher noch getrennte Objekte sind, können die Gravitationswellen ohne Probleme entkommen. Durch diese Abstrahlung werden die Schwarzen Löcher aber auf immer engere Orbits gelenkt, wobei dann immer mehr Gravitationswellen abgestrahlt werden. Erst in dieser Phase wird der Effekt von der Erde aus gesehen ausgeprägt genug, um als Gravitationswellen klar identfiziert werden zu können.

    Aber diese Phase des immer engeren Orbits endet natürlich, wenn sich die beiden Schwarzen Löcher so nahe kommen, dass sie schließlich als ein gemeinsames Schwarzes Loch erscheinen – und dann kann auch keine Information mehr von den “Komponenten” entkommen, inklusive Gravitationswellen.

  854. #865 noonscoomo
    Berlin
    18. November 2017

    @Ambi Valent
    Cool, danke. So ungefähr hab ich mir das vorgestellt. Ist denn folgende Überlegung von mir auch korrekt?
    Wenn ich (mal davon ab, dass es mich zerreissen würde) knapp über dem Ereignishorizont bleibe, indem ich mich extrem stark weg vom schwarzen Loch beschleunige, dann geht doch nicht nur für einen aussenstehenden Beobachter meine Uhr langsamer, sondern ich sehe, wie die Uhren der Aussenstehenden schneller gehen. Will sagen, wenn ich da verweile dann sehe ich das Universum schneller altern als wenn ich weit weg wäre, oder? Soweit ich das verstehe ist das eine Folge meiner Beschleunigung, richtig? Will sagen, wenn ich da einfach reinfalle sieht zwar ein Aussenstehender, wie meine Uhr immer langsamer geht und ich immer mehr rotverschoben erscheine, ich aber merke davon gar nix sonder falle umbeschleunigt in das schwarze Loch und das Universum altert für mich normal schnell, richtig?

  855. #866 Peter
    Berlin
    27. November 2017

    Hallo zusammen, hier meine ersten Fragen:

    Ich kann der Urknall-Theorie nicht recht Glauben schenken, weil ich mir nicht vorstellen kann, dass die gewaltige Masse von Trilliarden von Sternen und Planeten derart zusammengepresst sein kann, wie es angeblich beim Urknall der Fall war.

    Dann kann ich mir auch nicht vorstellen, dass unser Universum räumlich endlich ist. Es werden ja im Internet Bilder der Form des Universums gezeigt aber was ist außerhalb dieser Form? Für mich zumindest leerer Raum.

  856. #868 tau ceti
    27. November 2017

    Zusätzlich möchte ich noch auf diese Reihe hinweisen:

    https://urknall-weltall-leben.de/videos#

    Und wegen Glauben und Vorstellung. Es ist mehr ein Ding der Akzeptanz statt der Vorstellung. Ebenso wie man akzeptiert und ständig anwendet, das nicht durch Null geteilt werden kann, obwohl man sich das vielleicht “nicht vorstellen” kann. Weil 2 durch “garnichts” ist doch immer noch 2? Ein Mathematiker wird dies schlüssig nachweisen können, ebenso wie Astrophysiker schlüssig ihre Theorien nachweisen können.

  857. #869 Alderamin
    27. November 2017

    @Peter

    Ich kann der Urknall-Theorie nicht recht Glauben schenken, weil ich mir nicht vorstellen kann, dass die gewaltige Masse von Trilliarden von Sternen und Planeten derart zusammengepresst sein kann, wie es angeblich beim Urknall der Fall war.

    Wenn man Materie derart zusammenpresst, existiert sie nicht mehr als solche, und das kann man teilweise in Beschleunigern wie dem LHC am CERN in Genf nachvollziehen. Die Urknalltheorie besagt, dass es zunächst nur ein Vakuum mit einer gewissen, extrem hohen Vakuumenergie gab (auch ein Vakuum kann eine Energie haben). Die hohe Energie zwang das Vakuum, sich auszudehnen, und während einer kurzen Inflationsphase wuchs es gewaltig an, um den Faktor 10^30 bis 10^70 in nur 10^-32 s.

    Danach fiel das Vakuum spontan auf einen Zustand niedrigerer Energie und expandierte langsamer. Die überschüssige Vakuumenergie wurde als Strahlung (Photonen) frei. Das Universum hatte damals eine Temperatur von 10^95-10^99 K. Da gab es keine Teilchen, die Bestand haben konnten, außer Strahlung, und Strahlung benötigt keinen Platz, Photonen sind elektromagnetische Felder, die können sich beliebig überlagern, brauchen nur Platz für ihre Wellenlänge, und die war extrem klein bei solchen Energien.

    Mit weiterer, langsamerer Expansion kühlte das Universum dann weiter ab, bis bei 10^27- 10^12 K die ersten Teilchen, Quarks, Gluonen und ihre Antiteilchen, entstanden. Teilchen und Antiteilchen vernichteten sich meist gleich wieder, aber es entstand aus noch ungeklärten Gründen ein kleiner Überschuss von Teilchen über die Antiteilchen, während letztere verschwanden (mit Teilchenpartnern wieder zu Strahlung wurden).

    Quarks, Gluonen und andere Elementarteilchen sind punktförmig. Erst als sich Quarks zu Protonen und Neutronen zusammenfanden, gab es ausgedehnte Teilchen. Die Dichte des Universums war nach 0,0001s am Ende der als Hadronenära bezeichneten Zeit, in der die Protonen und Neutronen entstanden, fast so hoch wie in einem Atomkern oder Neutronenstern und das heute beobachtbare Universum hatte damals schon einen Radius von größenordnungsmäßig über 100 Milliarden km. Darin war genug Platz für alle heute vorhandene Materie im sichtbaren Universum, sie war nur extrem dicht gepackt; in Neutronensternen finden wir heute noch so dichte Materie.

    Nach 1 s waren alle Teilchenarten entstanden, das heute beobachtbare Universum maß schon rund 500 Lichtjahre im Radius. Das Universum war nun mit einem heißen Gas aus Atomkernen und Elektronen, einem Plasma gefüllt, wie im Inneren der Sonne. In den nächsten paar Minuten entstanden darin Helium- und Lithiumkerne durch Kernfusion, so wie in der Sonne Elemente entstehen, bis die weitere Expansion und Abkühlung die Fusionen beendeten.

    Nach 380000 Jahren hatte sich das Plasma soweit abgekühlt, dass die Kerne und Elektronen sich zu Atomen und Molekülen verbinden konnten. Das heute beobachtbare Universum hatte zu dieser Zeit einen Radius von 43 Millionen Lichtjahren. Danach bildeten sich allmählich die Strukturen aus, die wir heute kennen, Filamente aus Galaxienhaufen mit Leerräumen dazwischen.

    Heute beträgt der Radius des beobachtbaren Universums etwa 47 Milliarden Lichtjahre. Allerdings sehen wir beim Blick in die Ferne auch in die Vergangenheit, deswegen erscheint es uns kleiner, denn die fernsten Galaxien waren uns viel näher, als ihr Licht sich auf den Weg zu uns machte. Während das Licht unterwegs war, wuchs die Entfernung an und so erreicht es uns heute zu einem Weltalter von 13,8 Milliarden Jahren von Orten, die heute schon bis zu 47 Milliarden Lichtjahre entfernt sind.

    Dann kann ich mir auch nicht vorstellen, dass unser Universum räumlich endlich ist. Es werden ja im Internet Bilder der Form des Universums gezeigt aber was ist außerhalb dieser Form? Für mich zumindest leerer Raum.

    Bilder können das Universum nicht mathematisch getreu darstellen, sondern nur ein paar Grundideen vermitteln. Das Universum könnte einerseits unendlich groß sein (wobei wir aber nur einen endlichen Teil überblicken können, weil das Universum endlich alt ist und das Licht eine endliche Geschwindigkeit hat), es könnte aber auch endlich und in sich geschlossen sein. Eine Kugeloberfläche ist auch endlich groß und ohne Rand, braucht aber eine höhere Dimension, in der sie eingebettet ist, eben den Raum. Das Universum hat schon drei Raumdimensionen, muss dabei aber nicht in eine höhere Dimension eingebettet sein. Zum Beispiel krümmt nach der allgemeinen Relativitätstheorie Masse den Raum, eine Verzerrung der Geometrie, aber auch die braucht keine höhere Dimension.

    Eine mögliche Form des Universums ist ein 3-Torus. Ein Beispiel für einen 3-Torus ist ein dreidimensionaler Würfel, bei dem man wie in manchen Computerspielen beim Verlassen des Würfels auf der einen Seite an der gegenüberliegenden Seite wieder herein kommt. Ein Außerhalb gäbe es dann nicht, man bliebe in diesem Universum gefangen. Wäre der Blick nicht durch den Horizont der Lichtlaufzeit begrenzt und könnte man unendlich weit sehen, dann würde man sich selbst wie zwischen zwei Spiegeln in der Ferne endlos wiederholt sehen können (aber nur den Hinterkopf). Das Universum ist aber mit großer Sicherheit erheblich größer als der Teil, den wir überblicken können, deswegen werden wir die Milchstraße nicht in der Ferne wiederfinden.

    Möglicherweise ist unser Universum auch eingebettet in einen unermesslichen Raum, der seit der Inflationsphase immer weiter expandiert ist, und nur lokal entstehen in ihm kleine Blasen, die langsamer expandieren, wobei unser Universum eine solche Blase ist. Man muss sich die Expansion des Raums als ein Entstehen von neuem Raum aus sich sebst heraus vorstellen, der aufquillt wie ein Hefeteig, bei der Inflation explosionsartig und heute sehr viel langsamer. Raum ist dann nichts festes, unbewegliches, sondern beliebig dehn- und formbar. Die Materie hat vom Beginn der Expansion einen gewissen Schwung mitbekommen und zieht gemäß der Allgemeinen Relativitätstheorie (die die Raumkrümmung beschreibt) den Raum mit sich mit und auseinander. Es braucht also gar keinen umgebenden Raum, in den sich das Universum ausdehnen kann. Es kann aus sich heraus wachsen.

  858. #870 Alderamin
    27. November 2017

    @tau ceti

    Ebenso wie man akzeptiert und ständig anwendet, das nicht durch Null geteilt werden kann, obwohl man sich das vielleicht “nicht vorstellen” kann.

    Warum man nicht durch 0 teilen darf:
    Bekanntlich ist jede Zahl mal 0 = 0, also etwa 5*0 = 0 und 3*0=0. Also 5*0 = 3*0. Wenn man durch 0 teilen dürfte, fiele die 0 weg (denn jede Zahl geteilt durch sich selbst gibt 1, etwa 2/2, also auch 0/0), also wäre 5=3. Das ist offensichtlich falsch. Tatsächlich ist weder 0/0 definiert, doch darf man durch 0 teilen, sonst kommt Quatsch heraus.

    Weil 2 durch “garnichts” ist doch immer noch 2?

    Nein.
    2 durch 10 ist 0,2
    2 durch 1 ist 2.
    2 durch 1/10 ist 20.
    2 durch 1/1000 ist 2000.
    Und je kleiner die Zahl wird, durch die ich 2 teile, desto größer wird das Ergebnis. Das Ergebnis strebt gegen unendlich, wenn der Teiler gegen 0 strebt. 2 durch gar nichts ist unendlich viel. Wenn ich 2 g Gas auf 0 cm³ Volumen komprimieren wollte, würde der Druck unendlich groß.

    Ein Mathematiker wird dies schlüssig nachweisen können, ebenso wie Astrophysiker schlüssig ihre Theorien nachweisen können.

    Mathematiker können Beweise führen, die exakt sind, denn sie geben sich selbst ihre Regeln vor. Naturwissenschaftler versuchen die Regeln der Natur zu ergründen, sie können also nur Modelle der Natur anfertigen, daraus Schlüsse ziehen, und versuchen, aus diesen Vorhersagen abzuleiten, die sie dann in der Natur überprüfen können. Wenn die Vorhersagen erfolgreich bestätigt werden können, ist das Modell gut. Wenn nicht, braucht man ein anderes. Das, was in der Astrophysik gelehrt wird, sind die derzeit besten Modelle, die wir bis jetzt kennen. Und unter denen sind manche sehr gut bestätigt, wie die Urknalltheorie, und andere noch nicht, wie die Erklärungsversuche für die dunkle Materie.

  859. #871 tau ceti
    27. November 2017

    Genau, danke Alderamin.

    Wollte darauf hinweisen, das diese Wissenschaft zwar auf hohem Niveau ist, aber eben immer begründet. Auch wenn man es sich nicht vorstellen kann.

  860. #872 Peter
    27. November 2017

    Dankeschön @Florian Freistetter und @Alderamin!
    Ein wenig haben mir Eure Informationen geholfen.
    Ich bin kein grundsätzlicher Gegner der Urknalltheorie.

    Was ich weiterhin nicht verstehe: Der Urknall war doch eine Explosion, bei der normalerweise, wenn diese im luftleeren Raum stattfindet, alles in alle Richtungen davon fliegt, eine riesige, sich nach außen erweiternde Kugelform entstehen würde. Beim Urknall jedoch fliegt alles mehr oder weniger in Schlauchform in eine Richtung. Woran liegt das? Gab es eine Kraft, die ein nach hinten Explodieren verhinderte? Wie kommt es, dass Galaxien kollidieren? Müssten sie nicht alle die gleiche Geschwindigkeit haben und sich zumindest halbkreisförmig voneinander entfernen?

  861. #873 tau ceti
    27. November 2017

    ” Beim Urknall jedoch fliegt alles mehr oder weniger in Schlauchform in eine Richtung. Woran liegt das? ”

    Weil dies nur zweidimensionale Abbildungen sind.

  862. #874 Alderamin
    27. November 2017

    @Peter

    Der Urknall war doch eine Explosion, bei der normalerweise, wenn diese im luftleeren Raum stattfindet, alles in alle Richtungen davon fliegt, eine riesige, sich nach außen erweiternde Kugelform entstehen würde.

    Nein, wie ich oben schon sagte, gibt es kein „Außerhalb“, das Universum „explodierte“ überall, indem sich einfach sein Innenraum an jeder Stelle vergrößerte wie ein aufgehender Hefekuc‌hen (während der Inflationsphase ruckartig), insofern kann man von keiner bestimmten Form sprechen, es gibt keinen Rand (nur beim Kuch‌en, da versagt das Bild). Das Universum könnte ja auch unendlich groß sein, in dem Fall war es das auch schon zu Beginn, aber jedes Teilvolumen wuchs um einen riesigen Faktor an. So, als ob Du den Zahlenstrahl der reellen Zahlen von -unendlich bis +unendlich betrachtest, und alle Zahlen mit einem riesigen Faktor multiplizierst – da gibt‘s auch kein Maxim‌um oder Mini‌mum als Ränder.

    Beim Urknall jedoch fliegt alles mehr oder weniger in Schlauchform in eine Richtung. Woran liegt das?

    Meinst Du so was? Der „Schla‌uch“ entsteht hier dadurch, dass die Dime‌nsion links-rechts die Zeit darstellen soll. Der Raum ist hier auf die beiden Dimen‌sionen oben/unten und vorne/hinten reduziert. Der Schla‌uch soll auch nicht hohl sein, sondern sein Quersc‌hnitt soll das gleichmäßig ausgefüllte Volumen des Universums zu einer bestimmten Zeit darstellen. Ist halt kein Fil‌m, die Zeitdime‌nsion lässt sich nur andeuten. Das ist das Problem mit Bildern.

    Wie kommt es, dass Galaxien kollidieren? Müssten sie nicht alle die gleiche Geschwindigkeit haben und sich zumindest halbkreisförmig voneinander entfernen?

    Die Expansion lief (und läuft) auf kurzen Entfernungen langsam ab, z.B. wächst eine Strecke der Länge bis zur Andromedagalaxie nur mit etwa 50 km/s (die doppelte Strecke mit 100 km/s, die zehnfache mit 500 km/s und so weiter). Aber die Galaxien ziehen sich gegenseitig an, und Milchstraße und Andromeda fallen mit 400 km/s aufeinander zu. Das nach dem Urknall entstandene Plasma war nicht völlig homogen im Raum verteilt, sondern hatte geringe Dichteschwankungen aufgrund von Quantenfluktuationen im frühesten Vakuum, die von der Inflation gigantisch aufgebläht worden waren. Dorthin, wo die Dichte zufällig größer war, begann das umgebende Ga‌s zu fließen, und nur so konnten daraus Galaxien und Galaxienhau‌fen entstehen, die bis heute aneinander gebunden sind und sich umeinander und aufeinander zu bewegen. Das ganze Weltall ist wie von einem Gesp‌inst aus Fäde‌n von Materie durchzogen, entlang derer die Galaxien entstanden sind, mit Hohlrä‌umen dazwischen. Während die Fäd‌en sich zusammenziehen, wachsen die Hohlräu‌me dazwischen. Über Entfernungen von mehr als rund 60 Millionen Lichtjahre überwiegt die Expansion des Universums die Eigenbewegung der Galaxien und derart weit voneinander entfernte Galaxien werden durch das Anwachsen des Raums zwischen ihnen auseinander gezogen, obwohl sie ansonsten aufeinander zu fallen würden. Sie fallen zu langsam.

  863. #875 Rudolf
    Dreieich
    14. Dezember 2017

    Warum nennt man das Nichts Nichts, wenn daraus zufällig etwas entstehen kann?

  864. #876 Rudolf
    Dreieich
    14. Dezember 2017

    Kann ein “schwarzes Loch” dunkle Materie einfangen? Welche Auswirkungen dunkler Materie in einem “schwarzem Loch” könnte dies auf die Harkwing-Strahlung haben?

  865. #877 Alderamin
    14. Dezember 2017

    @Rudolf

    Zur ersten Frage:

    Es ist keineswegs eine gesicherte Erkenntnis, dass das Universum aus “Nichts” entstanden ist, sondern eine von mehreren Arbeitshypothesen. Mit unseren heutigen Möglichkeiten können wir keine gesicherten Aussagen darüber machen, wie das Universum genau begann, wir können nur so weit zurückrechnen, wie wir die Naturgesetze austesten können, z.B. extrem hohe Energien in Beschleunigern erzeugen, damit kommt man auf Bruchteile von Sekunden an den vermuteten Beginn des Universums heran, als die ersten Partikel entstanden, aber nicht beliebig nahe. Wir können auch nicht in so kleine Dimensionen vordringen, dass wir verifzieren können, wie der Raum sich auf kleinstem Niveau verhält, ob er beispielsweise gequantelt ist und ob Raumquanten einfach so entstehen können, wie die vielzitierten virtuellen Teilchen es im Vakuum tun sollen. Dazu fehlt uns eine Theorie, die Raumzeit (Relativitätstheorie) und Quantentheorie in Einklang bringt.

    Aber es gibt solche Überlegungen. Einer der lautesten Verfechter des “Universum aus dem Nichts” ist Lawrence Krauss. Hier (neben “Transcript” auf “Show” klicken, um den Text lesen zu können) erklärt er in einem Interview, wie er überhaupt “Nichts” definiert (ist es die Abwesenheit von Materie, die Abwesenheit von Raumzeit oder muss noch mehr fehlen?).

    Solange es noch keine Theorie der Quantengravitation gibt, ist das eine philosophische, keine naturwissenschaftliche Frage. Es geht letztlich darum, eine alternative Erklärung dafür zu geben, dass ein Schöpfer alles gemacht haben soll (was nicht besonders plausibel ist, denn wo soll der dann herkommen?). Alternativ könnte es auch ein Vorgängeruniversum gegeben haben, das kollabiert ist, oder von dem sich unseres abgespalten hat, oder unseres ist neben vielen anderen eine Art wachsende Schaumblase in einem darum herum inflationär expandierenden Raum, oder es war schon vorher da und kollidierte mit einem anderen, was den Urknall auslöste… es gibt da zahlreiche Ideen, und wir können derzeit nicht entscheiden, welche davon (und ob überhaupt eine) richtig ist.

  866. #878 Alderamin
    14. Dezember 2017

    @Rudolf

    Zur zweiten Frage:

    Kann ein “schwarzes Loch” dunkle Materie einfangen?

    Die Antwort lautet hier “ja, aber”. Wenn die dunkle Materie aus Partikeln (oder gar selbst aus schwarzen Löchern) besteht, können diese natürlich in ein schwarzes Loch hineinfallen. Aber ein schwarzes Loch ist ein sehr kleines Ziel; nur ein paar 10 bis 100 km groß, wenn es ein stellares ist, höchstens einige hundert Millionen km für supermassive schwarze Löcher, was kosmisch gesehen immer noch klein ist. Schwarze Löcher sind nun keine Staubsauger, die alles in ihrer Umgebung ansaugen, sondern sie sind nur sehr dichte Massen, um die Objekte genau so kreisen, wie sie dies um Sterne der gleichen Masse tun würden – nur kann man dem Massenzentrum bei Schwarzen Löchern viel näher kommen, als bei einem Stern, weil man bei letzterem vorher mit seiner Oberfläche kollidiert. Da die Schwerkraft mit abnehmendem Abstand von der Masse quadratisch größer wird, wird sie bei so dichten Objekten wie Schwarzen Löchern absurd hoch, so dass unterhalb eines bestimmten Radius selbst Licht (und damit auch nichts anderes) ihnen nicht mehr entkommen kann, ohne dass man dort schon eine Oberfläche angetroffen hätte. Möglicherweise gibt es gar keine und die Masse steckt in einem Punkt (auch hier fehlt noch ein Theorie der Quantengravitation, die darüber Auskunft geben könnte).

    Wenn ein Objekt von einem schwarzen Loch angezogen wird, fällt es also in Richtung des schwarzen Loches und wird dabei schneller. Wenn es nicht senkrecht auf den Ereignishorizont zu fällt, wird es in eine gekrümmte Keplerbahn (Ellipse, Parabel, Hyperbel) gezwungen, und eine gekrümmte Bahn bedeutet, es gibt eine Fliehkraft, die mit zunehmender Geschwindigkeit größer wird und der Schwerkraft entgegengerichtet ist. Diese hindert das Objekt daran, einfach in das schwarze Loch zu fallen – so, wie sie Satelliten und den Mond daran hindert, auf die Erde zu stürzen, und diese daran, geradewegs in die Sonne zu fallen. Sie fallen ständig an der anziehenden Masse vorbei. Falls Du Dich etwas in der Physik auskennst, es müssen Bahndrehimpuls und die Summe aus potenzieller und kinetischer Energie auf der Bahn erhalten bleiben. So entstehen die Keplerbahnen.

    Wie kann dann überhaupt etwas in ein schwarzes Loch fallen? Wenn ein normaler Stern eng an einem schwarzen Loch vorbei kommt, wird er zerrissen, es handelt sich ja um Gas, und das Gas kann durch Reibung und die entstehende Wärme Bewegungsenergie abstrahlen und sich so in einer Scheibe um das Schwarze Loch sammeln: Partikel, die sich relativ zur Scheibenebene auf und ab bewegen, kollidieren mit Partikeln in der Scheibe, und zwar so lange, bis sich alles in der Scheibenebene dreht. Und die Scheibe dreht sich innen schneller als außen (wegen der dort höheren Schwerkraft), also reiben benachbarte Partikel aneinander. Wenn das Material in der Scheibe durch Reibung ein paar tausend K heiß ist, dann ist es ein Plasma, Elektronen und Atomkerne sind getrennt und bewegen sich unabhängig, dann entstehen Magnetfelder, die die Bewegung der Teilchen zusätzlich beeinflussen und abbremsen können. So kann das Gas spiralförmig nach innen wandern und den Ereignishorizont erreichen.

    Wenn die dunkle Materie, wie bisher vermutet, aus Teilchen besteht, die nicht elektromagnetisch wechselwirken, dann können sie keine Strahlung abgeben und wirken auch nicht über elektromagnetische Kräfte aufeinander (letztlich ist es genau das, was normale Materie bei Kollisionen tut: Elektronen stoßen sich ab, Kerne ebenso, geraten dabei ins Schwingen und strahlen so Energie als Licht, Wärmestrahlung oder Radiowellen ab). Dann fallen sie einfach aneinander vorbei, und ihre Schwerkraft ist nur in der Lage, eine große Wolke von Teilchen in einem gewissen Volumen zusammen zu halten; Teilchen, die nahe aneinander vorbei fallen, beeinflussen sich kaum. Damit fehlt ihnen die Möglichkeit, ihre Bewegungsenergie los zu werden, und sie umschwirren das schwarze Loch einfach wie ein großer Bienenschwarm.

    Lange Rede, kurzer Sinn: Dunkle Materie hat es ungleich schwerer, in ein schwarzes Loch zu fallen, als normale Materie.

    Welche Auswirkungen dunkler Materie in einem “schwarzem Loch” könnte dies auf die Harkwing-Strahlung haben?

    Ich denke mal, keine besondere, weil es egal sein sollte, welches Teilchen in ein schwarzes Loch fällt; die Information darüber, was es für ein Teilchen war, geht verloren, bis auf seine Ladung, Masse und Drehimpuls. Aber da bin ich mir nicht sicher, es gibt ja mittlerweile die Aussage, dass schwarze Löcher vielleicht doch nicht alle Informationen löschen, sondern diese irgendwie kurz vor dem Ereignishorizont eingefroren bleiben, aber was das für Konsequenzen auf die Hawking-Strahlung hat, kann ich nicht beurteilen. Da ist im Moment noch einiges im Fluss.

  867. #879 Christian
    Wien
    25. Dezember 2017

    Servus Florian!

    Ich vereinfache das Doppelpulsarsystem PSR J0737-3039, deren Exponenten 1,34 und 1,25 Sonnenmassen aufweisen, der Einfachheit halber zu einem solchen, wo beide Neutronensterne jeweils eine Masse von 2,6*10^30 kg haben. Wie bei PSR J0737-3039 sollen beide Sterne einen Abstand von 850.000 km zueinander haben, die ihren gemeinsamen Schwerpunkt der Einfachheit halber kreisförmig umkreisen. Ihr Radius sei 10 km.

    1 Neutronenstern:
    m = 2,6*10^30 kg
    r1 = 10^4 m
    r2 = 4,25*10^8 m

    t = t0*√(1-rs/r) | rs = (2*m*G)/c^2

    t0 = 1 s
    t1 = 0,784 s
    t2 = 0,999995 s
    _______________________________________________________

    2 Neutronensterne:
    Im Abstand von r2 befindet sich aber auch der Schwerpunkt des Neutronensternsystems. Welche Zeitdilatation besteht im Schwerpunkt? t0 oder t2?

    Besten Dank!
    Christian

  868. #880 Christian
    Wien
    25. Dezember 2017

    Ergänzend:

    Im Schwerpunkt des obigen Neutronensternsystems heben einander die Schwerebeschleunigungen der beiden Sterne auf. Das tun sie auch im Mittelpunkt eines einzigen, freistehenden Neutronensterns. Welche Zeitdilatation besteht im Mittelpunkt dieses Neutronensterns? t0 oder tx?

  869. #881 Karl-Heinz
    25. Dezember 2017

    @Christian
    Eine sehr interessante Frage.
    Was meinst du, ist die Frage schwierig oder einfach zu lösen?

  870. #882 Higgs-Teilchen
    Im Standardmodell oben rechts
    25. Dezember 2017

    Kann man eigentlich irgendwie abschätzen, wie viele Sterne es bis heute gab? Also auch wie viele Population 2 und 3 Sterne es gab?

    Lg Higgs

  871. #883 Christian
    Wien
    26. Dezember 2017

    Nochmals Hallo!

    Wenn ich mich in Richtung eines Regenbogens bewege, dann sollten die Regenbogenfarben rotverschoben sein, bewege ich mich in die Gegenrichtung, also in Richtung der Sonne, sollten die Regenbogenfarben blauverschoben sein. Der Raumwinkel, unter dem ich den Regenbogen sehe, bleibt dabei immer gleich und somit bleibt auch der Abstand zum Regenbogen immer der gleiche.

    Besteht hier irgendeine Analogie zur Kosmischen Hintergrundstrahlung? Denn auch hier gibt es aus Sicht der Erde eine bewegungsrichtungsabhängige Farbverschiebung, was an und für sich bedeuten würde, dass sich die Kosmische Hintergrundstrahlung in einem bestimmten schalenförmigen Raumbereich rund um die Erde befindet, der theoretisch auch mit einem Raumschiff erreicht werden könnte. Tatsächlich erfüllt die Kosmische Hintergrundstrahlung aber den gesamten Raum, wobei beispielsweise ein Beobachter in der von der Milchstraße 2,5 Millionen Lichtjahre entfernten Nachbargalaxie “Andromeda” derzeit genau dasselbe sehen könnte wie wir auf der Erde, nämlich, dass die Kosmische Hintergrundstrahlung 13.810.000.000 minus 380.000 Lichtjahre von ihm entfernt ist.

    Nochmals besten Dank!
    Christian

  872. #884 Christian
    Wien
    26. Dezember 2017

    Im obigen Beitrag #883 soll es richtig heißen “… dass sich die Quelle der Kosmischen Hintergrundstrahlung in einem bestimmten schalenförmigen Raumbereich rund um die Erde befindet”.

  873. #885 Karl-Heinz
    29. Dezember 2017

    @Christian, Antwort zu #879,#880

    Im Feld einer statischen, sphärischen Masse M (Schwarzschild-Lösung) gilt für die Eigenzeitintervalle d\tau_{1} bzw. d\tau_{2} zweier Uhren die Relation
    \frac{d\tau_{2}}{d\tau_{1}}=\sqrt{\frac{1-\frac{r_{s}}{r_{2}}}{1-\frac{r_{s}}{r_{1}}}}=\sqrt{\frac{1+2\frac{\Phi_{2}}{c^2}}{1+2\frac{\Phi_{1}}{c^2}}} \approx 1+ \frac{\Phi_{2} - \Phi_{1}}{c^2}

    Dabei ist \Phi = \frac{-Gm}{r} das Newtonsche Gravitationspotential und r_{s}=\frac{2Gm}{c^2} der Schwarzschild-Radius.

    Wollen wir jetzt die Zeitdilatation bezüglich einem Beobachter_1 betrachten der sehr weit entfernt ist, so vereinfacht sich die Formel zu \frac{d\tau_{2}}{d\tau_{1}}=\sqrt{1-\frac{r_{s}}{r_{2}}}=\sqrt{1+2\frac{\Phi_{2}}{c^2}} \approx 1+ \frac{\Phi_{2} }{c^2}

    Deine Angaben:
    Zwei Neutronensterne die sich gegenseitig umkreisen mit je einer Masse von m = 2,6*10^{30} kg und einem Radius von R= 10^4 m und einem Abstand D=8,5 \cdot 10^8 m

    Gesucht:
    Zeitdilatation auf der Oberfläche des Neutronensterns und Zeitdilatation im gemeinsamen Schwerpunkt.

    Berechnung:
    Potential des weit entfernten Beobachters:
    \Phi_{1}=0
    Potential auf der Oberfläche des Neutronensterns:
    \Phi_{N}=\frac{-Gm}{R} + \frac{-Gm}{D} = -1,73 \cdot 10^{16} J/kg  -2,04  \cdot 10^{11} J/kg
    \Phi_{N}\approx -1,73 \cdot 10^{16} J/kg
    Potential im Schwerpunkt:
    \Phi_{S}= \frac{-Gm}{D} \cdot 4 = -8,16  \cdot 10^{11} J/kg
    Lösung:
    d\tau_{1}=1 s
    d\tau_{N}= \sqrt{1+2\frac{\Phi_{N}}{c^2}} =0,784 s
    d\tau_{S}= \sqrt{1+2\frac{\Phi_{S}}{c^2}}\approx 1+ \frac{\Phi_{2} }{c^2}   = 0,999990933 s

  874. #886 Karl-Heinz
    29. Dezember 2017

    @Christian, Antwort zu #880

    Hätte der Neutronenstern eine konstante Dichte so, wäre sein Gravitationspotential im Zentrum des Neutronensterns gleich Phi_Inneres = -1,5*G*m/r =2,6 * 10^16 J/kg.

    Damit würde, wenn für einen entfernten Beobachter genau eine Sekunde vergeht, im inneren des Neutronensterns (konstante Dichte natürlich vorausgesetzt) durch die gravitative Zeitdilatation eine Zeit von 0,649786 Sekunden vergehen.

  875. #887 Karl-Heinz
    29. Dezember 2017

    @Franz #843

    Eine Frage die vielleicht schon zu sehr in die Physik geht aber irgendwie aus der Astronomie kommt: Wenn die allgemeine Relativitätstheorie Gravitation als ein Produkt des gekrümmten Raums beschreibt, also quasi als Scheinkraft, warum wird Gravitation dann als Grundkraft definiert die man mit den 3 anderen zu vereinheitlichen versucht und weiters auch in der Quantenmechanik Teilchen (Gravitonen) definiert. Wenn sie nur eine Scheinkraft ist wozu das alles ?

    Problemfall Gravitation
    Warum die Schwerkraft nicht ins Schema passt.
    https://www.scinexx.de/dossier-detail-827-6.html

  876. #888 Karl-Heinz
    29. Dezember 2017

    Morgen muss ich mich leider auf meine Arbeit konzentrieren, damit mich das Stücklistenprogram mich nicht bis 2018 verfolgt also Gutes Neues Jahr und bis 2018 dann!!!!

  877. #889 Alderamin
    30. Dezember 2017

    @Christian #883

    Besteht hier irgendeine Analogie zur Kosmischen Hintergrundstrahlung? Denn auch hier gibt es aus Sicht der Erde eine bewegungsrichtungsabhängige Farbverschiebung, was an und für sich bedeuten würde, dass sich die Kosmische Hintergrundstrahlung in einem bestimmten schalenförmigen Raumbereich rund um die Erde befindet, der theoretisch auch mit einem Raumschiff erreicht werden könnte.

    Kann man so sehen.

    Tatsächlich erfüllt die Kosmische Hintergrundstrahlung aber den gesamten Raum, wobei beispielsweise ein Beobachter in der von der Milchstraße 2,5 Millionen Lichtjahre entfernten Nachbargalaxie “Andromeda” derzeit genau dasselbe sehen könnte wie wir auf der Erde, nämlich, dass die Kosmische Hintergrundstrahlung 13.810.000.000 minus 380.000 Lichtjahre von ihm entfernt ist.

    Nein. Es ist richtig, dass der Raum mit kosmischer Strahlung erfüllt ist. Diejenige, die sich jetzt hier befindet, muss Weltalter – 380000 Jahre alt sein (und demgemäß so lange unterwegs gewesen), weil sie zu dieser Zeit entstand. Sie kann aus jeder Richtung kommen (der Urknall war überall), aber nur aus einer bestimmten Entfernung (eben Lichtlaufzeit = Weltalter – 380000 Jahre oder Rotverschiebung z=1080), d.h. sie kommt aus einer kugelförmigen Schale mit diesem Radius (in Lichtlaufzeit; der tatsächliche Radius dieser Schale ist heute größer, rund 45,5 Milliarden Lichtjahre “Eigendistanz“, und war bei Weltalter 380000 Jahre viel kleiner, etwa 45,5 Milliarden Lichtjahre * 1/(z+1) = 42,1 Millionen Lichtjahre Eigendistanz). In der Andromeda-Galaxie gilt das gleiche, nur eben für deren Ort; da sie 2,5 Millionen Lichtjahre entfernt ist, ist ihre kugelförmige Schale der Hintergrundstrahlung 2,5 Millionen Lichtjahre gegen unsere versetzt (was nicht viel Unterschied macht).

    Das ist wie beim Regenbogen oder beim Horizont /z.B. auf dem Meer): jeder Beobachter sieht seinen eigenen Regenbogen und hat seinen eigenen Horizont.

  878. #890 Peter Paul
    10. Januar 2018

    Wieso sind Meteoriten hart wie Stein?

    Es sind doch eigentlich nur relativ kleine Zusammensetzungen von Staub, müssten also eher ganz “fluffig” sein. Selbst der “große” Komet Tschuri-Gerassimenko war ja recht fluffig. Nach einer Aussage von H.Lesch bestand sein Material zu 75% aus nichts.

  879. #891 Florian Freistetter
    10. Januar 2018

    @Peter Paul: Das ist ein Selektionseffekt. Das was auf der Erde aufgesammelt werden kann, ist zwangsläufig ein Stein. Wenn ein “fluffiges” Ding mit der Erde kollidiert, dann bricht es auseinander. Und nur das nicht-fluffige bleibt übrig und fällt auf den Boden. Außerdem gibts nen Unterschied zwischen Asteroiden und Kometen. Asteroiden sind sehr viel weniger fluffig als Kometen.

  880. #892 Alderamin
    10. Januar 2018

    @Peter Paul

    Wobei sich Philae am fluffigen Kometen Tschuri den Bohrhammer MUPUS in der höchsten Betriebsstufe zerdeppert hat… gut, das war wohl Eis, das bei uns schmilzen würde, aber da oben ganz schön fest war.

    Kometen kommen von weiter draußen und sind insgesamt weniger dicht, weil sie viele Gase und Wasser enthalten, sie sind höchstwahrscheinlich nie mit etwas größerem kollidiert und verlieren ihre Fluffigkeit an der Oberfläche durch kosmische Strahlung (was sie auch so schwarz macht) oder Annäherung an die Sonne (UV, Wärme, Sonnenwind). Innen sind sie aber nicht sehr kompakt und insgesamt leichter als Wasser.

    Viele Asteroiden sind demgegenüber Bruchstücke ehemals größerer Körper, in denen sich teilweise Gesteinsmantel und Eisenkern schon durch Aufschmelzen differenziert hatten. Bruchstücke solcher Objekte sind dann auch ensprechend fest und stabil, und nur die schaffen es, wie Florian sagt, dann bis bei uns auf den Erdboden.

  881. #893 Peter Paul
    10. Januar 2018

    @FF
    Ich hätte vielleicht die Frage anders formulieren sollen, damit klarer wird, worauf es mir ankommt:

    Wieso können kleine Meteoriten hart wie Stein sein?

    Sie bilden sich doch als Verzahnungen von fraktalen Objekten, ähnlich wie Staubfusseln. Wie können dann, unter den so gebildeten, welche sein, die dank Selektion durchkommen, weil sie hart wie Stein sind?

  882. #894 Peter Paul
    10. Januar 2018

    Danke, das mit der kosm. Strahlung wusste ich noch nicht.
    Die Idee mit den Bruchstücken größerer Asteroiden erklärt sicher einen Teil der “Steine”, aber es gab offenbar bereits steinerne Meteoroiden, bevor sich noch was Größeres bilden konnte.

    In dem Artikel : “Meteorite Formation Times and the Age of Jupiter” (https://www.psrd.hawaii.edu/Aug17/formation-times.html ; ich weiß leider immer noch nicht, wie ich einen Link anklickbar angeben kann!) ist von CAI-Chondriten die Rede, die quasi den Nullpunkt der Ausdifferenzierung des Sonnensystems setzen, noch bevor andere Chondriten oder gar Asteroiden entstehen konnten, wenn ich das richtig verstanden habe. Und die liegen auf der Erde rum, also waren sie hart genug um durchzukommen. Wie kann sich so etwas bilden?

  883. #895 Alderamin
    10. Januar 2018

    @Peter Paul

    CAIs sind keine Meteorite, sondern eingeschlossene Calcium-Aluminium-Splitter in einigen von ihnen, und genau diese Einschlüsse sind die ältesten bekannten Feststoffe im Sonnensystem. Die schafften es aber nur intakt auf die Erde, weil sie in Meteoriten (CCs – kohlige Chondrite, oder NCs Meteorite wie Stein oder Eisen-Nickel) eingeschlossen wurden, und das geschah in größeren Körpern. So verstehe ich das verlinkte Papier.

  884. #896 Peter Paul
    11. Januar 2018

    Das hatte ich anders verstanden, aber ich glaube, du hast recht.
    Dann wäre ich jetzt auf folgendem Stand: Feste Meteoroiden gibt es nur, wenn sie als Teile von größeren Körpern entstanden sind, in denen bereits Verfestigungsprozesse stattgefunden haben. Solche Prozesse können in Kometen durch kosmische Strahlung (wie?) oder in Asteroiden durch Gravitationsdruck (wie groß müsste dann so ein Asteroid sein?) bzw. durch radioaktive Heizung (auch da die Frage: wie viel Radioaktivität müsste da zufällig konzentriert sein, und wie wahrscheinlich ist so etwas?) realisiert worden sein. D´accord?

  885. #897 Alderamin
    11. Januar 2018

    @Peter Paul

    Soweit richtig bis auf

    Solche Prozesse können in Kometen durch kosmische Strahlung (wie?)

    da habe ich mich etwas zu weit aus dem Fenster gelehnt und voreilig geschlossen, die Kometenkerne würden schon schwarz und verändert aus der Oort-Wolke kommen. Tatsächlich ist beispielsweise Eris mit einer Albedo von 0,96 extrem hell, die dunkle Oberfläche von Kometen entsteht dann offenbar erst in Sonnennähe (diejenigen, die wir bis jetzt besucht haben, waren alle kurzperiodisch mit vielen Annäherungen an die Sonne). Die harte Oberfläche von Churyumov-Gerassimenko war dann ja auch eine Überraschung für die Wissenschaftler. Nach dieser Quelle entstand sie bei Temperaturen von > 150 K, also nicht schon in der Oort-Wolke. Es ist also die Sonne, die die Kometenoberflächen hart und dunkel macht.

    (wie groß müsste dann so ein Asteroid sein?)

    Weiß nicht, ich hab’ diese Liste mit der Dichte einiger Asteroiden gefunden, und der mit der kleinsten Dichte (617 Patroclus: 0,8 g/cm³) soll 130 km Durchmesser haben (ist ein Jupiter-Trojaner, hat wohl relativ hohen Anteil an flüchtigen Stoffen und wäre bei gleicher Größe dichter, wenn er in Sonnennähe entstanden wäre). Ich denke mal, das trifft ungefähr die Größenordnung. Die leichtesten Meteorite haben eine Dichte von 1,8, könnten also aus dem Inneren solcher Objekte entstammen.

    bzw. durch radioaktive Heizung (auch da die Frage: wie viel Radioaktivität müsste da zufällig konzentriert sein, und wie wahrscheinlich ist so etwas?)

    Auch alleine durch Einschläge erhitzten sich größere Objekte und schmolzen auf, wieviel Anteil der radioaktive Zerfall dabei hat, weiß ich nicht. Mike Brown gibt an, dass alle Objekte oberhalb von 500 km wahrscheinlich rund sein müssten, da gehe ich davon aus, dass sie innen differenziert sein müssen, aber das geht wohl schon bei kleineren Durchmessern los, bei denen die Schwerkaft nicht gereicht hat, das Objekt zur Kugel zu formen (Asteroiden mit sehr viel Eis sind hier natürlich auszuklammern).

    Asteroiden von wenigen km Durchmesser sind wohl eher Trümmer ehemals größerer Brocken.

  886. #898 Peter Paul
    11. Januar 2018

    Vielen und herzlichen Dank, du hast mir sehr geholfen. Ein ganz besonderer Dank für deine tollen Links!

  887. #899 ron
    12. Januar 2018

    Hallo Florian+Community!

    Ist das Thema der interplanetaren Kontamination mit Erdmikroben und ähnlichem gestorben? Ich kann mir nicht vorstellen, dass der Tesla, den Elon Musk mit der Falcon Heavy zum Mars schicken will, total sauber zu bekommen ist?! Oder spielt Kontamination auf dem Mars seit den Rovern keine Rolle mehr?

    Danke!

    ron

  888. #900 ron
    12. Januar 2018

    Hallo Florian+Community!

    Ist das Thema der interplanetaren Kontamination mit Erdmikroben und ähnlichem gestorben? Ich kann mir nicht vorstellen, dass der Tesla, den Elon Musk mit der Falcon Heavy zum Mars schicken will, total sauber zu bekommen ist?! Oder spielt Kontamination auf dem Mars seit den Rovern keine Rolle mehr?

    Danke!

    ron

  889. #901 Florian Freistetter
    12. Januar 2018

    @ron: Also NASA & Co denken schon darüber nach: https://scienceblogs.de/astrodicticum-simplex/2016/09/02/sternengeschichten-folge-197-planetarer-schutz/
    Was Musk macht, weiß ich nicht. Aber der landet ja auch nicht am Mars…

  890. #902 Alderamin
    12. Januar 2018

    @ron

    Musk schießt seinen Tesla nicht zum Mars. Dafür passt zum einen der Startzeitpunkt nicht (da müsste er bis Mai warten, wenn die anderen Marsmissionen dieses Jahr starten) und zum zweiten gibt es auch keine Steuer- und Bremsdüsen, mit denen man das Auto in einen Marsorbit oder überhaupt gezielt zum Mars bringen könnte (es wird zwar immer behauptet, die NASA könne mit ihren Raketen eine Raumsonde so präzise ins Ziel bringen, wie ein Golfball, der quer über die USA ins Loch geschlagen würde, aber das stimmt nicht; ganz ohne Korrekturmanöver [Trajectory Correction Maneuver, TCM] kommt keine Sonde exakt genug ins Ziel).

    Das ganze ist ein PR-Gag. Musk schießt, wenn alles gut geht, sein Auto in eine elliptische solare Umlaufbahn, die zwischen der Bahn der Erde und der Bahn des Mars verläuft, wo schon ein paar Asteroiden mit deutlich mehr Masse herumfliegen. Da Space-X das weiß, wird die Rakete wohl gar nicht ganz bis zur Marsbahn fliegen, schätze ich. Musk erwähnte ja, dass das Auto Milliarden Jahre oben verbleiben soll. Da wird er eine Kollision mit Mars sicherlich vermeiden wollen.

    Siehe auch das Statement von Emily Lakdawalla, Planetary Society, dazu. Die ist nun wirklich keine Freundin von Musks Idee, Weltraumschrott ins All zu schießen. Dummy-Nutzlasten sind aber vollkommen üblich, wenn Raketen das erste Mal getestet werden. Die eine oder andere Apollo-Oberstufe kreist übrigens auch noch um die Sonne.

  891. #903 tomto
    München
    15. Januar 2018

    Ich hätte eine Frage:

    Wie misst man Magnetfelder im Universum?

  892. #904 Alderamin
    15. Januar 2018

    Wenn’s großflächige, schwache Felder sind: die richten metallhaltige Staubteilchen aus. Das dadurch laufende Licht ist schwach linear polarisiert (längliche, parallele Staubteilchen wirken wie ein Gitter).

    Wenn’s um Sterne und starke Magnetfelder geht: Aufspaltung der Spektrallinien, weil das Magnetfeld die Energien von Elektronenspins, die nach dem Feld ausgerichtet sind oder eben nicht auseinander dividiert (verschiedene Niveaus mit verschiedenen Linien -> Zeeman-Effekt).

    Und dann gibt’s noch Synchotronstrahlung bei rotierenden Plasmen (Akkretionsscheiben), die müssen auch Magnetfelder haben.

  893. #905 DSC
    Deisenhausen
    21. Januar 2018

    Wie kommt es, dass die Bahngeschwindigkeiten der Planeten im Sonnensystem nach außen hin abnehmen, wo doch ein Satellit in eine Erdumlaufbahn beschleunigt werden muss, damit er da oben bleibt?

  894. #906 Karl-Heinz
    22. Januar 2018

    @DSC

    Wie kommt es, dass die Bahngeschwindigkeiten der Planeten im Sonnensystem nach außen hin abnehmen, wo doch ein Satellit in eine Erdumlaufbahn beschleunigt werden muss, damit er da oben bleibt?

    Ah, du meinst das Orbit-Paradoxon.
    Ein Satellit, der aus einem Kreisorbit heraus in Flugrichtung beschleunigt, wird weiter von der Erde weggetrieben und gleichzeitig langsamer – trotz Beschleunigung.

  895. #907 Alderamin
    22. Januar 2018

    @DSC

    Er muss nicht ständig beschleunigt werden, damit er oben bleibt, sondern nur ein- oder zweimal (s.u.), wenn er in die Bahn eingeschossen wird.

    Auf einer Kreisbahn (die Planetenbahnen sind in erster Näherung kreisförmig) besteht ein Gleichgewicht zwischen der Schwerkraft durch die Sonne und der Fliehkraft auf der Bahn. Die Schwerkraft nimmt mit zunehmender Entfernung von der Sonne ab, d.h. es braucht nach außen hin immer weniger Fliehkraft, um die Schwerkraft dort zu kompensieren. Die Fliehkraft nimmt mit dem Radius der Bahn bei gleicher Bahngeschwindigkeit linear ab (m*v²/r mit der Masse des Planeten m, der Bahngeschwindigkeit v und dem Bahnradius r), die Schwerkraft nimmt sogar mit dem Quadrat der Entfernung ab (G*M*m/ mit der Gravitationskonstanten G, der Sonnenmasse M, der Masse des Planeten m und dem Bahnradius r): Da beide bei einer Kreisbahn gleich groß sein müssen, muss die Fliehkraft in letzter Konsequenz also auch mit dem Radius quadratisch abnehmen. Da der Radius das alleine nicht hinbekommt, muss also zusätzlich die Bahngeschwindigkeit kleiner werden. Deswegen ist die Bahngeschwindigkeit und erst recht die Umlaufzeit weiter außen kreisender Planeten kleiner.

    Was das von Karl-Heinz beschriebene Orbit-Paradoxon betrifft: Wenn man einen Satelliten beschleunigt, steigt die Fliehkraft und wird größer als die Schwerkraft auf der Kreisbahn, also treibt es den Satelliten nach außen, d.h. die Bahn wird elliptisch und entfernt sich am gegenüber liegenden Ort der Beschleunigung weiter von der Erde als die vorherige Kreisbahn. Dabei steigt er im Schwerefeld empor, wie ein nach oben geworfener Stein, und wird dabei langsamer, bis er wieder umkehrt und zurück fällt. Wegen des dritten Keplerschen Gesetzes nimmt die Umlaufzeit mit der großen Halbachse der Bahn zu (längerer Halbmesser der Bahnellipse), also umkreist er die Erde danach langsamer (Umlaufzeit) als vorher auf der Kreisbahn. Gegenüber einem Objekt auf seiner alten Bahn fällt er folglich zurück.

    Bremst man den Satelliten hingegen ab, ist es umgekehrt: er nähert sich am gegenüberliegenden Punkt der Bahn mehr der Erde, die große Halbachse der Bahn wird kleiner, die Umlaufzeit ebenso, damit überholt er Objekte auf seiner alten Bahn.

    Um die elliptische Bahn wieder kreisförmig zu machen, muss man übrigens am gegenüberliegenden angehobenen bzw. abgesenkten Punkt der Bahn nochmal beschleunigen bzw. abbremsen, um dort die nötige Kreisbahngeschwindigkeit für diese Höhe herzustellen. Die ist nämlich ansonsten niedriger (nach Anhebung) bzw. größer (nach Absenkung), deswegen ist die Bahn elliptisch.

    Auf der elliptischen Bahn ist dann auch die Fliehkraft nicht mehr überall gleich der Schwerkraft (am erdnächsten Punkt ist sie größer, am erdfernsten kleiner als die Schwerkraft, deswegen ändert sich daraufhin die Entfernung zur Erde). Das gilt auch für die Planetenbahnen, die sind nicht exakt kreisförmig, nur angenähert, die Bahnen von Merkur und Mars sind fürs bloße Auge offensichtlich elliptisch; ich hab’s oben auf Kreisbahnen vereinfacht, damit das Prinzip klarer wird.

    Hoffe, das war halbwegs nachvollziehbar…

  896. #908 JosefG
    Aschach
    29. Januar 2018

    Hallo Florian
    Könnte es sein dass sich das Weltall zwischen zwei Membranen schnell kugelig ausgedehnt hat und jetzt nach dem Abkühlen von den Membranen flachgedrückt wird und sich so die exponential Ausdehnung erklären könnte.
    LG JosefG

  897. #909 Karl-Heinz
    29. Januar 2018

    @JosefG

    Könnte es sein dass sich das Weltall zwischen zwei Membranen schnell kugelig ausgedehnt hat und jetzt nach dem Abkühlen von den Membranen flachgedrückt wird und sich so die exponential Ausdehnung erklären könnte.

    Ich dachte bis jetzt immer, dass die Membrane selbst unser beobachtetes Universum darstellen, in dem sich die Prozesse der Elementarteilchenphysik abspielen.

  898. #910 Kurt Gminder
    73o66 Uhingen
    1. Februar 2018

    Ist eine Seitwärts Bewegung von fernsten Massen im All nicht wesentlich schlechter zu Detektieren als eine Hin und Zurück Bewegung? In einem womöglich unendlich massigen All gäbe es ja keine schneller werdende Auseinander Drift, sondern wäre nur eine Zunahme der Geschwindigkeit festzustellen.

  899. #911 ron
    8. Februar 2018

    @Alderamin, @FF (bezugnehmend auf #902)

    Und? Was sagt ihr nun? Eure Einschätzung zur gesamten Musk-Aktion würde mich mal interessieren! Ich fand es grandios und hoffe, dass jetzt mal ein bisschen was voran geht. Auch wenn der wissenschaftliche Aspekt gerade wohl ein wenig in den Hintergrund gerät…

    Viele Grüße!

    ron

  900. #912 Jens
    10. Februar 2018

    Warum hat es seit fast 400 Jahren keine Supernova in der Milchstraße gegeben?

  901. #913 Bullet
    10. Februar 2018

    Hat es doch:
    https://scienceblogs.de/astrodicticum-simplex/2008/05/14/jungste-supernova-in-unserer-galaxie-entdeckt/

    Und selbst wenn du jetzt gleich “okay, dann eben ‘seit 150 Jahren'” antwortest, bleibt die Antwort dieselbe:
    weil isso. Es gibt nun einmal keinen “Supernova-Explosionsterminkalender” für die nächsten 3000 Jahre, in der jede Galaxie ihre Supernova-Ereignise vorher eintragen muß.
    Und außerdem können selbst heute noch Supernovae komplett unbeobachtet bleiben, wenn sie in den falschen Regionen der Milchstraße stattfinden. (Nur die Neutrino-Observatorien könnten da Signale empfangen – aber leider haben die es nicht so mit Richtungsortung.)

  902. #914 Alderamin
    10. Februar 2018

    @Jens

    Es sollte ca. alle 100 Jahre eine Supernova in der Milchstraße auftreten. Aber einerseits treten die nicht mit einer regelmäßigen Periode auf, sondern sie können auch mal gehäuft und dann wieder seltener auftreten (so gab es 1572, 1604 und 1608 drei Stück kurz hintereinander), und außerdem können wir durch den Staub in der Milchstraßenebene nur 10% von ihr überblicken, so dass manche Supernova am gegenüberliegenden Ende der Milchstraße uns entgangen sein dürfte. Erst seit wenigen Jahrzehnten haben wir die Technologie (Radioteleskope, Infrarotteleskope, Neutrinodetektoren), dass uns keine mehr entwischen kann. Du kannst also quasi jeden Tag mit der Entdeckung der nächsten Milchstraßen-Supernova rechnen.

    Im Prinzip hat Bullet das alles schon gesagt, ich wollte es nur noch einmal mit eigenen Worten formulieren.

  903. #915 TomTest
    Hannover
    10. Februar 2018

    Hallo Florian,

    Wie stirbt man im Weltraum ohne Druckanzug?

    Hab dazu schon einiges gelesen über Wikipedia, NASA u.a. aber ich hab keine fundierte Antwort gefunden die auf Versuchen beruht. Kann mir Vorstellen das zu Zeiten des kalten Krieges einiges ausprobiert wurde. Was mir nicht klar ist: Im Vakuum ist ja kaum Materie daher kann der Wärme Verlust doch nur über Wärmeabstrahlung funktionieren und daher sollte doch ein Uberleben fur 10 bis 30 Minuten Moglich sein. Sind dir nachgewiesene Ergebnisse hierzu bekannt? Tier oder Menschenversuche, Moral außen vor? Wie stirbt man im All durch plotzliches erfrieren binnen Sekunden oder platzen die Augen weil das Blut anfängt zu kochen?

    Gruß Bob

    Sry wg. meinem gefährlichen Halbwissen.

  904. #916 Alderamin
    10. Februar 2018

    @TomTest

    Man stirbt wohl am schnellsten daran, dass sich Luftblasen im Blut bilden, die die Adern verstopfen, dasselbe wie bei der Taucherkrankheit. Lungenbläschen können platzen, man muss ausatmen, sonst explodiert das Gas in der Lunge nach außen, man kann so viel Druck nicht aufbringen, das Gas drinnen zu halten. Man kann aber überleben, dass einzelne Körperteile kurzfristig dem Vakuum ausgesetzt sind. Joe Kittinger, der als erster Mensch mit einem Ballon in die Stratosphäre flog, hatte einen undichten Handschuh und die Hand schwoll stark an, aber die Schwellung ging am Erdboden wieder zurück.

    https://en.wikipedia.org/wiki/Space_exposure

  905. #917 Jens
    10. Februar 2018

    Ist es möglich dass in älteren Galaxien weniger Supernovaexplosionen stattfinden als in Jüngeren?

  906. #918 Alderamin
    10. Februar 2018

    @Jens

    Das ist sogar zu erwarten, denn in alten Galaxien entstehen viel weniger Sterne. Die Sterne, die zur Supernova werden können, leben aber nur ein paar zehn Millionen Jahre lang, das ist ein tausendstel des Alters der Milchstraße.

    Letztlich ist es aber weniger eine Frage des absoluten Alters, sondern des verbliebenen Gases in der Galaxie. Wurde dieses schnell verbraucht oder fortgeblasen, dann ist kein Baumaterial mehr für neue Sterne vorhanden und es gibt keine Supernovae mehr; das ist in elliptischen Galaxien der Fall. Ist die Galaxie sparsam mit ihrem Gas umgegangen, kann sie heute noch neue Sterne bilden, wie die Milchstraße. Kollidieren Galaxien, wird das Gas in ihnen komprimiert und es kommt zu einem „Starburst“ mit stark verstärkter Sternentstehung.

    Als die Galaxien entstanden, bildeten sie aber generell viel mehr Sterne als heute und hatten auch mehr Supernovae. Für diese Phase gilt die Regel, je älter, desto weniger Supernovae.

  907. #919 HF(de)
    10. Februar 2018

    Ich hatte die Frage auch gelesen und bin jetzt etwas verwirrt: gibt es alte und junge Galaxien? Ich dachte, die wären alle gleich alt (~13 Mrd.), nur manche verschmolzen mit anderen, andere nicht…

  908. #920 Alderamin
    10. Februar 2018

    @HF(de)

    Im Prinzip entstehen keine Galaxien mehr, die meisten entstanden zu Beginn des Universums und verschmolzen dann nur noch zu größeren Galaxien. Aber früher waren sie eben jünger als heute – so habe ich meine Antwort gemeint.

  909. #921 HF(de)
    10. Februar 2018

    Ah ja, danke! Hab nicht dran gedacht, dass es da draußen etwas komplizierter ist. Wir sehen ja auch junge Exemplare – die, die weit weg sind.

  910. #922 daniel g.
    Kundl
    17. Februar 2018

    Die Theorie der Zeit

    Früher war die Zeit schneller und je weiter man zurückdenkt desto schneller war die Zeit.
    Durch die Ausdehnung der Zeit wurde sie auch langsamer, langsam genug um im Mikrowellenbereich zu strahlen. So entstand die Hintergrundstrahlung, die heute
    schwächer ist durch die Verlangsamung der Zeit.
    Dann dauerte es noch etwas bis die Zeit noch langsamer war, langsam genug um das sichtbare Licht für uns sichtbar zu machen, langsam genug damit sich die Teilchen
    verbinden konnten.
    Heute schauen wir durch das Teleskop in die Vergangenheit und sehen die damals schnellere Zeit das Lichtspektrum ins rote verschieben.
    Die gesamte Masse befindet sich heute in der gleichen Zeitlinie und der Raum fließt mit der Zeit mit. Da Massen sich langsamer durch die Zeit bewegen, bewegt
    sich der leere Raum relativ zu der Masse schneller in der Zeit. Dadurch entsteht eine Blase um die Masse und gibt der Masse noch mehr Schwerkraft.
    Je langsamer die Zeit durch die Masse desto mehr Schwerkraft durch die langsame Zeit. Ist wunderbar im Linseneffekt zu beobachten.

    Durch den Zeitunterschied und da die Zeit fließt beginne die Galaxien zu rotieren und die Begleiter in der Umgebung. Die Mehrheit der Galaxien sollten sich im Uhrzeigersinn drehen solange die Masse schön verteilt ist, da die Zeit auch nur in einer Richtung fließt.

    Die vier Grundkräfte vereinen geht nur wenn die Zeit früher schneller und schneller war, dann bekommt man ein beschleunigtes, heißes und lineares System,
    ohne Strings oder die unglaublichen Phantasie Dimensionen.

    Die Wahrscheinlichkeit, dass das Universum entstanden ist, ist nicht mehr so gigantisch sondern nur eine Frage der Zeit.
    Wann ist die Zeit langsam genug damit es entstehen kann auch ohne Paralleluniversen!

    PS: Wäre die Beobachtung von Hubble nicht falsch interpretiert, hätte auch Einstein nicht die Formel anpassen müssen.
    Warum sollte sich ein Man der mit allem Recht hatte, sich beim statischen Universum irren.
    Der Gesamte Raum bewegt sich in der Raumzeit mit, aber die Materie ist relativ statisch.

    Ich gehe soweit und behaupte damit die Antimaterie erklären zu können. Am Anfang dehnet sich die Zeit in zwei Richtungen auch, in der
    anderen Richtung dehnet sich auch die Zeit aus und wird immer langsamer aber alles ist andersherum. Die Galaxien drehen sich in der anderen Richtung,
    der Atomkern ist negativ geladene usw.

    Man könnte es so vereinfachen um es sich vorstellen zu können. Stellt ein Glas mit Wasser zugedeckt durch ein Blatt verkehrt auf dem Tisch, dann
    hebt das Glas auf das Wasser fließt in aller Richtungen erst ist es schnell und je weiter es sich ausdehne wird es langsamer.
    So verhält sich es sich mit der Zeit, nur das die Zeit sich in Zwei gegengesetzten Richtungen ausdehnte. Aber das liegt an der Materie Antimaterie.

    Die Zeit ist nicht ein Effekt der Raumausdehnung sondern der Raum ist ein Effekt der Zeitausdehnung und jetzt ist es eine Raumzeit.

    Man könnet ein Buch schreiben um alles zu erklären aber die Zeit habe ich nicht!

  911. #923 Karl-Heinz
    17. Februar 2018

    @daniel g. Nr. 922

    Die Theorie der Zeit
    Früher war die Zeit schneller und je weiter man zurückdenkt desto schneller war die Zeit.
    Man könnet ein Buch schreiben um alles zu erklären aber die Zeit habe ich nicht!

    Mit dem Buch schreiben würde ich noch warten. Soll ich dir beweisen, das die Zeit früher gleich schnell war wie heute?

  912. #924 Alderamin
    17. Februar 2018

    @Karl-Heinz

    Soll ich dir beweisen, das die Zeit früher gleich schnell war wie heute?

    Aber bitte nicht in diesem Thread, hier soll explizit nicht diskutiert werden, siehe Artikel oben. Und so eine Diskussion von Privatphysik hat das Potenzial, ein paar hundert Posts lange zu dauern, siehe Karl Gross et. al.

  913. #925 tau ceti
    17. Februar 2018

    Vor allem ärgere ich mich immer wieder, wenn Laien versuchen kosmologische Hypothesen aufzustellen, alleine wegen dem Hintergrund, gar keine Ahnung davon zu haben, wie Wissenschaft funktioniert.

    Florian hatte mal beschrieben, was man anstellen muss und welche Vorbedingungen man braucht, um sich über aktuelle Forschung zu informieren, um überhaupt eine Vorstellung davon zu haben.

    Einfach Rotwein trinken und der Phantasie freien Lauf lassen klappt hier nicht!

  914. #926 Hartl
    München
    24. Februar 2018

    Einige messbare Effekte der Raumkrümmung sind geometrische Verzerrungen. Wenn ich in einem ungekrümmten Raum ein Dreieck aufspanne, messe ich eine Winkelsumme von 180 Grad. Wenn ich anschließend den Raum krümme, z.B. indem ich eine Masse in die Nähe bringe, ist die Winkelsumme nicht mehr 180 Grad.

    Ich habe bei diesem Bild aber einige Verständnisprobleme, für die ich versucht habe, selber Lösungen zu entwickeln. Meine Frage ist: sind meine unten aufgeführten Überlegungen richtig?

    Mein erste Überlegung betrifft die Messung mit Hilfe eines Lasers. Ich sende einen Laserstahl von einer Ecke des Dreiecks aus und lenke ihn über zwei Spiegel (die die anderen beiden Ecken bilden) so ab, dass der Strahl an seinen Ursprungsort zurückgelangt, und z.B. senkrecht zu seiner Ursprungsrichtung einfällt. Dort bringe ich einen Sensor an, auf dem der Laserpunkt abgebildet wird. Wenn ich nun den Raum krümme, verschiebt sich der Auftreffpunkt des Lasers auf dem Sensor, woraus ich eine Winkelsumme ungleich 180 Grad errechne und ich auf die Raumkrümmung schließen kann (richtig…?)

    Meine zweite Überlegung betrifft nun den Fall, dass ich das Dreieck mit Stangen aufbaue, die an den Ecken fest verbunden sind. Dann kann sich der “Auftreffpunkt” aber nicht mehr verschieben. Ich interpretiere das so, dass sich durch die Raumkrümmung in den Stangen eine Biegespannung aufbaut und die Winkelsumme weiterhin 180 Grad bleibt. Die Stangen “wollen” zwar gerade (im Sinne einer Geodäte) bleiben, können aber wegen der festen Verbindung nicht. Erst wenn ich die Stangen an den Ecken löse, um ein wieder spannungsfreies Dreieck zusammenzusetzen, erhalte ich ein Winkelsumme von ungleich 180 Grad.

    Ich habe lange gegrübelt wie ich das Problem formulieren soll, ich hoffe es ist halbwegs klar geworden.

    Sind diese Überlegungen richtig? Vielleicht schlummert ja auch irgendwo ein ganz blamabler Gedankenfehler?

    Falls kein Gedankenfehler, hätte ich allerdings noch einer Reihe von Folgefragen, z.B. in welchen dieser Fällen die Stangen tatsächlich gerade oder gebogen sind oder nur so erscheinen (oder ob das überhaupt eine relevant Frage ist), oder wie es sich mit dem Verhältnis von Kreisradius und -fläche verhält, wenn ich einen Kreis mit Kacheln auslege und die Situation vor und nach der Krümmung betrachte.

  915. #927 Katja Binzenhöfer
    2. März 2018

    Hallo Herr Freistetter, ich bin eine Schülerin der 9ten Klasse und muss derzeit ein Vortrag in meinem Wahlpflichtfach Astronomie über die Sonne vorbereiten.
    Leider kursieren im Internet viele verschiedene Informationen dazu und ich bin mir nicht mehr sicher welchen ich glauben soll. Ich hoffe sie können mich aufklären.
    Zum einen würde ich gerne wissen aus wie viel Prozent Helium und wie viel Prozent Wasserstoff die Sonne momentan besteht?
    Außerdem erschließt sich mir der Lebensweg der Sonne nicht. Zwar weiß ich, dass die Sonne momentan Wasserstoff zu Helimum fusioniert, aber verbrennt sie jetzt auch schon etwas Helium ? Setzt nicht auch noch später das Heliumbrennen ein? Gibt es eigentlich einen Unterschied zwischen Hemiumbrennen und dem Heliumschalenbrennen?
    Des weiteren erschließt sich mir nicht ab wann das Hemiumbrennen einsetzt / eingesetzt hat. Dazu stellt sich mir die Frage, wie lang lebt die Sonne noch ? Oft lese ich von ca. 5 Milliarden Jahren. Ist damit gemeint, dass sich dann erst die Sonne zu einem Roten Riesen aufbläht oder schon zu einem schwarzen Zwerg verglüht?
    Eine letzte Frage zur Sonne hätte ich noch . Undzwar zu Sonnenaktivitäten: Was ist der Unterschied zwischen Flare und Protuberanz?
    Gruß Katja Binzenhöfer

  916. #928 Florian Freistetter
    2. März 2018

    @Katja: Das sind ja ganz schön viele Fragen… Darf ich einen Vorschlag machen? Geh mal in die Bücherei in deinem Ort – da findest du sicher ein Buch über Astronomie und da solltest du alle Antworten auf deine Fragen finden. Die Antworten auf diese Fragen sind auch nicht umstritten; die sind relativ klar und eindeutig. Aber im Internet findet man halt nicht immer nur die richtigen Antworten, sondern auch jede Menge anderes Zeug 😉

    Viele deiner Fragen habe ich auch schon in diversen Podcasts meiner Sternengeschichten-Sendung beantwortet, hier gibt es eine Übersicht: https://sternengeschichten.org/

    Alle Fragen kann ich nämlich hier jetzt auf die Schnelle leider nicht beantworten. Nur ein paar Anmerkungen:

    *) Die Sonne wird nicht zu einem schwarzen Zwerg, sondern einem weißen Zwerg. Das kommt nach der Phase, in der sie ein roter Riese war. Und die dauert im Vergleich zum Rest des Sonnenlebens so kurz, dass man sie bei der Frage “Wie lange lebt die Sonne noch” gut ignorieren kann. Exakte Werte gibt es da sowieso nicht. 5-6 Milliarden Jahre wird es noch dauern, bis die Sonne irgendwann als weißer Zwerg endet.
    *) Heliumbrennen und Heliumschalenbrennen können das gleiche bedeuten. Es kommt halt darauf an. “Heliumbrennen” bezeichnet allgemein den Kernfusionsprozess, bei dem drei Heliumkerne zu Kohlenstoff fusioniert werden. Das passiert im Kern eines Sterns, wenn es dort heiß genug ist. Auch in der Sonne (aber noch nicht jetzt). Wenn es im Kern eines Sterns heiß wird, wird es auch in den Schichten weiter draußen heißer. Irgendwann kann es dann sein, dass es im Kern dann zum Beispiel so heiß ist, dass dort Kohlenstoff fusioniert wird und kein Helium mehr. Das Helium wird dann stattdessen in einer Kugelschale um den Kern herum fusioniert, wo es dann heiß genug dafür geworden ist. Das bezeichnet man dann als “Heliumschalenbrennen”.

  917. #929 Katja Binzenhöfer
    3. März 2018

    Hallo Herr Freistetter, vielen Dank für Ihre Hilfe. In einer Bücherei war ich schon mal und habe nach Büchern geschaut, bin auch fündig geworden und trotzdem in machen Dingen nicht viel schlauer geworden. Aber ich suche und lese weiter.
    Doch möchte ich nochmal auf meine erste Frage zurück kommen. Ich finde leider zu viele verschiedene Zahlen dazu, ob im Internet oder in Büchern.
    Zum einen 90% Wasserstoff und 10% Helium, aber auch 75% Wasserstoff und 25% Helium. Zu meinem Verwundern dann auch noch 35% Wasserstoff und 65% Helium. Was davon halten Sie am plausibelsten?
    Gruß Katja Binzenhöfer

  918. #930 Dampier
    3. März 2018

    Hallo Katja,
    ich hab eben auch mal kurz nachgesehen. Tatsächlich: selbst die deutsche und die englische Wikipedia geben stark unterschiedliche Zahlen bezüglich Wasserstoff & Helium an. Is ja ‘n Ding.

    Du könntest in deinem Vortrag einfach sagen: es gibt da sehr unterschiedliche Angaben, sie schwanken zwischen diesem und jenem Wert (evtl. mit ein paar Quellenangaben). Damit zeigst Du, dass Du schon einiges recherchiert hast, aber das Ergebnis eben nicht eindeutig ist.

  919. #931 Dampier
    3. März 2018

    P.S. bin selbst Laie, kein Astronom.

  920. #932 Florian Freistetter
    3. März 2018

    @Katja, Dampier: Man muss auch aufpassen. Es kommt darauf an, ob man die Zusammensetzung der Sonne in der äußeren Schicht (der Photosphäre, die wir direkt beobachten können) betrachtet oder die Zusammensetzung im Kern. Im Kern ist mehr Helium als außen. Für die Photosphäre gibt zB die NASA Werte von 91/9 Prozent an: https://nssdc.gsfc.nasa.gov/planetary/factsheet/sunfact.html Bei der Gesamtmasse der Sonne dürfte das Verhältnis nähe an 75/25 liegen (zB laut DLR: https://solarsystem.dlr.de/RPIF/sonne0.shtml) Aber es stimmt: Es gibt hier keine allgemeingültigen Werte.

  921. #933 HF(de)
    3. März 2018

    Aber die endlische Wiki gibt für die Photospheric composition (by mass) ca 75/25 an mit dieser Quelle: https://solar-center.stanford.edu/vitalstats.html
    Die deusche Wiki sagt 91/9, wie der Link von FF. Wat denn nu? Bin da ganz auf Katjas Seite, sehr verwirrend. (35/65 ist im Kern, das dürfte klar sein.) [Danke Katja, war mir noch nie aufgefallen.]

  922. #934 HF(de)
    3. März 2018

    Oops, sehe grad, dass die Gesamtmasse bei 75/25 liegen soll. Liegt die englische Wikipedia falsch?

  923. #935 HF(de)
    3. März 2018

    *dass bei der Gesamtmasse das Verhältnis bei

  924. #936 Alderamin
    3. März 2018

    @HF(de)

    Vorsicht, Massenverhältnis (Massenanteil des jeweiligen Gases, englische Wikipedia) ist was anderes als Stoffmengenverhältnis (relative Anzahl der Teilchen, deutsche Wikipedia), denn Helium ist rund 4mal so schwer wie atomarer Wasserstoff! Daneben gibt’s noch Helium-3, das ist nur dreimal so schwer und Deuterium. Kann jetzt aber nicht sagen, wieviel davon in der Photosphäre vorkommt, aber das dürfte den Unterschied der Angaben ausmachen.

  925. #937 HF(de)
    4. März 2018

    Vielen Dank, jetzt versteh ich es! Ist also alles richtig: bei 91:9 betrachtet man die Anzahl der Atome in der äußeren Schicht, da zählen nur Wasserstoff und Helium, der Rest kommt auf parts per million (z.B. Sauerstoff 500ppm). Oder man betrachtet die Masse, dann stimmt ~73:25 und Sauerstoff kommt auf 0,77%.

  926. #938 Jürgen Ziegler
    Österreich
    10. März 2018

    Ich habe da eine Frage zur Entfernungsmessung zwischen Erde und Mond (Laser-Messung mit Spiegeln, die während der Apollo-Missionen installiert wurden): Um einen solchen Spiegel zu sehen bäuchte man ein Telekop, welches über 200m Spiegelfläche hat. Wie kann man etwas genau anvisieren und treffen, das man nicht sehen kann? Stimmt es, dass nur ein sehr geringer Bruchteil der Laserstrahlung wieder zurückkommt, und dieser kaum von anderen Signalen unterscheidbar ist? Was wird getan, um dieses “Problem” zu umgehen. Da ich Physik unterrichte, würde ich mich über eine detailierte physikalische Antwort sehr freuen. (Konkret brauche ich ein paar Gegenargumente gegen diverse Verschwörungstheorien, die behaupten die Mondlandung hätte nie stattgefunden (Van Alle-Gürtel, …) Herzlichen Dank,
    LG

  927. #939 Captain E.
    11. März 2018

    @Jürgen Ziegler

    Ausnahmsweise einmal eine Antwort von mir in diesem Thread, mit Bitte um Entschuldigung an den Hausherren. Auf diesen Seiten wurden alle diese Fragen gestellt und beantwortet:

    https://20600.foren.mysnip.de/list.php?6903
    https://www.mondlandung.pcdl.de/

  928. #940 Alderamin
    11. März 2018

    @Jürgen Ziegler

    Der Wikipedia-Artikel dazu ist ziemlich gut.

    Was das Anpeilen betrifft: Natürlich wusste man ziemlich gut, wohin auf dem Mond man den Laser schießen musste. Man ist ja nicht zufällig irgendwo gelandet, sondern die Apollo-Landestellen waren sorgfältig anhand von Nahaufnahmen vorheriger Mondsonden ausgewählt worden. Und der Laserstrahl ist auf dem Mond weit genug, dass man die Reflektoren nicht exakt treffen muss.

    Siehe auch: https://scienceblogs.com/builtonfacts/2010/12/16/all-right-im-gonna-delay/

  929. #942 Alexander
    12. März 2018

    “Fast Radio Bursts from Extragalactic Light Sails”

    https://arxiv.org/abs/1701.01109

    *kreisch* 🙂

    Wäre das vielleicht einen Artikel wert?

  930. #943 Katja Binzenhöfer
    15. März 2018

    Hallo Herr Freistetter,
    dauert der Vorgang der Umpolung der Sonne 11 Jahre oder ist der Vorgang der Sonne nur alle 11 Jahre und dauert kürzer an.
    Hat diese Umpolung auch etwas mit der “Bewegung” der Sonnenflecken zu tun?
    Gruß Katja Binzenhöfer

  931. #944 Alderamin
    16. März 2018

    @Katja Binzenhöfer

    Der Vorgang dauert ca. 11 Jahre (das ist keine feste Zahl, er kann auch schon einmal mehr oder weniger lange dauern) und ja, die Wanderung der Sonnenflecken gehört zum Zyklus dazu, sie wandern zum Maximum hin in Richtung Sonnenäquator. Sonnenflecken sind nämlich Erscheinungen des Magnetfelds der Sonne, und der Sonnenzyklus ist ein magnetischer Zyklus, der damit zusammenhängt, dass die Sonne aus leitfähigem Plasmagas besteht, dessen Strömung Magnetfelder verursacht, und damit, dass die Sonnenoberfläche am Äquator schneller rotiert als an den Polen. Dadurch verwirbeln die Magnetfeldlinien und wickeln sich auf, bis alles so durchgequirlt ist, dass das Feld zusammenbricht und sich neu aufbaut. Die Flecken hängen mit lokalen Verwirbelungen des Feldes zusammen.

    Siehe auch

    https://scienceblogs.de/astrodicticum-simplex/2013/02/04/das-maximum-der-sonnenaktivitat-sonnensturme-und-grose-katastrophen/?all=1

    https://scienceblogs.de/astrodicticum-simplex/2016/06/13/annie-maunder-der-schmetterling-in-der-sonne-und-die-sache-mit-der-kleinen-eiszeit/

  932. #946 Katja Binzenhöfer
    16. März 2018

    Vielen vielen Dank für Ihre Hilfe . Jetzt wird mir so einiges klarer.
    Gruß Katja Binzenhöfer

  933. #947 Jan
    23. März 2018

    Wenn ein Stern am Ende zu einem weißen Zwerg, Neutronenstern, schwarzem Loch… wird, wie schnell laufen diese Prozesse ab? Habe oft “Explosion”, “Kollaps”, etc gelesen, da denke ich an ein paar Sekunden. Vermute aber sowas dauert viel länger?

  934. #948 Bv
    Deutschland
    24. März 2018

    Hallo zusammen,
    ein Freund von mir und ich hatten letzthins eine Diskussion über eine Frage astronomischer Phänomenologie. Hab dazu nichts im Inet gefunden. Evtl. kann ja hier jemand Licht ins Dunkeln bringen.
    Vorausgesetzt der Bedingung, dass wir Menschen einen anderen Planeten besiedeln, wie würde der dortige Nachthimmel im Vergleich zur dem der Erde aussehen?
    Mein Standpunkt war, dass der Himmel anders aussieht. Aufgrund der unterschiedlichen Positionen, Winkeln und Entfernungen der beiden bewohnten Planeten zu bestimmten Objekten müssten sich doch z.B. Sternenbilder phänomenologisch unterscheiden oder? Klar sehen die Menschen auf beiden Planeten z.B. den gleichen großen Wagen (was in Richtung seiner Argumentation ging), aber hat eben meine Position, Winkel und Entfernung nicht einen Einfluss darauf, wie ich so ein Objekt wahrnehme?
    Schon mal vielen Dank
    Grüße

  935. #949 Krypto
    24. März 2018

    @Bv: Das hängt davon ab, wo der andere Planet sich befindet und welche Sternbilder man sich von dort anschaut: Viele auffällige Sterne liegen in der Nachbarschaft unseres Sonnensystems und je unterschiedlicher die Blickwinkel auf relativ nahe Objekte, desto größer die Abweichungen von vertrauten Konstellationen.

  936. #950 Werner
    Nürnberg
    24. März 2018

    Welche Längenangabe ist denn richtig?

    Pale Blue Dot – Wikipedia
    https://de.wikipedia.org/wiki/Pale_Blue_Dot
    Pale Blue Dot (PBD, englisch für blassblauer Punkt) ist der Name eines Fotos der Erde, aufgenommen von der Raumsonde Voyager 1 aus einer Entfernung von etwa 6 Milliarden Kilometern oder 40,5 AE, der größten Distanz, aus der bis dahin jemals ein Foto der Erde gemacht wurde. Das Bild entstand am 14. Februar …

    Pale Blue Dot – Wikipedia
    https://en.wikipedia.org/wiki/Pale_Blue_Dot
    Diese Seite übersetzen
    Pale Blue Dot is a photograph of planet Earth taken on February 14, 1990, by the Voyager 1 space probe from a record distance of about 6 billion kilometers (3.7 billion miles, 40.5 AU), as part of that day’s Family Portrait series of images of the Solar System.

  937. #951 HF(de)
    24. März 2018

    Beide sind richtig. 6 Mrd km bei uns = 3.7 Bio miles bei den Amis.

  938. #952 Karl-Heinz
    24. März 2018

    @Werner

    Das US-amerikanische billion entspricht der deutschen Milliarde.

  939. #953 Werner
    Nürnberg
    24. März 2018

    Danke.

  940. #954 Christian
    Wien
    24. März 2018

    Zunächst gibt es von mir einen späten Dank bei Karl-Heinz für die Antworten #885 und #886!

    Zum einen hätte ich gerne gewusst, wie man Φ(Sternmittelpunkt) = -1,5*G*m/r herleitet und zum anderen lässt sich aus den Gleichungen der Antworten #885 und #886 errechnen, ab wann die Zeitdilatation in einem Stermittelpunkt dτ = 0 ist:

    m/r = c²/(3*G)

    Angewandt auf die Sonnemasse von rund 2*10^30 kg ergibt das den folgenden Radius: r = 4449 m
    Der Schwarzschildradius für ein Objekt mit der obigen Masse hat den folgenden Wert: rS = 2966 m

    Das heißt: Schon bevor ein Objekt seinen Schwarzschildradius erreicht, würde in seinem Mittelpunkt eine imaginäre Zeitdilatation herrschen. dτ = 0, also für einen außenstehenden Beobachter einen Stillstand der Zeit gäbe es (unter Nichtbeachtung von Rotationseffekten) auf einer konzentrischen Kugelschale im Sterninneren. Wenn ein Stern also zu einem Schwarzen Loch kollabiert, dann stellt diese Kugelschale für jede beliebig große endliche Kollapsdauer den Konvergenzbereich dar. Da es beim Kollaps überdies zu einer Massenverlagerung nach innen kommt, sollte sich diese Kugelschale auf der dτ = 0 gilt aufblähen. Nach unendlicher Zeit würde der Abschluss des Kollapsprozesses so aussehen, dass sich die gesamte Sternenmasse in einer unendlich dünnen Schicht entlang seines Schwarzschildradius verteilt.

    So, jetzt frage ich mich und Euch, wie Einstein und andere auf eine punktförmige Singularität als Endpunkt eines Sternenkollaps gekommen sind?

  941. #955 Florian Freistetter
    24. März 2018

    @Christian: Ich weiß, nach dem jetzt schon fast 1000 Kommentare hier stehen und die Seite eh schon ewig lädt, erübrig sich der (eigentlich ganz, ganz oben stehende Hinweis), hier keine ausführlichen Diskussionen zu führen. Aber ich würde doch bitten, die Diskussion, die über reine, konkrete Fragen hinaus geht, woanders zu führen. Hier wäre zum Beispiel ein sehr passender Artikel zu dieser Frage: https://scienceblogs.de/astrodicticum-simplex/2018/03/19/stephen-hawking-die-singularitaeten-und-der-anfang-des-universums/

  942. #956 Christian
    Wien
    24. März 2018

    Ähm, ich “führe keine Diskussion”, sondern ich rechne, und komme zu einem Ergebnis, wo im Zentrum eines Sterns, der noch kein Schwarzes Loch, sondern zum Beispiel ein hypothetischer Quarkstern ist, eine Zeitdilatation herrschen würde, die ärger wäre als der Stillstand der Zeit, also beispielsweise eine negative Zeit oder eine imaginäre Zeit. Also mutmaße ich, dass die Singularität kein Punkt, sondern ein zu einer Kugelschale aufgeblähter Punkt ist und wundere mich, dass nur ich das bemerke und hätte daher gerne gewusst, wie Relativistiker es rechnerisch schaffen, die Masse eines Sterns in keinem aufgeblähten Punkt, sondern tatsächlich in einem Punkt unterzubringen? Florian, ich habe deinen Link überflogen und darin geht es ja eher darum, dass relativistische Physik und Quantenphysik nicht einfach auf einen Nenner zu bringen sind, was aber eine Voraussetzung für die Voraussage des Verhaltens von superdichten Materieansammlungen wäre. Die Sache mit der Singularität ist also ohnehin nur die halbe Wahrheit, aber was mich interessiert ist, ob ich in meiner Rechnung einen Fehler gemacht habe oder nicht und dazu müsste ich wissen, wie Realtivistiker auf den singulären Punkt gekommen sind anstatt auf die singuläre Kugelschale?

  943. #957 Florian Freistetter
    24. März 2018

    @Christian: Ich will dir einfach nur sagen, dass das, was du hier mitteilen/besprechen/etc willst, HIER nicht hinpasst. Aber nebenan passt es hin.

    Ich hatte ja mal den Plan, das hier Leute ihre Fragen posten und ich aus den Fragen dann welche aussuche und in eigenen Artikeln ausführlich beantworte. Das ist aber jetzt eh schon lange hinfällig, weil der Kommentarbereich so enorm zerfasert ist; über alles mögliche diskutiert wird und die Seite so unübersichtlich geworden ist, dass das ursprüngliche Konzept nicht mehr funktioniert. Also macht meinetwegen weiter… (Eventuell lösch ich irgendwann mal alle Kommentare hier und starte das Ding neu.)

  944. #958 HF(de)
    24. März 2018

    Warum so drastisch und alles löschen? Warum nicht wie damals beim Verschwoerungsgeplauder einfach ne neue Seite aufmachen und diese schließen? (Und den Text hier übernehmen oder drauf verweisen)
    https://scienceblogs.de/astrodicticum-simplex/2014/08/10/verschwoerungsgeplauder-xii-zelebration-der-staatsmacht/

  945. #959 PDP10
    24. März 2018

    Eventuell lösch ich irgendwann mal alle Kommentare hier und starte das Ding neu.

    Vielleicht gar keine so schlechte Idee.

    “Wenn nicht tut, tut Reboot gut!”

    (Alte Sysadmin Weisheit).

    Vorschlag zur Güte:

    Mach doch zwei:

    Einen Artikel für Fragen in dem du gnadenlos alles löscht was keine Frage ist (ja, auch die Beiträge vom Ursa Major Explicatoris ;-). ) und einen für Diskussionen zu Themen aus der Astronomie die grade nicht in deinen Artikeln vorkommen.

    Gemeint sind spezifisch astronomische (oder verwandte) Themen, die für den Plauder-Thread zu speziell sind. Also ein astronomischer Plauder-Thread sozusagen …

  946. #960 HF(de)
    24. März 2018

    Zusätzlich ein Astronomie-Plauder wie beim Verschwörungs-Plauder. Gute Idee.

  947. #961 PDP10
    25. März 2018

    Da tut vielleicht noch eine Übersetzung Not:

    Ich hoffe jedenfalls, das “Ursa Major Explicatoris” wenigstens sowas ähnliches wie “Großer Erklärbär” heißt ;-).

  948. #962 Alderamin
    25. März 2018

    @PDP10

    Ähem, Ursa wäre eine Bärin (und Ursula eine kleine Bärin).

    Bin ja eh schon umgezogen. Fragen kann man bei mir auch stellen unter Schreib doch mal was zu…. Für Fragen, die ich hier als Kommentare beantworten würde, würde ich da einen Artikel-Quickie aufmachen. Kam aber bisher noch keine.

  949. #963 PDP10
    25. März 2018

    @Alderamin:

    Ähem, Ursa wäre eine Bärin (und Ursula eine kleine Bärin).

    Ach echt? :-). Und weiß ich, ob du nicht ein Mädchen bist?!?

    ;-).

    Für Fragen, die ich hier als Kommentare beantworten würde,

    Wofür der Thread hier nie gedacht war du große Bärin.

    Die Seite bei dir nebenan habe ich übrigens schon bemerkt. Scheint ein interessantes Projekt zu werden!

  950. #964 HF(de)
    25. März 2018

    ok, dann “Urs Major Explicatoris”. “Großer Erklärbär” find ich gut 🙂 Und bei “Schreib doch mal” brauch ich ein Abo.

  951. #965 Orci
    29. März 2018

    Sicher haben einige Leser diese Meldung über die Entdeckung einer Galaxy mit nur wenig bzw. gar nicht vorhandener Dunkler Materie schon gesehen. Mir kommen da spontan zwei Fragen in den Sinn:

    1. Stellt das unsere Vorstellung, wie Galaxien entstehen tatsächlich so fundamental auf den Kopf?

    2. Bedeutet die Entdeckung nicht ein schwer zu erklärendes Problem für alternative Gravitationstheorien, wenn die Sterne sich nicht schneller, sondern langsamer bewegen, wie im Fall von NGC 1052-DF2?

    Danke im Voraus!

  952. #966 Dampier
    29. März 2018

    @Orci, danke. Schließe mich der Frage an.

  953. #967 Florian Freistetter
    29. März 2018

    @Orci: Wie üblich gilt: Es nicht alles dramatisch, wie es präsentiert wird.

    Zu 2): Die Sterne sind ja nicht langsamer als erwartet, sondern sie bewegen sich exakt so, wie sie es tun sollten, wenn die sichtbare Materie alles ist, was die Galaxie ausmacht. Was ein Hinweis darauf ist, dass da eben im Gegensatz zu den anderen Galaxien (wo das nicht so ist), hier keine dunkle Materie ist. Und mMn ist das für MONDartige Theorien tatsächlich ein Problem – aber wie solche Theorien eben sind, lassen die sich sicherlich leicht entsprechend anpassen.

    Zu 1): Na ja. Es ist halt etwas, was wir bis jetzt in der Form noch nicht beobachtet haben. “Auf den Kopf” stellen, würd ich nicht sagen. Aber es wirft halt Fragen auf, die man beantworten muss. Möglichkeiten, wie eine Galaxie dunkle Materie verlieren kann bzw. wie Galaxien entstehen können, ohne das dunkle Materie beteiligt ist, gibt es sicherlich (ein paar werden auf der NASA-Seite auch erwähnt). Bis jetzt war uns halt nicht bewusst, dass solche Mechanismen nötig sein können. Aber jetzt wird man sich das anschauen. Auf jeden Fall ist es nicht so, dass diese Galaxie irgendwelchen wichtigen Gesetzen fundamental entgegensteht…

  954. #968 Hannes Kranepuhl
    6. April 2018

    Im Zusammenhang mit der Expansion des Universums wird häufig ausgesagt, “dass sich Galaxien umso schneller entfernen, je weiter sie vom Beobachter entfernt sind.”
    Ich habe damit, dass diese Aussage in der Gegenwartsform formuliert ist, ein Problem.

    Erläuterung: Man misst bei einer 13 Milliarden Lichtjahre entfernten Galaxis eine größere Rotverschiebung des emittierten Lichtes als bei einer Galaxis, die nur 5 Millionen Lichtjahre entfernt ist.

    Mir stehen also Informationen zur Verfügung, die 13 Milliarden bzw. 5 Millionen Jahre alt sind.

    Wie komme ich da zur Aussage, ” “dass sich Galaxien umso schneller entfernen, je weiter sie vom Beobachter entfernt SIND.” ?

    Welche Rotverschiebung die 5 Millionen und 13 Milliarden Lichtjahre entfernten Galaxien HEUTE haben weiß kein Mensch.

    Man kann doch lediglich sagen, dass weiter entfernte Galaxien früher schneller “unterwegs´” WAREN. Mehr nicht.

  955. #969 Hannes Kranepuhl
    6. April 2018

    Es gibt einen interessanten Ansatz zur Quantengravitation, der ohne dunklen Materie auskommt:

    https://www.spektrum.de/news/der-anfang-vom-ende-dunkler-materie/1437827

    Wenn sich diese Theorie irgendwann bewahrheitete, hätte die jahrzehntelange erfolglose Suche nach der dunklen Materie ein Ende.

  956. #970 Rai
    Schleswig Holsten
    14. April 2018

    Hallo,
    in der Wikipedia habe ich über Neutronensterne gelesen: “Der Radius eines Neutronensterns ist doppelt so groß wie sein Schwarzschild-Radius.”
    Heißt das, dass im INNEREN eines Neutronenstern – also innerhalb des Schwarzschild Radius’ – (ähnliche) Bedingungen herrschen, wie in einem Schwarzen Loch? Falls das zutrifft,
    1. dann dürfte es eigentlich keinen Energieaustausch vom Zentrum in die Peripherie der Neutronensterns geben?!
    2. bestünde dann nicht die Gefahr, dass Neutronenstern doch (langsam) zu einem Schwarzen Loch kollabiert?!
    Oder wie ist diese Aussage zu verstehen?

  957. #971 Florian Freistetter
    14. April 2018

    @Rai: Der Schwarzschildradius ist die Größe, auf die man die Masse des Neutronensterns komprimieren müsste, damit ein schwarzes Loch daraus wird. Da der Radius des Neustronensterns aber größer ist, ist er kein schwarzes Loch geworden. Und auch in seinem Inneren ist keins – weil da eben nicht genug Masse vorhanden ist. Ein schwarzes Loch kriegst du nur, wenn du eine bestimmte Masse ausreichend stark komprimierst und das ist nirgendwo in nem Neutronenstern der Fall.

  958. #972 Dini
    Österreich
    24. April 2018

    Hi,
    Ich bin erst ein Teenager und hab versucht irgendwo eine Antwort auf meine Frage zu finden, aber ich weiß nicht, wen ich fragen soll, also würde ich mich sehr freuen, wenn sie meine Frage beantworten würden:
    Ich verstehe schwarze Löcher, Raum-Zeit-Krümmung und Einsteins Relativitätstheorie grob, aber ich bin auch auf weiße löcher gestoßen. Ich weiß, dass sie das Gegenteil von einem schwarzen Loch sind und wahrscheinlich durch eine Einstein-Rosen Brücke mit der Singularität des schwarzen Loches verbunden ist und somit seine Materie ausstößt und keine Aufnehmen kann. Hat dann ein weißes Loch, in dieser Abwandlung der Einsteinschen Relativitätstheorie, eine negative Masse?
    Danke für die Zeit.
    Schöne Grüße

  959. #973 Florian Freistetter
    24. April 2018

    @Dini: Was weiße Löcher sind bzw sein können, habe ich in diesem Artikel erklärt: https://scienceblogs.de/astrodicticum-simplex/2016/04/04/was-sind-weisse-loecher/

  960. #974 wohe
    Berlin
    25. April 2018

    Hallo Florian,
    im Forum der astronews.com wurde zum Michelson-Morley-Experiment gefragt, ob es einen Unterschied macht, ob man die “Lichtgeschwindigkeit aufwärts oder abwärts, also entgegen bzw. mit dem Gravitationsfeld, mißt”.
    Einer der Beiträge bestand aus der These, daß Photonen ja von der Gravitation beeinflußt werden, demnach also vom Gravitationszentrum weg verlangsamt werden müßten, in Richtung zum Gravitationszentrum jedoch von der Lichtgeschwindigkeitstempogrenze an einer höheren Geschwindigkeit gehindert werden.
    Demnach müßte, wenn das MM-Experiment nach “oben” statt zur Seite stattfände, dieser Teil doch langsamer sein.
    Dies erscheint mir plausibel – wurde jedoch in dem Forum nicht weiter beachtet und ich finde auch nirgens sonst etwas hierzu.
    Kannst Du mir bitte mitteilen, ob das so ist?
    Vielen Dank
    Wohe

  961. #975 klaus schmidt
    Deutschland
    26. April 2018

    Ich verstehe dieses zurückreisen in der Zeit nicht, wenn man weit ins Weltall hinausblickt, der Zeitpfeil ist doch nicht nur für uns relevant, auch der weit im Weltall erblickte Ort hat doch auch eine Vergangenheit, also blicken wir doch nie in Richtung Urknall, sondern eher auf einen parallelen Zeitpfeil, oder ist das falsch gedacht?

  962. #976 Karl-Heinz
    26. April 2018

    @wohe

    Antwort zu Frage #974
    Lokale Lichtgeschwindigkeit
    In Folge der Zeitdilatation und der Lorentzkontraktion ist die Lichtgeschwindigkeit im Gravitationsfeld geringer, wobei die Lorentzkontraktion nur in radialer Richtung wirksam ist:
    {\displaystyle {\vec {c}}_{rad}=c\cdot \left(1-{\frac {r_{S}}{r}}\right)}
    {\displaystyle {\vec {c}}_{tang}=c\cdot {\sqrt {1-{\frac {r_{S}}{r}}}}}
    {\displaystyle r_{S}={\frac {2\cdot G\cdot M}{c^{2}}}}

    r_{S} … Schwarzschildradius
    \vec {c}_{rad} wäre dann in deinem Fall die vertikale Richtung
    \vec {c}_{tang} … die horizontale Richtung

    Auf der Erdoberfläche würdest du dann folgende lokale Geschwindigkeit erhalten, wenn du die Gravitation berücksichtigst.

    \vec {c}_{rad} =0,999999998611235 \cdot c
    \vec {c}_{tang} =0,999999999305618 \cdot c

    Also in der horizontalen Richtung hast du durch die Gravitation eine etwas größere Geschwindigkeit als in der vertikalen Richtung. Sie ist aber in beiden Fällen kleiner als die Lichtgeschwindigkeit im flachen Raum.

  963. #977 Krypto
    27. April 2018

    @klaus: Mein reist auch nicht zurück in der Zeit.
    Das Licht von weit entfernten Objekten war nur lange unterwegs: Wir sehen also die Objekte, wie sie wesentlich jünger waren als solche in unserer Umgebung. Wenn Du Dir ein altes Foto anschaust, reist Du ja auch nicht zurück.

  964. #978 Ingolf Malik
    Berlin
    27. April 2018

    Allenthalben hört man, dass sich das Weltall ausdehnen würde und das immer schneller. Als Begründung wird dazu die Rotverschiebung angegeben. – Wenn ich als Laie so darüber nachdenke, frage ich mich warum nicht einfach das Licht im Lauf der Milliarden Jahre etwas seine Frequenz verlängsamt, sich also quasi abschwächt. Die Rotverschiebung würde dann nicht eine Folge der Ausdehnung, sondern des “natürlichen” Alterns des Lichts sein. – Wo liegt mein Denkfehler? Hat das schon mal jemand in Erwägung gezogen? – Besten Dank für die Anworten.

  965. #979 Alderamin
    27. April 2018

    @Ingolf Malik

    Hat das schon mal jemand in Erwägung gezogen?

    Selbstverständlich. Die damaligen Gegner der Urknalltheorie redeten ständig vom “ermüdenden Licht”. Passt aber absolut nicht zu den Beobachtungen.

    https://de.wikipedia.org/wiki/Lichterm%C3%BCdung
    https://www.astro.ucla.edu/~wright/tiredlit.htm

  966. #981 Captain E.
    27. April 2018

    @Ingolf Malik:

    Tja, und laut Einstein kann das Licht schon deshalb nicht “ermüden”, weil überhaupt keine Zeit vergeht. So ein Photon entsteht, fliegt mit Lichtgeschwindigkeit davon und wird vernichtet, z.B. weil es von den Atomen eines Auges oder eines Teleskops eingefangen wird. Aber egal, ob die zurückgelegte Strecke 1 Nanometer oder 13 Milliarde Lichtjahre betragen hat – für das lichtschnelle Photon ist keinerlei Zeit vergangen.

  967. #982 Karl-Heinz
    27. April 2018

    @wohe
    In #976 habe vergessen dazusagen, dass die Beziehung für die variable Lichtgeschwindigkeit nur für einen weit entfernten Beobachter gilt. Wenn du jetzt am Ort selbst als Beobachter misst, wird es für kurze Strecken also doch c sein unabhängig davon, ob du horizontal oder vertikal misst.

  968. #983 Ingolf Malik
    Berlin
    27. April 2018

    Besten Dank für eure Bemühungen, einen Laien zu erhellen. – Das Photon stammt aus der Teilchenbetrachtung, wenn ich das richtig überblicke. Ohne Zeit würde es allerdings keine Geschwindigkeit haben oder anders: auch keinen Weg zurücklegen. – Aber irgendwas muss doch am Photon dran sein, damit Licht mal ein bisschen röter erscheint und mal nicht. – Wie auch immer, an diesem Punkt steige ich aus. Wenn’s irgendwie nützlich ist, freue ich mich darüber, aber für mich Amöbe ist das zu theoretisch.

  969. #984 Alderamin
    27. April 2018

    @Ingolf Malik

    Ohne Zeit würde es allerdings keine Geschwindigkeit haben oder anders: auch keinen Weg zurücklegen

    Das ist Eigenzeit des Photons! Aus Sicht eines Photons erscheint ja auch jede Strecke unendlich kurz, Längenverkürzung der speziellen Relativitätstheorie. Ähnliches Argument gilt für die Neutrinos, sie ändern ihren Typ unterwegs, also vergeht für sie Zeit, also sind sie langsamer als das Licht, also haben sie eine Masse (die noch niemand kennt).

    Aber irgendwas muss doch am Photon dran sein, damit Licht mal ein bisschen röter erscheint und mal nicht. – Wie auch immer, an diesem Punkt steige ich aus.

    Das ist eigentlich total einfach. Licht ist eine Welle. Zeichne eine Wellenlinie auf einen Ballon und blas ihn auf. Dann wird die Welle länger. Genau das passiert dem Licht im expandierenden Universum. Das Universum (der Raum) ist der Ballon.

  970. #985 Captain E.
    27. April 2018

    @Ingolf Malik:

    Das Photon hat halt die Eigenschaften einer Welle und eines Teilchens. Du vergisst aber, dass Photonen keine Ruhemasse haben und sich nicht gar nicht anderes bewegen können als mit Ruhemasse. Die Zeitlosigkeit besteht natürlich auch nur aus der Perspektive eines Photons. Für den außenstehenden Beobachter kann es Milliarden von Jahren alt sein. Für das Photon selber vergeht aber gerade eben keine Zeit, und somit hat es auch nicht die Zeit zum “müde werden”.

  971. #986 PDP10
    27. April 2018

    dass Photonen keine Ruhemasse haben und sich nicht gar nicht anderes bewegen können als mit Ruhemasse.

    “als mit Lichtgeschwindigkeit

  972. #987 Niels
    28. April 2018

    Vorsicht.

    Photonen kann man schlicht kein Inertialsystem zuordnen. Es gibt also keine “Sicht eines Photons.”
    Damit kann man allgemein für masselose Teilchen grundsätzlich auch nicht sinnvoll eine Eigenzeit definieren.

    Das ist eine ganz andere Aussage als die “Eigenzeit ist Null” oder “für das Teilchen vergeht keine Zeit”.

  973. #988 Niels
    28. April 2018

    Nachtrag:
    Ich versuch mal eine Erklärung ganz ohne Mathematik:

    Die Eigenzeit ist die Zeit, die im Ruhesystem des Beobachters vergeht.

    Für die spezielle Relativitätstheorie braucht man nur zwei Postulate.
    1. Das Relativitätsprinzip
    Physikalische Gesetze sind invariant gegen den Wechsel des Inertialsystems.
    2. Konstanz der Lichtgeschwindigkeit
    Licht breitet sich im Vakuum in allen Inertialsystemen mit der gleichen Geschwindigkeit c aus.

    Mit 2. kann es offensichtlich kein Inertialsysteme geben, in dem Licht ruht. Damit dann auch keine Eigenzeit.

    (Allgemeiner und auch für die ART gültig kann man schlicht nur zeitartigen Weltlinien (auf denen sich alle Materie durch die Raumzeit bewegt) eine Eigenzeit zuordnen.
    Für lichtartige Pfade würde man etwa durch Null teilen, wenn man den Eigenzeitparameter verwenden will.)

  974. #989 Karl-Heinz
    28. April 2018

    @Niels

    Das ist eine ganz andere Aussage als die “Eigenzeit ist Null” oder “für das Teilchen vergeht keine Zeit”.

    Na ja, wenn die Eigenzeit 0 ist, dann ist die Eigenzeitzunahme auch 0. Das heißt für das Lichtteilchen vergeht keine Zeit. Aber du hast vollkommen Recht damit, dass man das Lichtteilchen nicht ruhigstellen kann. Scheint irgendwie hyperaktiv zu sein. Das hätte ich fast übersehen. (Smiley)

  975. #990 Captain E.
    28. April 2018

    @PDP10:

    “als mit Lichtgeschwindigkeit“

    Gut mitgedacht! Ich würde den Satz sofort korrigieren, wenn ich es könnte.

  976. #991 Niels
    28. April 2018

    @Karl-Heinz
    Die Eigenzeit ist aber eben nicht 0 und die Eigenzeitzunahme ist ebenfalls nicht 0.

    Vielmehr ist Eigenzeit ist einfach ein bedeutungsloser Begriff für masselose Teilchen.
    Die Frage, wie viel Eigenzeit für ein Photon vergeht ist genau so sinnvoll wie die Frage, wie stark ein Photon riecht.

  977. #992 PDP10
    28. April 2018

    @Niels:

    Die Frage, wie viel Eigenzeit für ein Photon vergeht ist genau so sinnvoll wie die Frage, wie stark ein Photon riecht.

    Du alter Prosaiker!

    In meiner Vorstellung sind Photonen gelb-rot und riechen nach Marakuja und Mandarine.

    Ausserdem trägt jedes eine kleine Armbanduhr auf der es immer Samstag-Nachmittag um kurz vor Vier ist.

    Dä!

    ;-).

  978. #993 Karl-Heinz
    28. April 2018

    @Niels

    Die Eigenzeit ist aber eben nicht 0 und die Eigenzeitzunahme ist ebenfalls nicht 0.

    Verstehe ich nicht.
    Die Eigenzeit eines Beobachters entspricht der Länge der Weltlinie dieses Beobachters, sie hängt mit dem Linienelement der Metrik über dτ=(√(-(ds)^2)/c
    zusammen (Riemannsche Mannigfaltigkeit).
    Wenn ds lichtartig ist dann ist halt ds gleich 0 und damit auch dτ gleich 0.
    Das Photon als Beobachter, das keine Zeit verfügt etwas zu beobachten. Das finde ich cool. Freue mich, dass du wieder mitkommentierst. Ich hoffe, das es gesundheitlich wieder so einigermaßen geht.

  979. #994 Niels
    29. April 2018

    @Karl-Heinz

    Überleg mal was sich etwa für die Vierergeschwindigkeit u = dx/dτ eines Photons ergibt, wenn dτ = 0 tatsächlich richtig wäre.

    Die Eigenzeit eines Beobachters entspricht der Länge der Weltlinie dieses Beobachters

    Richtig.

    Einen Beobachter kann es aber nur für zeitartige Weltlinien geben, nicht für raumartige oder lichtartige Kurven.

    Deswegen ist das Konzept Eigenzeit auch nur für zeitartige Kurven definiert.
    Man kann diese Formel also nicht einfach nur deswegen ohne nachzudenken anwenden, weil man weiß, welchen Wert das Linienelement ds annimmt.
    (Bei raumartigen Kurven ist auch sofort einsichtig, dass das Unsinn ist, da hier dτ eine imaginäre Zahl als Ergebnis liefert.)

    Ich zitiere einfach mal aus der englischen Wiki für “proper time” (Eigenzeit):
    https://en.wikipedia.org/wiki/Proper_time

    Proper time can only be defined for timelike paths through spacetime which allow for the construction of an accompanying set of physical rulers and clocks.

    The same formalism for spacelike paths leads to a measurement of proper distance rather than proper time.

    For lightlike paths, there exists no concept of proper time and it is undefined as the spacetime interval is identically zero.
    Instead an arbitrary and physically irrelevant affine parameter unrelated to time must be introduced.

  980. #995 Karl-Heinz
    29. April 2018

    @ Niels
    Danke für die Info. Klingt plausibel.
    Falls ich mal Fragen habe, hoffe ich, dass ich mich an dich wenden kann.

  981. #996 Alderamin
    29. April 2018

    @Niels

    Das ist die Tyrannei der Präzision. Ich weiß schon, dass es physikalisch nicht korrekt ist, dem Photon eine Eigenzeit zuzuschreiben. Genauso, wie es mathematisch nicht korrekt ist, vom Wert von sin (x)/x an der Stelle 0 zu sprechen (wo doch beidseitig der Grenzwert gegen 1 geht und derjenige der 1. Ableitung gegen 0).

    Allerdings kann ein Laie mit “zeitartig”, “Vierergeschwindigkeit” oder “Inertialsystem” nicht viel anfangen. Es ist ihm aber leicht nachvollziehbar, dass die Zeitdilatation für v->c gegen unendlich geht, die Längenkontraktion gegen 0 und im Grenzfall also jede beliebig lange Strecke in 0 Zeit wahrgenommen würde, wenn man sich so schnell bewegen könnte (auch wenn man es nicht kann). Damit kann man dann gedanklich nachvollziehen, warum ein Photon sich unterwegs nicht verändern kann, so wie ein Neutrino das tut.

    Wenn ein Physikstudent auf physics.stackexchange nachfragen würde, würde man ihm sicherlich eine andere Antwort geben, aber um jemand für Physik zu begeistern, kann man auch mal Fünfe gerade sein lassen, finde ich.

  982. #997 Niels
    29. April 2018

    @Alderamin
    Da bringst du aber leider einiges durcheinander.

    Zur Veranschaulichung:
    Wenn du als äußerer Beobachter ein Raumschiff betrachtest, dass fast mit Lichtgeschwindigkeit an dir vorbeifliegt, laufen alle Vorgänge im Raumschiff verlangsamt ab.
    Nach dem 1. Postulat der SRT, dem Relativitätsprinzip, sind alle Beobachter, die sich mit konstanter Geschwindigkeit gegeneinander bewegen, völlig gleichberechtigt.
    Sowohl du als auch ein Beobachter im Raumschiff nehmen also eine wechselseitige Verlangsamung wahr.

    Jetzt übertragen wir das einfach ein Photon und einen Niels.
    Aufgrund des Relativitätsprinzip bewegt sich Niels aus Sicht des Photons mit c.
    Da die Zeitdilatation für v->c gegen unendlich geht, die Längenkontraktion gegen 0 und im Grenzfall also jede beliebig lange Strecke in 0 Zeit wahrgenommen würde, kann man leicht nachvollziehen, dass sich ein Niels niemals verändern kann.
    Man kann ihm also die Eigenzeit 0 zuschreiben.
    Tatsächlich?

    um jemand für Physik zu begeistern, kann man auch mal Fünfe gerade sein lassen, finde ich.

    Man benötigt nur zwei Postulate, aus denen sich die ganze spezielle Relativitätstheorie ableiten lässt.
    Für deine Antwort musst du alle beiden verletzten.

    Kann das dann wirklich noch eine sinnvolle Antwort auf eine Frage zum Thema der speziellen Relativitätstheorie sein?
    Ich finde das schon ein bisschen mehr als nur “unpräzise”.

  983. #998 UMa
    30. April 2018

    Aus Sicht eines sich mit c bewegenden Photons ist nicht sinnvoll, da es kein solches Bezugssystem gibt.
    Interessanter ist, wenn das Licht langsamer als c ist. Z.B. in Luft. Dann gibt es ein Bezugssystem und die Eigenzeit ist größer 0.
    Was passiert dann?
    Eigentlich ist das immer der Fall, da es kein perfektes Vakuum gibt.

  984. #999 Niels
    30. April 2018

    @Karl-Heinz

    Kein Problem.
    Das Ganze liest man leider oft genug falsch.

    @UMa

    Photonen bewegen sich immer mit c, auch in Materie.

    Im Photonenbild kann man sich die langsamere Ausbreitungsgeschwindigkeit vereinfacht so veranschaulichen, dass ein Photon beim Durchgang durch ein Medium immer mal wieder von einem Atom absorbiert und kurze Zeit später reemittiert wird.

  985. #1000 PDP10
    30. April 2018

    @Niels:

    Im Photonenbild kann man sich die langsamere Ausbreitungsgeschwindigkeit vereinfacht so veranschaulichen, dass ein Photon beim Durchgang durch ein Medium immer mal wieder von einem Atom absorbiert und kurze Zeit später reemittiert wird.

    Argh! Schmerzen!

    Hast du nicht weiter oben implizit dafür plädiert “Laien” keinen Unsinn zu erzählen auch wenn er einfacher zu verstehen ist? ;-).

    https://scilogs.spektrum.de/quantenwelt/nicht-so-schnell-licht/

  986. #1001 Niels
    30. April 2018

    @PDP10

    Stimmt, die Erklärung ist falsch und es ist besonders scheinheilig, wenn ich noch unmittelbar zuvor Alderamin für etwas Ähnliches kritisiere habe.
    Mea culpa, ich bemühe mich zu bessern.
    (Sorry, Alderamin.)

    Die von dir verlinkte Erklärung ist allerdings die übliche Erklärung im Wellenbild.
    Für die Frage, was hier mit den Photonen passiert, hilft sie meiner Meinung nach überhaupt nicht weiter.

    Für das Teilchenbild kenne ich allerdings eben wirklich nur die Absorbtions-Reemissions-Erklärung.
    Da habe ich noch nirgends irgendwo etwas besseres gelesen.
    Ich selbst kann auch nicht besseres liefern als “da passieren komplizierte Wechselwirkungen und Streuvorgänge zwischen den Atomen/Elektronen/Quasiteilchen des Mediums und den Photonen”.

  987. #1002 Niels
    30. April 2018

    @PDP10

    Ich hab nach kurzem Googeln das hier gefunden:

    https://physics.stackexchange.com/questions/153904/how-does-light-speed-up-after-coming-out-of-a-glass-slab

    You can think, for a rough mind picture, of light propagating through a medium as somewhat like a game of Chinese Whispers.
    A photon is absorbed by one of the dielectric molecules, so, for a fantastically fleeting moment, it is gone.
    The absorbing molecule lingers for of the order of 10−15s in its excited state, then emits a new photon. The new photon travels a short distance before being absorbed and re-emitted again, and so the cycle repeats.

    The interaction is so short that the absorber interacts with nothing else, so the emitted photon must bear the same momentum as the incident one.
    Also take heed that we’re NOT a full absorption in the sense of forcing a transition between bound states of the atom (which gives the sharp spectral notches typical of the phenomenon), which is what David Richerby is talking about. It is a transition between virtual states – the kind of thing that enables two-photon absorption, for example – and these can be essentially anywhere, not at the strict, bound state levels. As I said, this is a rough analogy: it originated with Richard Feynman and is the best I can do for a high school student who likely has not dealt with quantum superposition before.
    The absorption and free propagation happen in quantum superposition, not strictly in sequence, so information is not being lost and when you write down the superposition of free photon states and excited matter states, you get something equivalent to Maxwell’s equations (in the sense I describe in my answer here or here) and the phase and group velocities naturally drop out of these.

    Another way of qualitatively saying my last sentence is that the absorber can indeed emit in any direction, but because the whole lot is in superposition, the amplitude for this to happen in superposition with free photons is very small unless the emission direction closely matches the free photon direction, because the phases of amplitudes the two processes only interfere constructively when they are near to in-phase, i.e. the emission is in the same direction as the incoming light.

    Das Absorbtions-Reemissions-Bild ist also doch richtig, wenn man berücksichtigt, dass die Atome zu virtuellen Energiezuständen angeregt werden und dass das Ganze als Superposition stattfindet?

    Kommt mir spontan ziemlich fishy vor, aber ich hab zu wenig Ahnung von der beteiligten Mathematik, um das einfach mal so beurteilen zu können.
    Habe auch nicht die richtigen Bücher da, um das nachschlagen zu können.
    Hm, vielleicht sollten wir mal bei MartinB nachfragen?

  988. #1003 PDP10
    30. April 2018

    @Niels:

    Für das Teilchenbild kenne ich allerdings eben wirklich nur die Absorbtions-Reemissions-Erklärung.
    Da habe ich noch nirgends irgendwo etwas besseres gelesen.

    Die ist aber so, wie du oben geschrieben hast – “dass ein Photon beim Durchgang durch ein Medium immer mal wieder von einem Atom absorbiert und kurze Zeit später reemittiert wird.” – mindestens missverständlich. Eigentlich falsch. Wie in dem Zitat, dass du oben gepostet hast geschrieben müsste man dann nämlich Absorbtions- oder Emissions-Linien sehen.

    Ferner schreibt ja der Joachim Schulz in dem von mir verlinkten Artikel dazu: “Dazu müsste die Photonenenergie, die proportional zur Frequenz ist, ausreichen um mindestens ein Elektron aus der Kristallbindung herauszulösen. Das geht bei Isolatoren nur mit Photonen, die deutlich jenseits der sichtbaren Wellenlängen im UV-Bereich liegen. Zu guter letzt würde ein Photon bei dem Prozess einer Absorption gefolgt von anschließender Emission seine Richtungsinformation verlieren. Glas wäre dann nicht mehr durchsichtig sondern diffus streuend wie Milchglas.”

    Mir gehts aber ansonsten genauso. Für das Teilchenbild kenne ich da bisher einfach keine griffige Erklärung.
    Die von dir zitierte finde ich allerdings ganz gut. Insbesondere, weil er / sie / es klar sagt, dass dabei die dielektrische Natur des Mediums die entscheidende Rolle spielt – das ist ja auch der Knackpunkt bei der Erklärung im Wellenbild.

    Den letzten Absatz finde ich allerdings etwas weit hergeholt – oder ich verstehe ihn einfach nicht.

    Vielleicht sollten wir wirklich mal den MartinB fragen – oder den Joachim :-).

  989. #1004 HF(de)
    30. April 2018

    Wird langsam spannend hier, sagt ein stiller Leser, sehr interessant! Und: sehr schön, dass Niels wieder da ist!

  990. #1005 PDP10
    30. April 2018

    @Niels:

    Nachtrag:

    Nach nochmaligem Lesen gefällt mir der letzte Absatz noch weniger. (Ausserdem kommt durch das Superpositionsprinzip ja doch wieder das Wellenbild da rein. Auch wenn es sich hier speziell um die Wellenfunktion des Photons handelt).

    Wenn ich mir das mal an frei fliegenden Molekülen – dem allseits beliebten CO2 zB. vorstelle: Da ist es ja gerade der Casus-Knacktus, dass die Schwingungsmoden des CO2 durch die Infrarotstrahlung die von der Erdoberfläche zurückkommen angeregt werden und diese die Photonen dann in alle Richtungen abstrahlen – eben auch wieder nach unten.

    Dreck! Jetzt verstehe ich das durch unsere Diskussion noch weniger als vorher … aber ok … dazu sind Diskussionen ja auch manchmal da :-).

  991. #1006 Alderamin
    1. Mai 2018

    @Niels, PDP10

    Soviel ich mich erinnere, regen die Photonen die Ladungen im durchlaufenen Material zu erzwungenenen Schwingungen an, die sich mit der Originalwelle überlagern und phasenverschoben sind. Durch die Phasenverschiebung in Ausbreitungsrichtung (und nur dort) ergibt sich irgendwie die Verzögerung, aber ich hab’s gerade nicht mehr komplett parat. Wichtig ist, nichts wird absorbiert oder re-immitiert. Bei frei beweglichen Ladungen (Metall) erhält man komplette Auslöschung im Material und Reflexion an der Oberfläche.

  992. #1007 Alderamin
    1. Mai 2018

    @myself

    Ähem, re-emmitiert. Muss ich mir nochmal Paroli passieren lassen 😉

  993. #1008 PDP10
    1. Mai 2018

    Das wäre dann Erklärung No. dreieinhalb … 🙂

  994. #1009 Niels
    1. Mai 2018

    @PDP10

    Wie in dem Zitat, dass du oben gepostet hast geschrieben müsste man dann nämlich Absorbtions- oder Emissions-Linien sehen.

    Nö, laut Zitat wird in einen virtuellen Energiezustand angeregt. Virtuelle Energiezustände haben vereinfacht gesagt ein kontinuierliches Spektrum. Das emittierte Photon hat immer genau die selbe Energie wie das zuvor absorbierte.
    https://en.wikipedia.org/wiki/Virtual_state

    Wo sollen da Spektrallinien herkommen?

    Dazu müsste die Photonenenergie, die proportional zur Frequenz ist, ausreichen um mindestens ein Elektron aus der Kristallbindung herauszulösen. Das geht bei Isolatoren nur mit Photonen, die deutlich jenseits der sichtbaren Wellenlängen im UV-Bereich liegen.

    Wie gesagt, es geht nicht um echte Absorption und echte erlaubte Energiezustände, sondern um imaginäre Energiezustände, die auch nur sehr kurzfristig eingenommen werden können.

    Hier noch eine kurze andere Erklärung dazu, runterscrollen bis zu Virtual Energy Levels:
    https://www.thephysicsmill.com/2014/03/30/a-quantum-of-scattering/

    Zu guter letzt würde ein Photon bei dem Prozess einer Absorption gefolgt von anschließender Emission seine Richtungsinformation verlieren.

    Da argumentiert der Autor meines obigen Zitats einerseits mit der Impulserhaltung und andererseits mit der Superposition.
    The interaction is so short that the absorber interacts with nothing else, so the emitted photon must bear the same momentum as the incident one.
    […]
    the absorber can indeed emit in any direction, but because the whole lot is in superposition, the amplitude for this to happen in superposition with free photons is very small unless the emission direction closely matches the free photon direction, because the phases of amplitudes the two processes only interfere constructively when they are near to in-phase, i.e. the emission is in the same direction as the incoming light.

    Den letzten Absatz finde ich allerdings etwas weit hergeholt – oder ich verstehe ihn einfach nicht.

    Wie gesagt, du müsste ich die konkreten Formeln dazu finden und mich dann eine Weile damit beschäftigen, um seriös beurteilen zu können, ob man das so sagen kann.
    Vom Bauchgefühl her kommt mir das aber auch komisch vor. Ist allerdings echt kein Gebiet, auf dem ich mich auskenne.

    Vielleicht sollten wir wirklich mal den MartinB fragen – oder den Joachim :-).

    Hiermit sei dir feierlich der Auftrag dazu erteilt. 😉
    .

    @Alderamin

    Klar. Das ist aber das Wellenbild bzw. der elektromagnetische Blickwinkel.
    Uns geht es aber eben um das Teilchenbild.

  995. #1010 Peter Paul
    15. Mai 2018

    Ich hätte eine sehr spezielle Frage, die sich mir stellte, als ich einen Artikel von S. Perlmutter über die beschleunigte Ausdehnung des Universums las (https://physicsforme.com/2011/10/04/supernovae-dark-energy-and-the-accelerating-universe/), und die mich jetzt schon länger, leider bisher ohne durchschlagenden Erfolg, beschäftigt hat:

    Kann man aus der Helligkeit, die man von einer SN Ia misst die Rückblickzeit zu dieser Supernova herleiten, ohne bereits ein spezielles Modell des Universums (damit sind die Omega-Werte gemeint) vorauszusetzen?

    Perlmutter tut das offenbar in seinem Diagramm 4, indem er dort der Rechtsachse zwei Bedeutungen gibt: Zeit und “Relative Brightness of Supernovae”. In dieses, für die verschiedenen Weltmodelle, gemeinsame Koordinatensystem zeichnet er dann die möglichen Ausdehnungskurven für diese verschiedenen Modelle ein.

  996. #1011 Jens
    22. Mai 2018

    Welche physikalischen Prozesse laufen bei der Explosion einer Paarinstabilitätssupernova ab und warum beobachtet man diese so selten?

  997. #1012 Alex B
    23. Mai 2018

    Folgender Artikel von Spektrum.de hat mich auf eine Frage gebracht:
    https://www.spektrum.de/news/neuer-starker-hinweis-auf-planet-neun/1566476

    Man vermutet, daß ein neuter Planet für diverse Bahnstörungen von Objekten im Kuiper-Gürtel verantwortlich seien.

    Meine Fragen nun:
    *) Könnte es nicht sein, daß einer der äußeren Planeten (Neptun z.b.) gar nicht in unserem Sonnensystem entstand, sondern ein ‘eingefangener Planet’ ist. Und dieser die beobachteten Bahnstörungen im äußeren Bereich (aufgrund des ‘eingefangen’ werdens) die Bahnstörungen verursacht hat ?
    *) Können Planeten überhaupt den interstellaren Raum durchqueren ? (Fluchtgeschwindigkeit)

    mit freundlichen Grüßen,

  998. #1013 Florian Freistetter
    23. Mai 2018

    @AlexB: “Könnte es nicht sein, daß einer der äußeren Planeten (Neptun z.b.) gar nicht in unserem Sonnensystem entstand, sondern ein ‘eingefangener Planet’ ist.”

    Das ist zwar theoretisch nicht unmöglich, aber so enorm unwahrscheinlich, das man es ausschließen kann. Dass ein Planet exakt mit der richtigen Geschwindigkeit zur richtigen Zeit am richtigen Ort an der Sonne vorbei fliegt um eingefangen zu werden, ist angesichts der Größe des Kosmos so extrem selten, dass es quasi nicht passieren kann.

    “Können Planeten überhaupt den interstellaren Raum durchqueren ? (Fluchtgeschwindigkeit)”

    Können sie: https://scienceblogs.de/astrodicticum-simplex/2011/05/18/ein-himmel-voller-planeten/ und https://scienceblogs.de/astrodicticum-simplex/2011/05/21/sind-die-vagabundierenden-planeten-gefahrlich/?all=1

  999. #1014 Dieter
    Cabarete
    28. Mai 2018
  1000. #1016 goellner
    volkach
    31. Mai 2018

    zeit und raum materie vor dem urknall sind nicht vorhanden. es war der geist. das universum ist geistlich anzuschauen, also erübrigt sich die frage was vor dem urknall war. es war geist, geist sonst nichts. es ist die dimmension des geistes. kein raum oder materie. der geist weht wo und wann er will. er ist nicht fassbar. der geist hat keinen anfang und kein ende. denken sie mal über den urknall nach. woher kommen die gedanken darüber. jawohl der geist ist und war. niemand kann seine gedanken im voraus vorhersagen. sie kommen aus dem nichts. ohne anfang und ende. also die gedanken sinds. die frage also ist, wer oder was hat sich gedanken gemacht. die bibel sagt den namen des schöpfers : sein nname ist : sein bzw. ist. mehr kann alle wissenschaft nicht dazu sagen. jesus sagt: die kreatur muss aus wasser und geist neu geboren werden . geist ohne raum und zeit, ohne materie, eben aus wasser und geist. der geist ist nicht fassbar aber er war vor dem urknall vorhanden. wir können die zeit vor dem urknall im geistlichen bereich, auf geistlicher ebene fassen….. der geist schwebte über das wasser… welchen schluss ziehen wir daraus, geist schuf das universum gedanken schufen das universum. wir kommen nur im geist zur erkenntnis geist ist ohne zeit und raum, geist hat keine grenzen .die gedanken eines menschen sind überall gleichzeitig . der geist iist unfassbar. der Geist ist der Geist an und für sich. der geist ist ohne anfang und ohne ende, der geist ist einfach da, wo wir menschen denken. in uns und auserhalb uns die frage also ist die was ist geist, woher kommte er. ich weis es nicht, aber ich denke darüber nach. ich weis nur eine inner stimme spricht mit mir und mit euch allen, hört einmal in euch hinein . der geist spricht ganz leise und doch real hörbar. der geist will den geist erhalten für unendlich . in diesem sinne bleibt geistlich. einen gesegneten geist wünsche ich euch allen, geistliche wesen .. immerfort bestehend imm geist, unsterblich.. ist das nicht eine schöne geistliches leben ……..

  1001. #1017 Noonscoomo
    Berlin
    31. Mai 2018

    Oha, welche Eigenschaften hat er denn, der Geist? Gibts/gabs nur einen oder von welchem sprichst du?

  1002. #1018 Alderamin
    31. Mai 2018

    Nicht hier diskutieren. Frage formulieren, Antwort bekommen. Ich habe jetzt allerdings keine Frage zur Astronomie in dem obigen Text gefunden, würde ich also als off-topic betrachten und an Florians Stelle löschen.

    So was kann man statt dessen im nicht themenbezogenen “Verschwörungsgeplauder” diskutieren, wenn man unbedingt will.

  1003. #1019 Florian Freistetter
    31. Mai 2018

    Irgendwann demnächst werd ich wohl mal hier aufräumen und alle Fragen löschen… damit wieder ein wenig Ordnung einkehrt 😉

  1004. #1020 Karl-Heinz
    31. Mai 2018

    @FF
    Diesen Thread schließen und einen neuen Thread aufmachen wäre ja auch eine Möglichkeit.
    Der jetzige Thread ist ja kein(e) threat. 😉

  1005. #1021 Dieter
    31. Mai 2018

    @Florian
    wenn man sich #1015 ansieht ergibt sich die Frage nicht automatisch? Dachte ich, entschuldigung!

    Astronomers Just Found 6 Bizarre Galaxies That Appear to Be ‘Empty’ of Stars

    hatte ich bisher nicht von gehöhrt, sind aber wohl nicht die ersten sternlosen Galaxien weißt du mehr darüber?
    Dieter

  1006. #1022 Ganymed
    13. Juli 2018

    Meine Frage hat zwar nicht direkt etwas mit Astronomie zu tun, aber wieso fanden die Apollo-Flüge stets bei zunehmenden Mond statt ?
    Hat das was mit Gravitationsverhältnissen zw. Erde-Mond zu tun ? (Treibstoff-Sparnis, verkürzte Flugzeit ?)

  1007. #1023 Alderamin
    13. Juli 2018

    @Ganymed

    Mit Gravitation hatte das nichts zu tun, die ändert sich ja nicht mit dem Winkel der Sonneneinstrahlung.

    Auf Quora habe ich gefunden, dass man einerseits eine relativ niedrig stehende Sonne haben wollte, damit der Pilot der Landefähre die Struktur des Bodens anhand der Schatten besser erkennen konnte. Und zur Morgenzeit des 29-tägigen Mondtages war der Boden noch nicht so von der Sonne aufgeheizt. Die Mondoberfläche kann an die 130°C erreichen. Die Mondlandungen fanden alle auf der Vorderseite des Mondes, nicht allzu weit von der Mitte der auf die Erde zugerichteten Seite statt, so dass Mond-Morgen dort mit einer zunehmenden Phase in der Nähe des Halbmonds zusammenfällt.

    Theoretisch hätte man auch in den Abendtagen des Mondzyklus landen können, aber der Morgen hatte den Vorteil, dass man nach hinten offen war für etwaige Verzögerungen, die ansonsten in die dunkle Mondnacht hätten führen können. Wie man sieht, haben sich die Planer bei der NASA damals eine Menge kluger Gedanken gemacht.

  1008. #1024 HF(de)
    14. Juli 2018

    Interessante Frage, interessante Antwort!

  1009. #1025 Bernd Wehmöller
    Essen
    17. Juli 2018

    Ich habe eine Frage zur Sternentstehung:
    Allgemein wird immer gesagt, dass die Materie klumpt und daraus Sterne mit einer Scheibe entstehen, und in der Scheibe entstehen dann die Planeten ebenfalls rein durch die Gravitation.
    Nun habe ich vor kurzem einen Vortrag von Prof. Wurm von der Universität Duisburg / Essen (Experimentelle Astronomie) gehört, in dem es um die Entstehung von Planeten geht. Er sagt, dass die Annahme des Klumpens aufgrund von Gravitation in einem bestimmten Bereich nicht mehr funktioniert, es gibt dort eine Bouncing Barriere (ih meine das war im Bereich von Millimetern zu dezimetern). Ist das allgemein anerkannt? Eine Ausgabe des Podcasts zu dem Thema wäre bestimmt spannend.

  1010. #1026 Florian Freistetter
    17. Juli 2018

    Muss ich mir mal anschauen; davon hab ich noch nix gehört. Danke für den Hinweis.

  1011. #1027 Bernd Wehmöller
    Essen
    17. Juli 2018

    Hier noch ein link auf das Thema
    https://arxiv.org/abs/1703.07124
    Gruß
    Bernd

  1012. #1028 Anderas Palm
    Dorf
    18. Juli 2018

    Als totaler Laie habe ich mal eine (vermutlich blöde) Frage: Es heißt, kurz nach dem Urknall war das Universum viele Millionen Grad heiß. Dann fing es an, sich allmählich abzukühlen. Ein Körper gibt seine Wärme an seine Umgebung und gleicht sich dabei Temperaturtechnisch an. Wohin aber hat das Universum seine Wärme abgegeben wenn außerhalb nichts ist und innerhalb alles gleich heiss war – wie konnte es abkühlen?

  1013. #1029 Alderamin
    18. Juli 2018

    @Anderas Palm

    Indem es sich vergrößerte. Damit wurde die Energie auf mehr Volumen verteilt und der Druck sank. Führt zu sogenannter “adiabatischer Abkühlung”.

    Ähnliches passiert, wenn man Gas aus einer Flasche ablässt, sie kühlt ab. Das Gas auch. Schickt man CO2-Gas durch einen Jute-Sack, bekommt man auf diese Weise Trockeneis, gefrorenes CO2.

    Ist auch der Grund, warum mit zunehmender Höhe im Gebirge (oder Flugzeug) die Temperatur sinkt.

  1014. #1030 Anderas Palm
    18. Juli 2018

    Schnell und verständlich erklärt. Vielen Dank.Werd Euch weiterempfehlen.

  1015. #1031 Peter
    19. Juli 2018

    Kürzlich hatte ich mit einem anderen Hobby-Astronomen folgende Diskussion: Wandern Planetenbahnen mit der Zeit vom Mutterstern weg oder zum Mutterstern hin?

    Unsere Sonne verliert ja ständig Masse durch den Sonnenwind. Wenn die Sonne leichter wird, müsste der mittlere Abstand der umlaufenden Planeten mit der Zeit immer größer werden. Aber wie groß ist dieser Effekt?

    Auch könnten Gezeitenkräfte wirken. Denn die Sonne rotiert doch bestimmt schneller, als die Planeten für einen Umlauf brauchen. Bei der Erde ist es ja auch so, dass sich der Mond mit der Zeit von der Erde entfernt, weil die Erde sich schneller dreht aus der Mond für einen Umlauf braucht (der Mond erzeugt einen Flutberg, den die Erde schneller mitdreht, als der Mond folgen kann. Der Flutberg läuft also dem Mond voraus, diese zusätzliche Masse beschleunigt den Mond leicht in Bahnrichtung).
    Allerdings werden die Gezeitenkräfte zwischen einem Planeten und der Sonne eher klein sein, da ein Planet maximal rund 1 ‰ der Sonnenmasse hat (der massereichste Planet Jupiter hat ca. 1/1048 der Sonnenmasse) und daher auf der Sonne keinen hohen Flutberg erzeugt.

    Oder wird der mittlere Abstand der umlaufenden Planeten mit der Zeit immer kleiner?

    Es heißt ja, dass durch den Umlauf eines Körpers um einen anderen Gravitationswellen erzeugt werden. Diese führen etwas Energie nach außen ab, wodurch die Bandgeschwindigkeit sinkt und die Umlaufbahn nach innen driftet.

    Auch könnten die Planeten bei ihrem Umlauf in Bahnrichtung mit Staub, Gasen, Meteoroiden, Asteroiden u. Kometen kollidieren, wodurch die Planeten abgebremst werden.
    Dieser Bremseffekt wird sicher in der Frühzeit des Sonnensystems relativ groß gewesen sein. Aber heutzutage ist der Raum zwischen den Planeten ja fast leer.
    Außerdem könnten manche Asteroiden o. Kometen einen Planeten ja auch „von hinten“ treffen und ihn in Umlaufrichtung beschleunigen.

    Gibt es noch weitere Effekte, welche die Planeten beschleunigen oder abbremsen könnten?

    Wie wirken sich z. B. andere Sterne, die nahe am Sonnensystem vorbeiziehen, auf die Umlaufbahnen der Planeten aus?

    Oder ist es so, dass sich beschleunigende und abbremsende Effekte die Waage halten und sich der mittlere Abstand der Planeten zum Mittelpunkt des Systems nicht verändert?

  1016. #1032 Karl-Heinz
    20. Juli 2018

    @Peter

    Kürzlich hatte ich mit einem anderen Hobby-Astronomen folgende Diskussion: Wandern Planetenbahnen mit der Zeit vom Mutterstern weg oder zum Mutterstern hin?

    Unsere Sonne verliert ja ständig Masse durch den Sonnenwind. Wenn die Sonne leichter wird, müsste der mittlere Abstand der umlaufenden Planeten mit der Zeit immer größer werden. Aber wie groß ist dieser Effekt?

    Man hat die Bahndaten der Merkur-Raumsonde MESSENGER aus sieben Jahren ausgewertet und so kleine Veränderungen im Schwerkraftregime des Merkurs und der Sonne zu registriert.
    Die Planeten rücken natürlich nach außen, wenn die Sonne Masse verliert. So um die 1,5 Zentimeter pro Jahr und astronomischer Einheit. Gravitationswellen kann man für diese Betrachtung vernachlässigen, da nur ganz wenig Energie und damit Masse nach außen getragen wird.

    Planetenbahnen rücken durch abnehmende Anziehung immer weiter nach außen.

  1017. #1033 UMa
    20. Juli 2018

    @Peter:
    Die Abstände der Erde und der anderen Planeten von der Sonne werden mit der Zeit größer. Hauptgrund ist der Masseverlust der Sonne durch Strahlung und Sonnenwind, der stärker als alle anderen, nichtperiodischen Effekte ist.

    Hier hatte ich 2006 schon mal meine Ergebnisse beschrieben:
    https://forum.astronomie.de/phpapps/ubbthreads/ubbthreads.php/topics/305864/Sonne_und_Erde_driften_immer_w

  1018. #1034 Karl-Heinz
    20. Juli 2018

    @UMa

    Gezeiten:
    In 100 Jahren:
    Sonne bremst Erdrotation +0.33 mm
    Erde bremst Sonnenrotation +0.06 nm (??um Faktor ~10 unsicher)

    Genau diese Werte hätte ich für die Gezeiten erwartet. 😉

  1019. #1035 Max Mustermann
    21. Juli 2018

    Auf dem Mond und auf dem Mars wurden Löcher entdeckt, die vermutlich Eingänge zu Höhlen sind.

    Wie seht es aber mit Asteroiden aus? Wären auch auf/in Asteroiden Höhlen möglich? Einige Asteroiden wurden ja schon von Raumsonden aus der Nähe fotographiert, aber Berichte über etwaige Höhlen konnte ich per Google nicht finden. Auch auf den Zwergplaneten Ceres und Vesta wurden keine gefunden, soweit ich weiß.

    Vielleicht gibt es ja einen Zusammenhang zwischen der Größe eines Himmelskörpers und der Möglichkeit von Höhlen?

  1020. #1036 Peter
    23. Juli 2018

    Ob es auf einem Himmelskörper Höhlen gibt, wird vor allem von dessen Zusammensetzung abhängen sowie von seinen Bahneigenschaften, der Stärke von Gezeitenkräften u. der Rotation, aber weniger von seiner Größe:

    Metallische Asteroiden, welche in der Frühzeit des Sonnensystems wahrscheinlich vollständig aufgeschmolzen waren, werden eher keine Höhlen besitzen, höchstens durch tiefe Einschläge.

    Asteroiden, die aus relativ porösem Gestein u./o. aus Eis bestehen, könnten dagegen durchaus Höhlen besitzen.

    In Kometen wären Höhlen relativ wahrscheinlich, da dort oft unterhalb der Oberfläche leicht flüchtiges Material ausgast u. an der Oberfläche oft eine festere Kruste (aus Material welches nicht schnell ausgast) zurückbleibt. Durch das Ausgasen könnten unter dieser Kruste Höhlen entstehen.

    Objekte in den Außenbereichen des Sonnensystems (Transneptunische Objekte) mit stärker elliptischen Bahnen erwärmen sich in Sonnennähe, wodurch feste Stoffe gasförmig (o. flüssig) werden könnten und sich Richtung Oberfläche o. nach unten bewegen, so dass dadurch Hohlräume entstehen könnten.

    Auf Eismonden können durch wechselnde Temperaturen (durch Tag/Nacht u. evtl. die Reibung durch die Gezeiten) u. durch Bewegungen infolge der Gezeitenkräfte Spalten u. Höhlen im Eis entstehen.
    Z. B. besteht der Saturn-Mond Hyperion zum Großteil aus relativ porösem Wassereis, seine Oberfläche ähnelt im Aussehen einem Schwamm.

    Auf eishaltigen Monden u. Zwergplaneten kann durch Gezeitenkräfte u. Tag/Nacht-Temperaturschwankungen Kryovulkanismus entstehen (wie z. B. auf Saturn-Mond Enceladus, Neptun-Mond Triton u. Pluto-Mond Charon), welcher dafür sorgen könnte, das sich unterirdische Hohlräume bilden.
    Selbst auf Ceres wurde ein Eisvulkan entdeckt [https://www.n-tv.de/wissen/Eisvulkan-auf-Zwergplanet-Ceres-entdeckt-article18544221.html].

    Auf Himmelskörpern mit ehemaligem Vulkanismus (wie dem Erdmond) oder noch vorhandener vulkanischer Aktivität (wie dem Jupiter-Mond Io) könnte es leere Magmaschlote/-gänge u. Lavahöhlen geben.

  1021. #1037 Peter
    23. Juli 2018

    Nach welchen Regeln werden eigentlich Exoplaneten benannt?

    Ich habe gelesen, dass der wissenschaftliche Name eines Exoplaneten aus dem Namen des Zentralsterns gebildet wird, an den ein Kleinbuchstabe angehängt wird. Und zwar würde beim zuerst entdeckten Planeten eines Systems ein „b“ angehangen, beim als 2. entdeckten Planeten ein „c“, usw.
    Beispiele:
    Stern: Proxima Centauri, zugehöriger Planet: Proxima Centauri b
    Stern: 109 Piscium; zugehöriger Planet: 109 Piscium b
    Stern: HD 12661, zugehörige Planeten: HD 12661 b, HD 12661 c

    Warum haben eigentlich die Sterne so unterschiedliche Namensformen (manche nur lateinische u./o. griechische Namen; andere einen Namen mit Nummer; manche nur Buchstaben-Zahlen-Folgen)?

    Wenn sich bei Exoplaneten der Buchstabe nach der Reihenfolge der Entdeckung richtet, was passiert dann, wenn gleichzeitig mehrere Planeten eines Systems entdeckt werden?

    Wird die Reihenfolge der Buchstaben im Nachhinein verändert, wenn diese nicht mit der Reihenfolge der Planeten vom Stern nach außen übereinstimmt?
    Z. B. wenn der innerste Planet den Namensanhang „c“ hat, dessen äußerer Nachbar „e“, der 3. Planet „d“ und der äußerste „b“.

    Wie geht man bei der wissenschaftlichen Benennung von Planeten in Mehrfach-Sternsystemen vor, wenn z. B. 2 o. 3 Sterne umeinander kreisen und außerhalb davon ein Planet um das Doppel-/Dreifach-Sternsystem kreist? Dieser Planet hat ja dann 2 oder 3 Zentralsterne.

    Bei Mehrfach-Sternsystemen müssten ja auch Sterne von den Planeten durch ihren Namen eindeutig unentschieden werden.
    Z. B. heißt der 2. Stern im Sirius-System Sirius B. Wenn man einen Planeten entdecken würde, der um Sirius A u. Sirius B kreist, und ihn Sirius b nennt, könnte man diesen Planeten ja leicht mit dem Stern Sirius B verwechseln.

    Wie wird eigentlich ein Exoplanet wissenschaftlich benannt, wenn er sich allein durchs Weltall bewegt, also nicht um einen Stern, Braunen Zwerg o. ein Schwarzes Loch kreist?
    Wurden überhaupt schon Exoplaneten entdeckt, die um Braune Zwerge o. Schwarze Löcher kreisen?

    Und wie würde man Zwergplaneten und Monde außerhalb des Sonnensystems benennen?

  1022. #1038 Dampier
    23. Juli 2018

    @Peter #1036
    Danke für die schöne Zusammenfassung. Ich hatte mir das intuitiv so ähnlich beantwortet, ohne aber Details zu wissen. Das war mal echt lesenswert.

  1023. #1039 Alex
    26. Juli 2018

    Zu diesem Artikel habe ich folgende Frage:
    https://scienceblogs.de/astrodicticum-simplex/2018/07/26/warum-sind-pflanzen-gruen-und-welche-farbe-hat-die-sonne/
    — Zitat —
    Die Antwort auf die zweite Frage ist simple: Die Sonne ist nicht gelb, sondern leuchtet weiß.

    Sind eigentlich alle Sterne weiß ? Ich hab immer gelernt, die Sonne sei deshalb ‘gelb’ weil ihre Oberflächen-Temperatur ~6000°C sei.

  1024. #1040 Mattis
    Bad Nauheim
    28. Juli 2018

    Beim Blutmond am 28.07.2018 fiel mir auf, dass die Verschattung linksseitig begann. Wie kann das sein, wenn der Mond sich doch nach rechts bewegt und in den Schatten “eintritt”, wie man so sagt? Ebenso wurde am Ende der Finsternis der linke Rand wieder strahlend hell, nicht der rechte, was man von einem “Austritt” aus dem Schatten erwarten würde. Wie erklärt sich das?

  1025. #1041 HF(de)
    28. Juli 2018

    Oh, die Frage kann glaub ich sogar ich erklären: es sieht nur so aus, als ob der Mond nach “rechts” wandert, weil die Erde sich nach “links” dreht. Dabei wandert der Mond allerdings ebenfalls nach links (nur viel langsamer als die Erddrehung) und taucht mit der linken Seite zuerst in den Schatten ein.

  1026. #1042 Mattis
    28. Juli 2018

    Danke HF(de) für die schnelle Antwort, das klingt plausibel. Merkwürdig, dass ich trotz eifriger Suche nichts dazu gefunden hatte, und niemand schien sich über diesen Aspekt zu wundern.
    Fast alle Grafiken stellen die Sache nur statisch dar, und man kann daraus eben nicht ersehen, wieso die Verschattung “links” beginnt, wo sich der Mond doch nach “rechts” zu bewegen scheint!
    Auf manchen Skizzen ist sogar ein Pfeil eingezeichnet, der bedeuten soll, dass der Mond von links nach rechts in den Schatten eintritt.

  1027. #1043 Peter
    28. Juli 2018

    Wie sieht eine Mondfinsternis aus, wenn man auf dem Mond steht?

    Was würde man sehen, wenn man auf der erdzugewandten Seite des Mondes steht und sich die Erde vor die Sonne schiebt?
    Bei einer Sonnenfinsternis auf der Erde sind ja Mond und Sonne etwa gleich groß.
    Vom Mond aus gesehen müsste die Erde ja größer sein als die Sonne. Aber wie viel größer (doppelt so groß, dreimal so groß o. noch mehr)?

    Wenn die Erde zum Teil vor der Sonne steht, würde man vom Mond aus am Rand der Erde irdische Gebirge oder die Erdatmosphäre sehen?
    Würde man, wenn man sich auf der Mondoberfläche umschaut, die leichte Verdunkelung durch den Halbschatten der Erde mit bloßem Auge sehen oder würde diese nicht auffallen?

    Wie würde es vom Mond aus aussehen, wenn die Erde die Sonne komplett verdeckt (der Mond komplett im Kernschatten liegt)?
    Sieht man die Erdatmosphäre dann als roten Ring, weil diese ja rotes Sonnenlicht in den Kernschatten bricht?
    Würde man auf dem Mond das schwache rote Licht mit bloßem Auge auf der Mondoberfläche sehen oder wäre es dort komplett dunkel?
    Und wäre vom Mond aus mit bloßem Auge gesehen die Rückseite der Erde komplett dunkel? Oder würde man dort einen schwachen Lichtschein sehen? Kann man künstliche Lichtquellen (z. B. große Städte/Ballungsräume) bis zum Mond sehen?

  1028. #1044 Mattis
    28. Juli 2018

    Kennt jemand eine gute Computer-Animation der Mondfinsterniss, bei der die Bewegungen aller drei beteiligten Himmelskörper gezeigt werden?

  1029. #1045 Alderamin
    29. Juli 2018

    @Mattis

    Eine Animation habe ich nicht anzubieten, aber folgendes: Der Erdschatten ist – logischerweise – der Sonne genau gegenüber. Die Sonne bewegt sich scheinbar täglich von Ost nach West über den Himmel (die Sterne nachts auch, Mond und Planeten ebenso), weil sich die Erde von West nach Ost dreht. Und der Erdschatten gegenüber der Sonne folglich auch. Deshalb wanderte der verfinsterte Mond mitsamt Erdschatten von Ost nach West.

    Das tun im Sonnensystem fast alle Körper: von „oben“ (Norden) gesehen, kreisen die Planeten gegen den Uhrzeigersinn um die Sonne, der Mond gegen den Uhrzeigersinn um die Erde. Die Sonne dreht sich in entsprechender Richtung um sich selbst. Einfach, weil das ganze Sonnensystem aus einer rotierenden Gas- und Staubscheibe entstanden ist, die sich in dieser Richtung dreht.

    Entsprechend wandert der Mond auf seiner Bahn von West nach Ost, gegen den Uhrzeigersinn um die Erde und auch in dieser Richtung durch den Erdschatten. Auf der Nordhablkugel von rechts nach links (auf der Südhalbkugel von links nach rechts).

  1030. #1046 Alderamin
    29. Juli 2018

    @Peter

    Wie sieht eine Mondfinsternis aus, wenn man auf dem Mond steht?

    Dazu habe ich was in meinem Artikel über die Mondfinsternis geschrieben.

    Bei einer Sonnenfinsternis auf der Erde sind ja Mond und Sonne etwa gleich groß.
    Vom Mond aus gesehen müsste die Erde ja größer sein als die Sonne. Aber wie viel größer (doppelt so groß, dreimal so groß o. noch mehr)?

    Die Erde durchmisst etwa 12750 km, der Mond 3480 km. Die Erde ist also etwa 3,5-mal so groß wie der Mond.

    Wenn die Erde zum Teil vor der Sonne steht, würde man vom Mond aus am Rand der Erde irdische Gebirge oder die Erdatmosphäre sehen?

    Nein, keine Gebirge, man sieht ja auch mit bloßem Auge keine Berge und Krater auf dem Mond (der größte Krater, Clavius, durchmisst über 300 km). Mit der Auflösung des bloßen Auges kann man bestenfalls 1/30 des Monddurchmessers (also rund 100 km) ausmachen, wenn der Kontrast optimal ist. Auf der Erde gilt in umgekehrter Richtung das gleiche. Mit dem Teleskop kommt man runter bis auf 0,5-1 km Auflösung.

    Würde man, wenn man sich auf der Mondoberfläche umschaut, die leichte Verdunkelung durch den Halbschatten der Erde mit bloßem Auge sehen oder würde diese nicht auffallen?

    Würde wohl bis über 50% Bedeckung nicht auffallen, auch bei einer Sonnenfinsternis merkt man die Verdunklung erst jenseits der 50% und richtig dunkel wird es erst jenseits 99%. Das Auge passt sich an die verfügbare Helligkeit an.

    Wie würde es vom Mond aus aussehen, wenn die Erde die Sonne komplett verdeckt (der Mond komplett im Kernschatten liegt)?
    Sieht man die Erdatmosphäre dann als roten Ring, weil diese ja rotes Sonnenlicht in den Kernschatten bricht?

    Ja, siehe auch meinen Artikel mit Bildern.

    Würde man auf dem Mond das schwache rote Licht mit bloßem Auge auf der Mondoberfläche sehen oder wäre es dort komplett dunkel?

    Es würde sicher reichen, dass man sich umsehen könnte, aber nicht zum Lesen.

    Und wäre vom Mond aus mit bloßem Auge gesehen die Rückseite der Erde komplett dunkel? Oder würde man dort einen schwachen Lichtschein sehen? Kann man künstliche Lichtquellen (z. B. große Städte/Ballungsräume) bis zum Mond sehen?

    Denke nicht, dass man die Lichter ohne optische Hilfsmittel erkennen könnte, der Ring um die Erde würde blenden, die Lichter recht schwach und klein sein, ein Pendant zur Milchstraße an unserem Himmel. Mit einem Teleskop könnte ich es mir schon vorstellen.

  1031. #1047 Mattis
    31. Juli 2018

    Auch an Alderamin vielen Dank. Der springende Punkt ist, dass von meiner Position aus (Deutschland), nach Süden blickend, der Mond sich ja nur scheinbar nach rechts bewegt; wenn ich es richtig verstanden habe deshalb, weil die Erde sich unter ihm Richtung Osten dreht (Rotation) und zwar schneller als die Mondbewegung, die “eigentlich” auch Richtung Osten erfolgt.
    Im Falle der jüngsten Mondfinsternis müsste das bedeuten, dass der Mond von rechts nach links in den Schatten eintritt – immer von meiner o.g. Position aus gesehen – während Schatten und Mond optisch nach rechts driften wegen der schnelleren Eigenrotation der Erde nach Osten.
    Was mich aber wundert: dass es kaum jemanden erstaunt hat, wieso die Verschattung zuerst links zu sehen ist.

  1032. #1048 Peter
    2. August 2018

    Vielen Dank an Alderamin für die Erklärungen (#1046), wie eine Mondfinsternis vom Mond aus aussieht.

    Ich habe dazu noch ein Video der NASA von 2014 gefunden:
    https://www.der-mond.org/mondfinsternis/sonnenfinsternis-vom-mond-aus-gesehen-video/

    Nach deinen Erläuterungen sind in diesem Video die Lichter der Städte auf der Nachtseite der Erde übertrieben hell dargestellt.

    Komisch an dem Video finde ich auch, dass die Mondoberfläche zweimal kurz stockdunkel wird: Als sich die Erde komplett vor die Sonne geschoben hat, wird es zuerst stockdunkel. Erst etwas später sieht man auf der Mondoberfläche das schwache rote Licht, welches durch die Erdatmosphäre in den Erdschatten abgelenkt wird. Auch kurz bevor die Sonne wieder hinter der Erde hervorkommt, wird die Mondoberfläche noch einmal stockdunkel.

    Ich hätte eher erwartet, dass man das rote Licht schon auf der Mondoberfläche sieht, wenn die Erde erst ganz knapp vor der Sonne steht.
    Und dass es dann am dunkelsten auf der Oberfläche ist, wenn sich wenn die Sonne vom Mond aus gesehen am weitesten von den Rändern der Erde entfernt ist, also genau in der Mitte der totalen Finsternis.
    Von der Erde aus gesehen sieht man ja auf dem Mond, dass der Kernschatten in der Mitte am dunkelsten ist und zu den Rändern hin allmählich heller und gelblicher wird.

  1033. #1049 Peter
    2. August 2018

    Ich habe gelesen, dass es an den Polen des Erdmondes Berggipfel gibt, auf die immer die Sonne scheint, und Täler u. Krater, die immer im Schatten liegen.
    Wenn das stimmt, müsste die Rotationsachse des Mondes senkrecht zur Sonneneinstrahlung stehen.

    Der Mond hat laut Wikipedia eine Achsneigung von 6,68°. Diese Zahl bezieht sich wahrscheinlich auf die Achsneigung zur Mondbahn.
    Die Mondbahn selbst ist gegenüber der Ekliptik (scheinbare Bahn der Sonne am Himmel von der Erde aus gesehen um 5,2° geneigt ist.

    Nach diesen Zahlen bleibt also immer noch eine Neigung der Mondachse von 1,48° zur Ekliptik.
    Aber das müsste ja für ständige Helligkeit bzw. Dunkelheit reichen, wenn die Erhebungen hoch genug u. die Täler/Krater tief genug sind.

    Wie stehen eigentlich Mondachse und Erdachse zueinander?
    Wenn man sich in einem relativ weit nördlichen Land wie Deutschland befindet, ist ja der Nordpol des Mondes ziemlich genau ganz oben und nicht am Rand oben links o. oben rechts.
    Aber Erdachse u. Mondachse können ja nicht parallel zueinander sein. Schließlich hat die Erdachse eine Neigung von 23,44° zur Ekliptik.

  1034. #1050 Reinhard 49
    Stockstadt
    6. August 2018

    Ein Satellit umkreist die Erde. Die für den Satelliten notwendige Zentripetalkraft (*) ist gleich der Gravitationskraft (*) zwischen Satellit und Erde.
    (* Bei Gleichsetzung der entsprechenden Formeln kürzt sich m (Masse des Satelliten) heraus. Ist das so zu verstehen, dass die Masse des Satelliten keine Rolle spielt? Und wie ist das bei der ISS, wenn z.B. sich die Masse durch ein neues Modul erhöht? Nach meinem Verständnis müsste nach dem Andocken die Geschwindigkeit und/oder die Umlaufbahn geändert werden?

  1035. #1051 Alderamin
    6. August 2018

    @Peter #1048

    Komisch an dem Video finde ich auch, dass die Mondoberfläche zweimal kurz stockdunkel wird: Als sich die Erde komplett vor die Sonne geschoben hat, wird es zuerst stockdunkel. Erst etwas später sieht man auf der Mondoberfläche das schwache rote Licht, welches durch die Erdatmosphäre in den Erdschatten abgelenkt wird. Auch kurz bevor die Sonne wieder hinter der Erde hervorkommt, wird die Mondoberfläche noch einmal stockdunkel.

    Das soll wohl simulieren, dass man die Blende der Optik während der totalen Bedeckung öffnet, so dass mehr Licht zur Aufnahme der Landschaft zur Verfügung steht, dadurch wird es heller. Bevor die Sonne dann am Erdrand wieder auftaucht, wird die Blende wieder weiter geschlossen.

    Ich hätte eher erwartet, dass man das rote Licht schon auf der Mondoberfläche sieht, wenn die Erde erst ganz knapp vor der Sonne steht.

    Solange noch ein Teil der Sonne nicht bedeckt ist, ist deren direktes Licht viel zu hell, als dass die Landschaft rötlich erscheinen würde; auch wenn man auf die Erde schaute, würde man so stark geblendet, dass man die Atmosphäre nicht wird sehen können. Um die rot erleuchtete Landschaft zu sehen, wird man sich erst an die Dunkelheit gewöhnen müssen.

  1036. #1052 Peter
    6. August 2018

    Ein Brennpunkt der elliptischen Umlaufbahn (o. der Mittelpunkt der kreisförmigen Umlaufbahn) eines Körpers, der die Erde umläuft, ist nicht genau der Erdmittelpunkt, sondern das gemeinsame Massezentrum von Erde und Satellit (künstlich o. natürlich). Erde u. Satellit umlaufen sich also gegenseitig.

    Die Masse eines künstlichen Satelliten o. einer Raumstation ist im Vergleich zur Masse der Erde sehr, sehr viel kleiner. Daher liegt das gemeinsame Massezentrum mit der Erde sehr, sehr nahe am Erdmittelpunkt.
    Beim Erdmond (ca. 1/81 der Erdmasse) liegt das gemeinsame Massezentrum von Erde u. Mond ca. 1700 km unter der Erdoberfläche, das heißt der Mond bringt die Erde zum „Eiern“.
    Wenn man das Gewicht eines künstlichen Satelliten o. einer Raumstation erhöht, wandert das gemeinsame Massezentrum ein sehr kleines Stückchen weiter vom Erdmittelpunkt weg.
    Dieser Effekt ist aber so gering, dass er in der Realität nicht ins Gewicht fällt. Was sind schon die 20 Tonnen eines zusätzlichen ISS-Moduls gegen die ca. 5,97 Trilliarden Tonnen der Erde? Das gemeinsame Massezentrum dürfte sich geschätzt nur um einige Atomkerne weit vom Erdmittelpunkt weg verschieben.

    Einen viel größeren Einfluss hat im erdnahen Orbit die Abbremsung durch die Reibung mit der Erdatmosphäre. Z. B. verliert die ISS, welche in ca. 400 km Höhe kreist, pro Tag ca. 50 – 150 m Höhe. Deshalb muss sie regelmäßig angehoben werden, ansonsten würde sie in wenigen Jahren verglühen. Wenn Raumtransporter wie z. B. die „Progress“ an die ISS andocken, wird diese um ca. 3 km angehoben.

  1037. #1053 Peter
    6. August 2018

    Kennt jemand gute Webseiten, wo man astronomische Koordinaten eingeben kann und dann herausfindet, welcher Himmelskörper sich dort befindet?

    Ein Bekannter von mir ist zur Zeit mit einem kleinen Fernrohr in Afrika und hat mir geschrieben, er habe an einer bestimmten Stelle etwas gesehen, was er nicht in seiner analogen Sternkarte gefunden hat, und hat mir die Koordinaten geschickt.
    Leider weiß er nicht, ob es ein lichtschwacher Stern, ein Gasnebel, eine Galaxie etc. war. Jedenfalls habe es sich gegenüber den Sternen nicht bewegt.

    Ich kann gerade selbst nicht am Himmel nachsehen, da in meinem Urlaubsort fast nur Wolken sind.

  1038. #1054 Alderamin
    6. August 2018

    @Peter #1049

    Merke ich mir für einen Artikel vor.

    @Peter #1053

    Bei WikiSky.org kann man Himmelskoordinaten in das Suchfeld eingeben (im gleichen Format, wie sie rechts oben angezeigt werden, also xxhyymzzs, ±xx°yy’zz.z”) und dann auf Objekte klicken. Jedes Planetariumsprogramm (Stellarium, Cartes du Ciel) tut’s auch.

    Man muss aber schon wissen, was man sucht, bei hinreichender Grenzgröße sind ja etliche Objekte (Sterne, Galaxien) zu sehen. Ansonsten muss die Position schon sehr exakt angegeben werden, und ob Dein Bekannter dazu in der Lage ist (Messung relativ zu benachbarten Sternen bekannter Position) würde ich nicht ohne weiteres erwarten.

    Planeten sind auch nicht auf Sternkarten eingetragen, weil es “Wandelsterne” sind, es ist nicht zufällig der Mars, den er da nicht auf der Karte findet?

  1039. #1055 Florian Freistetter
    6. August 2018

    @Peter: stellarium.org Kostenlose Planetariumssoftware, die dir den Himmel an einem beliebigen Ort zu einer beliebige Zeit zeigt.

  1040. #1056 Jens
    6. August 2018

    Warum wurde seit fast einem Jahr keine Gravitationswelle mehr nachgewiesen?

  1041. #1057 Alderamin
    7. August 2018

    @Jens

    LIGO beobachtet in jeweils mehrmonatigen “observation runs” und wird zwischen den runs verbessert und ausgebaut. Der letzte run endete am 25. August 2017 und der nächste soll erst Anfang 2019 beginnen. Am Ende des 3. Runs wird man dann wohl erst dessen Ergebnisse erfahren.

  1042. #1058 Anderas Palm
    13. August 2018

    Gibt es eigentlich eine “Oberlimit” für die Größe eines schwarzen Loches und was würde passieren wenn es diesen Punkt erreicht?

  1043. […] einigen Jahren gibt es in meinem Blog ja die Rubrik Fragen zur Astronomie. Dort kann man alle Fragen stellen die man eben so stellt. Früher habe ich es noch geschafft, eine […]

  1044. #1060 Metalgeorge
    16. August 2018

    Stern hat Chaos am Rande unseres Sonnensystems hinterlassen.
    https://www.businessinsider.de/vor-70000-jahren-hat-ausserirdischer-stern-unser-sonnensystem-beruehrt-2018-7
    Scholz’ Stern , hatte ich noch nie zuvor gehört.
    Stimmt das so?

  1045. #1061 Metalgeorge
    16. August 2018

    @Anderas Palm
    habe hierzu etwas hier gefunden,
    https://www.scinexx.de/wissen-aktuell-19664-2015-12-21.html

  1046. #1062 Florian Freistetter
    16. August 2018

    @Metalgeorge: Abgesehen davon dass das eine grottenschlechte Quelle ist und enorm missverständlich/hysterisch formuliert ist: Ja, das ist echte Wissenschaft. Hier ist ein seriösere Artikel: https://www.nature.com/news/star-buzzed-solar-system-during-human-prehistory-1.16958

  1047. #1063 Metalgeorge
    16. August 2018

    @FF
    …dachte ich mir:)
    Vielen Dank für den Link.

  1048. #1064 Florian Freistetter
    16. August 2018

    @Metalgeorge: Sorry, der nature-Artikel ist älter. Das ist die aktuelle Sache: https://arxiv.org/abs/1802.00778 Ist aber, wie gesagt, lange nicht so hysterisch wie es der BI (der mir in letzter Zeit immer mehr negativ auffällt wenn es um Wissenschaft geht) darstellt.

  1049. #1065 Metalgeorge
    16. August 2018

    @FF
    super!
    Nochmals vielen Dank.
    Die erwähnte site ist mir auch schon negativ aufgefallen,
    taucht aber oft im Zusammenhang mit wissenschaftlichen Entdeckungen auf,
    daher u.A. auch meine Frage:)

  1050. #1066 Peter
    16. August 2018

    Zu nahen Sternenpassagen habe ich noch folgendes gefunden: Unter

    https://www.scinexx.de/wissen-aktuell-23033-2018-08-13.html

    wird geschrieben, dass in der Frühzeit des Sonnensystems vor ca. 4 Milliarden Jahren wahrscheinlich ein Stern mit ca. einer halben bis einer Sonnenmasse die Sonne etwa im Abstand von 100 astronomischen Einheiten (ca. 3-facher Abstand Sonne-Neptun) passierte und dabei einem Teil des Material der protoplanetare Wolke des Sonnensystems mit sich riss.
    Dies könnte die Ursache dafür sein, dass es heute jenseits des Neptuns relativ wenige Objekte gibt.
    Eine nahe Sternenpassage hätte viele Himmelskörper aus ihren ursprünglichen Bahnen gelenkt, was erklären könnte, warum viele transneptunische Himmelskörper Bahnen haben, die gegen die Ebene der inneren Planetenbahnen geneigt und relativ elliptisch sind.

  1051. #1067 Metalgeorge
    16. August 2018

    @Peter
    vielen Dank für den Link.
    Habe ich bisher auch noch nicht gewusst.

  1052. #1068 Peter
    16. August 2018

    Auf der Erde wurden ja schon Meteoriten gefunden, die ursprünglich von Mond o. vom Mars o. aus dem Asteroidengürtel stammen.

    Doch wie sieht es mit Meteoriten von den Himmelskörpern weiter draußen aus?
    Wurden auf der Erde schon Meteoriten gefunden, die z. B. Stücke eines Jupiter-Mondes / Saturn-Mondes / Uranus-Mondes / Neptun-Mondes sind, oder die von einem transneptunischen Himmelskörper oder gar von einem Himmelskörper außerhalb des Sonnensystems stammen?

  1053. #1069 Florian Freistetter
    17. August 2018

    @Peter: Siehe dazu hier: https://scienceblogs.de/astrodicticum-simplex/2013/12/16/hat-die-erde-ihr-leben-ins-all-exportiert/?all=1 (Und auch die Sternengeschichten-Folge 117)

  1054. […] stibitze mal eine Frage aus Florian Freistetters “Fragen zur Astronomie“. Dort fragte @Peter #1049  nach der Achsneigung des Mondes, mit der das im ewigen Schatten […]

  1055. #1071 Mirko
    HH
    24. August 2018

    Hallo FF, kannst du mal was zu Sabine Hossenfelder bzw ihrem aktuellen Buch schreiben? Wie ist Deine Meinung dazu? Ist das überfällig/populistisch/unsinnig/genau richtig? Lohnt sich das Buch? Also ich meine „Lost in Math…“, erscheint Ende September auf deutsch mit dem komischen Titel „Hässliches Universum„ o.s.ä.

  1056. #1073 Peter
    24. August 2018

    Bevor Pluto entdeckt wurde, hatte man doch Bahnstörungen des Neptun festgestellt. Anhand der Bahnstörungen berechnete man, wo sich der gesuchte 9. Planet befinden würde. In diesem Suchgebiet tauchte dann zufällig Pluto auf. Nach der Entdeckung Plutos stellte man aber schnell fest, dass Plutos Masse viel zur gering ist, um die Bahnstörungen des Neptun zu erklären.

    Gab es damals wirklich Bahnstörungen des Neptun oder waren das nur Messfehler?

    Astronomen suchen ja heute noch nach einem massereichen 9. Planeten.

  1057. #1074 Florian Freistetter
    24. August 2018

    @Peter: Die Probleme bei Neptun waren auf eine ungenau bestimmte Masse zurückzuführen. Hab ich in dieser Serie genauer erklärt: https://scienceblogs.de/astrodicticum-simplex/2016/01/28/die-lange-suche-nach-planet-x-2016-die-entdeckung-von-planet-9/

  1058. #1075 Mirko
    24. August 2018

    Uups, Danke!

  1059. #1076 Dieter Schläger
    Deisenhausen
    29. August 2018

    Immer wieder höre und lese ich, dass Asteroiden und Kometen die die Erde bedrohen, mit einem “daneben” geparkten Satelliten auf längere Sicht durch die geringe Anziehungskraft des Satelliten aus der Bahn gelenkt werden können. Das kommt mir recht unwahrscheinlich vor, denn ein Asteroid der auf der Erde beträchtlichen Schaden anrichten kann, müsste doch deutlich grösser sein als der Satellit. Damit wird der Satellit stärker an den Asteroiden herangezogen als umgekehrt. Um sich dagegen zu wehren wären doch unheimliche Mengen Antriebsenergie – also Treibstoff – erforderlich! Bringts das denn?

  1060. #1077 HF(de)
    29. August 2018

    Keine Ahnung, ob das hilft, aber es gibt hier eine ganze Serie zur Asteroidenabwehr, s. hier:
    https://scienceblogs.de/astrodicticum-simplex/2013/07/17/dunkle-materie-exoplaneten-und-asteroidenabwehr-alle-serien-bei-astrodicticum-simplex/

  1061. #1078 Alderamin
    29. August 2018

    @Dieter Schläger

    Es macht keinen Unterschied im Energieaufwand (Treibstoffverbrauch), ob man einen Asteroiden abschleppt, (unelastisch) rammt oder mit einem Triebwerk anschiebt. Die übertragene Energie ist bei gleichem Treibstoffaufwand (beim Rammen während der Beschleunigung des Satelliten) immer dieselbe. Der Vorteil beim Traktor ist, dass die Kraft sehr lange wirkt und besser gesteuert werden kann. Bei einem Einschlag könnte ja auch ein Teil der Energie in Rotation übergehen, weil man den Asteroiden nicht genau mittig über dem Schwerpunkt trifft.

    Nach Newton ziehen sich Asteroid und Satellit wechselseitig mit der gleichen Kraft an. Da Beschleunigung = Kraft durch Masse ist, wird der leichte Satellit mehr beschleunigt, wenn er auf den Asteroiden fällt, als umgekehrt. Aber auch der Asteroid fällt auf den Satelliten zu. Wenn der Satellit aber seine Position mit den Triebwerken hält, fällt nur noch der Asteroid auf den Satelliten zu. Sehr, sehr langsam, aber stetig.

  1062. #1079 Karl-Heinz
    30. August 2018

    @Dieter Schläger

    Was vielleicht ein bisschen untergegangen ist.
    Wenn ich es schaffe die Geschwindigkeit des Asteroiden um delta v zu ändern, so ist der Distanzunterschied d bis zum möglichen Einschlag gleich d=Δv*T. T ist die restliche Zeit bis zum möglichen Einschlag. Man sieht auf Anhieb, je früher desto besser.

  1063. #1080 Peter L.
    1. September 2018

    Kann man Neutronenstern-Materie hier auf der Erde (im Labor) o. im erdnahen Weltraum (in der Schwerelosigkeit) künstlich herstellen, z. B. durch Verschmelzung von Atomkernen u. Elektronen?

    Es muss ja nicht viel sein, z. B. wurden auch von vielen Transuranen oft nur wenige Mikrogramm künstlich hergestellt.

    Oder fehlen der Menschheit heute noch die technischen Mittel, um Neutronenstern-Materie herzustellen?

  1064. #1081 Karl-Heinz
    1. September 2018

    @Peter L.

    Na ja.
    Ohne entsprechende Gravitation wird dir die Neutronenstern-Materie um die Ohren fliegen.

  1065. #1082 tomtoo
    1. September 2018

    @Karl-Heinz,@Peter L.
    Sammeln kann man sie nicht, aber macht, man nicht genau das in Kollidern? Für ultrakurze Mommente.

  1066. #1083 tomtoo
    1. September 2018

    @Karl-Heinz,@Peter L.
    Sammeln kann man sie nicht, aber macht, man nicht genau das in Kollidern? Für ultrakurze Mommente.

  1067. #1084 Karl-Heinz
    1. September 2018

    @tomtoo
    Jo… 😉

    Mit dem Experiment CBM (Compressed
    Baryonic Matter) kann dies untersucht
    werden, denn dort wird solch hochkom-
    primierte Materie im winzigen Maßstab
    experimentell hergestellt. Dazu lassen
    die Wissenschaftler zwei schwere Atom-
    kerne mit hoher Energie zusammensto-
    ßen, sodass sie zu einem extrem dichten
    Feuerball zusammengepresst werden.
    Dieser Feuerball existiert zu kurz für eine
    direkte Untersuchung, aber die nachfol-
    gende Explosion lässt sich beobachten.

  1068. #1085 Peter L.
    13. September 2018

    Wo kann man Meteoriten bestimmen lassen?

    Ich habe einen Meteoriten auf einem Flohmarkt gekauft, dazu gehört ein Zettel, auf welchem steht: „Type unclassified“, nur der Fundort (die Sahara-Wüste) ist angegeben.

    Wo kann man in Deutschland bestimmen lassen, um welchen Typ es sich handelt, also ob es ein Steinmeteorit, Stein-Eisen-Meteorit oder Eisen-Meteorit ist und welcher Unterklasse er angehört (bei Steinmeteoriten z. B. Achondrite; Enstatit-Chondrite, Forsterit-Chondrite, Gewöhnliche Chondrite, Kakangari-Chondrite, Kohlige Chondrite, Rumuruti-Chondrite)?

    Der Meteorit ist von außen rotbräunlich und ist leicht magnetisch, enthält also wahrscheinlich Eisen.

  1069. #1086 Karl-Heinz
    13. September 2018

    @Peter L.
    Frage an Peter: Wie viel hat der Meteorit vom Flohmarkt gekostet?

  1070. #1087 Peter L.
    13. September 2018

    Zitat Karl-Heinz „Wie viel hat der Meteorit vom Flohmarkt gekostet?“

    5 € mit „Mengenrabatt“, weil ich am gleichen Stand noch ein paar andere Steine u. Fossilen gekauft habe.
    Der Meteorit ist 4,7 cm lang, 2,5 cm breit, wiegt etwa 25 g und hat eine Schmelzkruste, die zum Teil verwittert ist.

  1071. #1088 Jason
    13. Oktober 2018

    Ich hätte eine Frage. Hat die NASA wirklich eine interstellare Mission zu Proxima Centauri im Jahr 2069 geplant?
    Hier gibt es einige Artikel über dieses Thema:

    https://www.heise.de/newsticker/meldung/NASA-Ueberlegungen-fuer-interstellare-Mission-zu-Alpha-Centauri-ab-2069-3924533.html

    https://www.derstandard.de/story/2000071214236/nasa-forscher-erwaegen-mission-nach-alpha-centauri

    https://bgr.com/2017/12/19/alpha-centauri-mission-nasa-2069/

    Wenn man das googlet, findet man noch viel mehr. Sind das wirklich ernsthafte Konzepte? Werden wir dann die Technologie dazu haben? Würde eine solche Raumsonde mit einer Rakete gestartet werden oder mit Lasersegel. Sind solche Reisen technologisch möglich?

  1072. #1089 Peter Paul
    24. Oktober 2018

    Wenn ich es recht verstanden habe, entsteht die kosmische Hintergrundstrahlung durch Entkopplung von der normalen Materie, als das primordiale Plasma durch Abkühlung in ein Atomgas überging.
    Ein paar hundert Millionen Jahre später ging es aber wieder in den ionisierten Zustand über. Dieser Vorgang, der auf die Strahlung der ersten Sterne zurückgeführt wird, wird Reionisation genannt. Wieso führte diese Zustand eigentlich nicht wieder zur Kopplung mit der Hintergrundstrahlung?

  1073. #1090 Alderamin
    24. Oktober 2018

    @Peter Paul

    Na, weil zur Zeit der Entstehung der Hintergrundstrahlung die Gasdichte noch sehr hoch und das Gas gleichverteilt im Raum war. Da war das Weltall ja gerade erst 380.000 Jahre alt.

    Nach ein paar hundert Millionen Jahren bildeten sich Galaxien, weil das Gas sich dort verdichtete, dort entstanden Sterne und nur dort wurde das Gas ionisiert. Im leeren Raum dazwischen nicht (da fliegt heute noch neutrales Gas durch die Gegend). Im Nahbereich um die Galaxien herum wurde das Gas durch Supernovaexplosionen mit Ionen angereichert.

    Aber das Gas ist so dünn, dass die meisten Photonen da problemlos hindurchkommen, ohne mit Elektronen zu interagieren. Der eigentliche Grund ist, dass zur Zeit der Reionisation das Weltall schon viel größer war und das Gas nicht mehr gleichverteilt, sondern konzentriert in Galaxien und stark verdünnt dazwischen.

  1074. #1091 Peter Paul
    25. Oktober 2018

    VIelen Dank, Alderamin. So etwa hatte ich mir das bis jetzt auch vorgestellt, aber dann las ich in dem Buch “Kosmologische Strukturbildung” von Helmut Hetzneckerder, in Reihe “Astrophysik aktuell” des Spektrum Verlags, herausgegeben 2009, folgendes : “Aus den Spektren zahlreicher Quasare weiß man heute, dass das Universum bei z ungefähr 6 bis 7 vollständig (!) ionisiert war! Man spricht in diesem Zusammenhang von der Reionisierung des Universums.” ( S. 121; Ausrufezeichen ist vom Autor!).
    Er begründet das dort damit, dass die ionisierten Blasen, die sich um die Sterne bilden, immer mehr anwachsen und sich immer mehr überlappen, und deshalb “bald den gesamten Kosmos einnehmen!”.
    Das scheint doch deiner/unserer Meinung deutlich zu widersprechen.

  1075. #1092 Alderamin
    25. Oktober 2018

    @Peter Paul

    Selbst wenn die Ionisation (dann wohl durch die Supernovae) den ganzen Raum erfasste, war die Materiedichte bei z=6-7 im Mittel schon 5 Millionenmal geringer als bei z=1100 zur Zeit der Hintergrundstrahlung (Volumenzunahme mit (1100/6,5)^3 überschlagen). Und weil das Gas sich schon in den Galaxien konzentriert hatte, war es in den Räumen dazwischen noch viel dünner verteilt.

    Wir können ja auch heute durch die Blasen ionisierten Gases hindurch die Galaxien sehen, wie z.B. M82 im Großen Bären.

  1076. #1093 Peter Paul
    25. Oktober 2018

    Ich denke, damit hast du im Prinzip schon recht, aber ein wenig möchte ich doch noch den Griffel spitzen, das kennst du ja schon von mir.
    Folgender Artikel (https://the-educational-blog.quora.com/The-EDGES-Experiment-This-May-Deserve-Two-Nobel-Prizes) habe ich entnommen, dass die ersten Sterne etwa bei z = 18 bis 16 maximal ionisiert haben, denn da geht die 21cm Strahlung am meisten zurück.Der Grund dafür ist Ionisation des Wasserstoffs.
    Ich bin mir jetzt nicht sicher, ob das schon die kosmische Reionisation ist, aber in dem von mir zitierten Buch werden diese ersten Sterne dafür verantwortlich gemacht, nämlich die Population III Sterne
    Rechnet man mal mit z = 17, dann ist die Volumenvergrößerung nicht mehr ganz so eindrucksvoll , nämlich “nur” noch das ca. das 270 000 Fache, aber das reicht wohl/vielleicht zur Beibehaltung deiner Argumentation aus.

    Andererseits schreibt Hetznecker (2009) auf Seite 122: “WMAP, das große Orakel der modernen Kosmologie, lehrt uns, dass sich der Prozess Reionisierung um z = 11 abspielte”. Wenn das stimmt, dann kann man zwei interessante Folgerungen daraus ziehen :

    a) Die kosmische Hintergrundstrahlung wird doch durch die Reionisierung beeinflusst, denn sonst könnte man ihren Zeitpunkt nicht aus WMAP ableiten.

    b) Die Reionisierung hat doch nichts mit den Population III Sternen zu tun, denn die gab es da schon lange nicht mehr.

  1077. #1094 Alderamin
    25. Oktober 2018

    @Peter Paul

    Ich denke, damit hast du im Prinzip schon recht, aber ein wenig möchte ich doch noch den Griffel spitzen, das kennst du ja schon von mir.

    Wer nicht fragt bleibt dumm, und wenn ich mich bemühe, solche Fragen zu beantworten, komme ich auch ins Grübeln, schlage nach, und lerne…

    Ich denke, damit hast du im Prinzip schon recht,

    Wikipedia sieht das auch so:

    The second phase change occurred once objects started to condense in the early universe that were energetic enough to re-ionize neutral hydrogen. As these objects formed and radiated energy, the universe reverted from being neutral, to once again being an ionized plasma. This occurred between 150 million and one billion years after the Big Bang (at a redshift 6 < z < 20) At that time, however, matter had been diffused by the expansion of the universe, and the scattering interactions of photons and electrons were much less frequent than before electron-proton recombination. Thus, the universe was full of low density ionized hydrogen and remained transparent, as is the case today.

    Ich bin mir jetzt nicht sicher, ob das schon die kosmische Reionisation ist, aber in dem von mir zitierten Buch werden diese ersten Sterne dafür verantwortlich gemacht, nämlich die Population III Sterne

    Siehe unten.

    a) Die kosmische Hintergrundstrahlung wird doch durch die Reionisierung beeinflusst, denn sonst könnte man ihren Zeitpunkt nicht aus WMAP ableiten.

    Die Hintergrundstrahlung des Mikrowellenhintergrunds ist bei z=1100, die Strahlung nach der Reionisation bei z=11-20, das wäre eher im Infraroten (das re-ionisierte Gas, war glaube ich, deutlich heißer als das des CMB, d.h. die maximale Wellenlänge läge irgendwo im UV und wird durch z in den langen IR-Bereich gezogen, aber nicht bis zum Mikrowellenbereich).

    Ich habe ein Paper gefunden, leider zu komplex, als dass ich da auf Anhieb durchsteige, aber in den Conclusions steht jedenfalls, dass es eine infrarote Hintergrundstrahlung gibt und dass das interstellare Gas zwischen z=6 und 30 eine optische Tiefe von τ=0,09 haben müsse, was auch von WMAP gemessen worden sei. Wobei τ so definiert ist, dass τ=1 eine Transmission von 1/e = 37% bedeutet (Wikipedia “Optische Tiefe”). Für kleine τ ist die Transmission ungefähr 1-τ (burro.astr.cwru.edu/Academics/Astr221/StarPhys/opticaldepthprimer.html), also für τ=0,09 wäre das 0,91 oder 91%. Optisch dicht ist das nicht.

    Die Mikrowellenstrahlung des CMB leuchtet da hindurch und drunter weg, hauptsächlich bei längeren Wellenlängen.

    b) Die Reionisierung hat doch nichts mit den Population III Sternen zu tun, denn die gab es da schon lange nicht mehr.

    Doch, schon, die allerersten Sterne waren riesig, kurzlebig, heiß und explosiv, und genau denen ist zuzutrauen, das Gas ionisiert zu haben. Die Re-Ionisation hat mit der Entstehung der ersten Galaxien und Sterne begonnen, die nur Wasserstoff und Helium als Grundstoff hatten, und das führt zu großen Sternen, weil diese reinen Gase Strahlung nicht so effektiv absorbieren wie das heute mit Staub und Metallen angereicherte Gas, das die Sterne nun besser wegblasen können, wenn sie gezündet haben. Das bremst heute das Sternwachstum; damals war das anders.

    Im verlinkten Paper steht aber auch, dass die Population-II-Sterne, insgesamt einen größeren Anteil an der Ionisation hatten, weil sie viel länger lebten. Es ist also zwischen Pop III und Pop II kein entweder-oder, sondern ein sowohl-als-auch.

  1078. #1095 Peter Paul
    26. Oktober 2018

    @Alderamin

    Doch, schon, die allerersten Sterne waren riesig, kurzlebig, heiß und explosiv, und genau denen ist zuzutrauen, das Gas ionisiert zu haben.

    Ich zweifle aus mehreren Gründen :

    a) Die Lyman Alpha Linie liegt schon relativ weit im UV. Dazu passt die Erklärung der Veränderung der 21cm-Strahlung durch den ” Wouthuysen–Field mechanism” in https://the-educational-blog.quora.com/The-EDGES-Experiment-This-May-Deserve-Two-Nobel-Prizes, die ohne Ionisation auskommt.

    b) Das Originalpapier (https://www.nature.com/articles/nature25792.epdf?referrer_access_token=mke8Nct9eF8FcSmJgQ6wbNRgN0jAjWel9jnR3ZoTv0OPTMBuYbvtRoHeOiTS9UYdiDlmksSuzKW7rnZSsmFRcTnqcHkOmkQRAw2xPwRH-PChYr_RuvHJ0306XBwi4vdIxk_upwHH5_SHAb85fdvIpeKX13caw9ja4h9xIsIi7GM–IQaAhpNFuBN1XftzSsdxljtRVZ5df4ndEIFBaVYzv0-eqZ_z-LgqfZd2g0P_rMJiOHHHCqi9Y25Cv3BO8Dy&tracking_referrer=derstandard.at) spicht nicht von Ionisation sondern davon, dass das UV-Licht eine Änderung des “excitation state of its 21-centimetre hyperfine” herbeiführt.” (siehe a)). Wenn aber gar kein Elektron mehr im H-Atom wäre, wäre eine solche Sprechweise doch kaum sinnvoll.

    c) Aber wieso nennt sich das EDGES-Experiment, das sich genau um das Auffinden der ersten Sterne dreht, dann so : “Experiment to Detect the Global Epoch of Reionization Signature” ?

    d) Das verschwinden des Signals in der oben erwähnten Fig.2 wird ja mit der Erwärmung des umgebenden Gases begründet. Dafür brauchen die Pop. III Sterne wohl ca. 150 Mio. Jahre. In dieser Zeit spucken sie aber schon massenhaft “Metall” aus, denn ihre individuelle Lebensdauer ist ja sehr kurz (ca. 3 bis 10 Mio Jahre).

    e) Die Epoche der Reionisation wird bei ca. z=10 angenommen (https://www.lofar.org/astronomy/eor-ksp/epoch-reionization), d.h. zwischen dem Beginn der ersten Sterne und dieser Periode liegen ca. 4 Mrd. Jahre. Aber zu dieser Zeit gabe es ganz bestimmt keine Pop. III Sterne mehr, denn die hatten fast 4 Mrd. Jahre Zeit, den Kosmos mit “Metallen” anzureichern.

    f) Es bleiben also nur Pop.II-Sterne als Verursacher der Reionisation übrig. Und wenn das so ist, leben wir dann vielleicht heute immer noch im Zeitalter der “Reionisierung”?

  1079. #1096 Alderamin
    26. Oktober 2018

    @Peter Paul

    Aber wieso nennt sich das EDGES-Experiment, das sich genau um das Auffinden der ersten Sterne dreht, dann so : “Experiment to Detect the Global Epoch of Reionization Signature” ?

    Na, weil EDGES doch den Anfang und das Ende des 21-cm-Signals gesehen hat. In dem Bild aus dem von Dir verlinkten Quora-Artikel sieht man doch, wie bei z=21 die Kurve sinkt, da wurden die ersten Elektronen von Sternenlicht angeregt, 21-cm-Licht auszusenden, und bei z=16 geht das Signal wieder zurück, weil das Gas ionisiert wurde. 21 cm entsteht ja beim Umklappen des Elektronenspins zwischen den Zuständen “parallel zum Kernspin” und “antiparallel zum Kernspin”. Wenn die Elektronen nicht mehr gebunden sind, dann gibt’s den Übergang nicht mehr. Deswegen ist das Verschwinden des Signals zwischen z=16 und z=14 ein Nachweis für die Re-Ionisation.

    Die Epoche der Reionisation wird bei ca. z=10 angenommen

    Wird oder wurde angenommen! Es gab vorher noch keine Messung, EDGES hat eine geliefert. Es gibt aber auch Zweifel an dem Ergebnis, wenn ich mich recht entsinne. Bevor alle sagen, ja, das ist es, die Reionisation fand bei z=15 statt, wird man noch unabhängige Beobachtungen fordern, vielleicht mit JWST.

    d.h. zwischen dem Beginn der ersten Sterne und dieser Periode liegen ca. 4 Mrd. Jahre

    Ned Wrights Cosmology Calculator verortet z=10 bei einem Weltalter von 478 Millionen Jahren, und z=15 bei 272 Millionen (jedenfalls mit den vorgegebenen Parametern; einfach z=10 eingeben und “Flat” drücken, dann rechts bei “age at redshift” nachschauen). Da gab es mit Sicherheit noch Population-III-Sterne.

    Und wenn das so ist, leben wir dann vielleicht heute immer noch im Zeitalter der “Reionisierung”?

    Die Ionisation wird jedenfalls heute noch aufrecht erhalten.

  1080. #1097 Peter Paul
    26. Oktober 2018

    @Alderamin

    und bei z=16 geht das Signal wieder zurück, weil das Gas ionisiert wurde.

    Ich glaube, zumindest nach dem Quora-Papier liegst du hier ganz weit daneben. Das hat alles gar nichts mit Ionisation zu tun, sondern es ist ein Temperatureffekt.

    Kurz gesagt (Quora macht es ausführlicher und besser): Im thermischen Gleichgewicht zwischen Strahlung und Materie nimmt die Wasserstoff-Wolke so viel 21cm Strahlung auf, wie sie auch wieder abgibt.
    Ist der Wasserstoff kälter als die Strahlung, dann absorbiert er mehr 21cm Strahlung als er emittiert.
    Das ist bei z=20 so. Bei z=14 ist er, dank der ersten Sterne wieder so warm wie die Strahlung. Deshalb gleicht sich das hier wieder aus.
    Eben weil es ein Temperatur-Effekt ist, der aber doppelt so stark ausfiel, wie man vermutet hatte, kam die “Dunkle Materie” in die Argumentation mit herein. Sie scheint das Gas weiter gekühlt zu haben, so zumindest die erste Erklärung.

  1081. #1098 UMa
    26. Oktober 2018

    Hallo Peter Paul, Alderamin,
    hier sind die letzten Planck-Resultate, auch zur Reionisierung:
    https://arxiv.org/abs/1807.06209

  1082. #1099 Alderamin
    26. Oktober 2018

    @Peter Paul

    Du hast recht, ich hatte gelesen, dass das Profil der Linie von UV Licht beeinflusst wird und das mit der Reionisation (die ja auch durch UV erfolgt) gleichgesetzt. Es ist eine Absorption, die entsteht, weil das Gas aufgrund mit der Expansion weniger werdender Kollisionen die Spin-Temperatur von der Gas-Temperatur entkoppelt, dadurch wird die Spin-Temperatur kühler und absorbiert Strahlung der CMB von weiter weg.

    Dann kommen die Sterne mit ihrem UV-Licht hinzu und heben die Spin-Temperatur wieder an, die Linie verschwindet. Die Intensität des UV-Lichts ist da aber noch zu gering, um die Ionisation zu bewirken.

    Danach steigt die Spin-Temperatur allmählich über die Gastemperatur, was zu einer Emissionslinie führt. Und die lässt dann wieder nach, wenn das Gas ionisiert wird. Das Profil der 21-cm-Linie über die Rotverschiebung z (und die Wellenlänge) ist hier gezeigt und erklärt. Die Reionisation findet danach zwischen z=15 und z=7 statt. Die Arbeit zu dem Bild (hinter Paywall) stammt schon von 2010.

  1083. #1100 Alderamin
    26. Oktober 2018

    @Peter Paul

    Aber wieso nennt sich das EDGES-Experiment, das sich genau um das Auffinden der ersten Sterne dreht, dann so : “Experiment to Detect the Global Epoch of Reionization Signature” ?

    Wenn mit EDGES das gesamte Profil der 21-cm-Linie wie im oben verlinkten Bild gemessen werden soll, dann passt es wieder. Nur war das Signal, das im Frühjahr veröffentlicht worden war, erst der Anfang dieses Profils, und die Veröffentlichung fokussierte sich auf die unerwartete Tiefe der Linie, die mit DM begründet wurde.

  1084. #1101 Peter Paul
    27. Oktober 2018

    @Alderamin
    Vielen Dank! Ich glaube, ich habe wieder einiges dazu gelernt.

  1085. #1102 Eufura
    28. Oktober 2018

    Hallo,
    Ich lese mich zurzeit durch die Informationen zu den Planetenringen und ihrer Entstehung.
    Nun frage ich mich allerdings, nach dem ich hier auf das “was wäre wenn die Erde einen solchen Ring hätte” gestossen bin, was wären die Auswirkungen auf unseren Planeten wenn sie Voraussetzungen für einen Ring gegeben wäre und dieser bestehen bleiben würde. Hätte er Einfluss auf Ebe und Flut, auf die Gravitation und was wäre mit dem Schatten den er wirft? Gäbe es andere Auswirkungen?
    Besten dank für die Antwort

  1086. #1103 Peter L.
    29. Oktober 2018

    zu #1102:
    Die Masse eines Planetenrings ist meist (sehr) klein im Vergleich zu einem größeren Mond (wie z. B. unserem Erdmond). Die Auswirkungen eines Rings um die Erde auf die Gravitation am Erdboden und auf Ebbe und Flut wären daher sehr gering, so dass man sie nicht bemerken würde.

    Die Erdachse ist gegenüber der Ebene der Erdumlaufbahn um 23,44° geneigt. Ein Planetenring liegt in der Regel in der Ebene des Äquators. Die Tropen und bei einem breiteren Ring auch die Subtropen u. evtl. auch die gemäßigten Zonen würden daher etwas weniger Sonneneinstrahlung erhalten. Wenn der Ring relativ breit und dicht ist, hätte das auch Auswirkungen auf das Klima, in den schattierten Regionen wäre es etwas kühler.

    Ein dichterer, breiterer Ring würde sich tagsüber durch einen (leichten) Schattenwurf bemerkbar machen. Wenn der Ring nicht sehr dicht ist, würde man evtl. eine leichte Abschwächung des Sonnenlichts (als wären dünne Schleierwolken vor der Sonne) bemerken. Diese Abschwächung kann aber bei einem Ring mit geringer Dichte so klein sein, dass man sie mit bloßem Auge nicht bemerkt.

    Nachts würde der Ring von der Sonne angestrahlt werden (jedenfalls der Teil, der sich nicht im Kernschatten der Erde befindet). Je höher die Albedo (das Rückstrahlvermögen) des Materials des Ringes und je dichter der Ring ist, umso heller würde man den Ring am Nachthimmel sehen.

  1087. #1104 Peter L.
    29. Oktober 2018

    Können sich neue Sterne rings um Weiße Zwerge oder Neutronensterne bilden?

    Damit meine ich, dass es doch theoretisch passieren könnte, dass ein Weißer Zwerg oder ein Neutronenstern in eine Gaswolke wandert, welche rings um diesen Sternenrest kollabiert, so dass sich um den alten Kern ein neuer Protostern bildet und schließlich einen neuer Stern entsteht.

    Wurde so etwas schon einmal beobachtet?

    Wie würde sich so etwas bemerkbar machen? Ein so entstandener Stern hätte ja eine etwas andere Zusammensetzung als ein „normal“ entstandener Stern, was man evtl. auch im Spektrum sehen könnte.

  1088. #1105 Florian Freistetter
    29. Oktober 2018

    @Peter: “Können sich neue Sterne rings um Weiße Zwerge oder Neutronensterne bilden?”

    Sterne eher nicht. Aber Planeten um Neutronensterne gibt es: https://scienceblogs.de/astrodicticum-simplex/2013/04/01/die-wunderbare-welt-der-exoplaneten-iii-tote-sterne-mit-planeten/?all=1

  1089. #1106 Andreas Palm
    Provinz
    1. November 2018

    Habe letztens eine Bericht gesehen da wurde erklärt, dass sich das bisher nicht nachgewiesene Graviton Berechnungen zu Folge exakt so verhalten müsste wie zwei Gluonen. Also das Gravitation eigentlich nichts anderes ist als eine Unterform der starken Kernkraft und deshalb bisher auch kein “eigenes” Trägerteilchen gefunden wurde. Ist etwas drann an dieser Theorie oder ist es nur TV-Unterhaltung für Laien wie mich? Die Sendung hieß Mysterien des Universums-Ist Schwerkraft nur ein Schwindel (oder so ähnlich).

  1090. #1107 Karl-Heinz
    1. November 2018

    @Andreas Palm

    Schwindel ist das falsche Wort. 😉

    Morgan Freeman: Mysterien des Weltalls – Ist Schwerkraft eine Täuschung?

    Darin kommt der Physikers Erik Verlinde vor.
    Dann geht es sicher um https://de.m.wikipedia.org/wiki/Entropische_Gravitation

  1091. #1108 Peter L.
    1. November 2018

    Zitat #1106: „das Gravitation eigentlich nichts anderes ist als eine Unterform der starken Kernkraft“

    Das ist sehr unwahrscheinlich. Denn die Gravitation hat eine sehr weite Reichweite (über Millionen/Milliarden von Lichtjahren), ist aber sehr schwach.
    Die starke Kernkraft (starke Wechselwirkung) hat dagegen einen extrem kurze Reichweite von nur etwa 1 Femtometer (ein billiardstel Meter = 0,000000000000001 m) und ist um viele Größenordnungen stärker als die Gravitation.

    Die einzige Wechselwirkung, die ebenfalls eine sehr weite Reichweite hat, ist die elektromagnetische Wechselwirkung.

    Die Higgs-Bosonen könnten als Trägerteilchen der Gravitation bezeichnet werden.
    Zitat https://erlangen.physicsmasterclasses.org/sm_et/sm_et_07.html:
    „das Higgs-Feld, das im ganzen Raum herrscht. Teilchen, die sich in diesem Feld aufhalten, erhalten Masse.
    Wie bei anderen Feldern, wird auch die Wirkung des Higgs-Feldes durch ein Austauschboson, das Higgs-Boson, vermittelt. Das Higgs-Boson koppelt an Teilchen, wobei die Stärke seiner Kopplung proportional zur Masse des Teilchens ist. Durch seine Kopplung an alle Teilchen verschafft das Higgs-Boson ihnen Masse.“

  1092. #1109 Sven
    1. November 2018

    @Peter L.
    Der erste Teil stimmt. Der zweite Teil (das mit dem Higgs-Boson als Austauschteilchen der Gravitation) ist falsch.

  1093. #1110 Karl-Heinz
    1. November 2018

    @Sven

    Den gleichen Gedanken wegen dem Higgs-Boson hatte ich gerade auch. 😉

    Als Graviton bezeichnet man das hypothetische Eichboson einer Quantentheorie der Gravitation. Dieser Annahme zufolge ist es der Träger der Gravitationskraft.

  1094. #1111 Peter L.
    1. November 2018

    Zitat #1109: „Der zweite Teil (das mit dem Higgs-Boson als Austauschteilchen der Gravitation) ist falsch.“

    Okay, vielen Dank für die Richtigstellung. Ich bin kein Physiker.
    In vielen populärwissenschaftlichen Darstellungen gibt es leider diese falschen Aussagen über die Higgs-Bosonen.

    Was mir aber bei Austausch noch einfällt, sind die Gravitationswellen. Das sind natürlich keine Teilchen, sondern wenn ich es richtig verstanden habe, ist es die Raumzeit selbst, die sich wellenförmig verformt. Diese Wellen breiten sich maximal mit Lichtgeschwindigkeit aus.

  1095. #1112 Sven
    1. November 2018

    @Peter L.
    Das Higgs-Feld ist verantwortlich für die Ruhemasse von Elementarteilchen (aber die Masse von z.B. Protonen und Neutronen kommt nur zum kleineren Teil vom Higgs). Aber mit Gravitation hat es nichts zu tun. Das sieht man auch daran, dass das Austauschteilchen der Gravitation masselos sein muss (so wie das Photon), da sonst die Reichweite der Gravitation nicht so groß wäre. Und das Higgs hat ja eine sehr große Masse. Und Gravitation koppelt auch an sämtliche Formen von Energie, nicht nur an Ruhemasse.

    Ja, in einer Quantentheorie der Gravitation würden Gravitationswellen aus Gravitonen bestehen. Und Gravitationswellen breiten sich stets mit Lichtgeschwindigkeit aus.

  1096. #1113 Captain E.
    2. November 2018

    Tja, mal kurz überlegt: Die Austauschteilchen (Bosonen) von drei der vier Kernkräften sind ruhemasselos. Die Schwache Kernkraft mit ihren W-und Z-Bosonen weicht davon ab. Dadurch können sich Gravitonen (hypothetisch!), Photonen und Gluonen mit Lichtgeschwindigkeit (und nur damit) bewegen, womit die Reichweite theoretisch unendlich ist. Bei der Starken Kernkraft ist das aber trotzdem nicht der Fall. Andererseits sind die elektromagnetische und die schwache Kraft bei hohen Energien nicht mehr voneinander zu unterscheiden, so dass die Physik die Theorie der elektroschwachen Kraft hat entwickeln können. Dass die Photonen mit einer Ruhemasse von 0 und die extrem schweren W-und Z-Bosonen unter gewissen Bedingungen dasselbe sein können, hört sich auf den ersten oder selbst den zweiten Blick ziemlich widersinnig an, scheint aber genau so zu sein.

    So, und was ist nun ein Boson? Das ist ein Teilchen mit ganzzahligem Spin. Fermionen wie Quarks, Protonen, Elektronen oder Neutronen haben keinen ganzzahligen Spin. Das Higgs-Boson hat Spin 0, Photonen und Gluonen haben Spin 1 und das Graviton müsste Spin 2 haben. Bosonen müssen allerdings keine Elementarteilchen sein, denn Cooper-Paare, Mesonen oder Atomkerne mit einer geraden Nukleonenzahl (H-2, He-3, Li-6, B-10, N-14, Cl-36, Co-60) sind auch Bosonen.

    Also ist es zumindest denkbar, dass zwei Gluonen mit Spin 1 und Ruhemasse 0, die jeder für sich eine Grundkraft über extrem kurze Distanz vermitteln können, in Interaktion miteinander ein zusammengesetztes Teilchen ergeben könnten mit immer noch Ruhemasse 0 (bliebe nur die Frage nach der Bindungsenergie und wieso diese die Ruhemasse nicht erhöhte?), Spin 2 und der Fähigkeit, eine weitere Grundkraft durch das gesamte Universum zu vermitteln. Falls dem so wäre, könnte man also irgendwann einmal die Theorie der gravitativ-schweren Kraft entwickeln. Oder vielleicht, weil es lustiger klänge, die “Kraft der schweren Schwerkraft”.

    Muss ja alles nicht so sein, aber dieses Universum ist ja so schon dermaßen verrückt, dass diese Idee ohne weiteres hinein passen könnte.

  1097. #1114 Dieter
    Deisenhausen
    2. November 2018

    So ein paar Attosekunden nach dem Urknall muss doch alles, was das Universum heute ausmacht in einem winzigen Raum komprimiert gewesen sein. Da muss doch die Raumkrümmung von dieser Winzigkeit irrwitzig gewesen sein. Warum ist da nicht sofort bei erreichen der passenden Größe für den dazugehörigen Schwarzschildradius ein Schwarzes Loch aus unserem Universum geworden?

  1098. #1115 Karl-Heinz
    4. November 2018

    @Dieter

    Antwort zu #1114

    Gesamtmasse des sichtbaren Universums: ~ 10^54 kg
    Schwarzschildradius für 10^54 kg: 157 Milliarden Lichtjahre. Nein, wir leben nicht in einem schwarzen Loch.

    Du sprichst von ein paar Attosekunden nach dem Urknall. Die GUT-Ära ist da schon längst vorbei und wir können uns der Friedmann-Gleichungen (Friedmann-Lemaître-Gleichung) zuwenden. Und die beschreibt die zeitliche Entwicklung des Universums. Es wird davon ausgegangen, dass das Universum von jedem Standpunkt aus isotrop und homogen aussieht (kopernikanisches Prinzip). Das kosmologische Prinzip macht nun die weitere Annahme erforderlich, dass die räumliche Krümmung des Raumes unabhängig von der Position im Raum sein soll. Das ist bei einem Schwarzen Loch nicht der Fall. 😉

  1099. #1116 Alderamin
    4. November 2018

    @Karl-Heinz, Dieter

    Vor allem gilt die Schwarzschildlösung in einem expandierenden Universum nicht mehr, sondern sie ist eine statische Lösung für eine ruhende Masse in einem leeren Minkowski-Raum.

    https://math.ucr.edu/home/baez/physics/Relativity/BlackHoles/universe.html

    https://www.forbes.com/sites/civicnation/2018/10/30/putting-the-party-in-participation-how-votetogether-parties-are-driving-youth-voter-turnout/

  1100. #1117 Christian
    Wien
    4. November 2018

    Hallo an alle!

    > Nein, wir leben nicht in einem schwarzen Loch. > Das kosmologische Prinzip macht nun die weitere Annahme erforderlich, dass die räumliche Krümmung des Raumes unabhängig von der Position im Raum sein soll. > Das ist bei einem Schwarzen Loch nicht der Fall.

    Wenn das Holographische Prinzip gilt, dann bildet unser Universum z. B. die Oberfläche eines Schwarzen Lochs und da ist die Krümmung überall gleich (im nichtrotierenden Fall) oder annähernd gleich (bei Rotation des Schwarzen Lochs)!

    > Vor allem gilt die Schwarzschildlösung in einem expandierenden Universum nicht mehr, sondern sie ist eine statische Lösung für eine ruhende Masse in einem leeren Minkowski-Raum.

    Das ist ein mathematisches Problem. Es gibt ja auch die Kerr-Lösung für rotierende Schwarze Löcher. Und wenn Schwarze Löcher einen Massenzuwachs von außen erfahren, dann expandiert auch deren Ereignishorizont.

  1101. #1118 Alderamin
    4. November 2018

    @Christian

    Wenn das Holographische Prinzip gilt, dann bildet unser Universum z. B. die Oberfläche eines Schwarzen Lochs

    Ich glaube, da schmeißt Du zwei Dinge durcheinander: nach dem holographischen Prinzip soll der Ereignishorizont eines schwarzen Lochs die Information über in das Schwarze Loch hineingefallene 3D-Objekte eingeprägt in 2D enthalten. Und das holographische Universum soll eine hypothetische 2-dimensionale Struktur des Universums sein, die uns dreidimensional erscheint. Wobei dies aber nicht der Ereignishorizont eines Schwarzen Lochs zu sein braucht.

    In der Frage von Dieter ging es aber um etwas ganz anderes, nämlich warum der Raum, wenn er so dicht mit Energie/Masse gefüllt war, überhaupt expandieren konnte und nicht gleich zu einem Schwarzen Loch kollabierte. Nicht darum, ob das Universum ein schwarzes Loch ist. Ist es nicht, wenn überhaupt, dann ein Weißes Loch.

    Das ist ein mathematisches Problem.

    Ist es nicht, weil die Voraussetzungen für die Existenz eines Schwarzschildschen (oder Kerrschen) Ereignishorizonts gar nicht erfüllt sind und es daher auch keinen gibt (es gibt allerdings einen kosmologischen, aber das ist was ganz anderes).

    Und wenn Schwarze Löcher einen Massenzuwachs von außen erfahren, dann expandiert auch deren Ereignishorizont.

    Das Universum erfährt aber keinen Massenzuwachs, sondern seine Materiedichte nimmt mit der Expansion stetig ab.

  1102. #1119 Christian
    Wien
    5. November 2018

    > Ist es nicht, weil die Voraussetzungen für die Existenz eines Schwarzschildschen (oder Kerrschen) Ereignishorizonts gar nicht erfüllt sind

    Schwarze Löcher sind ein mathematisches Konstrukt der relativistischen Physik, es gibt aber auch noch andere mathematisch konsistente Modelle für solche Gebilde wie die des Grava- und Holosterns. Der experimentelle Nachweis, welches Konzept in der Natur verwirklicht ist, steht ja noch aus.

    > Das Universum erfährt aber keinen Massenzuwachs, sondern seine Materiedichte nimmt mit der Expansion stetig ab.

    Gegenbeispiel:

    Radius beobachtbares Universum: r
    r = (4,7*10^10)*(9,641*10^15) = 4,447*10^26 m

    mittlere Energiedichte des beobachtbaren Universums: ρ
    Baryonische Materie + Dunkle Materie + Dunkle Energie: ρ = 110*10^-28 kg/m³
    Baryonische Materie + Dunkle Materie: ρ = 30*10^-28 kg/m³
    Baryonische Materie: ρ = 5*10^-28 kg/m³

    Schwarzschild-Radius: rs
    rs = (2*m*G)/c^2 | m = ρ*V
    rs = [(2*ρ*G)/c^2]*V | Vkugel = (4/3)*π*rs^3
    rs = [(8*π*ρ*G)/(3*c^2)]*rs^3 | *[(3*c^2)/(8*π*ρ*G*rs)] | √
    rs = √[(3*c^2)/(8*π*ρ*G)]

    Baryonische Materie + Dunkle Materie + Dunkle Energie: rs = ? Dunkle Energie wirkt definitionsgemäß antigravitativ.
    Baryonische Materie + Dunkle Materie: rs = 2,316*10^26 m r

    Für die obigen Werte der Baryonischen Materie für ρ und rs ergibt sich bei Kugelform des Universums-Schwarzen-Lochs/-Holosterns folgende Masse: ms
    ms = 3,826*10^53 kg

    Und daraus das folgende Flächendichteäquivalent am Ereignishorizont: P
    P = ms/A | Akugel =4*π*rs^2
    P = ms/(4*π*rs^2)
    P = 0,095 kg/m²

    Infolge der Expansion des Universums wird die Dichte der Baryonischen Materie in Zukunft beispielsweise die Hälfte des heutigen Werts annehmen, also:
    ρ2 = 2,5*10^-28 kg/m³

    Daraus ergibt sich der folgende √2-fache Schwarzschild-Radius des gegenwärtigen Schwarzschild-Radius’:
    rs2 = 8,024*10^26 m

    Daraus ergibt sich die folgende √2-fache Masse der gegenwärtigen Masse:
    ms2 = 5,410*10^53 kg

    Und daraus die folgende (1/√2)-fache Flächendichte der gegenwärtigen Flächendichte:
    P2 = 0,067 kg/m²

  1103. #1120 Christian
    Wien
    5. November 2018

    1119 Beiträge sind für diese Seite einfach zu viel! Ein Teil meines obigen Beitrags wurde – ganz nach Art eines Schwarzen Lochs – geschluckt. Zum Glück habe ich diese Information noch am Ereignishorizont gefunden:

    Baryonische Materie + Dunkle Materie + Dunkle Energie: rs = ? Dunkle Energie wirkt definitionsgemäß antigravitativ.
    Baryonische Materie + Dunkle Materie: rs = 2,316*10^26 m [kleiner] r
    Baryonische Materie: rs = 5,674*10^26 m > r

  1104. #1121 Karl-Heinz
    5. November 2018

    @Christian

    Bekommt das Schwarze Loch (SL) Nahrung, so wird seine Masse größer aber die mittlere Dichte in Bezug auf den Schwarzschildradius wird kleiner. Man beachte, dass das SL kein abgeschlossenes System ist.

    Das Universum würde ich als abgeschlossen bezeichnen und damit gilt hier auch der Energieerhaltungssatz über die Zeit.

  1105. #1122 Alderamin
    5. November 2018

    @Christian

    Schwarzschild-Radius: rs

    Wie gesagt gilt die Formel für das expandierende Universum nicht, das hatte ich auch verlinkt. Da kann rauskommen, was will.

    1119 Beiträge sind für diese Seite einfach zu viel! Ein Teil meines obigen Beitrags wurde – ganz nach Art eines Schwarzen Lochs – geschluckt.

    Das liegt daran, dass das “<“-Zeichen HTML-Tags einleitet, und die werden nicht angezeigt. Bis zum nächsten “>” geht dann alles verloren. Zur Verwendung muss man &lt; und &gt; schreiben (und um das zu schreiben muss man &amp;lt; / &amp;gt; schreiben 😉 )

    Hier soll aber auch nicht diskutiert werden, sonst wird’s noch länger. Zum freien Diskutieren hat Florian die Verschwörungsgeplauderseite eingerichtet. Oder noch besser einen passenden Artikel suchen. Hier nur Frage – kurze Antwort oder idealerweise ein Artikel.

  1106. #1123 Christian
    Wien
    5. November 2018

    > Das Universum würde ich als abgeschlossen bezeichnen

    Warum? Gibt es irgendwelche experimentellen Hinweise, dass es nicht auch so sein kann, wie in #1119 und #1120 beschrieben? Abgeschlossen scheint das beobachtbare Universum zu sein, aber dass das, was beobachtbar ist, alles ist, ist so etwas von unwahrscheinlich, weil das – unter Korrektur von galaktischen und innermilchstraßigen Relativbewegungen – bedeuten würde, dass sich das Sonnensystem exakt im Mittelpunkt des Universums befindet. – Ein Ergebnis, das den Klerus des Mittelalters sehr gefreut hätte, aber dessen Wahrscheinlichkeit eben beliebig nahe bei Null liegt. Das Universum könnte also mit gleichen Wahrscheinlichkeiten abgeschlossen oder nicht abgeschloss sein. Wieso gehst du also offenbar mit Überzeugung von einer Abgeschlossenheit aus?

  1107. #1125 Christian
    Wien
    6. November 2018
  1108. #1126 Noonscoomo
    Berlin
    22. November 2018

    Vor kurzem las ich von einem Schwarzen Loch das sich sehr schnell dreht. So weit so gut. Wenn ein großer Stern am Ende seiner Fusionsreaktion kollabiert sollte der Drehimpuls erhalten bleiben. D.h. wenn das Schwarze Loch immer kleiner wird, sollte es immer schneller rotieren? Dann kann es doch unmöglich unendlich klein werden, da es dann ja auch unendlich schnell rotieren müsste, oder?

  1109. #1127 Peter L.
    22. November 2018

    Zitat #1126: „Wenn ein großer Stern am Ende seiner Fusionsreaktion kollabiert sollte der Drehimpuls erhalten bleiben.“

    Am Ende der Fusionsreaktion gibt es ja eine Supernova, bei der die äußeren Schichten, die sich ja auch drehen, ins All geschleudert werden.

    Daher würde ich annehmen, dass das Material, welches bei der Supernova ins All geschleudert wird, einen Teil des Drehimpulses des ursprünglichen Sterns hat.

    Nur die innersten Schichten kollabieren, das heißt das Schwarze Loch müsste dann den Drehimpuls der innersten Schichten haben.

    Später kann ein Schwarzes Loch Drehimpuls hinzugewinnen, wenn Material von außen spiralförmig in das Schwarze Loch fällt.

  1110. #1128 Noonscoomo
    Berlin
    23. November 2018

    @Peter L
    Ja, schon richtig und soweit auch klar, aber wenn ein Objekt, egal welche Masse es hat, mit einem Drehimpuls ungleich null unendlich klein wird muss doch der Drehimpuls unendlich gross werden. Da ist es doch unerheblich, ob es vorher einen Teil des Drehimpulses los geworden ist…
    Das einzige was das rettet ist die Erkenntnis, dass sich alles auf der anderen Seite des Ereignishorizontes befindet unserer Beschreibung entzieht. Aber wenn wir kurz für einen Moment annehmen dass die Physik da drin erst mal noch immer gilt müsste es doch eine Grenze für die Rotation geben. Oder nicht? Da versteh ich die Mathe zu wenig. Wenn das Ding immer schneller rotiert aber immer kleiner wird, dann könnte es ja sein dass die zurück gelegte Strecke pro Zeit auf der Oberfläche immer noch kleiner als c bleibt

  1111. #1129 Alderamin
    23. November 2018

    @Noonscoomo

    Ja, schon richtig und soweit auch klar, aber wenn ein Objekt, egal welche Masse es hat, mit einem Drehimpuls ungleich null unendlich klein wird muss doch der Drehimpuls unendlich gross werden.

    Drehimpuls ist eine Erhaltungsgröße, der kann nur ausgetauscht werden, bleibt ansonsten aber konstant.

    Wa sich beim Komprimieren erhöht, ist die Rotationsgeschwindigkeit. Der Drehimpuls enthält auch die Masse eines Objekts, und dreht sich das Objekt mit annähernd Lichtgeschwindigkeit, dann erhöht sich die bewegte Masse und damit kann der Drehimpuls steigen, ohne dass die Rotationsgeschwindigkeit wesentlich zunimmt.

    Bei Schwarzen Löchern steckt der Drehimpuls aber in der rotierenden Raumzeit. Was innerhalb des Ereignishorizonts ist, ist von der Außenwelt abgeschnitten, das kann tun was es will, außen merkt man davon nichts. Fällt z.B. mehr Masse in ein Schwarzes Loch und erhöht diese beim Einfall aus einer rotierenden Akkretionsscheibe den Drehimpuls, dann scheint sie mit dem Ereignishorizont zu verschmelzen und dort einzufrieren, der Ereignishorizont wächst ein bisschen und dadurch erhöht sich auch der Drehimpuls. Wie Niels neulich woanders schrieb, ist die Annahme, dass die Raumzeit am Ereignishorizont nicht schneller als mit Lichtgeschwindigkeit rotieren kann. Man sieht die Materie nicht den Ereignishorizont überschreiten und mit der Ringsingularität im Inneren verschmelzen. Das gilt übrigens schon beim Kollaps des Schwarzen Lochs, deswegen muss der Drehimpuls außerhalb des Ereignishorizonts verbleiben.

    Hoffe, das stimmt soweit.

  1112. #1130 Mysterion1000
    Salzburg
    21. Dezember 2018

    Es sind ja einige Missionen in Planung, die Leben auf anderen Exoplaneten suchen werden, insofern stellt sich folgende Frage für mich.
    Gibt es eigentlich Namensvorschläge für den Planeten, wo man zum ersten mal Leben vorfindet?

  1113. #1131 Florian Freistetter
    21. Dezember 2018

    @Mysterion1000: Ich glaube das ist eine Frage, die sich erst stellt, wenn es denn mal so weit sein sollte. Bis jetzt haben nur ne Handvoll Exoplaneten offizielle Namen bekommen: https://scienceblogs.de/astrodicticum-simplex/2015/12/15/poltergeist-dagon-und-dimidium-die-ersten-exoplaneten-haben-offizielle-namen-bekommen-2/?all=1

  1114. #1132 Peter
    21. Januar 2019

    Warum gibt es erst innerhalb von „nur“ 6 Monaten eine totale Mondfinsternis (27. 7. 2018 und 21. 1.2019), und danach dauert es aber 9 Jahre, bis in Deutschland die nächste totale Mondfinsternis zu sehen ist?

    In der Zeitung stand heute, dass eine totale Mondfinsternis meist 1- bis 2-mal im Jahr auftritt. Weiter hinten im gleichen Artikel stand dann, dass erst 2028 wieder eine totale Mondfinsternis in Deutschland gut zu beobachten sein wird.
    Woran liegt das, dass die Abstände zwischen 2 totalen Mondfinsternissen so unterschiedlich lang sind?

  1115. #1133 Florian Freistetter
    21. Januar 2019

    @Peter: 1-2 mal pro Jahr verfinstert sich der Mond zumindeste teilweise. Eine TOTALE Mondfinsternis gibt es aber viel seltener.

  1116. #1134 HF(de)
    21. Januar 2019

    viel seltener

    Wenn mich meine Erinnerung nicht im Stich läßt: MoFi allgemein 2x pro Jahr (partielle und/oder totale). Totale ca. alle 4 Jahre. Und dann muss man auf der richtigen (Nacht-)Seite sein. 9 Jahre passt.

  1117. #1136 HF(de)
    21. Januar 2019

    PS: dass es jetzt 2 totale MoFis in einem halben Jahr waren, ist sozusagen Zufall: mal rauschte der Mond “nördlich” durch den Erdschatten, mal “südlich” – und DE war beide mal auch noch auf der nächtlichen (also richtigen) Seite.
    Der Mond kann auch ziemlich zentral durchrauschen, dann gibt es nur eine totale MoFi in dem Jahr.

  1118. #1137 HF(de)
    21. Januar 2019

    @PDP10: bedankt für den Link, da liege ich wohl falsch mit 1x im Jahr, geht sogar bis 4x in 2 Jahren. Auf die Nachtseite kommt es an…

  1119. #1138 Peter
    22. Januar 2019

    Danke an PDP10 für den Wikipedia-Link.

    Laut diesem Wikipedia-Artikel ist die nächste totale Mondfinsternis in 2 Jahren, nämlich am 26. Mai 2021, diese ist in Europa aber nicht zu sehen.
    Die folgende totale Mondfinsternis ist dann am 16. Mai 2022, die ist in Europa sichtbar.
    Am 8. Nov. 2022 folgt dann schon wieder eine totale Mondfinsternis, die nicht bei uns, sondern auf der anderen Erdseite sichtbar ist.
    Die folgende totale Mondfinsternis am 14. Mrz. 2025 ist in Westeuropa sichtbar, darauf folgt eine am 7. Sep. 2025 die in Europa sichtbar ist.
    Darauf folgt eine am 3. Mrz. 2026 die bei uns nicht sichtbar ist.
    Danach kommt eine am 31. Dez. 2028 die in Europa sichtbar ist. Danach folgen: 26. Jun. 2029 und 20. Dez. 2029 (beide in Europa sichtbar).
    Die nächste ist dann am 25. Apr. 2032 (hier nicht sichtbar).

    Also sind die Abstände zwischen 2 totalen Mondfinsternisse ungefähr jeweils:
    2 Jahre;
    1 Jahr;
    6 Monate,
    2 Jahre und 4 Monate;
    6 Monate;
    6 Monate;
    2 Jahre;
    6 Monate;
    6 Monate;
    6 Monate;
    2 Jahre und 4 Monate.

    Und etwa die Hälfte aller (totalen) Mondfinsternisse ist bei uns sichtbar, weil die Wahrscheinlichkeit, dass man auf der Nachtseite ist, bei 50 % liegt.

  1120. #1139 Peter
    22. Januar 2019

    Heute habe ich einen Vortrag an der Uni gehört, wo ein Teilchenphysiker gesagt hat, dass die schwereren Elemente im Periodensystem so etwa ab Silber bis Gold und schwerer (z. B. Blei, Uran) nicht durch Supernovae gebildet wurden, sondern durch den Zusammenstoß von Neutronensternen entstanden sind.

    In Lehrbüchern steht, dass schwerere Elemente als Eisen, auch Gold, Uran etc. in Kernkollaps-Supernovae (massereiche Sterne ab etwa 8 bis 10 Sonnenmassen) gebildet wurden.

    Wer hat denn jetzt Recht: Der Physiker, die Lehrbücher oder werden schwerere Elemente durch beide Prozesse gebildet?

    In Supernova-Überresten (vom Stern ins All geschleuderte Plasma-/Gaswolken) müsste man doch die Absorptionslinien aller vorhandenen Elemente nachweisen können.

    Neutronensterne bestehen doch nur aus Neutronen, oder?
    Atomkerne enthalten ja außer Neutronen auch Protonen, woher kommen die Protonen?
    Entstehen diese durch den Beta-Minus-Zerfall, wobei je 1 Neutron zu einem ein Proton, einem Elektron und einem Elektron-Antineutrino zerfällt?
    Falls ja, wann geschieht dass: Schon im normalen Neutronenstern vor dem Zusammenstoß oder erst während oder nach dem Zusammenstoß?

    Wenn 2 Neutronensterne zusammenstoßen, müssten diese nicht zu einem Schwarzen Loch (oder evtl. zu einem massereicheren Neutronenstern) verschmelzen?

    Entstehen dabei auch Bruchstücke, die nach außen geschleudert werden? Falls ja, wie groß sind die Bruchstücke, die beim Zusammenstoß von 2 Neutronensternen entstehen?
    Um Atomkerne schwerer Elemente zu bilden, müssten ja auch sehr kleine Stücke entstehen, die je ca. 110 – 240 Kernteilchen (Neutronen u. evtl. Protonen) enthalten.

  1121. #1140 Niels
    23. Januar 2019

    Allgemein werden schwere Elemente durch den sogenannten r-Prozess (rapid neutron-capture process) erzeugt.
    https://en.wikipedia.org/wiki/R-process

    Früher dachte man, dass dieser Prozess hauptsächlich während bestimmter Typen von Supernovae abläuft.
    Das Problem war aber, dass es zu wenige solcher Supernovae gab. Man wusste nicht, wo die “überzähligen” schweren Elemente herkommen könnten.

    Später schlug man vor, dass der r-Prozess auch in Neutronensternen stattfinden kann, die sich in einem Doppelsternsystem mit einem schwarzen Loch befinden und in dieses stürzen.

    Mittlerweile geht man davon aus, dass die Mehrheit der r-Prozesse bei der Verschmelzung von zwei Neutronensternen entsteht. Kommt aber immer auch auf das genaue Periodensystem-Element an.
    Nachgewiesen wurde das Auftreten des r-Prozesses bei Neutronenstern-Neutronenstern-Kollisionen sogar erst 2017.

    Welche Anteile welchen schweren Elements aus welcher dieser Quellen stammen, weiß momentan niemand so ganz genau. Verschiedene Simulationen verschiedener Forschungsgruppen liefern verschiedene Ergebnisse.
    Im verlinkten Wiki-Artikel ist eine aktuelle Übersicht eingebettet:
    https://en.wikipedia.org/wiki/File:Nucleosynthesis_periodic_table.svg
    (Supernovae gehören zu “Exploding massive stars”.)

    Zu deinen Detailfragen über den genauen Ablauf muss ich leider auf diesen Wiki-Artikel verweisen oder du kannst einfach mal nach “r-process” googeln.
    Da kenne ich mich auch nicht so richtig gut aus.

  1122. #1141 Peter
    23. Januar 2019

    Niels, vielen Dank für den Link.
    Unter https://en.wikipedia.org/wiki/R-process#Astrophysical_sites
    steht: “the very low abundance of r-process nuclei in the interstellar gas … either that only a small fraction of supernovae eject r-process nuclei to the interstellar medium, or that each supernova ejects only a very small amount of r-process material. …
    discovered in 2017 … collapse of the two neutron stars into a black hole, but with significant spun off mass of highly neutronized matter, … rich in actinides (such as uranium, thorium, californium etc).”

    Also die Supernovae allein können offenbar die Häufigkeiten schwerer Elemente nicht erklären. Entweder stoßen nur wenige Supernovae (sehr) schwere Elemente aus, oder jede Supernova stößt nur sehr wenige davon aus.

    2017 hat man in den Überresten einer Kollision 2er Neutronensterne sehr schwere Elemente gefunden.
    Da es erst 2017 entdeckt wurde, steht es noch nicht in den Lehrbüchern.

    Das Periodensystem unter
    https://en.wikipedia.org/wiki/File:Nucleosynthesis_periodic_table.svg
    zeigt nur, wo die Elemente in unserem Sonnensystem herkommen.
    Fürs ganze Weltall müsste man das Periodensystem modifizieren, z. B. kommt Technetium (Tc) in Roten Riesen vor und Californium (Cf) wurde nach Kollision 2er Neutronensterne gefunden.

    Dass viele schwere Elemente weniger in Supernovae und mehr in der Kollision von Neutronensternen entstehen, würde auch erklären, warum z. B. Silber, Gold, Platin, Osmium, Iridium u. Uran auf der Erde so selten vorkommen. Denn Supernovae kommen ja deutlich häufiger vor als Kollisionen von Neutronensternen.

  1123. #1142 Niels
    23. Januar 2019

    @Peter

    Denn Supernovae kommen ja deutlich häufiger vor als Kollisionen von Neutronensternen.

    Ist das so?
    r-Prozesse gibt es wohl nur in Supernovae vom Typ Ib, Ic und II.
    Keine Ahnung, wie groß deren Anteil an allen Supernovae ist.

    Wie häufig Kollisionen von Neutronensternen sind wissen wir schlicht nicht, oder?
    Dass der r-Prozess bei der Kollision von Neutronensternen erstmals 2017 entdeckt wurde, ist ja kein Zufall. Neutronensternkollisionen kann man optisch praktisch nicht aufspüren.
    Das geht erst, seit man Gravitationswellendetektoren hat. Wenn man die Gravitationswellen der Kollision gemessen hat, weiß man dann ganz genau, auf welche Stelle man die optischen Teleskope ausrichten muss.
    Funktionierende Gravitationswellendetektoren haben wir aber erst seit 2015 oder so.

  1124. #1143 Frank
    24. Januar 2019

    Vor kurzem war der 21. Januar. Dieser Tag ist einen Monat entfernt vom 21. Dezember, der Wintersonnenwende, dem kürzesten Tag. Gehen wir noch einen Monat zurück, so sind wir beim 21. November. Man kann also sagen, dass der 21. November gewissermaßen der “Spiegeltag” vom 21. Januar ist. Wenn man sich entsprechende Tabellen ansieht, so sieht man tatsächlich, dass die Tageslänge des 21. November ungefähr der Tageslänge des 21. Januar entspricht (nämlich ca. 8,5 Stunden). Schaut man sich jedoch die Sonnenauf- und untergänge dieser beiden Tage an, so sind beide Zeitpunkte am 21. November ca. eine halbe Stunde früher, als am 21. Januar. Dies ist mehr, als man sich durch “kalendarische Varianzen” (Schaltjahre, unterschiedlich lange Monate etc.) erklären kann. Wir hatten an allen drei Daten Winterzeit. Auch bleibt diese “Verschiebung” über die Jahre konstant. Frage: Woher kommt diese Verschiebung?

  1125. #1144 UMa
    24. Januar 2019

    @Frank:
    Diese Verschiebung kommt durch die Überlagerung zweier Effekte. Zum einen der Exzentrizität der Erdbahn. Die Winkelgeschwindigkeit ist bei Sonnennähe (Perihel am 3. Januar) größer als bei Sonnenferne. Außerdem ist die Erdachse gegen die Erdbahn geneigt.
    Eine Ausführlich Erklärung ist hier auf Wikipedia zu finden:
    https://de.wikipedia.org/wiki/Zeitgleichung
    https://de.wikipedia.org/wiki/Analemma

  1126. #1145 Peter
    24. Januar 2019

    Zitat Frank: „Woher kommt diese Verschiebung?

    Diese Verschiebung kommt daher, dass die Erdbahn eine Ellipse ist und die kleinste Sonnenentfernung (Perihel) u. die größte Sonnenentfernung (Aphel) beide nicht den gleichen zeitlichen Abstand zur Wintersonnenwende oder zur Sommersonnenwende haben:
    Die Erde erreicht den sonnennächsten Punkt (Perihel) am 2. bis 5. Januar. Rund um diesen Punkt bewegt sie sich schneller auf ihrer Bahn.
    Den sonnenfernsten Punkt (Aphel) erreicht die Erde am 3. bis 6. Juli. Rund um diesen Punkt bewegt sie sich langsamer auf ihrer Bahn.

  1127. #1146 Alderamin
    24. Januar 2019

    @Frank

    Dazu habe ich zufällig vor kurzem einen Artikel geschrieben.

  1128. #1147 Peter
    25. Januar 2019

    In einem Vortrag hat ein Physiker heute erzählt, dass sich das Weltall in der Inflation kurz nach dem Urknall mit Überlichtgeschwindigkeit ausgedehnt hätte.
    Als dazu eine Frage kam, war er sich selber aber nicht ganz sicher, ob es nicht doch „bloß“ Lichtgeschwindigkeit war, er sagte auch, dass er kein Kosmologe ist.

    Ist eine kosmische Inflation mit Überlichtgeschwindigkeit physikalisch möglich?
    Es heißt ja immer, nichts könne schneller sein als das Licht. Gilt dies auch für die Raumzeit an sich?

    Falls das mit der Überlichtgeschwindigkeit nicht stimmt, mit welcher Geschwindigkeit hat sich das Universum während der Inflation ausgedehnt?
    Etwa mit Lichtgeschwindigkeit im Vakuum oder mit (deutlich) weniger als der Lichtgeschwindigkeit im Vakuum?

    Und wie genau kann man die Geschwindigkeit der Inflation im Nachhinein überhaupt bestimmen?

    Auf Wikipedia [ https://de.wikipedia.org/wiki/Inflation_(Kosmologie) ] habe ich nur gefunden, dass die Inflation etwa 10^(−35) s nach dem Urknall begann und bis zu einem Zeitpunkt zwischen 10^(−33) s und 10^(−30) s nach dem Urknall dauerte. Während der Inflation dehnte sich das Universum laut Wikipedia mindestens um den Faktor 10^26 aus.
    In einem Lehrbuch zur Kern- u. Teilchenphysik stand, dass der Faktor zwischen etwa 10^30 und 10^50 lag.

    Daraus lässt sich aber nicht die Geschwindigkeit berechnen, denn dazu müsste man ja noch die Größe zum Beginn bzw. zum Ende der Inflation kennen.

    Die Lichtgeschwindigkeit im Vakuum beträgt 299792458 m/s.
    Die Inflation dauerte minimal 9,9*10^(-34) s lang und maximal 9,9999*10^(-31) s lang.
    In 9,9*10^(-34) s legt Licht im Vakuum 2,9679*10^(-25) m zurück.
    In 9,9999*10^(-31) s legt Licht im Vakuum 2,9979*10^(-22) m zurück.
    Beide Strecken sind kleiner als ein Femtometer ( 1 fm = 10^(-15) m ) und kleiner als der Durchmesser eines Protons von ca. 1,7*10(−15) m.

    Wenn das Universum zu Beginn der Inflation sehr viel kleiner als ein Proton war, wäre eine Inflation mit Lichtgeschwindigkeit möglich.

  1129. #1148 Karl-Heinz
    25. Januar 2019

    @Peter
    Bei der Inflation hat sich das Universum exponentiell ausgedehnt. Am Besten du nimmst eine e-Funktion und legst zwischen zwei Punkten, die auf einer e-Funktion liegen, eine Gerade. Die Gerade würde dann eine lineare Ausdehnung entsprechen. Die Steigung der Gerade würde dann die Geschwindigkeit bei linearer Ausdehnung entsprechen. Die Tangente an der e-Funktion würde dann die Momentangeschwindigkeit bei exponentieller Ausdehnung entsprechen. Fällt dir dabei etwas auf? Morgen muss ich Nachhilfe in Mathe geben. 😉

  1130. #1149 Noonscoomo
    Berlin
    25. Januar 2019

    @Peter
    Die Inflationstheorie geht davon aus, dass sich der Raum mit einem grossen Vielfachen der Lichtgeschwindigkeit ausdehnte.
    Das steht nicht im Widerspruch zur Speziellen Relativitätstheorie (SRT), denn laut dieser kann Infomation nicht schneller als mit Lichtgeschwindigkeit übertragen werden. Das ist aber auch nicht der Fall, im Gegenteil.
    Wenn sich der Raum zwischen Dingen ausdehnt wird die Entfernung zwischen allen Dingen, die nicht gravitativ aneinander „kleben“ grösser, und zwar mit Überlichtgeschwindigkeit. D.h. Information kann zwischen denen gar nicht mehr ausgetauscht werden. Und genau das ist ja auch der Witz bei der Inflationstheorie.

  1131. #1150 Karl-Heinz
    26. Januar 2019

    @Peter

    Abschätzen der Expansionsgeschwindigkeit der Inflation zum Zeitpunkt $latex $t_{1}=10^{-30}s$
    $latex $R(t)=R_{0}e^{at}$
    $latex $\frac{R(t)}{R_{0}}$ … Ausdehnungsfaktor(t)
    $latex $\frac{\mathrm{d} R(t)}{\mathrm{d} t}=\frac{\mathrm{d} (R_{0}e^{at})}{\mathrm{d} t}=aR_{0}e^{at}=v(t)$
    $latex $t_{0}=10^{-35}s$
    $latex $t_{1}=10^{-30}s$
    $latex $\tau =t-t_{0}=t-10^{-35}\approx t$
    $latex $\frac{R(t_{1})}{R_{0}}=10^{26}$
    $latex $\frac{R(t_{1})}{R_{0}}=\frac{R^{0}e^{at_{1}}}{R_{0}}=e^{at_{1}}=e^{a10^{-30}}=10^{26}\rightarrow a\approx6\cdot10^{31}$
    $latex $v(t_{1})=aR_{0}e^{at_{1}}=6\cdot10^{31}\cdot R_{0}\cdot10^{26}\approx 6\cdot10^{57}\cdot R_{0}$

    R_0 kann nicht kleiner als die Planck-Länge (1,616 10^(-35)) sein. Die Überlichtgeschwindigkeit wird zumindest am Ende der Inflation locker überboten. 😉

  1132. #1151 Karl-Heinz
    26. Januar 2019

    @Peter

    Abschätzen der Expansionsgeschwindigkeit der Inflation zum Zeitpunkt t_{1}=10^{-30}s
    $latex $R(t)=R_{0}e^{at}$
    $latex $\frac{R(t)}{R_{0}}$ … Ausdehnungsfaktor(t)
    $latex $\frac{\mathrm{d} R(t)}{\mathrm{d} t}=\frac{\mathrm{d} (R_{0}e^{at})}{\mathrm{d} t}=aR_{0}e^{at}=v(t)$
    $latex $t_{0}=10^{-35}s$
    $latex $t_{1}=10^{-30}s$
    $latex $\tau =t-t_{0}=t-10^{-35}\approx t$
    $latex $\frac{R(t_{1})}{R_{0}}=10^{26}$
    $latex $\frac{R(t_{1})}{R_{0}}=\frac{R^{0}e^{at_{1}}}{R_{0}}=e^{at_{1}}=e^{a10^{-30}}=10^{26}\rightarrow a\approx6\cdot10^{31}$
    $latex $v(t_{1})=aR_{0}e^{at_{1}}=6\cdot10^{31}\cdot R_{0}\cdot10^{26}\approx 6\cdot10^{57}\cdot R_{0}$

    R_0 kann nicht kleiner als die Planck-Länge (1,616 10^(-35)) sein. Die Überlichtgeschwindigkeit wird zumindest am Ende der Inflation locker überboten. 😉

  1133. #1152 Karl-Heinz
    26. Januar 2019

    @Peter

    Sorry wegen Mehrfachpost
    Abschätzen der Expansionsgeschwindigkeit der Inflation zum Zeitpunkt t_{1}=10^{-30}s
    R(t)=R_{0}e^{at}
    \frac{R(t)}{R_{0}} … Ausdehnungsfaktor(t)
    \frac{\mathrm{d} R(t)}{\mathrm{d} t}=\frac{\mathrm{d} (R_{0}e^{at})}{\mathrm{d} t}=aR_{0}e^{at}=v(t)
    t_{0}=10^{-35}s
    t_{1}=10^{-30}s
    \tau =t-t_{0}=t-10^{-35}\approx t
    \frac{R(t_{1})}{R_{0}}=10^{26}
    \frac{R(t_{1})}{R_{0}}=\frac{R^{0}e^{at_{1}}}{R_{0}}=e^{at_{1}}=e^{a10^{-30}}=10^{26}\rightarrow a\approx6\cdot10^{31}
    v(t_{1})=aR_{0}e^{at_{1}}=6\cdot10^{31}\cdot R_{0}\cdot10^{26}\approx 6\cdot10^{57}\cdot R_{0}

    R_0 kann nicht kleiner als die Planck-Länge (1,616 10^(-35)) sein. Die Überlichtgeschwindigkeit wird zumindest am Ende der Inflation locker überboten. 😉

  1134. #1153 Metalgeorge
    26. Januar 2019

    @Peter

    Ist eine kosmische Inflation mit Überlichtgeschwindigkeit physikalisch möglich?
    Es heißt ja immer, nichts könne schneller sein als das Licht. Gilt dies auch für die Raumzeit an sich?

    Falls das mit der Überlichtgeschwindigkeit nicht stimmt, mit welcher Geschwindigkeit hat sich das Universum während der Inflation ausgedehnt?
    Etwa mit Lichtgeschwindigkeit im Vakuum oder mit (deutlich) weniger als der Lichtgeschwindigkeit im Vakuum?

    Ich glaube hier liegt das Verständnisproblem, was genau beim Urknall passierte.
    Es gab vor dem Urknall eben nichts , kein Vakuum und auch keine Raumzeit.
    Das Universum entstand nicht in einem vorhandenen Raum (Vakuum)
    mit den entsprechenden Gesetzen (Lichtgeschwindigkeit) oder dehnte sich darin aus.
    Die Raumzeit und das Vakuum und die entsprechenden Gesetze etablierten sich erst und gelten innerhalb dieses entstandenen Universums.
    Auf Grund des expandierenden, sogar beschleunigt expandierenden (dunkle Energie), Universums (Raumes), können sich darin enthaltenene Objekte durchaus mit Überlichtgeschwindigkeit voneinander entfernen. s.u.
    Von Geschwindigkeiten während der Inflationsphase zu sprechen halte ich für sehr schwierig,
    da man hierzu ja irgendwelche Bezugsgrössen zu diesem Zeitpunkt festlegen müsste.
    @Karl-Heinz versucht es hier #1152 an 2 theoretischen Zeitpunkten und dazugehörigen Durchmessern (Radien) festzumachen.
    Was wir wissen ist , dass offenbar so gut wie alle Regionen des beobachtbaren Universums
    eine identische Information enthalten , die sie aber auf Grund Ihrer Entfernung
    zueinander und dem Alter des Universums gar nicht haben dürften.
    Die Temperatur der Hintergrundstrahlung.
    (Begrenzung der Informationsausbreitung auf die Lichtgeschwindigkeit.)
    https://scienceblogs.de/astrodicticum-simplex/2018/12/14/sternengeschichten-folge-316-die-kosmische-hintergrundstrahlung/

    Dies führte schliesslich zur Entwicklung der Inflationsmodelle.
    Siehe auch Beobachtungshorizont hier

    https://de.m.wikipedia.org/wiki/Beobachtbares_Universum

  1135. #1154 UMa
    26. Januar 2019

    @Peter:
    Nichts verbietet in der allgemeinen Relativitätstheorie, dass sich der Raum schneller ausdehnt als das Licht, nicht nur während der Inflation.
    So können Abstände zwischen weit entfernten Galaxien schneller zunehmen, als mit Lichtgeschwindigkeit. Das Licht dieser Galaxien kann uns trotzdem erreichen, obwohl anfangs der Abstand zwischen Beobachter und dem Licht noch größer wird. Aber allmählich kommt das Licht in Raumgebiete, die sich nicht mehr so schnell ausdehnen und kaum dadurch näher und kann uns erreichen.

  1136. #1155 Peter
    26. Januar 2019

    Zitat Metalgeorge #1153: „Ich glaube hier liegt das Verständnisproblem, was genau beim Urknall passierte. Es gab vor dem Urknall eben nichts, kein Vakuum und auch keine Raumzeit. Das Universum entstand nicht in einem vorhandenen Raum (Vakuum) mit den entsprechenden Gesetzen (Lichtgeschwindigkeit) oder dehnte sich darin aus.“

    Das die Raumzeit unseres Universums und dessen physikalische Gesetze vor dem Urknall nicht existierten, ist mir klar.
    Ich hatte ja nach der Inflation gefragt und diese begann ja nicht direkt mit dem Urknall. Jedenfalls wird es unter
    https://de.wikipedia.org/wiki/Urknall#Frühes_Universum so dargestellt:
    Direkt nach dem Urknall (Zeitpunkt = 0) folgte die Planck-Ära bis zum Zeitpunkt 10^(-43) s. Erst danach kam die GUT-Ära, in der die Kosmische Inflation etwa 10^(-35) s nach dem Urknall begann und bis zu einem Zeitpunkt zwischen 10^(-33) s und 10^(-30) s nach dem Urknall andauerte (Quelle: https://de.wikipedia.org/wiki/Inflation_(Kosmologie) ).

    Wenn die Inflation „erst“ ca. 10^(-35) s nach dem Urknall begann, müssen zu diesem Zeitpunkt ja schon physikalische Gesetze gegolten haben.

    @ Karl-Heinz: Wie kann man hier Formeln einfügen?

  1137. #1156 Karl-Heinz
    26. Januar 2019

    @Peter

    $-Zeichen + Latex + Leerzeichen + Latexformel + $-Zeichen

  1138. #1157 Metalgeorge
    27. Januar 2019

    @Peter #1155

    Das die Raumzeit unseres Universums und dessen physikalische Gesetze vor dem Urknall nicht existierten, ist mir klar…..

    prima..:)
    Dann musst du nur noch verstehen, dass durch die Ausdehnung
    des Universums, auch während der Inflationsphase, keine Objekte
    innerhalb des Universums beschleunigt und mit einer zusätzlichen
    Geschwindigkeit versehen werden, die dann den relativistischen
    Gesetzen unterliegen würden.
    Sondern sie entfernen sich voneinander, weil sich der Raum ausdehnt.
    (Hierzu wird vielfach die Haut eines Luftballons als Gleichnis verwendet.)
    Und diese Ausdehnungsgeschwindigkeit #1152 ist eben nicht
    den relativistischen Gesetzen unterworfen.
    Wir wissen seit der Entdeckung, dass sich das Universum ausdehnt,
    dass es es einen Beobachtungshorizont für uns in unserem
    Universum gibt. Jenseits dieses Horizonts bewegen sich Objekte
    mit Überlichtgeschwindigkeit von uns weg. Das Licht dieser
    Objekte wird uns also nie erreichen.
    Wir wissen auch , dass das reale Universum,
    übrigens auch auf Grund der Inflationsmodelle, viel Grösser (ca 4 mal) ist
    als das, das wir sehen.
    Es ist also leicht ersichtlich, dass sich 2 fiktiv “gegenüberligende”
    Punkte am “Rande” des Universums sich mit weit über Lichtgeschwindigkeit
    voneinander wegbewegen müssen.
    Das heißt, nicht nur damals zur Zeit der Inflation,
    sondern auch “heute” existiert dieser Effekt.
    Siehe nochmals Artikel über das beobachtbare Universum.

  1139. #1158 Christian
    Wien
    27. Januar 2019

    > dass das reale Universum, übrigens auch auf Grund der Inflationsmodelle, viel Grösser (ca 4 mal) ist als das, das wir sehen.

    Der Durchmesser des beobachtbaren Universums beträgt 93 Mrd. Lichtjahre. Bezieht sich “ca 4 mal” auf die Länge oder das Volumen? Soll der tatsächliche Durchmesser also 372 Mrd. oder 148 Mrd. Lichtjahre sein? Wer ist wie auf diesen Wert gekommen? Hast du vielleicht einen Link dazu?

  1140. #1159 Niels
    27. Januar 2019

    @Peter

    Ein paar Ergänzungen zum schon von Karl-Heinz, Noonscoomo, Metalgeorge und UMa gesagten:

    Es heißt ja immer, nichts könne schneller sein als das Licht. Gilt dies auch für die Raumzeit an sich?

    Nein.
    Durch die Expansion des Universums “entfernen sich” auch in dieser Sekunde sehr weit entfernte Galaxien mit deutlicher Überlichtgeschwindigkeit von uns.

    Daraus lässt sich aber nicht die Geschwindigkeit berechnen, denn dazu müsste man ja noch die Größe zum Beginn bzw. zum Ende der Inflation kennen.

    Eigentlich nicht.
    Ist dir klar, wie man die aktuelle Expansionsgeschwindigkeit des Universums üblicherweise angibt?
    Das ist der berühmte Hubble-Parameter, der in den Einheiten [(Kilometer) pro (Sekunde mal Megaparsec)] angegeben wird.

    Der Hubble-Parameter H ist definiert als
    H = a'(t)/a(t)
    Also
    H = [“zeitliche Ableitung a'(t) des Skalenfaktors” geteilt durch den “Skalenfaktor a(t)”]
    Der heutige Wert ist ungefähr H = 70.

    Der Skalenfaktor a(t) gibt die relative Expansion des Universums an.
    Während er Inflation expandierte das Universum exponentiell, d.h. es galt
    a(t) = C*exp[t]
    Die Ableitung der E-Funktion ist wieder die e-Funktion, a'(t) folgt in dieser Zeit also ebenfalls einem exponentiellen Verlauf.
    H = a'(t)/a(t) ist also gleich einer Konstanten.

    Aus der Inflationsdauer (zwischen 10^(−33) s und 10^(−30) s nach dem Urknall) und dem Ausdehnungsfaktor (zwischen etwa 10^30 und 10^50) kann man also problemlos H berechnen und mit dem heutigen Wert (H = 70) vergleichen.

    Ist aber für die Intuition nicht besonders nützlich, der H-Wert der Inflation ist offensichtlich in jedem Fall eine gigantische Zahl mit unfassbar vielen Nullen.
    So eine Zahl kann man sich schlicht nicht vorstellen.

    Wenn das Universum zu Beginn der Inflation sehr viel kleiner als ein Proton war, wäre eine Inflation mit Lichtgeschwindigkeit möglich.

    Nach aktuellen Modellen ist das Universum höchstwahrscheinlich unendlich groß. Dann muss es zur Inflationszeit natürlich ebenfalls unendlich groß gewesen sein.

    Das beobachtbare(!) Universum war damals nur protonengroß und wurde durch die Inflation um einen gigantischen Faktor aufgeblasen.

    Wie schon von Metalgeorge erwähnt wurde die Inflation erfunden, weil aus den Messungen der Hintergrundstrahlung folgt, dass sich das komplette beobachtbare Universums einmal in einem thermischen Gleichgewicht befunden haben muss.
    Heute ist das beobachtbare Universum aber so groß, dass seine Ränder nie miteinander in kausalem Kontakt gewesen sein können, wenn man ohne Inflation rechnet.
    Wenn sie nie in Kontakt waren, können sie aber auch nicht ihre Temperatur ausgeglichen haben.
    Die Inflation löst wie gesagt dann (unter anderem) dieses sogenannte Horizont-Problem.
    Winzige Quantenfluktuationen wurden dadurch so aufgeblasen, dass daraus gigantische Strukturen wie Galaxien-Superhaufen und Leer-Räume ohne Materie (Voids) entstanden.

    .
    Zur Expansion des Universums lohnt sich dieser Artikel bei “Hier wohnen Drachen”:
    https://scienceblogs.de/hier-wohnen-drachen/2010/09/19/wie-gross-ist-das-beobachtbare-universum/

    Zum Skalenfaktor und dem Hubble-Paramet ein Artikel bei Alpha Cephei “Was ist eine “mitbewegte Entfernung”?”, speziell eben dort die Abschnitte “Die Hubble-Konstante (?)” und “Der Skalenfaktor und die Rotverschiebung”.
    https://scienceblogs.de/alpha-cephei/2018/05/28/was-ist-eine-mitbewegte-entfernung/

    .

    @Christian

    Bezieht sich “ca 4 mal” auf die Länge oder das Volumen?

    Auf den Skalenfaktor, also letztlich auf den Radius des beobachtbaren Universums.

  1141. #1160 Karl-Heinz
    27. Januar 2019

    @Niels
    Danke für die ausführliche Erläuterung. (freu)

    So auf die Schnelle ohne Gewähr auf Richtigkeit.
    a(t)=a_0 * e^(H*t)
    a'(t)=H * a_0 * e^(H*t) falls H für den zu betrachtenden Zeitraum konstant ist.
    a'(t)/a(t) =H * a_0 * e^(H*t)/(a_0 * e^(H*t))=H

  1142. #1161 Niels
    27. Januar 2019

    @Karl-Heinz

    Jo, sollte passen.

    Vor ewigen Zeiten hab ich mal einen Kommentar mit Kurven zu diesen ganzen Parametern verfasst:
    https://scienceblogs.de/astrodicticum-simplex/2014/02/06/video-wie-wird-das-universum-sterben/#comment-238748
    (Bitte auch den Kommentar 68 direkt darunter betrachten.)

    Mir hat das bei der Veranschaulichung enorm geholfen.
    (Das sind allerdings Schaubilder für die “normale” Expansion, die Inflationsphase ist also nicht berücksichtigt.)

    Wenn man nach der Anleitung in #68 mal den Skalenfaktor a(t) und die “Geschwindigkeit” der Expansion a'(t) beispielsweise für die Zeitspanne 10 Milliarden Jahre nach dem Urknall bis 100 Milliarden Jahre danach plottet, sieht man, dass die Expansion momentan beinahe exponentiell und in Zukunft völlig exponentiell verläuft.

    Dann überwiegt nämlich der Einfluss der dunklen Energie auf die Expansion.
    Dunkle Energie und Inflationsfeld sind (in der reinen ART-Beschreibung) verblüffend ähnliche Phänomene.
    Nur die Größenordnung der Expansion ist völlig unterschiedlich.
    Vereinfacht zusammengefasst:
    Durch die dunkle Energie gibt es 0,000000001% “Zinsen” im Jahr, durch die Inflation 10000000000000000%.
    .

    Hier sieht man, dass H = a'(t)/a(t) in Zukunft gegen einen Grenzwert läuft.
    Schaubild Hubble-Parameter
    Das ist wie gesagt das erwartete Verhalten bei vollständig exponentiellem Verlauf der Expansion.

  1143. #1162 Mysterion1000
    Salzburg
    30. Januar 2019

    Einer hat mir diese frage gestellt: “Eine Laienfrage: Wenn die Bindungsenergie so hoch ist, warum zerfällt es dann?”
    Kann das jemand beantworten? Es geht um diesen Artikel bzw diesen Kontext. Ich glaube zu wissen, dass die Fragestellung falsch formuliert ist aber die Antwort darauf weiß ich auch nicht. Ich denke, es ist ein wenig komplizierter, als die Fragesteller es sich denkt.
    https://derstandard.at/2000097189634/Physiker-erzeugen-neue-Form-von-Materie#posting-1038304607

  1144. #1163 Peter
    30. Januar 2019

    @ Mysterion1000 #1162:

    Ein Anti-Kaon besteht aus einem leichten Up-Antiquark und einem mittelschweren Strange-Quark. Das Strange-Quark ist instabil und zerfällt in das leichtere Up-Quark.
    https://deacademic.com/dic.nsf/dewiki/86046

    Dieser Zerfall findet nur über die Schwache Wechselwirkung statt und dauert aus Sicht der Teilchenphysik relativ „lang“, die mittlere Lebensdauer von freien Anti-Kaonen liegt bei (1,2385 ± 0,0024) * 10^(−8) s.

    Im Atomkern zerfällt ein gebundenes Anti-Kaon auch bei einer hohen Bindungsenergie nach einer gewissen Zeit.

    Was ich in dem Abstract leider nicht gefunden habe, ist wie groß die mittlere Lebensdauer des gebundenen Anti-Kaons im gebundenen Zustand mit 2 Protonen ist.

    Denn die mittlere Lebensdauer eines Teilchens kann im gebundenen Zustand (deutlich) länger sein als die mittlere Lebensdauer eines gleichen freien (ungebundenen) Teilchens.

  1145. #1164 Mysterion1000
    Bitte eine PLZ eingeben!
    30. Januar 2019

    @Peter Vielen dank für die Ausführung. Ich habe schon befürchtet, dass es in diese Richtung geht. Ich werde es so weiterleiten. Für die meisten Laien ist so eine Antwort nicht befriedigend, da der Inhalt der Antwort einem klar macht, dass man durch die Antwort zwar weiß worum es geht aber beim Blick auf das gesamte, eigentlich genauso ratlos dasteht, wie davor. 😀

  1146. #1165 Mysterion1000
    Salzburg
    30. Januar 2019

    Peter, vielen Dank für die ausführliche Antwort.
    Ps:
    Ich bin mir nicht sicher, ob meine Nachricht die ich zuvor gesendet habe angekommen ist, da ich genau beim Absenden gemerkt habe, dass der Ort fehlt und eventuell nicht durchgekommen ist, falls doch, sorry Florian :P:

  1147. #1166 Peter
    1. Februar 2019

    Zitat Mysterion1000: „Für die meisten Laien ist so eine Antwort nicht befriedigend …“

    Quanten- und Teilchenphysik ist nun mal leider nicht so einfach zu verstehen* und die meisten Fachleute können es leider auch nicht einfach erklären.
    * An der Uni hatte ich eine Vorlesung über Teilchenphysik, bei der lesende Professor sagte, dass selbst die Fachleute nicht alles verstehen würden.

    Das wäre eigentlich mal etwas für ScienceBlogs (z. B. für den Blog-Schreibwettbewerb): Quanten- und Teilchenphysiker/-innen sind aufgerufen, ihre Themen so allgemeinverständlich wie möglich zu erklären.

    Was man sich merken kann, ist das die leichtesten Elementarteilchen stabil sind (z. B. die Photonen = Lichtteilchen, die Neutrinos, die Elektronen, die Up-Quarks und die Down-Quarks). Die Bausteine der Atomkerne, die Protonen und Neutronen, bestehen aus Up-Quarks und Down-Quarks.
    Schwerere Elementarteilchen sind dagegen instabil (z. B. wie z. B. die Charm-Quarks, Strange-Quarks, Bottom-Quarks und Top-Quarks), sie zerfallen nach relativ kurzer Zeit in leichtere Teilchen.

  1148. #1167 PDP10
    1. Februar 2019

    @Peter, Mysterion1000:

    Man könnte auch einfach bei einem Scienceblogger nebenan stöbern. Martin Bäker hat viel, viel, viel über Elementarteilchen und so Kram geschrieben.

    Hier sind alle seine Artikel-Serien verlinkt:

    https://scienceblogs.de/hier-wohnen-drachen/artikelserien/

    Insbesondere die Serien über Quantenmechanik (kurz) und Quantenfeldtheorie für Laien (sehr lang!) sind da vielleicht interessant.

  1149. #1168 Mysterion1000
    Salzburg
    2. Februar 2019

    @PDP10 ich lese schon sehr lange mit, war auch unter einem anderen Namen hier aktiv. Ich kenne auch die Ausführungen von Martin Bäker. Für einen Laien verstehe ich schon viel aber im Hinblick auf größere Zusammenhänge, komme ich einfach ins strudeln. So wie ich es verstehen will, müsste ich es eigentlich studieren, was ich ernsthaft in Erwägung gezogen habe^^ . Analogien, veranschauliche Beispiele, helfen mir nicht weiter, weil es höchstens dazu führt, dass man dieses eine spezifische Problem versteht. Im Bezug auf das Verständnis von zusammenhängen, ist es sogar hinderlich.

  1150. #1169 PDP10
    2. Februar 2019

    @Mysterion1000:

    Dann hätte ich vielleicht einen Buchtipp für dich:

    David Griffiths: Introduction to Elementary Particles

    Leider hinreichend teuer …

    Das Buch ist vor allem eine detaillierte qualitative Einführung. Man wird größtenteils nicht mit kilometerlangen Formeln behelligt. Es setzt allerdings schon einiges an Physik- und Mathematikkenntnissen voraus. Abi-Niveau ungefähr. Und man sollte sich einigermaßen mit den Grundlagen der Quantenmechanik auskennen.

    Dafür bekommt man eine solide Einführung. Insbesondere mit einem langen historischen Überblick am Anfang. Du kannst ja mal beim großen A die “Blick ins Buch” Funktion benutzen. Oder du guckst mal obs in der nächsten Uni-Bib rumsteht. Wie geschrieben: leider recht teuer. Aber eventuell ist die erste Auflage preiswerter antiquarisch zu bekommen – die hatte ich während des Studiums oft in der Hand und war begeistert.

  1151. #1170 Peter
    2. Februar 2019

    @ PDP10:
    An meiner Uni wurde das Buch „David Griffiths: Introduction to Elementary Particles” eher für Physik-Studierende in höheren Semestern empfohlen. Das klingt nicht gerade nach einem Buch, was auch für Nicht-Physiker/-innen gut verständlich ist.

    Kennt jemand zufällig Bücher über Quanten- u. Teilchenphysik, die zumindest die wichtigen Grundlagen allgemeinverständlich erklären (also z. B. auf komplizierte Herleitungen und zu viele Formeln verzichten)?

  1152. #1171 PDP10
    2. Februar 2019

    @Peter:

    Das klingt nicht gerade nach einem Buch, was auch für Nicht-Physiker/-innen gut verständlich ist.

    Hmja… kommt drauf an, denke ich. Das ist keine Einführung in die Elementarteilchenphysik für Studenten, die sich darauf spezialisieren wollen sondern eher für die, die sich da aus Interesse einen tiefergehenden Überblick anlesen wollen. Ich habe das zum ersten Mal in der Hand gehabt, als ich eine Vorlesung über theoretische Elementarteilchen-Physik gehört (und meine intellektuellen Grenzen kennen gelernt) habe. Das Buch war nicht geeignet mir beim Lösen der Übungsaufgaben zu helfen aber ohne das Buch hätte ich nie einen vernünftigen Überblick über das Feld bekommen.

    Wie gesagt: Man braucht solide Grundlagen in Physik und Mathe und ein wenigstens Anfänger-Verständnis der QM. Besonders der Grundlagen des Formalismus.
    Von alldem braucht man aber kein “working Knowledge”.

    Und so, wie sich @Mysterion1000 ausgedrückt hat, denke ich, dass sie / er diese Grundlagen hat. Verstehe ich jedenfalls so. Und man kann das Buch ja, wie geschrieben auch “Ausprobieren”.

    Kennt jemand zufällig Bücher über Quanten- u. Teilchenphysik

    Gute Frage. Ich kenne nicht mal besonders viele vernünftige Lehrbücher für Physiker über QM. Genau genommen nur eins. Das von F. Schwabl. Alle anderen, die ich jemals benutzt habe waren nur in Teilen zu gebrauchen oder kompletter Mist. Das ist aber mein Stand aus den frühen Neunzigern.

    Wenns Formeln sein dürfen, aber “keine komplizierten Herleitungen etc.”: Vielleicht siehst du dir mal die Bücher aus der “The theoretical minimum” Serie von Leonard Susskind an:

    https://theoreticalminimum.com/

  1153. #1172 Ernst Haugeneder
    Wien
    4. Februar 2019

    In einer kürzlich gesendeten Rätselsendung und in mehreren Artikeln im Internet (zB Spektrum.de vom 24.2.2014) wurde die Behauptung aufgestellt, dass sich die Erde ohne Mond (theoretisch) etwa 3 Mal so schnell drehen würde als mit Mond. Begründet wird dies mit dem Wegfall der Bremswirkung der Gezeiten. Wie kann das erklärt werden? Der Wegfall einer Bremse bewirkt noch keine Beschleunigung, oder?

  1154. #1173 PDP10
    4. Februar 2019

    @Ernst Haugeneder:

    Wie kann das erklärt werden? Der Wegfall einer Bremse bewirkt noch keine Beschleunigung, oder?

    Nee. Aber der Wegfall der Bremse bewirkt einen Wegfall des Bremsens.

    Die Erde hat sich früher (meint Hunderte Millionen Jahre früher) deutlich schneller gedreht. Ohne den Mond würde sie das immer noch tun.

  1155. #1174 Peter
    4. Februar 2019

    Als sich der Mond gebildet hat, war er noch viel näher an der Erde.
    https://de.wikipedia.org/wiki/Mond#Entstehung_des_Mondes
    Zitat: „Nach aktuellen Simulationen bildete sich der Mond in einer Entfernung von rund drei bis fünf Erdradien, also in einer Höhe zwischen 20.000 und 30.000 km.“

    Die Anziehungskraft des Mondes bewirkt auf der Erde Gezeiten. Weil sich die Erde schneller dreht als der Mond für einen Umlauf braucht, bremsen die Gezeitenwellen die Erdrotation. Seit die Erde Meere hat, bremst vor allem die Reibung zwischen dem Wasser (welches bei Flut aufläuft u. bei Ebbe wieder abläuft) und der festen Erdkruste (die feste Erdkruste hebt sich bei Flut zwar auch, aber deutlich schwächer als das Wasser, nämlich nur um wenige cm).

    https://de.wikipedia.org/wiki/Mondbahn#Säkulare_Akzeleration
    Zitat: „Die vom Mond auf den Erdozeanen aufgetürmten Gezeitenwellen werden von der Erdrotation seitlich versetzt, so dass sie nicht exakt in der Verbindungslinie Erde-Mond liegen und ihrerseits ein Drehmoment auf den Mond ausüben. Dieses Drehmoment führt dem Mond Drehimpuls und Energie zu, so dass er auf eine höhere, energiereichere Bahn gehoben wird … Infolge der gezeitenbedingten Anhebung seiner Bahn entfernt sich der Mond pro Jahr um 3,8 cm von der Erde.“

    In der Frühzeit der Erde war dieser Effekt noch stärker: Flut und Ebbe waren stärker, wodurch die Erdrotation stärker abgebremst wurde und der Mond sich schneller von der Erde entfernte als heutzutage.

    https://de.wikipedia.org/wiki/Erdrotation#Langfristige_Änderungen

    Zitat: „Für prähistorische Zeiten lässt sich die Geschwindigkeit der Erdrotation aus täglichen Wachstumsringen fossiler Meeresorganismen mit Kalkskelett ablesen.[8]
    Entsprechende Untersuchungen deuten beispielsweise an, dass vor 400 Millionen Jahren das Jahr etwa 400 Tage hatte; bei angenommener gleicher Jahresdauer dauerte ein Tag also nur circa 21,9 Stunden.
    … Mathematische Modelle für die frühe, gerade im Entstehen befindliche Erde, also vor rund 4 Milliarden Jahren, legen eine ursprüngliche Tageslänge von lediglich 14 Stunden nahe.[9] Andere Wissenschaftler nehmen für diese Phase der Erdgeschichte eine Rotationsdauer von sechs bis sieben Stunden an.[10]“

  1156. #1175 Mysterion1000
    Salzburg
    4. Februar 2019

    @PDP10 So teuer ist dein Buchvorschlag gar nicht. Auf deutsch wäre mir das Buch aber schon lieber. Ich habe nicht die besten Englischkenntnisse. Nebenbei, wenn ich schon im richtigen Thread bin. Darf das Neutrino im Standardmodell deswegen keine Masse haben, weil sie schwach wechselwirken und im Hinblick auf das postulierte Higgsfeld es entsprechend nicht zu Wechselwirkung gekommen sein kann?

  1157. #1176 PDP10
    4. Februar 2019

    @Mysterion1000:

    Auf deutsch wäre mir das Buch aber schon lieber.

    Gibts. Aber nur Antiquarisch bzw. gebraucht und nur die erste Auflage. Griffiths: “Einführung in die Elementarteilchenphysik”.

    Darf das Neutrino im Standardmodell deswegen keine Masse haben, weil […]

    Sorry, da bin ich überfragt. Aber sowas von 🙂

  1158. #1177 Karl-Heinz
    4. Februar 2019

    @Mysterion1000

    Darf das Neutrino im Standardmodell deswegen keine Masse haben, weil […]

    https://t2k-experiment.org/neutrinos/in-the-standard-model/

  1159. #1178 PDP10
    4. Februar 2019

    @Karl-Heinz:

    Coole Web-Site!

    Insbesondere auch das andere Kapitel:
    https://t2k-experiment.org/neutrinos/beyond-the-standard-model/

    In dem es darum geht, wie die Neutrinos ihre Masse “bekommen” haben.

    Aber nix da beantwortet die eigentlich Frage in Bezug auf das Higgs-Feld.

    Selbiges brauchte man ja vor allem deswegen zu Erklärung der Masse der Teilchen, die keine Austauschteilchen sind weil man gemessen hat, dass W- und Z-Bosonen – die doch Austauschteilchen sind – eine Masse haben.

    Ob es da einen Zusammenhang mit der Masse der Neutrinos gibt? Keine Ahnung. Und in den Links auf der von dir verlinkten Seite steht – soweit ich das nach kurzem Überfliegen sagen kann – auch nichts.

    Müstisch!

  1160. #1179 Mysterion1000
    Salzburg
    5. Februar 2019

    @PDP10 Der Link ist zwar interessant aber die eigentliche Frage wird da nicht beantwortet. Wenn man dann versucht aus der Perspektive der Chiralität wie im verlinkten Artikel die Sache zu betrachten und entsprechend Wikipedia aufsucht, landet man in der Vorhölle Satzkonstruktionen, die dazu gedacht ist, einem zu quälen. Beim Versuch bestimmte Begrifflichkeiten zu verstehen, klickt man ja die Hyperlinks an und landet dann bei Sätzen wie: Jede Clifford-Algebra ist isomorph zu einer Teil-Algebra einer reellen, komplexen oder quaternionischen Matrix-Algebra. Diese hat eine kanonische Darstellung durch Spaltenvektoren, die Spinoren”… Du mich auch, habe ich mir gedacht 😀

  1161. #1180 Dieter Schläger
    Deisenhausen
    5. Februar 2019

    Eine Frage zur Dunklen Materie:
    Die Dunkle Materie zeigt keine Wechselwirkung mit der “normalen” Materie, außer der Gravitation. Mit anderen Worten – es gibt keine elektromagnetische Wechselwirkung, sonst könnten wir sie sehen? Wenn es eine schwache Wechselwirkung gäbe, dann könnte sie sich umwandeln? Wie ist es mit der Starken Kernkraft? Ich habe noch nie etwas über “Dunkle Sterne” und “Dunkle Löcher” gehört oder gelesen! Was hält die Dunkle Materie davon a, zu einem gigantischen schwarzen Loch zusammen zu fallen, wenn es die anderen Wechselwirkungen bei ihr nicht gibt?

  1162. #1181 PDP10
    5. Februar 2019

    @Dieter Schläger:

    Mit anderen Worten – es gibt keine elektromagnetische Wechselwirkung, sonst könnten wir sie sehen?

    Exakt. Das war der Grund, warum Fritz Zwicky in den dreißiger Jahren des letzten Jahrhunderts den Begriff überhaupt geprägt hat. Er hat beim Vermessen von Bewegungen von Galaxienhaufen eben festgestellt, dass da mehr Materie sein muss als man “sehen” kann, dh. als Materie, die elektromagnetisch wechselwirkt.

    Wenn es eine schwache Wechselwirkung gäbe, dann könnte sie sich umwandeln? Wie ist es mit der Starken Kernkraft?

    So ganz verstehe ich die Frage nicht, aber Neutrinos, die wegen oben auch zur dunklen Materie gehören wechselwirken auch per schwacher WW. Sonst könnten sie nicht am Betazerfall beteiligt sein.

    Was hält die Dunkle Materie davon a, zu einem gigantischen schwarzen Loch zusammen zu fallen, wenn es die anderen Wechselwirkungen bei ihr nicht gibt?

    Eben, dass es andere Wechselwirkungen nicht gibt. DM klumpt nicht. Materie zB. die zu Staub zusammenklumpt tut das immer aufgrund elektrostatischer Kräfte (Kohäsion). Du kannst dich also bei Maxwell bedanken für die Staubmäuse unter deinem Bett :-). DM fliegt einfach immer aneinander vorbei …

  1163. #1182 PDP10
    5. Februar 2019

    @Dieter Schläger:

    Nachtrag:

    Florian hat mal eine ganze Artikel-Serie über DM geschrieben:

    https://scienceblogs.de/astrodicticum-simplex/2013/06/26/dunkle-welten-alles-uber-dunkle-materie-die-komplette-serie/

    Da werden möglicherweise alle deine Fragen beantwortet.

  1164. #1183 Alderamin
    5. Februar 2019

    @Dieter Schläger

    Mit anderen Worten – es gibt keine elektromagnetische Wechselwirkung, sonst könnten wir sie sehen?

    Richtig.

    Wenn es eine schwache Wechselwirkung gäbe, dann könnte sie sich umwandeln?

    Dann könnte sie mit anderen Teilchen, die auch schwach Wechselwirken, interagieren und Zerfälle auslösen, wie Neutrinos das gelegentlich tun, und ma könnte die Zerfallsprodukte nachweisen (bisher ohne Erfolg). Oder sogar mit sich selbst annihilieren (vielleicht hat man solche Strahlung schon gemessen, vielleicht auch nicht).

    Wie ist es mit der Starken Kernkraft?

    https://en.wikipedia.org/wiki/Strongly_interacting_massive_particle
    Werden durch Beobachtungen weitgehend ausgeschlossen.

    Ich habe noch nie etwas über “Dunkle Sterne” und “Dunkle Löcher” gehört oder gelesen! Was hält die Dunkle Materie davon a, zu einem gigantischen schwarzen Loch zusammen zu fallen, wenn es die anderen Wechselwirkungen bei ihr nicht gibt?

    Impulserhaltung und Energieerhaltung (im Wesentlichen genau das, was Planeten davon abhält, in die Sonne zu fallen).

    Wie entsteht massive Materie? Atome gehen Molekülbindungen ein (DM nicht). Moleküle können Gitter bilden oder durch Van-der-Waals-Kräfte aneinander haften (alles elektromagnetische Kräfte; DM kann das nicht). So können Staubteilchen entstehen, die sich mechanisch verhaken oder aneinander festfrieren. Staub kann in Gas abgebremst werden oder unelastisch kollidieren (DM sieht nichts von anderen Teilchen und fliegt einfach an allem vorbei). So können größere Objekte entstehen, die dann zu noch größeren kollidieren können. Die Aufprallenergie wird zunehmend in Wärme umgewandelt und abgestrahlt. Am Ende entstehen kugelförmige Körper, die durch ihre Schwerkraft Gas und andere Körper anziehen und weiter wachsen. Wachsen sie zu Sternen und haben sie ihre Fusionsphasen durchlebt, dann kann der Kern sehr großer Sterne unter seiner Eigengravitation irgendwann zu einem Schwarzen Loch kollabieren. Die DM scheitert aber schon daran, zwei Partikel aneinander zu binden. Keine Moleküle, kein Staub, schon gar keine Sterne entstehen so. Also auch keine Schwarzen Löcher.

    Gut, hat man eben ein normales Schwarzes Loch, kann das jetzt nicht sehr viel Dunkle Materie anziehen?

    Wie fällt Materie in ein Schwarzes Loch? Zunächst kreist sie drum herum und wird zerbröselt. Die Brösel kollidieren (kann DM nicht) und heizen sich auf (tut DM nicht). Das bremst sie ab (DM aber nicht). Dann können sie die Wärme als Infrarot- und Licht abstrahlen, weil sie elektromagnetisch wechselwirken (DM nicht). Durch Reibung geht fortwährend Energie verloren und die Teilchenbahnen werden enger (DM kennt keine Reibungskräfte). Die Reibung wird so stark, dass die Materie sich in ein Plasma aus Atomkernen und freien Elektronen verwandelt, die starke Magnetfelder erzeugen (DM ist neutral), die wiederum 90% der Materie und einen großen Teil des Drehimpulses fortblasen, so dass ein kleiner Teil der Teilchen weiter nach innen rücken kann. Wenn sie einen bestimmten Radius (abhängig davon, ob das Schwarze Loch rotiert) unterschreiten, kreuzen ihre Bahnen zwangsläufig den Ereignishorizont und sie vereinigen sich mit dem Schwarzen Loch.

    Dunkle Materie würde hingegen einfach ungestört kreisen, wenn sie nicht zufällig auf direktem Kollisionskurs mit dem Schwarzen Loch wäre, das mit wenigen km Radius ein winziges Ziel im riesigen All ist. Deswegen fällt nur sehr wenig DM jemals in ein Schwarzes Loch.

  1165. #1184 PDP10
    5. Februar 2019

    @Mysterion1000:

    Der Link ist zwar interessant aber die eigentliche Frage wird da nicht beantwortet.

    Sag ich ja. Aber eine bessere Quelle habe ich im Moment auch nicht. Wenn ich eine finde, lasse ich es dich wissen 🙂

    landet man in der Vorhölle Satzkonstruktionen, die dazu gedacht ist, einem zu quälen.

    Nee. Dann landet man bei Algebra und Gruppentheorie, dem Werkzeugkasten der Elementarteilchenphysiker. Ich schrieb ja schon, dass mich das Thema an meine intellektuellen Grenzen gebracht hat 🙂

  1166. #1185 Karl-Heinz
    6. Februar 2019

    @PDP10

    In dem es darum geht, wie die Neutrinos ihre Masse “bekommen” haben.
    Aber nix da beantwortet die eigentlich Frage in Bezug auf das Higgs-Feld.

    Die meisten Teilchen wie die Quarks, aus denen die Protonen und Neutronen des Atomkerns bestehen, erwerben ihre Masse durch Wechselwirkung mit dem HiggsFeld, das zu dem am LHC entdeckten Higgs-Boson gehört. Doch dieser Mechanismus wirkt nur bei Teilchen, die sowohl rechtshändig als auch linkshändig vorkommen – das heißt, deren Spin sowohl parallel als auch antiparallel zur Bewegungsrichtung orientiert sein kann. Bisher hat man nur linkshändige Neutrinos gefunden. Stammte ihre Masse vom HiggsFeld, müsste es auch rechtshändige Neutrinos geben. Da solche Teilchen noch nie beobachtet wurden, dürften sie, falls vorhanden, mit keinerlei anderen Teilchen und Feldern wechselwirken – und das erscheint vielen Physikern sehr unwahrscheinlich. Außerdem: Würden die Neutrinomassen vom Higgs-Mechanismus verursacht, dann sollten sie der Theorie zufolge mit anderen Teilchenmassen vergleichbar sein. Doch Neutrinos sind ungeheuer leicht; selbst das winzige Elektron hat gewiss mehr als 100000-mal mehr Masse. »Kaum jemand glaubt, dass der Higgs-Mechanismus den Neutrinos Masse verleiht«, meint Fermilab-Direktor Nigel Lockyer.»Wahrscheinlich liegt es an einem völlig anderen Effekt, an dem neue Mitspieler beteiligt sind.«
    Quelle: Spektrum Kompakt Neutrinos

  1167. #1186 Mysterion1000
    Salzburg
    6. Februar 2019

    @PDP10 Bei solchen Formulierungen, bin ich schon im Schattenreich. Das ist schon jenseits von gut und böse. Ich habe jemanden gefragt, der kurz davor ist, sein Mathestudium abzuschließen und gehört zu den Jahrgangsbesten und er hat gesagt, dass die Begriffe zu speziell sind und er hätte bei der Wahlauswahl sich in Algebra vertiefen müssen, um überhaupt zu verstehen, wobei es sich dabei handelt.

  1168. #1187 Dieter Schläger
    Deisenhausen
    6. Februar 2019

    @Alderamin
    um meine Frage genauer zu spezifizieren: Ohne elektromagnetische Wechselwirkung sollte es bei der DM weder einen Druck, noch eine Temperatur geben. Es könnten sich die Teilchen also beliebig nahe kommen. Müsste sich da nicht im laufe der Zeit eine unheimliche Dichte einstellen, auch dann, wenn die Teilchen ihre durch die Gravitation gewonnene Energie nicht abbauen können. Sie würden ja beim Wegstreben vom Potentialtopf wieder langsamer und dann irgendwann wieder der gravitativen Anziehung folgend zurückfallen.

  1169. #1188 PDP10
    6. Februar 2019

    @Karl-Heinz:

    Erstens …

    Bisher hat man nur linkshändige Neutrinos gefunden. Stammte ihre Masse vom HiggsFeld, müsste es auch rechtshändige Neutrinos geben.

    … beantwortet das die Frage irgendwie so gar nicht. Wieso, weshalb, warum (wer nicht fragt bleibt dumm … dideldidum 🙂 ) hängt das mit der Händigkeit zusammen?

    Und zweitens scheint dein Spektrum-Sonderheft ein wenig älter zu sein.

    Jedenfalls steht hier:

    “Für Neutrinos konnte experimentell lange keine Masse nachgewiesen werden. Weil sie nur schwach mit Materie wechselwirken, nahm man an, dass es nur linkshändige Neutrinos und rechtshändige Antineutrinos gibt. Aus der Entdeckung der Neutrinooszillationen lässt sich ableiten, dass Neutrinos eine nicht verschwindende Masse besitzen. Daraus folgt nach aktuellem physikalischem Verständnis, dass es auch rechtshändige Neutrinos und linkshändige Antineutrinos geben muss. Eine weitere Folge einer endlichen Masse ist, dass Neutrinos sich nicht ganz mit Lichtgeschwindigkeit bewegen.”

    … alles nicht so einfach *seufz*.

    Ich schließe mich @Peters und @Mysterion1000s Wunsch an: Ein aktuelles populärwissenschaftliches Werk über Elementarteilchenphysik zu haben, wäre jetzt nicht schlecht. Ich kenne nur keins …

  1170. #1189 Karl-Heinz
    6. Februar 2019

    @PDP10

    Und zweitens scheint dein Spektrum-Sonderheft ein wenig älter zu sein.

    Nö, …

    Erscheinungsdatum der Ausgabe: 27.08.2018
    Ich muss aber zugeben, dass ich bis jetzt nur einen kurzen Blick reingeworfen habe.

  1171. #1191 Alderamin
    6. Februar 2019

    @Dieter Schläger

    Ohne elektromagnetische Wechselwirkung sollte es bei der DM weder einen Druck, noch eine Temperatur geben.

    Man spricht schon von einer Temperatur, deswegen “kalte”, “warme” und “heiße” Dunkle Materie. Gemeint ist die Geschwindigkeit, mit der die Teilchen umhersausen. (Gewöhnliche) Neutrinos sind heiße Dunkle Materie, die sind mit annähernd Lichtgeschwindigkeit unterwegs und werden deshalb von Galaxienhaufen nicht festgehalten (schneller als deren Fluchtgeschwindigkeit). Ein Grund, warum sie nicht als die Dunkle Materie taugen.

    Es könnten sich die Teilchen also beliebig nahe kommen.

    Sehr nahe. Die Schwache Wechselwirkung (wenn die DM sie denn spürt) hat eine sehr kurze Reichweite (kleiner als ein Atomkern). Die elektromagnetische Kraft wirkt unendlich weit (nimmt natürlich mit dem Quadrat der Entfernung ab). Atome stoßen spätestens mit ihren Elektronenhüllen zusammen, die sind viel größer als die Kerne. Um nicht von Staubteilchen zu reden.

    Müsste sich da nicht im laufe der Zeit eine unheimliche Dichte einstellen, auch dann, wenn die Teilchen ihre durch die Gravitation gewonnene Energie nicht abbauen können. Sie würden ja beim Wegstreben vom Potentialtopf wieder langsamer und dann irgendwann wieder der gravitativen Anziehung folgend zurückfallen.

    Sie würden langsamer und zurückfallen, das ist ja auch der Grund, warum sie um Galaxien Halos bildet. Nur werden sie eben immer mit ein paar 10 bis 100 km/s unterwegs sein und nie zum Stillstand kommen. Die Teilchen kreisen einfach wie die Sterne in einem Kugelsternhaufen um das Zentrum ihrer gemeinsamen Masse. Es gibt keinen Dämpfungsmechanismus, der aus diesem System insgesamt Energie abführen könnte, so dass es dichter zusammen rückt. Im Gegenteil, in einem Artikel habe ich neulich beschrieben, wie sogar die Materie über die Schwerkraft auf die Dunkle Materie zurück wirken und sie auseinander treiben kann.

    Die Bewegung der Teilchen ist wie die einer Schaukel oder eines Pendels ohne Reibung und Luftwiderstand. Sie werden mal langsamer, mal schneller, im ständigen Austausch von potenzieller und kinetischer Energie. Solange sie die Energie nicht loswerden, treibt es sie immer wieder in die Ferne bis zum Umkehrpunkt.

    Und sollten sie ihre eigenen Antiteilchen sein, dann wäre eine Frontalkollision sogar tödlich für sie, die Teilchen würden sich vernichten und zu Gammastrahlung werden, die die DM-Wolke verlassen würde. Dann würde mit der Zeit ihre Masse abnehmen und sie sich eher weiter verteilen, als verdichten.

  1172. #1192 Peter
    6. Februar 2019

    Zitat Alderamin: „sollten sie ihre eigenen Antiteilchen sein, dann wäre eine Frontalkollision sogar tödlich für sie, die Teilchen würden sich vernichten und zu Gammastrahlung werden, die die DM-Wolke verlassen würde.“

    Falls die Dunkle-Materie-Teilchen wirklich ihre eigenen Antiteilchen sind und sich vernichten, müsste die dabei entstehende Annihilationsstrahlung dann nicht von unseren Hochenergie-Weltraumteleskopen gemessen werden können?
    Hochenergetische elektromagnetische Strahlung, die von einem Ort kommt, wo gar keine sichtbare Materie nachweisbar ist, sollte doch auffallen.

    Und müsste hochenergetische elektromagnetische Strahlung dann nicht auch Paare aus Dunkle-Materie-Teilchen bilden können?
    An Teilchenbeschleunigern kommt doch hochenergetische elektromagnetische Strahlung vor und falls durch diese Strahlung Paare aus Dunkle-Materie-Teilchen gebildet werden, dann könnten diese Teilchen doch detektiert werden.

  1173. #1193 PDP10
    6. Februar 2019

    @Peter:

    Falls die Dunkle-Materie-Teilchen wirklich ihre eigenen Antiteilchen sind und sich vernichten, müsste die dabei entstehende Annihilationsstrahlung dann nicht von unseren Hochenergie-Weltraumteleskopen gemessen werden können?

    Die Idee gabs schon. Darum hat man damals den AMS Detektor auf die ISS geschafft. Hat aber leider keine eindeutigen Ergebnisse gebracht:

    https://www.deutschlandfunk.de/iss-experiment-dunkle-materie-noch-nicht-nachweisbar.676.de.html?dram:article_id=424070

  1174. #1194 PDP10
    6. Februar 2019

    @Karl-Heinz:

    Erscheinungsdatum der Ausgabe: 27.08.2018

    Von wann ist denn der Artikel aus dem du zitiert hast?

    Meiner Erfahrung nach werden in den Spektrum Sonderheften auch oft Artikel reingepackt, die schon ein paar Jährchen alt sind und nicht auf dem neuesten Stand.

  1175. #1195 Karl-Heinz
    Graz
    6. Februar 2019

    @PDP10

    Hmm…
    Es könnte dieser Artikel sein.
    https://www.scientificamerican.com/article/the-neutrino-puzzle/

    19.09.2017 · The Neutrino Puzzle. Clara Moskowitz … It will be the biggest neutrino experiment on the planet. It will also mark the …

    Im Spektrum fängt der Artikel so an.
    Während ich in einer riesigen Höhle stehe und über eine Balustrade auf eine Fülle technischer Geräte hinunterblicke, jagen durch jeden Zentimeter meines Körpers sekündlich Billionen nahezu lichtschnelle Neutrinos. Allerdings spüre ich davon nichts – die fast masselosen Teilchen durchqueren den leeren Raum zwischen meinen Atomen, ohne eine Spur zu hinterlassen. Selbst in der busgroßen Metallkiste inmitten der Höhle kollidiert nur einige Male pro Tag ein Neutrino mit …

  1176. #1196 PDP10
    6. Februar 2019

    @Karl-Heinz:

    Ich dachte, du hast das Heft vor dir und kannst mal kurz nachsehen … 🙂

  1177. #1197 Karl-Heinz
    6. Februar 2019

    @PDP10

    Die Artikeln selbst sind leider nicht mit einem Datum versehen. Aber es gibt ja mehrere Artikeln.

    Frage: Hat man schon rechtshändige Neutrinos nachgewiesen oder gibt’s die nur in der Theorie?

  1178. #1198 Niels
    7. Februar 2019

    @Karl-Heinz

    Das Problem ist, dass man rechtshändige Neutrinos grundsätzlich nur indirekt nachweisen kann.
    Rechtshändige Neutrinos müssten sogenannte sterile Neutrinos sein, d.h. sie wechselwirken ausschließlich über die Gravitation und über keine der anderen Kräfte.

    Genau deswegen kommen sie ja als Bestandteile der dunklen Materie in Frage.

    Es gibt mittlerweile mehrere große Experimente, die Neutrinooszillation zu messen versuchen.
    Ich kenne Double Chooz (Frankreich), RENO (Südkorea), Daya Bay (China), MINOS (USA) und T2K (Japan).

    An diesen Experimenten wird immer mal wieder etwas gemessen, dass als Hinweis auf die Existenz rechtshändiger Neutrinos aufgefasst werden kann.
    Dann widersprechen anderen Forscher dieser Interpretation der Messung. Dann liefert eine andere Messung eines konkurrierenden Experiments ein ganz anderes Ergebnis. Daraufhin findet wieder ein anderes Experiment einen ganz anderen heißen Hinweis, der aber leider dem ersten widerspricht…

    Vielleicht kommt da demnächst etwas konkretes, kann aber auch sehr gut sein dass das alles Quatsch war.
    Siehe die zig Leaks und Meldungen der letzten Jahre vom LHC, dass man jetzt endlich demnächst mit fast absoluter Sicherheit neue Physik nachweisen wird.
    Was sich leider alles nach genaueren Messungen und Datenauswertungen zerschlagen hat.

  1179. #1199 Noonscoomo
    Berlin
    7. Februar 2019

    @Alderamin
    Sollte dunkle Materie nicht Energie durch Gravitationswellen verlieren?

  1180. #1200 Karl-Heinz
    7. Februar 2019

    @Niels

    Danke

    @Noonscoomo
    Unsere Milchstraße zum Beispiel besteht wie alle Spiralgalaxien aus dem Bulge,
    der Scheibe (dicke und dünne Scheibe), sowie dem Halo, der
    näherungsweise als isotherme Kugel angesehen werden kann.
    • Diesem wird die Masse der dunklen Materie zugerechnet.
    • Es handelt sich um ein stoßfreies System und die Sterne und das
    Gas bewegen sich im gemeinsamen Gravitationspotential.
    • Das System relaxiert durch Phasenmischung. Relaxiert heisst, dass sich dass sich das ganze System in einem Gleichgewichtszustand befindet.
    Und selbst wenn es pulsieren würde, so würden nach einem bestimmten Theorem keine Gravitationswellen abgestrahlt.

  1181. #1201 Noonscoomo
    Berlin
    7. Februar 2019

    @Karl-Heinz
    Öh, hab ich nich verstanden.
    Irgendwie sehe ich da keine Erklärung über “is halt so” hinaus.

  1182. #1202 Karl-Heinz
    7. Februar 2019

    @Noonscoomo

    Die Dichte des Halos (Dunkle Materie) ist durch die Phasenmischung heutzutage zeitlich konstant. Wenn du aber ein einzelnes Teilchen der Dunklen Materie betrachtest, so handelt es sich um ein dynamisches System, ähnlich einem Masse Federsystem, welches zum Schwingen neigt.
    Meine Vermutung: Das Halo selbst hat sicher einen Drehimpuls, denn sonst würden die Teilchen durch das Zentrum sausen und unter Umständen im Schwarzen Loch landen.

  1183. #1203 Noonscoomo
    Berlin
    7. Februar 2019

    @Karl-Heinz
    Hä? Versteh ich gerade krass was nicht oder passt die Antwort wirklich nicht zu meiner Frage?
    Ich hätte erwartet, dass dunkle Materie, die um ein SL kreist genau so Energie verlieren müsste wie jede andere Masse, indem sie (seeehr kleine) Gravitationswellen erzeugt.
    Kann ja gut sein dass der Energieverlust so irre klein ist, dass es billionen Jahre dauert. Oder erzeugen die wirklich keine Gravitationswellen? Und wenn nein, wo ist der Unterschied zu anderen massebehafteten Objekten?

  1184. #1204 Alderamin
    7. Februar 2019

    @Niels

    Es gibt mittlerweile mehrere große Experimente, die Neutrinooszillation zu messen versuchen.
    Ich kenne Double Chooz (Frankreich), RENO (Südkorea), Daya Bay (China), MINOS (USA) und T2K (Japan).

    Und MiniBooNE!

  1185. #1205 Karl-Heinz
    7. Februar 2019

    @Noonscoomo

    Von außen betrachtet hat das Halo ein konstante Dichte über die Zeit.
    Dir Raumkrümmung bleibt dadurch konstant und es werden keine Gravitationswellen ausgesandt. Selbst wenn eine kugelförmiges Halo pulsieren würde, werden keine Gravitationswellen ausgesandt.

  1186. #1206 Alderamin
    7. Februar 2019

    @Noonscoomo

    Die Abstrahlung von Gravitationswellen ist umso stärker, je stärker die Massen beschleunigt werden und je größer sie sind. Deswegen strahlen Schwarze Löcher kurz vor der Verschmelzung, wenn sie sich hunderte Male pro Sekunde umkreisen, ganze Sonnenmassen an Energie ab, die Erde beim Umlauf um die Sonne aber nur ein paar Watt. Wenn so ein winziges DM-Teilchen in hunderten Millionen Jahren seine Bahn um das Milchstraßenzentrum zieht, dann ist das ein noch viel gemächlicherer Umlauf, da sollte Gravitationswellenabstrahlung keinerlei Rolle spielen.

  1187. #1207 Ernst Haugeneder
    Wien
    7. Februar 2019

    @ Peter #1174 vom 4.2.2019
    Deinen Ausführungen entnehme ich, dass eine Rotationszeit von ca. 8 Stunden einer Situation ohne Mond von Anfang an entspricht. Das wurde in den von mir genannten “Zitaten” nicht genau genug definiert.
    Wenn jedoch, theoretisch, der Mond heute plötzlich verschwinden würde, würde sich die Tageslänge nicht weiter ändern, weil die Gezeiten zwar nicht mehr auftreten aber sich das Trägheitsmoment der Erde nur unwesentlich ändern würde? Ein Piruetteneffekt wäre ja nicht gegeben. Aufgrund fehlender äußerer Einflüsse (abgesehen von der Sonne) bliebe die Rotation etwa konstant, ok?

  1188. #1208 Noonscoomo
    Berlin
    7. Februar 2019

    @Alderamin
    Danke, damit kann ich was anfangen.
    Es ist also wirklich so, dass die Dinger bedeutungslos wenig Energie verlieren und daher ewig und Kugelförmig um das gemeinsame Massezentrum kreisen.

  1189. #1209 PDP10
    7. Februar 2019

    @Ernst Haugeneder:

    Wenn jedoch, theoretisch, der Mond heute plötzlich verschwinden würde, würde sich die Tageslänge nicht weiter ändern, weil die Gezeiten zwar nicht mehr auftreten aber sich das Trägheitsmoment der Erde nur unwesentlich ändern würde?

    Es würde sich eine ganze Menge ändern. Hier ist das Ganze einigermaßen ausführlich erklärt:

    https://www.spektrum.de/frage/was-passiert-wenn-der-mond-verschwindet/1223162

  1190. #1210 Ernst Haugeneder
    Wien
    8. Februar 2019

    @PDP10
    Genau diesen Beitrag verstehe ich nicht. Die Flutberge der Ozeane betragen doch nur einige -zig Meter (in Le Havre zB 20m – 30m). Auch über die großen Flächen der Ozeane ergibt der Wegfall der Flutberge und -Täler nur eine kleine Reduktion des Trägheitsmomentes. Das führt zu einer kurzfristigen Beschleunigung der Drehung. Aber wenn sich alles beruhigt hat, gibt es keine Änderung mehr: Woher kommt die fortwährende Beschleunigung?

  1191. #1211 Niels
    8. Februar 2019

    @Karl-Heinz

    Und selbst wenn es pulsieren würde, so würden nach einem bestimmten Theorem keine Gravitationswellen abgestrahlt.

    Heißt übrigens Birkhoff-Theorem.
    Gilt allerdings nur für sphärisch symmetrische Pulsation.

    Noonscoomo schrieb:
    Hä? Versteh ich gerade krass was nicht oder passt die Antwort wirklich nicht zu meiner Frage?
    Ich hätte erwartet, dass dunkle Materie, die um ein SL kreist genau so Energie verlieren müsste wie jede andere Masse, indem sie (seeehr kleine) Gravitationswellen erzeugt.

    Karl-Heinz schrieb:
    Von außen betrachtet hat das Halo ein konstante Dichte über die Zeit.

    Konstante Dichte über die Zeit ist für das Birkhoff-Theorem nicht das Entscheidende.
    Es geht vielmehr darum, ob ein Vorgang sphärisch symmetrisch stattfindet.

    Geht man davon aus, dass die im Halo kreisende dunkle Materie (oder die um ein schwarzes Loch kreisende) dies insgesamt sphärisch symmetrisch tut?
    Kommt mir zwar nicht unplausibel vor, ich habe davon aber leider keine Ahnung. Tut mir leid.

    .

    Alderamin weiß das aber bestimmt?

    .

    @Alderamin

    Und MiniBooNE!

    Danke!
    Ist vielleicht sogar das wichtigste, wenn es um die mögliche Entdeckung von sterilen Neutrinos geht.
    Darf man deswegen auf keinen Fall vergessen.

  1192. #1212 Karl-Heinz
    8. Februar 2019

    @Niels

    Danke für die Antwort.

    @Noonscoomo

    Hä? Versteh ich gerade krass was nicht oder passt die Antwort wirklich nicht zu meiner Frage?
    Ich hätte erwartet, dass dunkle Materie, die um ein SL kreist genau so Energie verlieren müsste wie jede andere Masse, indem sie (seeehr kleine) Gravitationswellen erzeugt.

    Damit ein System Gravitationswellen abstrahlt muss die Massenverteilung ein zeitlich veränderliches Multipolmoment von mindestens Quadrupolcharakter besitzt. Ich gehe mal davon aus, dass du das sowieso nicht vetstehst. Nur soviel. Ein rotierendes schwarzes Loch würde keine Gravitationswellen aussenden. Ein Halo welches nicht mindest ein Quadrupolmoment hat, sendet ebenfalls keine Gravitationswellen aus, du Banause.

  1193. #1213 Alderamin
    8. Februar 2019

    @Niels

    Geht man davon aus, dass die im Halo kreisende dunkle Materie (oder die um ein schwarzes Loch kreisende) dies insgesamt sphärisch symmetrisch tut?

    Die Bahnen der DM Teilchen dürften, soweit sie nicht durch Sterne gestört werden, eher langgezogene Ellipsen sein, sie bildet ja keine Akkretionsscheiben, sondern fällt unter dem Einfluss der eigenen Schwerkraft auf ein Massenzentrum zu und wird nur durch gravitative Interaktion mit sichtbarer Materie und durch den Verlust der schnellsten Teilchen abgebremst.

  1194. #1214 Noonscoomo
    Berlin
    9. Februar 2019

    @Karl-Heinz
    Oh, ok, alles klar. Warum lässt du dich denn dann herab, um mir überhaupt zu antworten? Worum geht’s dir beim Antworten? Darum, darzustellen, was du alles tolles weißt, oder darum, jemandem dabei zu helfen einen Sachverhalt zu verstehen?
    Weißt du, wenn jemand was nicht versteht kann’s auch an dem liegen, der‘s zu erklären versucht.

  1195. #1215 Alderamin
    9. Februar 2019

    @Niels

    Ach so, darum ging’s, insgesamt ist der DM-Halo drehsymmetrisch, kugelrund oder ein Rotationsellipsoid (Navarro-Frenk-White-Pofil). Die einzelnen Teilchen bewegen sich aber auf unsymmetrischen, langgestreckten Bahnen. Könnten die also auch dann nicht (wenigstens theoretisch in kleinem Umfang) Gravitationswellen abstrahlen, weil sich die Gesamtverteilung nicht ändert? Ist ja was anderes als bei einem massiven rotierenden Objekt.

    @Karl-Heinz

    Deine Schnippigkeit geht zu weit, wenn Du andere indirekt als dumm bezeichnest.

  1196. #1216 Niels
    9. Februar 2019

    @Alderamin
    Klar könnten die Gravitationswellen aussenden. Tun sie offenbar auch.

    Sie täten das nur genau dann nicht, wenn das ganze System perfekt symmetrisch (sphärisch symmetrisch oder eben rotationssymmetrisch) wäre.
    Wenn ein DM-Halo ein Navarro-Frenk-White-Pofil hat, ist das wohl im Allgemeinfall nicht gegeben.

    Auch die Erde, die Sonne und sogar Neutronensterne strahlen zumindest theoretisch aufgrund ihrer Rotation, da sie eben nie perfekte Symmetrie haben.

    Könnten die also auch dann nicht (wenigstens theoretisch in kleinem Umfang) Gravitationswellen abstrahlen, weil sich die Gesamtverteilung nicht ändert? Ist ja was anderes als bei einem massiven rotierenden Objekt.

    Weiß ich ehrlich gesagt nicht.
    Ein massives Objekt ist auch aus einzelnen Teilchen aufgebaut. Es sind nur so viele, dass man das zu einer kontinuierlichen Massenverteilung vereinfacht.
    Dadurch hat man dann perfekte Symmetrie.
    (Wie erwähnt, in der Realität ist sowieso nichts perfekt symmetrisch.)

    Nehmen wir mal an, statt des massiven Objektes betrachten wir einen Schwarm dunkler Materie, dessen Einzelteilchen um einen Mittelpunkt kreisen.
    Ab welcher Teilchendichte ist dann die Näherung als kontinuierliche Massenverteilung so gut, dass wir von praktisch perfekter Symmetrie ausgehen können?
    Keine Ahnung.

    .

    @Karl-Heinz

    Damit ein System Gravitationswellen abstrahlt muss die Massenverteilung ein zeitlich veränderliches Multipolmoment von mindestens Quadrupolcharakter besitzt.

    Na ja, ich finde es überhaupt nicht trivial, dass kreisende dunkle Materie kein zeitlich veränderliches Multipolmoment von mindestens Quadrupolcharakter aufweisen kann.
    Ist offenbar ja auch falsch, ein Navarro-Frenk-White-Pofil besitzt im Allgemeinfall wohl immer auch höhere nicht-konstante Multipolmomente.

    Ein theoretisches rotierendes schwarzes Loch, dass durch die Kerr-Metrik beschrieben wird, sendet tatsächlich keine Gravitationswellen aus.
    Bei tatsächlich exisiterenden schwarzen Löchern samt Akkretionsscheibe sieht das aber eben auch schon wieder anders aus.
    Auch sonst gibt es im realen Weltall höchstwahrscheinlich keine Köper, die die nötige perfekte Symmetrie aufweisen. Daher werden sie immer winzigste Mengen rotationsverursachter Gravitationswellen abstrahlen.

    Ich gehe mal davon aus, dass du das sowieso nicht vetstehst. Nur soviel.
    […] du Banause.

    War wahrscheinlich eher scherzhaft gemeint?
    Das Problem dabei ist einfach, dass das bei rein geschriebenem Text leider extrem leicht misszuverstehen ist und deswegen praktisch immer falsch aufgefasst wird…

    Vielleicht wäre einfach ein kurzer klärender Kommentar dazu ganz hilfreich?

  1197. #1217 Alderamin
    9. Februar 2019

    @Niels

    Ein massives Objekt ist auch aus einzelnen Teilchen aufgebaut. Es sind nur so viele, dass man das zu einer kontinuierlichen Massenverteilung vereinfacht.

    Der wesentliche(?) Unterschied ist, dass bei einem massiven Objekt Kreisbahnen um die Achse vorliegen, bei einem Halo angenäherte Ellipsenbahnen um das Zentrum.

    Angenähert, weil die wirksame Schwerkraft nach innen geringer wird (Schalentheorem) und die Teilchen an Sternen gestreut werden können.

    Jedes Teilchen für sich betrachtet müsste eigentlich ein wenig (irrelevant wenig, um den Halo schrumpfen zu lassen) GW abstrahlen – löschen sich die Wellen von allen gegenseitig wieder aus?

  1198. #1218 Alderamin
    9. Februar 2019

    @myself

    Blödes Tablet, jetzt ist oben genau das zitiert, was nicht zitiert gehört und umgekehrt…

  1199. #1219 Karl-Heinz
    9. Februar 2019

    @Niels

    War wahrscheinlich eher scherzhaft gemeint?
    Das Problem dabei ist einfach, dass das bei rein geschriebenem Text leider extrem leicht misszuverstehen ist und deswegen praktisch immer falsch aufgefasst wird…

    Stimmt es ist scherzhaft gemeint. Habe mich ein bisschen geärgert, dass meine Antwort an Noonscoomo von Noonscoomo so verrissen wurde. Bin aber eh selber schuld, dass die Antwort aus Zeitgründen so dilettantisch formuliert war.
    Damit bitte ich Noonscoomo um Entschuldigung. War nicht so gemeint.

    In der Elektrotechnik gibt es ja so was wie ein Nah und Fernfeld. Angenommen ich betrachte ein Teilchen Dunkler Materie, dass gemeinsam mit der Erde um die Galaxie kreist. Ein galaktisches Jahr = 225 Millionen Jahre. Die Gravitationswellenlänge müsste in diesem Fall gigantisch groß sein. Überschlagen würde ich sagen, die ganze Galaxie liegt sicher im Nahfeld. Jetzt muss ich mal scharf überlegen, ob im Nahfeld Energie transportiert wird. Weiß das zufällig jemand?

    @Noonscoomo
    Friede? 😉

  1200. #1220 Niels
    9. Februar 2019

    @Alderamin

    Der wesentliche(?) Unterschied ist, dass bei einem massiven Objekt Kreisbahnen um die Achse vorliegen, bei einem Halo angenäherte Ellipsenbahnen um das Zentrum.

    Ich glaube, du hast recht.
    Habe mich selbst verwirrt, weil ich annahm, dass bei der Überlagerung unzähliger Ellipsenbahnen in drei Dimensionen etwas mit sphärischer Symmetrie rauskommt.

    Ist aber falsch, das Ganze hat zeitlich veränderliche Multipolmomente. Deswegen muss es auch strahlen.
    Oder bin ich immer noch verwirrt?
    Irgendwie setzt meine Vorstellung bei Ellipsen-Überlagerungen gerade völlig aus…

    löschen sich die Wellen von allen gegenseitig wieder aus?

    Kann man so eigentlich nicht sagen.
    Man muss in der ART immer die resultierende Gesamt-Raumzeit anschauen, da die ART nicht-linear ist.
    Deswegen kann man das Ganze nicht wie etwa bei der Elektrodynamik in einzelne Strahler aufteilen und einfach linear überlagern.

  1201. #1221 Karl-Heinz
    9. Februar 2019

    @Niels

    Kann man so eigentlich nicht sagen.
    Man muss in der ART immer die resultierende Gesamt-Raumzeit anschauen, da die ART nicht-linear ist.
    Deswegen kann man das Ganze nicht wie etwa bei der Elektrodynamik in einzelne Strahler aufteilen und einfach linear überlagern.

    Wenn die Gravitationswellen nicht zu stark sind kann man ohne schlechtem Gewissen linear rechnen, d.h. linear überlagern (um Schelte zu vermeiden: meine Meinung, die aber auch falsch sein kann 😉 ).

  1202. #1222 Noonscoomo
    Berlin
    9. Februar 2019

    @Karl-Heinz
    Ok, hast recht, ich habs auch bisschen provoziert. Sorry. Schwamm drüber.
    Ich fasse mal zusammen was bei mir angekommen ist:
    Da gibt es eine Kugelförmige Wolke in locker mal Galaxiengrösse die nach aussen immer dünner wird und deren einzelne „Teilchen“ jedes für sich unbeeinflusst von irgendwas elipsoid um ein gemeinsames Zentrum kreisen. Jedes für sich kreist so langsam, dass man dessen Gravitationswellen einfach mal vernachlässigen kann aber alle zusammen strahlen dann doch irgendwie, weils ja dann doch ne echt grosse Menge ist und näher am Zentrum schneller unterwegs ist als aussen.
    Richtig soweit?

  1203. #1223 Noonscoomo
    Berlin
    9. Februar 2019

    Wenn der leere Raum ein Energiepotenzial hat das grösser Null ist. Sollte er dann nicht auch eine Masse grösser Null haben und damit auch eine gravitative Wirkung? Die Energie des Raumes kennen wir nicht sehr gut, aber ich finde:
    DE = DM löst das Problem doch sehr elegant!

  1204. #1224 Alderamin
    9. Februar 2019

    @Noonscoomo

    DE = DM löst das Problem doch sehr elegant!

    Nein, denn DE stößt ab, DM zieht an. Und DM verdichtet sich zu großen Strukturen (Halos, Filamente), während DE eine Eigenschaft des leeren Vakuums zu sein scheint, also überall gleich stark wirkt.

    Schwerkraft wird nicht nur von Masse erzeugt, sondern auch von Druck, und Druck kann negativ sein. Negativer Druck erzeugt abstoßende Schwerkraft. DE verhält sich so, als ob das Vakuum einen negativen Druck hätte. Der allerdings bis heute nicht aus der Quantenphysik heraus erklärbar ist. Schon gar nicht, dass er sich seit kurz nach dem Urknall geändert zu haben scheint.

  1205. #1225 Karl-Heinz
    9. Februar 2019

    @Noonscoomo
    Freu 😉
    Ich finde es total interessant, wenn diskutiert wird. Sehr oft ist es so, dass eine Frage die andere ergibt. Zu Beginn habe ich darüber nach gedacht, welche Form, Größe und Dichteverteilung das Dunkle Halo haben könnte. Dann fängt man zu Stöbern an und erfährt, dass die Teilchen des Halos als stoß und reibungsfreie Teilchen aufgefasst werden. Es dauert nicht lange, man fragt nach der Eigengeschwindigkeit der Teilchen und da sie ja stoß und reibungsfrei sind, nach deren Bahn. Anschließend ist auch noch deine Frage hinzugekommen, ob einzelne Teilchen, wenn sie schon das Zentrum umrunden, Gravitationswellen abgeben sollten und daher ein klein wenig Energie verlieren müssten. Zu deiner Frage. Was ich so gelesen habe, ist dass die DM-Halos um einiges größer ist als die Galaxie selbst, die sie umgibt. Wie gesagt, ein sehr interessantes Thema. Dann tauchen noch weitere Fragen auf, die natürlich auch noch erörtert werten müssen. Nachdem die Teilchen stoßfrei sind können sie nicht direkt ihren Impuls austauschen, aber sehr wohl über das gemeinsame Gravitationspotential. Dann fragt man sich auch ob das Halo schon immer statisch war oder es früher mal eine Dynamik gegeben hat. Nach langem suchen findet man endlich so was wie “System relaxiert durch Phasenmischung” will das mitteilen, teil es aber so ungeschickt mit, dass man voll ins Fettnäpfchen tritt.
    So jetzt muss ich nachdenken, was es mit dem Nahfeld auf sich hat. 😉

  1206. #1226 Noonscoomo
    Berlin
    10. Februar 2019

    @Alderamin
    Das ist mir schon klar. Ein Vorzeichenproblem.
    Aber evtl. kriegen wir das ja weg.
    Zunächst mal, wenn der Raum selbst ein Energiepotenzial hat, dann hat er auch Masse und damit auch Gravitation. Korrekt soweit?
    Und ein sehr schlauer Mensch, Martin Bäker, schrieb mal irgendwo, dass es equivalent sei zu sagen, „der Raum dehnt sich aus“ und „die Materie (und damit auch unsere Messlatte) schrumpft.
    Wenn die Raumzeit also Gravitation hat und sich daher in und um die Galaxien sammelt und dort also dichter ist und mehr Gravitation hat und wenn wir jetzt noch annehmen, dass Gravitation Materie schrumpft, (und hierdurch die Raumzeit quasi im Verhältnis grösser wird) dann sieht das so aus als würde der Raum zwischen den Galaxien grösser und es sieht so aus als würde die Zeit langsamer und es sieht so aus als würde zusätzlich irgend eine geheimnisvolle Materie im Spiel sein die die Galaxien zusammen hält.
    Ich gehe also noch einen Schritt weiter und sage:
    Das Energiepotenzial des leeren Raumes = DE = DM
    Aber vermutlich ist da ein schlimmer Denkfehler drin…

  1207. #1227 Mysterion1000
    Salzburg
    10. Februar 2019

    @PDP10
    Auf der Suche nach Antworten bezüglich Neutrinothematik, habe ich dieses Video gefunden. Hier geht es zwar nicht um die Fragestellung die wir und gestellt haben, ist aber sehr interessant zusammengefasst. Die Dame, die es vorträgt schafft es nämlich sehr verständlich darzulegen, wie man versucht beim Katrin Experiment in Karlsruhe die Neutrinomasse zu bestimmten. Die ersten Minuten kannst du eigentlich vorspulen, da sie dort nichts sagt, was man nicht weiß, wenn man sich ein wenig mit Physik beschäftigt. Ich war tatsächlich erstaunt darüber, dass ich in diesen Vortrag wirklich alles auf Anhieb verstanden habe.

  1208. #1228 Noonscoomo
    Berlin
    10. Februar 2019

    Vielleicht wird es verständlicher wenn ich das so ausdrücke.
    Das Universum ist (von einem nicht mitschrumpfenden Beobachter aus gesehen) heute noch immer genau so gross wie beim Urknall, aber alle Materie darin schrumpft unaufhörlich zunächst sehr schnell, dann viel langsamer genau eben gerade wegen der Gravitation = Raum mit Energie drin, der sich aus Gründen ungleichmässig verteilt hat.
    Da, soweit ich das verstehe, die elementarsten Bausteine auch keinen Parameter „Grösse“ haben kann das ja auch beliebig weiter schrumpfen.
    Das Universum hatte also schon immer eine konstante Grösse und alles darin wird permanent immer kleiner und dichter (wie gesagt, vom nicht mitschrumpfenden Beobachter aus).
    Der mitschrumpfende Beobachter nimmt das natürlich nicht wahr. Der denkt er und seine Umgebung hätte konstante Grösse und der Raum dehne sich aus (was ja nicht weniger unerklärlich ist)
    Den Zustand zu t = 0 könnten wir also als völlig homogen beschreiben und völlig homogen ist ja genau so gut wie überall null, und heisenbergs Unschärfe hat da zu ner inhomogenität geführt und dadurch fing die Raumzeit an sich hier und da zu verdichten und die Energie und dann die Materie mit sich zu nehmen und zu verdichten.

  1209. #1229 Karl-Heinz FRUHMANN
    10. Februar 2019

    @Mysterion1000

    Cooler Link.

    Schon jetzt gibt es Abschätzungen auf Basis kosmologischer Beobachtungen, die eine Höchstgrenze von 120 Millielektronvolt setzen.
    Wenn das Neutrino aber zu leicht ist, wird man mit KATRIN nur eine Obergrenze von 200 Millielektronvolt setzen können. Macht aber nichts. Es ist total wichtig, dass eine Obergrenze auch im Labor bestimmt wird. Sind nämlich die Annahmen der Abschätzungen auf Basis kosmologischer Beobachtungen falsch, so ist die Abschätzungen wertlos. Das kann im Labor nicht passieren.

  1210. #1230 Bullet
    10. Februar 2019

    @Alderamin:

    Schwerkraft wird nicht nur von Masse erzeugt, sondern auch von Druck

    Den mußt du mir jetzt erklären.

  1211. #1231 Alderamin
    10. Februar 2019

    @Bullet

    Können andere viel besser. Achte auf die Diagonalelemente >00 im Energie-Impuls-Tensor.

  1212. #1233 Alderamin
    10. Februar 2019

    @Noonscoomo

    Das Universum ist (von einem nicht mitschrumpfenden Beobachter aus gesehen) heute noch immer genau so gross wie beim Urknall, aber alle Materie darin schrumpft unaufhörlich

    Es gibt eine Entfernungsskala, in der die Galaxien ihre Entfernungen untereinander beibehalten, genannt “mitbewegte Entfernung“. Man könnte auf Basis derselben sicherlich eine Physik formulieren, in der alles ständig schrumpft, außer den Entfernungen der Galaxien, aber diese Physik sähe sehr anders aus und wäre komplizierter als diejenige, mit der wir rechnen. Z.B. werden in unserer Physik Gase erwärmt, wenn man sie komprimiert. In der “mitbewegten Physik” würden sie nur wärmer, wenn sie schneller komprimierten, als die “natürliche Schrumpfung”. Die ART sähe dann auch anders aus.

    Oder anders gesagt, die Physik, so wie wir sie benutzen, passt nicht zu einem Universum, in dem alles schrumpft, außer dem Universum selbst.

  1213. #1234 Bullet
    11. Februar 2019

    @Alderamin, Karl-Heinz:
    danke für eure Links. Es tut mir in der Seele weh, und ich kann mich hoofentlich jetzt damit retten, daß ich gerade ganz andere Dinge tun muß, aber:
    ich habs nicht verstanden. Ich werde das so hinnehmen müssen. 🙁

  1214. #1235 Alderamin
    11. Februar 2019

    @Bullet

    Die verständliche (aber wie meistens nicht ganz korrekte) Variante geht so: Wenn Du ein Gas komprimierst und damit unter Druck setzt, musst Du Arbeit leisten und in dem Gas steckt dann mehr Energie. Diese Energie hat ein Masseäquivalent und dieses erzeugt eine Schwerkraft.

    Pulsare kollabieren genau wegen des hohen Drucks im Kern ab einer bestimmten Dichte unvermeidlich zum Schwarzen Loch (wobei Pulsare wie inkompressible Flüssigkeiten gerechnet werden – da hakt dann das obige Modell schon).

    Aber Druck kann auch negativ sein, und negativer Druck führt nach der Allgemeinen Relativitätstheorie zu abstoßender Schwerkraft. Man kann herleiten, dass eine positive Vakuumenergie einen negativen Druck verursacht. Genau so soll die Inflation funktioniert haben. Und möglicherweise funktioniert auch die Dunkle Energie so.

    Meine ich aber schon 3-4mal in diesem Blog kund getan zu haben.

  1215. #1236 Peter
    11. Februar 2019

    Wozu braucht man überhaupt eine Dunkle Energie?
    Könnte die beschleunigte Expansion des Universums nicht auch damit erklärt werden, dass durch die fortlaufende Expansion die durchschnittlichen Entfernungen zwischen großen Massen wie (Super-)Galaxienhaufen u. Galaxien immer größer werden? Die Bremsung der kosmischen Expansion durch die Gravitation wird also zunehmend geringer.
    Die Gravitationskraft nimmt ja mit 1/Entfernung^2 ab. Das Ergebnis müsste also eine mehr oder weniger exponentielle Expansion des Universums sein.

  1216. #1237 alex
    11. Februar 2019

    @Peter:
    “Das Ergebnis müsste also eine mehr oder weniger exponentielle Expansion des Universums sein.”

    Nein. Bei einer exponentiellen Expansion ist die Expansion immer beschleunigt (mit einer positiven Beschleunigung). In deinem Modell ist die Expansion aber gebremst (oder anders gesagt beschleunigt mit einer negativen Beschleunigung). Die Größe dieser Bremsung nimmt zwar mit der Zeit ab, aber sie ist immer eine Bremsung. Ohne Dunkle Energie (oder ähnliches) kommt es nie zu einer positiven Beschleunigung.

  1217. #1238 Jutta Grübler
    Aschach
    11. Februar 2019

    Hallo Florian könnte ich mir die Dunkle Energie im Universum so vorstellen wie Gluonen in einem Proton
    LG Josef

  1218. #1239 Florian Freistetter
    11. Februar 2019

    @Josef: Ich vermute mal nicht… “Dunkle Energie” ist nur ein Begriff um die Tatsache zu beschreiben, dass sich das Universum immer schneller ausdehnt. Die Ursache dafür ist noch unbekannt.

  1219. #1240 Bullet
    12. Februar 2019

    @Alderamin: okay, das Beispiel versteh ich. Selbst mit dem Modell einer inkompressiblen Flüssigkeit. Soweit hilfts. Aber: “Druck kann auch negativ sein”? So wie negative Kelvin?

  1220. #1241 Alderamin
    12. Februar 2019

    @Bullet

    Ja, man kann auch etwas auseinanderziehen, was von sich aus zusammensacken will.

  1221. #1242 Noonscoomo
    Berlin
    12. Februar 2019

    @Alderamin
    Die Mitbewegte Entfernung gefällt mir.
    Und ja, klar, die Physik ist bei schrumpfender Materie komplizierter, aber die aktuelle tut’s ja halt nicht, drum kann es ja gut sein, dass wir eine kompliziertere brauchen.
    Letztlich ist es einem ja aber selbst überlassen zu wählen, welche Betrachtung man wählt, wenn beide Betrachtungen äquivalent sind. Mir geht’s ja in erster Linie um die Anschaulichkeit und ich finde es viiiiiel anschaulicher, mir vorzustellen, wie Gravitation sowohl den Raum (der ja Energie enthält und damit auch Masse hat) als auch die Materie anzieht und dadurch verkleinert als mir vorzustellen dass der Raum sich ausdehnt aufgrund von irgendwas ganz anderem wofür wir jetzt grade keine Erklärung haben und deshalb abstossende dunkle Energie verantwortlich machen .
    Daher noch mal meine Frage, hat der leere Raum Gravitation und hindert den irgendwas daran da wo er angezogen wird hat quasi “dichter” zu werden und weit weg von Masse z.B. in den voids wird er also quasi “dünner” oder eben ernergieärmer?
    Mir ist schon klar, dass die Nullpunktenergie eigentlich nicht ausreicht um die dunkle Materie zu erklären, aber evtl. ginge es ja doch, wenn man annimmt, dass der Raum nicht gleichmässig verteilt ist. Passt doch auch sehr schön zur Relativitätstheorie mit gekrümmter Raumzeit. Und nein, das soll jetzt überhaupt nicht esoterisch klingen, ich versuche nur mal ne andere Sicht auf die Dinge und freue mich, wenn jemand mitdenkt und mir erklärt warum das totaler Humbug ist.

  1222. #1243 Karl-Heinz
    12. Februar 2019

    @Noonscoomo

    und freue mich, wenn jemand mitdenkt und mir erklärt warum das totaler Humbug ist.

    Aber gerne. Ich nehme einen Meterstab, der wie gesagt genau einen Meter hat. Natürlich kann man den Meterstab zusammendrücken aber dafür müsste man eine Kraft aufwenden. Es stellt sich eine neue Länge ein, die etwas kürzer ist als als ein Meter. Bleibt die Kraft, die auf den Stab wirkt konstant, so bleibt auch seine Länge konstant. Der Stab schrumpft nur soweit, bis sich ein neuer Gleichgewichtszustand einstellt.
    Ohne Kraft, die auf den Stab wirkt hat er jedenfalls genau einen Meter. Ich habe also eine Längeneinheit, die nicht der Schrumpfung unterworfen ist. Würdest du jetzt mit dieser Längeneinheit den Abstand zu einer weit entfernten Galaxie vermessen, so kommst du im Moment sicher zur Erkenntnis, dass sich die weit entfernte Galaxie von uns entfernt. Im Moment schrumpft also nichts. 😉

  1223. #1244 Noonscoomo
    Berlin
    14. Februar 2019

    @Karl-Heinz
    Da ist noch ein Missverständnis. Ich meinte nicht, das jemand den Meterstab drückt, sondern dass das komplette Bezugssystem schrumpft.
    Es ist doch gleichwertig zu sagen, der Raum dehnt sich aus, oder, alles im Raum, inclusive dem Meterstab schrumpft (in Bezug auf den Raum). Denn wenn alles schrumpft inclusive meinem Massstab und der Lichtgeschwindigkeit und alles, dann ist das ununterscheidbar von “der Raum dehnt sich aus”. So zumindest mein Vorschlag.
    Wenn also alles kontinuierlich schrumpft, und die Schrumpfung von der Gravitation verursacht wird, dann sollte es da schneller schrumpfen, wo mehr Gravitation ist.

    Oder jetzt noch mal von der anderen Seite betrachtet. Wenn Raum Masse hat und damit auch Gravitation und wenn Raum sich “verdichtet” durch die Gravitation, dann kommt da, wo mehr Masse ist auch mehr Raum zusammen und wenn sich um eine konstante Menge an Materie mehr Raum sammelt dann ist, je nach Betrachter, die Materie geschrumpft bzw. der Raum hat sich ausgedehnt. Ok, ich gebe zu, die Betrachtung der schrumpfenden Materie verlangt recht viel Vorstellungsvermögen aber vielleicht wird so klarer was ich meine.
    Daher noch mal meine Frage, hat Raum Gravitation und wird der angezogen von Gravitation und spricht was dagegen dass der an verschiedenen Stellen unterschiedlich “dicht” ist(was auch immer das in Bezug auf Raum bedeutet)?

  1224. #1245 Karl-Heinz
    14. Februar 2019

    @Noonscoomo

    Die Längeneinheit Meter kann nicht schrumpfen. Würde das komplette Bezugssystem inklusive der Längeneinheit Meter schrumpfen oder expandieren, so würde bzw. könnte man keine gravitative Rotverschiebung beobachten. Bitte wende dich diesbezüglich an Alderamin. Er hat mehr Erfahrung als ich. 😉

  1225. #1246 Noonscoomo
    Berlin
    14. Februar 2019

    @Karl-Heinz
    Ich denke nicht, dass das stimmt was du sagst.
    Wenn ich Alderamins Link zur mitbewegten Entfernung richtig verstehe ist das lediglich eine Frage des Bezugssystems

  1226. #1247 Nina
    Köln
    12. März 2019

    Guten Abend Herr Freistetter,

    für ein literarisches Projekt benötige ich einen Asteroideneinschlag. Da es ein postapokalyptisches Szenario ist, muss dieser entsprechend groß sein – tatsächlich habe ich einen Durchmesser von 36 km (Material: Eisen) angenommen. Dank eines Fachstudiums habe ich eine hoffentlich ausreichende Idee davon, was mit der Welt in dem Fall passiert. Leider gehört Astronomie aber eher nicht zu meinem Kenntnisbereich – trotzdem möchte ich keinen gravierenden Unsinn schreiben. Zwei Dinge fehlen mir aktuell noch:
    Da kein derartiges Schätzchen bekannterweise zum Einschlagszeitpunkt (2026) in unserem Sonnensystem herumfliegt, wollte ich ein Objekt von außerhalb des Sonnensystems nutzen. Gibt es da irgendwelche Fallstricke zu beachten?
    Und das zweite: Vom Einschlag an rückwärts gerechnet: Ab wann können professionelle Astronomen das gute Stück sehen? Kurz vor dem Einschlag habe ich folgende Zahlen recherchiert: 4 Wochen vorher passiert er die Sonne, 1 ¾ Stunde vorher den Mond, 10 Sekunden vorher die äußere Erdatmosphäre. Aber in welchem Zeitraum könnte er zum ersten Mal gesichtet werden?

    Ich bedanke mich im Voraus sehr herzlich bei Ihnen und finde es ganz wunderbar, einen Ort für derartige Fragen gefunden zu haben 🙂

    Nina

  1227. #1248 Florian Freistetter
    12. März 2019

    @Nina: “Da kein derartiges Schätzchen bekannterweise zum Einschlagszeitpunkt (2026) in unserem Sonnensystem herumfliegt,”

    Asteroiden dieser Größe gibt es jede Menge im Sonnensystem – nur halt nicht auf einer Kollisionsbahn. Aber da kann man doch ein paar literarische Freiheiten nehmen, oder?

    “wollte ich ein Objekt von außerhalb des Sonnensystems nutzen. Gibt es da irgendwelche Fallstricke zu beachten?”

    Na ja, sowas ist halt sehr unwahrscheinlich. Mehr dazu hier: https://scienceblogs.de/astrodicticum-simplex/2017/11/14/ein-botschafter-aus-dem-raum-zwischen-den-sternen-oumuamua-der-erste-interstellare-asteroid/

    “Aber in welchem Zeitraum könnte er zum ersten Mal gesichtet werden?”

    Das lässt sich pauschal schwer sagen. Das hängt von der Umlaufbahn ab; von den Teleskopen; davon ob ein Teleskop zum richtigen Zeitpunkt in die richtige Richtung schaut oder grad was anderes beobachtet. Ist also auch sehr viel Glück dabei; besonders bei interstellaren Asteroiden. Da kann die Vorwarnzeit ein paar Jahre bis ein paar Tage dauern…

  1228. #1249 Bullet
    12. März 2019

    @Nina: 1 ¾ Stunde vom Mond bis zur Erde? Der Asteroid wäre dann mit einer Relativgeschwindigkeit von knapp 89 km/s unterwegs. Das wäre eine Frontalkollision mit einem Körper, der ohnehin schon zu schnell ist, um im Sonnensystem zu verbleiben. (Ein Körper, der auf Höhe der Erdbahn noch mit knapp 60 km/s relativ zur Sonne unterwegs ist, ist zu schnell, um hier zu bleiben, es sei denn, er hat eine seehr exzentrische Bahn – aber die ist dann wieder nicht stabil, so daß es ein wenig fragwürdig ist, wieso er erst nach 4 Milliarden Jahren einen der Planeten trifft.)
    Für so ein Szenario wäre dann ein interstellarer Asteroid durchaus passend. Nur eben – wie Florian ja sagt – mordsunwahrscheinlich.
    Bahntechnisch könnte ich mir so etwas wie eine ekliptiknahe Hyperbel, deren auslaufender Ast, also nach der Ablenkung durch die Sonne, entgegen der Bahndrehung der inneren Planeten verläuft. Dann bekommst du auch deine 89 km/s.

  1229. #1250 Alderamin
    12. März 2019

    @Nina

    Man könnte auch einen Kometen nehmen. Die kommen hinreichend überraschend (Vorwarnzeit 1-2 Jahre) und können auch groß sein (der Kern von Hale-Bopp hatte 60 km Durchmesser). Besteht dann natürlich aus Eis, mittlere Dichte ca. 500 kg/m³. Die Geschwindigkeit auf Höhe der Erdbahn wäre knapp unter der Fluchtgeschwindigkeit von 41,7 km/s, die sich je nach Winkel zwischen frontal zur Erdbewegung (29,5 km/s -> 41,7 + 29,5 = 70,2 km/s; kurz vor Einschlag kommen noch 11,2 km/s durch die Erdanziehungskraft hinzu: 81,4 km/s) und die Erde einholend (41,7 – 29,5 = 12,2, Einschlag mit 23,4 km/s) gestalten kann (dazwischen ist alles möglich).

    Die Folgen eines Einschlags kann man sich hier vorrechnen lassen.

  1230. #1251 Spritkopf
    12. März 2019

    @Alderamin
    Ich schätze, du wolltest diese Seite verlinken, oder?

  1231. #1252 Peter L.
    12. März 2019

    @Nina:

    Wenn der Meteorit von der Erde aus gesehen aus einer Richtung kommt, die nahe an der Sonne liegt, dann wird er gar nicht oder erst sehr spät mit Teleskopen gesehen.

    Die Sonne ist so hell, dass es schwierig ist, mit Teleskopen so lichtschwache Objekte wie Asteroiden zu entdecken, deren Bahn man nicht kennt.

    Der Meteor von Tscheljabinsk kam z. B. aus Richtung Sonne:

    https://de.wikipedia.org/wiki/Meteor_von_Tscheljabinsk

    Zitat: „Das Objekt kam aus Richtung der Sonne und konnte daher von keinem der Himmelsüberwachungsprogramme (wie z. B. NEAT, LINEAR, LONEOS, CSS, CINEOS, Spacewatch) entdeckt werden.[22]…

    22.↑ Additional Details on the Large Fireball Event over Russia on Feb. 15, 2013 „The second diagram shows the impactor’s final trajectory over the last several hours, as it approached the Earth along a direction that remained within 15 degrees of the direction of the Sun. Asteroid detection telescopes cannot scan regions of the sky this close to the Sun.“ neo.jpl.nasa.gov“

  1232. #1253 Alderamin
    12. März 2019

    @Spritkopf

    yep, genau die. Danke.

  1233. #1254 Alderamin
    12. März 2019

    @Peter L.

    Der Meteoroid von Tscheljabinsk war nur ein paar Meter groß, den konnte man nur in Erdnähe aufspüren, deswegen konnte er durch seinen Anflug aus Richtung der Sonne nicht entdeckt werden. Ein 39 km großes Objekt wäre aber schon im Asteroidengürtel sichtbar und damit lange, bevor es hinter die Sonne abbiegt.

    Objekte von mehr als 10 km Größe, die der Erde nahe kommen können, sind alle schon entdeckt, 95% der Objekte von mehr als 1 km ebenso. Im Moment arbeitet man an einer Liste über 90% der Objekte > 100 m.

    Ein 39-km-Objekt kann also nur von weit außerhalb kommen, und da wäre ein Komet (nicht unwahrscheinlich) oder ein interstellarer Asteroid (sehr unwahrscheinlich) denkbar.

  1234. #1255 Spritkopf
    12. März 2019

    @Peter L.

    Die Sonne ist so hell, dass es schwierig ist, mit Teleskopen so lichtschwache Objekte wie Asteroiden zu entdecken, deren Bahn man nicht kennt.

    Laut dieser Tabelle der NASA hätte ein Eisen-Nickel-Asteroid mit 36 km Durchmesser ungefähr eine Absolutmagnitude von 10. Welche scheinbare Helligkeit das ergibt, hängt, wie du richtig schreibst, neben der Entfernung auch davon ab, aus welcher Richtung er kommt und wie er von der Sonne beleuchtet wird. Jedoch wäre er selbst dann, wenn er aus Richtung der Sonne käme, vermutlich schon vorher sichtbar, da die Erde eine Kreisbahn um die Sonne beschreibt und es somit mehrere Monate vor dem Einschlag Konstellationen gäbe, in denen man nicht in Richtung Sonne schauen müsste, um ihn zu entdecken.

    Die Frage ist halt, ob in dem Moment jemand sein Teleskop auf diese Stelle richtet und ob der Asteroid sich genügend bewegt, dass man ihn als Nicht-Stern identifiziert.

  1235. #1256 UMa
    13. März 2019

    Hallo Nina,

    die Kollision mit einem 36 km Eisenasteroiden der von außerhalb des Sonnensystems kommt und mit sagen wir 85 km/s einschlägt dürfte wohl kaum etwas auf der Erde überleben. Z.B.
    https://impact.ese.ic.ac.uk/ImpactEarth/cgi-bin/crater.cgi?dist=5000&diam=36000&pdens=8000&pdens_select=0&vel=85&theta=60&tdens=2500&tdens_select=0
    Die Energie des Einschlag ist 1000 mal größer als dem Asteroiden vor 66 Millionen Jahren.
    Der Einschlag hat einen Transienten Krater von 522km und 184 km Tiefe. Das ist bis in den Erdmantel. Am ende verformt sich der Krater zu einem Durchmesser von 1180 km. Ein guter Teil der Erde wird hoch mit Auswurfmaterial bedeckt, welches beim Wiedereintritt in die Atmosphäre diese selbst in 8000-10000 km Entfernung auf hunderte Grad erhitzt. Die Energie dürfte ausreichen, um die Ozeane weltweit auf über 100 Grad zu erhitzen. Selbst im Erdorbit würden Satelliten und Raumstationen durch das Auswurfmaterial zerstört.
    Auch unterirdische Anlagen dürften durch Erdbeben zerstört werden.

    Wenn das so gewollt ist…

    Asteroiden im inneren Sonnensystem dürfte nicht infrage kommen, da dort alle dieser Größe schon bekannt sind.

    Bleiben Objekte von außerhalb des Sonnensystems, die aus Eisen sein könnten und auch die erforderliche Größe und Geschwindigkeit hätten. Die sind zum Glück aber selten, ich schätze im Schnitt einmal 10 hoch 17 Jahren.

    Kometen aus der Oortschen Wolke wären langsamer (ca. Parabelbahn) und nicht aus Eisen. Dafür aber häufiger. Ein solcher Einschlag könnte im Schnitt einmal 10 hoch 11 Jahren passieren.

    Zur Entdeckung:
    Ich habe mich an der Entdeckung von Kometen orientiert. Früher war es üblich, dass Kometen durch Amateure entdeckt wurden bei Helligkeiten um 10 mag. Inzwischen gibt es aber automatische Suchprogramme wie Catalina Sky Survey und Pan-STARRS.

    Die dürften die meisten Objekte so bei 19-20 mag Helligkeit finden.
    Da das Objekt so groß und damit hell ist, dürfte das schon im äußerem Sonnensystem der Fall sein

    2022 geht das LSST in Betrieb, welches den größten Teil des Südhimmels beobachtet (ca 44% des Gesamthimmels) und derartige Objekte bei ca 24,5 mag finden können soll.

    Realistisch wäre bei einem (schnellen) interstellaren Objekt eine Entdeckung entweder mit dem neuen LSST Mitte/Ende 2022 in ca. 25 AE Entfernung zwischen Uranus und Neptun.
    Oder, falls nicht aus Richtung der Südhalbkugel kommend, mit Pan-STARRS oder Catalina ca. 9 Monate vor dem Einschlag in ca. 6 AE Entfernung hinter Jupiter.

    Das Objekt dürfte sehr intensiv beobachtet werden um die Bahn genauer zu bestimmen, trotzdem dürfte es sehr lange dauern bis fest steht ob es die Erde trifft oder knapp verfehlt.

    Bei einem langsameren Kometen dieser Größe (wie Hale-Bopp, 60km Durchmesser) dürfte die Entdeckung mit Pan-STARRS oder Catalina schon 2021 in 14-15 AE Entfernung gelingen, vor Fertigstellung des LSST.

    Wie groß die Chance ist, dass er diesen Teleskopen lange Zeit entgeht, kann ich nicht sagen.

  1236. #1257 Peter L.
    13. März 2019

    @Nina:
    Was ebenfalls einen Einfluss darauf hat, mit welcher Wahrscheinlichkeit ein Asteroid zeitig genug entdeckt wird, um ihn noch vom Kollisionskurs abzubringen, ist seine Albedo, also sein Rückstrahlvermögen:
    Ein Himmelskörper mit hoher Albedo wie ein heller Asteroid (z. B. mit viel Eis an der Oberfläche) o. ein Komet (mit Eis u. Schweif) wird wahrscheinlich viel eher entdeckt als ein Himmelskörper mit sehr kleiner Albedo, d. h. mit sehr dunkler Oberfläche, welche kaum Sonnenlicht reflektiert.

    Die Albedo von Asteroiden u. Himmelkörpern jenseits der Neptunbahn (TNOs: Transneptunische Objekte) liegt zwischen 1 (so hell wie frisch gefallender Schnee) und 0 (so schwarz wie Kohlenruß), siehe 1. Graphik auf
    https://www.johnstonsarchive.net/astro/astalbedo.html

    Relativ dunkle Asteroiden sind z. B.:
    https://de.wikipedia.org/wiki/Asteroid#Zusammensetzung

    Zitat: „C-Asteroiden: Dies ist mit einem Anteil von 75 Prozent der häufigste Asteroidentyp. C-Asteroiden weisen eine kohlen- oder kohlenstoffartige (das C steht für Kohlenstoff), dunkle Oberfläche mit einer Albedo um 0,05 auf. Es wird vermutet, dass die C-Asteroiden aus dem gleichen Material bestehen wie die kohligen Chondriten, einer Gruppe von Steinmeteoriten. Die C-Asteroiden bewegen sich im äußeren Bereich des Hauptgürtels. …

    D-Asteroiden: Dieser Typ ist ähnlich zusammengesetzt wie die P-Asteroiden, mit einer geringen Albedo und einem rötlichen Spektrum. …

    T-Asteroiden: T-Asteroiden findet man im mittleren und äußeren Bereich des Hauptgürtels sowie bei den Jupiter-Trojanern. Sie weisen ein dunkles rötliches Spektrum auf.“

    Zitat https://www.irs.uni-stuttgart.de/skript/RaL/050628_Bedrohliche_Himmelskoerper_www.pdf:

    „Erdnahe Objekte – Asteroiden und Kometen
    Alle Objekte, die der Erde nahe kommen, werden als NEOs (Near Earth Objects) bezeichnet. Ihre Größe ist sehr unterschiedlich, sie reicht bis zu einigen 10 km. Die meisten Objekte zeigen … eine geringe Albedo (0,03 – 0,08).“

    Erdnahe Objekte sind aber fast alle bekannt.

    Deutlich heller sind z. B.:

    https://de.wikipedia.org/wiki/Asteroid#Zusammensetzung

    Zitat: „M-Asteroiden: Der überwiegende Rest der Asteroiden wird diesem Typ zugerechnet. Bei den M-Meteoriten (das M steht für metallisch) dürfte es sich um die metallreichen Kerne differenzierter Asteroiden handeln, die bei der Kollision mit anderen Himmelskörpern zertrümmert wurden. Sie besitzen eine ähnliche Albedo wie die S-Asteroiden. Ihre Zusammensetzung dürfte der von Nickel-Eisenmeteoriten gleichen.* …

    S-Asteroiden besitzen eine hellere Oberfläche mit einer Albedo von 0,15 bis 0,25.“

    https://en.wikipedia.org/wiki/M-type_asteroid

    Zitat: „M-type asteroids are asteroids of partially known composition; they are moderately bright (albedo 0.1–0.2). Some, but not all, are made of nickel–iron, either pure or mixed with small amounts of stone.”

    Die allermeisten Eisen-Asteroiden sind also wahrscheinlich relativ hell, was die Wahrscheinlichkeit erhöht, dass ein Eisen-Asteroid auf Kollisionskurs zeitig entdeckt wird, besonders wenn er relativ groß ist.

    Asteroiden u. Kometen senden aber nicht nur sichtbares Licht aus, sondern auch Wärmestrahlung im Infrarot-Bereich.
    Dies ist auch bei sehr dunklen Körpern der Fall, selbst wenn sie relativ weit von der Sonne entfernt sind (z. B. hinter der Neptun-Bahn).

    Im Erdorbit gibt es mehrere Infrarot-Teleskope. Es besteht also die Chance, auch sehr dunkle Asteroiden zu entdecken.

  1237. #1259 Kerstin
    13. März 2019

    Hallo Florian Freistetter, ich würde gerne verstehen, wie heiße Gase im Weltraum abkühlen. Die Vorstellung, Wärme ist Bewegung der Teilchen, reicht da irgendwie nicht so ganz. Und um das heiße Gas beispielsweise ist ja viel “nichts” an das keine Wärme abgegeben werden kann? Oder doch?
    Außerdem heißt es immer, die Materie klumpt sich etwa bei der Sternentstehung zusammen. Aber wieso klumpt sich die Materie auf der Erde nicht zusammen? Die Dichte, beispielsweise meiner Brötchenkrümel, müsste doch höher sein als in einem Nebel, dennoch passiert auf meinem Teller nichts weiter? (Das Beispiel ist natürlich nicht ernst gemeint, aber die Frage schon!)
    Vielen Dank auch für die vielen Sternengeschichten! Falls die Antworten auf meine Fragen dort doch schon vorkam, würde ich mich über die Nummer freuen!
    Beste Grüße Kerstin

  1238. #1260 Alderamin
    13. März 2019

    @Kerstin

    Bin zwar nicht Florian, aber…

    Und um das heiße Gas beispielsweise ist ja viel “nichts” an das keine Wärme abgegeben werden kann? Oder doch?

    Doch, jede Menge leerer Raum. Warum wird es im Sonnenlicht warm und in der Nacht kalt? Weil die Sonnenstrahlung auch durch den leeren Raum die Erde erwärmt und die Erde auf der Nachtseite durch Wärmestrahlung in den kalten Weltraum abkühlt. Neben Wärmetransport durch Wärmleitung, den Du anscheinend angesprochen hast, gibt es auch den Transport durch Strahlung. Und jeder warme Körper, auch Gas, strahlt Wärme ab.

    Außerdem heißt es immer, die Materie klumpt sich etwa bei der Sternentstehung zusammen. Aber wieso klumpt sich die Materie auf der Erde nicht zusammen? Die Dichte, beispielsweise meiner Brötchenkrümel, müsste doch höher sein als in einem Nebel, dennoch passiert auf meinem Teller nichts weiter?

    Weil die Brötchenkrümel sich auf dem Teller unter ihrem Gewicht auf der Erde nicht bewegen können. Die Gewichtskraft drückt sie auf den Teller, und die Reibungskraft hält sie unter dem Gewichtsdruck auf der Stelle fest. Wenn die Erde nicht wäre und die Krümel frei schwebend im Raum wären, dann würden sie sich anziehen und ganz langsam aufeinander zu driften, bis sie zusammenstoßen. Auf der Erde verklumpen sie nur – mit der Erde!

  1239. #1261 PDP10
    14. März 2019

    @Kerstin:

    Ergänzend zu @Alderamins Post:

    Und jeder warme Körper, auch Gas, strahlt Wärme ab.

    Bei Gasen zB. indem die Moleküle oder Atome aneinander stoßen und kinetische Energie (aus ihrer Bewegung) aufeinander übertragen. Dann gerät ein Molekül zB. in Schwingung und wird mit der Zeit diese Energie wieder in Form von Strahlung verlieren.

    Wenn so eine Wolke aus Teilchen (Gas) lange genug irgendwo rumfliegt, wird diese Ansammlung von Gas – so sie denn nicht in einem Dampfkochtopf oder sowas eingeschlossen ist – darum immer größer. Durch die Stöße untereinander.
    Die Wege zwischen den Teilchen werden immer länger und die kinetische Energie der Teilchen immer kleiner.
    Die Wolke kühlt ab.

    Wenn die Erde nicht wäre und die Krümel frei schwebend im Raum wären, dann würden sie sich anziehen und ganz langsam aufeinander zu driften, bis sie zusammenstoßen.

    Und dann zB. durch Van-der-Vaals-Kräfte (Elektrostatik, vereinfacht gesagt) aneinander kleben bleiben. So wie die Papier-Schnipsel an einem elektrisch aufgeladenem Stück Plastik.

  1240. #1262 Kerstin
    15. März 2019

    @alderamin und @pdp10
    Danke für eure Erklärungen, wenn man es so sieht, klingt es eigentlich ganz einfach (Marke “hätte ich auch selbst drauf kommen können!”)

  1241. #1263 Horst Fleischer
    D-74211 Leingarten
    17. März 2019

    Der Mond dreht sich kaum um die Erde, nur 15° auf seiner Umlaufbahn. Vielmehr dreht sich die Erde unter dem Mond viel schneller und damit auch der gemeinsame Schwerpunkt zwischen ihnen auf ca. 4687 km vom Erdmittelpunkt, Um diesen Punkt tanzt die Erde täglich, also müßte auch der Nordpol tanzen und einen großen Kreis täglich beschreiben? Wobei sich die Erde normal um ihre Achse weiter dreht. Stimmt das ?
    mfg
    Fleischer H.

  1242. #1264 Florian Freistetter
    17. März 2019

    @Horst Fleischer: Nein, der Mond dreht sich definitiv um die Erde; einmal im Monat rundherum: https://scienceblogs.de/astrodicticum-simplex/2013/05/02/der-mond-dreht-sich-um-seine-achse/
    Aber es stimmt, dass die Erde um das Baryzentrum Erde-Mond wackelt. Einmal pro Monat: https://de.wikipedia.org/wiki/Erde-Mond-Schwerpunkt

  1243. #1265 HF(de)
    17. März 2019

    Um diesen Punkt tanzt die Erde täglich

    Nee, nicht täglich sondern monatlich…

  1244. #1266 Peter
    19. März 2019

    Der Ostersonntag ist doch eigentlich immer der Sonntag nach dem 1. Vollmond im Frühling.

    Frühlingsanfang ist dieses Jahr am 20. 3. um 22:58 Uhr Mitteleuropäischer Zeit (MEZ).
    Der 1. Vollmond im Frühling ist am 21. 3. um 2:42:52 Uhr MEZ.
    Warum ist Ostersonntag dann nicht am darauffolgenden Sonntag, dem 24. 3., wie man es erwarten würde, sondern erst am 21. 4.?

    Vor dem 21. 4. ist ja noch mal Vollmond, nämlich am 19. 4. um 13:12:11 Uhr Mitteleuropäischer Sommerzeit (MESZ). Das ist dann aber schon der zweite Vollmond im Frühling.

    Ich kann mich erinnern, dass Ostern in manchen Jahren auch schon Ende März war, also warum ist es dieses Jahr anders?

  1245. #1267 PDP10
    19. März 2019

    @Peter:

    Weil für die Berechnung des Termins des Osterfestes der 21. März fix als Tag des Frühlingsanfangs benutzt wird.

    So einen Termin für Ostern wie dieses Jahr nennt man darum Osterparadoxon

  1246. #1268 Florian Freistetter
    19. März 2019

    @Peter: Wenn du es ganz genau erklärt haben möchtest, dann lies dir das mal durch: https://www.der-orion.com/universum/10-lichtsekunden/119-ostern-und-der-mond

  1247. #1269 Peter
    20. März 2019

    Vielen Dank an PDP10 und Florian Freistetter für die Links.

    So etwas Ähnliches hatte ich schon vermutet.
    Ein Bekannter meinte auch, dass Kirchenkalender häufig von den heutigen astronomischen Ereignissen abweichen, zum Teil um mehrere Tage.*
    Und im Fall „erster Vollmond im Frühling“ kann eine Abweichung um mehrere Tag eben dafür sorgen, dass sich der Ostertermin fast um einen vollen Monat nach hinten verschiebt.

    * Ein anderes Beispiel für mehrtägige Abweichungen ist Weihnachten:
    Die Westkirche legte Weihnachten im 4. Jahrhundert auf den Tag der Wintersonnenwende fest, welche schon von den Römern, Griechen usw. gefeiert wurde. Der Tag der Wintersonnenwende war damals der 25. Dezember. In der heutigen Zeit findet die Wintersonnenwende schon am 21. Dezember oder am 22. Dezember statt.
    In der Ostkirche (Orthodoxe) gilt noch der Julianische Kalender, dort ist Weihnachten erst am 7. Januar.

  1248. #1270 Nina
    Rheinland :-)
    21. März 2019

    Guten Abend,

    erst einmal vielen (vielen!) Dank für die zahlreichen und hochspannenden Antworten. Sollte das Projekt so etwas wie Veröffentlichungsreife bekommen, landet ihr auf jeden Fall in der Danksagung. Ich mag künstlerische Freiheit, aber im Herzen bin und bleibe ich eben Naturwissenschaftler und möchte alles schön faktenunterfüttert haben. Und nun eins nach dem anderen: 🙂

    Mangels Kollisionsbahn würde ich einen Asteroiden von außerhalb wählen, Kometen sind mir zu hell. Dass die Wahrscheinlichkeit extrem niedrig ist, ist in dem Fall egal, es soll ja nur einmal passieren. Frei nach dem Motto: Irgendjemand gewinnt auch im Lotto 😀 (Hier zieht die Sache mit der künstlerischen Freiheit ;-))

    Danke für den Hinweis, die Geschwindigkeit betreffend, ich habe sie nun auf 45 km/s runtergesetzt, das passt immer noch prima in den Rahmen. Ein bis zwei Jahre Vorwarnzeit sind gut und scheinen im möglichen Rahmen zu liegen, ein bisschen Panik und nutzloses Rumgeballer muss sein! 🙂

    Die Folgen betreffend habe ich eine Reihe Artikel bemüht und mich an einigen Seminarstunden orientiert, die jene zum Inhalt hatten (Biologie & Geografie). Für konkrete Zahlen habe ich mit
    https://www.purdue.edu/impactearth/
    rumgespielt (geiles Teil!).

    Zu den Folgen (alles Schätzwerte):
    – „mein“ Asteroid schlägt im Westpazifik ein, Standort meiner Protagonistin ist Mitteldeutschland.
    – Die Winde liegen bei 400-600 km und damit bei Tornadowerten.
    – Die Tsunamiwelle erreicht 130-260 m Höhe an der nächstgelegenen amerikanischen Küste, aufgrund der Rückbewegungen sind auch scheinbar gegenläufige Küsten betroffen.
    – Das Erdbeben erreicht Stärke 12-13 ab dem Einschlagspunkt, davon ist an meinem Standort – 11.000 km entfernt – aber nicht mehr als ein mittelschweres Beben zu spüren.
    – Die Druckwelle ist mir Abstand das Gefährlichste, da der entstehende Druck nicht schnell genug abgegeben werden kann, erhitzt sich die Luft kurzfristig auf bis zu mehrere hundert Grad.
    – Der Pazifik kocht kurz, aber nur an der Oberfläche (jedenfalls ab einigen tausend Kilometern Entfernung). Hier wird oft die Wassermasse unterschätzt, man darf nicht vergessen, dass der Pazifik im Durchschnitt (!) fast 4000 km tief ist und mehr als 700 Millionen Kubikmeter Wasser enthält. Der Atlantik wird nicht übermäßig erhitzt.
    – Mangels Strom, Personal, Kühlung und Sarkophagbau und unter Eindruck der diversen Naturkatastrophen geht jedes Atomkraftwerk hoch, um es mal salopp zu formulieren.
    – Mittel- und langfristige Folgen sehen sauren und radioaktiven Regen, eine Versauerung der Meere, eine staubbedingte Eiszeit […] vor.
    So viel dazu 🙂
    – Die Menschheit soll auf >300.000 beim Einschlag und auf > 200.000 Individuen in den nächsten Jahren reduziert werden. Dazu braucht es bei unserer Bevölkerungszahl schon ordentliches stellares Feuerwerk. Überlebende sollen sich in Bunker- und anderen Tiefbauanlagen (nur einschlagsabgewandte Seite, nur in entsprechend großer Entfernung von der Küste) sowie sehr tief eingeschnittenen Tälern bei entsprechender Lage (windbrechend) finden.

    Noch mal vielen Dank & liebe Grüße,

    Nina

  1249. #1271 Uli Schoppe
    31. März 2019

    Ich bin eben über den YORP Effekt gestolpert:

    https://www.spektrum.de/news/ein-asteroid-zerstoert-sich-selbst/1635512

    Warum zerlegt der nicht sämtliche Asteroiden? Da muss sich doch im LAufe der Zeit bei allen richtig fies viel Drehmoment angesammelt haben? 🙂

    Vielen Dank schon mal für die Aufklärung darüber wo ich zu simpel denke 🙂

  1250. #1272 Florian Freistetter
    31. März 2019

    @Uli Schoppe: Der YORP-Effekt war im Blog schon öfter Thema: https://scienceblogs.de/astrodicticum-simplex/tag/yorp-effekt/

    Damit ein Asteroid “zerlegt” werden kann, muss er so schnell rotieren, dass die Bindungskräfte im Gestein ihn nicht mehr halten können. Dazu muss er wirklich schnell rotieren. Und das schafft der Yorp-Effekt halt nicht immer…

  1251. #1273 Uli Schoppe
    31. März 2019

    Hallo Florian 🙂

    Stimmt habe ich aus was für Gründen auch immer dauernd überlesen. Aber eigentlich haben doch alle Asteroiden eine unsymmetrische Oberfläche. Das heißt ja das der Effekt immer alle betrifft. Warum schaukelt sich der Effekt nicht bei allen Asteroiden auf?

  1252. #1274 Peter
    Dresden
    1. April 2019

    Ich habe heute während der Abenddämmerung um 20:21 Uhr am Osthimmel von Dresden eine helle, punktförmige Leuchterscheinung gesehen, die am Himmel etwa in Richtung Nordosten bis Osten erschien und sich auf gerader Linie in Richtung Osten bis Südosten schräg auf den Horizont zu bewegte. Das Licht war fast so hell wie die Venus, leuchtete etwa 10 – 15 s lang relativ gleichmäßig, bevor es innerhalb von 1 – 2 s dunkler wurde und verschwand, weit bevor es den Horizont erreichte.

    Für ein Flugzeug war es zu schnell. Die ISS scheidet aus, da diese in der Abenddämmerung von Westen her kommend nicht plötzlich mitten am Himmel auftaucht (sie wird von der Sonne angestrahlt, ist also schon ab dem westlichen Horizont sichtbar) und meist auch deutlich länger zu sehen ist.

    Als Auslöser der Leuchterscheinung vermute ich einen Satelliten (z. B. Antennen, die das Sonnenlicht einige Sekunden lang in meine Richtung reflektierten) oder einen hellen Meteor (Boliden).

    Kann jemand Websites empfehlen, wo man sich ansehen bzw. danach suchen kann, an welchen Orten an Himmel an welchen Tagen u. zu welchen Uhrzeiten helle Satelliten zu sehen waren bzw. sind?

    Gibt es Websites, wo potentielle oder bestätigte Boliden mit Orten der Sichtbarkeit und Zeit des Auftretens erfasst werden?

  1253. #1275 Florian Freistetter
    1. April 2019

    @Peter: War die ISS – die war heute von 20.15 bis 20.21 über Dresden sichtbar: https://spotthestation.nasa.gov/sightings/view.cfm?country=Germany&region=None&city=Dresden

  1254. #1276 Alderamin
    1. April 2019

    @Peter

    Hier sind alle Überflüge von heute Abend gelistet.

    Deiner Beschreibung nach würde Iridium 64 passen. Der wird gerade de-orbitet, d.h. er kann früher auftauchen, als in der Liste vorhergesagt und evtl. auch “flaren”, obwohl nicht vorausgesagt. Wenn die Geometrie stimmt und die Sonne im richtigen Winkel auf den Satelliten scheint, dann kann er sehr hell werden und problemlos die ISS übertreffen. Die Flares dauern 10-15 s. Passt also auch.

  1255. #1277 Peter
    Dresden
    1. April 2019

    @ Florian: Also taucht die ISS manchmal doch weiter oben am Himmel auf?

    Bei der vorletzten totalen Mondfinsternis im Sommer 2018 erschien die ISS direkt am Horizont u. beschrieb dann einen großen Kreisbogen am Himmel.

    Vielleicht waren heute am Anfang der ISS-Sichtbarkeit zunächst Wolken davor u. dann tauchte die ISS plötzlich hinter einer Wolke auf. Denn als heute Abend die Sonne noch schien, waren einige großflächige Schichtwolken/Schleierwolken unterwegs. 20:21 Uhr war es schon fast dunkel, da waren evtl. noch vorhandene Wolken vom Innenraum durchs Fenster schlecht zu sehen.

    Dass die ISS in Richtung Osten am Himmel relativ plötzlich verschwindet, ist logisch, wenn sie in den Erdschatten eintaucht.

  1256. #1278 HF(de)
    2. April 2019

    @Peter: wenn ich mich recht erinnere, tauchte bei der MoFi 2018 die ISS zwei mal auf, hab sie auch (erstmalig an einem Tag) zwei mal gesehen. Beim zweiten Durchgang war sie schon fast über meinem Kopf, als ich sie bemerkte… Das klappt, dass man sie erst spät sieht, auch wenn man drauf achtet. Aber wie Alderamin schon sagte: kann auch ein Satellit gewesen sein.

  1257. #1279 Bullet
    2. April 2019

    @Peter: ich hab zwar nicht alle Iridium-Umlaufbahnen im Kopf wie unser Obernerd ( 😛 ) Alderamin, aber wenn der Leuchtpunkt nur etwa 10 s lang und dann richtig hell wird, dann isses mit hoher Wahrscheinlichkeit ein Iridium. Diese Flares sind echt knallhell und entstehen, wie du korrekt vermutest, durch “Antennen, die das Sonnenlicht einige Sekunden lang in meine Richtung reflektierten”.
    Siehe hier:
    https://de.wikipedia.org/wiki/Iridium-Flare

  1258. #1280 Alderamin
    2. April 2019

    @Bullet

    Im Kopf, haha 😀

    Eigentlich wollte ich nur nachschauen, ob Iridium 64 ein alter ist, oder einer der neuen Generation. Die neuen flaren nicht mehr. 64 ist ein alter, und da fand ich zufällig, dass er deorbitet wird, also stimmen die Vorhersagen auf Heavens-Above wahrscheinlich nicht, weil sich die Bahnelemente durch den Luftwiderstand laufend ändern. Daher könnte es sein, dass er in Dresden geflaret hat, obwohl kein Flare vorhergesagt war.

  1259. #1281 Bernd
    Essen
    5. April 2019

    Habe heute etwas über “Anomale Rotverschiebung” gelesen. Demnach gäbe es in der gemessenen Rotverschiebung der Galaxien keine echte Linearität sondern Häufungen. Dies sei insbesondere bei Quasaren häufig zu beobachten. Ist da was dran?

  1260. #1282 Alderamin
    5. April 2019

    @Bernd

    Falls es um Quasare geht: darüber bin ich neulich bei der Recherche zu meinem Quasar-Artikel gestolpert. Arp behauptete, solche Periodizitäten in den Rotverschiebungen von Quasaren gefunden zu haben. Die Autoren dieser Arbeit haben keine gefunden.

  1261. #1283 AndreasL
    Wolfenbüttel
    25. April 2019

    Warum haben Sterne unterschiedliche Farben?

    Das sichtbare Licht eines Sterns entsteht in dessen Photosphäre. Dazu muss sich das Plasma im Sterninneren so weit abgekühlt haben, dass sich Elektronen und Protonen zu neutralem Wasserstoff zusammenfinden können. Diese Temperatur ist aber nun mal von den Eigenschaften des Wasserstoffs abhängig. Und da Sterne in guter Näherung Schwarzkörperstrahlung abgeben, sollten dann nicht alle Sterne in derselben Farbe leuchten?

  1262. #1284 Karl-Heinz
    25. April 2019

    Diese Temperatur ist aber nun mal von den Eigenschaften des Wasserstoffs abhängig

    Nö …Ist vom Typ (Spektraltypen) abhängig.

    Die heißesten violetten Sterne sind die Zentralsterne Planetarischer Nebel mit Oberflächentemperaturen zwischen 100.000 K und 50.000 K (Typ W).
    Bis zu 30.000 K erscheinen sie Violett-Weiß (Typ O),
    bis zu 10.000 K Blau-Weiß (Typ B),
    bis zu 7.500 K Weiß (Typ A),
    bis zu 6.000 K Gelb-Weiß (Typ F),
    bis zu 4.500 K Gelb (Typ G),
    bis zu 3.500 K Gelb-Rot (Typ K),
    und bis zu 2.000 K Rot (Typ M).

  1263. #1285 Alderamin
    25. April 2019

    @Andreas L

    Dazu muss sich das Plasma im Sterninneren so weit abgekühlt haben, dass sich Elektronen und Protonen zu neutralem Wasserstoff zusammenfinden können.

    Das ist definitiv nicht korrekt, die Photosphäre ist ein Plasma. Die darüber liegende Chromosphäre enthält neutrale Atome, und da entstehen die Spektrallinien.

  1264. #1286 UMa
    25. April 2019

    @AndreasL: Die effektive Temperatur des Sterns ist so hoch, dass die von Innen ankommende Energie abgestrahlt werden kann.
    Die Durchsichtigkeit der Atmosphäre hängt neben der Temperatur auch von der Dichte ab.
    So kann die Atmosphäre der Sterne auch bei hohen Temperaturen durchsichtig werden, wenn die Dichte nur genügend gering ist. Und die Dichte fällt nach oben in der Atmosphäre schnell ab.

  1265. #1287 Peter
    25. April 2019

    Zitat AndreasL #1283: „Warum haben Sterne unterschiedliche Farben?“

    Sterne bestehen aus Plasma, die Gas-Atome darin sind also teilweise oder komplett ionisiert. Im Plasma befinden sich außerdem freie Elektronen. Freie Elektronen haben eine höhere Energie als Elektronen, die in der Atomhülle gebunden sind.

    Fängt ein Atom o. Ion ein freies Elektron ein
    (z. B. H+ wird zu H;
    H zu H-;
    He2+ zu He1+;
    He1+ zu He;
    He zu He-;
    Be2+ zu Be+;
    Be+ zu Be;
    C4+ zu C3+;
    C3+ zu C2+
    usw.),
    wird die überschüssige Energie des eingefangenen Elektrons in Form von Licht abgegeben.

    Je nach Oberflächentemperatur des Sterns sind die Atome unterschiedlich stark ionisiert. Je heißer ein Plasma ist, desto weniger (elektrisch neutrale) Atome und desto mehr Ionen gibt es. Bei sehr hohen Temperaturen sind auch mehr Atome vollständig ionisiert, das heißt man hat nur noch Atomkerne ohne Elektronenhülle (z. B. H+, He2+, Be4+).

    In der Atomhülle gibt es unterschiedliche Energieniveaus („Schalen“).
    Bei steigender Temperatur verliert ein Atom zuerst Elektronen aus der äußersten Schale der Atomhülle, weil diese am schwächsten gebunden sind.
    Fängt ein Ion ein Elektron auf die äußerste Schale ein, wird Licht mit geringer Energie, also mit größerer Wellenlänge abgegeben, z. B. rotes Licht.

    Wenn die Temperaturen höher sind, verlieren Atome auch Elektronen, die sich auf weiter innen liegenden Schalen der Atomhülle befanden (diese waren stärker gebunden).
    Ein solches Ion mit „Lücken“ auf inneren Schalen kann freie Elektronen auf weiter innen liegende Schalen der Atomhülle einfangen. Dabei wird Licht mit höherer Energie, also mit kleinerer Wellenlänge frei, z. B. blaues Licht oder noch energiereichere UV-Strahlung.

  1266. #1288 Judith
    Österreich
    6. Mai 2019

    Hallo Florian,
    Meine Tochter (6 Jahre) fragt mich ob es ältere bzw.auch jüngere Erden gibt?
    Ebenso wo die Sonne und die Planeten herkommen?

    Ich kann ihr nur Schulwissen anbieten.
    Sie will das sehr genau wissen.☺️

    Sie freut sich auf eine Antwort von einem echten Astronomen.

    Judith und Maya

  1267. @Judith: Also die Planeten des Sonnensystems sind alle gleich alt. Aber Planeten gibt es auch bei anderen Sternen und da sind jede Menge dabei, die auch jünger oder älter sind. Und wo die Sonne und die Planeten herkommen ist eine etwas längere Geschichte 😉 Vielleicht könnt ihr euch gemeinsam ein paar passende Folgen des Sternengeschichten-Podcasts (sternengeschichten.org) anhören? Oder bei “Astronomie in 365 Tagen” vorbei schauen: https://www.instagram.com/astrodicticum/

  1268. #1290 Marc
    Bonn
    14. Mai 2019

    Hallo Florian,

    bei dem angedachten “Bergbau” im All geht es darum Rohstoffe im All zu gewinnen. Wenn es die Idee ist diese zur Erde zu bringen, dann folgende Frage:

    Kann der Rohstofftransport aus dem All zur Erde zu einer Gewichtsveränderung der Erde führen, so dass die Umdrehung um die eigene Achse und auch Umlaufbahn um die Sonne beeinflusst werden könnte?

    Ich weiß, noch wissen wir nicht um welche Massen es sich handelt.

    Wahrscheinlich kann man die Frage auch umkehren:

    Kann das Verbrennen der fossilen Rohstoffe zu einer bemerkenswerten Gewichtsveränderung der Erde führen, so dass … (siehe oben)?

    Besten Dank und viele Grüße

    Marc.

  1269. #1291 Marc
    Bonn
    14. Mai 2019

    Zusatzbemerkung:

    Das hier habe ich gelesen:

    https://scienceblogs.de/astrodicticum-simplex/2015/03/30/wird-die-erde-durch-meteoriteneinschlaege-im-laufe-der-zeit-immer-schwerer/

    In diesem SPON Artikel wird vom ziemlich bald beginnendem Bergbau im All gesprochen. Daher meine Frage. Dein Blogeintrag hat diese Thematik ja ausgeklammert.

    SPON Artikel:

    https://www.spiegel.de/wissenschaft/weltall/bergbau-im-weltraum-forscher-warnen-vor-raubbau-im-all-a-1267126.html

    Nochmals besten Dank und viele Grüße

  1270. #1292 Florian Freistetter
    14. Mai 2019

    @Marc: “Kann der Rohstofftransport aus dem All zur Erde zu einer Gewichtsveränderung der Erde führen”

    Na ja, das Gewicht wird sich schon verändern. Aber es wird absolut keine Auswirkungen haben. Ich hab dazu hier was geschrieben: https://scienceblogs.de/astrodicticum-simplex/2015/03/30/wird-die-erde-durch-meteoriteneinschlaege-im-laufe-der-zeit-immer-schwerer/

  1271. #1293 Bullet
    14. Mai 2019

    @Marc:

    Kann das Verbrennen der fossilen Rohstoffe zu einer bemerkenswerten Gewichtsveränderung der Erde führen, so dass […]

    Eher nicht. Denn die Massen der fossilen Brennstoffe werden (allergrößtenteils) in Wasser und CO2 verwandelt. Wsser fällt sofort wieder runter, und CO2 mag zwar als Gas nicht unbedingt in Brocken herumliegen, aber wenn es in der Atmospäre bleibt (was definitiv der Fall ist), dann bleibt es Teil der Gesamtmasse der Erde. Allenfalls die Rotationsperiode der Erde könnte im Mikrosekundenbereich verändert werden. Wenn ich mich daran erinnere, daß der Pirouetteneffekt, der durch das Fallen der Blätter im Herbst der Nordhalbkugel ausgelöst wird, ebenfalls meßbar ist, dann ziehe ich die Halloween-Maske vor derlei Genauigkeit.

  1272. #1294 Captain E.
    15. Mai 2019

    @Bullet:

    Naja, aber noch exakter wird ein Teil des Wassers durch das Sonnenlicht (UV?) aufgespalten in Wasserstoff und Sauerstoff, und wiederum ein Teil des Wasserstoffs entweicht danach ins Weltall.

    Und da hätten wir ja noch das Helium. Helium entsteht unter anderem durch Fusion von Wasserstoff, aber auch durch den Zerfall schwerer instabiler Atomkerne. Die Fusion findet auf der Erde nun wirklich so gut wie gar nicht statt, aber der Alpha-Zerfall schon. Der Erdkern ist ja auch deshalb so warm, weil tief unter unseren Füßen Uran, Thorium und ähnlich schwere Isotope zerfallen. Interessanterweise scheint Methan die Fähigkeit zu haben, die dabei abgestrahlten Heliumkerne abzubremsen. Zumindest findet sich in Erdgaslagerstätten meistens auch Helium. Wurde dieses aber erst einmal gebraucht, als Traggas für Luftschiffe und Wetterballons, als Kühlmittel oder als Atemgasbeimischung für Taucher, gelangt es in die Atmosphäre und entweicht dann ebenso wie der Wasserstoff.

    Und tatsächlich ist es wohl, dass der tägliche Eintrag von festerem Material durch Asteroiden und Meteoroiden den Verlust dieser flüchtigen Gase bei weitem nicht ausgleichen kann. Die Erde wird also leichter! Klar ist aber auch: Angesichts der Gesamtmasse der Erde ist das alles völlig bedeutungslos.

  1273. #1295 Mirko
    25. Mai 2019

    Neue Entdeckung am CERN bzgl CP Violation?

    Hallo Florian @ Co. Gibt da (leider wenige) Berichte, dass am CERN ein neuer Unterschied zwischen Materie und Antimaterie gefunden wurde beim Zerfall von D° bzw Charm-Quarks. Will nicht behaupten, dass ich auch nur die Hälfte verstanden hab. Warum nach diesen Unterschieden gesucht wird, ist mir grundsätzlich klar.
    Kannst du mal darüber berichten .. auf etwas volksnahe Weise? Auch wenn es im Kern (schönes Wortspiel, oder?) nicht Astronomie ist.

    Danke!

  1274. #1296 Florian Freistetter
    25. Mai 2019

    @Mirko: Schau ich mir an!

  1275. #1297 Jutta Grübler
    Aschach\Steyr
    28. Mai 2019

    Hallo Florian,
    wie kann ich feststellen ob ein außergewöhnlicher Stein ein Meteorit ist oder nur ein außergewöhnlicher Stein.
    Lg Josef

  1276. #1298 Florian Freistetter
    28. Mai 2019

    @Josef: Selbst ist sowas schwer zu machen. Das können am Ende nur Experten zweifelsfrei tun. Auf jeden Fall muss ein Meteorit metallhaltig sein. Wenn ein Magnet nicht dran haften bleibt, ist es keiner. Aber selbst dann kann es noch etwas anderes sein. Um das zweifelsfrei zu klären bräuchte man Chemie um die Widmanstätten-Strukturen sichtbar zu machen: https://de.wikipedia.org/wiki/Widmanst%C3%A4tten-Struktur

  1277. #1299 Schufti
    30. Mai 2019

    Hallo!
    In deinen Podcasts erzählst du oft das Asteroiden oder Planeten aus dem System geworfen werden. Kannst du bitte einmal genau erklären wie das abläuft?
    Was ich in diesem Zusammenhang noch nicht verstehe ist warum bei der Planetaren Migration Jupiter Asteroiden nach außen schleudert, die anderen Planeten aber nach innen.
    Danke im voraus

  1278. #1300 Peter Paul
    1. Juni 2019

    @Alderamin #371

    Die Sternentstehungszone in einem Spiralnebel bewegt sich nun langsam um die Spiralgalaxie herum,

    Hast du auch ´ne Ahnung davon, in welche Richtung sich diese Spiralarme drehen ?
    a) In die gleiche Richtung wie die Sterne, aber viel langsamer?”
    b) In die konvexe Richtung (in Bauch-Richtung) oder in die konkave?
    c) Oder ist jeweils beides möglich?
    d) Oder weiß man das alles noch gar nicht recht?

  1279. #1301 Alderamin
    1. Juni 2019

    @Peter Paul

    So rum.. Gilt, meine ich, allgemein für Spiralgalaxien, dass die Arme nachhängen, wie bei einem Rasensprenger. Deutet auf ein Von-innen-nach-außen hin, aber da bin ich nicht sicher.

  1280. #1302 Peter Paul
    1. Juni 2019

    @Alderamin
    Ich glaube, mit der Rotationsrichtung ist da wohl die Rotation der Spiralarme gemeint, oder vielleicht doch die Richtung der Sterne, oder vielleicht doch beides?
    Es ist wirklich sehr schwer, darüber Informationen zu bekommen, aber trotzdem erst mal Danke!.

  1281. #1303 Alderamin
    1. Juni 2019

    @Peter Paul

    Die Arme rotieren im gleichen Drehsinn wie die Sterne. Bin mir nur nicht sicher, welche Komponente schneller ist. Wäre vielleicht mal einen Artikel wert.

  1282. #1304 UMa
    1. Juni 2019

    @Peter Paul:
    Wie Alderamin schon gesagt hat: Die Arme rotieren im gleichen Drehsinn wie die Sterne.

    Die Arme rotieren dabei mit ungefähr gleicher Winkelgeschwindigkeit unabhängig von der Entfernung vom Milchstraßenzentrum, sonst hätten sie sich längst völlig aufgewickelt.

    Die Sterne der dünnen Scheibe rotieren dagegen mit ungefähr gleicher absoluter Geschwindigkeit von 235km/s, zumindest wenn sie nicht ganz innen oder evtl. außen sind.

    Daher sollte es eine Entfernung vom Zentrum geben in der die Sterne gleich schnell wie die Arme rotieren, den Korotationsradius. Weiter innen sind die Sterne der dünnen Scheibe schneller als die Arme und laufen von hinten durch.
    Weiter außen sind die Arme schneller, als die Sterne.

    Bis vor kurzem war die Rotationsgeschwindigkeit der Arme, und damit der Korotationsradius noch nicht genau bekannt. Man vermutete ihn ungefähr in der Größenordnung der Entfernung der Sonne oder knapp außerhalb.
    Vor ein paar Jahren war der beste Wert für den Korotationsradius glaube ich bei etwa 9000 pc, mittlerweile eher bei 8500 pc.
    Zum Vergleich, die Entfernung der Sonne zum Milchstraßenzentrum ist 8178 pc, auch erst kürzlich genauer bestimmt. Die Messfehler müsste ich erst nachschlagen.

  1283. #1305 Peter Paul
    2. Juni 2019

    @Uma, Alderamin
    Wenn das stimmt, was du sagst, wäre vielleicht gleich die Frage mitbeantwortet, ob die Arme mit der konvexen Seite nach vorne rotieren oder mit der konkaven.
    Für konvex spräche dann vielleicht: Innen sind die Sterne und das Gas schneller, was eine Wechselwirkung mit dem Gas in den Armen bringt, die dieses Gas etwas in Rotations-Richtung schiebt. Weiter außen sind die Sterne langsamer, was ganz analog die Arme dort etwas nach hinten schiebt. Dadurch wäre dann auch die Biegung “erklärt”.
    Ich schreibe “erklärt”, weil mir das eigentlich so lange als reine Spekulation erscheint, wie man keine richtige Erklärung für das Entstehen der Spiralarme hat.

  1284. #1306 Peter Paul
    2. Juni 2019

    @Uma, Alderamin
    Jetzt habe ich gemerkt, dass man auch genau das Gegenteil daraus ableiten könnte, wenn man die Spiralarme wie (Auto-)Staus betrachtet. Diese Veranschaulichung der Dinge habe ich schon in mehreren Quellen gefunden.
    So ein Stau wächst ja nach hinten, wenn von hinten schnelle Autos bzw. Sterne in ihn einfahren, d.h, er wächst gegen die Fahrtrichtung. Der “Bauch” ginge, nachdieser Argumentation gegen die Rotation.
    Und er löst sich nach hinten auf, wenn weitere Sterne nach hinten weggehen.
    Zusammen hieße das: die Rotation ginge in die konkave Richtung.

  1285. #1307 Peter Paul
    4. Juni 2019

    Die Sache mit den Dichtewellen und den Spiralarmen scheint ungeheuer komlex zu sein.

    Siehe : https://www.spektrum.de/magazin/sternentstehung-in-spiralgalaxien/826767

    Der Artikel ist sehr ausführlich und eigentlich für Laien geschrieben, aber schaut selbst ob ihr da von profitieren könnt.

  1286. #1308 Mirko
    HH
    8. Juni 2019

    Hab zwar auf meine letzte Frage noch keine Antwort, versuche es aber erneut. Was ist von folgender News zu halten? Der Guardian ist ja eigentlich seriös und der Link hoffentlich lesbar. Wenn sich das bestätigen sollte, wäre die QT dann nicht Geschichte? Hab es absichtlich etwas drastisch formuliert…

    https://www.theguardian.com/science/2019/jun/03/feline-fine-fate-of-schrodingers-cat-can-be-reversed-study?utm_term=RWRpdG9yaWFsX0xhYk5vdGVzLTE5MDYwNw%3D%3D&utm_source=esp&utm_medium=Email&utm_campaign=LabNotes&CMP=labnotes_email

  1287. #1309 Ambi Valent
    14. Juni 2019

    @Schufti

    Zuerst machen alle Riesenplaneten das Gleiche, nämlich Asteroiden ablenken. Und weil die äußeren Asteroiden, die zu dem Zeitpunkt noch übrig waren, außerhalb der Planetenbahn waren, werden sie nach innen abgelenkt.

    Dann gibt es einen zweiten Effekt: die neue Asteroidenbahn kommt immer wieder zurück an den Punkt ihrer letzten Ablenkung, also würde der Planet den Asteroiden wieder und wieder ablenken (solange der Asteroid nicht direkt kollidiert).

    Bei den äußeren Planeten ist es nun so, dass der Asteroid weiter innen einem weiteren Planeten begegnet, und er deshalb nicht wieder zur äußeren Planetenbahn zurückkommt.

    Jupiter hat aber keine inneren Planeten mehr. Also wird der Asteroid Jupiter immer wieder begegnen, bis eine von drei Varianten passiert:
    a) Der Asteroid kollidiert mit Jupiter.
    b) Der Asteroid wird so extrem abgelenkt, dass er ganz aus dem System fliegt
    c) Der Asteroid wird auf eine Bahn gelenkt, wo er auf einen der äußeren Planeten trifft.

    Die Möglichkeit c ist aber viel unwahrscheinlicher als die, dass der Asteroid von einem der äußeren Planeten zu Jupiter gelenkt wird. Jupiter hat da einen viel stärkeren Einfluss, einmal wegen seiner großen Masse und darum stärkeren Gravitation, und andererseits, weil für einen Asteroden in Jupiterentfernung von der Sonne eben weniger Abstand zu Jupiter möglich ist.

    Generell kreist ein Asteroid ja im 3-dimensionalen Raum, und seine Bahn ist mehr oder weniger gegen die Planetenbahnen geneigt. Also kann er den äußeren Planeten viel leichter aus dem Weg gehen als Jupiter.

    Insgesamt ist es natürlich so, dass auch vorkommt, dass ein Asteroid zum Beispiel zuerst von Jupiter abgelenkt wird und danach von Saturn, und so nicht wieder zur Jupiterbahn zurückkommt. Der andere Fall ist aber viel wahrscheinlicher, und so summiert sich alles so auf, dass ein Asteroid, der aus dem Sonnensystem geschleudert wird, am ehesten von Jupiter abgelenkt wurde, und vorher von den anderen Planeten hin zu Jupiter.

  1288. #1310 Ksaima
    Frankfurt
    17. Juli 2019

    Hallo Herr Freistetter

    Ich bin neu hier und hätte auch direkt schon mal drei Fragen.

    1. Je näher man sich dem Ereignishorizont eines Schwarzen Lochs annähert desto langsamer vergeht die Zeit. Am Ereignishorizont geht die Zeit gegen Unendlich. Würde sich nicht ein Riesiger Stau bilden da alle Materie vor dem Ereignishorizont förmlich in der Zeit festfriert. Und würde das Universum und mit ihm das Schwarze Loch nicht schon vorher in einem Big Rip sterben bevor das Objekt jemals im inneren ankommt?

    2. Nehmen wir an ein Magnetar und Schwarzes Loch bilden ein enges Doppelsternsystem. Könnten die Magnetfelder des Magnetars das Schwarze Loch unbeschadet durchdringen oder würde es im Schwarzen Loch zerstört werden?

    3. Warum fällt bei der Hawkingstrahlung immer das Teilchen mit der Negativen Masse ins Schwarze Loch und nicht sein Gegenstück?

    Tut mir leid das ich hier so einfach mit 3 Fragen hereinplatze, aber die brennen mir schon seit längerem unter den Fingern

  1289. #1311 Florian Freistetter
    18. Juli 2019

    @Ksaima: Zu Frage 3: Das ist ein weit verbreitete falsche Vorstellung. Ich hab probiert, das hier ausführlich zu erklären: https://scienceblogs.de/astrodicticum-simplex/2017/06/16/sternengeschichten-folge-238-die-hawking-strahlung/

    Zu 2) Magnetfelder können den Ereignishorizont nicht duchdringen; das ist ja auch Elektromagentismus. Und der kommt genau so wenig wie alles andere aus nem schwarzen Loch raus.

    Zu 1) Das ist ein Missverständnis, die Sache mit der Verlangsamung der Zeit gilt nur für weit entfernte Beobachter. Nicht für das Material, das ins Loch fällt. Das fällt ins Loch und aus seiner Sicht verlangsamt sich da gar nix.

    Ich hab hier noch ein wenig mehr zum Thema geschrieben: https://scienceblogs.de/astrodicticum-simplex/2018/03/25/die-wissenschaftliche-arbeit-von-stephen-hawking/

    Und allgemein gibts in meinem Blog jede Menge Artikel über schwarze Löcher (schau einfach mal mit der Blog-Suchfunktion nach). Wenn du Zugang zu ner Bücherei hast (oder dir das Buch kaufen willst): In dem Buch “Warum landen Asteroiden immer in Kratern?” habe ich auch ein Kapitel über die Sache mit der Zeit beim Fall ins schwarze Loch geschrieben.

  1290. #1312 Ksaima
    Frankfurt
    19. Juli 2019

    Hallo Herr Freistetter

    Vielen dank für die Antworten.

  1291. #1313 Karl-Heinz
    20. Juli 2019

    Meine Frage lautet:

    Besitzen rotierende schwarze Löcher ein magnetisches Dipolmoment?

  1292. #1314 Karl-Heinz
    20. Juli 2019

    myself

    Meine Frage lautet:
    Besitzen elektrisch geladene rotierende schwarze Löcher ein magnetisches Dipolmoment?

    Wenn ja, wie groß ist das magnetische Dipolmoment, als Funktion der Masse, Ladung und Drehimpuls? Kann man diese Art von magnetischem Dipolmoment in der Praxis vernachlässigen?

  1293. #1315 Oliver Mulzet
    Wiener Neudorf, Österreich
    5. August 2019

    Lieber Florian,
    Meine Frage bezieht sich auf Dunkle Energie bzw. Dunkle Materie:
    Man hört immer wieder, dass nur ca. 10% des sichtbaren Universums “sichtbar” sind und 90% unsichtbar. Das ergibt sich meines Wissens daraus, dass die Bewegung der sichtbaren Materie nur in Verbindung mit einer noch größeren Masse die man nicht sieht erklärt werden kann. (Bitte korrigiere falls das nicht korrekt ist) Es wird hier immer davon gesprochen, dass man die dunkle Materie und Energie indirekt nachweisen kann, aber der Ursprung unbekannt ist. (es ist halt einfach da)
    Hat man hier eigentlich schon daran gedacht, dass es sich “einfach” um eine große Anzahl an schwarzen Löchern handelt, die im Raum verteilt sind und nicht sichtbar sind, weil sich einfach keine Materie in unmittelbarer Umgebung befindet, die es zum Leuchten bringt? Liebe Grüße,
    Oliver

  1294. #1316 Florian Freistetter
    5. August 2019

    @Oliver: Dunkle Materie und dunkle Energie sind zwei ganz unterschiedliche Dinge. Du scheinst von der dunklen Materie zu sprechen. Und ja, man hat die Sache mit schwarzen Löchern natürlich untersucht. Aber das reicht nicht bzw. würde man das auf andere Weise merken. Hier hab ich ne Serie zu dem Thema geschrieben wo alles erklärt ist: https://scienceblogs.de/astrodicticum-simplex/2013/06/26/dunkle-welten-alles-uber-dunkle-materie-die-komplette-serie/

  1295. #1317 UMa
    5. August 2019

    Lieber Oliver,

    ja, allerdings dürften es dann keine schwarzen Löcher sein, die als Überreste schwerer Sterne entstanden sind.

    Eine Möglichkeit wären aber schwarze Löcher, die schon vor der primordialen (ursprünglichen) Nukleosynthese in den ersten Sekunden entstanden sind, da die Dichte an der normalen baryonischen Materie zu diesem Zeitpunkt durch die beobachteten Verhältnisse an Helium Deuterium usw. nicht wesentlich höher sein kann.

    Das wären dann sogenannte primordiale schwarze Löcher.

    Allerdings ist dann die Frage, welche Verteilung im Raum und welche Massen diese primordiale schwarzen Löcher haben könnten, so dass sie durch die Suche nach MACHOs (Massive Astrophysical Compact Halo Objects) im Halo der Milchstraße über den Mikrolinseneffekt noch nicht entdeckt wurden.

  1296. #1318 Peter
    6. August 2019

    Um wie viele Kilometer müsste der durchschnittliche Abstand der Erde zur Sonne größer sein, um die globale Durchschnittstemperatur um 1 °C (genauer um 1 K) zu senken?
    Vereinfachend sei dabei angenommen, dass die Sonnenaktivität so groß ist wie die letzten 100 Jahre und konstant bleibt. Außerdem sollen die heutigen Werte für die Neigung der Erdachse, die Exzentrizität der Erdbahn und die Konzentrationen der Treibhausgase in der Erdatmosphäre konstant bleiben.

  1297. #1319 Ambi Valent
    7. August 2019

    @Peter

    Ich kann leider nicht sagen, wie es mit Rückkoppelungen laufen würde, aber ohne Rückkoppelungen lässt sich ein Schätzwert berechnen.

    Dann wäre bei 1% mehr Abstand 2% weniger Energie da und damit 0,5% weniger Temperatur in Kelvin. 1 Kelvin weniger als Ziel wären etwa 1/300 Temperatur weniger und etwa 1/150 Abstand mehr von der Sonne, also etwa 1 Million Kilometer.

    Bei positiver Rückkoppelung bräuchte man weniger Kilometer für dasselbe Temperaturziel.

    Aber selbst wenn man die Erde verschieben könnte, würde doe Verschiebung zwar dem Treibhauseffekt entgegenwirken, aber die Meere wären durch mehr CO2 immer noch saurer und damit weniger lebensfreundlich.

  1298. #1320 Karl-Heinz
    7. August 2019

    @Peter

    Wenn du die Erde 10 Millionen km näher an die Sonne schiebst ( 6,7 % näher als heute), dann tritt nach Rasool und De Bergh (1970) ein galoppierender Treibhauseffekt ein.

    https://de.wikipedia.org/wiki/Galoppierender_Treibhauseffekt

  1299. #1321 Paul
    München
    25. August 2019

    Hallo Herr Freistetter,

    wurde die Zeitgleichung neu definiert? Auf älteren drehbaren Sternkarten, die ich habe (vom Kosmos-Verlag und auch “Sirius” vom Hallwag-Verlag) sind die vier Nullstellen im Jahresverlauf so eingezeichnet, wie es der (bisherigen) Fachliteratur entspricht.

    Vor einigen Wochen kaufte ich eine drehbare Sternkarte vom Kosmos-Verlag, nach Angabe eine Neuauflage von 2017, und stellte fest, dass die Skala für die “wahre Sonne” deutlich verschoben ist gegenüber den bisherigen Sternkarten. Ist das die Folge einer neuen Definition, oder ein Fehler in der Herstellung der Karte?

  1300. #1322 Paul
    München
    27. August 2019

    Nachdem ich feststellte, dass dies bei der drehbaren Sternkarte aus dem Oculum-Verlag genauso ist, nehme ich an, dass die Änderung darauf zurückzuführen ist, dass die Karte vom Kosmos-Verlag früher auf den 15. Längengrad bezogen war, nun jedoch auf den 10. Längengrad. Verwirrend, da man in der früheren Version z.B. die vier Nullstellen im Jahresverlauf unmittelbar durch Vergleich der Skalen erkennen konnte.

  1301. #1323 Karl-Heinz
    27. August 2019

    @Paul

    Die drehbare Sternkarte hat meine Neugier geweckt. Sorry für Link.

    https://www.abenteuer-sterne.de/wp-content/uploads/2016/10/ausfuehrliche_anleitung_kosmos_sternkarte.pdf

  1302. #1324 Peter
    27. August 2019

    @ Paul:

    Ein Bezug auf den 15. Längengrad Ost macht eigentlich Sinn, weil an diesem Längengrad die Zeit der Mitteleuropäischen Zeitzone (also die MEZ und von Anfang März bis Ende Oktober die MESZ) mit der mittleren Ortszeit übereinstimmt.

    Die Verschiebung auf 10° Ost ist vermutlich damit zu erklären, dass 15° Ost in Deutschland sehr weit im Osten – nämlich in Görlitz – schneidet. Fast alle anderen Orte in Deutschland liegen mehr o. weniger weit westlich vom 15. östlichen Längenmeridian. Dadurch ergibt sich eine mehr o. weniger starke Abweichung von Zeitangaben (z. B. wann geht ein Stern auf, in MEZ oder MESZ) zur mittleren Ortszeit und zur wahren Ortszeit von bis zu ca. 45 Minuten für Aachen (Quelle: https://www.timeanddate.de/astronomie/sonnenzeit-zonenzeit ).

    10° Ost (z. B. Hamburg) liegt fast an der West-Ost-Mittellinie von Deutschland.
    Wenn man 10° Ost als Referenz nimmt, sind die Zeitabweichungen bis zur westlichen deutschen Grenze und bis zur östlichen deutschen Grenze kleiner, als wenn man 15° Ost als Referenz nimmt.
    Wenn man 10° Ost als Referenz nimmt, muss man dabei aber beachten, dass die mittlere Ortszeit von der Zonenzeit (MEZ oder MESZ) abweicht.

  1303. #1325 UMa
    1. September 2019

    @Peter

    Um wie viele Kilometer müsste der durchschnittliche Abstand der Erde zur Sonne größer sein, um die globale Durchschnittstemperatur um 1 °C (genauer um 1 K) zu senken?

    Genauer lässt sich angeben, um wie viele Kilometer der Abstand der Erde zur Sonne größer sein müsste, um bei einer Verdopplung des CO2 Gehaltes der Atmosphäre die Durchschnittstemperatur konstant zu halten. Das wären in diesem Fall 1,1 bis 1,2 Millionen Kilometer.

    Um daraus den Wert für 1 K Temperaturabsenkung zu machen, benötigt man die Klimasensitivität, die leider noch nicht sehr genau bekannt ist, da die Rückkopplung der Wolken noch sehr unsicher ist.

    Nimmt man an, dass die Konzentration aller Treibhausgase, außer Wasserdampf (hängt sehr von der Temperatur ab), konstant bleibt, komme ich auf 400000 Kilometer, mit einem Unsicherheitsbereich zwischen 250000 Kilometer und 700000 Kilometern.
    Das ist etwa die Entfernung zwischen Erde und Mond.

    Allerdings verändert sich der Abstand zwischen Erde und Sonne, abgesehen von der Exzentrizität der Erdbahn, nicht so stark. In der nächsten Milliarde Jahre wird der Abstand durch den Massenverlust der Sonne um 14000 Kilometer größer.

  1304. #1326 Helena
    6. September 2019

    Haben die Expansion des Universums und die Zunahme der Entropie etwas miteinander zu tun?

  1305. #1327 Dieter Schläger
    Deisenhausen
    21. September 2019

    Kurz vor der Jahrtausendwende wurde entdeckt, dass die Expansion des Universums seit einigen Milliarden Jahren beschleunigt abläuft. Vordem ist sie also wohl durch die Gravitation bedingt gebremst abgelaufen. Aus dem immer Größeren Abstand der Massen untereinander lässt sich schließen, dass diese “Bremsung” sich wohl im Laufe der Zeit verringert haben muss.
    In der Physik ist immer die Rede von 4 Kräften, von denen unser Universum beherrscht wird. Die schwächste davon ist die Gravitation. Irgendwie ist mir bei der Überlegung zu diesem Thema die Idee gekommen, dass es doch eine fünfte Kraft geben könnte, die dem Raum innewohnt, Linear mit dem Abstand zunimmt, sehr viel schwächer ist als die Gravitation und natürlich abstoßend ist.
    Stellt sich mir natürlich die Frage, ob schon jemand vor mir auf diese Idee gekommen ist und warum sich diese Idee nicht durchgesetzt hat. Ist die Dunkle Energie nicht Ausdruck einer “fünften” Kraft?

  1306. #1328 Alderamin
    21. September 2019

    @Dieter Schläger

    Irgendwie ist mir bei der Überlegung zu diesem Thema die Idee gekommen, dass es doch eine fünfte Kraft geben könnte, die dem Raum innewohnt, Linear mit dem Abstand zunimmt, sehr viel schwächer ist als die Gravitation und natürlich abstoßend ist.

    Genau das leistet aber schon die Gravitation nach der Allgemeinen Relativitätstheorie bei negativem Druck. Warum also eine neue Kraft postulieren, wenn die 4 bekannten ausreichen?

  1307. #1329 Uwe Kerstel
    Löhne
    3. Oktober 2019

    Man Rätselt ja noch wieso die Erde einen so großen Mond hat der auch noch in der richtigen Position ist um das Wetter zu stabilisieren. Wenn man in der Lage wäre ein Objekt von der Größe des Mondes wie eine Sonde zu bewegen, bestünde die Möglichkeit ihn auf die derzeitige Position zu navigieren?

  1308. #1330 Magnetar
    Wien
    6. November 2019

    Hallo Herr Freistetter,

    meine Frage lautet:
    Welches Bild zeichnet sich von der Erde aus gesehen am Himmel ab, wenn es eine Sonnenfinsternis gäbe und einen weiteren Mond am Himmel?
    Würde man den zweiten Mond am Himmel sehen können oder wäre er nicht sichtbar?

    Danke und Liebe Grüße

  1309. #1331 Magnetar
    Wien
    7. November 2019

    Verzeihung, habe mich nicht klar ausgedrückt.
    Ich meinte eine totale Sonnenfinsternis! und ein zusätzlicher Mond am Himmelsbild.
    Wäre der zweite Mond überhaupt sichtbar und wenn ja wie würde er aussehen, stünde man im Kernschatten und macht es einen Unterschied wenn man sich nicht im Kernschatten befindet? Leuchten sollte er doch nicht oder? Da er ja das Licht der Sonne nicht reflektiert?

  1310. #1332 felix
    Niederbayern
    7. November 2019

    Hallo ” Magnetar”

    Natürlich sähe man den anderen Mond wenn er an einem anderen Ort steht.
    Der Schatten von “unserem ” Mond ist ja nur maximal so breit wie er selbst (3470 km) und der Kernschatten wo die Sonne komplett verschwindet reicht grad mal so bis zur Erde.
    Deshalb dauern totale Sonnenfinsternisse auch nur ein paar Minuten und außerhalb des Kernschattens scheint die Sonne wenn auch die Sonne angebissen erscheint. Unser Mond ist dann deshalb nicht hell weil wir ja genau in den Schatten schauen.
    Ein Mond an anderer Stelle wird stärker von der Seite beleuchtet und ist deshalb sichtbar.
    Es gibt übrigens schöne Bilder von der Raumsonde Cassini die jahrelang um Saturn geflogen ist und viele Bilder mit seinen Monden aufgenommen hat.
    Ich hoffe ich konnte Deine Frage beantworten.
    Nochwas – sowas kann man auch im Modell nachstellen.
    Man nehme ein paar Bälle, Kugeln etc und beleuchte sie aus größerer Entfernung. Da kannst Du die Phasen (Sichel, Halbmond, Vollmond) nachstellen sowie Sonnen- oder Mondfinsternisse

    Gruß Felix

  1311. #1333 Klaus
    15. November 2019

    Läuft die Zeit auf der Erde und einem Planeten dichter am Zentrum der Milchstraße synchron? Wie groß ist die Abweichung in etwa?

  1312. #1334 Karl-Heinz
    16. November 2019

    @Klaus

    Nicht synchron zu einem Beobachter ausserhalb der Milchstraße. Zwei Dinge müssen berücksichtigt werden. Zeitdilatation durch Gravitationspotential und Geschwindigkeit durch Rotation. Kann man wirklich sehr einfach ausrechnen. 😉

  1313. #1335 Klaus
    18. November 2019

    @Karl-Heinz Danke für die Antwort. Ich habe mir die Wiki-Seite zur Zeitdilatation angesehen. Mit lateinischen Buchstaben in Mathe-Formeln komm ich meist noch klar. Aber griechische Buchstaben und komische Formelzeichen überfordern mich zugegebenermaßen etwas. Wenn es so einfach zu berechnen ist, wie wäre es mit ein paar Eckwerten? Wenn die Erde der neutrale Punkt ist, wie hoch ist der Zeitlaufunterschied sagen wie in 100.000 Lj. Entfernung vom Zentrum, in 50.000 und 1.000 Lichtjahren? Ist es ein lineares oder expentionelles Verhältnis?

  1314. #1336 Jonas
    1. Dezember 2019

    Frage zum Higgs Boson

    Es wird gesagt, dass das higgs Boson einst das ganze Universum zerstören wird. Ist das einfach nur Angst gemache der Wissenschaftler? Oder besteht die Gefahr wirklich :/?

  1315. #1337 Florian Freistetter
    1. Dezember 2019

    Das hast du doch nebenan schon mal gefragt, oder? Die Antwort ist immer noch die gleiche: Das ist etwas, vor dem man absolut keine Angst zu haben braucht! https://scienceblogs.de/astrodicticum-simplex/2014/09/08/schlechte-schlagzeilen-4-warnung-vor-dem-gottesteilchen-stephen-hawking-befuerchtet-kollaps-des-universums/?all=1

  1316. #1338 Ralph Scheider
    Trier
    11. Januar 2020

    Hallo Florian,
    In dem Beitrag “Wie ist das Sonnensystem innerhalb der Milchstraße ausgerichtet und wie bewegt es sich durch die Galaxis?” schilderst du, dass das Sonnensystem sich in seiner Bahn um das Milchstraßenzentrum in einem Auf und Ab von ingesamt etwa 500 Lj bewegt. Welche Ursache hat diese Schwankung?

    Vielen Dank für die Antwort im voraus
    Gruß Ralph

  1317. #1339 Florian Freistetter
    11. Januar 2020

    @Ralph: Die Gravitation der anderen Sterne in der Milchstraße

  1318. #1340 Jan Stahlhut
    Hochdorf
    12. Januar 2020

    Ich habe mir wiedereinmal zum Thema Ausdehnung des Universums meine Gedanken gemacht (bin kein Astronom und hab auch nicht studiert, aber ich interessiere mich dafür) und hab für mich jetzt eine Theorie anhand geltender physikalischer Gesetze (Energie verschwindet nicht, und Gravitation) aufgestellt, die für mich persönlich (mit meinem ganzen Wissen/Unwissen) und vieleicht anderer die meiner theoretischen Überlegung folgen können, vieleicht einige Fragen beantworten könnten.

    Nun Gravitation ist die bestimmende Kraft in unserem Universum, und es wurde ja nun ein “Anfang” des Universums (Urknall) nachgewiesen. Dadurch stellt sich mir persönlich das Bild eines Rings (bildlich aussehend wie ein Donat, mit dem Urknall als Loch dargestellt) dar.

    Es ist belegt das Maße, Energie mit Kraft X weggeschleudert wurde und sich dadurch ja verklumpt hat (es entstanden Galaxien, Sonnensysteme etc), durch immer mehr Verklumpung also immer mehr Masse sich zusammen fasste, diese quasi durch die Gravitation des Urknalls (man könnte Sagen eine Art schwarzes Urloch) auf elyptische Bahnen irgendwann wieder zum Zentrum zurück bewegen und (somit die Aussage das Energie nicht verschwindet also auch) irgendwann wieder zum Ursprung zurückkehrt.

    Daher würde ich behaupten das der Urknall vortwärend stattfindet, wenn man das plastische Beispiel Donat, der auf dem Tisch liegt, sich bildlich versucht vorzustellen das alles vom Loch aus Richtung äußeren Donatrand bewegt und dann auf der Unterseite sich wieder Richtung Loch bewegt (ganz genau wie bei dem “herkomlichen” schwarzen Loch nur eben in ultra groß), quasi alles wieder durch ein “Nadelör” geführt wird (also ein Ring um den Urknall), die Energie, Hitze, Gravitation etc so groß wird das es auf neuem wieder auf der Oberseite heraus tritt.

    Desweiteren würde sich eventuell die (nicht erklärbare) dunkle Energie zwischen dem oberen Teil des Rings (vom Knall wegbewegend) und des unteren Teil (auf den Knall zu bewegend) befinden, entstanden durch die entgegengesetzten Richtungen in der Mitte des Ringes um den Urknall selbst (was wiederum ein Gravitationsfeld erzeugen müsste).

    Desweiteren könnte man sich so vieleicht auch die Funktionsweisen “normaler” schwarzer Löcher erklären (eine Seite geht rein eine andere hinaus)

    Ich lehne mich jetzt mal ganz weit hinaus mit meiner theoretischen Vorstellung, das einige Licht-/Wärme-/Energie-/Gravitationsquellen im Universum die nicht auf Sterne oder ähnliches zurückzuführen werden können, eventuell quasi die Rückseite (oder jeh nach Blickwinkel andere Ansicht) eines Loches sind.

    So weit meine Vorstellung/Denkansätze/Theorien.

    Mich würde interessieren was sie und andere davon halten, eventuelle Berechnungen oder ähnliches anstellen könnten (falls möglich) und mich eventuell sogar persönlich anschreiben würden.

  1319. #1341 Dieter Schläger
    Deisenhausen
    19. April 2020

    Man liest immer wieder, dass beim Sturz von Materie in ein schwarzes Loch – resp. beim Verdunsten desselben durch Hawking-Strahlung information verloren geht. Laut Relativitätstheorie sollte doch eigentlich aus unserer Sicht gar nichts in ein Schwarzes Loch hineinfallen, da die Zeitdilatation am Ereignishorizont ja unendlich wird, die Zeit – und damit die Geschwindigkeit des Einsturzes 0 wird. Wie passt denn das zusammen?

  1320. #1342 Alexander
    8. Mai 2020

    Ich möchte nur schnell etwas in den Briefkasten einwerfen:

    https://www.scinexx.de/news/technik/elemente-unerklaerliche-gesetzmaessigkeit-entdeckt/

    Klingt spannend, oder?

  1321. #1343 Karl-Heinz
    10. Mai 2020

    @Dieter Schläger zu #1341

    Wenn man sehr viel Materie in die Nähe des Schwarzen Loches bringt, hat dies Auswirkung auf den Ereignishorizont. Der Ereignishorizont wird zunehmend größer, obwohl keine Masse diesen überschreitet. Es setzt eine Dynamik ein und Schwupp­di­wupp ist auch diese Materie aus unserer Sicht ein Teil des Schwarzen Loches. Wenn also sehr viel Materie im Spiel ist, gilt die Erkenntnis, dass diese aus unserer Sicht den Ereignishorizont nie überqueren wird, nicht mehr.

  1322. #1344 Dennis M.
    Dortmund
    25. Juni 2020

    Habe gehört, das man Entfernungen durch Rotverschiebungen misst, nun stellt sich mir die frage wie man die Rotation der Galaxien mitberechnet, oder die eigene bewegung zum mittelpunkt der Galaxie

  1323. #1345 Dietmar Fischer
    64653 Lorsch
    8. Oktober 2020

    Sehr geehrte Damen und Herren,
    ich habe eine Verständnisfrage.
    Wenn ein Stern ein Lichtjahr von uns entfernt ist, beobachten Sie dann nicht nur die Vergangenheit dieses Sternes? Es könnte doch sein, dass der _Stern real gar nicht mehr existiert. Das Licht ist also noch unterwegs, obwohl der Stern gar nicht mehr existent ist.
    Vielen Dank
    Dietmar V. Fischer

  1324. #1346 Jan
    8. Oktober 2020

    @Dietmar Fischer:

    Ja.

    Aber Sterne leben sehr lange; unsere Sonne ist z.B. etwa 5 Milliarden Jahre alt und man geht davon aus, dass sie nochmal etwa genauso lange durchhalten wird. Die meiste Zeit während dieser Lebensdauer wird sich innerhalb von einem Jahr oder auch von ein paar tausend Jahren nicht allzu viel verändern. Wenn man heute einen Stern in der Milchstraße beobachtet, ist die Wahrscheinlichkeit also sehr hoch, dass dieser Stern immer noch existiert.

    Selbst bei einem Stern wie Beteigeuze, von dem man erwartet, dass er in astronomischer Sicht bald explodieren wird, ist das noch so. Denn mit “bald” ist dort üblicherweise ein Zeitpunkt in den nächsten zehntausend bis hunderttausend Jahren gemeint. Und das Licht von Beteigeuze braucht nur ein paar hundert Jahre bis zu uns.

    Zudem interessiert man sich in der Astronomie meistens gar nicht so sehr für die Details eines einzelnen bestimmten Objekts. Sondern man interessiert sich eher für allgemeinere Zusammenhänge und dafür, welche Eigenschaften den Objekten einer Kategorie gemein sind und wodurch sie sich von denen anderer Kategorien unterscheiden. Von daher würde man einen bestimmten Stern den man beobachtet eher als Repräsentanten ähnlicher Sterne (ähnliche Masse, Alter, Zusammensetzung, …) sehen. Und dann spielt es keine besonders große Rolle mehr, dass man nur die Vergangenheit dieses Sterns sieht.

    Wirklich wichtig wird dieser Effekt aber bei sehr weit entfernten Galaxien, von denen das Licht Milliarden von Jahren braucht, um uns zu erreichen. Und dort verwendet man diesen Effekt bewusst, um Dinge über das frühe Universum herauszufinden.

    Wenn man es ganz genau nehmen will, gibt es auch noch das Problem mit der Relativität der Gleichzeitigkeit. Je nachdem wie schnell und in welche Richtung man sich bewegt, betrachtet man andere Ereignisse an einem weit entfernten Ort als gleichzeitig zu “jetzt”. In diesem Sinn gibt es kein objektives “jetzt” bei einem fernen Stern oder einer fernen Galaxie. Und die Frage, ob dieser Stern “jetzt” noch existiert, ist nicht sinnvoll. Aber ich vermute, dass dieser Effekt innerhalb der Milchstraße nicht allzu groß ist (wegen der vergleichsweise kleinen Entfernungen und Relativgeschwindigkeiten).

  1325. #1347 Wohe
    Berlin
    9. Oktober 2020

    Hallo Freunde,

    in letzter Zeit las ich des Öfteren von auch großen Namen, die ein zyklisches Universum für möglich halten (Penrose u.a.), d.h. Expansion, Kontraktion, Expansion, … …
    Allerdings wird meines Wissens nicht angezweifelt, dass die Gravitation im Universum für dessen sich-Zusammenziehen nicht ausreicht (Florian in diesem Blog u.a.), d.h. ewige Expansion.
    Könnt Ihr mir diesen Widerspruch erklären?

    Vielen Dank
    Wohe

  1326. #1348 Alderamin
    10. Oktober 2020

    @Wohe

    Penroses Modell habe ich hier vorgestellt.

    Ansonsten gilt, dass nicht sicher ist, ob die Dunkle Energie konstant ist oder vielleicht auch irgendwann anziehend werden könnte. Dann könnte das Universum auch wieder kollabieren.

    Und dann gibt es noch ein paar Leute, die glauben, dass sich die Zeitrichtung irgendwann umkehren könnte (Gold-Universum). Siehe diesen Artikel.

  1327. #1349 Wohe
    Berlin
    11. Oktober 2020

    @Alderamin,

    vielen Dank für Deine Erläuterung. Das mit der variablen DE scheint mir als Laien sehr plausibel.

    MfG Wohe

  1328. #1350 Kmpr_rak
    25. Oktober 2020

    Ich habe nur zwei Fragen, und zwar

    1) wie errechne ich aus Galaktischen Koordinaten Supergalaktische Koordinaten?

    2) wie errechne ich aus Galaktischen oder auch Ekliptikalen Koordinaten räumliche x-y-z Koordinaten?

    Ich würde mich über klare Lösungswege freuen, nicht über die üblichen Verweise an wikipedia. Vielen Dank im voraus!

  1329. #1351 Karl-Heinz
    26. Oktober 2020

    @Kmpr_rak

    Mit Matrizen kennst dich aus?

  1330. #1352 Kmpr_rak
    26. Oktober 2020

    Na ja, eher weniger. Wäre schön, wenn ich so was hätte wie “sin x * sin y”, weil ich ja die Galaktischen Koordinaten schon habe.

  1331. #1353 Karl-Heinz
    27. Oktober 2020

    @Kmpr_rak

    Auf https://en.wikipedia.org/wiki/Supergalactic_coordinate_system habe ich gesehen, dass es dafür eine Transformationsmatrix von supergalakitsch nach galaktisch gibt. Die Transformationsmatrix von galakitsch nach supergalaktisch müsste demnach die Inverse vielleicht auch die transponierte Matrix sein.

    Intuitiv würde die Vorgehensweise für die Umrechnung von galaktisch nach supergalaktisch so aussehen.

    l,b [°] &#10139 SGL,SGB [°]

    1) galaktische Koordinaten in kartesische Koordinaten umwandeln (Einheitskugel)
    2) Transformation mit der inversen(transponierten?) Matrix durchführen.
    3) die erhaltenen kartesischen Werte in Polarkoordinaten zurückrechnen.

    Das ganze könne man im Excel sehr gut abbilden. Ich werde mich mal demnächst hinsetzen und das versuchen und wenn plausibel durch einen Link veröffentlichen.

    Für was benötigst du eigentlich Supergalaktische Koordinaten? 🙂

  1332. #1354 Kmpr_rak
    27. Oktober 2020

    Nun, ich habe in Excel mit Formeln experimentiert und bin auch noch Star Trek Fan. Da bin ich dahin gekommen! 😀

    Ja, wie man die Galaktischen Koordinaten in Kartesische, also x-y-z, umwandelt, das interessiert mich auch! Wie gesagt, so auf Basis von “sin x * cos y”, mit der Matrix stehe ich eher auf Kriegsfuß!

    Ich danke Dir auf jeden Fall für Deine Mühen!

  1333. #1355 PDP10
    27. Oktober 2020

    @Kmpr_rak:

    Kurz mal ein Einwurf:

    Du wirst nicht darum herum kommen, dich mit der zugrunde liegenden Mathematik zu beschäftigen, wenn du dich wirklich dafür interessierst.

    Glaub mir. Erst dann wirst du damit umgehen können und vor allem damit Spaß haben.

    Das dauert. Lohnt sich aber.

    Ich kann dir keine konkreten Literaturtips geben, weil ich deine Vorrausetzungen nicht kenne (und ausserdem seit Jahren komplett aus dem Thema raus bin). Aber vielleicht versuchst du es mal mit irgendeinem brauchbaren “Höhere Mathematik für Ingenieure” Buch oder so. Dafür gibts sicher auch ergänzend Bücher in der “Schaums Outline” Reihe. Die haben jede Menge Übungsaufgaben mit Lösungen und sowas darin.

    Und diese Übungsaufgaben – wenigstens ein paar davon – wirst du lösen wollen, wennn du daran Spaß haben willst.
    “Rechnen” heißt hier Gleichungen lösen, Geometrie verstehen und Koordinatentransformationen etc. Mathematik halt. Irgendwelche Rechnungen mit Excel werden dir da nichts bringen. Das ist nicht das gleiche, wie Mathematik. Der Spaß fängt erst da an, wo man die Grundlagen verstanden hat und sich den ganzen Kram mit Bleistift und Papier klar machen kann …

    (Nur meine 2 Cent.)

  1334. #1356 Lars Kohlhauer
    Greifenstein-Beilstein
    30. Oktober 2020

    Hallo.

    Ich habe eine Frage zur Expansion des Universums. Und zwar:

    Ist es möglich, dass die Expansion des Universums die 4. Dimension respektive Zeit darstellt? Diese Überlegung will mir einfach nicht aus dem Kopf. Ich interessiere mich solang ich denken kann für Astronomie und diese Frage stelle ich mir schon ein paar Jahre. Kann natürlich sein, dass es kompletter Nonsense ist, denn ich bin Laie,aber eine fundierte Antwort würde mich wirklich überaus freuen!!!! Vielen Dank im Voraus!

  1335. #1357 Kmpr_rak
    31. Oktober 2020

    @ Karl-Heinz
    Das wäre mega, danke!

    @ PDP10
    Danke, aber mir geht es rein um die Möglichkeiten in Excel.

  1336. #1358 wohe
    berlin
    7. November 2020

    @Lars Kohlhauer,
    also: Laie bin ich auch, aber das Problem beim Dimensionsbegriff ist ja nicht nur die begrenzte menschliche Vporstellungskraft (so kann ich z.B. mit den zig Dimensionen der String-Theorie und ihren Zuständen (“aufgerollt”) absolut nichts anfangen (wie gesagt: Laie halt)), bez. des Begriffs und dem Zusammenhang mit der Expansion des Universums fällt mir aber folgendes ein:
    – Dimension ist vom Wortsinn her ja eigentlich nichts anderes als eine Größe (siehe Wiki), also z.B. Temperatur oder Länge (mit den Einheiten Grad Celsius oder Meter). Insofern kann man natürlich jedem vorstellbaren “Etwas” eine Dimension zuordnen, also auch der Expansion = Ausdehnung (gemessen in Weg/Zeit = Geschwindigkeit).
    – Die Zeit wurde mir früher in der Schule tatsächlich als “4. Dimension” beschrieben und dies ist bez. der Begrifflichkeit auch zulässig (gemessen in z.B. Sekunden), aber sie als Folge der Ausdehnung zu betrachten, scheint mir nicht schlüssig, denn die Existenz von Raum und Zeit (also der Raumzeit) ist ja Voraussetzung für die Ausdehnung. Eine conditio sine qua non. Auch in einem sich nicht ausdehnenden Universum wäre sie vorhanden. Meine Antwort auf Deine Frage ist also “nein”.
    MfG Wohe

  1337. #1359 Karl-Heinz
    8. November 2020

    @Kmpr_rak

    Das Ganze natürlich ohne Gewähr auf Richtigkeit.
    Ich finde einfach keine supergalaktische Koordinaten um die Berechnung ausgiebig zu testen. 🙁

    Umrechnung zwischen galaktische und supergalaktische Koordinaten.xlsx

  1338. #1360 Kmpr_rak
    12. November 2020

    Wow, das sieht ja mega aus. Ich teste das mal durch und gebe Bescheid. Danke auf jeden Fall!!! Sogar x, y und z, ich nehme an, das sind die Kartesischen Koordinaten? Wow.

  1339. #1361 Karl-Heinz
    12. November 2020

    @Kmpr_rak

    Ja bei x,y und z handelt es sich um kartesische Koordinaten. Bin schon sehr gespannt auf dein Gutachten. 🙂

  1340. #1362 Kmpr_rak
    12. November 2020

    Soweit mega, denke ich. Aber wenn ich etwas in die Eingabefelder eingebe, dann bekomme ich in der Ausgabe die Fehlermeldung NAME?, oder hab ich einen Denkfehler?

  1341. #1363 Karl-Heinz
    12. November 2020

    @Kmpr_rak

    Ich vermute, dass du “Öffnen mit Google Tabellen” gedrückt hast. Hier kannst du leider keine Berechnungen mit Excel durchführen.

    Vorgehensweise:
    1) Auf Link klicken
    2) Rechts oben neben Druckersymbol befindet sich ein Pfeil nach unten zum Herunterladen.
    3) Auf dieses Symbol klicken
    4) Öffnen mit Excel(Standard) bzw. Datei speichern.
    4a) Wurde Öffnen mit Excel gewählt, so befindet sich das Excel in geschützter Ansicht. Den Button rechts oben im Excel “Bearbeitung aktivieren” drücken.
    4b) Alternativ kann auch “Datei speichern” ausgewählt werden. Anschließend die heruntergeladene Datei öffnen. Auch hier den Button rechts oben im Excel “Bearbeitung aktivieren” drücken.

    PS: Bitte bei nicht vertrauenswürdigen Personen auf keinen Fall “Bearbeiten aktivieren” drücken, da unter Umständen Schadsoftware durch ein Makro nachgeladen wird.

  1342. #1364 Kmpr_rak
    12. November 2020

    Das Problem besteht dennoch.

  1343. #1365 Karl-Heinz
    13. November 2020

    @Kmpr_rak

    Welche Excel bzw. Office-Version verwendest du?

  1344. #1366 Kmpr_rak
    13. November 2020

    Office 2010.

  1345. #1367 Karl-Heinz
    13. November 2020

    @Kmpr_rak

    Sollte jetzt funktionieren. Bitte diesen Link zum Herunterladen verwenden.

    Umrechnung zwischen galaktische und supergalaktische Koordinaten.xlsx

  1346. #1368 Rolf
    14. November 2020

    Hallo Florian, ich habe eine Frage zu Roten Riesen. Wenn unsere Sonne dereinst ein Roter Riese wird und sich aufbläht, so soll der Sonnenradius ja evtl bis zur Erdbahn oder sogar darüber hinaus reichen. Aber würde man in den Außenbezirken dieser aufgeblähten roten Riesensonne abgesehen vom gleißend hellen Licht – zumindest theoretisch – überhaupt mitbekommen, dass man sich innerhalb der Sonne befindet? Die Masse unseres Sternes müsste ja auf das Volumen gerechnet sehr verdünnt sein, d. h. auch die Dichte sehr gering, oder mache ich da einen Denkfehler? Vielen Dank für Deine Antwort und schöne Grüße aus Erlagen, Rolf

  1347. #1369 Kmpr_rak
    14. November 2020

    FUNKTIONIERT. Ich checke das mal mit Daten. Danke sehr!!!!

  1348. #1370 Kmpr_rak
    24. November 2020

    Karl-Heinz, es funktioniert! Wow, ich bin echt glücklich damit. Ich werde mal die Excel-Datei “sezieren”, damit ich das alles nachvollziehen kann!

  1349. #1371 Karl-Heinz
    24. November 2020

    @Kmpr_rak

    Gerne geschehen. 🙂

  1350. #1372 Mike coopet
    München
    2. Dezember 2020

    Meine Frage betrifft die Singularität.
    Wenn das Komprimierte (in) einer Singularitat als existentes Etwas gesehen werden darf, so benötigt Singularität Raum – mag dieser auch “unendlich klein” sein. Etwas braucht Raum.

    Wie erklärt sich ggf. ein solcher Raum, gewissermaßen der “Platz, der Ort der Singularität” vor ihrer Expansion, wenn erst durch eben diese Expansion Raum (und natürlich Zeit) entstehen wird?
    Oder wo ist der Fehler in meiner Betrachtung?

  1351. #1373 Kmpr_rak
    7. Dezember 2020

    Ich habe eine Frage bezüglich Koordinaten verschiedener Systeme und deren Veränderung durch unterschiedliche Standorte.

    Also: der Standpunkt sei Punkt 1, räumliche Koordinaten 0 zu 0 zu 0. An diesem Punkt 1 haben die Sterne am Himmel sowohl in Rektaszension/Deklination, in ekliptikaler Länge/Breite, in galaktischen und auch supergalaktischen Koordinaten ihre jeweiligen Werte! Danke nochmal an Karl-Heinz, dank Dir habe ich mein Excel-Wissen nochmals deutlich aufpeppen können!

    Was wäre aber, wenn ich von Punkt 1 zu Punkt 2 reise, die räumlichen Koordinaten sich also meinetwegen von 0 zu 0 zu 0 an Punkt 1 zu 23,9 zu 61,8 zu 0 an Punkt 2 ändern würden? Die Werte der räumlichen Koordinaten sind in Parsec, was aber grundsätzlich erst einmal egal ist.

    Die Werte der Koordinaten in den verschiedenen Systemen, wenn ich von Punkt 2 in den Himmel gucke, ändern sich ja nun alle. Ich vermute, ich muss die Punkte irgendwie rechnerisch verschieben!

    Und nun, tada, wie?

  1352. #1374 Kmpr_rak
    24. Dezember 2020

    Hm, okay, schade, leider keiner. Ich knabbere auch noch daran herum.

    Nun, eine weitere Frage: es gibt 88 nach der IAU offizielle Sternbilder. Habe ich Rektaszension und Deklination eines Sternes, kann ich nachsehen und den Stern einem Sternbild zuordnen.

    Nun habe ich allerdings versucht, rechnerisch via Excel Koordinaten einem Sternbild zuzuordnen. Das funktioniert ganz gut, allerdings überschneiden sich die Koordinaten der Sternbilder, so zum Beispiel bei Theta Pictoris (RA 81,1 Grad, Dekl -52,31 Grad). So läßt sich Theta Pictoris sowohl dem Pictor als auch dem Doradus zuordnen, da die Koordinaten von Theta Pictoris sowohl innerhalb der offiziellen Koordinaten von Pictor als auch von Doradus liegen.

    Puuuuhh. Viele Zahlen. Gibt es eine mögliche Lösung zur Vermeidung des Problems? Wenn ich einen Stern X habe, dessen Koordinaten ich habe, kann ich diesen ja kaum zwei Sternbildern zuordnen.

  1353. #1375 hans
    nürnberg
    26. Dezember 2020

    hallo mich quält schon seid langen die frage wie z.b. bei bildern des hubble teleskopes das unterschiedliche alter der sterne und dadurch ja auch überhaupt deren existenz zur jetztzeit bewertet wird, was ja möglicherweise auch heissen müßte das sich durch die unvorstellbaren entfernungen bilder zeigen die , so wie wir sie jetzt sehen gar nicht mehr existent sind und erscheinungen (himmelskörper,nebel usw.) nicht mehr in der situation gesehen werden in der sie sich momentan befinden,möglicherweise sehen wir ja auch dinge,in unterschiedlichen entwicklungsstufen oder sogar mehrfach.
    vielleicht kann mich jemand aufklären.
    gruß hans

  1354. #1376 Florian Freistetter
    26. Dezember 2020

    @Hans: Diese Frage habe ich schon in einem eigenen Artikel beantwortet (siehe die Liste oben): https://scienceblogs.de/astrodicticum-simplex/2016/01/18/sind-die-sterne-die-wir-sehen-alle-schon-laengst-wieder-verschwunden/

  1355. #1377 hans
    nürnberg
    26. Dezember 2020

    Danke dir Florian
    werde mich gleich mal darüber machen

  1356. #1378 Karl-Heinz
    Graz
    27. Dezember 2020

    @Kmpr_rak zur Frage #1374

    Nun, eine weitere Frage: es gibt 88 nach der IAU offizielle Sternbilder. Habe ich Rektaszension und Deklination eines Sternes, kann ich nachsehen und den Stern einem Sternbild zuordnen.

    Nun habe ich allerdings versucht, rechnerisch via Excel Koordinaten einem Sternbild zuzuordnen. Das funktioniert ganz gut, allerdings überschneiden sich die Koordinaten der Sternbilder, so zum Beispiel bei Theta Pictoris (RA 81,1 Grad, Dekl -52,31 Grad). So läßt sich Theta Pictoris sowohl dem Pictor als auch dem Doradus zuordnen, da die Koordinaten von Theta Pictoris sowohl innerhalb der offiziellen Koordinaten von Pictor als auch von Doradus liegen.

    Puuuuhh. Viele Zahlen. Gibt es eine mögliche Lösung zur Vermeidung des Problems? Wenn ich einen Stern X habe, dessen Koordinaten ich habe, kann ich diesen ja kaum zwei Sternbildern zuordnen.

    Dass sich die Koordinaten der Sternbilder überschneiden ist so nicht richtig. Die Werte, die du bei Wikipedia findest geben halt nur den Bereich (von-bis) von Rektaszension und Deklination an.

    Eine einfache geometrische Figur der Sternbildgrenzen wie ein (sphärisches) Viereck kommt fast nur im Süden vor, und hier neunmal, während ein zehntes auf dem Äquator liegt (Sternbild Sextans). Die meisten Sternbilder werden durch Polygonzüge abgegrenzt. Benachbarte Punkte der dabei entstehenden Linienzüge haben also entweder gleiche Deklination oder gleiche Rektaszension. Siehe Beispiel ganz unten. Damit liegt Theta Pictoris eindeutig im Sternbild Pictor.

    Sternbild Maler (Pictor)
    Kürzel: Pic

    Rektaszension: 04h 32m 52s bis 06h 52m 03s
    Deklination: −64° 09′ 07″ bis −42° 47′ 47″

    Sternbild Schwertfisch (Sternbild)
    Kürzel: Dor

    Rektaszension: 03h 53m 17s bis 06h 35m 45s
    Deklination: −70° 06′ 15″ bis −48° 40′ 12″

    Theta Pictoris ist ein Mehrsternsystem
    θ Pic AB

    Rektaszension: 05h 24m 46.28819s
    Deklination: −52° 18′ 58.4836″

    ———————————————-
    1 Spalte: Rektaszension ( Äquinoktium 1875.0)
    2 Spalte: Deklination ( Äquinoktium 1875.0)
    3 Spalte: Sternbild
    PIC
    6.00000 -43.00000 PIC
    5.00000 -43.00000 PIC
    4.83333 -43.00000 PIC
    4.83333 -46.50000 PIC
    4.50000 -46.50000 PIC
    4.50000 -49.00000 PIC
    4.50000 -54.00000 PIC
    5.00000 -54.00000 PIC
    5.00000 -57.50000 PIC
    5.50000 -57.50000 PIC
    5.50000 -61.00000 PIC
    6.00000 -61.00000 PIC
    6.00000 -64.00000 PIC
    6.58333 -64.00000 PIC
    6.83333 -64.00000 PIC
    6.83333 -58.00000 PIC
    6.50000 -58.00000 PIC
    6.50000 -55.00000 PIC
    6.16667 -55.00000 PIC
    6.16667 -52.50000 PIC
    6.00000 -52.50000 PIC
    6.00000 -50.75000 PIC

    3.83333 -53.16667 DOR
    4.00000 -53.16667 DOR
    4.00000 -56.50000 DOR
    4.33333 -56.50000 DOR
    4.33333 -59.00000 DOR
    4.58333 -59.00000 DOR
    4.58333 -67.50000 DOR
    4.58333 -70.00000 DOR
    6.58333 -70.00000 DOR
    6.58333 -64.00000 DOR
    6.00000 -64.00000 DOR
    6.00000 -61.00000 DOR
    5.50000 -61.00000 DOR
    5.50000 -57.50000 DOR
    5.00000 -57.50000 DOR
    5.00000 -54.00000 DOR
    4.50000 -54.00000 DOR
    4.50000 -49.00000 DOR
    4.26667 -49.00000 DOR
    4.08333 -49.00000 DOR
    4.08333 -51.00000 DOR
    3.83333 -51.00000 DOR

  1357. #1379 Kyllyeti
    27. Dezember 2020

    “Sed imago plus dicit quam mille numeri” sagte der Maler (Pictor) indigniert und verreiste grummelnd mit dem Polygonzug in den Süden.

  1358. #1380 Karl-Heinz
    Graz
    27. Dezember 2020

    @Kmpr_rak

    Konstellationsgrenzen

    Sternbild auswählen,
    dann “Data as a Table” rechts unten, für die Zahlenwerte als Grenzen.

  1359. #1381 Kmpr_rak
    27. Dezember 2020

    @ Karl-Heinz:

    Vielen Dank für Deine Infos. Soweit alles klar, und wenn ich auf einem Bild nachsehe, kann ich das alles verstehen.

    Wenn ich allerdings nur nach den Daten gehe, dann liegen Rektaszension und Deklination von Theta Pictoris sowohl zwischen der von Pictor als auch von Doradus. Meine Formel spielt darauf ab, wenn Rektaszension und Deklination eines Sterns zwischen Rektaszension MIN und Rektaszension MAX bzw. Deklination MIN und Deklination MAX sind, welches Sternbild ist es dann? Die Tabelle zeigt eben auch die MIN und MAX Ausdehnungen der 88 Konstellationen.

    Und ich denke, da liegt der Hund begraben. Ich müsste Excel darauf trimmen, nach welcher Polygonlinie es sich richten soll (obwohl ich das so richtig auch nicht nachvollziehen kann).

  1360. #1382 Kmpr_rak
    27. Dezember 2020

    Ah ja, jetzt habe ich das begriffen mit den verschiedenen “Grenzpunkten”. Oh je, das alles für 88 Konstellationen in Excel einpflegen und eine Formel basteln …..

  1361. #1383 Karl-Heinz
    Graz
    27. Dezember 2020

    @Kmpr_rak

    Oder man macht das Online.

    http://goblack.de/php/sternbild/sternbild.php
    Eingabe siehe Hilfe

    Beispieleingabe: Theta Pictoris (θ Pic AB)

    (05 24 46.28819) (-52 18 58.4836)

  1362. #1384 Kmpr_rak
    28. Dezember 2020

    Ja, nur das sollte nicht Sinn der Sache sein in einer Excel-Tabelle. Aber danke trotzdem für die Erklärung!!!

  1363. #1385 Karl-Heinz
    Graz
    28. Dezember 2020

    @Kmpr_rak
    Na dann halt ausprogrammieren.

    Eine der vielen Möglichkeiten.
    http://www.querbeet-astronomie.de/artikel/SternbildZuKoordinate.shtml

    @FF
    Sorry, ich weiß. Hier ist nicht die Plattform für Diskussionen.

  1364. #1386 Moritz Lerchbaumer
    Wien
    4. Januar 2021

    Hallo ich schreibe gerade an einem kleinen Text über die Sonne und da ist mir diese Frage in den Weg gestolpert. Ich hoffe sie ist verständlich genug formuliert und ich hoffe ich nerve mit ihr nicht.
    Mfg, Moritz Lerchbaumer

    Gibt es Leben ohne Sonnen. Oder kennt jedes Lebewesen dass auf einem Planeten lebt eine Sonne? Gibt es Planeten im Universum die in völliger Dunkelheit schweben?

  1365. #1387 Florian Freistetter
    4. Januar 2021

    @Moritz: Es gibt tatsächlich jede Menge Planeten ohne Stern: https://scienceblogs.de/astrodicticum-simplex/2020/08/31/vagabundierende-planeten-im-interstellaren-raum-und-das-roman-space-telescope/ Ob es da auch Leben geben kann ist umstritten. Wenn, dann eher kein “höheres Leben”; sondern vielleicht irgendwelche unterirdischen Mikroben oder Zeug das in irgenwelchen dunklen Ozeanen unter ner kilometerdicken Eisdecke rumschwimmt.

  1366. #1388 Kmpr_rak
    8. Januar 2021

    Ich habe eine Frage bezüglich Koordinaten verschiedener Systeme und deren Veränderung durch unterschiedliche Standorte.

    Also: der Standpunkt sei Punkt 1, räumliche Koordinaten 0 zu 0 zu 0. An diesem Punkt 1 haben die Sterne am Himmel sowohl in Rektaszension/Deklination, in ekliptikaler Länge/Breite, in galaktischen und auch supergalaktischen Koordinaten ihre jeweiligen Werte!

    Was wäre aber, wenn ich von Punkt 1 zu Punkt 2 reise, die räumlichen Koordinaten sich also meinetwegen von 0 zu 0 zu 0 an Punkt 1 (sagen wir ERDE) zu 23,9 zu 61,8 zu 0 an Punkt 2 (sagen wir ERDE 2) ändern würden? Die Werte der räumlichen Koordinaten sind in Parsec, was aber grundsätzlich erst einmal egal ist.

    Die Werte der Koordinaten in den verschiedenen Systemen, wenn ich von Punkt 2 in den Himmel gucke, ändern sich ja nun alle. Ich vermute, ich muss die Punkte irgendwie rechnerisch verschieben!

    Tja, aber ich weiß immer noch nicht, wie …

  1367. #1389 Karl-Heinz
    8. Januar 2021

    @Kmpr_rak

    Ich denke, dass bei deiner Überlegung was fehlt. Man kann ja ein Koordinatensystem von einer Stelle zu einer anderen Stelle verschieben und anschließend x,y und z im Raum neu ausrichten.
    Zum Beispiel von galaktisch auf supergalaktisch. Da wird der Ort nicht gewechselt sondern die Ebene (x,y) auf die
    supergalaktischen Ebene ausgerichtet. Der Nullpunkt der supergalaktischen Länge wird definiert als die Schnittlinie zwischen dieser Ebene und der galaktischen Ebene.

    Du könntest doch mal posten, wie jetzt das galaktischen Koordinatensystems absolut im Raum liegt. 🙂

  1368. #1390 Kmpr_rak
    8. Januar 2021

    Danke Karl-Heinz. Na ja, das bleibt alles so wie es ist, sagen wir mal, ich bin ein Erdling und habe alles so, wie festgelegt. Jetzt steige ich aber in ein Raumschiff und fliege zur ERDE 2. Wie verschieben sich dann die Koordinaten?

    Wie gesagt, das System bleibt gleich, nur ich verschiebe meinen Standort.

  1369. #1391 Kmpr_rak
    9. Januar 2021

    Eine weitere Frage “plagt” mich. Wie bestimme ich bei einem Stern die habitable Zone? Soweit ich weiß, funktioniert das über die bolometrische Helligkeit.

    Diese berechnet sich aus scheinbarer Helligkeit MINUS bolometrische Korrektur. Komischerweise funktioniert das auch für die absolute Helligkeit. Die Korrektur hat auch so eine tolle Formel, man muss da die Temperatur eines Sterns eingeben. Ich habe da sogar ein tolles EXCEL-Beispiel gefunden, für einen Stern von 3.500 K. Sobald ich allerdings mit der Formel und der Temperatur der Sonne die Korrektur berechne, stimmt da was nicht.

    Laut Quellen beträgt die Korrektur der Sonne etwa 0,09, bei absoluter Helligkeit 4,83 wäre die bolometrische Helligkeit nach Abzug also 4,74. Was aber mit meinem berechneten Wert (siehe oben) nicht passt.

    Nun, das ist Teilproblem 1. Wenn ich dieses aber lösen kann, wie gelange ich dann mit der bolometrischen Helligkeit zur Berechnung der habitablen Zone?

    Wie gesagt, ich bin kein Profiastronom. Aber eben sehr interessiert.

  1370. #1392 Jakob Wassili Siepmann
    Berlin
    27. Januar 2021

    Wahnsinn, genau so ein Forum habe ich gesucht!
    Seit dem ich populärwissenschaftliche Bucher lese plagt mich die Frage woher Stephen Hawking und Konsorten wissen, wie sich der Urknall zugetragen hat. Immer wieder wird davon geschrieben, was in den ersten Sekundenbruchteilen des Urknalls entstanden ist, wie groß das Universum zu diesem Zeitpunkt war und welche Temperatur es z.B. hatte.
    Woher kommt die Gewissheit über den Ablauf? Anhand welcher Daten wird das rekonstruiert?

    Coronafreundliche Onlineküsse an die Person, die mir das beantworten kann!

  1371. #1393 H
    27. Januar 2021
  1372. #1394 Captain E.
    12. Februar 2021

    In den Nachrichten wurde vorhin gemeldet, dass in der Grenzregion Deutschland, Niederlande und Belgien ein neues Teleskop gebaut werden soll: Das Einstein-Teleskop. Worum handelt es sich dabei?

  1373. #1395 Peter L.
    12. Februar 2021

    Zitat: “Das Einstein-Teleskop. Worum handelt es sich dabei?”

    Um einen Gravitationswellen-Detektor, siehe auch:
    https://de.wikipedia.org/wiki/Einstein-Teleskop

  1374. #1396 Dr. Horst Martin Stubenrauch
    52249 Eschweiler
    13. Februar 2021

    Hab 2 Fragen zu Sternendichte und Sternenlicht:

    Wenn man den nächtlichen Sternenhimmel in dunkler Umgebung beobachtet und den Raum zwischen zwei leuchtenden Sternen betrachtet, so erscheinen in dem dunklen Feld bald weitere Sterne. Und wenn man sich dann auf die kleineren, dunklen Felder konzentriert, so passiert dasselbe erneut. – Ich weiß nicht, ob man diesen Effekt mit Teleskopen erweitern kann, mich aber inspirierte diese Beobachtung zu 2 Fragen:

    1) Ist die Sternendichte bis zum Rand des Universums so groß, dass wir (in jede Richtung in die man blickt) quasi komplett von Masse umhüllt sind oder könnte man theoretisch eine Gerade (zu irgendeinem Zeitpunkt) bis zum Rand des Universums ziehen, ohne dabei auch nur einen einzigen Himmelskörper zu berühren?

    2) Falls sich letztendlich in jedem “dunklen Fleck” zwischen sichtbaren Sternen ebenfalls leuchtende Sterne befinden, warum ist dann der Nachthimmel nicht komplett hell?

    Wenn jemand dazu eine Antwort geben, würde es mich freuen, sie zu lesen.

  1375. #1397 Florian Freistetter
    13. Februar 2021

    Diese Frage ist ein altes (scheinbares Paradoxon): Heißt “Olbers Paradox” – hier gibt es mehr dazu: https://scienceblogs.de/astrodicticum-simplex/2017/11/03/sternengeschichten-folge-258-warum-ist-es-nachts-dunkel-das-olberssche-paradoxon/

  1376. #1398 Dieter Schläger
    Deisenhausen
    13. Februar 2021

    @ Herr Stubenrauch
    Dazu ist in der Wikipedia ein Artikel zu finden über das Olbersche Paradoxon.
    https://de.wikipedia.org/wiki/Olberssches_Paradoxon

  1377. #1399 Dieter Schläger
    Deisenhausen
    13. Februar 2021

    Erschwerend kommt ja noch hinzu, dass das Universum sich ausdehnt und das Licht von weit entfernten Sternen nicht nur schwächer, sondern auch kühler bei uns ankommt. Die ca. 3000 Kelvin der Hintergrundstrahlung haben sich in den 13,4 Mrd. Jahren auf ca. 2,7 Kelvin reduziert. Das entspricht einer Frequenz die nicht mehr im für uns sichtbaren Licht liegt.

  1378. #1400 Kmpr_rak
    22. Februar 2021

    Wie kann ich für einen Stern die Habitable Zone ermitteln? Angeblich funktioniert das über die Bolometrische Hellligkeit. Wie kann ich denn diese wiederum ermitteln?

  1379. #1401 Volkmar
    Wedemark
    26. Februar 2021

    Dunkle Materie, schwarze Löcher und Galaxien

    Hallo Florian,

    wir wissen noch sehr wenig über die dunkle Materie … aber wären die folgenden Hypothesen, die ich als Fragen formuliert habe, verträglich mit dem aktuellen Stand des Wissens?

    Wird die dunkle Materie, die selbst eine Gravitation bewirkt (oder den Raum krümmt), auch selbst durch Gravitation (oder die Raumkrümmung) beeinflusst?

    Gehen wir für die weiteren Überlegungen davon aus, dass es so ist (wenn dem nicht so ist, dann erübrigen sich die Fragen leider).
    Falls es so ist dann würde sich die dunkle Materie in einer Galaxis mit der baryonischen Materie gemeinsam um das Zentrum bewegen.

    Was würde nun passieren, wenn sich dunkle Materie aufgrund von zufällig vorhandenen Fluktuationen lokal verdichtet?
    Wären Prozesse denkbar, die eine weitere Kontraktion dunkler Materie begünstigen?
    (z.B. analog der inneren Reibung einer Gaswolke)

    Könnte sich so ein schwarzes Loch aus dunkler Materie bilden?
    Im Unterschied zu baryonischer Materie gibt es ja keinen „Widerstand“ durch eine anlaufende Kernfusion die einer weiteren Kontraktion entgegenwirkt.

    Könnte dies eventuell sogar eine Erklärung für die supermassiven schwarzen Löcher im Zentrum von Galaxien sein?
    D.h. dass die supermassiven Löcher in den Zentren der Galaxien im wesentlichen aus dunkler Materie bestehen?

    Ein Indiz hierfür ist für mich, dass die Diagramme Rotationsgeschwindigkeit über Abstand vom Zentrum v.a. im äußeren Bereich eine Diskrepanz aufweisen zwischen beobachteter baryonischer Materie und gesamt vorhandener Materie, siehe z.B. https://de.wikipedia.org/wiki/Dunkle_Materie
    Im Zentrum gibt es also keine dunkle Materie – aber warum sollte ausgerechnet da die dunkle Materie fehlen?
    Für mich wäre eine plausible Erklärung, dass dort anfangs dunkle Materie war, die dann jedoch das supermassive schwarze Loch im Zentrum gebildet hat.
    Interessant wäre daher auch ein Vergleich zwischen den Massen der schwarzen Löcher in den Galaxienzentren und der Verteilung der dunklen Materie – vielleicht besteht hier ein Zusammenhang?

    Die Gedanken lassen sich natürlich noch weiter spinnen, damit möchte ich es aber für’s erste bewenden lassen.
    Über eine Beantwortung meiner Fragen würde ich mich sehr freuen!

    VG, Volkmar

    PS Vielen Dank für diesen interessanten Blog, bei dem man auch als interessierter Laie noch einiges lernen kann. 🙂

  1380. #1402 alex
    27. Februar 2021

    @Volkmar:
    Ich bin zwar nicht Florian, aber ich versuche mich trotzdem mal an ein paar Antworten:

    Wird die dunkle Materie, die selbst eine Gravitation bewirkt (oder den Raum krümmt), auch selbst durch Gravitation (oder die Raumkrümmung) beeinflusst?

    Ja.

    Wären Prozesse denkbar, die eine weitere Kontraktion dunkler Materie begünstigen?
    (z.B. analog der inneren Reibung einer Gaswolke)

    Denkbar ist vieles. Momentan kennen wir keinen Mechanismus der bei dunkler Materie zu Reibung führen würde. Bei baryonischer Materie geht das über den Elektromagnetismus, und damit wechselwirkt dunkle Materie ja nicht. Die anderen bekannten Grundkräfte sind dazu auch nicht geeignet, also bräuchte es eine fünfte Kraft.

    Könnte sich so ein schwarzes Loch aus dunkler Materie bilden?

    Wenn man eine ausreichend große Menge dunkler Materie auf einen ausreichen kleinem Raum komprimiert, dann sollte das auch ein schwarzes Loch bilden. Die Art der Materie ist der ART da egal. Aber ich sehe nicht wie das in der Realität ohne Reibung passieren sollte.

    Im Zentrum gibt es also keine dunkle Materie – aber warum sollte ausgerechnet da die dunkle Materie fehlen?

    Naja, im Inneren gibt es auch weniger Raum (genauer: Kugelschalen fester Dicke haben für kleine Radien kleinere Volumina). Wenn man das durchrechnet mit der Annahme, dass die dunkle Materie eine konstante Dichte hat, kommt heraus dass ihr Einfluss auf die Rotationsgeschwindigkeit im Inneren am kleinsten ist.

  1381. #1403 Volkmar Fries
    Wedemark
    28. Februar 2021

    Hallo Alex,

    danke für deine Kommentare.
    Zum Punkt „warum finden wir keine dunkle Materie in den zentralen Bereichen der Galaxien“ noch folgende Anmerkungen, wiederum als Fragen formuliert mit zusätzlichen Anmerkungen zur Erläuterung.

    Gibt es eine plausible Hypothese warum in den zentralen Bereichen der Galaxien weniger dunkle Materie vorhanden sein sollte als in den Außenbereichen – also umgekehrt wie es sich für die baryonische Materie (gemeint ist jeweils die Masse / Volumen = Dichte)?
    (Die Annahme, dass die dunkle Materie eine konstante Dichte haben soll finde ich nicht einleuchtend. Zum einen ist die dunkle Materie in den Bereichen der Galaxien konzentriert und verteilt sich nicht gleichmäßig über das Universum.)
    Wäre es nicht sogar naheliegend, dass die dunkle Materie ähnlich wie die baryonische Materie stärker in den zentralen Bereichen konzentriert sein sollte?
    (Das ist wie gesagt jedoch nicht der Fall, daher sollte es dafür einen Grund geben.)

    Gibt es Modelle, die sich mit dem Zusammenhang zwischen dunkler Materie und der Bildung der Galaxien und deren zentralen schwarzen Löchern beschäftigen?
    (Irgendwie legen doch schon die Bezeichnungen dunkle Materie und schwarzes Loch einen Zusammenhang nahe!) 😉

    Volkmar

  1382. #1404 Volkmar
    Wedemark
    28. Februar 2021

    PS
    … und noch eine Spekulation: Primordiale schwarze Löcher aus dunkler Materie als Galaxienkerne – wäre so etwas denkbar?

  1383. #1405 Florian Freistetter
    28. Februar 2021

    @Volkmar: Wo wäre da der Unterschied zur aktuellen Situation? Es SIND ja schwarze Löcher in den Zentren der Galaxien.

  1384. #1407 Volkmar
    Wedemark
    1. März 2021

    @ Karl-Heinz
    Vielen Dank für den Link!
    Sehr schön, genau das meinte ich.
    Damit gibt es also nun auch ein physikalisches Modell zur Hypothese.
    🙂

  1385. #1408 alex
    2. März 2021

    @Volkmar Fries:
    Da scheint es ein Missverständnis gegeben zu haben. Ich behaupte nicht, dass die dunkle Materie gleichverteilt ist. Das ist sie nicht (aber sie ist gleicher verteilt als die sichtbare Materie). Aber du kannst aus dem Diagramm das du verlinkt hast nicht schließen, dass es in den Zentren von Galaxien keine dunkle Materie gibt (oder auch nur dass ihre Dichte dort geringer ist als in den Außenbereichen von Galaxien).

  1386. #1409 Peter
    1. April 2021

    Ich habe folgende Frage zum interstellaren Kometen 2I/Borisov: Gibt es eine Möglichkeit, den ungefähren Ort zu bestimmen, an dem dieser Komet entstanden ist? Möglicherweise sogar den Stern/das Sternensystem, aus dem er kam? Oder zumindest grob, aus welchem Bereich der Milchstraße er kam?

    Die SZ schrieb, der Komet sei ein Gast „Gast aus einer fremden Galaxie“. Ich habe aber nichts im Internet dazu gefunden, dass die Wissenschaft irgendwelche Beweise dafür hätte, dass 2I/Borisov von außerhalb der Milchstraße stammt.
    Könnte ein Eiskörper von der Größe eines Kometen*, der noch nie nahe an einem Stern war, die Passage zischen 2 Galaxien überleben? Z. B. die rund 163000 Lichtjahre Entfernung von der Großen Magellanschen Wolke oder die rund 2,5 Millionen Lichtjahre von der Andromedagalaxie bis zu uns? So ein Eiskörper wird ja auch durch die kosmische Strahlung erodiert.
    * Laut https://de.qaz.wiki/wiki/Comet_nucleus sind die meisten Kometenkerne nicht mehr als 16 km groß, die größten wären rund 100 km groß.

  1387. #1410 Florian Freistetter
    1. April 2021

    @peter: Schau 8ich mir an. Spontan würd ich sagen: Das mit der Galaxie ist Quatsch

  1388. #1412 Peter
    2. April 2021

    Nicht in der Süddeutschen, sondern in der Sächsischen Zeitung (auch mit SZ abgekürzt), vom 1. 4. 2021, Seite 24, Artikel „Gast aus einer fremden Galaxie“.
    War aber kein Aprilscherz, sondern fast alle Fakten im Artikel sind wissenschaftlich korrekt. Nur in der Überschrift steht eben, dass der Komet aus einer fremden Galaxie käme, aber darauf wird im Artikeltext selbst gar nicht eingegangen.

    Im Artikeltext steht, dass Komet Borisov aus dem interstellaren Raum stammt und vor Millionen Jahren aus seinem eigenen Sonnensystem ausgestoßen worden war.
    Auch steht dort, dass sich Komet Borisov in einem wesentlich ursprünglichen Zustand bestfindet als alle bekannten Kometen unseres Sonnensystems: Die Polarisation des Lichts, das von der Koma von Borisov reflektiert wird, ist stärker als für Kometen unseres Sonnensystems üblich ist. Das würde zeigen, dass er in seinem Ursprungssystem vermutlich nie in die Nähe des Zentralsterns und dessen Strahlung gelangt sei.
    Außerdem hat man viele Gesteinsteilchen mit Größen über 1 mm in der Koma gemessen. Daraus schlussfolgern Wissenschaftler, dass der Komet in einer Region entstanden sein muss, in der es schon viele größere Gesteinskörper gab, so dass es viele Kollisionen zwischen diesen großen Gesteinskörpern gab. Denn nur durch viele Kollisionen zwischen großen Gesteinskörpern könnten die kleinen Gesteinspartikel entstanden sein, die sich in der Koma von Borisov befinden.

  1389. #1413 mysterion10
    Salzburg
    4. April 2021

    Ich werde mit allerlei physikalischen fragen bombardiert, von meinem Neffen. Eines davon übersteigt einfach mein wissen, insofern würde ich gerne wissen, ob in einem Universum was aus Antimaterie bestünden würde, die Kausalität genauso gegeben wäre, wie in unseren Universum. Bei Wiki unter CPT-Theorem und der CP-Verletzung, habe ich keine befriedigende Antwort bekommen. mfg

  1390. #1414 Peter
    Dresden
    7. April 2021

    Ich habe bei der Sächsischen Zeitung zum Artikel über Komet Borisov nachgefragt und sie haben mir geantwortet: Beim Verfassen der Überschrift („Gast aus einer fremden Galaxie“) ist ein Fehler passiert. Komet Borisov kommt aus einem fremden Sonnensystem, aber aus unserer Galaxie.

  1391. #1415 Karl-Heinz
    Graz
    8. April 2021

    @Peter
    Danke für die Info.
    Wäre der Komet 2I/Borisov wirklich von außerhalb in unsere Milchstraße reingefallen, dann hätte er bei uns eine beachtliche Geschwindigkeit gehabt. Wahrscheinlich so um die 500 km/s.

  1392. #1416 Peter
    Dresden
    8. April 2021

    Zitat Karl-Heinz „Wäre der Komet 2I/Borisov wirklich von außerhalb in unsere Milchstraße reingefallen, dann hätte er bei uns eine beachtliche Geschwindigkeit gehabt. Wahrscheinlich so um die 500 km/s.“

    Wurden denn überhaupt schon einmal in unserer Milchstraße so schnelle Himmelskörper beobachtet, die keine Sterne, Sternreste oder Braune Zwerge waren (also z. B. Planeten ohne Mutterstern, Planetoiden, Asteroiden, Kometen) und die aus einer Richtung kamen, die auf einen Ursprung außerhalb unserer Milchstraße schließen lässt?

  1393. #1417 Peter
    7. Juni 2021

    Warum gibt es Mondkarten, auf denen der Südpol des Mondes oben ist und der Nordpol des Mondes unten ist?
    Hat das rein historische Gründe, weil Mondkarten früher nur mit erdgebundenen Teleskopen erstellt wurden? Das Bild, das man durch ein Teleskop sieht, steht ja auf dem Kopf.
    Nur wenn man ins Teleskop eine Umkehrlinse (zusätzliche Linse im Strahlengang vorm Okular) einbaut, steht das Bild wieder so herum, wie man es auch mit bloßem Auge sieht. Wurden früher keine Umkehrlinsen benutzt?

  1394. #1418 Peter
    8. Juni 2021

    https://www.t-online.de/leben/familie/id_90075922/ringfoermige-sonnenfinsternis-10-juni-2021-das-ist-zu-sehen.html
    Zitat: “Am 21. Juni erreicht die Sonne um 5.32 Uhr im Sternbild Stier den Gipfel ihrer Jahresbahn, der astronomische Sommer beginnt. Am Nachmittag tritt sie in das Sternbild Zwillinge. Der Sommerpunkt markiert den Beginn des Tierkreiszeichens Krebs, weshalb man auch vom Wendekreis des Krebses spricht.”

    Wenn der Punkt der Sommersonnenwende schon im Stier liegt, weichen dann die astrologischen Tierkreiszeichen schon um mehr als ein echtes (astronomisches) Sternbild vom echten Stand der Sonne ab? Also liegt z. B. der Frühlingpunkt nicht mehr am Anfang des Sternbilds Widder, sondern am Ende der Fische (dieses Sternbild ist ja ziemlich groß) oder schon knapp im Wassermann?
    Kann man irgendwo nachschauen, in welchem astronomischen Sternbild (nicht astrologischen Tierkreiszeichen) die Sonne zu einem bestimmten Datum stand, z. B. als man geboren wurde?

  1395. #1419 Florian Freistetter
    8. Juni 2021

    @Peter: Das geht auf jeden Fall mit der freien Software Stellarium: https://stellarium.org/de/

  1396. #1420 Kmpr'rak
    23. Juni 2021

    wie kann ich bei einem Stern die bolometrische Helligkeit ermitteln?

    Und wie finde ich die “Habitable Zone”?

  1397. #1421 Karl-Heinz
    Graz
    23. Juni 2021

    @Kmpr’rak

    Bei normalen Sternen (Hauptreihensternen) hängen Ausmaß und Ausdehnung der habitablen Zone nur von der Masse des Sterns ab, denn die Masse bestimmt die Oberflächentemperatur der Sterne sowie den gesamten Energieausstoß (Leuchtkraft). Weil massearme Sterne kühler und weniger leuchtkräftig sind, sind ihre habitablen Zonen weiter innen und weniger ausgedehnt.
    Das Leben, wie wir es kennen, benötigt flüssiges Wasser. Daher definieren wir die habitable Zone um einen Stern als die Zone, in der ein hinreichend großer Planet auf seiner Oberfläche flüssiges Wasser aufweisen kann.

  1398. #1422 Karl-Heinz
    Graz
    23. Juni 2021

    @Kmpr’rak

    Berechnung der habitablen Zone eines Hauptreihenstern

    http://www.sternwarte-eberfing.de/Aktuell/2017/Juli/EingabeZentralst.php

  1399. #1423 Florian Freistetter
    24. Juni 2021

    ich hab hier ein bisschen was zum Konzept der habitablen Zone geschrieben: https://scienceblogs.de/astrodicticum-simplex/2020/04/06/ausserirdisches-gin-tonic-die-wirklich-habitable-zone/

  1400. #1424 Carlota
    15. Dezember 2021

    Hallo, ich hätte eine Frage bezüglich schwarzer Löcher und Gravitationswellen: Wie kann man denn mit Hilfe von Gravitationswellen schwarze Löcher entdecken?
    Danke!

  1401. #1425 Captain E.
    15. Dezember 2021

    @Carlota:

    Hallo, ich hätte eine Frage bezüglich schwarzer Löcher und Gravitationswellen: Wie kann man denn mit Hilfe von Gravitationswellen schwarze Löcher entdecken?
    Danke!

    Vorbehaltlich einer fachlich fundierteren Antwort formuliere ich das mal so: Gravitationswellen werden erzeugt, wenn sich Massen anders anordnen. Allerdings sind diese Gravitationswellen dermaßen schwach, dass es schon gewaltige Massen braucht, um überhaupt eine Chance zu haben, diese messen zu können. Wenn die Gravitationswellendetektoren also etwas auffangen, dann muss etwas wirklich großes dahinter stecken, und das sind im Universum Neutronensterne und Schwarze Löcher. Nun hat die Astronomie eine Vorstellung davon, wie schwer ein Neutronenstern mindestens sein muss und wie schwer er maximal sein darf. Wenn da also etwas Gravitationswellen verursacht hat, das größer als Neutronenstern gewesen ist, dürfte es sich um ein Schwarzes Loch handeln.

  1402. #1426 Bernd W.
    Essen
    15. Dezember 2021

    Die Allgemeine Relativitätstheorie macht (mathematische) Vorhersage über die Existenz von Gravitationswellen und ermöglicht es auch, Amplitude, Frequenzen und Verlauf dieser Wellen für verschiedene Objekte vorherzusagen. Jedenfalls können die Theoretiker:innen das 🙂 Und es gibt unterschiedliche “Formen” für die Gravitationswellen unterschiedlicher astronomischer Objekte, die Masse der Objekte etc. So kann man das “Bild” der aufgefangenen Welle gegen diese “Vorhersagen” Mappen und erkennt dann welches Objekte beteiligt sind, wie schwer sie sind (zumindest in gewissem Rahmen) und wahrscheinlich auch noch andere Dinge. So habe ich es jedenfalls verstanden.

  1403. #1427 Christopher
    27. Dezember 2021

    Könnte es theoretisch Planeten geben, die nur oder größtenteils flüssig sind und keinen festen Kern haben? Und könnte es in einem dieser Planeten Leben geben?

  1404. #1428 Florian Freistetter
    28. Dezember 2021

    @Christopher: Komplett flüssig geht nicht; irgendwann sorgt der Druck dafür, dass Flüssigkeiten fest werden. Aber es gibt mit Sicherheit Planeten, die zum Beispiel SEHR viel mehr Wasser enthalten; mit Ozeanen die hunderte Kilometer tief sind.

  1405. #1429 Dieter Schläger
    Deisenhausen
    29. Dezember 2021

    Gibt es eigentlich so etwas wie eine Größenverteilung (Massenverteilung) von Sternen z. B. als Diagramm. Es ist ja bekannt, das die Anzahl der Zwergsterne wesentlich größer ist als die der größeren Sterne. Es täte mich auch interessieren ob es viele “Sterne” gibt, die es noch nicht einmal bis zum braunen Zwerg geschafft haben, praktisch so etwas wie das Jupiter- oder Saturn-System, nur halt als Sonnensystem.

  1406. #1430 Peter
    31. Dezember 2021

    Zitat #1427: „Könnte es theoretisch Planeten geben, die nur … flüssig sind und keinen festen Kern haben?“
    Einen Planeten, der nur flüssig ist, stelle ich mir schwierig vor:
    1.) Ohne ein festes Planeten-Inneres wäre die Anziehungskraft auf die Flüssigkeit in der äußersten Schicht deutlich geringer. Würde sich eine Kugel oder ein Rotationsellipsoid aus einer Flüssigkeit im Orbit um einen Stern befinden, so könnten die Gezeitenkräfte des Sterns (und eventuell vorhandener Monde, anderer Planeten bzw. Sterne des Systems) mit der Zeit den Flüssigkeitskörper zerreißen und die Flüssigkeit entlang des Orbits verteilen.
    2.) a) Ein Planet, der nur aus flüssigem Wasser, Methan oder einer anderen nicht-metallischen Flüssigkeit bestünde, hätte kein Magnetfeld. Der Sternenwind würde die Atmosphäre des Planeten mit der Zeit davontragen. Die Flüssigkeit des Planeten würde an der Oberfläche verdunsten, die Gasteilchen würden vom Sternenwind ionisiert und weggeweht. Der Planet würde also mit der Zeit immer kleiner werden oder sogar ganz verschwinden.
    2.) b) Ein Planet, der nur aus flüssigem Metall (z. B. Eisen, Nickel) bestünde, müsste sehr eng um seinen Stern kreisen. Die Temperaturen lägen bei 1000 °C und mehr, Leben wäre nicht möglich.

  1407. #1431 Peter
    31. Dezember 2021

    Zitat #1428 Florian Freistetter: „Komplett flüssig geht nicht; irgendwann sorgt der Druck dafür, dass Flüssigkeiten fest werden.“
    Das kommt doch bestimmt auch die Größe des Planeten an und den chemischen Stoff an, oder?

    Würde eine Kugel aus flüssigem Wasserstoff oder aus flüssigem Helium, die nur so klein wie Merkur oder noch kleiner ist, innen fest werden?

    Was ist bei Stoffen mit einer Anomalie (Feststoff ist weniger dicht als die Flüssigkeit) wie z. B. Wasser: Wenn man eine Kugel aus Wasser so groß wie der Mars oder so groß wie die Erde hätte, würde das Wasser in tieferen Schichten dann nur allein durch den Druck zu Eis werden? Bei einem Planeten nimmt ja nach innen hin auch die Temperatur immer weiter zu.

    Und falls Salze gelöst sind: salzigeres Wasser ist ja schwerer als weniger salziges Wasser und sinkt nach unten. Es würde sich eine Schichtung ausbilden: Süßwasser an der Oberfläche, darunter salziges Wasser, ganz unten sehr salziges Wasser. Würde auch sehr salziges Wasser, welches in großer Tiefe warm ist (durch die innere Hitze des Planeten), durch hohen Druck zu Eis werden?
    Vielleicht würden sich auch Wasser und Salze entmischen, also dass reine Eiskristalle entstehen, zwischen denen dann Salzkristalle oder kleine Einschlüsse mit extrem salzigem flüssigem Wasser liegen.

  1408. #1432 Manfred Muckenhuber
    Schleißheim
    2. Januar 2022

    Kann es nicht sein das die beschleunigte Expansion des Universums nur ein Trugschluss ist, da durch die Entfernung zu den entferntesten Galaxien auch schon mehr Zeit vergangen ist in der die Expansions- Geschwindigkeit höher war als zu den nahen Galaxien in der das Licht zu einem späteren Zeitpunkt ausgesandt wurde, also die Expansions- Geschwindigkeit zu heute keinen so großen Unterschied hat?

  1409. #1433 Christian Werenka
    Ansfelden Österr.
    17. Januar 2022

    Es wird behauptet (Hubble) dass: “je weiter Objekte (Galaxien) von uns entfernt sind , desto scheller entfernen sie sich von uns” (Rotverschiebung). Aber ist es nicht genauso möglich, dass da diese info Aufgrund der langen Zeit die das Licht gebraucht hat auch sehr alt ist, gesagt werden kann: “früher haben sich Objekte schneller von uns weg bewegt.”
    Woher weis man welche der beiden Aussagen oder ob beide Richtig oder Falsch sind? Die Info über weitentfernte Objekte ist leider immer älter als die von nahen Objekten. Wir vergleichen also Ojekte zu verschiedenen Zeiten mit verschieden Entfernungen.
    Wir können nichts über die aktuelle geschwindikeit eines entferntes Objektes oder die frühere Geschwindikeit eines nahen Objektes sagen.
    Wie löst man dieses Problem?
    mfg. Christian Werenka

  1410. #1435 noonscoomo
    Berlin
    17. Januar 2022

    Ich glaub, Christian meint das anders. Es geht ihm wohl nicht darum, dass das Licht langsamer wird, sondern, so verstehe ich das, wir überhaupt keine Aussage machen können, wie schnell sich eine Galaxie _heute_ von uns weg bewegt, die weit entfernt ist, weil wir ja nur Licht sehen, das vor vielen Milliarden Jahren (mit konstanter Lichtgeschwindigkeit) los geflogen ist. Und, so seine Argumentation, je weiter weg, desto älter. Was wir also mit Sicherheit sagen können, je älter eine Galaxie, um so schneller bewegt sie sich von uns weg. Die Frage lautet also, könnte es nicht sein, dass all diese weit entfernten Galaxien inzwischen viel langsamer sind und daher die Aussage, “das Universum dehnt sich beschleunigt aus” nicht valide ist, weil wir das gar nicht wissen können, weil wir keine Ahnung haben, wie schnell sich die Galaxien _heute_ von uns weg bewegen und es ja sein kann, dass die inzwischen um so langsamer sind.
    Nun ist das ja aber so, dass die Wellenlänge des Lichts aus zwei Gründen grösser (also roter) sein kann, erstens, weil sich das Universum ausdehnt und zweitens, weil die Galaxie (auch ohne Ausdehnung) eine Relativgeschwindigkeit zu uns hat.
    Eigentlich kriegen wir erst raus, um welche Art Rotverschiebung es sich handelt, wenn wir eine weit entfernte Galaxie lange genug beobachtet haben um eine Änderung der Geschwindigkeit direkt beobachtet zu haben. Ich finde die Frage also durchaus berechtigt.

  1411. #1436 Manfred
    Schleißheim
    20. Januar 2022

    Auch ich hab in #1432 nicht an Lichtermüdung gedacht, sondern das sich der Raum 13,8 Milliarden Jahren nach dem Urknall nicht mehr so schnell ausdehnt als zB. 1Milliarde Jahre nach dem Urknall. Das hätte dann aber auch weitreichende Folgen bzgl Dunkle Energie und wie sich das Universum entwickelt. Oder liege ich da falsch?
    MfG Manfred Muckenhuber

  1412. #1437 Robert Müller
    Jena
    19. April 2022

    Ich habe am Nachthimmel eine sehr auffällige Leuchterscheinung gesehen (weißes Licht, Lichtfleck ca. 1/2 der Vollmondgröße, flache fallende Flughahn, ca. von O nach W, Dauer 2-3 sec). Gibt es ein Register, homepage oder ähnliches, wo man nachsehen kann, was das gewesen sein könnte?
    MfG Robert Müller

  1413. #1438 Frank Egermann
    Hannover
    19. Juli 2022

    Weshalb kann man eine Galaxie nicht zweimal sehen? Also z.B. von dort wo sie vor 13,6 Mrd. Jahren stand und von dort, wo sie vor 4 Mrd. Jahren stand?

  1414. #1439 Bullet
    19. Juli 2022

    Hä? Die Frage ist … .ehm … inkonsistent.
    Das vor 13,6 Milliarden ausgesandte Licht ist schon vor 13,5 Milliarden Jahren “hier” angekommen. Und als dieselbe Galaxie vor 4 Milliarden Jahren Licht ausgesendet hat, hatte dieses einen Weg vor sich, der sich ständig vergrößerte, so daß dieses Licht extrem rotverschoben möglicherweise erst in acht oder zehn Milliarden Jahren “hier” ankommt.
    Warum jetzt zweimal?

  1415. #1440 Bernd Wehmöller
    İstanbul
    19. Juli 2022

    @Frank… Du denkst da ein bisschen falsch, denke ich. Nehmen wir an, wir sehen heute im Teleskop ein Objekt, z. B. eine Galaxie, und bestimmen die Entfernung zu 10 Milliarden Lichtjahren. Das bedeutet, dass Licht hat von dort wo wir das Objekt sehen 10 Milliarden Jahre gebraucht um zu uns zu kommen. In diesen 10 Milliarden Jahren hat sich das Objekt selbst ja auch bewegt, und nun ist deine Frage, warum sehen wir das Objekt nicht auch z. B. dort, wo es vor vier Milliarden Jahren war.

    Es gibt zwei Möglichkeiten.
    1. Das Objekt hat sich von uns weg bewegt in den zwei Milliarden Jahren. Dann ist ja sofort klar, dass das Licht von der neuen Position länger zu uns brauchen wird. Da sich das Objekt ja nicht mit Lichtgeschwindigkeit bewegen wird, wird es also mehr als weitere zwei Milliarden Jahre dauern, bis wir es an der Position zu sehen bekommen. Wir könnten es also die ganze Zeit beobachten und irgendwann feststellen, dass es an einer anderen Stelle ist.

    2. Das Objekt bewegt sich auf und zu. Aber auch dann sehen wir es erst später an der neuen Position, denn es ist immer noch zwei Milliarden Jahre unterwegs bis es an der neuen Position ist. Und da es sich auf keinen Fall schneller als Licht bewegen kann, werden wir es auch nicht eher zu Gesicht bekommen. Was wir feststellen werden ist dann, daß die Rotverschiebung abgenommen hat.

    Du musst bedenken, wir sehen die Objekte nicht dort wo sie gerade sind, sondern immer dort wo sie mal waren. Das gilt auch für den Mond und die Sonne, denn das Licht braucht ja einige Zeit bis es bei uns ist. Von der Sonne aus sind das schon Minuten. Und wir sehen es in dem Moment, wo das Licht bei uns eintrifft an der Position an der es damals war.

    Also noch mal einfach. Nimm einmal an, ein neuer Stern entsteht heute in einer Million Lichtjahre Entfernung. Es dauert also eine Million Jahre bis wir den neuen Stern aufgeben sehen. Danach können wir ihn verfolgen und sehen dann wo er später “war”. Egal ob näher bei uns oder weiter entfernt. Da nichts mit mehr als Lichtgeschwindigkeit unterwegs sein kann, kann der Stern das Licht nicht überholen.

    Hoffe das war verständlich. Das Problem ist der Gedanke, daß man etwas”jetzt” sieht. Tut man nicht. Es gibt kein allgemeingültiges “jetzt”, das ist immer nur für einen selbst das jetzt. Alles was man “jetzt” sieht ist schon nicht mehr da wo man es sieht.

  1416. #1441 Peter
    51,1° N 15,0 ° E
    19. Juli 2022

    Zitat #1438: „Weshalb kann man eine Galaxie nicht zweimal sehen? Also z.B. von dort wo sie vor 13,6 Mrd. Jahren stand und von dort, wo sie vor 4 Mrd. Jahren stand?“

    @Frank: Man kann – unter gewissen Umständen – eine Galaxie durchaus 2-mal oder manchmal z. B. auch 4-mal sehen: Nämlich dann, wenn sich die eine Galaxie hinter einer Gravitationslinse befindet.

    Siehe z. B.: https://www.derstandard.at/story/2000137417790/webb-teleskop-zeigt-bei-gravitationslinsen-irrte-einstein-zum-glueck

    Zitat: „Gravitationslinsen eröffnen noch eine Reihe weiterer Möglichkeiten. Manchmal kommt es zu einer Vervierfachung des weiter entfernten Objekts. …
    Das Besondere dabei: Die vier Abbilder zeigen das Objekt üblicherweise nicht zum selben Zeitpunkt. Der Unterschied wird durch die verschieden lange Laufzeit des Lichtes der einzelnen Abbilder verursacht und kann Tage oder sogar Wochen betragen. … 2014 gelang es, einen explodierenden Stern zu beobachten, der von einer Gravitationslinse vervielfältigt wurde. Die Abbilder erschienen im Lauf eines Jahres an verschiedenen Stellen.“

    Ein Zeitunterschied von Tagen bis Wochen ist also „machbar“.

    Einen Unterschied von mehreren Milliarden Jahren, wie Sie ihn erwähnt haben, können wir aber auch bei den stärksten uns bekannten Gravitationslinsen nicht beobachten:

    Denn dazu enthält der uns bekannte Teil des Universums (auch inklusive der Dunklen Materie) zu wenig Masse. Die größten Masse-Strukturen des Universums wie Filamente und Super-Galaxienhaufen (Supercluster) sind nicht massereich und nicht groß genug, um die Raumzeit über sehr viele Milliarden oder wahrscheinlich eher Billionen – Billiarden Lichtjahre hinweg zu so stark zu krümmen, dass Laufzeitunterschiede des Lichts von mehreren Milliarden Jahren entstehen.
    Und die Schwarzen Löcher enthalten zwar viel Masse, diese konzentriert sich aber auf sehr kleinen Raum, was nicht ausreicht, um sehr ausgedehnte Gravitationslinsen zu erzeugen.

  1417. #1442 Norbert
    Greifswald , Hansestadt
    24. Juli 2022

    Wenn wir das Universum in den beobachtbaren Teil mit einer Ausdehnung von 93 Milliarden Lichtjahre Durchmesser und in den nicht beobachtbaren Teil, dessen Größe keiner kennt, von vielen Wissenschaftlern aber als mindestens, 200 bzw. 250 Mal so groß wie das beobachtbare Universum angesetzt wird, aufteilen. Wie kann es dann sein, das wir jetzt Galaxien sehen, die 13,5 Milliarden Jahre alt sind, und sich zwar jetzt aktuell 35-40 Milliarden Lichtjahre von uns entfernt befinden, aber 300 Millionen Jahre nach dem Urknall entstanden sein sollen, also die frühesten Galaxien überhaupt. Was befindet sich dann noch in dem 200-250 Mal so großen nicht sichtbaren Universum, wenn wir doch schon bis Nähe zum Urknall herankommen

  1418. #1443 Norbert
    Greifswald
    24. Juli 2022

    Um den Gedankengang noch einmal aufzugreifen und auf die Spitze zu treiben, dann würde das nicht sichtbare Universum nach Meinung einiger Wissenschafter bei 20 Billionen Lichtjahren immer noch nicht Ende sein, Wo aber soll die Materie aus der Entfernung vor 13,5 Milliarden Lichtjahren einzuordnen sein, wenn die weiteste derzeit 40 Milliarden Lichtjahre weit entfernt ist

  1419. #1444 Bernd
    24. Juli 2022

    @Norbert
    Da kommt die kosmische inflation ins Spiel. Schau mal bei Wikipedia.

    Kurz gesagt hast sich das Universum direkt nach dem Urknall für kurze Zeit mit weit mehr als Lichtgeschwindigkeit ausgedehnt. Dabei ist ein großer Teil des Universums aus “unserem” Sichtfeld verschwunden. Dieser Teil mag zwar auch seit Milliarden von Jahren Licht ausstrahlen, aber das kann uns nie erreichen, da er zu Anfang mit mehr als Lichtgeschwindigkeit von uns weg getrieben wurde. Und Licht kann nicht schneller als Lichtgeschwindigkeit sein, also holt es den Unterschied nicht auf.

    Diese Theorie ist nicht bestätigt, aber erklärt sehr vieles, was man sich sonst nicht erklären könnte. Zum Beispiel die Flachheit des Universums. Wir messen, dass es flach ist, also die Ränder – der Rand des uns sichtbaren Universums – nicht gekrümmt sind. Das kann aber nur sein, wenn außerhalb des Randes ebensoviel oder gar mehr Masse ist, als innerhalb. Sonst würde die Gravitation den Raum am Rand krümmen. Wenn dort also noch Masse ist, muss das Universum sehr viel größer sein als das was wir sehen. So jedenfalls die Logik…

    Hoffe das hilft, zumindest was die Richtung angeht. Man findet viel zum Thema bei Google.

  1420. #1445 Norbert
    Greifswald , Hansestadt
    25. Juli 2022

    Vielen Dank Bernd, das ist mir alles bekannt, wobei ich fast der Meinung bin das in ein Paar Jahren Theorien über Inflation, dunkle Materie und dunkle Energie sich als Irrwege herausstellen werden und es sich in Wirklichkeit um jeweils etwas anderes handelt nur Mangels Wissen heute verwendet wird. Womit ich ein Problem habe ist folgendes, diese nun am weitesten im sichtbaren Universum von Webb entdeckte Galaxie, war meinen Recherchen nach vor 13,5 Milliarden Jahren nur 60 Millionen Lichtjahre von der Erde entfernt, also sehr nahe. Alle von uns aus gesehenen Galaxien dahinter müssen also genau so alt oder älter sein. Das bedeutet das die mit aus dieser Frühphase des Universums sein müssen. Was im Umkehrschluss bedeutet daß alle Materie die wir heute in 93 Milliarden Lichtjahren gerade noch sehen vor 13,5 Milliarden Jahren innerhalb eines Radius von 60 Millionen Jahre befanden.

  1421. #1446 Bullet
    26. Juli 2022

    @Norbert:

    diese nun am weitesten im sichtbaren Universum von Webb entdeckte Galaxie, war meinen Recherchen nach vor 13,5 Milliarden Jahren nur 60 Millionen Lichtjahre von der Erde entfernt

    Woher hast du diese Zahl?

    Was im Umkehrschluss bedeutet daß alle Materie[,] die wir heute in 93 Milliarden Lichtjahren [Entfernung] gerade noch sehen[, sich] vor 13,5 Milliarden Jahren innerhalb eines Radius von 60 Millionen [Licht-]Jahre[n] befand[–].

    Du hast vergessen, dein “Problem” damit zu schildern.

  1422. #1447 Norbert Schmied
    Greifswald , Hansestadt
    11. August 2022

    Die Aussage mit 40 bis 60 Millionen Jahren am Anfang ist mir schon ein paar Mal begegnet habe ich mir nicht ausgedacht. Und ja ich habe wirklich vergessen mein Problem darzustellen, das tut mir leid das ich es nicht weiter ausgeführt habe, aber es betrifft den inneren Wiederspruch den ich verspüre wenn ich höre wir nähern uns bis auf aktuell 230 Millionen Jahre nach dem Urknall, minus 380.000 Jahre wo die ersten Atome entstanden, minus dem Dunklen Zeitalter wo es noch keine Sterne gab, und dann hört man das das unsichtbare Universum noch 200 bis 250 Mal so groß wie das sichtbare Universum ist, tut sich bei anderen da nicht auch dieser Wiederspruch auf das man sich fragt wie passt das zusammen, wo ist das einzuordnen,wie kann das sein. Da ist doch kein Platz mehr dazwischen wir sind doch schon fast an der Grenze der ersten überhaupt möglich entstandenen Sterne.

  1423. #1448 A. Balser
    Frankfurt am Main
    25. August 2022

    Seit Einstein wissen wir, dass die Lichtgeschwindigkeit eine absolute Konstante ist und zwar unabhängig vom Bezugssystem.
    Meine Frage: Wie kann es überhaupt eine messbare Geschwindigkeit bzw. eine Bewegung geben, ohne sie zu einem fixen Bezugssystem (quasi einem ruhenden Punkt) in Beziehung zu setzen?

  1424. #1449 noonscoomo
    Berlin
    25. August 2022

    Geschwindigkeiten (ausser die Lichtgeschwindigkeit) sind immer relativ, d.h. sie existieren nur in Bezug auf etwas anderes und verschiedene Beobachter sind sich nicht einig, wie die Geschwindigkeit eines Objektes ist, wenn sie nicht im selben Inertialsystem sind.
    Oder anders gesagt, in einem komplett leeren Universum gibt es keine Möglichkeit für einen Menschen in einer Rakete, seine Geschwindigkeit festzustellen und wenn er die Rakete an macht und eine Kraft auf die Rakete ausübt ist er hinterher nicht “schneller”, weil der Begriff “schneller” eben nur Sinn ergibt, wenn da noch was anderes ist.

  1425. #1450 Manfred Neumann
    Balamban, Cebu, Philippinen
    25. August 2022

    Ich habe ein Verstaendnisproblem zur Ausdehnung des Weltalls. Es wurde beobachtet, dass sich die am weitesten von uns entfernten Galaxien deutlich schneller von uns weg bewegen als die naeher befindlichen (staerkere Rotverschiebung im Spektrum des Lichtes der weiter entfernten Galaxien). Nun ist aber die Lichtgeschwindigkeit konstant. Das heisst, das Licht der weiter von uns entfernten Galaxien stammt aus einem Zeitraum weit in der Vergangenheit, z. B. von den ersten Galaxien nach dem Urknall. Zu diesem Zeitpunkt war die Ausdehnungsgeschwindigkeit des Weltalls also deutlich hoeher als zum gegenwaertigen Zeitpunkt (geringere Rotverschiebung bei den naeher befindlichen Galaxien, deren Licht uns aus der juengeren Vergangenheit erreicht). Somit hat es den Anschein, dass in der weit entfernesten Vergangenheit die Ausdehnung des Weltalls deutlich hoeher (!) war als aktuell. Wo ist mein Denkfehler?

  1426. #1451 Christian
    Wien
    25. August 2022

    > Wo ist mein Denkfehler?

    Nimm ein Gummiband und zeichne in gleichen Abständen etwa mit einem Filzstift Querstriche darauf. Dann bestimme den Abstand zwischen einem mittleren Strich und dessen Nachbarstrich sowie diesem mittleren Strich und einem Strich am Ende des Gummibands.
    Danach ziehe das Gummiband auseinander und befestige es in dieser Position. Wenn du wieder nachmisst, dann wirst du feststellen, dass sich die jeweiligen Abstände zum Beispiel verdoppelt haben. Aus zum Beispiel 5 cm wurden 10 cm, aus zum Beispiel 25 cm 50 cm. Für die Dehnung des Gummibands hast du vielleicht 2 Sekunden gebraucht. Der dem mittleren Strich nächstgelegene Strich hat sich also in 2 Sekunden um 5 cm von diesem entfernt. Der Strich am Ende des Gummibands hat sich ebenfalls in 2 Sekunden um 25 cm vom mittleren Strich entfernt.
    Also obwohl sich das Gummiband überall gleichmäßig mit derselben Geschwindigkeit ausgedehnt hat, hat sich der Strich am Emde des Gummibands mit der 5-fachen Geschwindigkeit vom mittleren Strich entfernt wie der dem mittleren Strich benachbarte Strich.
    Ich denke, jetzt ist alles klar, oder?

  1427. #1452 Manfred Neumann
    Balamban, Cebu, Philippinen
    26. August 2022

    Wenn ich es richtig verstanden habe, koennte Dein Gummiband-Modell die unmittelbar nach dem Urknall (vermutlich) stattgefundene Inflation (kurzzeitige Ausdehnung des Raumes mit Ueberlichtgeschwindigkeit, was zur aktuell beobachteten Homogenitaet des Weltalls fuehrte) veranschaulichen. Allerdings ist es mir nicht moeglich, damit mein Verstaendnisproblem zu loesen.
    Nach wie vor “kaempfen” zwei Vorstellungen in meinem Denken um die Vorherrschaft:
    1.) aktuelle Lehrmeinung: beschleunigte Ausdehnung des Weltraums, belegt durch die hoeheren Fluchtgeschwindigkeiten weiter von uns entfernter Objekte
    2.) meine Denkweise: verminderte Ausdehnung des Raumes, belegt durch die hoeheren Fluchtgeschwindigkeiten der weiter von uns entfernten (und damit wegen der Lichtgeschwindigkeitskonstante deutlich aelteren Informationen) Objekte.
    Je juenger die Infos sind (z. B. von naeheren Galaxien), desto langsamer entfernen sie sich von uns, d. h., der Raum dehnte sich in der Vergangenheit viel schneller aus als gegenwaertig.

  1428. #1453 Christian
    Wien
    26. August 2022

    Nein, nein es geht nicht um die (hypothetische) Inflation, sondern das Gummiband-Modell beschreibt den Zustand des Universums danach bis in die heutige Zeit, wo die Expansionsrate des Universums einigermaßen gleichbleibend war. Das Gummiband-Modell zeigt ja, wie es zustande kommt, dass sich Galaxien umso schneller von unserem Standort im Universum entfernen, je weiter sie weg sind, obwohl das Universum beziehungsweise das Gummiband überall die gleiche Ausdehnungsrate pro Zeiteinheit hat. Wenn du ein Gummiband dehnst, dann ist ja nicht am Rand stärker gedehnt als in der Mitte, was aber notwendig wäre, wenn man davon ausgeht, dass sich der Raum in der Vergangenheit viel schneller ausgedehnt hat als in der Gegenwart. Möglicherweise ist es sogar umgekehrt: https://www.scinexx.de/news/kosmos/kosmologie-beschleunigte-expansion-im-zwielicht/

  1429. #1454 Dieter Schläger
    Deisenhausen
    26. August 2022

    Hallo Manfred Neumann,

    wenn ich da auch mal meinen Senf dazu geben darf:
    Zeichne auf das Gummiband eine Wellenlinie vom mittleren Strich bis zum äußersten Strich, dann kannst Du sehen wie die Rotverschiebung zustande kommt. Die rührt nicht von der Geschwindigkeit der Galaxien bei der Lichtemission her, sondern von der Ausdehnung des Raumes während der gesamten “Reise” des Lichtes bis zu uns. Diese “Reisezeit” ist bei den Galaxien, die heute von uns aus gesehen am Rande des Universums sind, einfach viel länger!
    MfG
    Dieter

  1430. #1455 A. Balser
    Frankfurt am Main
    26. August 2022

    Danke an “noonscoomo” für die Antwort auf meine Frage (#1448), aber ich hatte sie wohl nicht präzise genug gestellt.
    Ich verstehe, dass es keine Bewegung bzw. Geschwindigkeit gibt unabhängig von einem festen Bezugssystem, anhand dessen sie sich definieren und messen lässt.
    Doch wie kann die Geschwindigkeit oder überhaupt die Bewegung von Licht angenommen werden unabhängig von irgendeinem fixen Punkt?

  1431. #1456 noonscoomo
    Berlin
    26. August 2022

    Ah, ja, gute Frage. Da die Lichtgeschwindigkeit im Vakuum immer und für jede Beobachter:in dieselbe ist braucht sie keinen Bezugspunkt bzw. hat immer die Beobachter:in als Bezugspunkt. Egal mit welcher Geschwindigkeit du dich “in Bezug auf etwas anderes” bewegst, du wirst _immer_ dieselbe Lichtgeschwindigkeit messen, weil du, solange du dich geradlinig und gleichförmig bewegst (in Bezug auf etwas anderes) behaupten kannst, dass du dich gar nicht bewegst (Relativitätsprinzip). Das Licht hat kein fixes Medium (früher Lichtäther genannt) durch das es sich bewegt. Michelson-Morley-Experiment und all seine Varianten und Überprüfungen zeigen das eindrücklich und das geht selbst aus den Maxwell-Gleichungen hervor. Denn würden wir selbst mit Lichtgeschwindigkeit reisen (was eh nicht geht, aber nur der Anschauung wegen) dann würden wir, wenn wir neben einem Lichtstrahl her reisen eine stehende elektromagnetische Welle sehen, die kann es aber nicht geben.
    Jetzt kann man natürlich die Frage stellen, wie messen wir die Lichtgeschwindigkeit. Das ist recht einfach für die s.g. Zweiwege-Lichtgeschwindigkeit (also hin und zurück). Da bauen wir einfach irgendwo einen Spiegel hin, schiessen einen Lichtpuls hin und messen wie lange das dauert bis der zurück kommt. Schwieriger wird es bei der Einweg-Lichtgeschwindigkeit. Dazu bräuchten wir zwei absolut synchrone Uhren an beiden Orten. Da hast du dann aber das Problem der Uhrensynchronisierung, aber das ist wieder ein anderes Problem.

  1432. #1457 A. Balser
    Frankfurt am Main
    26. August 2022

    Aha, das hilft meinem Verständnis tatsächlich schon mal etwas weiter – besten Dank!
    Ich hatte mal gelesen, bestimmte Geschwindigkeiten ließen sich messen in Bezug auf den Mikorwellenhintergrund. Doch das konnte ich nicht nachvollziehen, da diese Strahlung zwar in alle Richtungen verläuft, aber ja nicht “still hält” und somit als Bezugssystem für eine definierte gerichtete Bewegung schwer vorstellbar ist.

  1433. #1458 Manfred Neumann
    Balamban, Cebu, Philippinen
    26. August 2022

    Vielen Dank fuer die Antworten von Christian (#1453) und Dieter (#1454) zur Ausdehnung des Weltraumes. Mein Denkfehler war, dass die Beobachtungsergebnisse des Lichtes von fernen und nahen Galaxien vergleichbar waeren. Das sind sie aber nicht, weil das Licht von ferneren Galaxien wegen des laengeren Weges (und des damit laenger andauernden Energieverlustes durch die Raumausdehnung) staerker geschwaecht wird als dasjenige von naeheren Galaxien.

  1434. #1459 Christian
    Wien
    26. August 2022

    @ Manfred Neumann

    Gerne, und auch danke an den Dieter Schläger für die kompetente Unterstützung!

  1435. #1460 Dieter Schläger
    Deisenhausen
    26. August 2022

    Oh bitte sehr, es freut einen doch immer, wenn man es mal fertig bringt in etwas komplizierteren Zusammenhängen etwas verständliches von sich zu geben.

  1436. #1461 Olivia
    Pfaffenhofen a.d.Ilm
    31. August 2022

    Hallo, ich habe eine Frage bezüglich der Perseiden, da ich bisher nicht verstanden habe warum es bei den Meteorströmen so aussieht als würden sie aus dem Sternbild Perseus kommen. Warum genau dieser Punkt?

  1437. #1462 Frieder
    Halberstadt
    1. September 2022

    Entfernungen und Alter im Universum

    Bei großen Entfernungen greift man auf die Laufzeit des Lichtes zurück und auf die Expansion der Raum-Zeit. Je höher die Rotverschiebung desto größer die Entfernung.

    Woher wissen wir mit hinlänglicher Sicherheit, wann die Expansion der Raumzeit wie groß war? Beruht die Bestimmung von sehr großen Entfernungen und Zeiträumen letztlich auf mehr oder weniger unsicheren Annahmen?
    Hubble hat sich bei seiner kosmologischen Konstante auch mehrfach korrigieren lassen müssen.
    Also: Wie genau sind überhaupt Entfernungsbestimmungen im All. Eine Methode stützt sich auf die andere. Damit vervielfachen sich zwangsläufig die Ungenauigkeiten.

    Die Altersbestimmung über die Laufzeit des Lichtes gibt aber nur ein Mindestalter an.
    Wie alt war das Objekt, als es die elektromagnetische Strahlung oder auch die Gravitationswelle ausgesandt hat?
    Woher wissen wir das Alter von Objekten die offensichtlich deutlich älter sind als die Laufzeit des Lichtes zu uns?
    Wie groß ist das Universum?
    Es dehnt sich seit 13,8 Milliarden Jahren aus. So muss es doch deutlich größer als diese 13,8 Ly sein.
    Wie geht die Wissenschaft damit um, dass aus weiten Bereichen uns wegen der Laufzeit elektromagnetische Strahlung gar nicht erreichen kann?
    Es muss so Bereiche geben, über die wir spekulieren können, die dennoch verborgen bleiben.

  1438. #1463 Bullet
    1. September 2022

    Wie geht die Wissenschaft damit um, dass aus weiten Bereichen uns wegen der Laufzeit elektromagnetische Strahlung gar nicht erreichen kann?

    klick mich, dann siehste

  1439. #1464 Spritkopf
    4. September 2022

    @Olivia

    Hallo, ich habe eine Frage bezüglich der Perseiden, da ich bisher nicht verstanden habe warum es bei den Meteorströmen so aussieht als würden sie aus dem Sternbild Perseus kommen. Warum genau dieser Punkt?

    Weil die Erde auf ihrer Bahn um die Sonne genau in Richtung des Sternbildes Perseus fliegt, wenn das Maximum der Perseiden auftritt.

    Anders gesagt: Wenn du eine Tangente an die Erdbahn dort anlegst, wo sich die Erde am 13. August befindet, zeigt diese Tangente auf das Sternbild Perseus.

    https://de.wikipedia.org/wiki/Radiant_(Astronomie)

  1440. #1465 Karl-Heinz
    Graz
    4. September 2022

    @Olivia

    Der Fachbegriff heißt Radiant (Astronomie).
    Als Radiant bezeichnet man den Punkt am Nachthimmel, von dem aus Meteorschauer ihren Anfang zu nehmen scheinen.
    https://de.m.wikipedia.org/wiki/Radiant_(Astronomie)

    Der Radiant oder Ausstrahlungspunkt
    ist jenes Gebiet am Himmel, aus dem die Sternschnuppen eines Meteorstromes zu kommen scheinen. Durch die Perspektive (wir sehen Bruchstücke die aus ein und derselben Richtung auf parallelen Bahnen aus dem Weltall kommen) sieht es für uns auf der Erde so aus, als würden alle Meteore aus einem kleinen Himmelsgebiet kommen. D.h. alle Sternschnuppen eines Meteorstromes treffen sich bei Rückverlängerung Ihrer Bahnen an einem Punkt am Himmel. die Meteorströme werden dann nach dem Sternbild benannt, in dem dieser Punkt liegt. die Perseiden haben ihren Radianten im Sternbild Perseus, daher der Name Perseiden.

    https://www.astronomie.at/Meteor/metstart.asp

    Weitere Links:
    https://www.meteorshowers.org/
    Bitte die Perseiden auswählen.

    http://www.sternwarte-eberfing.de/Fuehrung/Objekbeschreibung/Meteorstroeme.html

    PS: Wenn du mit dem Auto bei starkem Schnefall fährst, tritt der gleiche Effekt bezüglich Perspektive auf.
    So ich hoffe, ich konnte damit alle Klarheiten beseitigen. 😉

  1441. #1466 Karl-Heinz
    Graz
    5. September 2022

    @Olivia

    Der Fachbegriff heißt Radiant (Astronomie).
    Als Radiant bezeichnet man den Punkt am Nachthimmel, von dem aus Meteorschauer ihren Anfang zu nehmen scheinen.
    https://de.m.wikipedia.org/wiki/Radiant_(Astronomie)

    Der Radiant oder Ausstrahlungspunkt
    ist jenes Gebiet am Himmel, aus dem die Sternschnuppen eines Meteorstromes zu kommen scheinen. Durch die Perspektive (wir sehen Bruchstücke die aus ein und derselben Richtung auf parallelen Bahnen aus dem Weltall kommen) sieht es für uns auf der Erde so aus, als würden alle Meteore aus einem kleinen Himmelsgebiet kommen. D.h. alle Sternschnuppen eines Meteorstromes treffen sich bei Rückverlängerung Ihrer Bahnen an einem Punkt am Himmel. die Meteorströme werden dann nach dem Sternbild benannt, in dem dieser Punkt liegt. die Perseiden haben ihren Radianten im Sternbild Perseus, daher der Name Perseiden.

    PS: Wenn du mit dem Auto bei starkem Schnefall fährst, tritt der gleiche Effekt bezüglich Perspektive auf.
    So ich hoffe, ich konnte damit alle Klarheiten beseitigen. 😉

  1442. #1467 Karl-Heinz
    Graz
    5. September 2022

    @Spritkopf #1464
    Ich würde das ein bisschen abschwächen.
    Die Perseiden kommen in Bezug auf den Beobachter, der nach den Perseiden ausschau hält, aus Richtung des Sternbildes Perseus.

  1443. #1468 Bullet
    5. September 2022

    Ich hab das mal mit Celestia nachvollzogen, weil ich das auch dachte. Da sich aber die Erde auf der Ekliptik bewegt, können die Richtungen, in die sich die Erde bewegt, immer nur auf der Ekliptik abgebildet werden.
    Perseus is kein Ekliptiksternbild.
    Das hat mich überrascht. Also – daß Perseus kein Ekliptiksternbild ist, hat mich nicht überrascht, aber daß der naive Ansatz, den Spritti dargestellt hat, nicht stimmen kann, schon. Und das ist keine Kleinigkeit: da liegen mindestens 20° nach Norden zwischen Perseus und Widder (wohin etwa die Erde am 5.8. zielt).

  1444. #1469 Karl-Heinz
    Graz
    5. September 2022

    @1468 Bullet

    Ja genau. Ich würde mal als Bezugssystem das Sonnensystem nehmen. Dann haben sowohl die Perseiden als auch der Beobachter einen Geschwindigkeitsvektor. Dann muss man noch in das System des Beobachters transformieren.
    Ich erwarte mir, dass der Geschwindigkeitsvektor aus dem die Perseiden für mich kommen auf der Line von mir und Perseus liegt.

  1445. #1470 Detlef
    Mülheim an der Ruhr
    10. September 2022

    Hallo Florian
    Ich habe da eine Frage zum Thema ‘Schwarze Löcher und Galaxien’. Ich habe vor kurzem gelesen, das es einen Zusamenhang zwischen zentralem SL und der Bulge geben soll. Kein SL, keine Bulge. Als Beispiel wurde Messier 33, die Dreiecksgalaxie, genannt. Weist Du etwas darüber ob es da wirklich einen Zusammenhang gibt ? Oder wurde da nur ein Einzelfall verallgemeinert ?

  1446. #1471 Johannes
    Heidelberg
    14. Oktober 2022

    Hallo Herr Freistetter,
    liebe Fachleute,

    ich habe heute Folge 84 und 85 der Sternengeschichten gehört, und dabei kam bei mir die Frage auf, warum wir uns unserer klimaschädlichen Gase nicht “einfach” ins Weltall entledigen?
    Es gibt ja Anlagen, die CO2 aus der Luft filtern können, man weiß nur kaum, wohin damit (Unterirdisch in Bimsgestein speichern ist eine mögliche, aufwändige Lösung). Was würde passieren, wenn man diese, salopp gesprochen, mittels eines CO2-und-Kuhpups-Fahrstuhls ins Weltall pusten würde, außerhalb der Atmosphäre, so dass die Erde sie nicht mehr anzieht? Wie würde sich der dadurch eintretende Gewichtsverlust auf die Erdbahn, -rotation etc. auswirken? Ich bin gespannt, wie die Gemeinschaft hierüber diskutiert.

  1447. #1472 Captain E.
    14. Oktober 2022

    @Johannes:

    Ein Fahrstuhl ins All wird noch für eine ziemlich lange Zeit ausschließlich ein Gedankenspiel sein. Und mit Raketen würden wir niemals nennenswerte Mengen wegschaffen können.

  1448. #1473 Bullet
    17. Oktober 2022

    außerhalb der Atmosphäre, so dass die Erde sie nicht mehr anzieht?

    Das ist ganz schön weit weg… selbst der Mond ist noch ziemlich tief im Gravitationsbrunnen der Erde.

  1449. #1474 Oliver Gabath
    19. Oktober 2022

    @1472:
    Ganz zu schweigen vom dem Kohlenstoffdioxid, dass wir für Produktion, Vorbereitung und Flug der Raketen in die Atmosphäre bliesen.

  1450. #1475 Haberich
    Erde
    26. Oktober 2022

    “Die Raumstruktur des Universums dehnt sich mit über 160.000 Meilen pro Stunde aus.” habe ich im Internet gelesen. Meine zwei Fragen dazu. Wie kann man das ermitteln? Ist diese Ausbreitungsgeschwindigkeit gleichbleibend bzw. gleichmäßig beschleunigend oder fluktuiert diese Ausbreitungsgeschwindigkeit?

  1451. #1476 alex
    26. Oktober 2022

    @Haberich:
    Der zitierte Satz ist Unsinn. Die Ausdehnung des Universums hat keine Geschwindigkeit im gewöhnlichen Sinne (also etwas das man in km/h oder Meilen pro Stunde messen könnte). Wie schnell sich ein bestimmtes Stück des Raums ausdehnt, hängt davon ab wie groß es ist.

    Wenn zwei Galaxien z.B. 10 Millionen Lichtjahre voneinander entfernt sind, dann dehnt sich der Raum zwischen ihnen mit etwa 750.000 Kilometer/Sekunde aus. Wenn sie doppelt so weit voneinander entfernt sind, also 20 Millionen Lichtjahre, dann dehnt sich der Raum zwischen ihnen auch doppelt so schnell aus, also mit etwa 1,5 Millionen Kilometer/Sekunde.

    Der Proportionalitätsfaktor zwischen Ausdehnungsgeschwindigkeit und Entfernung heißt Hubble-Konstante. Oder auch Hubble-Parameter, weil er eben nicht konstant ist, sondern zu unterschiedlichen Zeiten in der Geschichte des Universums unterschiedliche Werte hatte.

    Im Prinzip misst man den Hubble-Parameter, indem man für möglichst viele möglichst weit entfernte Objekte deren Entfernung und Geschwindigkeit misst.

  1452. #1477 Karl-Heinz
    Graz
    27. Oktober 2022

    @Haberich

    So, dank Alex wissen wir jetzt bescheid.
    Jetzt kommt die Verständnisfrage. Man hört ja immer, dass dich das Universum beschleunigt ausdehnt.
    Meine Frage: Wird es uns in naher Zukunft zerreißen?

    PS: Achtung: Die Hubble-Konstante war in der Vergangenheit immer höher als heute.

    PS2: Ich hoffe ich konnte damit alle Klarheiten beseitigen. Bei Mensch Ärgere dich nicht heißt dies, zurück an den Start. 😉

  1453. #1478 Domenik H.
    Jena
    30. November 2022

    Guten Tag,
    Nachdem ich mir, anhand von dem Luftballon- und Hefeteigbespiel, erklären lassen habe, wie das Universum sich ausdehnt, kam mir eine Frage in den Kopf. Und zwar, ob denn nicht das gesamte Universum die Singularität ist? Also ob denn nicht das Universum heute die selbe Singularität wie vor 13,8 Mrd. Jahren (Zeitpunkt des Urknalls) ist, nur eben sehr viel gröser als eben zum Zeitpunkt des Urknalls.
    MfG Domenik H.

  1454. #1479 Christian
    Wien
    30. November 2022

    Wenn gefragt wird, ob unser Universum ein Schwarzes Loch sein könnte, heißt es meist: Nie und nimmer. Für den Fall eines Großen Rückpralls – https://de.m.wikipedia.org/wiki/Big_Bounce – könnte es aber durchaus so sein. Denn solange der Rückprall symmetrisch erfolgt (und unser Universum ist ja homogen und isotrop), ist einem Schwarzen Loch von außen betrachtet nicht anzumerken, ob die Energie unterhalb seines Schwarzschild-Radius in einer Singularität oder kugelsymmetrisch verteilt ist.

  1455. #1480 Regine B.
    Weil der Stadt
    13. Dezember 2022

    Hallo,
    mich beschäftigt folgende Frage zur kosmischen Hintergrundstrahlung:

    Wenn sich die elektromagnetische Strahlung seit dem Urknall von einem Punkt aus in allen Richtungen explosionsartig und kugeloberflächenförmig mit Lichtgeschwindigkeit fortbewegt, und wenn Materie sich langsamer als die Lichtgeschwindigkeit vom selben Punkt ausgehend luftballonartig verteilt, dann müsste die Strahlung des Urknalls die Materie doch von Anfang an “überholt” haben und sich jetzt vom Ausgangspunkt des Urknalls viel weiter weg befinden als z. B. unsere Galaxie / Erde. Wie können wir dann hier noch etwas von dieser Strahlung messen? Der Urknall ist doch ein längst vergangenes Ereignis. Die Strahlung müsste doch “auf und davon” sein!

  1456. #1481 Spritkopf
    13. Dezember 2022

    @Regine B.

    Wir messen nicht die Strahlung des eigentlichen Urknalls, sondern die Lichtstrahlung ca. 380.000 Jahre nach dem Urknall, nachdem das Universum durchsichtig wurde. Zu diesem Zeitpunkt war das Universum nicht mehr punktförmig (wenn es das überhaupt jemals war).

    Außerdem handelte es sich beim Urknall nicht um eine Explosion im Raum, bei der die Materie sich mit Unterlichtgeschwindigkeit ausbreitet. Vielmehr war es der Raum selber, der expandiert und die in ihm enthaltene Materie “mitnimmt”.

  1457. #1482 Bullet
    15. Dezember 2022

    Wie ein Luftballon das eben so macht. Da hat jemand das Luftballonmodell nicht verstanden.

  1458. #1483 Petra Kühne
    65510 Idstein
    25. Januar 2023

    Vielen Dank für die super gute, interessante und verständliche Beschreibung zur Bewegung unseres Sonnensystems durch die Milchstraße. Mich würde dazu brennend interessieren, was mit der „Galaktischen Ausrichung“ gemeint sein könnte. Ich habe da on gehört finde aber nix dazu. Bedeutet das, das Sonnensystem ist immer im 63 Grad Winkel zum Zentrum der Milchstraße, bewegt sich damit auf und ab und ist aber irgendwann auch immer mal genau auf der gleichen Ebene (also Sonne und Milchstraßenzentrum auf gleicher Höhe?). Ganz liebe Grüße, Petra

  1459. #1484 Schufti
    16. April 2023

    Hallo!
    Da ich auf der neuen Seite von Florian (https://astrodicticum-simplex.at/) keinen Link für Fragen gefunden habe versuche ich es nochmal hier.

    Ich möchte wissen ob Himmelskörper nur durch eine nahe Begegnung mit einem Planeten aus dem Sonnensystem geworfen werden können. Oder kann das auch nach einer nahen Begegnung mit der Sonne selbst passieren? Wenn ja, welcher Mechanismus ist das? Der Swing by Effekt kann es ja nicht sein, denn der beruht ja auf der relativen Bewegung eines Planeten um die Sonne.
    Falls es eine neue Seite für Astronomie Fragen gibt wäre ich auch für einen Hinweis dankbar.
    Danke im voraus

  1460. #1485 Manfred Neumann
    Balamban, Cebu, Philippinen
    3. Juli 2023

    Warum werden die Galaxien nicht durch die Wirkung der Dunklen Energie “Auseinandergerissen”, da die Dunkle Energie seit dem Urknall der “Gegenspieler” der Dunklen Materie ist? Und warum sind wenigstens durch die Wirkung der Dunklen Energie die uralten Galaxien im Mittel nicht deutlich groesser als die jungen Galaxien?

  1461. #1486 rolak
    3. Juli 2023

    moin Manfred, der Hausherr ist umgezogen – dieser Frage-thread lebt jetzt dort.

  1462. #1487 Carsten
    12. Juli 2023

    Hallo Herr Freistetter,

    immer wieder liest und hört man davon, dass die Bewegung von Sternen bestimmt wurde. Nun scheint ja die Positionsänderung von Sternen derartig gering zu sein (pro Jahr), dass sie eben “Fixsterne” heißen.
    Wie misst man denn die Bewegung eines Sternes in der Milchstraße, in einem Kugelsternhaufen, einer anderen Galaxie? Wenn der Stern sich auf mich zu oder wegbewegt dürfte dies ja wahrscheinlich durch eine Analyse der Verschiebung der Spektrallinien möglich sein, aber wie ist es bei seitlichen Bewegungen? Und wie lassen sich daraus Bahnen bestimmen?
    Wie bestimmt man die Bewegung eines Kugelsternhaufens?

    Beste Grüße,

    Carsten

  1463. #1488 rolak
    12. Juli 2023

    Hallo Herr Freistetter

    moin Carsten, schon mal einen vorsichtigen Blick in den Kommentar genau vor Deinem geworfen?

  1464. #1489 Hello World! https://national-team.top/go/hezwgobsmq5dinbw?hs=1c51898923cbccf9cb0d07bf220ca118
    aurcxm
    13. Juli 2023

    wahzd1

  1465. #1490 Get free iPhone 14 Pro Max: http://uhycentra.com/uploads/go.php hs=1c51898923cbccf9cb0d07bf220ca118*
    8d8axo
    27. August 2023

    75zrn3

  1466. #1491 Get free iPhone 15: https://cncdost.com/uploads/go.php hs=1c51898923cbccf9cb0d07bf220ca118*
    dxvwdp
    28. November 2023

    8m8i6m